Download as pdf or txt
Download as pdf or txt
You are on page 1of 158

5/26/24, 6:17 PM Exam Automation

College Knowledge
.

Test / Exam Name: Differentiation Standard: 12th Science Subject: Mathematics


Student Name: Section: Roll No.:
Questions: 880 Time: 01:00 hh:mm Marks: 3420
3 3
Q1. sin t cos t dy 7 Marks
If x = ,y = , find dx
√cos 2
t √cos 2t

Ans: sin 3 t cos 3 t


We have, x = and y =
√cos 2t √cos 2t
dx d sin 3 t
⇒ dt
= dt [ √cos 2t ]
d d
3
dx √cos 2t dt ( sin ) − sin 3 t dt √cos 2t
⇒ =
dt cos 2t
[Using quotient rule]
d 1 d
3
√cos 2t ( 3sin t ) dt ( sin t ) − sin 3 × ( cos 2t )
dx 2√cos 2t dt
⇒ =
dt cos 2t
sin 3 t
2
3√cos 2t ( sin tcos t ) − ( − 2sin 2t )
dx 2√cos 2t
⇒ dt
= cos 2t
dx 3cos 2tsin 2 tcos t + sin 3 tsin 2t
⇒ =
dt cos 2t√cos 2t
dy d cos 3
Now, dt
= dt [ √cos 2t ]
d d
3 t ) − cos 3 t dt √cos 2t
dy √cos 2t dt ( cos
⇒ =
dt cos 2t
[Using quotient rule]
d 1 d
2
√cos 2t ( 3cos t ) dt ( cos t ) − cos 3 t × ( cos 2t )
dy 2√cos 2t dt
⇒ =
dt cos 2t
cos 3 t
3√cos 2tcos 2 t − ( sin t ) − ( − 2sin 2t )
dy 2√cos 2t
⇒ dt
= cos 2t
dy − 3cos 2tcos 2 t + cos 3 tsin 2t
⇒ dt
=
cos 2t√cos 2t
dy
dy dt − 3cos 2tcos 2 tsin t + cos 3 tsin 2t cos 2t√cos 2t
∴ = dx
= ×
dt cos 2t√cos 2t 3cos 2tsin 2 tcos t + sin 3 tsin 2t
dt
dy sin tcos t [ − 3cos 2tcos t + 2cos 3 t ]
⇒ =
dt sin tcos t [ 3cos 2tsin t + 2sin 3 t ]
dy [ − 3 ( 2cos 2 t − 1 ) cos t + 2cos 3 t ]
⇒ =
dt [ 3 ( 1 − 2sin 2 t ) sin t + 2sin 3 t ]

[ cos2t = 2cos 2t − 1
cos2t = 1 − 2sin 2t ]
dy − 4cos 3 t + 3cos t
⇒ =
dt 3sin t − 4sin 3 t
dy − cos 3t
⇒ dt
= sin 3t

[ cos3t = 4cos 3t − 3cost


sin3t = 3sin 2t − 4sin 3t
dy
]
∴ = − cos3t
dx
Q2. dy 7 Marks
Find dx
y= x log x + (logx x)

Ans: Let y = x log x + (logx x)


Also, let u = (logx) x and v = x log x
∴ y=v+u
dy dv du
⇒ dx
= dx
+ dx
. . . . . (i)
Now, u = (logx) x

⇒ logu = log [(logx) x ]


⇒ logu = xlog(logx)
Differentiating both sides with respect to x,
1 du d d
u dx
= log(logx) dx (x) + x dx [log(logx) ]
du 1 d
⇒ dx [ ]
= u log(logx) + x log x dx (logx)
du x 1
⇒ dx = (logx) x [log(logx) + log x × x ]
du 1
⇒ dx = (logx) x [log(logx) + log x ] . . . . . (ii)
Also, v = x log x
⇒ logv = logx log x
⇒ logv = logxlogx = (logx) 2
Differentiating both sides with respect to x,
1 dv d
v dx
=
dx [(log) 2 ]
1 dv d
⇒ = 2(logx) dx (logx)
v dx
dv 1
⇒ dx
= 2v(logx) x
dv log x
⇒ dx
= 2x log x x
dv log x log x
⇒ dx
= 2x x
. . . . . (iii)
From (i), (ii) and (iii), we obtain

https://bls.smartstudies.co.in/#/exam/pdf-preview/c59cb220-8e86-4716-9ff7-82aec16b1ade/1 1/158
5/26/24, 6:19 PM Exam Automation
dy log x log x 1
dx
= 2x
x [ log x ]
+ (logx) x log(logx) +
Q3. Differentiate sin − 1 (4x√1 − 4x 2 ) with respect to √1 − 4x 2, if: 7 Marks

1 1
x ∈ (−
2√ 2 √ 2√ 2 )
,

Ans: Let u = sin − 1 4x 1 − 4x 2


( √ )
Put 2x = cosθ

(
⇒ u = sin − 1 2 × cosθ 1 − cos 2θ √ )
⇒u= sin − 1(2cosθsinθ)
⇒ u = sin − 1(sin2θ) . . . . . (i)
2
Let, v = √1 − 4x . . . . . (ii)
Here,
1 1
x∈ ( − 2√ 2 , 2√ 2 )
1 1
⇒ 2x ∈ ( − . )
√2 √2
π 3π
⇒ θ ∈ (4, 4 )
So, from equation (i),
u = π = 2θ
π
[Since, sin − 1(sinθ) = π − θ, if θ ∈ ( 2 , π )]
⇒ u = π − 2cos − 1(2x) [Since, 2x = cosθ ]
Differentiating it with respect to x,
du
dx
=0−2 (√ −1
1 − ( 2x ) 2
) d
dx
(2x)

du 2
⇒ dx
= (2)
√ 1 − 4x 2
du 4
⇒ = . . . . . (iii)
dx 2
√1 − 4x
From equation (ii)
dv − 4x
⇒ dx
=
√1 − 4x 2
1 1
But, x ∈ ( − 2 , 2√ 2 )
dv −4( −x)
⇒ dx
=
√1 − 4 ( − x ) 2
dv 4x
⇒ = . . . . . (iv)
dx 1 − 4x 2

Diferentiating equation (ii) with respect to x,
dv 1 d
dx
= (1 − 4x 2)
2 1 − 4x 2 dx

dv 1
⇒ dx
= ( − 8x)
2 1 − 4x 2

dv − 4x
⇒ = . . . . . (v)
dx 2
√1 − 4x
Dividing equation (iii) by (v)
du 2
dx 4 √1 − 4x
dv
= ×
2 − 4x
dx √1 − 4x
du 1
∴ = −
dv x
Q4. x−1 1 1 7 Marks
Differentiate tan − 1 ( x + 1 ) with respect to sin − 1 (3x − 4x3 ), if − 2 < x < 2
Ans: Let, u = tan − 1 x−1
(x+1 )
Put x = tanθ
tan θ − 1
⇒ u = tan − 1 ( tan θ + 1 )
π
tan θ − tan 4
⇒ u = tan −1
( 1 + tan θtan 4
π )
π
⇒ utan − 1 tan (θ − [ 4 ) ] . . . . . (i)
1 1
Here, − 2 < x < 2
1 1
⇒ − < tanθ <
2 2
1 1
⇒ − tan − 1 2 ( ) < θ < tan − 1 ( 2 )
So, from equation (i)
π
u=θ−
4
π π
[Since,tan − 1(tanθ) = θ, if θ ∈ ( − 2 , 2 )]
π
⇒ u = tan − 1x − 4
[Since,x = tanθ]
Differentiating it with respect to x,
du 1
= −0
dx 1 + x2
du 1
⇒ = . . . . . (ii)
dx 1 + x2
And,
Let, v = sin − 1(3x − 4x 3)
Put x = sinθ
⇒ v = sin − 1(3sinθ − 4sin 3θ)
⇒ v = sin − 1(sin3θ) . . . . . (iii)
1 1
Now, − 2 < x < 2
1 1
⇒ − 2
< sinθ < 2

https://bls.smartstudies.co.in/#/exam/pdf-preview/c59cb220-8e86-4716-9ff7-82aec16b1ade/1 2/158
5/26/24, 6:19 PM Exam Automation
1 π
⇒ − 6
<θ< 6
So, from equation (iii),
π π
[
v = 3θ Since,sin − 1(sinθ) = θ, if θ ∈ [ − 2 , 2 ]]
⇒ v = 3sin − 1x[Since, x = sinθ]
Dirrerentiating it with respect to x,
dv 3
= . . . . . (iv)
dx 2
√1 − x
Dividing equation (iii) by (iv),
du 2
dx 1 √1 − x
dv
= ×
1 + x2 3
dx
2
du √1 − x
∴ =
dv 3 ( 1 + x2 )
Q5. Differentiate the following functions from first principles: 7 Marks
sin − 1(2x + 3)

Ans: Let f(x) = sin − 1(2x + 3)


⇒ f(x + h) = sin − 1(2(x + h) + 3)
⇒ f(x + h) = sin − 1(2x + 2h + 3)
d f ( x + h ) − f(x)
∴ dx
{f(x)} = lim h
h→0
sin − 1 ( 2x + 2h + 3 ) − sin − 1 ( 2x + 3 )
= lim h
h→0
sin − 1 [ ( 2x + 2h + 3 ) √1 + ( 2x + 3 )2 − ( 2x + 3 ) √1 − ( 2x + 2h + 3 )2 ]
= lim
h
h→0

[Since, sin − 1x − sin − 1y = sin − 1 [x√1 − y2 − y√1 − x2 ] ]


sin − 1 z z
= lim z
× h
h→0
Where, z = (2x + 2h + 3) 1 − (2x + 3) 2 − (2x + 3) 1 − (2x + 2h + 3) 2 √ √
sin − 1 h
and lim h
=1
h→0
z
= lim h
h→0
2 2
( 2x + 2h + 3 ) √1 − ( 2x + 3 ) − ( 2x + 3 ) √1 − ( 2x + 2h + 3 )
= lim
h
h→0
( 2x + 2h + 3 ) − ( 2x + 3 ) − ( 2x + 3 ) 2 ( 1 − ( 2x + 2h + 3 ) 2 )
2 2
= lim
h→0 { ( 2x + 2h + 3 ) √1 − ( 2x + 3 ) 2 − ( 2x + 3 ) √1 − ( 2x + 2h + 3 ) 2 }
[Since, rationalizing numerator]
[ ( 2x + 3 ) 2 + 4h 2 + 4h ( 2x + 3 ) ] ( 1 − ( 2x + 3 ) 2 ) − ( 2x + 3 ) 2 [ 1 − ( 2x + 3 ) 2 − 4h 2 − 4h ( 2x + 3 ) ]
= lim 2 2
h→0 h { ( 2x + 2h + 3 ) √1 − ( 2x + 3 ) + ( 2x + 3 ) √1 − ( 2x + 2h + 3 ) }
[ ( 2x + 3 )2 + 4h2 + 4h ( 2x + 3 ) − ( 2x + 3 )4 − 4h2 ( 2x + 3 )2 − 4h ( 2x + 3 )3 − ( 2x + 3 )2 + ( 2x + 3 )3 + 4h2 ( 2x + 3 )2 + 4h ( 2x + 3 )3 ]
= lim 2 2
h→0 h { ( 2x + 2h + 3 ) √1 − ( 2x + 3 ) + ( 2x + 3 ) √1 − ( 2x + 2h + 3 ) }
4h [ h + ( 2x + 3 ) ]
= lim
h→0 h { ( 2x + 2h + 3 ) √1 − ( 2x + 3 ) 2
+ ( 2x + 3 ) √1 − ( 2x + 2h + 3 ) }
2

4h [ h + ( 2x + 3 ) ]
=
( 2x + 3 ) √1 − ( 2x + 3 ) 2 + ( 2x + 3 ) √1 − ( 2x + 3 ) 2
4 ( 2x + 3 )
=
2 ( 2x + 3 ) √1 − ( 2x + 3 ) 2
2
= 2
√1 − ( 2x + 3 )
So,
d 2
dx (sin − 1(2x + 3) ) =
√1 − ( 2x + 3 ) 2
Q6. x2 + x + 1 2 √3x dy 6 Marks
If y = log 2
x −x+1
+
√3
tan − 1 ( 1 − x ), find dx
2

Ans: Here,
x2 + x + 1 2 √3x
y = log 2
x −x+1
+
√3
tan − 1 (1−x ) 2

Differentiating it with respect to x using chain rule and quotient rule,


dy d x2 + x + 1 2 d √3x

dx
=
dx
log ( x − x + 1 ) + √3 dx tan − 1 ( 1 − x )
2 2

) + √3 { }
dy 1 d x2 + x + 1 2 1 d √3x

dx
= x2 + x + 1 (
dx x 2 − x + 1 √3x (
dx 1 − x 2 )
( x −x+1 )
2 1+ ( 1−x 2 )
d d d d
( x 2 − x + 1 ) dx ( x 2 + x + 1 ) − ( x 2 + x + 1 ) dx ( x 2 − x + 1 ) ( 1 − x 2 ) 2 dx ( √3x ) − √3x dx ( 1 − x ) 2
( x − x + 1 )( )+ √ { }{ }
dy x2 + x + 1 2 ( 1 − x )2
⇒ = 2
dx ( x2 − x + 1 )2 3 1 + x 4 − 2x 2 + 3x 2 ( 1 − x2 )2
2
dy 1 ( x 2 − x + 1 ) ( 2x + 1 ) − ( x 2 + x + 1 ) ( 2x − 1 ) 2 ( 1 − x2 )2 ( 1 − x ) ( √3 ) − √3x ( − 2x )

dx
= ( x − x + 1 )(
2 2
(x −x+1)
) + √3 ( 1 + x + x )( 2 4
( 1 − x2 )2
)
2 2
dy 2x 3 − 2x 2 + 2x + x 2 − x + 1 − 2x 3 − 2x 2 − 2x + x 2 + x + 1 2 √3 − √3x √3x

dx
= ( 4
x + 2x + 1 − x 2 2 ) + √3 ( 1 + x2 + x4
)
2
dy − 2x 2 + 2 2√ 3 ( x + 1 )
⇒ dx
= ( x + x + 1 ) + √3 ( 1 + x + x )
4 2 2 4

2 2
dy 2(1−x ) 2(x +1)
⇒ = +
dx ( x4 + x2 + 1 ) 1 + x2 + x4
dy 2 ( 1 − x2 + x2 + 1 )
⇒ =
dx 1 + x2 + x4
dy 4
⇒ =
dx 1 + x2 + x4
Q7. dy 6 Marks
Find dx
, when

https://bls.smartstudies.co.in/#/exam/pdf-preview/c59cb220-8e86-4716-9ff7-82aec16b1ade/1 3/158
5/26/24, 6:19 PM Exam Automation
2t 1 − t2
x= and y =
1 + t2 1 + t2

Ans: Here, x = 2t
1 + t2
Differentiating it with respect to t using quotient rule,
d d
( 1 + t 2 ) dt ( 2t ) − 2t dt ( 1 + t 2 )
dy
dx
= [ ( 1 + t2 )2 ]
2
( 1 + t ) ( 2 ) − 2t ( 2t )
= [ (1+t ) 2 2 ]
2 2
2 + 2t − 4t
= [ ] 2 2
(1+t )
2
2 − 2t
= [ ] 2
(1+t )
dx 2 ( 1 − t2 )
= . . . (i)
dt ( 1 + t2 )2
2 ( 1 − t2 )
And, y =
( 1 + t2 )2
Differentiating it with respect to t using quotient rule,
d d
( 1 + t 2 ) dt ( 1 − t 2 ) − ( 1 − t 2 ) dt ( 1 + t 2 )
dy
dt
= [ ( 1 + t2 )2 ]
( 1 + t 2 ) ( − 2t ) − ( 1 − t ) 2 ( 2t )
= [ (1+t ) 2 2 ]
3 3
− 2t − 2t − 2t + 2t
= [ ] 2 2
(1+t )
dy − 4t
= [
(1+t ) ]
. . . (ii)
2 2
dx
Dividing equation (ii) by (i),
dy
dt − 4t ( 1 + t2 )2
dx
= ×
( 1 − t2 )2 2 ( 1 + t2 )
dt
− 2t
=
( 1 − t2 )
dy x
dx
= − y
x 2t 1 + t2 2t
[Since y = 1 + t 2 ×
1 − t2
=
1 − t2
]
Q8. Show that the function defined by f(x) = cos (x2 ) is a continuous function. 6 Marks

Ans: It is given function is f(x) = cos(x 2)


This function f is defined for every real number and f can be written as the composition of two function as,
f = goh, where, g(x) = cosx and h(x) = x 2
First we have to prove that g(x) = cosx and h(x) = x 2 are continuous functions.
We know that g is defined for every real number.
Let k be a real number.
Then, g(k) = cos k
Now, put x = k + h
lf x → k, then h → 0
lim lim
x → kg(x) = x → kcosx
lim
= h → -0cos(k + h)
= lim
h→0
− [coskcosh − sink. sinh]
lim lim
= h → -0coskcosh − h → -0sinksinh
= coskcos0 − sinksin0
= cosk × 1 − sin × 0
= cosk
lim
∴ x→k
g(x) = g(k)
Thus, g(x) = cosx is continuous function.
Now, h(x) = x2
So, h is defined for every real number.
Let c be a real number, then h(c) = c2
lim
x → ch(x) = xlim
→ cx
2

lim
x→c
h(x) = h(c)
Therefore, h is a continuous function.
We know that for real valued functions g and h, Such that (fog) is continuous at c.
Therefore, f(x) = (goh)(x) = cos(x2) is a continuous function.
Q9. Differentiate the following functions with respect to x: 6 Marks
√1 + x + √1 − x
sin − 1 { 2 }, 0 < x < 1
Ans: √1 + x + √1 − x
Let y = sin − 1 { 2 }
Put x = cos2θ, So
√1 + cos 2θ + √1 − cos 2θ
= sin − 1 { 2 }
√2cos 2
θ+ √2sin 2
θ
= sin − 1 { 2 }
√ 2cos θ + √ 2sin θ
= sin − 1 { 2 }
1 1
= sin − 1 {cosθ ( ) + ( )sinθ }
√2 √2
π π
= sin − 1 {cosθsin ( 4 ) + cos 4 sinθ }
π
y = sin − 1 {sin (θ + 4 )} . . . . . (i)
Here, 0 < x < 1
⇒ 0 < cos2θ < 1
π
⇒ 0 < 2θ < 2
π
⇒0<θ< 4
π π π
⇒ 4
< θ+ ( 4 )< 2
https://bls.smartstudies.co.in/#/exam/pdf-preview/c59cb220-8e86-4716-9ff7-82aec16b1ade/1 4/158
5/26/24, 6:19 PM Exam Automation
So from eqaution (i),
π π π
y=θ+
4 [Since, sin − 1(sinθ) = θ, if θ ∈ ( − 2 , 2 )]
1 π
y = cos − 1x +
2 4
Differentiate it with respect to x,
dy 1 −1
dx
=
2 ( √1 − x ) + 0 2

dy −1
dx
=
2 1 − x2

Q10. Discuss the continuity and differentiability of f(x) = e | x | . 6 Marks

Ans: Given:
f(x) = e | x |

⇒ f(x) =
{ e x,
e − x,
x≥0
x<0
f is Continuity:
(LHL at x = 0)
lim f(x)
x→0−
= lim f(0 − h)
h→0
= lim e − ( 0 − h )
h→0
= lim e − h
h→0
=1
(RHL at x = 0)
lim f(x)
x→0+
= lim f(0 + h)
h→0
= lim e ( 0 + h )
h→0
=1
and f(0)
= e0 = 1
Thus, lim − f(x) = lim − f(x) = f(0)
x→0− h→0+
Hence, function is continuous at x = 0.
Differentiability at x = 0.
(LHL at x = 0)
f(x) − f ( 0 )
= lim
x−0
x→0−
f(0−h) −f(0)
= lim 0−h−0
h→0
e − (0−h) −1
= lim −h
h→0
eh − 1 eh − 1
= lim
h→0
−h
= −1 [ ∵ hlim
→0
h
=1 ]
(RHL at x = 0)
f(x) − f ( 0 )
= lim x−0
+
x→0
f(0−h) −f(0)
= lim
0+h−0
h→0
− (0−h)
e −1
= lim
h
h→0
eh − 1 eh − 1
= lim
h→0
h
=1 [ ∵ hlim
→0
h
=1 ]
LHL at (x = 0) ≠ RHL at (x = 0)
Hence the function is not differentiable at x = 0.
Q11. dy sin 2 ( a + y ) 6 Marks
If xsin(a + y) + sinacos(a + y) = 0, prove that dx
= sin a

Ans: We have, xsin(a + y) + sinacos(a + y) = 0


Differentiate with respect to x, we get
d d
⇒ dx [xsin(a + y) ] + dx [sinacos(a + y) ] = 0
d d d
[ ]
⇒ x dx sin(a + y) + sin(a + y) dx (x) + sina dx cos(a + y) = 0
d d
⇒ [xcos(a + y) dx (a + y) + sin(a + y)(1) ] + sina [ − sin(a + y) dx (a + y) ] = 0
dy dy
⇒ xcos(a + y) dx (a + y) + sin(a + y) − sinasin(a + y) dx = 0
dy
⇒ dx [xcos(a + y) − sinasin(a + y) ] = − sin(a + y)
2
dy cos ( a + y )

dx [ − sina sin ( a + y ) ]
− sinasin(a + y) = − sin(a + y)
cos ( a + y )
[∵ x = − sina
sin ( a + y ) ]
dy sin acos 2 ( a + y ) + sin asin 2 ( a + y )
⇒ −
dx [ sin ( a + y ) ]= − sin(a + y)
dy sin ( a + y )
⇒ dx
= sin(a + y) [ sin a { cos 2
( a + y ) + sin 2 ( a + y ) } ]
dy sin 2 ( a + y )
⇒ =
dx sin a
Q12. Differentiate the following functions with respect to x: 6 Marks
4x 1 1
tan − 1 ( 1 − 4x ), − 2 < x < 2
2

Ans: Let y = tan − 1 4x


{ 1 − 4x } 2

Put 2x = tanθ, so
2tan θ
y = tan − 1 { 1 − tan θ } 2

https://bls.smartstudies.co.in/#/exam/pdf-preview/c59cb220-8e86-4716-9ff7-82aec16b1ade/1 5/158
5/26/24, 6:19 PM Exam Automation

y= tan − 1{tan2θ} . . . . . (i)


1 1
Here, − 2 < x < 2
⇒ − 1 < 2x < 1
⇒ − 1 < tanθ < 1
π π
⇒ − <θ<
4 4
π π
⇒ − < (2θ) <
2 2
So, from equation (i),
π π
[
y = θ Since, tan − 1(tanθ) = θ, if θ ∈ [ − 2 , 2 ]]
y = 2tan − 1(2x) [Since, 2x = tanθ ]
Differentiating ti with respect to x using chain rule,
dy 1 d
dx
=2 ( 1 + ( 2x ) ) dx (2x) 2
dy 4
=
dx 1 + 4x 2
Q13. 6 Marks

{
π
x+ a√2sinx, if 0 ≤ x <
4
π π
Find the value of a and b so that the function f(x) defind by f(x) = 2xcot x + b, if 4
≤x< 2 becomes continuous on [0, π]
π
acos2x − bsinx, if 2
≤x≤π

Ans: Given, f is continuous on [0, π]


π π
∴ f is continuous at x and
4 2
π
At x = 4 , we have
π π π π π π
lim f(x) = lim
π− h→0
( 4 − h ) = hlim
→0
[( 4 − h ) + a√2sin ( 4 − h )] = [ 4 + a√2sin ( 4 )] = [ 4 + a ]
x→ 4
π π π π π π
lim f(x) = lim
π+ h→0
( 4 + h ) = hlim
→0
[2 ( 4 + h )cot ( 4 + h ) + b ] = [ 2 cot ( 4 ) + b ] = [ 2 + b ]
x→ 4
π
At x = 2 , we have
π π π
lim f(x) = lim
π− h→0
( 2 − h ) = hlim
→0
[2 ( 2 − h )cot ( 2 − h ) + b ] = b
x→ 2
π π π
lim f(x) = lim
π+ h→0
( 2 + h ) = hlim
→0
[acos2 ( 2 + h ) − bsin ( 2 + h )] = −a−b
x→ 2
π π
Since f is continuous at x = 4
and x = 2 , we have
lim f(x) = lim f(x) and lim f(x) = lim f(x)
π− π+ π− π+
x→ 2 x→ 2 x→ 4 x→ 4
π π
⇒ − b − a = b and +a= +b
4 2
−a −π
⇒b= . . . (i) and = b − a . . . (ii)
2 4
−π − 3a

4
=
2
[Substituting the value of b in eq. (ii)]
π
⇒a=
6
−π
⇒b= 12
[From eq. (i)]
Q14. dx y ( x 2y + x + y ) 6 Marks
If xylog(x + y) = 1, prove that = −
dx x ( xy 2 + x + y )

Ans: Here,
xy log(x + y) = 1
Differentiating with respect to x, we get
d d
⇒ dx [xylog(x + y) ] = dx
(1)
d dy d
⇒ xy log(x + y) + xlog(x + y) + ylog(x + y) (x) = 0
dx dx dx
[Using chain rule and product rule]
1 d dy
⇒ xy × ( x + y ) dx (x + y) + xlog(x + y) dx + ylog(x + y)(1) = 0
xy dy dy
⇒ (
x + y )(
1 + ) + xlog(x + y) + ylog(x + y)(1) = 0
dx dx
xy dy xy 1 dy 1
⇒ (
x + y ) dx
+(
x+y )
+ x( ) + y( ) = 0
xy dx xy
1
[Since from equation (i)log(x + y) = xy ]
dy xy 1 1 xy
⇒ [ + ]= −[ +
dx x + y y x x+y ]
2 2
dy xy + x + y x+y+x y
dx [ ( x + y ) y ]
= −[
x(x+y) ]
2
dy x+y+x y (x+y)y
dx
= − ( x ( x + y ) )( ) 2
xy + x + y
2
y x+y+x y
= − (
x x + y + xy ) 2

So,
dy y x 2y + x + y
dx
= − (
x xy 2 + x + y )
Q15. Differentiate the following functions with respect to x: 6 Marks
x+ √ 1 − x3
sin − 1 ( √2
), − 1 < x < 1
Ans: x+ √1 − x 3
Let y = sin − 1 ( √2
)
Put x = asinθ, So
2
sin θ + √1 − sin θ
= sin − 1 { √2
}
https://bls.smartstudies.co.in/#/exam/pdf-preview/c59cb220-8e86-4716-9ff7-82aec16b1ade/1 6/158
5/26/24, 6:19 PM Exam Automation
sin θ + cos θ
= sin − 1 { √2 }
1 1
= sin − 1 {sinθ ( ) + cosθ ( )}
√ 2 √2
π π
= sin − 1 {sinθcos 4 + cosθsin 4 }
π
y = sin − 1 {sin (θ + 4 )} . . . . . (i)
Here, − 1 < x < 1
⇒ − 1 < sinθ < 1
π π
⇒ − 2
<θ< 2
π π π 3π
⇒ ( − 2 + 4 ) < ( 4 + θ) < 4
So, from equation (i),
π π π
y=θ+ 4 [Since, sin − 1(sinθ) = θ, as θ ∈ [ − 2 , 2 ]]
π
y = sin − 1x + 4 [Since, sinθ = x ]
Differentiating it with respect to x,
dy 1
dx
= +0
√ 1 − x2
dy 1
=
dx 1 − x2

Q16. dy b 6 Marks
If x = asin2t(1 + cos2t) and y = b cos2t(1 − cos2t), show that ( dx ) at t = π
4
= .
a

Ans: We have, x = asin2t(1 + cos2t) and y = bcos2t(1 − cos2t)


dx d d
∴ dt
= a sin2t ⋅ [ dt
(1 + cos2t) + (1 + cos2t) dt sin2t ]
= a [sin2t ⋅ ( − 2sin2t) + (1 + cos2t) ⋅ 2cos2t ]
= − 2asin 22t + 2acos2t(1 + cos2t)
dx
⇒ dt
= − 2a [sin 22t − cos2t(1 + cos2t) ] …(i)
dy d d
and dt
= b cos2t ⋅ [ dt
(1 − cos2t) + (1 − cos2t) ⋅ dt
cos2t ]
= b [cos2t ⋅ (2sin2t) + (1 − cos2t)( − 2sin2t) ]
= 2b [sin2t ⋅ cos2t − (1 − cos2t)sin2t ]
dy
dy dt 2b [ sin 2t ⋅ cos 2t − 1 ( 1 − cos 2t ) sin 2t ]
∴ dx
= dx =
− 2a [ sin 2 2t − cos 2t ( 1 + cos 2t ) ]
dt
π π π π
dy b [ sin 2 cos 2 ( 1 − cos 2 ) sin 2 ]
⇒ ( dx ) t = π
4
= −
a π π π
[ sin2 2 − cos 2 ( 1 + cos 2 ) ]
b (0−1) b
= a
⋅ (1−0)
= a
Q17. dy 6 Marks
Find dx
y= x sin x + (sinx ) x

Ans: Let y = x sin x + (sinx) x


Also, let u = x sin x and v = (sinx) x
∴y=u+v
dv du dv
⇒ dx
=
dx
+ dx
. . . . . (i)
u= x sin x
⇒ logu = log (x sin x )
⇒ logu = sinxlogx
Differentiating both sides with respect to x, we obtain
1 du d d
u dx
= dx
(sinx) × logx + sinx × dx
(logx)
du 1
⇒ dx [
= u cotxlogx + sinx × x ]
du sin x
⇒ dx
= x sin x cosxlogx + [ x ] . . . . . (ii)
x
v = (sinx)
⇒ logv = log(sinx) x
⇒ logv = xlog(sinx)
Differentiating both sides with respect to x, we obtain
1 dv d d
v dx
= dx
(x) × log(sinx) + x × dx [log(sinx) ]
dv 1 d
⇒ dx [
= v log(sinx) + x × sin x
× dx
(sinx) ]
dv x
⇒ dx
= (sinx) x logsinx + [ sin x
cosx ]
dv
⇒ dx
(sinx) x [logsinx + xcotx ]
dv
⇒ dx
= (sinx) x [logsinx + xcotx ] . . . . . (iii)
From (i), (ii) and (iii), we obtain
dy sin x
dx
= x sin x cosxlogx + ( x ) + (sinx) x [logsinx + xcotx ]
Q18. Verify the hypothesis and conclusion of Lagrange's mean value theorem for the function 6 Marks
1
f(x) = 4x − 1
, 1 ≤ x ≤ 4.

Ans: The given function f(x) = 1


4x − 1
.
Clearly, f(x) is does not exist for x = 0
Since for each x ∈ [1, 4], the function attains a unique definite value, f(x) is continuous on [1, 4].
−4
Also, f ′ (x) = exists for all x ∈ [1, 4],
( 4x − 1 ) 2
Thus, both the conditions of Lagrange's mean value theorem are verified.
Concequently, there exists some c ∈ [1, 4] such that
f(4) −f(1)
f ′ (c) =
4−1
f(4) −f(1)
=
3
Now,

https://bls.smartstudies.co.in/#/exam/pdf-preview/c59cb220-8e86-4716-9ff7-82aec16b1ade/1 7/158
5/26/24, 6:19 PM Exam Automation
1 −4
f(x) = ⇒ f ′ (x) =
4x − 1 ( 4x − 1 ) 2
1 1
f(4) = 15
, f(1) = 3
f(4) −f(1)
∴ f ′ (x) = 4−1
1 1
15
−3 −4

⇒ f (x) = 4−1
== 45
−4 −4
⇒ =
( 4x − 1 ) 2 45
⇒ (4x − 1) 2 = 45
⇒ 16x 2 − 8x − 44 = 0
⇒ 4x 2 − 2x − 11 = 0
1
⇒x= 4 (1 + 3√5 )
1 f(4) −f(1)
Thus, c = 4 (1 + 3√5 ) ∈ (1, 4) such that f ′ (c) = 4−1
.
Hence, Lagrange's theorem is verified.
Q19. Determine the values of a, b, c for which the function 6 Marks

{
sin (a + 1 ) x + sin x
, for x < 0,
x
c, for x = 0
f(x) = is continuous at x = 0.
√ x + bx 2 − √x
3 , for x > 0
bx 2

Ans: The given function can be rewritten as,

{
sin (a + 1 ) x + sin x
, for x < 0,
x
c, for x = 0
f(x) = 2
√x + bx − √x
3 , for x > 0
bx 2

{
sin (a + 1 ) x + sin x
x
, for x < 0,

⇒ f(x) = c, for x = 0
√1 + bx − 1
, for x > 0
bx

We observe
(LHL at x = 0) = lim f(x) = lim f(0 − h) = lim f( − h)
x→0− h→0 h→0
− sin ( a + 1 ) h − sin ( − h ) − sin ( a + 1 ) h sin h
= lim
h→0
[ h ] = hlim
→0
[ h
− h ]
− sin ( a + 1 ) h sin h
= − (a − 1) lim
h→0
[ (a+1)h ] − hlim
→0
h
= −a−1

(RHL at x = 0) = lim f(x) = lim f(0 + h) = lim f(h)


x→0+ h→0 h→0
√1 + bh − 1 bh 1 1
= lim
h→0
( bh ) = hlim
→0
( bh ( √1 + bh + 1 ) ) = hlim ( 1 + bh + 1 ) = 2
→0 √
And, f(0) = c
If f(x) is continuous at x = 0, then
= lim f(x ) = lim f(x ) = f(0)
x→0− x→0+
1
⇒ −a−1= =c
2
1 1
⇒ −a−1= 2
and c = 2
−3 1
⇒a= 2
,c = 2
√1 + bx − 1
Now, bx
exists only if bx ≠ 0 ⇒ b ≠ 0.
∴ b ∈ R − {0}
Q20. y2 6 Marks


dy y
If √y + x + √y − x = c, show that dx
=
x
− 2
−1
x

Ans: Here,
√y + x + √y − x = c
Differentiating with respect to x,
d d d
⇒ (√y + x) + √y − x = (c)
dx dx dx
1 d 1 d
⇒ (y + x) + (y − z) = 0
2√y + x dx 2√y − x dx
1 dy 1 dy

2√y + x dx ( ( − 1) = 0
+1 + ) 2√y − x dx
dy 1 dy 1 1 1
⇒ dx (
2√ y + x )
+ dx (
2√ y − x )
= −
2√ y − x 2√ y + x

dy 1 1 1 √y + x − √y − x
⇒ dx
× [ √ y + x + √ y − x ] = 2 [ √ y − x√ y + x ]
dy √y − x − √y + x √y + x − √y − x
dx [ √y + x√y − x ]
⇒ = [ ]
√ y − x√ y + x
dy √y + x − √y − x √y + x − √y − x
⇒ = ×
dx √y − x + √y − x √y + x − √y + x
[Rationalizing the denominator]
dy ( y + x ) + ( y − x ) − 2√ y + x√ y − x
⇒ dx
= y+x−y+x

https://bls.smartstudies.co.in/#/exam/pdf-preview/c59cb220-8e86-4716-9ff7-82aec16b1ade/1 8/158
5/26/24, 6:19 PM Exam Automation
2 2
dy
2y − 2 y − x√
⇒ dx
= 2x

dy 2y √
2 y2 − x2
⇒ dx
= 2x
− 2x
y2 − x2


dy y
⇒ = −
dx x x2

y2


dy y
⇒ = − −1
dx x x2

Q21. Find a point on the parabola y = (x − 3)2, where the tangent is parallel to the chord joining (3, 0) and (4, 1). 6 Marks

Ans: Let,
f(x) = (x − 3)2 = x2 -6x + 9
The tangent to the curve is parallel to the chord joining the points (3, 0) and (4, 1).
Assume that the chord joins the points (a, f(a)) and (b, f(b)).
∴ a = 3, b = 4
The polynomial function is everywhere continuous and differentiable.
So, f(x) = x2 -6x + 9 is continuous on (3, 4) and differentiable on (3, 4).
Thus, both the conditions of Lagrange's theorem are satisfied.
f(4) −f(3)
Concequently, there exists c ∈ (3, 4) such that f ′ (c) = 4−3
Now,
f(x) = x2 -6x + 9
⇒ f'(x) = 2x - 6, f(3) = 0, f(4) = 1
f(4) −f(3)
∴ f ′ (x) = 4−3
1−0
⇒ 2x − 6 = 4−3
⇒ 2x = 7
7
⇒x= 2
7 f(4) −f(3)
Thus, c = 2
∈ (3, 4) such that f ′ (c) = 4−3
7 1
Clearly, f(c) = ( 2 − 3) 2
= 4
7 1
Thus, c, f(c), i.e. 2 , 4 , is a point on the given curve where the tangent is parallel to the chord joining the points (3, 0) and (4, 1).
Q22. Find the value of 'a' for which the function f defined by 6 Marks

{
π
asin 2 (x + 1), x≤0
f(x) = tan x-sinx is discontinuous at x = 0.
3
x>0
x

Ans: Since f(x) is continuous at x = 0, L.H.L = R.H.L.


Thus, we have
lim f(x) = lim f(x)
x→0− x→0+
π tan x − sin x
⇒ lim asin (x + 1) = lim
2 x3
x→0− x→0+
tan x − sin x
⇒ a × 1 = lim
x3
x→0
sin x
cos x
− sin x
⇒ a = lim
x→0 x3
sin x 1
x ( cos x − 1 )
⇒ a = lim
x→0 x2
sin x 1 − cos x
x ( cos x )
⇒ a = lim
x2
x→0
sin x 1 1 − cos x
⇒ a = lim × lim × lim
x cos x x2
x→0 x→0 x→0
1 − cos x
⇒ a = 1 × 1 × lim
x→0 x2
1 − cos x 1 + cos x
⇒ a = lim ×
x→0 x2 1 + cos x
2
1 − cos x
⇒ a = lim
x 2 ( 1 + cos x)
x→0
sin 2 x
⇒ a = lim 2
x ( 1 + cos x)
x→0
sin 2 x 1
⇒ a = lim × lim
x→0 x2 x→0
1 + cos x
1
⇒ a = 1 × lim 1 + cos x
x→0
1
⇒a=1× 1+1
1
⇒a= 2
Q23. Differentiate the following functions with respect to x: 6 Marks
x
cos − 1 { √x + a } 2 2

Ans: Let y = cos − 1 x


{ √x + a } 2 2

Let x = acotθ
acot θ
⇒ y = cos − 1 { √a cot 2 2 θ + a2
}
acot θ
⇒ y = cos − 1 { √a ( cot 2 2
θ+1)
}
acot θ
⇒ y = sin − 1 ( a cosecθ )
https://bls.smartstudies.co.in/#/exam/pdf-preview/c59cb220-8e86-4716-9ff7-82aec16b1ade/1 9/158
5/26/24, 6:19 PM Exam Automation
cos θ

⇒ y = cos − 1 ( ) sin θ
1
sin θ

⇒ y = cos − 1(cosθ)
⇒ y=θ
x
⇒ y = cot − 1 ( a ) [Since, x = acotθ ]
Differentiating it with respect to x using chain rule,
dy 1 d x
dx
= x
2 dx (a )
1+ (a )
dy − a2 1
⇒ dx
=
a2 + x2
× (a )
dy −a
∴ =
dx a2 + x2
Q24. dy 6 Marks
Find dx
y = (tanx) cot x + (cotx) tan x

Ans: Here,
y = (tanx) cot x + (cotx) tan x
( tan x ) cot x ( cot x ) tan x
y = e log + e log
[Since, log ee = 1, loga b = bloga ]
y = e cot xlog tan x + e tan xlog ( cot x )
Differentiating it with respect to x using rule and product rule,
dy d d
dx
=
dx (e cot xlog tan x ) + dx (e tan xlog cot x )
d d
= e cot xlog tan x (cotxlogtanx) + e tan xlog cot x (tanxlogcotx)
dx dx
d d d d
[cotx dx logtanx + logtanx dx cotx ] + elog ( cot x ) [tanx dx logcotx + logcotx dx (tanx) ]
cot x tan x
= e log ( tan x )

1 d 1 d
= (tanx) cot x [cotx × (
tan x ) dx
(tanx) + logtanx( − cosec 2x) ] + (cotx) tan x [tanx ( (cotx) + logcotx (sec 2x ) ]
cot x ) dx

= (tanx) cot x [(1) (sec 2x ) − cosec 2xlogtanx ] + (cot) tan x [(1) ( − cosec 2x ) + sec 2xlogcotx ]
dy
= (tanx) cot x [sec 2x − cosec 2xlogtanx ] + (cot) tan x [sec 2xlogcotx − cosec 2x ]
dx
Q25. Discuss the continuity of the cosine, cosecant, secant and cotangent functions. 6 Marks

Ans: 1. Let a be an arbitrary real number then lim + f(x) ⇒ lim + cosx ⇒ lim cos(a + h)
x→a x→a h→0
lim lim lim
⇒ h→0
(cosacos h − sinasinh) = cosa h→0
cosh − sina h→0
sinh
= cosa × 1 − sina × 0 = cosa = f(a)
Lt
∴ x → af(x) = f(a) for all a ∈ R
Therefore, f(x0 is continuous at x = a.
Since, a is an arbitrary real number, therefore cosx is continuous.
1
2. f(x) = cosec x = sin x
and domain x = R − (xπ), x ∈ I
lim 1 1 1 1
⇒ x → a sinx = lim = lim = sinacos 0 + cos asin 0
h → 0sin (a + h) h → 0 ( sinacos h + cos asin h )
1 1
= = = f(a)
sina ( 1 ) + cos a ( 0 ) sin a
Therefore, f(x) is continuous at x = a.
Since, a is an arbitrary real number, therefore, f(x) = cosec x is continuous.
1 π
3. f(x) = secx = and domain x = R − (2x + 1) 2 , x ∈ I
cos x
lim 1 1 1 1
⇒x → a = lim
= lim
=
cosx cosacos 0 − sin asin 0
h → 0cos (a + h) h → 0 ( cosacos h − sin asin h )
1 1
= = = f(a)
cosa ( 1 ) − sin a ( 0 ) cos a
Therefore, f(x) is continuous at x = a.
Since, a is an arbitrary real number, therefore, f(x) = sec x is continuous.
1
4. f(x) = cotx = and domain x = R − (xπ), x ∈ I
tan x
lim 1 1 1 1
⇒x → a = = tan a + tan h
= tan a + 0
tanx lim tan ( a + h )
h→0 lim
h→0
( 1 − tan atan h ) 1 − tan atan 0

1−0 1
tan a
= tan a
= f(a)
Therefore, f9x) is continuous at x = a.
Since, a is an arbitrary real number, therefore, f(x) = cotx is continuous.
Q26. If ysin(x x), prove that dy = cos(x x) × x x(1 + logx) 6 Marks
dx

Ans: Let y = sin(x x) . . . . . (i)


Also, Let u = x x . . . . . (ii)
Taking log on both sides,
⇒ logu = logx x
⇒ logu = xlogx
Differentiating both sides with respect to x,
1 du d
= (xlogx)
u dx dx
1 du d d
⇒ =x (logx) + logx x
u dx dx dx
1 du 1
⇒ = x( ) + logx(1)
u dx x
1 du
⇒ = 1 + logx
u dx
du
⇒ = u(1 + logx)
dx
du
⇒ = x x(1 + logx) . . . . . (iii)
dx
[Using equation (ii)]
Now, using equation (ii) in equation (i),
y = sinu
Differentiating with respect to x,
dy d
= (sinu)
dx dx
dy du
⇒ = cosu dx
dx

https://bls.smartstudies.co.in/#/exam/pdf-preview/c59cb220-8e86-4716-9ff7-82aec16b1ade/1 10/158
5/26/24, 6:19 PM Exam Automation
Using equation (ii) and (iii),
dy
= cos(x x) × x x(1 + logx)
dx
Q27. Differentiate the following functions with respect to x: 6 Marks
1
+ x (1+ x )
1 x
(x + ) x
1
( x )x + x ( 1 + )
Ans: 1
Let y = x + x
1
Also, let (x + ) x and v = x ( 1 + )
1
x
x
∴y=u+v
dy du dv
⇒ = + . . . . (1)
dx dx dx
1
Then, u = x ( + x
x )
1 x
⇒ logu = log x + ( x )
1
⇒ logu = xlog x + ( x )
Differentiating both sides with respect to x, we obtain,
1 du d 1 d 1
u dx
⋅ = ( x ) + x × dx [log (x + x )]
dx
(x) × log x +
1 du 1 1 d 1
= 1 × log (x + ) + x × ⋅ (x + ) 1
u dx x dx x
(x+ ) x
du 1 x 1 x2 − 1
⇒ ( ) [log (x + x ) + x + 1 ]
dx
= x+
x 2
2
du 1 x −1 1
⇒ dx = (x + x )x [ + log (x + x )] 2
x +1
1
v = x (1+ x )

[ ]
1
⇒ logv = log x ( 1 + x )
1
⇒ logv = 1 + ( x )logx. . . . (2)
Differentiating both sides with respect to x, we obtain
1 dv d 1 1 d
v
⋅ [ dx (1 + x )] × logx + (1 + x ) ⋅ dx logx
dx
=
1 dv 1 1 1
⇒ v dx = ( − )logx + (1 + x ) ⋅ x 2
x
1 dv log x 1 1
⇒ = − 2
+ +
v dx x x x2
dv − log x + x + 1

dx
=v [ x2 ]
dv 1 x + 1 − log x
⇒ dx
= x (1+ x ) ( x2
). . . . . (3)
Therefore, from (i), (ii) and (iii), we obtain

)[ )] + x ( 1 + ) ( x )
dy 1 x x2 − 1 1 1 x+1 − log x
dx
= x+( x x2 + 1
+ log x + ( x
x
2

Q28. 1 6 Marks
Differentiate sin − 1 (2x√1 − x 2 ) with respect to sec − 1 ( ), if:
√ +x
1 2

1
x∈ ( √2 , 1 )
Ans: Let u = sin − 1 2x 1 − x 2
( √ )
Put x = sinθ
⇒ u = sin − 1 2sinθ 1 − sin 2θ ( √ )
⇒u= sin − 1(2sinθcosθ)
⇒ u = sin − 1(sin2θ) . . . . . (i)
And,
1
Let v = sec − 1 ( √1 − x ) 2

1
⇒ v = sec − 1 ( √1 − sin θ ) 2

1
⇒ v = sec − 1 ( cos θ )
⇒ v = sec − 1(secθ)

⇒ v = cos − 1 ( ) [Since, sec


1
1
cos θ
−1
x = cos − 1 ( x )
1
]
⇒ v = cos − 1(cosθ) . . . . . (ii)
Here,
1
x∈ ( √2 , 1 )
1
⇒ sinθ ∈ ( , 1 )
√2
π π
⇒θ∈ ( , )
4 2
So, from equation (i),
π π
[
u = 2θ Since, sin − 1(sinθ) = θ, if θ ∈ ( − 2 , 2 )]
Let, u = 2sin − 1x [Since, x = sinθ ]
du 1
dx
=2 ( √1 − x ) 2

du 2
⇒ = . . . . . (iii)
dx 2
√1 − x
And, from equation (ii),
v = θ [Since, cos − 1(cosθ) = θ, if θ ∈ [0, π] ]
⇒ v = sin − 1x [Since, x = sinθ ]
dv 1
= . . . . . (iv)
dx 1 − x2

Dividing equation (iii) by (iv),

https://bls.smartstudies.co.in/#/exam/pdf-preview/c59cb220-8e86-4716-9ff7-82aec16b1ade/1 11/158
5/26/24, 6:19 PM Exam Automation
du
dx 2 √ 1 − x2
dv = × 1
dx √1 − x 2

du
∴ =2
dv
Q29. Differentiate the function given in Exercise: 6 Marks
1
(xcosx) x + (xsinx) x
1
Ans:
Let y = (xcosx) x + (xsinx) x
1
Putting u = (xcosx) xand v(xsinx) x , we have y = u + v
dy du dv
∴ = + …(i)
dx dx dx
Now u = (xcosx) x ⇒ logu = log(xcosx) 2 = xlog(xcosx)
d d
⇒ logu = x(logx + logcosx) ⇒
dx
logu =
dx {x(logx + logcosx) }
1 du 1 1

u dx
=x [ x + cos x ( − sinx) ] + (logx + logcosx).1
du
⇒ = u[1 − xtanx + log(xcosx)]
dx
du
⇒ = (xcosx) x[1 − xtanx + log(xcosx)] … (ii)
dx
1 1 1
Again v = (xsinx) x ⇒ logv = log(xsinx) x = x log(xsinx)
1 d d 1
⇒ logv = x (logx + logsinx) ⇒ { x (logx + logsinx)}
dx
logv = dx
1 dv 1 1 1 −1
⇒ v dx
= x [ x + sin x . cosx ] + (logx + logsinx) ( ) 2
x
dv 1 cot x log ( xsin x )
⇒ dx
= v[ + x − 2 ] 2
x x
dv 1 cot x 1 log ( xsin x )
⇒ = (xsinx) [ + −
x
2 ] …(iii) 2
dx x x x
Putting the values from eq. (ii) and (iii) in eq. (i),
dy 1 1 cot x log ( xsin x
dx
= (xcosx) x[1 − xtanx + log(xcosx)] + (xsinx) x [x 2
+
x

x2
]
Q30. Verify Rolle's theorem for the following function on the indicated intervals 6 Marks
f(x) = (x - 1) (x - 2)2 on [1, 2]

Ans: Given:
f(x) = (x - 1)(x - 2)2
i.e. f(x) = x3 + 4x - 4x2 - x2 - 4 + 4x
i.e. f(x) = x3 - 5x2 + 8x - 4
We know that a polynominal function is everywhere derivable and hence continuous.
So, being a polynomial function, f(x) is continuous and derivable on [1, 2]
Also,
f(1) = (1)3 - 5(1)2 + 8(1) - 4 = 0
f(2) = (2)3 - 5(2)2 + 8(2) - 4 = 0
∴ f(1) = f(2) = 0
Thus, all the continuous of Rolle's theorem are satisfied.
Now, we have to show that there exists c ∈ (1, 2) such that f'(c) = 0.
We have,
f(x) = x3 + - 5x2 - 8x - 4
⇒ f'(x) = 3x2 + 8 - 10x
∴ f'(x) = 0 ⇒ 3x2 - 10x + 8 = 0
⇒ 3x2 - 6x - 4x + 8 = 0
⇒ 3x(x - 2) - 4(x - 2) = 0
⇒ (x - 2)(3x - 4)
4
⇒ x = 2,
3
Thus, c ∈ (1, 2) such that f'(c) = 0.
Hence, Rolle's theorem is verified.
Q31. 1 − y2 6 Marks


dy
If y 1 − x 2 + x 1 − y 2 = 1, prove that
√ √ = −
dx 1 − x2

Ans: We have, y 1 − x 2 + x 1 − y 2 = 1
√ √
Let x = sinA, y = sinB
⇒ sinB 1 − sin 2A + sinA 1 − sin 2B = 1
√ √
⇒ sinBcosA + sinAcosB = 1
[ ∵ sin(x + y) = sinxcosy + cosxsiny ]
⇒ sin (A + B ) = 1
⇒ A + B = sin − 1(1)
π
⇒ sin − 1x + sin − 1y = [ ∵ x = sinA, y = sinB ]
2
Differentiating with respect to x, we get
d d d π

dx (sin − 1x ) + dx (sin − 1y ) = dx 2( )
1 1 dy
⇒ + =0
2 2 dx
√1 − x √1 − y
1 − y2


dy
⇒ = −
dx 1 − x2

Q32. Differentiate the following functions with respect to x: 6 Marks


log {x + 2 + √ x2 + 4x + 1 }

Ans: Let y = log {x + 2 +


√x2 + 4x + 1 }
Differentiate it with respect to x,
dy d
dx
=
dx[log {x + 2 + √x2 + 4x + 1 } ]
1 d 1
=
[ x + 2 + √x + 4x + 1 ]2dx [x + 2 + (x2 + 4x + 1 ) ] 2

[Using chain rule]

https://bls.smartstudies.co.in/#/exam/pdf-preview/c59cb220-8e86-4716-9ff7-82aec16b1ade/1 12/158
5/26/24, 6:19 PM Exam Automation
1 1 1 d
=
[ x + 2 + √x 4 + 4x + 1 ]
× 1+0+ [ 2 (x 2 + 4x + 1 ) 2 dx (x 2 + 4x + 1) ]
( 2x + 4 )
1+
2( √x2 + 4x + 1 )
=
[ x + 2 + √x 4 + 4x + 1 ]
√x 2 + 4x + 1 + x + 2
=
[ x + 2 + √x 2 + 4x + 1 ] × √x 2 + 4x + 1
1
= 2
√x + 4x + 1

So,
d 1
dx [log {x + 2 + √x2 + 4x + 1 } ] = √x + 4x + 1 2

Q33. Differentiate the following w.r.t. x: 6 Marks


√ 1 + x2 + √ 1 − x2
tan − 1 (√ 1+x − 2
√1 − x
2 ), − 1 < x < 1, x ≠ 0
Ans: √1 + x 2 + √1 − x 2
Let y = tan − 1 (√ 1 + x2 − √1 − x
2 )
Substituting x 2 = cos2θ, we get
√1 + cos 2θ + √1 − cos 2θ
y = tan − 1 (√ 1 + cos 2θ − √1 − cos 2θ )
√ 1 + 2cos 2 θ−1+ √1 − 1 + 2sin 2 θ
= tan − 1 (√ 1 + 2cos 2 θ − 1 − √1 − 1 + 2sin 2 θ
)
√2cos θ + √2sin θ
= tan − 1 (√ 2cos θ − √2sin θ )
2 ( cos θ + sin θ )
= tan − 1 [ √√ 2 ( cos θ − sin θ ) ]
cos θ + sin θ
= (
tan − 1 cos θ − sin θ )
cos θ + sin θ

= tan − 1 ( cos θ
cos θ − sin θ
cos θ
)
1 + tan θ
= tan − 1( 1 − tan θ )
π tan a + tan b
= tan − 1tan ( 4 + θ ) [ ∵ tan(a+b) = 1 − tan a ⋅ tan b ]
π π 1 1
∴ y= 4
+θ= 4
+ 2 cos − 1x 2 [∵ 2θ = cos − 1x 2 ⇒ θ = 2 cos − 1x 2 ]
dy 1 −1 d
⇒ dx
=0+ 2
⋅ ⋅ dx
x2
√1 − x 4
1 − 2x −x
= ⋅ =
2 1 − x4 1 − x4
√ √
dy
Find dx
of each of the functions expressed in parametric form.
Q34. 6 Marks
Show that f(x) =
{ 12x − 13,
2x 2 + 5,
if x ≤ 3
if x > 3
is differentiable at x = 3. Also, find f(3).

Ans:
Given: f(x) =
{ 12x − 13,
2x 2 + 5,
if x ≤ 3
if x > 3
We have to show that the given function is differentiable at x = 3.
We have,
f(x) − f ( 3 )
(LHL at x = 3) = lim x−3
x→3−
12(x) − 13 − 23
= lim
x−3
x→3
12x − 36
= lim
x−3
x→3
12 ( x − 3 )
= lim
x−3
x→3
= lim 12
x→3
= 12
f(x) − f ( 3 )
(RHL at x = 3) = lim x−3
x→3+
2
2x + 5 − 23
= lim x−3
x→3
2
2x − 18
= lim x−3
x→3
2 ( x2 − 9 )
= lim x−3
x→3
= lim 2(x + 3)
x→3
=2×6
= 12
Thus, (LHL at x = 3) = (RHL at x = 3) = 12.
So, f(x) is differentiable at x = 3 and f(3) = 12.
Q35. 2x 1 − x2 6 Marks
Differentiate cos − 1 2
with respect to cos − 1(1+x ) ( 1 + x ), if 0 < x < 1
2

Ans: Let u = sin − 1 2x


(1+x ) 2

Put x = tanθ,
2tan θ
u = sin − 1 ( 1 + tan θ )2

u= sin − 1(sin2θ) . . . . . (i)


1 − x2
Let v = cos − 1
1 + x2
( )
https://bls.smartstudies.co.in/#/exam/pdf-preview/c59cb220-8e86-4716-9ff7-82aec16b1ade/1 13/158
5/26/24, 6:19 PM Exam Automation
1 − tan 2 θ
= cos − 1 (
1 + tan 2 θ )
−1
v = cos (cos2θ) . . . . . (ii)
Here, 0 < x < 1
0 < tanθ < 1
π
⇒0<θ< 4
So, from equation (i),
π π
[
u = 2θ Since,sin − 1(sinθ) = θ, if θ ∈ [ 2 , 2 ]]
π
u = 2tan − 1x [Since, x = tan 2 ]
Differentiating it with respect to x,
dv 2
= . . . . . (iii)
dx 1 + x2
From equation (ii),
v = 2θ [since,cos − 1(cosθ) = θ, if θ ∈ [0, π] ]
v = 2tan − 1x[since,x = tanθ]
Differentiating it with respect to x,
dv 2
= . . . . . (iv)
dx 1 + x2
Dividing equation (iii) by (iv),
du
dx 2 ( 1 + x2 )
dv = 2 × 2
(1+x )
dx
du
=1
dv
Q36. 2x dy π 6 Marks
If y = {log cos xsinx }{log sin xcosx } − 1 + sin − 1 ( 1 + x 2 ), find dx
at x = 4

Ans: We have, y = 2x
{log cos xsinx }{log sin xcosx } − 1 + sin − 1 ( 1 + x 2 )
2x
⇒ y {log cos xsinx }{log cos xsinx } + sin − 1 ( 1 + x ) [ ∵ logab = (logba) − 1 ]
2
log sin x 2 2x log b
⇒y= [ log cos x ] + sin − 1 (1+x 2 ) [ ∵ logab = log a ]
Differentiating with respect to x,
d d log sin x 2 d 2x
dx
= [
dx log sin x ( )} ] +
dx {
sin − 1
1 + x2
d log sin x 1 d log sin x d 2x
= 2[
log sin x ] dx ( log sin x ) [ ]
⇒ + ×
dx 2x dx 1 + x 2
−1

d
√ ( 1+x ) 2
2

d
( log cos x ) dx ( log sin x ) − log sin x dx ( log cos x )
[ log sin x ] [ ]
d log sin x ( 1 + x2 ) ( 1 + x 2 ) ( 2 ) − ( 2x ) ( 2x )

dx
=2
( log cos x ) 3
+ [ √1 + z − 2x ][
2 2 ( 1 + x2 )3 ]
1 d 1 d
log cos × sin x dx ( sin x ) − log sin x × cos x dx ( cos x )
[ log cos x ] [ ] + [√
dy log sin x ( 1 + x2 ) ( 1 + x 2 ) ( 2 ) − ( 2x ) ( 2x )

dx
=2
( log cos x )3 1 + x 4 − 2x 2
][ ( 1 + x2 )2 ]
cos x sin x
log cos x ( sin x ) + log sin x × ( cos x )
[ log cos x ] [ ] + [√
dy log sin x 1 + x2 2 + 2x 2 − 4x 2

dx
=2
( log cos x )2 (1−x ) 2 3 ][ ( 1 + x2 )2 ]
dy log sin x 2
⇒ =2 (cosxlogcosx + tanxlogsinx) +
dx ( log cos x ) 3 1 + x2
π
Put x = 4
π

{ }(
log sin 4

dy
dx
=2
( log cos 4 ) 3
π
π π
cot 4 logcos 4 + tan 4 logsin 4 + 2
π π
) { 1+
1

( 4 )2
π }

dy
dx
=2 { ( log
1
1

√2
) 2 }( 1 × log
√2
1
+ 1 × log
√2
1
) + 2 ( 16 + π )
16
2

dy
2log ( √2 ) 32
⇒ =2 1
+
dx 16 + π 2
{ log ( √2 ) } 2

dy 1 32
⇒ =4 1
+
dx 16 + π 2
log ( √2 )
dy 1 32
⇒ =4 1
+
dx
− 2 log 2 16 + π 2

dy 8 32
⇒ = − +
dx log 2 16 + π 2
dy 4 1
So, ( dx ) a = π
4
=8 [ 16 + π 2 − log 2 ]
Q37. Differentiate w.r.t. x the function in Exercise: 6 Marks
2 2
xx − 3 + (x − 3) x , for x > 3
2−3 2
Ans: Let y = x x + (x − 3) x
2 2
Also, let u = x x − 3 and v = (x − 3) x
∴ y=u+v
Differentiating both sides with respect to x, we obtain
dy du dv
dx
= dx dx
+ …(1)
2
u= xx − 3
2
∴ logu = log(x x − 3)

logu = (x 2 − 3)logx
Differentiating with respect to x, we obtain
1 du d d
u
⋅ dx
= logx. dx
(x 2 − 3) + (x 2 + 3) ⋅ dx
(logx)
1 du 1
⇒ u dx
= logx ⋅ 2x + (x 2 − 3) ⋅ x


du
dx
= xx
2−3
⋅ [ x2 − 3
x
+ 2xlogx ]
Also,
2
v = (x − 3) x
2
∴ logv = log(x − 3) x

https://bls.smartstudies.co.in/#/exam/pdf-preview/c59cb220-8e86-4716-9ff7-82aec16b1ade/1 14/158
5/26/24, 6:19 PM Exam Automation

⇒ logv = − 3) x 2log(x
Differentiating both sides with respect to x, we obtain
1 dv d d
⋅ = log(x − 3) ⋅ (x 2) + x 2 ⋅ [log(x − 3) ]
v dx dx dx
1 dv 1 d
⇒ = log(x − 3) ⋅ 2x + x 2 ⋅ ⋅ (x − 3)
v dx x−3 dx


dv
dx [
= v 2xlog(x − 3) +
x2
x−3
⋅1 ]

dv
dx
= (x − 3) x
2
[ x2
x−3
+ 2xlog(x − 3) ]
du dv
Substituting the expressions of dx
and dx
in equation (1), we obtain

[ ] [ ]
2
dy 2
−3 x −3 xx
dx
= xx x
+ 2xlogx + (x − 3) x x−3
+ 2xlog(x − 3)

Q38. Differentiate the following w.r.t. x: 6 Marks


(x + 1) 2(x + 2) 3(x + 3) 4

Ans: Let y = (x + 1) 2(x + 2) 3(x + 3) 4


∴ logy = log {(x + 1) 2 ⋅ (x + 2) 3(x + 3) 4 }
= log(x + 1) 2 + log(x + 2) 3 + log(x + 3) 4
d dy d d d
and dy
logy ⋅
dx
=
dx [2log(x + 1) ] + dx [3log(x + 2) ] + dx [4log(x + 3) ]
1 dy 2 d 1 d 1 d d 1
y

dx
=
(x+1)

dx
(x + 1) + 3 ⋅
(x+2)

dx
(x + 2) + 4 ⋅
(x+3)

dx [
(x + 3) ∵
dx
(logx) =
x ]
2 3 4
= [x+1 + x+2 + x+3 ]
dy 2 3 4
= y[
x+3 ]
∴ + +
dx x+1 x+2
2 3 4
= (x + 1) 2 ⋅ (x + 2) 3 ⋅ (x + 3) 4 [
x+3 ]
+ +
x+1 x+2
= (x + 1) 2 ⋅ (x + 2) 3 ⋅ (x + 3) 4

[ 2(x+2) (x+3) +3(x+1) (x+3) +4(x+1) (x+2)


(x+1) (x+2) (x+3) ]
2 3 4
(x+1) (x+2) (x+3)
=
(x+1) (x+2) (x+3)
[2(x 2 + 5x + 6) + 3(x 2 + 4x + 3) + 4(x 2 + 3x + 2) ]
= (x + 1)(x + 2) 2(x + 3) 3
[2x 2 + 10x + 12 + 3x 2 + 12x + 9 + 4x 2 + 12x + 8 ]
= (x + 1)(x + 2) 2(x + 3) 3 [9x 2 + 34x + 29 ]
Q39. dx 6 Marks
If x√1 + y + y√1 + x = 0, prove that (1 + x) 2 dx + 1 = 0

Ans: We have x√1 + y + y√1 + x = 0


⇒ x√1 + y = − y√1 + x
Squaring both sides, we get,
⇒ (x√1 + y ) 2 = ( − y√1 + x ) 2
⇒ x 2 (1 + y ) = y 2(1 + x )
⇒ x 2 + x 2y = y 2 + y 2x
⇒ x 2 − y 2 = y 2x − x 2y
⇒ (x − y)(x + y) = xy(y − x)
⇒ (x + y) = − xy
⇒ y + xy = − x
⇒ y(1 + x) = − x
−x
⇒y= (1+x)
Differentiating with respect to x, we get
d d
− ( 1 + x ) dx ( x ) − ( − x ) dx ( x + 1 )

dy
dx
= [ ( 1 + x )2 ]
dy − (1+x) (1) +x(1)
⇒ [ (1+x)
dx
= 2 ]
dy
−1−x+x
⇒ dx = [ ] 2
(1+x)
dy −1
⇒ =
dx ( 1 + x )2

⇒ (1 + x) 2 dy = −1
dx

⇒ (1 + x) 2 dy +1=0
dx
Q40. Find the local maxima and local minima, if any, of the following function. Also find the local maximum and the local minimum values, as the case may be: 6 Marks
1 π
f(x) = sinx + 2 cos2x, 0 ≤ x ≤ 2

Ans: f(x) = sinx + 1 cos2x,


2
⇒ f ′ (x) = cosx − sin2x
f ′x = 0
⇒ cosx − 2sinxcosx = 0
⇒ cosx(1 − 2sinx) = 0
π π
⇒x= 2
or x = 6
π π
x= 6
∈ 0, ( 2)
f ″ (x)
= − sinx − 2cos2x
\text{f}''\Big(\frac{\pi}{6}\Big)<0\Rightarrow\text{x}=\frac{\pi}{6} is a local maxima.
Local Max. Value =\text{f}\Big(\frac{\pi}{6}\Big)=\frac{3}{4}
Local extreme values do exist at end points \text{x}=0,\ \text{x}=\frac{\pi}{2} but no marks are alotted here for that.
Q41. Differentiate the following functions with respect to x: 6 Marks
\frac{3\text{x}^2\sin\text{x}}{\sqrt{7-\text{x}^2}}

Ans: Let \text{y}=\frac{3\text{x}^2\sin\text{x}}{\sqrt{7-\text{x}^2}}


Differentiate it with respect to x,
\frac{\text{dy}}{\text{dx}}=\frac{\text{d}}{\text{dx}}\bigg\{\frac{3\text{x}^2\sin\text{x}}{(\sqrt{7-\text{x}^2})^\frac{1}{2}}\bigg\}
=\frac{(7-\text{x}^2)^\frac{1}{2}\times\frac{\text{d}}{\text{dx}}(3\text{x}^3\sin\text{x})-(3\text{x}^2\sin\text{x})\frac{\text{d}}{\text{dx}}(7-\text{x}^2)^\frac{1}{2}}{\Big[(7-
\text{x}^2)^\frac{1}{2}\Big]^2}

https://bls.smartstudies.co.in/#/exam/pdf-preview/c59cb220-8e86-4716-9ff7-82aec16b1ade/1 15/158
5/26/24, 6:19 PM Exam Automation
[Using quotient rule, chain rule and product rule]
\Bigg[\frac{(7-\text{x}^2)^\frac{1}{2}\times3\Big[\text{x}^2\frac{\text{d}}{\text{dx}}(\sin\text{x})+\sin\text{x}\frac{\text{d}}{\text{dx}}(\text{x}^2)\Big]-3\text{x}^2\sin\text{x}\times\frac{1}
{2}(7-\text{x}^2)\times\frac{\text{d}}{\text{dx}}(7-\text{x}^2)}{(7-\text{x}^2)}\Bigg]
\Bigg[\frac{(7-\text{x}^2)^\frac{1}{2}3(\text{x}^2\cos\text{x}+2\text{x}\sin\text{x})-3\text{x}^2\sin\text{x}\times\frac{1}{2}(7-\text{x}^2)^\frac{-1}{2}(-2\text{x})}{(7-\text{x}^2)}\Bigg]
=\Bigg[\frac{(7-\text{x}^2)^\frac{1}{2}\times3(\text{x}^2\cos+2\text{x}\sin\text{x})}{(7-\text{x}^2)}+\frac{3\text{x}^2\sin\text{x}(7-\text{x}^2)^\frac{-1}{2}}{(7-\text{x}^2)}\Bigg]
\bigg[\frac{6\text{x}\sin\text{x}+3\text{x}^2\cos\text{x}}{\sqrt{(7-\text{x}^2)}}+\frac{3\text{x}^3\sin\text{x}}{(7-\text{x}^2)^\frac{3}{2}}\bigg]
So,
\frac{\text{d}}{\text{dx}}\Big(\frac{3\text{x}^2\sin\text{x}}{\sqrt{7-\text{x}^2}}\Big)\bigg[\frac{6\text{x}\sin\text{x}+3\text{x}^2\cos\text{x}}{\sqrt{(7-
\text{x}^2)}}+\frac{3\text{x}^3\sin\text{x}}{(7-\text{x}^2)^\frac{3}{2}}\bigg]
Q42. Find \frac{\text{dy}}{\text{dx}}, when 6 Marks
\text{x}=\text{e}^{\theta}\Big(\theta+\frac{1}{\theta}\Big)\text{ and y}=\text{e}^{-\theta}\Big(\theta-\frac{1}{\theta}\Big)

Ans: We have, \text{x}=\text{e}^\theta\Big(\theta+\frac{1}{\theta}\Big)


\frac{\text{dx}}{\text{d}\theta}=\text{e}^\theta\frac{\text{d}}{\text{d}\theta}\Big(\theta+\frac{1}{\theta}\Big)+\Big(\theta+\frac{1}{\theta}\Big)\frac{\text{d}}{\text{d}\theta}
(\text{e}^\theta)
[Using product rule]
\Rightarrow\frac{\text{dx}}{\text{d}\theta}=\text{e}^\theta\Big(1-\frac{1}{\theta^{2}}\Big)+\Big(\frac{\theta^{2}+1}{\theta}\Big)\text{e}^{\theta}
\Rightarrow\frac{\text{dx}}{\text{d}\theta}=\text{e}^{\theta}\Big(1-\frac{1}{\theta^{2}}+\frac{\theta^{2}+1}{\theta}\Big)
\Rightarrow\frac{\text{dx}}{\text{d}\theta}=\text{e}^\theta\Big(\frac{\theta^{2}-1+\theta^{3}+\theta}{\theta{2}}\Big)
\Rightarrow\frac{\text{dx}}{\text{d}\theta}=\frac{\text{e}^\theta(\theta^{3}+\theta^{2}+\theta-1)}{\theta^{2}}\ .....(\text{i})
And, \text{y}=\text{e}^\theta\Big(\theta-\frac{1}{\theta}\Big)
Q43. If the functions f(x), defined below is continuous at x = 0, find the value of k. 6 Marks
\text{f(x)}=\begin{cases}\frac{1-\cos2\text{x}}{2\text{x}^2},&\text{x}<0\\\text{k},&\text{x}=0\\\frac{\text{x}}{|\text{x}|},&\text{x}>0\end{cases}

Ans: Given, \text{f(x)}=\begin{cases}\frac{1-\cos2\text{x}}{2\text{x}^2},&\text{x}<0\\\text{k},&\text{x}=0\\\frac{\text{x}}{|\text{x}|},&\text{x}>0\end{cases}


\text{f(x)}=\begin{cases}\frac{1-\cos2\text{x}}{2\text{x}^2},&\text{x}<0\\\text{k},&\text{x}=0\\1,&\text{x}>0\end{cases}
We have,
(\text{LHL at x}= 0)=\lim_\limits{\text{x}\rightarrow0^-}\text{f(x)}=\lim_\limits{\text{h}\rightarrow0}\text{f}(0-\text{h})
=\lim_\limits{\text{h}\rightarrow 0}\Big(\frac{1-\cos2(-\text{h})}{2(-\text{h})^2}\Big)
=\lim_\limits{\text{h}\rightarrow 0}\Big(\frac{1-\cos2\text{h}}{2\text{h}^2}\Big)
=\frac{1}{2}\lim_\limits{\text{h}\rightarrow 0}\Big(\frac{2\sin^2\text{h}}{\text{h}^2}\Big)
=\frac{2}{2}\lim_\limits{\text{h}\rightarrow 0}\Big(\frac{\sin^2\text{h}}{\text{h}^2}\Big)
=\frac{2}{2}\lim_\limits{\text{h}\rightarrow 0}\Big(\frac{\sin^2\text{h}}{\text{h}}\Big)^2
=1\times1
=1
(\text{RHL at x}= 0)=\lim_\limits{\text{x}\rightarrow0^+}\text{f(x)}=\lim_\limits{\text{h}\rightarrow0}\text{f}(0+\text{h})
=\lim_\limits{\text{h}\rightarrow 0}\text{f(h)}=\lim_\limits{\text{h}\rightarrow 0}(1)=1
Also, \text{f}(0)=\text{k}
If f(x) is continuous at x = 0, then
\lim_\limits{\text{x}\rightarrow 0^-}\text{f(x)}=\lim_\limits{\text{x}\rightarrow 0^+}=\text{f}(0)
\Rightarrow1=1=\text{k}
Hence, the required value of k is 1
Q44. If yx + xy + xx = ab, find \frac{\text{dy}}{\text{dx}} 6 Marks

Ans: Given that yx + xy + xx = ab


Putting u = yx, v = xy and w = xx, we get u + v + w = ab
Therefore \frac{\text{du}}{\text{dx}}+\frac{\text{dv}}{\text{dx}}+\frac{\text{dw}}{\text{dx}}=0\ .....(\text{i})
Now, u = yx. Taking logrithm on both sides, we have
\log\text{u}=\text{x}\log\text{y}
Differentiating both sides w.r.t. x, we have
\frac{1}{\text{u}}\times\frac{\text{du}}{\text{dx}}=\text{x}\frac{\text{d}}{\text{dx}}(\log\text{y})+\log\text{y}\frac{\text{d}}{\text{dx}}(\text{x})
=\text{x}\frac{1}{\text{y}}\times\frac{\text{dy}}{\text{dx}}+\log\text{y}\times1
So, \frac{\text{du}}{\text{dx}}=\text{u}\Big(\frac{\text{x}}{\text{y}}\frac{\text{dy}}{\text{dx}}+\log\text{y}\Big)=\text{y}^\text{x}\Big[\frac{\text{x}}{\text{y}}\frac{\text{dy}}
{\text{dx}}+\log\text{y}\Big]\ .....(\text{ii})
Also v = xy
Taking logarithm on both sides, we have
\log\text{v}=\text{y}\log\text{x}
Differentiating both sides w.r.t. x, we have
\frac{1}{\text{v}}\times\frac{\text{dv}}{\text{dx}}=\text{y}\frac{\text{d}}{\text{dx}}(\log\text{x})+\log\text{x}\frac{\text{dy}}{\text{dx}}
=\text{y}\times\frac{1}{\text{x}}+\log\text{x}\times\frac{\text{dy}}{\text{dx}}
So, \frac{\text{dv}}{\text{dx}}=\text{v}\Big[\frac{\text{y}}{\text{x}}+\log\text{x}\times\frac{\text{dy}}{\text{dx}}\Big]
=\text{x}^\text{y}\Big[\frac{\text{y}}{\text{x}}+\log\text{x}\times\frac{\text{dy}}{\text{dx}}\Big]
Again w = xx
Taking logarithm on both sides, we have
\log\text{w}=\text{x}\log\text{x}
Differentiating both sides w.r.t x, we have
\frac{1}{\text{w}}\times\frac{\text{dw}}{\text{dx}}=\text{x}\frac{\text{d}}{\text{dx}}(\log\text{x})+\log\text{x}\frac{\text{x}}{\text{dx}}(\text{x})
=\text{x}.\frac{1}{\text{x}}+\log\text{x}\times1
i.e. \frac{\text{dw}}{\text{dx}}=\text{w}(1+\log\text{x})
=\text{x}^\text{x}(1+\log\text{x})\ .....(\text{iv})
From (i), (ii), (iii), (iv), we have
\text{y}^\text{x}\Big(\frac{\text{x}}{\text{y}}\frac{\text{dy}}{\text{dx}}+\log\text{y}\Big)+\text{x}^\text{y}\Big(\frac{\text{y}}{\text{x}}+\log\text{x}\frac{\text{dy}}{\text{dx}}\Big) \\
+\text{x}^\text{x}(1+\log\text{x})=0
\big(\text{x}\times\text{y}^{\text{x}-1}+\text{x}^\text{y}\times\log\text{x}\big) \\ \frac{\text{dy}}{\text{dx}}=-\text{x}^\text{x}(1+\log\text{x})-\text{y}\times\text{x}^{\text{y}-1}-
\text{y}^\text{x}\log\text{y}=0
\therefore\frac{\text{dy}}{\text{dx}}=\frac{-\big[\text{y}^\text{x}\log\text{y}+\text{y}\times\text{x}^{\text{y}-1}+\text{x}^\text{x}(1+\log\text{x})\big]}
{\text{x}\times\text{y}^{\text{x}-1}+\text{x}^\text{y}\log\text{y}}
Q45. Differentiate the following functions with respect to x: 6 Marks
\frac{\sqrt{\text{x}^2+1}+\sqrt{\text{x}^2-1}}{\sqrt{\text{x}^2+1}-\sqrt{\text{x}^2-1}}

Ans: We have, \frac{\sqrt{\text{x}^2+1}+\sqrt{\text{x}^2-1}}{\sqrt{\text{x}^2+1}-\sqrt{\text{x}^2-1}}


By rationalising we get,
\frac{\sqrt{\text{x}^2+1}+\sqrt{\text{x}^2-1}}{\sqrt{\text{x}^2+1}-\sqrt{\text{x}^2-1}}\times\frac{\sqrt{\text{x}^2+1}+\sqrt{\text{x}^2-1}}{\sqrt{\text{x}^2+1}+\sqrt{\text{x}^2-1}}
=\frac{\big(\sqrt{\text{x}^2+1}+\sqrt{\text{x}^2-1}\big)^2}{\big(\sqrt{\text{x}^2+1}\big)^2-\big(\sqrt{\text{x}^2-1}\big)^2}
=\frac{\big(\sqrt{\text{x}^2+1}\big)^2+\big(\sqrt{\text{x}^2-1}\big)^2+2\big(\sqrt{\text{x}^2+1}\big)\big(\sqrt{\text{x}^2-1}\big)}{\text{x}^2+1-\text{x}^2+1}
=\frac{\text{x}^2+1+\text{x}^2-1+2\sqrt{\text{x}^4-1}}{2}
=\frac{2\text{x}^2+2\sqrt{\text{x}^4-1}}{2}
=\text{x}^2+\sqrt{\text{x}^4-1}

https://bls.smartstudies.co.in/#/exam/pdf-preview/c59cb220-8e86-4716-9ff7-82aec16b1ade/1 16/158
5/26/24, 6:19 PM Exam Automation
Now, Let \text{y}=\text{x}^2+\sqrt{\text{x}^4-1}
Differentiate it with respect to x we get,
\frac{\text{dy}}{\text{dx}}=\frac{\text{d}}{\text{dx}}\big(\text{x}^2+\sqrt{\text{x}^4-1}\big)
=2\text{x}+\frac{1}{2\sqrt{\text{x}^4-1}}\times\frac{\text{d}}{\text{dx}}(\text{x}^4-1)
=2\text{x}+\frac{1}{2\sqrt{\text{x}^4-1}}\times(4\text{x}^3)
=2\text{x}+\frac{2\text{x}^3}{\sqrt{\text{x}^4-1}}
Q46. Discuss the continuity and differentiability of the f(x) = |x| + |x - 1| in the interval (-1, 2). 6 Marks

Ans: f(x) = |x| + |x - 1| in the interval (-1, 2).


\text{f(x)}=\begin{cases}\text{x}+\text{x}+1 & -1<\text{x}<0\\1 & 0\leq\text{x}\leq1\\-\text{x}-\text{x}+1&1<\text{x}<2\end{cases}
\text{f(x)}=\begin{cases}2\text{x}+1 & -1<\text{x}<0\\1 & 0\leq\text{x}\leq1\\-2\text{x}+1&1<\text{x}<2\end{cases}
We know that a polynomial and a constant function is continuous and differentiable everywhere.
So, f(x) is continuous and differentiable for \text{x}\in(-1,0),\text{x}\in(0,1) and (1, 2).
We need to check continuitly and differentiability at x = 0 and x = 1
Continuity at x = 0
\lim_\limits{\text{x}\rightarrow0^{-}}\text{f(x)}=\lim_\limits{\text{x}\rightarrow0^{-}}2\text{x}+1=1
\lim_\limits{\text{x}\rightarrow0^{+}}\text{f(x)}=\lim_\limits{\text{x}\rightarrow0^{+}}1=1
\text{f(0)}=1
\lim_\limits{\text{x}\rightarrow0^{+}}\text{f(x)}=\lim_\limits{\text{x}\rightarrow0^{-}}\text{f(x)}=\text{f(x)}
\therefore f(x) is continuous at x = 0.
Continuity at x = 1
\lim_\limits{\text{x}\rightarrow1^{-}}\text{f(x)}=\lim_\limits{\text{x}\rightarrow1^{-}}1=1
\lim_\limits{\text{x}\rightarrow1^{+}}\text{f(x)}=\lim_\limits{\text{x}\rightarrow1^{+}}1=1
\text{f(x)}=1
\lim_\limits{\text{x}\rightarrow1^{-}}\text{f(x)}=\lim_\limits{\text{x}\rightarrow1^{+}}\text{f(x)}=\text{f}(1)
\therefore f(x) is continuous at x = 1.
Q47. Show that the function \text{f(x)}=\begin{cases}|2\text{x}-3||\text{x}|, & \text{x}\geq1\\\sin\Big(\frac{\pi\text{x}}{2}\Big),& \text{x}>1\end{cases} is continuous but not differentiable at x 6 Marks
= 1.

Ans: Given: \text{f(x)}=\begin{cases}|2\text{x}-3||\text{x}|, & \text{x}\geq1\\\sin\Big(\frac{\pi\text{x}}{2}\Big),& \text{x}>1\end{cases}


Continuity at x = 1:
(LHL at x = 1) = \lim_\limits{\text{x}\rightarrow1^{-}}\text{f(x)}=\lim_\limits{\text{h}\rightarrow0}\text{f}(1-\text{h})=\lim_\limits{\text{h}\rightarrow0}\sin\Big(\frac{\pi(1-\text{h})}
{2}\Big)=\sin\frac{\pi}{2}=1
(RHL at x = 1) =\lim_\limits{\text{x}\rightarrow1^{+}}\text{f(x)}=\lim_\limits{\text{h}\rightarrow0}\text{f}(1+\text{h})
\Rightarrow\ \lim_\limits{\text{h}\rightarrow0}|2(1+\text{h})-3|[1+\text{h}]=\lim_\limits{\text{h}\rightarrow0}|2(1+\text{h})-3|=1
Hence, (LHL at x = 1) = (RHL at x = 1)
Differentiable at x = 1:
(LHL at x = 1) =\lim_\limits{\text{x}\rightarrow1^{-}}\frac{\text{f(x)}-\text{f}(1)}{\text{x}-1}
(LHL at x = 1) =\lim_\limits{\text{h}\rightarrow0}\frac{\text{f}(1-\text{h})-\text{f}(1)}{1-\text{h}-1}
(LHL at x = 1) =\lim_\limits{\text{h}\rightarrow0}\frac{\text{f}(1-\text{h})-\text{f}(1)}{-\text{h}}
(LHL at x = 1) =\lim_\limits{\text{h}\rightarrow0}\frac{\sin\Big(\frac{\pi(1-\text{h})}{2}\Big)-1}{-\text{h}}
(LHL at x = 1) =\lim_\limits{\text{h}\rightarrow0}\frac{\cos\frac{\pi\text{h}}{2}-1}{-\text{h}}
(LHL at x = 1) =-\frac{\pi}{2}\lim_\limits{\text{h}\rightarrow0}\frac{\cos\frac{\pi\text{h}}{2}-1}{\frac{\pi}{2}\text{h}}=0
(RHL at x = 1) =\lim_\limits{\text{x}\rightarrow1^{+}}\frac{\text{f(x)}-\text{f}(1)}{\text{x}-1}
(RHL at x = 1) =\lim_\limits{\text{h}\rightarrow0}\frac{\text{f}(1+\text{h})-\text{f}(1)}{1+\text{h}-1}
(RHL at x = 1) =\lim_\limits{\text{h}\rightarrow0}\frac{\text{f}(1+\text{h})-\text{f}(1)}{\text{h}}
(RHL at x = 1) =\lim_\limits{\text{h}\rightarrow0}\frac{-(2(1+\text{h})-3)-1}{\text{h}}
(RHL at x = 1) =\lim_\limits{\text{h}\rightarrow0}\frac{-2\text{h}}{\text{h}}=-2
\text{LHL}\neq\text{RHL}
Hence, the function is continuous but not differentiable at x = 1.
Q48. Using integration, find the area of the region bounded by the line y = 3x + 2, the x-axis and the ordinates x = -2 and x = 1. 6 Marks

Ans:

Required Area =\Big|\int\limits_{-2}^{\frac{-2}{3}}(3\text{x}+2)\text{dx}\Big|\int\limits^{1}_{\frac{-2}{3}}(3\text{x}+2)\text{dx}


=\bigg|\frac{+(3\text{x}+2)}{6}\bigg|^\frac{-2}{3}_{2}+\frac{(3\text{x}+2)}{6}\bigg|_{\frac{-2}{3}}^1
=\frac{8}{3}+\frac{25}{6}
=\frac{41}{6}
Q49. Differentiate the following functions with respect to x: 6 Marks
\log\sqrt{\frac{1-\cos\text{x}}{1+\cos\text{x}}}

Ans: Let, \text{y}=\log\sqrt{\frac{1-\cos\text{x}}{1+\cos\text{x}}}


\Rightarrow\ \text{y}=\log\Big(\frac{1-\cos\text{x}}{1+\cos\text{x}}\Big)^\frac{1}{2}
\Rightarrow\ \text{y}=\frac{1}{2}\log\Big(\frac{1-\cos\text{x}}{1+\cos\text{x}}\Big) \big[\text{Using }\log\text{a}^\text{b}=\text{b}\log\text{a}\big]
Differentiate with respect to x we get,
\frac{\text{dy}}{\text{dx}}=\frac{\text{d}}{\text{dx}}\Big\{\frac{1}{2}\log\Big(\frac{1-\cos\text{x}}{1+\cos\text{x}}\Big)\Big\}
=\frac{1}{2}\times\frac{1}{\Big(\frac{1-\cos\text{x}}{1+\cos\text{x}}\Big)}\times\frac{\text{d}}{\text{dx}}\Big(\frac{1-\cos\text{x}}{1+\cos\text{x}}\Big)
[Using chain rule]
=\frac{1}{2}\Big(\frac{1+\cos\text{x}}{1-\cos\text{x}}\Big)\bigg[\frac{(1+\cos\text{x})\frac{\text{d}}{\text{dx}}(1-\cos\text{x})-(1-\cos\text{x})\frac{\text{d}}{\text{dx}}(1+\cos\text{x})}
{(1+\cos\text{x})^2}\bigg]
=\frac{1}{2}\Big(\frac{1+\cos\text{x}}{1-\cos\text{x}}\Big)\bigg[\frac{(1+\cos\text{x})(\sin\text{x})-(1-\cos\text{x})(-\sin\text{x})}{(1+\cos\text{x})^2}\bigg]
=\frac{1}{2}\Big(\frac{1+\cos\text{x}}{1-\cos\text{x}}\Big)\Big[\frac{\sin\text{x}+\sin\text{x}\cos\text{x}+\sin\text{x}-\sin\text{x}\cos\text{x}}{(1+\cos\text{x})^2}\Big]
=\frac{1}{2}\Big(\frac{1+\cos\text{x}}{1-\cos\text{x}}\Big)\Big[\frac{2\sin\text{x}}{(1+\cos\text{x})^2}\Big]
=\frac{\sin\text{x}}{(1-\cos\text{x})(1+\cos\text{x})}
=\frac{\sin\text{x}}{1-\cos^2\text{x}}
=\frac{\sin\text{x}}{\sin^2\text{x}}
=\frac{1}{\sin\text{x}}
=\text{cosec x}
So,
\frac{\text{d}}{\text{dx}}\Big(\log\sqrt{\frac{1-\cos\text{x}}{1+\cos\text{x}}}\Big)=\text{cosec x}
Q50. Differentiate (\cos\text{x})^{\sin\text{x}} with respect to (\sin\text{x})^{\cos\text{x}} 6 Marks

Ans: Let, \text{u} = (\cos)^{\sin\text{x}}

https://bls.smartstudies.co.in/#/exam/pdf-preview/c59cb220-8e86-4716-9ff7-82aec16b1ade/1 17/158
5/26/24, 6:19 PM Exam Automation
Taking log on both sides,
\log\text{u} = \log(\cos\text{x})^{\sin \text{x}}
\Rightarrow \log\text{u} = \sin \text{x}\log(\cos\text{x})
Differentiating it with respect to x using rule,
\frac{1}{\text{u}}\frac{\text{du}}{\text{dx}}=\sin \text{x}\frac{\text{d}}{\text{dx}}(\log\cos\text{x})+\log \cos \text{x}\frac{\text{d}}{\text{dx}}(\sin\text{x})
[using product rule]
\Rightarrow\frac{1}{\text{u}}\frac{\text{du}}{\text{dx}}=\sin \text{x}\big(\frac{1}{\cos\text{x}}\big)\frac{\text{d}}{\text{dx}}(\cos\text{x})+\log\cos\text{x}(\cos\text{x})
\Rightarrow\frac{\text{du}}{\text{dx}}=\text{u}[(\tan\text{x})\times(-\sin\text{x})+\log\log\text{x}(\cos\text{x})]
\Rightarrow\frac{\text{du}}{\text{dx}}=(\cos\text{x})^{\sin\text{x}}[\cos\text{x}\log\cos\text{x}-\sin\text{x}\tan\text{x}]\ .....\text{(i)}
Let, \text{v = }(\sin\text{x})^{\cos\text{x}}
Taking log on both sides,
\log\text{v}=\log(\sin\text{x})^{\cos\text{x}}
\Rightarrow\log\text{v}=\cos\text{x}\log(\sin\text{x})
Differentiating it with respect to x using chain rule,
\frac{1}{\text{v}}\frac{\text{dv}}{\text{dx}}=\cos\text{x}\frac{\text{d}}{\text{dx}}(\log\sin\text{x})+\log\sin\text{x}\frac{\text{d}}{\text{dx}}(\cos\text{x})
[using product rule]
\Rightarrow\frac{1}{\text{v}}\frac{\text{dv}}{\text{dx}}=\cos\text{x}\big(\frac{1}{\sin\text{x}}\big)\frac{\text{d}}{\text{dx}}(\sin\text{x})+\log\sin\text{x}(-\sin\text{x})
\Rightarrow\frac{\text{dv}}{\text{dx}}=\text{v}[\cot\text{x}(\cos \text{x})-\sin\text{x}\log\sin\text{x}]
\Rightarrow\frac{\text{dv}}{\text{dx}}=(\sin\text{x})^{\cos\text{x}}[\cot \text{x} (\cos \text{x})-\sin \text{x}\log\sin\text{x}]\ ...(\text{ii})
Dividing equation (i) by (ii)
\therefore\frac{\text{du}}{\text{dv}}=\frac{(\cos\text{x})^{\sin\text{x}}[\cos \text{x}\log\cos\text{x}-\sin\text{x}\tan\text{x ]}}{(\sin\text{x})^{\cot\text{x}}[\cot \text{x}(\cos\text{x})-
\sin\text{x}\log\sin\text{x}]}
Q51. Differentiate the following functions with respect to x: 6 Marks
\cos^{-1}\Big(\frac{\text{x}+\sqrt{1-\text{x}^2}}{\sqrt{2}}\Big),-1<\text{x}<1

Ans: Let \text{y}=\cos^{-1}\Big(\frac{\text{x}+\sqrt{1-\text{x}^2}}{\sqrt{2}}\Big)


Put \text{x}=\cos\theta
\text{y}=\sin^{-1}\Big\{\frac{\cos\theta+\sqrt{1-\cos^2\theta}}{\sqrt{2}}\Big\}
\text{y}=\cos^{-1}\Big\{\frac{\cos\theta+\sin\theta}{\sqrt{2}}\Big\}
\text{y}=\cos^{-1}\Big\{\cos\theta\Big(\frac{1}{\sqrt{2}}\Big)+\sin\theta\Big(\frac{1}{\sqrt{2}}\Big)\Big\}
\text{y}=\cos^{-1}\Big\{\cos\theta\cos\frac{\pi}{2}+\sin\theta\sin\frac{\pi}{2}\Big\}
\text{y}=\cos^{-1}\Big\{\cos\Big(\theta-\frac{\pi}{4}\Big)\Big\}\ .....(\text{i})
Here, -1<\text{x}<1
\Rightarrow -1<\cos\theta<1
\Rightarrow\frac{3\pi}{4}<\theta<\frac{5\pi}{4}
\Rightarrow\Big(\frac{3\pi}{4}-\frac{\pi}{4}\Big)<\Big(\theta-\frac{\pi}{4}\Big)<\frac{5\pi}{4}-\frac{\pi}{4}
\Rightarrow\Big(\frac{\pi}{4}\Big)<\Big(\theta-\frac{\pi}{4}\Big)<\pi
So, from equation (i),
\text{y}=\Big(\theta-\frac{\pi}{4}\Big)\ \big[\text{Since}, \cos^{-1}(\cos\theta)=\theta,\text{ if }\theta\in[0,\pi]\big]
\text{y}=\cos^{-1}\text{x}-\frac{\pi}{4}\big[\text{Since, x}=\sin\theta\big]
Differentiating it with respect to x,
\frac{\text{dy}}{\text{dx}}=-\frac{1}{\sqrt{1-\text{x}^2}}+0
\frac{\text{dy}}{\text{dx}}=-\frac{1}{\sqrt{1-\text{x}^2}}
Q52. If \text{y}^\text{x}=\text{e}^{\text{x}-\text{e}}, prove that \frac{\text{dy}}{\text{dx}}=\frac{(1+\log\text{y})^2}{\log\text{y}} 6 Marks

Ans: We have, \text{y}^\text{x}=\text{e}^{\text{y}-\text{x}},


\Rightarrow\ \log\text{y}^\text{x}=\log^{\text{y}-\text{x}}
\Rightarrow\ \text{x}\log\text{y}=\text{y}-\text{x}\log_\text{e}=(\text{y}-\text{x}) [\because\log_\text{e}=1]
\Rightarrow\ \log\text{y}=\frac{(\text{y}-\text{x})}{\text{x}}\ \ \dots(\text{i})
Now, differentiating w.r.t. x, we get
\frac{\text{d}}{\text{dy}}\log\text{y}\cdot\frac{\text{dy}}{\text{dx}}=\frac{\text{d}}{\text{dx}}\frac{(\text{y}-\text{x})}{\text{x}}
\Rightarrow\ \frac{1}{\text{y}}\cdot\frac{\text{dy}}{\text{dx}}=\frac{\text{x}\cdot\frac{\text{d}}{\text{dx}}(\text{y}-\text{x})-(\text{y}-\text{x})\cdot\frac{\text{d}}{\text{dx}}\text{x}}
{\text{x}^2}
\Rightarrow\ \frac{1}{\text{y}}\frac{\text{dy}}{\text{dx}}=\frac{\text{x}\Big(\frac{\text{dy}}{\text{dx}}-1\Big)-(\text{y}-\text{x})}{\text{x}^2}
\Rightarrow\ \frac{\text{x}^2}{\text{y}}\cdot\frac{\text{dy}}{\text{dx}}=\text{x}\frac{\text{dy}}{\text{dx}}-\text{x}-\text{y}+\text{x}
\Rightarrow\ \frac{\text{dy}}{\text{dx}}\Big(\frac{\text{x}^2}{\text{y}}-\text{x}\Big)=-\text{y}
\therefore\ \frac{\text{dy}}{\text{dx}}=\frac{-\text{y}^2}{\text{x}^2-\text{xy}}=\frac{-\text{y}^2}{\text{x}(\text{x}-\text{y})}
=\frac{\text{y}^2}{\text{x}(\text{y}-\text{x})}\cdot\frac{\text{x}}{\text{x}}=\frac{\text{y}^2}{\text{x}^2}\cdot\frac{1}{\frac{(\text{y}-\text{x})}{\text{x}}}
=\frac{(1+\log\text{y})^2}{\log\text{y}} \Big[\because\log\text{y}=\frac{\text{y}-\text{x}}{\text{x}}\log\text{y}=\frac{\text{y}}{\text{x}}-1\Rightarrow1+\log\text{y}=\frac{\text{y}}
{\text{x}}\Big]
Hence proved.
Q53. Differentiate the following functions from first principles: 6 Marks
\log\cos\text{x}

Ans: Let \text{f(x)} = \log \cos \text{x}


\Rightarrow\ \text{f}(\text{x}+\text{h})=\log\cos(\text{x}+\text{h})
\therefore \frac{\text{d}}{\text{dx}}\{\text{f(x)}\}=\lim\limits_{\text{h}\rightarrow0}\frac{\text{f}(\text{x}+\text{h})=\text{f(x)}}{\text{h}}
=\lim\limits_{\text{h}\rightarrow0}\frac{\log\cos(\text{x}+\text{h})-\log\cos\text{x}}{\text{h}}
=\lim\limits_{\text{h}\rightarrow0}\frac{\log^{\log(\text{x}+\text{h})}_{\cos\text{x}}}{\text{h}}\ \Big[\because\ \log\text{A}-\log\text{B}=\log\Big(\frac{\text{A}}{\text{B}}\Big)\Big]
=\lim\limits_{\text{h}\rightarrow0}\frac{\log\Big[1+\left\{\frac{\cos(\text{z}+\text{h})}{\cos\text{z}}-1\right\}\Big]}{\text{h}}
=\lim\limits_{\text{h}\rightarrow0}\frac{\log\left\{1+\frac{\cos(\text{x}+\text{h})-\cos\text{z}}{\cos\text{z}}\right\}}{\left\{\frac{\cos(\text{x}+\text{h})-\cos\text{x}}
{\cos\text{x}}\right\}}\times\lim\limits_{\text{h}\rightarrow0}\left\{\frac{\cos(\text{x}+\text{h})-\cos\text{x}}{\cos\text{x}}\right\}
=1\times\lim\limits_{\text{h}\rightarrow0}\frac{\cos(\text{x}+\text{h})-\cos\text{x}}{\cos\text{x}\times\text{h}}\ \Big[\because\lim\limits_{\text{h}\rightarrow0}\frac{\log(1+\text{x})}
{\text{x}}=1\Big]
=\lim\limits_{\text{h}\rightarrow0}\frac{-2\sin\Big(\frac{\text{x}+\text{h}+\text{x}}{2}\Big)\sin\Big(\frac{\text{x}+\text{h}+\text{x}}{2}\Big)}{\cos\text{x}\times\text{h}}
=-2\lim\limits_{\text{h}\rightarrow0}\frac{\sin\Big(\frac{2\text{x}+\text{h}}{2}\Big)\times\sin\big(\frac{\text{h}}{2}\big)}{2\cos\text{x}\times\big(\frac{\text{x}}{2}\big)}
=\frac{-2\sin\text{x}}{2\cos\text{x}}\Big[\because \lim\limits\frac{\sin\text{x}}{\text{x}}=1\Big]
=-\tan\text{x}
So,
\frac{\text{d}}{\text{dx}}(\log\cos\text{x})=-\tan\text{x}
Q54. If \text{x}=\text{a}(\cos\text{t}+\log\tan\frac{\text{t}}{2})\ \text{and}\ \text{y}=\text{a}(\sin\text{t}),evaluate \frac{\text{d}^2\text{y}}{\text{dx}^2}\ \text{at}\ \text{t}=\frac{\pi}{3}. 6 Marks

Ans: We have,
\text{x}=\text{a}(\cos\text{t}+\log\tan\frac{\text{t}}{2})\ \text{and}\ \text{y}=\text{a}(\sin\text{t}),
On differentiating with respect to t, we get
\frac{\text{dx}}{\text{dt}}=\frac{\text{d}}{\text{dt}}\Big[\text{a}\Big(\cos\text{t}+\log\tan\frac{\text{t}}{2}\Big)\Big]=\text{a}\Bigg(-\sin\text{t}+\frac{1}{\tan\frac{\text{t}}
{2}}\times\sec^2\frac{\text{t}}{2}\times\frac{1}{2}\Bigg)
=\text{a}\Bigg(-\sin\text{t}+\frac{1}{2\sin\frac{\text{t}}{2}\cos\frac{\text{t}}{2}}\Bigg)=\text{a}\Big(-\sin\text{t}+\frac{1}{\sin\text{t}}\Big)

https://bls.smartstudies.co.in/#/exam/pdf-preview/c59cb220-8e86-4716-9ff7-82aec16b1ade/1 18/158
5/26/24, 6:19 PM Exam Automation
=\text{a}\Big(\frac{-\sin^2\text{t}+1}{\sin\text{t}}\Big)=\text{a}\Big(\frac{\cos^2\text{t}}{\sin\text{t}}\Big)
And
\frac{\text{dy}}{\text{dt}}=\frac{\text{d}}{\text{dt}}(\text{a}\sin\text{t})=\text{a}\cos\text{t}
Now, \Big(\frac{\text{dy}}{\text{dx}}\Big)=\frac{\frac{\text{dy}}{\text{dt}}}{\frac{\text{dX}}{\text{dt}}}=\frac{\text{a}\cos\text{t}}{\text{a}\frac{\cos^2\text{t}}{\sin\text{t}}}=\tan\text{t}
Therefore
\frac{\text{d}^2\text{y}}{\text{dx}^2}=\frac{\text{d}}{\text{dx}}\Big(\frac{\text{dy}}{\text{dx}}\Big)=\frac{\text{d}}{\text{dx}}(\tan(\text{t}))
=\frac{\text{d}}{\text{dt}}(\tan(\text{t}))\times\frac{\text{dt}}{\text{dx}}=\sec^2\text{t}\times\frac{\sin\text{t}}{\text{a}\cos^2\text{t}}
=\Big(\frac{\sin\text{t}}{\text{a}\cos^4\text{t}}\Big)
\Big(\frac{\text{d}^2\text{y}}{\text{dx}^2}\Big)_{\text{t}=\frac{\pi}{3}}=\Bigg(\frac{\sin\Big(\frac{\pi}{3}\Big)}{\text{a}\cos^4\Big(\frac{\pi}{3}\Big)}\Bigg)=\frac{\frac{\sqrt{3}}{2}}
{\text{a}\Big(\frac{1}{16}\Big)}=\frac{8\sqrt{3}}{\text{a}}
Hence, \text{at}\ \text{t}=\frac{\pi}{3},\frac{\text{d}^2\text{y}}{\text{dx}^2}=\frac{8\sqrt{3}}{\text{a}}
Q55. If \text{y}=\text{a}\{\text{x}+\sqrt{\text{x}^2+1}\}^\text{n}+\text{b}\{\text{x}-\sqrt{\text{x}^2+1}\}^{-\text{n},} prove that (\text{x}^2-1)\frac{\text{d}^2\text{y}} 6 Marks
{\text{dx}^2}+\text{x}\frac{\text{dy}}{\text{dx}}-\text{n}^2\text{y}=0.

Ans: We have
\text{y}=\text{a}\{\text{x}+\sqrt{\text{x}^2+1}\}^\text{n}+\text{b}\{\text{x}-\sqrt{\text{x}^2+1}\}^{-\text{n},}
Differentiating y with respect to x, we get
\frac{\text{dy}}{\text{dx}}=\text{an}\{\text{x}+\sqrt{\text{x}^2+1}\}^{\text{n}-1}\Big(1+\frac{1}{2\sqrt{\text{x}^2+1}\times2\text{x}}\Big)-\text{bn}\{\text{x}-\sqrt{\text{x}^2+1}\}^{-
\text{n}-1}\Big(1-\frac{1}{2\sqrt{\text{x}^2+1}\times2\text{x}}\Big)
=\text{an}\{\text{x}+\sqrt{\text{x}^2+1}\}^{\text{n}-1}\Big(1+\frac{\text{x}}{\sqrt{\text{x}^2+1}}\Big)-\text{bn}\{\text{x}-\sqrt{\text{x}^2+1}\}^{-\text{n}-1}\Big(1-\frac{\text{x}}
{\sqrt{\text{x}^2+1}}\Big)
=\text{an}\{\text{x}+\sqrt{\text{x}^2+1}\}^{\text{n}-1}\Big(\frac{\sqrt{\text{x}^2+1}+\text{x}}{\sqrt{\text{x}^2+1}}\Big)-\text{bn}\{\text{x}-
\sqrt{\text{x}^2+1}\}^{\text{n}-1}\Big(\frac{\sqrt{\text{x}^2+1}-\text{x}}{\sqrt{\text{x}^2+1}}\Big)
=\text{an}\{\text{x}+\sqrt{\text{x}^2+1}\}^{\text{n}-1}\Big(\frac{\text{x}+\sqrt{\text{x}^2+1}}{\sqrt{\text{x}^2+1}}\Big)+\text{bn}\{\text{x}-\sqrt{\text{x}^2+1}\}^{-
\text{n}-1}\Big(\frac{\text{x}\sqrt{\text{x}^2+1}}{\sqrt{\text{x}^2+1}}\Big)
=\Big\{\text{a}\{\text{x}+\sqrt{\text{x}^2+1}\}^\text{n}\Big(\frac{\text{n}}{\sqrt{\text{x}^2+1}}\Big)+\text{b}\{\text{x}-\sqrt{\text{x}^2+1}\Big\}^{-\text{n}}\Big(\frac{\text{n}}
{\sqrt{\text{x}^2+1}}\Big)
=\Big(\frac{\text{n}}{\sqrt{\text{x}^2+1}}\Big)\text{y}
\Rightarrow\sqrt{\text{x}^2+1}\frac{\text{dy}}{\text{dx}}=\text{ny}
Squaring both sides, we get
(\text{x}^2+1)\Big(\frac{\text{dy}}{\text{dx}}\Big)^2=\text{n}^2\text{y}^2...(2)
Differentiating (2) with respect to x , we get
(\text{x}^2+1)2\frac{\text{dy}}{\text{dx}}\times\frac{\text{d}^2\text{y}}{\text{dx}^2}+2\text{x}\Big(\frac{\text{dy}}{\text{dx}}\Big)^2=\text{n}^2\Big(2\text{y}\frac{\text{dy}}{\text{dx}}\Big)
\Rightarrow(\text{x}^2+1)\frac{\text{d}^2\text{y}}{\text{dx}^2}+\text{x}\Big(\frac{\text{dy}}{\text{dx}}\Big)=\text{n}^2(\text{y})
\Rightarrow(\text{x}^2+1)\frac{\text{d}^2\text{y}}{\text{dx}^2}+\text{x}\Big(\frac{\text{dy}}{\text{dx}}\Big)=\text{n}^2\text{y}=0
Hence, (\text{x}^2+1)\frac{\text{d}^2\text{y}}{\text{dx}^2}+\text{x}\Big(\frac{\text{dy}}{\text{dx}}\Big)=\text{n}^2\text{y}=0
Q56. If xx + yx = 1, prove that \frac{\text{dy}}{\text{dx}}=-\Big\{\frac{\text{x}^\text{x}(1+\log\text{x})+\text{y}^\text{x}\times\log\text{y}}{\text{x}\times\text{y}^{\text{x}-1}}\Big\} 6 Marks

Ans: Here,
xx + y x = 1
\text{e}^{\log\text{x}^\text{x}}+\text{e}^{\log\text{y}^\text{x}}=1
\text{e}^{\text{x}\log\text{x}}+\text{e}^{\text{x}\log\text{y}}=1
\big[\text{Since},\text{e}^{\log\text{a}}=\text{a}.\log\text{a}^\text{b}=\text{b}\log\text{a}\big]
Differentiating it with respect to x using product rule and chain rule,
\frac{\text{d}}{\text{dx}}\big(\text{e}^{\text{x}\log\text{x}}\big)+\frac{\text{d}}{\text{dx}}\big(\text{e}^{\text{x}\log\text{y}}\big)=\frac{\text{d}}{\text{dx}}(1)
\text{e}^{\text{x}\log\text{x}}\frac{\text{d}}{\text{dx}}(\text{x}\log\text{x})+\text{e}^{\text{x}\log\text{y}}\frac{\text{d}}{\text{dx}}(\text{x}\log\text{y})=0
\text{e}^{\text{x}\log\text{x}}\Big[\text{x}\frac{\text{d}}{\text{dx}}(\log\text{x})+\log\text{x}\frac{\text{d}}{\text{dx}}(\text{x})\Big] \\
+\text{e}^{\log\text{y}^\text{x}}\Big[\text{x}\frac{\text{d}}{\text{dx}}(\log\text{y})+\log\text{y}\frac{\text{d}}{\text{dx}}(\text{x})\Big]=0
\text{x}^\text{x}\Big[\text{x}\Big(\frac{1}{\text{x}}\Big)+\log\text{x}(1)\Big]+\text{y}^\text{x}\Big[\text{x}\Big(\frac{1}{\text{y}}\Big)\frac{\text{dy}}{\text{dx}}+\log\text{y}(1)\Big]=0
\text{x}^\text{x}[1+\log\text{x}]+\text{y}^\text{x}\Big(\frac{\text{x}}{\text{y}}\frac{\text{dy}}{\text{dx}}+\log\text{y}\Big)=0
\text{y}^\text{x}\times\frac{\text{x}}{\text{y}}\frac{\text{dy}}{\text{dx}}=-\big[\text{x}^\text{x}(1+\log\text{x})+\text{y}^\text{x}\log\text{y}\big]
\big(\text{xy}^{\text{x}-1}\big)\frac{\text{dy}}{\text{dx}}=-\big[\text{x}^\text{x}(1+\log\text{x})+\text{y}^\text{x}\log\text{y}\big]
\frac{\text{dy}}{\text{dx}}=-\Big[\frac{\text{x}^\text{x}(1+\log\text{x})+\text{y}^\text{x}\log\text{y}}{\text{xy}^{\text{x}-1}}\Big]
Q57. Differentiate the following functions with respect to x: 6 Marks
(\text{x}\cos\text{x})^\text{x}+(\text{x}\sin\text{x})^\frac{1}{\text{x}}

Ans: Let \text{y}=(\text{x}\cos\text{x})^\text{x}+(\text{x}\sin\text{x})^\frac{1}{\text{x}}


Also, \text{u}=(\text{x}\cos\text{x})^\text{x}\text{ and }\text{v}(\text{x}\sin\text{x})^\frac{1}{\text{x}}
\therefore\ \text{y}=\text{u}+\text{v}
\Rightarrow\frac{\text{dy}}{\text{dx}}=\frac{\text{du}}{\text{dx}}+\frac{\text{dv}}{\text{dx}}\ .....(\text{i})
Now, \text{u}=(\text{x}\cos\text{x})^\text{x}
\Rightarrow\log\text{u}=\log(\text{x}\cos\text{x})^\text{x}
\Rightarrow\log\text{u}=\text{x}\log(\text{x}\cos\text{x})
\Rightarrow\log\text{u}=\text{x}\big[\log\text{x}+\log\cos\text{x}\big]
\Rightarrow\log\text{u}=\text{x}\log\text{x}+\text{x}\log\cos\text{x}
Differentiate both sides with respect to x,
\frac{1}{\text{u}}\frac{\text{du}}{\text{dx}}=\frac{\text{d}}{\text{dx}}(\text{x}\log\text{x})+\frac{\text{d}}{\text{dx}}(\text{x}\log\cos\text{x})
\Rightarrow\frac{\text{du}}{\text{dx}}=\text{u}\Big[\Big\{\log\text{x}\frac{\text{d}}{\text{dx}}(\text{x})+\text{x}\frac{\text{d}}{\text{dx}}(\log\text{x})\Big\} \\ +\Big\
{\log\cos\text{x}\frac{\text{d}}{\text{dx}}(\text{x})+\text{x}\frac{\text{d}}{\text{dx}}(\log\cos\text{x})\Big\}\Big]
\Rightarrow\frac{\text{du}}{\text{dx}}=(\text{x}\cos\text{x})^\text{x}\Big[\Big(\log\text{x}(1)+\text{x}\big(\frac{1}{\text{x}}\big)\Big) \\ +\Big\{\log\cos\text{x}(1)+\text{x}\frac{1}
{\cos\text{x}}\frac{\text{d}}{\text{dx}}(\cos\text{x})\Big\}\Big]
\Rightarrow\frac{\text{du}}{\text{dx}}=(\text{x}\cos\text{x})^\text{x}\Big[(\log\text{x}+1)+\Big\{\log\cos\text{x}\frac{\text{x}}{\cos\text{x}}(-\sin\text{x})\Big\}\Big]
\Rightarrow\frac{\text{du}}{\text{dx}}=(\text{x}\cos\text{x})^\text{x}\big[(1+\log\text{x})+(\log\cos\text{x}-\text{x}\tan\text{x})\big]
\Rightarrow\frac{\text{du}}{\text{dx}}=(\text{x}\cos\text{x})^\text{x}\big[1-\text{x}\tan\text{x}+(\log\text{x}+\log\cos\text{x})\big]
\Rightarrow\frac{\text{du}}{\text{dx}}=(\text{x}\cos\text{x})^\text{x}\big[1-\text{x}\tan\text{x}+\log(\text{x}\cos\text{x})\big]\ .....(\text{ii})
Again, \text{v}=(\text{x}\sin\text{x})^\frac{1}{\text{x}}
\Rightarrow\log\text{v}=\log(\text{x}\sin\text{x})^\frac{1}{\text{x}}
\Rightarrow\log\text{v}=\frac{1}{\text{x}}\log(\text{x}\sin\text{x})
\Rightarrow\log\text{v}=\frac{1}{\text{x}}(\log\text{x}+\log\sin\text{x})
\Rightarrow\log\text{v}=\frac{1}{\text{x}}\log\text{x}+\frac{1}{\text{x}}\log\sin\text{x}
Differentiating both sides with respect to x,
\frac{1}{\text{v}}\frac{\text{dv}}{\text{dx}}=\frac{\text{d}}{\text{dx}}\Big(\frac{1}{\text{x}}\log\text{x}\Big)+\frac{\text{d}}{\text{dx}}\Big[\frac{1}{\text{x}}\log(\sin\text{x})\Big]
\Rightarrow\frac{1}{\text{v}}\frac{\text{dv}}{\text{dx}}=\Big[\log\text{x}\frac{\text{d}}{\text{dx}}\Big(\frac{1}{\text{x}}\Big)+\frac{1}{\text{x}}\frac{\text{d}}{\text{dx}}(\log\text{x})\Big] \\
+\Big[\log(\sin\text{x})\frac{\text{d}}{\text{dx}}\Big(\frac{1}{\text{x}}\Big)+\frac{1}{\text{x}}\frac{\text{d}}{\text{dx}}\big\{\log(\sin\text{x})\big\}\Big]
\Rightarrow\frac{1}{\text{v}}\frac{\text{dv}}{\text{dx}}=\Big[\log\text{x}\Big(-\frac{1}{\text{x}^2}\Big)+\Big(\frac{1}{\text{x}}\Big)\Big(\frac{1}{\text{x}}\Big)\Big] \\
+\Big[\log(\sin\text{x})\Big(-\frac{1}{\text{x}^2}\Big)+\frac{1}{\text{x}}\Big(\frac{1}{\sin\text{x}}\Big)\frac{\text{d}}{\text{dx}}(\sin\text{x})\Big]
\Rightarrow\frac{1}{\text{v}}\frac{\text{dv}}{\text{dx}}=\frac{1}{\text{x}^2}(1-\log\text{x})+\Big[-\frac{\log(\sin\text{x})}{\text{x}^2}+\frac{1}{\text{x}\sin\text{x}}(\cos\text{x})\Big]

https://bls.smartstudies.co.in/#/exam/pdf-preview/c59cb220-8e86-4716-9ff7-82aec16b1ade/1 19/158
5/26/24, 6:19 PM Exam Automation
\Rightarrow\frac{\text{dv}}{\text{dx}}=(\text{x}\sin\text{x})^{\frac{1}{\text{x}}}\Big[\frac{1-\log\text{x}}{\text{x}^2}+\frac{\log(\sin\text{x}+\text{x}\cos\text{x})}{\text{x}^2}\Big]
\Rightarrow\frac{\text{dv}}{\text{dx}}=(\text{x}\sin\text{x})^\frac{1}{\text{x}}\Big[\frac{1-\log\text{x}-\log(\sin\text{x})+\text{x}\cot\text{x}}{\text{x}^2}\Big]
\Rightarrow\frac{\text{dv}}{\text{dx}}=(\text{x}\sin\text{x})^\frac{1}{\text{x}}\Big[\frac{1-\log(\text{x}\sin\text{x})+\text{x}\cot\text{x}}{\text{x}^2}\Big]\ .....(\text{iii})
From (i), (ii) and (iii), we obtain
\frac{\text{dy}}{\text{dx}}=(\text{x}\cos\text{x})^\text{x}\big[1-\text{x}\tan\text{x}+\log(\text{x}\cos\text{x})\big]\\+(\text{x}\sin\text{x})^\frac{1}{\text{x}} \Big[\frac{\text{x}\cot\text{x}+1-
\log(\text{x}\sin\text{x})}{\text{x}^2}\Big]
Q58. Differentiate \sin^{-1}\sqrt{1-\text{x}^2} with respect to \cot^2\Big(\frac{\text{x}}{\sqrt{1-\text{x}^2}}\Big), if 0 < x < 1. 6 Marks

Ans: Let \text{u}=\sin^{-1}\Big(\sqrt{1-\text{x}^2}\Big)


Put \text{x}=\cos\theta\Rightarrow\theta=\cos^{-1}\text{x},\text{ so}
\text{u}=\sin^{-1}(\sin\theta)\ .....(\text{i})
And,
Let \text{v}=\cot^{-1}\Big(\frac{\text{x}}{\sqrt{1-\text{x}^2}}\Big)
=\cot^{-1}\Big(\frac{\cot\theta}{\sqrt{1-\cos^2\theta}}\Big)
=\cot^{-1}\Big(\frac{\cos\theta}{\sin\theta}\Big)
\text{v}=\cot^{-1}(\cot\theta)\ .....(\text{ii})
Here, 0<\text{x}<1
\Rightarrow0<\cos\theta<1
\Rightarrow0<\theta<\frac{\pi}{2}
So, from equation (i),
\text{u}=\theta\Big[\text{Since,}\sin^{-1}(\sin\theta)=\theta,\text{if }\theta\in\Big[-\frac{\pi}{2},\frac{\pi}{2}\Big]\Big]
\text{u}=\cos^{-1}\text{x}
Differentiating it with respect to x,
\frac{\text{du}}{\text{dx}}=\frac{-1}{\sqrt{1-\text{x}^2}}\ .....(\text{iii})
From equation (ii),
\text{v}=\theta\big[\text{Since,}\cot^{-1}(\cot\theta)=\theta,\text{if }\theta\in(0,\pi) \big]
\text{v}=\cos^{-1}\text{x}
Differentiating it with respect to x,
\frac{\text{dv}}{\text{dx}}=\frac{-1}{\sqrt{1-\text{x}^2}}\ .....(\text{iv})
Dividing equation (iii) by (iv),
\frac{\frac{\text{du}}{\text{dx}}}{\frac{\text{dv}}{\text{dx}}}=\frac{-1}{\sqrt{1-\text{x}^2}}\times\frac{\sqrt{1-\text{x}^2}}{-1}
\frac{\text{du}}{\text{dv}}=1
Q59. Differentiate the following functions from first principles: 6 Marks
\text{e}^{\cos\text{x}}

Ans: Let \text{f(x)}=\text{e}^{\cos\text{x}}


\Rightarrow\text{f}(\text{x}+\text{h})=\text{e}^{\cos(\text{x}+\text{h})}
\therefore\frac{\text{d}}{\text{dx}}(\text{f(x)})=\lim\limits_{\text{h}\rightarrow0}\frac{\text{f}(\text{x}+\text{h})-\text{f}(\text{x})}{\text{h}}
=\lim\limits_{\text{h}\rightarrow0}\frac{\text{e}^{\cos(\text{x}+\text{h})}-\text{e}^{\cos\text{x}}}{\text{h}}
=\lim\limits_{\text{h}\rightarrow0}\text{e}^{\cos\text{x}}\Big[\frac{\text{e}^{\cos(\text{x}+\text{h})-\cos\text{x}}-1}{\text{h}}\Big]
=\lim\limits_{\text{h}\rightarrow0}\text{ e}^{\cos\text{x}}\Big[\frac{\text{e}^{\cos(\text{x}+\text{h})-\cos\text{x}}-1}{\cos(\text{x}+\text{h})-
\cos\text{x}}\Big]\times\frac{\cos(\text{x}+\text{h})-\cos\text{x}}{\text{h}}
=\lim\limits_{\text{h}\rightarrow0}\text{ e}^{\cos\text{x}}\times\Big(\frac{\cos(\text{x}+\text{h})-\cos\text{x}}{\text{h}}\Big)
\Big[\text{Since},\lim\limits_{\text{h}\rightarrow0}\frac{\text{e}^\text{x}-1}{\text{x}}=1\Big]
=\lim\limits_{\text{h}\rightarrow0}\ \text{e}^{\cos\text{x}}\times\bigg(\frac{-2\sin\frac{\text{x}+\text{h}+\text{x}}{2}\times\sin\frac{\text{x}+\text{h}-\text{x}}{2}}{\text{h}}\bigg)
\begin{bmatrix} \cos\text{A}-\cos\text{B}=-2\sin\frac{\text{A}+\text{B}}{2}\sin\frac{\text{A}-\text{B}}{2} \end{bmatrix}
=\text{e}^{\cos\text{x}}\lim\limits_{\text{h}\rightarrow0}\frac{-\sin\Big(\frac{2\text{x}+\text{h}}{2}\Big)}{2}\times\frac{\sin\Big(\frac{\text{h}}{2}\Big)}{\frac{\text{h}}{2}}
=\text{e}^{\cos\text{x}}\lim\limits-2\sin\Big(\frac{2\text{x}+\text{h}}{2}\Big)\times\frac{1}{2}
\Big[\text{Since},\lim\limits_{\text{x}\rightarrow0}\frac{\sin\text{x}}{\text{x}}=1\Big]
=\text{e}^{\cos\text{x}}(-\sin\text{x})
=-\sin\text{xe}^{\cos\text{x}}
Hence,
\frac{\text{d}}{\text{dx}}\big(\text{e}^{\cos\text{x}}\big)=-\sin\text{xe}^{\cos\text{x}}
Q60. If xy + yx = (x + y)x+y, find \frac{\text{dy}}{\text{dx}} 6 Marks

Ans: Here,
xy + yx = (x + y)x+y
\text{e}^{\log\text{x}^\text{y}}+\text{e}^{\log\text{y}^\text{x}}=\text{e}^{\log(\text{x}+\text{y})^{(\text{x}+\text{y})}}
\text{e}^{\text{y}\log\text{x}}+\text{e}^{\text{x}\log\text{y}}=\text{e}^{{(\text{x}+\text{y})}\log(\text{x}+\text{y})}
\big[\text{Since},\text{e}^{\log\text{a}}=\text{a},\log\text{a}^\text{b}=\text{b}\log\text{a}\big]
Differentiating it with respect to x using chain rule, product rule,
\Rightarrow\frac{\text{d}}{\text{dx}}\big(\text{e}^{\text{y}\log\text{x}}\big)+\frac{\text{d}}{\text{dx}}\big(\text{e}^{\text{x}\log\text{y}}\big)=\frac{\text{d}}
{\text{dx}}^{(\text{x}+\text{y})\log(\text{x}+\text{y})}
\Rightarrow\text{e}^{\text{y}\log\text{x}}\Big[\text{y}\frac{\text{d}}{\text{dx}}(\log\text{x})+\log\text{x}\frac{\text{dy}}{\text{dx}}\Big] \\
+\text{e}^{\text{x}\log\text{y}}\Big[\text{x}\frac{\text{d}}{\text{dx}}\log\text{y}+\log\text{y}\frac{\text{d}}{\text{dx}}(\text{x})\Big] \\
=\text{e}^{(\text{x}+\text{y})\log(\text{x}+\text{y})}\frac{\text{d}}{\text{dx}}\big[(\text{x}+\text{y})\log(\text{x}+\text{y})\big]
\Rightarrow\text{e}^{\text{y}\log\text{x}}\Big[\text{y}\big(\frac{1}{\text{x}}\big)+\log\text{x}\frac{\text{dy}}{\text{dx}}\Big]+\text{e}^{\log\text{x}}\Big[\frac{\text{x}}
{\text{y}}\frac{\text{dy}}{\text{dx}}+\log\text{y}(1)\Big] \\ =\text{e}^{\log(\text{x}+\text{y})^{(\text{x}+\text{y})}}\Big[(\text{x}+\text{y})\frac{\text{d}}
{\text{dx}}\log(\text{x}+\text{y})+\log(\text{x}+\text{y})\frac{\text{d}}{\text{dx}}(\text{x}+\text{y})\Big]
\Rightarrow\text{x}^\text{y}\Big[\frac{\text{y}}{\text{x}}+\log\text{x}\frac{\text{dy}}{\text{dx}}\Big]+\text{y}^\text{x}\Big[\frac{\text{x}}{\text{y}}\frac{\text{dy}}{\text{dx}}+\log\text{y}\Big]
\\ =(\text{x}+\text{y})^{(\text{x}+\text{y})}\Big[(\text{x}+\text{y})\frac{1}{(\text{x}+\text{y})}\frac{\text{d}}{\text{dx}}(\text{x}+\text{y})+\log(\text{x}+\text{y})\Big(1+\frac{\text{dy}}
{\text{dx}}\Big)\Big]
\Rightarrow\text{x}^\text{y}\times\frac{\text{y}}{\text{x}}+\text{x}^{\text{y}}\log\text{x}\frac{\text{dy}}{\text{dx}}+\text{y}^\text{x}\times\frac{\text{x}}{\text{y}}\frac{\text{dy}}
{\text{dx}}+\text{y}^\text{x}\log\text{y} \\ =(\text{x}+\text{y})^{(\text{x}+\text{y})}\Big[1\times\Big(1+\frac{\text{dy}}{\text{dx}}\Big)+\log(\text{x}+\text{y})\Big(1+\frac{\text{dy}}
{\text{dx}}\Big)\Big]
\Rightarrow\text{x}^{\text{y}-1}\times\text{y}+\text{x}^\text{y}\log\text{x}\frac{\text{dy}}{\text{dx}}+\text{y}^{\text{x}-1}\times\text{x}\frac{\text{dy}}
{\text{dx}}+\text{y}^{\text{x}}\log\text{y} \\ =(\text{x}+\text{y})^{(\text{x}+\text{y})}+(\text{x}+\text{y})^{(\text{x}+\text{y})}\frac{\text{dy}}{\text{dx}} \\ +
(\text{x}+\text{y})^{(\text{x}+\text{y})}\log(\text{x}+\text{y})+(\text{x}+\text{y})^{(\text{x}+\text{y})}\log(\text{x}+\text{y})\frac{\text{dy}}{\text{dx}}
\Rightarrow\frac{\text{dy}}{\text{dx}}\Big[\text{x}^{\text{y}}\log\text{x}+\text{xy}^{\text{x}-1}-(\text{x}+\text{y})^{\text{x}+\text{y}}(1+\log(\text{x}+\text{y}))\Big] \\ =
(\text{x}+\text{y})^{\text{x}+\text{y}}(1+\log(\text{x}+\text{y}))-\text{x}^{\text{y}-1}\times\text{y}-\text{y}^{\text{x}}\log\text{y}
\Rightarrow\frac{\text{dy}}{\text{dx}}\bigg[\frac{(\text{x}+\text{y})^{\text{x}+\text{y}}(1+\log(\text{x}+\text{y}))-\text{x}^{\text{y}-1}\times\text{y}-\text{y}^\text{x}\log\text{y}}
{\text{x}^\text{y}\log\text{x}+\text{xy}^{\text{x}-1}-(\text{x}+\text{y})^{\text{x}+\text{y}}(1+\log(\text{x}+\text{y}))}\bigg]
Q61. If \text{y}=\sin^{-1}\Big(\frac{\text{x}}{1+\text{x}^2}\Big)+\cos^{-1}\Big(\frac{1}{\sqrt{1+\text{x}^2}}\Big), 0<\text{x}<\infty prove that \frac{\text{dy}}{\text{dx}}=\frac{4}{1+\text{x}^2} 6 Marks

Ans: Let \text{y}=\sin^{-1}\Big(\frac{\text{x}}{\sqrt{1+\text{x}^2}}\Big)+\cos^{-1}\Big(\frac{1}{\sqrt{1+\text{x}^2}}\Big)


Put \text{x}=\tan\theta
\therefore\text{y}=\sin^{-1}\Big(\frac{\tan\theta}{\sqrt{1+\tan^2\theta}}\Big)+\cos^{-1}\Big(\frac{1}{\sqrt{1+\tan^2\theta}}\Big)
\Rightarrow \text{y}=\sin^{-1}\bigg(\frac{\frac{\sin\theta}{\cos\theta}}{\sec\theta}\bigg)+\cos^{-1}\Big(\frac{1}{\sec\theta}\Big)

https://bls.smartstudies.co.in/#/exam/pdf-preview/c59cb220-8e86-4716-9ff7-82aec16b1ade/1 20/158
5/26/24, 6:19 PM Exam Automation
\Rightarrow\text{y}=\sin^{-1}\bigg(\frac{\frac{\sin\theta}{\cos\theta}}{\frac{1}{\cos\theta}}\bigg)+\cos^{-1}(\cos\theta)
\Rightarrow\text{y}=\sin^{-1}(\sin\theta)+\cos^{-1}(\cos\theta)\ .....(\text{i})
Here, 0<\text{x}<\infty
\Rightarrow 0<\tan\theta<\infty
\Rightarrow 0 <\theta<\frac{\pi}{2}
So, from equation (i),
\text{y}=\theta+\theta
\begin{bmatrix} \text{Since}, \sin^{-1}(\sin\theta)=\theta,\text{ if }\theta\in \Big[-\frac{\pi}{2},\frac{\pi}{2}\Big] \\ \cos^{-1}(\cos\theta)=\theta,\text{ if }\theta\in[0,\pi] \end{bmatrix}
\Rightarrow\text{y}=2\theta
\Rightarrow\text{y}=2\tan^{-1}\text{x}\ \big[\text{Since, x}=\tan\theta\big]
Differentiate it with respect to x,
\therefore\frac{\text{dy}}{\text{dx}}=\frac{2}{1+\text{x}^2}
Q62. Discuss the continuity and differentiability of, 6 Marks
\text{f(x)}=\begin{cases}(\text{x}-\text{c})\cos\Big(\frac{1}{\text{x}-\text{c}}\Big), & \text{x}\neq 0\\0, & \text{x}= 0\end{cases}

Ans: \text{f(x)}=\begin{cases}(\text{x}-\text{c})\cos\Big(\frac{1}{\text{x}-\text{c}}\Big), & \text{x}\neq 0\\0, & \text{x}= 0\end{cases}


(LHL at x = c) =\lim_\limits{\text{x}\rightarrow\text{c}^{-}}\text{f(x)}
=\lim_\limits{\text{h}\rightarrow0}\text{f}(-\text{h})
=\lim_\limits{\text{h}\rightarrow0}(\text{c}-\text{h}-\text{c})\cos\Big(\frac{1}{\text{c}-\text{h}-\text{c}}\Big)
=\lim_\limits{\text{h}\rightarrow0}-\text{h}\cos\Big(-\frac{1}{\text{h}}\Big)
=\lim_\limits{\text{h}\rightarrow0}-\text{h}\cos\Big(\frac{1}{\text{h}}\Big)
=0
(RHL at x = c) =\lim_\limits{\text{x}\rightarrow\text{c}^{+}}\text{f(x)}
=\lim_\limits{\text{h}\rightarrow0}\text{f}(\text{c}+\text{h})
=\lim_\limits{\text{h}\rightarrow0}(\text{c}+\text{h}-\text{c})\cos\Big(\frac{1}{\text{c}+\text{h}-\text{c}}\Big)
=\lim_\limits{\text{h}\rightarrow0}-\text{h}\cos\Big(\frac{1}{\text{h}}\Big)
=0
f(c) = 0
Since, LHL = f(x) = RHL at x = c
⇒ f(x) is continuous at x = c
(LHL at x = c) =\lim_\limits{\text{h}\rightarrow0}\frac{\text{f}(\text{c}-\text{h})-\text{f(c)}}{-\text{h}}
=\lim_\limits{\text{h}\rightarrow0}\frac{(\text{c}-\text{h}-\text{c})\cos\Big(\frac{1}{\text{c}-\text{h}-\text{c}}\Big)-0}{-\text{h}}
=\lim_\limits{\text{h}\rightarrow0}\cos\Big(-\frac{1}{\text{h}}\Big)
=\lim_\limits{\text{h}\rightarrow0}\cos\Big(\frac{1}{\text{h}}\Big)
(RHL at x = c) =\lim_\limits{\text{h}\rightarrow0}\frac{\text{f}(\text{c}+\text{h})-\text{f(c)}}{\text{h}}
=\lim_\limits{\text{h}\rightarrow0}\frac{(\text{c}+\text{h}-\text{c})\cos\Big(\frac{1}{\text{c}+\text{h}-\text{c}}\Big)-0}{\text{h}}
=\lim_\limits{\text{h}\rightarrow0}\frac{\text{h}\cos\Big(\frac{1}{\text{h}}\Big)}{\text{h}}
=\lim_\limits{\text{h}\rightarrow0}\cos\Big(\frac{1}{\text{h}}\Big)
(LHL at x = c) = (RHL at x = c)
So,
f(x) is differentiable and continuous at x = c.
Q63. If \text{y}=\sin^{-1}\Big(\frac{2\text{x}}{1+\text{x}^2}\Big)+\sec^{-1}\Big(\frac{1+\text{x}^2}{1-\text{x}^2}\Big), 0<\text{x}<1 prove that \frac{\text{dy}}{\text{dx}}=\frac{4}{1+\text{x}^2} 6 Marks

Ans: Let \text{y}=\sin^{-1}\Big(\frac{2\text{x}}{1+\text{x}^2}\Big)+\sec^{-1}\Big(\frac{1+\text{x}^2}{1-\text{x}^2}\Big)


\text{y}=\sin^{-1}\Big(\frac{2\text{x}}{1+\text{x}^2}\Big)+\cos^{-1}\Big(\frac{1-\text{x}^2}{1+\text{x}^2}\Big)
Put \text{x}=\tan\theta
\text{y}=\sin^{-1}\Big(\frac{2\tan\theta}{1+\tan^2\theta}\Big)+\cos^{-1}\Big(\frac{1-\tan^2\theta}{1+\tan^2\theta}\Big)
\text{y}=\sin^{-1}(\sin2\theta)+\cos^{-1}(\cos2\theta)\ .....(1)
Here, 0<\text{x}<1
\Rightarrow 0<\tan\theta<1
\Rightarrow 0<\theta<\frac{\pi}{4}
\Rightarrow0<(2\theta)<\frac{\pi}{2}
So, from equation (i),
\text{y}=2\theta+2\theta
\begin{bmatrix} \text{Since}, \sin^{-1}(\sin\theta)=\theta,\text{ if }\theta\in \Big[-\frac{\pi}{2},\frac{\pi}{2}\Big] \\ \cos^{-1}(\cos\theta)=\theta,\text{ if }\theta\in[0,\pi] \end{bmatrix}
\text{y}=4\tan^{-1}\text{x}\ [\text{Since, x}=\tan\theta]
Diffrentiating it with respect to x,
\frac{\text{dy}}{\text{dx}}=\frac{4}{1+\text{x}^2}
Q64. For what value of k is the following function continuous at x = 2 6 Marks
f(x) = \begin{matrix} 2x + 1 & ; & x< 2 \\ k & ; & x = 2 \\ 3x - 1 & ; & x> 2 \end{matrix}

Ans: For continuity of the function at x = 2


\lim\limits_{h \rightarrow 0} f (2 -h) = f(2) = \lim\limits_{h \rightarrow 0} f(2 + h)
Now, f (2 - h) = 2 (2 - h) + 1 = 5 - 2h
\therefore \lim\limits_{h \rightarrow 0} f(2- h) = 5
Also, f(2 + h) = 3(2 + h) -1 = 5 + 3h
\lim\limits_{h \rightarrow 0} f(2 + h) = 5
So, for continuity f(2) = 5.
\therefore k = 5.
Q65. If \sin(\text{xy})+\frac{\text{y}}{\text{x}}=\text{x}^2-\text{y}^2, find \frac{\text{dy}}{\text{dx}} 6 Marks

Ans: We have, \sin(\text{xy})+\frac{\text{y}}{\text{x}}=\text{x}^2-\text{y}^2


Differentiating with respect to x, we get
\Rightarrow\frac{\text{d}}{\text{dx}}(\sin\text{ xy})+\frac{\text{d}}{\text{dx}}\Big(\frac{\text{y}}{\text{x}}\Big)=\frac{\text{d}}{\text{dx}}(\text{x}^2)-\frac{\text{d}}{\text{dx}}(\text{y}^2)
\Rightarrow \cos(\text{xy})\frac{\text{d}}{\text{dx}}(\text{xy})+\Bigg\{\frac{\text{x}\frac{\text{dy}}{\text{dx}}-\text{y}\frac{\text{d}}{\text{dx}}(\text{x})}
{\text{x}^2}\Bigg\}=2\text{x}-2\text{y}\frac{\text{dy}}{\text{dx}}
\Rightarrow \cos(\text{xy})\Big\{\text{x}\frac{\text{dy}}{\text{dx}}+\text{y}\frac{\text{d}}{\text{dx}}(\text{x})\Big\}+\Bigg\{\frac{\text{x}\frac{\text{dy}}{\text{dx}}-\text{y}(1)}
{\text{x}^2}\Bigg\}=2\text{x}-2\text{y}\frac{\text{dy}}{\text{dx}}
\Rightarrow \cos(\text{xy})\Big\{\text{x}\frac{\text{dy}}{\text{dx}}+\text{y}(1)\Big\}+\frac{1}{\text{x}^2}\Big(\text{x}\frac{\text{dy}}{\text{dx}}-
\text{y}\Big)=2\text{x}-2\text{y}\frac{\text{dy}}{\text{dx}}
\Rightarrow \text{x}\cos(\text{xy})\frac{\text{dy}}{\text{dx}}+\text{y}\cos(\text{xy})+\frac{1}{\text{x}}\frac{\text{dy}}{\text{dx}}-\frac{\text{y}}{\text{x}^2}=2\text{x}-2\text{y}\frac{\text{dy}}
{\text{dx}}
\Rightarrow \frac{\text{dy}}{\text{dx}}\Big\{\text{x}\cos(\text{xy})+\frac{1}{\text{x}}+2\text{y}\Big\}=\frac{\text{y}}{\text{x}^2}-\text{y}\cos(\text{xy})+2\text{x}
\Rightarrow\frac{\text{dy}}{\text{dx}}\Big\{\frac{\text{x}^2\cos(\text{xy})+1+2\text{xy}}{\text{x}}\Big\}=\frac{1}{\text{x}^2}\big(\text{y}-\text{x}^2\text{y}\cos(\text{xy})+2\text{x}^2\big)
\Rightarrow \frac{\text{dy}}{\text{dx}}=\frac{2\text{x}^3+\text{y}-\text{x}^2\text{y}\cos(\text{xy})}{\text{x}\big(\text{x}^2\cos(\text{xy})+1+2\text{xy}\big)}
Q66. An isosceles triangle of vertical angle 2\theta is inscribed in a circle of radius a. Show that the area of the triangle is maximum when \theta=\frac{\pi}{6}. 6 Marks

Ans:

https://bls.smartstudies.co.in/#/exam/pdf-preview/c59cb220-8e86-4716-9ff7-82aec16b1ade/1 21/158
5/26/24, 6:19 PM Exam Automation

\text{CD}=\sqrt{\text{a}^2-\text{x}^2}
Area, \text{A}=\frac{1}{2}\times2\sqrt{\text{a}^2-\text{x}^2}(\text{a}+\text{x})
\text{Z}=\text{A}^2=(\text{a}-\text{x})(\text{a}+\text{x})^3
\frac{\text{dZ}}{\text{dx}}=2(\text{a}+\text{x})^2(\text{a}-2\text{x})
\frac{\text{dZ}}{\text{dx}}= 0 \
\Rightarrow\text{x}= \frac{\text{a}}{2}
\frac{\text{d}^2\text{Z}}{\text{dx}^2}=-12 (\text{a}+\text{x})\text{x}
\Big(\frac{\text{d}^2\text{Z}}{\text{dx}^2}\Big)_{\text{x}=\frac{\text{a}}{2}}=-9\text{a}^2<0
\therefore \text{Z} is maximum when \text{x}=\frac{\text{a}}{2}
i.e., Area is maximum when \text{x}=\frac{\text{a}}{2}
For maximum area
\tan\theta =\frac{\text{CD}}{\text{AD}}=\frac{\sqrt{\text{a}^2-\frac{\text{a}^2}{4}}}{\text{a}+\tfrac{\text{a}}{2{}}}=\frac{1}{\sqrt{3}}
\theta=\frac{\pi}{6}
Q67. If \text{y}=\frac{1}{2}\log\Big(\frac{1-\cos2\text{x}}{1+\cos2\text{x}}\Big), Prvoe that \frac{\text{dy}}{\text{dx}}=2\text{ cosec }2\text{x} 6 Marks

Ans: Given, \text{y}=\frac{1}{2}\log\Big(\frac{1-\cos2\text{x}}{1+\cos2\text{x}}\Big)


\Rightarrow\text{y}=\frac{1}{2}\log\Big(\frac{2\sin^2\text{x}}{2\cos^2\text{x}}\Big)
\begin{bmatrix} \text{Since}, 1-\cos2\text{x}=2\sin^2\text{x}, \\ 1+\cos2\text{x}=2\cos^2\text{x} \end{bmatrix}
\Rightarrow\text{y}=\frac{1}{2}\log\big(\tan^2\text{x}\big)
\Rightarrow\text{y}=\frac{2}{2}\log\tan\text{x}
\big[\text{Since}, \log\text{a}^{\text{b}}=\text{b}\log\text{a}\big]
\Rightarrow\text{y}=\log\tan\text{x}
Differentiate it with respect to x,
\frac{\text{dy}}{\text{dx}}=(\log\tan\text{x})
=\frac{1}{\tan\text{x}}\times\frac{\text{d}}{\text{dx}}(\tan\text{x})
[Using chain rule]
=\frac{\sec^2\text{x}}{\tan\text{x}}
=\frac{1}{\cos^2\text{x}\times\frac{\sin\text{x}}{\cos\text{x}}}
=\frac{1}{\sin\text{x}\cos\text{x}}
=\frac{2}{2\sin\text{x}\cos\text{x}}
=\frac{2}{\sin2\text{x}}\Big[\text{Since},\frac{1}{\sin\text{x}=\text{cosec x}}\Big]
So,
\frac{\text{dy}}{\text{dx}}=2\text{ cosec }2\text{x}
Q68. Find the derivative of the function given by f(x) = (1 + x) (1 + x2) (1 + x4) (1 + x8) and hence find f′(1). 6 Marks

Ans: Given: \text{f(x)}=(1+\text{x})(1+\text{x}^2)(1+\text{x}^4)(1+\text{x}^8)\ \dots\text{(i)}


\Rightarrow\ \log\text{f(x)}=\log\big\{(1+\text{x})(1+\text{x}^2)(1+\text{x}^4)(1+\text{x}^8)\big\}
\Rightarrow\ \log\text{f(x)}=\log(1+\text{x})+\log(1+\text{x}^2)+\log(1+\text{x}^4)+\log(1+\text{x}^8)
\Rightarrow\ \frac{\text{1}}{\text{f(x)}}\frac{\text{d}}{\text{dx}}\text{f(x)}=\frac{\text{1}}{1+\text{x}}\frac{\text{d}}{\text{dx}}(1+\text{x})+\frac{\text{1}}{\text{1+x}^2}\frac{\text{d}}
{\text{dx}}(1+\text{x}^2)+\frac{\text{1}}{\text{1+x}^4}\frac{\text{d}}{\text{dx}}(1+\text{x}^4)+\frac{\text{1}}{\text{1+x}^8}\frac{\text{d}}{\text{dx}}(1+\text{x}^8)
\Rightarrow\ \frac{1}{\text{f(x)}}\text{f}'\text{(x)}=\frac{1}{1+\text{x}}.1+\frac{1}{1+\text{x}^2}.2\text{x}+\frac{1}{1+\text{x}^4}.4\text{x}^3+\frac{1}{1+\text{x}^8}8\text{x}^7
\Rightarrow\ \text{f}'\text{(x)}=\text{f}\text{(x)}\Big[\frac{1}{1+\text{x}}+\frac{2\text{x}}{1+\text{x}^2}+\frac{4\text{x}^3}{1+\text{x}^4}+\frac{8\text{x}^7}{1+\text{x}^8}\Big]
Putting the value of f(x) from eq. (i),
\text{f}'\text{(x)}=(1+\text{x})(1+\text{x}^2)(1+\text{x}^4)(1+\text{x}^8)\Big[\frac{1}{1+\text{x}}+\frac{2\text{x}}{1+\text{x}^2}+\frac{4\text{x}^3}{1+\text{x}^4}+\frac{8\text{x}^7}
{1+\text{x}^8}\Big]
\Rightarrow\ \text{f}'\text{(x)}=(1+1)(1+1^2)(1+1^4)(1+1^8)\Big[\frac{1}{1+1}+\frac{2\times1}{1+1^2}+\frac{4\times1^3}{1+1^4}+\frac{8\times1^7}{1+1^8}\Big]
\Rightarrow\ \text{f}'(1)=(2)(2)(2)(2)\Big[\frac{1}{2}+\frac{2}{2}+\frac{4}{2}+\frac{8}{2}\Big]
\Rightarrow\ \text{f}'(1)=16\Big[\frac{15}{2}\Big]=8\times15=120
Q69. Differentiate the following functions with respect to x: 6 Marks
\tan^{-1}\Big(\frac{2^{\text{x}+1}}{1-4^{\text{x}}}\Big),-\infty<\text{x}<0

Ans: Let \text{y}=\tan^{-1}\Big\{\frac{2^{\text{x}+1}}{1-4^{\text{x}}}\Big\}


Put 2\text{x}=\tan\theta,\text{ so}
\text{y}=\tan^{-1}\Big\{\frac{2^{\text{x}}\times2}{1-(2^\text{x})^2}\Big\}
=\tan^{-1}\Big\{\frac{2\tan\theta}{1-\tan^2\theta}\Big\}
\text{y}=\tan^{-1}\big\{\tan(2\theta)\big\}\ .....(\text{i})
Here, -\infty<\text{x}<0
\Rightarrow 2^{-\infty}<2^{\text{x}}<2^{0}
\Rightarrow 0<2^{\text{x}}<1
\Rightarrow 0< \theta< \frac{\pi}{4}
\Rightarrow 0 < (2\theta) <\frac{\pi}{2}
From equation (i),
\text{y}=2\theta \Big[\text{Since}, \tan^{-1}(\tan\theta)=\theta, \text{ if }\theta\in \Big(-\frac{\pi}{2},\frac{\pi}{2}\Big)\Big]
\text{y}=2\tan^{-1}(2^\text{x})
Differentiate it with respect to x using chain rule,
\frac{\text{dy}}{\text{dx}}=\frac{2}{1+(2^\text{x})^2}\frac{\text{d}}{\text{dx}}(2^\text{x})
=\frac{2\times2^\text{x}\log2}{1+4^{\text{x}}}
\frac{\text{dy}}{\text{dx}}=\frac{2^{\text{x}+1}\log2}{1+4^{\text{x}}}
Q70. Differentiate the following functions with respect to x: 6 Marks
\cos^{-1}\Big(\frac{1-\text{x}^{2\text{n}}}{1+\text{x}^{2\text{n}}}\Big), <\text{x}<\infty

Ans: Let \text{y}=\cos^{-1}\Big(\frac{1-\text{x}^{2\text{n}}}{1+\text{x}^{2\text{n}}}\Big)


Put \text{x}=\tan\theta,\text{ so}
\text{y}=\cos^{-1}\bigg(\frac{1-(\text{x}^{\text{n}})^2}{1+(\text{x}^{\text{n}})^2}\bigg)
=\cos^{-1}\Big(\frac{1-\tan^2\theta}{1+\tan^2\theta}\Big)

https://bls.smartstudies.co.in/#/exam/pdf-preview/c59cb220-8e86-4716-9ff7-82aec16b1ade/1 22/158
5/26/24, 6:19 PM Exam Automation
\text{y}=\cos^{-1}(\cos2\theta)\ .....(\text{i})
Here, 0<\text{x}<\infty
\Rightarrow 0<\text{x}^{\text{n}}<\infty
\Rightarrow 0<\theta<\frac{\pi}{2}
\Rightarrow 0<(2\theta)<\pi
So, from equation (i),
\text{y}=2\theta\big[\text{Since}, \cos^{-1}(\cos\theta)=\theta,\text{ if }\theta\in[0,\pi]\big]
\text{y}=2\tan^{-1}\big(\text{x}^{\text{n}}\big)
Differantiating it with respect to x using chain rule,
\frac{\text{dy}}{\text{dx}}=2\Big(\frac{1}{1+(\text{x}^{\text{n}})^2}\Big)\frac{\text{d}}{\text{dx}}(\text{x}^{\text{n}})
=\frac{2}{1+\text{x}^{2\text{n}}}\times(\text{nx}^{\text{n}-1})
\frac{\text{dy}}{\text{dx}}=\frac{2\text{nx}^{\text{n}-1}}{1+\text{x}^{2\text{n}}}
Q71. Find \frac{\text{dy}}{\text{dx}}, when 6 Marks
\text{x}=\cos^{-1}\frac{1}{\sqrt{1+\text{t}^2}}\text{ and y}=\sin^{-1}\frac{\text{t}}{\sqrt{1+\text{t}^2}},\text{t}\in\text{R}

Ans: We have, \text{x}=\cos^{-1}\Big(\frac{1}{\sqrt{1+\text{t}^{2}}}\Big)


\Rightarrow\frac{\text{dx}}{\text{dt}}=\frac{-1}{\sqrt{1-\Big(\frac{1}{\sqrt{1+\text{t}^{2}}}\Big)^{2}}}\frac{\text{d}}{\text{dt}}\Big(\frac{1}{\sqrt{1+\text{t}}^{2}}\Big)
\Rightarrow\frac{\text{dx}}{\text{dt}}=\frac{-1}{\sqrt{1-\frac{1}{(1+\text{t}^{2})}}}\left\{\frac{-1}{2(1+\text{t}^{2})^\frac{3}{2}}\right\}\frac{\text{d}}{\text{dt}}(1+\text{t}^{2})
\Rightarrow\frac{\text{dx}}{\text{dt}}=\frac{(1+\text{t}^{2})^{\frac{1}{2}}}{\sqrt{1+\text{t}^{2}-1}}\times\frac{1}{2(1+\text{t}^{2})^\frac{3}{2}}(2\text{t})
\Rightarrow\frac{\text{dx}}{\text{dt}}=\frac{\text{t}}{\sqrt{\text{t}^{2}}\times(1+\text{t}^{2})}
\Rightarrow\frac{\text{dx}}{\text{dt}}=\frac{1}{1+\text{t}^{2}}...(\text{i})
Now, \text{y}=\sin^{-1}\Big(\frac{1}{\sqrt{1+\text{t}^{2}}}\Big)
\Rightarrow\frac{\text{dy}}{\text{dt}}=\frac{1}{\sqrt{1-\Big(\frac{1}{\sqrt{1+\text{t}^{2}}}\Big)^{2}}}\frac{\text{d}}{\text{dt}}\Big(\frac{1}{\sqrt{1+\text{t}^{2}}}\Big)
\Rightarrow\frac{\text{dy}}{\text{dt}}=\frac{1}{\sqrt{1-\frac{1}{(1+\text{t}^{2})}}}\left\{\frac{-1}{2(1+\text{t}^{2})\frac{3}{2}}\right\}\frac{\text{d}}{\text{dt}}(1+\text{t}^{2})
\Rightarrow\frac{\text{dy}}{\text{dt}}=\frac{(1+\text{t}^{2})^{\frac{1}{2}}}{\sqrt{1+\text{t}^{2}-1}}\times\frac{-1}{2(1+\text{t}^{2})^{\frac{3}{2}}}(2\text{t})
\Rightarrow\frac{\text{dy}}{\text{dt}}=\frac{-1}{2\sqrt{\text{t}^{2}}\times(1+\text{t}^{2})}(2\text{t})
\Rightarrow\frac{\text{dy}}{\text{dt}}=\frac{-1}{1+\text{t}^{2}}....(\text{ii})
Dividing equation (ii) by (i),
\frac{\frac{\text{dy}}{\text{dt}}}{\frac{\text{dx}}{\text{dt}}}=\frac{1}{(1+\text{t}^{2})}\times\frac{(1+\text{t}^{2})}{-1}
\Rightarrow\frac{\text{dy}}{\text{dx}}=-1
Q72. Differentiate the functions given in Exercise: 6 Marks
(\log\text{x})^\text{x}+\text{x}^{\log\text{x}}

Ans: Let \text{y}=(\log\text{x})^\text{x}+\text{x}^{\log\text{x}}=\text{u}+\text{v }\text{ where u}=(\log\text{x})^\text{x}\text{ and v}=\text{x}^{\log\text{x}}


\therefore\ \frac{\text{dy}}{\text{dx}}=\frac{\text{du}}{\text{dx}}+\frac{\text{dv}}{\text{dx}}\ \dots\text{(i)}
Now \text{u}=(\log\text{x})^\text{x}\ \Rightarrow\ \log\text{u}=\log(\log\text{x})^\text{x}=\text{x}\log(\log\text{x})
\Rightarrow\ \frac{\text{d}}{\text{dx}}\log\text{u}=\frac{\text{d}}{\text{dx}}[\text{x}\log(\log\text{x})]\ \Rightarrow\ \frac{1}{\text{u}}\frac{\text{du}}{\text{dx}}=\text{x}\frac{\text{d}}
{\text{dx}}[\log(\log\text{x})]+\log(\log\text{x})\frac{\text{d}}{\text{dx}}\text{x}
\Rightarrow\ \frac{1}{\text{u}}\frac{\text{du}}{\text{dx}}=\text{x}\frac{1}{\log\text{x}}\frac{\text{d}}{\text{dx}}\log\text{x}+\log(\log\text{x}).1
\Rightarrow\ \frac{1}{\text{u}}\frac{\text{du}}{\text{dx}}=\text{x}\frac{1}{\log\text{x}}\frac{1}{\text{x}}+\log(\log\text{x})\ \Rightarrow\ \frac{\text{du}}{\text{dx}}=\text{u}\Big[\frac{1}
{\log\text{x}}+\log(\log\text{x})\Big]
\Rightarrow\ \frac{\text{du}}{\text{dx}}=(\log\text{x)}^\text{x}\Big[\frac{1}{\log\text{x}}+\log(\log\text{x})\Big]\ \dots\text{(ii)}
Again \text{v}=\text{x}^{\log\text{x}}\ \Rightarrow\ \log\text{v}=\log\text{x}^{\log\text{x}}=\log\text{x}.\log\text{x}=(\log\text{x})^2
\Rightarrow\ \frac{\text{d}}{\text{dx}}\log\text{v}=\frac{\text{d}}{\text{dx}}(\log\text{x})^2\ \Rightarrow\ \frac{1}{\text{v}}\frac{\text{dv}}{\text{dx}}=2\log\text{x}\frac{\text{d}}{\text{dx}}
(\log\text{x})
\Rightarrow\ \frac{1}{\text{v}}\frac{\text{dv}}{\text{dx}}=2\log\text{x}.\frac{1}{\text{x}}\ \Rightarrow\ \frac{\text{dv}}{\text{dx}}=\text{v}\Big(\frac{2}
{\text{x}}\log\text{x}\Big)=\text{x}^{\log\text{x}}\frac{2}{\text{x}}\log\text{x}
\Rightarrow\ \frac{\text{dv}}{\text{dx}}=2\text{x}^{\log\text{x}-1}\log\text{x}\ \dots\text{(iii)}
Putting the values from eq. (ii) and (iii) in eq. (i),
\frac{\text{dy}}{\text{dx}}=(\log\text{x})^\text{x}\Big[\frac{1}{\log\text{x}}+\log(\log\text{x})\Big]+2\text{x}^{\log\text{x}-1}\log\text{x}
\Rightarrow\ \frac{\text{dy}}{\text{dx}}=(\log\text{x})^\text{x}\Big[\frac{1+\log\text{x}\log(\log\text{x})}{\log\text{x}}\Big]+2\text{x}^{\log\text{x}-1}\log\text{x}
\Rightarrow\ \frac{\text{dy}}{\text{dx}}=(\log\text{x})^{\text{x}-1}(1+\log\text{x}\log(\log\text{x}))+2\text{x}^{\log\text{x}-1}\log\text{x}
Q73. Find a point on the curve y = x2 + x, where the tangent is parallel to the chord joining (0, 0) and (1, 2). 6 Marks

Ans: Let,
f(x) = x2 + x
The tangent to the curve is parallel to the chord joining the point (0, 0) and (1, 2).
Assume that the chord joins the points (a, f(a)) and (b, f(b)).
\therefore a = 0, b = 1
The polynomial function is everywhere continuous and differentiable.
So, f(x) = x2 + x is continuous on [0, 1] and differentiable on (0, 1).
Thus, both the conditions of Lagrange's theorem are satisfied.
Concequently, there exists \text{c}\in(0,1) such that \text{f}'(\text{c})=\frac{\text{f}(1)-\text{f}(0)}{1-0}
Now,
f(x) = x2 + x ⇒ f'(x) = 2x + 1, f(1) = 2, f(0) = 0
\therefore\ \text{f}'(\text{x})=\frac{\text{f}(1)-\text{f}(0)}{1-0}
\Rightarrow2\text{x}+1=\frac{2-0}{1-0}
\Rightarrow2\text{x}=1
\Rightarrow\text{x}=\frac{1}{2}
Thus, \text{c}=\frac{1}{2}\in(0,1) such that \text{f}'(\text{c})=\frac{\text{f}(1)-\text{f}(0)}{1-0}.
Clearly,
\text{f}(\text{c})=\Big(\frac{1}{2}\Big)^2+\frac{1}{2}=\frac{3}{4}.
Thus, (c, f(c)), i.e. \Big(\frac{1}{2},\frac{3}{4}\Big), is a point on the given curve where the tangent is parallel to the chord joining the points (4, 0) and (5, 1).
Q74. Is |\sin\text{x}| differentible? What about \cos|\text{x}|? 6 Marks

Ans: Let, d(x) = |sin x|


\sin\text{x}=0, for \text{x}=\text{n}\pi,
|\sin\text{x}|=\begin{cases}-\sin\text{x}\ (2\text{m}-1)\pi<\text{x}<2\text{mx},&\text{where m}\in\text{Z}\\\sin\text{x}\ 2\text{mx}<\text{x}<(2\text{m}+1)\pi,&\text{where
m}\in\text{Z}\\-\sin\text{x}\ (2\text{m}+1)\pi<\text{x}<2(\text{m}+1)\pi,&\text{where m}\in\text{Z}\end{cases}
(\text{LHL at x}=2\text{mx})=\lim_\limits{\text{x}\rightarrow2\text{mx}^{-}}\frac{\text{f(x)}-\text{f}(2\text{mx})}{\text{x}-2\text{mx}}
=\lim_\limits{\text{x}\rightarrow2\text{mx}^{-}}\frac{-\sin(\text{x}-0)}{\text{x}-2\text{mx}}
=\lim_\limits{\text{h}\rightarrow0}\frac{-\sin(2\text{mx}-\text{h})}{2\text{mx}-\text{h}-2\text{mx}}
=\lim_\limits{\text{h}\rightarrow0}\frac{\sin(\text{h})}{\text{h}}=-1
(\text{RHL at x}=2\text{mx})=\lim_\limits{\text{x}\rightarrow2\text{mx}^{+}}\frac{\text{f(x)}-\text{f}(2\text{mx})}{\text{x}-2\text{mx}}
=\lim_\limits{\text{x}\rightarrow2\text{mx}^{+}}\frac{\sin(\text{x)}-0}{\text{x}-2\text{mx}}
=\lim_\limits{\text{h}\rightarrow0}\frac{\sin(2\text{mx}+\text{h})}{2\text{mx}+\text{h}-2\text{mx}}
=\lim_\limits{\text{h}\rightarrow0}\frac{\sin(\text{h})}{\text{h}}=1
Here, \text{LHL}\neq\text{RHL} So, function is not differentiable at \text{x}=2\text{m}\pi, where, \text{m}\in\text{Z}\ \dots(1)

https://bls.smartstudies.co.in/#/exam/pdf-preview/c59cb220-8e86-4716-9ff7-82aec16b1ade/1 23/158
5/26/24, 6:19 PM Exam Automation
[\text{LHL at x}=(2\text{m}+1)\pi]=\lim_\limits{\text{x}\rightarrow(2\text{m}+1)\pi^{-}}\frac{\text{f(x)}-\text{f}[(2\text{m}+1)\pi]}{\text{x}-(2\text{m}+1)\pi}
=\lim_\limits{\text{x}\rightarrow(2\text{m}+1)\pi^{-}}\frac{\sin(\text{x})-0}{\text{x}-(2\text{m}+1)\pi}
=\lim_\limits{\text{x}\rightarrow0}\frac{\sin[(2\text{m}+1)\pi-\text{h}]}{(2\text{m}+1)\pi-\text{h}-(2\text{m}+1)\pi}
=\lim_\limits{\text{x}\rightarrow0}\frac{\sin(\text{h})}{\text{h}}=-1
[\text{RHL at x}=(2\text{m}+1)\pi]=\lim_\limits{\text{x}\rightarrow(2\text{m}+1)\pi^{+}}\frac{\text{f(x)}-\text{f}[(2\text{m}+1)\pi]}{\text{x}-(2\text{m}+1)\pi}
=\lim_\limits{\text{x}\rightarrow(2\text{m}+1)\pi^{+}}\frac{-\sin(\text{x})-0}{\text{x}-(2\text{m}+1)\pi}
=\lim_\limits{\text{x}\rightarrow0}\frac{-\sin[(2\text{m}+1)\pi+\text{h}]}{(2\text{m}+1)\pi+\text{h}-(2\text{m}+1)\pi}
=\lim_\limits{\text{x}\rightarrow0}\frac{\sin(\text{h})}{\text{h}}=1
Here, \text{LHL}\neq\text{RHL}.
So, function is not differentiable at \text{x}=(2\text{m}+1)\pi, where, \text{m}\in\text{Z}\ \dots(2)
From, (1) and (2), we get
\text{f(x)}=|\sin\text{x}| is not differentiable at \text{x}=\text{n}\pi
We know that,
\cos|\text{x}|=\cos\text{x} For all \text{x}\in\text{R}
Also we know that cos x is differentiable at all real points.
Therefore, cos |x| is differentiable everywhere.
Q75. Differentiate the following functions with respect to x: 6 Marks
\sqrt{\frac{1+\sin\text{x}}{1-\sin\text{x}}}

Ans: Let \text{y}=\sqrt{\frac{1+\sin\text{x}}{1-\sin\text{x}}}


Differentiate it with respect to x,
\frac{\text{dy}}{\text{dx}}=\frac{\text{d}}{\text{dx}}\Big(\frac{1+\sin\text{x}}{1-\sin\text{x}}\Big)^{\frac{1}{2}}
=\frac{\text{1}}{\text{2}}\Big(\frac{1+\sin\text{x}}{1-\sin\text{x}}\Big)^{\frac{1}{2}-1}\frac{\text{d}}{\text{dx}}\Big(\frac{1+\sin\text{x}}{1-\sin\text{x}}\Big)
=\frac{\text{1}}{\text{2}}\Big(\frac{1+\sin\text{x}}{1-\sin\text{x}}\Big)^{\frac{1}{2}}\Big[\frac{(1-\sin\text{x})(\cos\text{x})-(1+\sin\text{x})(-\cos\text{x})}{(1-\sin\text{x})^2}\Big]
=\frac{\text{1}}{\text{2}}\frac{(1+\sin\text{x})^\frac{1}{2}}{(1-\sin\text{x})^\frac{1}{2}}\Big[\frac{\cos\text{x}-\cos\text{x}+\cos\text{x}\sin\text{x}+\sin\text{x}\cos\text{x}}{(1-
\sin\text{x})^2}\Big]
=\frac{1}{2}\times\frac{2\cos\text{x}}{\sqrt{1+\sin\text{x}}(1-\sin\text{x})^\frac{3}{2}}
=\frac{\cos\text{x}}{\sqrt{1+\sin\text{x}}(1-\sin\text{x})^\frac{3}{2}}
=\frac{\cos\text{x}}{\sqrt{1+\sin\text{x}}\sqrt{1-\sin\text{x}}(1-\sin\text{x})}
=\frac{\cos\text{x}}{\sqrt{1-\sin^2\text{x}}(1-\sin\text{x})}
=\frac{\cos\text{x}}{\cos\text{x}(1-\sin\text{x})} \big[\text{Using }1-\sin^2\text{x}=\cos^2\text{x}\big]
=\frac{1}{(1-\sin\text{x})}\times\frac{(1+\sin\text{x})}{(1+\sin\text{x})}
Thus, \frac{\text{dy}}{\text{dx}}=\frac{1}{\cos^2\text{x}}+\frac{\sin\text{x}}{\cos^2\text{x}}
\frac{\text{dy}}{\text{dx}}=\sec^2\text{x}+\tan\text{x}\sec\text{x}
\frac{\text{dy}}{\text{dx}}=\sec\text{x}[\tan\text{x}+\sec\text{x}]
Q76. Find \frac{\text{dy}}{\text{dx}} in the following cases: 6 Marks
\sin\text{ xy}+\cos(\text{x}+\text{y})=1

Ans: We have, \sin\text{ xy}+\cos(\text{x}+\text{y})=1


Differentiating with respect to x,
\frac{\text{d}}{\text{dx}}\sin\text{xy}+\frac{\text{d}}{\text{dx}}\cos(\text{x}+\text{y})=\frac{\text{d}}{\text{dx}}(1)
\Rightarrow\cos \text{xy}\frac{\text{d}}{\text{dx}}(\text{xy})-\sin(\text{x}+\text{y})\frac{\text{d}}{\text{dx}}(\text{x}+\text{y})=0
[Using chain rule]
\Rightarrow\cos(\text{xy})\Big[\text{x}\frac{\text{dy}}{\text{dx}}+\text{y}\frac{\text{d}}{\text{dx}}(\text{x})\Big]-\sin(\text{x}+\text{y})\Big[1+\frac{\text{dy}}{\text{dx}}\Big]=0
\Rightarrow\cos(\text{xy})\Big[\text{x}\frac{\text{dy}}{\text{dx}}+\text{y}(1)\Big]-\sin(\text{x}+\text{y})+\sin(\text{x}+\text{y})\frac{\text{dy}}{\text{dx}}=0
\Rightarrow\text{x}\cos(\text{xy})\frac{\text{dy}}{\text{dx}}+\text{y}\cos(\text{xy})-\sin(\text{x}+\text{y})+\sin(\text{x}+\text{y})\frac{\text{dy}}{\text{dx}}=0
\Rightarrow\big[\text{x}\cos(\text{xy})+\sin(\text{x}+\text{y})\big]\frac{\text{dy}}{\text{dx}} \\ =\big[\sin(\text{x}+\text{y})-\text{y}\cos(\text{xy})\big]
\Rightarrow\frac{\text{dy}}{\text{dx}}=\Big[\frac{\sin(\text{x}+\text{y})-\text{y}\cos\text{xy}}{\text{x}\cos\text{xy}+\sin(\text{x}+\text{y})}\Big]
Q77. If \text{x}=\text{a}\sin\text{t}-\text{b}\cos\text{t},\text{y}=\text{a}\cos\text{t}+\text{b}\sin\text{t}, Prove that \frac{\text{d}^2\text{y}}{\text{dx}^2}=-\frac{\text{x}^2+\text{y}^2}{\text{y}^2} 6 Marks

Ans: We have
\text{x}=\text{a}\sin\text{t}-\text{b}\cos\text{t},\text{y}=\text{a}\cos\text{t}+\text{b}\sin\text{t},
On differentiating with respect to t, we get
\frac{\text{dx}}{\text{dt}}=\frac{\text{d}}{\text{dt}}(\text{a}\sin\text{t}-\text{b}\cos\text{t})=\text{a}\cos\text{t}+\text{b}\sin\text{t}
and
\frac{\text{dy}}{\text{dt}}=\frac{\text{d}}{\text{dt}}(\text{a}\cos\text{t}+\text{b}\sin\text{t})=-\text{a}\sin\text{t}+\text{b}\cos\text{t}
\Big(\frac{\text{dy}}{\text{dx}}\Big)=\frac{\frac{\text{dy}}{\text{dt}}}{\frac{\text{dx}}{\text{dt}}}=\frac{-\text{a}\sin\text{t}+\text{b}\cos\text{t}}{\text{a}\cos+\text{b}\sin\text{t}}
Therefore
\frac{\text{d}^2\text{y}}{\text{dx}^2}=\frac{\text{d}}{\text{dx}}\Big(\frac{\text{dy}}{\text{dx}}\Big)=\frac{\text{d}}{\text{dx}}\Big(\frac{-\text{a}\sin\text{t}+\text{b}\cos\text{t}}
{\text{a}\cos\text{t}+\text{b}\sin\text{t}}\Big)
=\frac{\text{d}}{\text{dt}}\Big(\frac{-\text{a}\sin\text{t}+\text{b}\cos\text{t}}{\text{a}\sin\text{t}+\text{b}\sin\text{t}}\Big)\times\frac{\text{dt}}{\text{dx}}
=\frac{(\text{a}\cos\text{t}+\text{b}\sin\text{t})\frac{\text{d}}{\text{dt}}(-\text{a}\sin\text{t}+\text{b}\cos\text{t})-(-\text{a}\sin\text{t}+\text{b}\cos\text{t})\frac{\text{d}}{\text{dt}}
(\text{a}\cos\text{t}+\text{b}\sin\text{t})}{(\text{a}\cos\text{t}+\text{b}\sin\text{t})^2}
=\frac{(\text{a}\cos\text{t}+\text{b}\sin\text{t})(\text{a}\cos\text{t}+\text{b}\sin\text{t})(-\text{a}\sin\text{t}+\text{b}\cos\text{t})-(-\text{a}\sin\text{t}+\text{b}\cos\text{t})}
{(\text{a}\cos\text{t}+\text{b}\sin\text{t})^2}
=\frac{-(\text{a}\cos\text{t}+\text{b}\sin\text{t})^2-(-\text{a}\sin\text{t}+\text{b}\cos\text{t})^2}{(\text{a}\cos\text{t}+\text{b}\sin\text{t})}
=\frac{-(\text{a}\cos\text{t}+\text{b}\sin\text{t})^2-(\text{a}\sin\text{t}+\text{b}\cos\text{t})^2}{(\text{a}\cos\text{t}+\text{b}\sin\text{t})}
=\frac{-\text{y}^2-\text{x}^2}{\text{y}^3}
Q78. Find the values of a and b so that the function \text{f(x)}\begin{cases}\text{x}^2+3\text{x}+\text{a}, & \text{if x}\leq1\\\text{bx}+2, & \text{if x} > 1\end{cases} is differentiable at 6 Marks
each \text{x}\in\text{R}.

Ans: It is given that the function is differentiable at each \text{x}\in\text{R} and every differentiable function is continuous.
Therefore,
Given: \text{f(x)}=\begin{Bmatrix}\text{x}^2+3\text{x}+\text{a}, & \text{if x}\leq1\\\text{bx}+2, & \text{if x} > 1 \end{Bmatrix}
Therefore,
f(x) is continuous at x = 1.
Therefore,
\lim_\limits{\text{x}\rightarrow1^{-}}\text{f(x)}=\lim_\limits{\text{x}\rightarrow1^{+}}\text{f(x)}=\text{f}(1)
Implies that \lim_\limits{\text{x}\rightarrow1}\text{x}^2+3\text{x}+\text{a}=\lim_\limits{\text{x}\rightarrow1}\text{bx}+2=\text{a}+4 [Using def. of f(x)]
Implies that a + 4 = b + 2 = a + 4 ......(1)
Since, f(x) is differentiable at x = 1. Therefore,
(LHL at x = 1) = (RHL at x = 1)
\lim_\limits{\text{x}\rightarrow1^{-}}\frac{\text{f(x)}-\text{f}(1)}{\text{x}-1}=\lim_\limits{\text{x}\rightarrow1^{+}}\frac{\text{f(c)}-\text{f}(1)}{\text{x}-1}
Implies that \lim_\limits{\text{x}\rightarrow1}\frac{\text{x}^2+3\text{x}+\text{a}-\text{a}-4}{\text{x}-1}=\lim_\limits{\text{x}\rightarrow1}\frac{\text{bx}+2-4-\text{a}}{\text{x}-1} [Using def.
of f(x)]
Implies that \lim_\limits{\text{x}\rightarrow1}\frac{(\text{x}+4)(\text{x-1})}{\text{x}-1}=\lim_\limits{\text{x}\rightarrow1}\frac{\text{bx}-2-\text{a}}{\text{x}-1}
Implies that \lim_\limits{\text{x}\rightarrow1}\frac{(\text{x}+4)(\text{x}-1)}{\text{x}-1}=\lim_\limits{\text{x}\rightarrow1}\frac{\text{bx}-\text{b}}{\text{x}-1} [Using (1)]
Implies that \lim_\limits{\text{x}\rightarrow1}\frac{(\text{x}+4)(\text{x}-1)}{\text{x}-1}=\lim_\limits{\text{x}\rightarrow1}\frac{\text{b}(\text{x}-1)}{\text{x}-1}

https://bls.smartstudies.co.in/#/exam/pdf-preview/c59cb220-8e86-4716-9ff7-82aec16b1ade/1 24/158
5/26/24, 6:19 PM Exam Automation
Implies that 5 = b
From (1), we have
a+4=b+2
Implies that a + 4 = 5 + 2
Implies that a = 7 - 4
Implies that a = 3
Hence, a = 3, b = 5.
Q79. If \text{x}=2\cos\theta-\cot2\theta and \text{y}=2\sin\theta-\sin2\theta, prove that \frac{\text{dy}}{\text{dx}}=\tan\big(\frac{3\theta}{2}\big) 6 Marks

Ans: Here, \text{x}=2\cos\theta-\cos2\theta


Diffierentiating it with respect to \theta using chain rule,
\frac{\text{dx}}{\text{d}\theta}=2(-\sin\theta)-(-\sin2\theta)\frac{\text{d}}{\text{d}\theta}(2\theta)
=-2\sin\theta+2\sin2\theta
\frac{\text{dx}}{\text{d}\theta}=2(\sin2\theta-\sin\theta)...(\text{i})
And, \text{y}=2\sin\theta-\sin\theta
Differentiating it with respect to \theta using chain rule,
\frac{\text{dt}}{\text{d}\theta}=2\cos\theta-\cos2\theta\frac{\text{d}}{\text{d}\theta}(2\theta)
=2\cos\theta-\cos2\theta(2)
=2\cos\theta-\cos2\theta(2)
\frac{\text{dy}}{\text{d}\theta}=2(\cos\theta-\cos2\theta)...(\text{ii})
Dividing equation (ii) by equation (i),
\frac{\frac{\text{dy}}{\text{d}\theta}}{\frac{\text{dx}}{\text{d}\theta}}=\frac{2(\cos\theta-\cos2\theta)}{2(\sin2\theta-\sin\theta)}
=\frac{(\cos\theta-\cos2\theta)}{(\sin2\theta-\sin\theta)}
\frac{\text{dy}}{\text{dx}}=\frac{-2\sin\big(\frac{\theta+2\theta}{2}\big)\sin\big(\frac{\theta-2\theta}{2}\big)}{2\cos\big(\frac{2\theta+\theta}{2}\big)\sin\big(2\theta-\theta\big)}
\Big[\sin\text{A}-\sin\text{B}=2\cos\Big(\frac{\text{A+B}}{2}\Big)\sin\Big(\frac{\text{A+B}}{2}\Big)\Big]
\Rightarrow-\frac{\sin\big(\frac{3\theta}{2}\big)\big(\sin\big(\frac{-\theta}{2}\big)\big)}{\cos\big(\frac{3\theta}{2}\big)\sin\big(\frac{\theta}{2}\big)}
\Rightarrow-\frac{\sin\big(\frac{3\theta}{2}\big)\big(-\sin\frac{-\theta}{2}\big)}{\cos\big(\frac{3\theta}{2}\big)\sin\big(\frac{\theta}{2}\big)}
\Rightarrow\frac{\sin\big(\frac{3\theta}{2}\big)}{\cos\big(\frac{3\theta}{2}\big)}
\frac{\text{dy}}{\text{dx}}=\tan\big(\frac{3\theta}{2}\big)
Q80. Differentiate \sin^{-1}\Big(\frac{2\text{x}}{1+\text{x}^2}\Big) with respect to \tan^{-1}\Big(\frac{2\text{x}}{1-\text{x}^2}\Big), if -1 < x < 1. 6 Marks

Ans: Let \text{u}=\sin^{-1}\Big(\frac{2\text{x}}{1+\text{x}^2}\Big)


put \text{x}=\tan\theta\Rightarrow \theta=\tan^{-1}\text{x},\text{so}
\text{u}=\sin^{-1}\Big(\frac{2\tan\theta}{1+\tan^2\theta}\Big)
\text{u}=\sin^{-1}(\sin2\theta)\ .....(\text{i})
Let \text{v}=\tan^{-1}\Big(\frac{2\text{x}}{1-\text{x}^2}\Big)
=\tan^{-1}\Big(\frac{2\tan\theta}{1-\tan^2\theta}\Big)
\text{v}=\tan^{-1}(\tan2\theta)\ .....(\text{ii})
Here, -1<\text{x}<1
\Rightarrow-1<\tan\theta<1
\Rightarrow -\frac{\pi}{4}<\theta<\frac{\pi}{4}
So, from equation (i),
\text{u}=2\theta
\Big[\text{Since,}\sin^{-1}(\sin\theta)=\theta,\text{if }\theta\in\Big(-\frac{\pi}{2},\frac{\pi}{2}\Big)\Big]
\text{u}=2\tan^{-1}\text{x}
Diffrerentiating it with respect to x,
\frac{\text{du}}{\text{dx}}=\frac{2}{(1+\text{x}^2)}\ .....\text{(iii)}
From equation (ii),
\text{v}=2\theta
\Big[\text{Since,}\tan^{-1}(\tan\theta)=\theta,\text{if }\theta\in\Big(-\frac{\pi}{2},\frac{\pi}{2}\Big)\Big]
\text{v}=2\tan^{-1}\text{x}
Differentiating it with respect to x,
\frac{\text{dv}}{\text{dx}}=\frac{2}{1+\text{x}^2}\ .....\text{(iv)}
Dividing equation (iii) by (iv),
\frac{\frac{\text{du}}{\text{dx}}}{\frac{\text{dv}}{\text{dx}}}=\frac{2}{1+\text{x}^2}\times\frac{1+\text{x}^2}{2}
\frac{\text{du}}{\text{dv}}=1
Q81. If \text{x}=\sin^{-1}\Big(\frac{2\text{t}}{1+\text{t}^2}\Big) and \text{y}=\tan^{-1}\Big(\frac{2\text{t}}{1-\text{t}^2}\Big),-1<\text{t}<1, prove that \frac{\text{dy}}{\text{dx}}=1 6 Marks

Ans: We have, \text{x}=\sin^{-1}\Big(\frac{2\text{t}}{1+\text{t}^{2}}\Big)


Put \text{t}=\tan\theta
\Rightarrow-1<\tan\theta<1
\Rightarrow-\frac{\pi}{4}<\theta<\frac{\pi}{4}
\Rightarrow-\frac{\pi}{2}<2\theta<\frac{\pi}{2}
\therefore\text{x}=\sin^{-1}\Big(\frac{2\tan\theta}{1+\tan^{2}\theta}\Big)
\Rightarrow\text{x}=\sin^{-1}(\sin2\theta)
\Rightarrow\text{x}=2\theta
\Big[\therefore-\frac{\pi}{2}<2\theta<\frac{\pi}{2}\Big]
\Rightarrow\text{x}=2(\tan^{-1}\text{t})
\big[\therefore \text{t}=\sin\theta\big]
\Rightarrow\frac{\text{dx}}{\text{dt}}=\frac{2}{1+\text{t}^{2}}\ .....(\text{i})
Now, \text{y}=\tan^{-1}\Big(\frac{2\text{t}}{1-\text{t}^{2}}\Big)
Put \text{t}=\tan\theta
\Rightarrow\text{y}=\tan^{-1}\Big(\frac{2\text{t}}{1-\text{t}^{2}}\Big)
\Rightarrow\text{y}=\tan^{-1}\Big(\frac{2\tan\theta}{1-\tan\theta}\Big)
\Rightarrow\text{y}=\tan^{-1}(\tan2\theta)
\Rightarrow\text{y}=2\theta
\Big[\therefore-\frac{\pi}{2}<2\theta<\frac{\pi}{2}\Big]
\Rightarrow\text{y}= 2 \tan^{-1}\text{t}
\big[\therefore\text{t}=\tan\theta\big]
\Rightarrow\frac{\text{dy}}{\text{dt}}=\frac{2}{1+\text{t}^{2}}\ .....(\text{ii})
Dividing equation (ii) by (i),
\frac{\frac{\text{dy}}{\text{dt}}}{\frac{\text{dx}}{\text{dt}}}=\frac{2}{1+\text{t}^{2}}\times\frac{1+\text{t}^{2}}{2}
\Rightarrow\frac{\text{dy}}{\text{dx}}=1
Q82. If \text{x}=\cos\text{t}+\log\tan\frac{\text{t}}{2},\text{y}=\sin\text{t}, Then find the value of \frac{\text{d}^2\text{y}}{\text{dt}^2}\ \text{and}\ \frac{\text{d}^2\text{y}}{\text{dx}^2}\ 6 Marks
\text{at}\ \text{t}=\frac{\pi}{4}.

Ans: \text{x}=\cos\text{t}+\log\tan\frac{\text{t}}{2},\text{y}=\sin\text{t}
On differentiating with respect to t, we get

https://bls.smartstudies.co.in/#/exam/pdf-preview/c59cb220-8e86-4716-9ff7-82aec16b1ade/1 25/158
5/26/24, 6:19 PM Exam Automation
\frac{\text{dx}}{\text{dt}}=\frac{\text{d}}{\text{dt}}\Big(\cos\text{t}+\log\tan\frac{\text{t}}{2}\Big)=-\sin\text{t}+\frac{1}{\tan\frac{\text{t}}{2}}\times\sec^2\frac{\text{t}}{2}\times\frac{1}{2}
=-\sin\text{t}+\frac{1}{2\sin\frac{\text{t}}{2}\cos\frac{\text{t}}{2}}=-\sin\text{t}+\frac{1}{\sin\text{t}}
=\frac{-\sin^2\text{t}+1}{\sin\text{t}}=\frac{-\sin^2\text{t}+1}{\sin\text{t}}
=\frac{\cos^2\text{t}}{\sin\text{t}}
and
\frac{\text{dy}}{\text{dt}}=\frac{\text{d}}{\text{dt}}(\sin\text{t})=\cos\text{t}
\frac{\text{d}^2\text{y}}{\text{dt}^2}=\frac{\text{d}}{\text{dt}}\Big(\frac{\text{dy}}{\text{dt}}\Big)=\frac{\text{d}}{\text{dt}}(\cos\text{t})=-\sin\text{t}
\Big(\frac{\text{d}^2\text{y}}{\text{dt}^2}\Big)_{(\text{t}=\frac{\pi}{4})}=-\sin\Big(\frac{\pi}{4}\Big)=-\frac{1}{\sqrt{2}}...(1)
Also, \Big(\frac{\text{dy}}{\text{dx}}\Big)=\frac{\frac{\text{dy}}{\text{dt}}}{\frac{\text{dx}}{\text{dt}}}=\frac{\cos\text{t}}{\frac{\cos^2\text{t}}{\sin\text{t}}}=\frac{\sin\text{t}}
{\cos\text{t}}=\tan\text{t}
Now, \frac{\text{d}^2\text{y}}{\text{dx}^2}=\frac{\text{d}}{\text{dx}}\Big(\frac{\text{dy}}{\text{dx}}\Big)=\frac{\text{d}}{\text{dx}}(\tan\text{x})
\frac{\text{d}}{\text{dt}}(\tan\text{t})\times\frac{\text{dt}}{\text{dx}}=\sec^2\text{t}\times\frac{\sin\text{t}}{\cos^2\text{t}}
=\frac{\sin\text{t}}{\cos^4\text{t}}
\Big(\frac{\text{d}^2\text{y}}{\text{dx}^2}\Big)_{\text{t}=\frac{\pi}{4}}=\frac{\sin\Big(\frac{\pi}{4}\Big)}{\cos^4\Big(\frac{\pi}{4}\Big)}=2\sqrt{2}...(2)
Hence, at \text{t}=\frac{\pi}{4},\frac{\text{d}^2\text{y}}{\text{dt}^2}=-\frac{1}{\sqrt{2}}\ \text{and}\ \frac{\text{d}^2\text{y}}{\text{dx}^2}=2\sqrt{2}
Q83. Show that f(x) = |x − 2| is continuous but not differentiable at x = 2. 6 Marks

Ans: Given: \text{f}(\text{x})=|\text{x}-3|=\begin{vmatrix}\text{x}-3,&\text{x}\geq3 \\-\text{x}+3, & \text{x}<3 \end{vmatrix}


Continuity at X = 2: We have,
(LHL at x = 3)
=\lim_\limits{\text{x} \rightarrow 2}\text{f}(\text{x})
=\lim_\limits{\text{x} \rightarrow 0}\text{f}(3-\text{h})
=\lim_\limits{\text{x} \rightarrow 0}0(-3+\text{h})+3
=0.
(RHL at x = 3)
=\lim_\limits{\text{x} \rightarrow 2^{-}}\text{f}(\text{x})
=\lim_\limits{\text{x} \rightarrow 0}\text{f}(3+\text{h})
=\lim\limits_{\text{x} \rightarrow 0}3+\text{h}-3
=0.
and f(2) = 0
Thus, \lim_\limits{\text{x} \rightarrow 2^{-}}\text{f}(\text{x})=\text{f}(2).
Hence,
f(x) is continuous at x = 3
We have,
(LHL at x = 3)
=\lim_\limits{\text{x} \rightarrow 2^{-}}-\frac{\text{f}(\text{x})-\text{f}(2)}{\text{x}-3}
=\lim_\limits{\text{x} \rightarrow 2}\frac{(-\text{x}+3)-0}{\text{x}-3}
=\lim_\limits{\text{x} \rightarrow 2}\text{f}(\text{x}-3)\text{x}-2
\lim_\limits{\text{x} \rightarrow 2}(-1)
=-1
(RHL at x = 3)
=\lim_\limits{\text{x} \rightarrow 2^{-}}\frac{\text{f}(\text{x})-\text{f}(2)}{\text{x}-3}
=\lim_\limits{\text{x} \rightarrow 2}\frac{(\text{x}-3)-0}{\text{x}-3}
=\lim_\limits{\text{x}\rightarrow2}1=1
Thus, =\lim_\limits{\text{x}\rightarrow2^{-}}\text{f}(\text{x})\neq\lim_\limits{\text{x}\rightarrow2^{-}}\text{f}(\text{x}).
Hence, f(x) is not differentiable at x = 3.
Q84. Find \frac{\text{dy}}{\text{dx}} 6 Marks
\text{y}=\frac{(\text{x}^2-1)^3(2\text{x}-1)}{\sqrt{(\text{x}-3)(4\text{x}-1)}}

Ans: We have, \text{y}=\frac{(\text{x}^2-1)^3(2\text{x}-1)}{\sqrt{(\text{x}-3)(4\text{x}-1)}}.....(\text{i})


\text{y}=\frac{(\text{x}^2-1)^3(2\text{x}-1)}{(\text{x}-3)^{\frac{1}{2}}(4\text{x}-1)^{\frac{1}{2}}}
Taking log on both sides,
\log\text{y}=\log\Bigg[\frac{(\text{x}^2-1)^3(2\text{x}-1)}{(\text{x}-3)^{\frac{1}{2}}(4\text{x}-1)^{\frac{1}{2}}}\Bigg]
\Rightarrow\log\text{y}=\log(\text{x}^2-1)^3+\log(2\text{x}-1)-\log(\text{x}-3)^{\frac{1}{2}}-\log(4\text{x}-1)^{\frac{1}{2}}
\Rightarrow\log\text{y}=\log(\text{x}^2-1)^3+\log(2\text{x}-1)-\frac{1}{2}\log(\text{x}-3)-\frac{1}{2}\log(4\text{x}-1)
Differentiating with respect to x using chain rule,
\frac{1}{\text{y}}\frac{\text{dy}}{\text{dx}}=3\frac{\text{d}}{\text{dx}}\Big\{\log(\text{x}^2-1)\Big\}+\frac{\text{d}}{\text{dx}}\Big\{\log(2\text{x}-1)\Big\}\\-\frac{1}{2}\frac{\text{d}}
{\text{dx}}\Big\{\log(\text{x}-3)\Big\}-\frac{1}{2}\Big\{\log(4\text{x}-1)\Big\}
\Rightarrow\frac{1}{\text{y}}\frac{\text{dy}}{\text{dx}}=3\Big(\frac{1}{\text{x}^2-1}\Big)\frac{\text{d}}{\text{dx}}(\text{x}^2-1)+\frac{1}{(2\text{x}-1)}\frac{\text{d}}{\text{dx}}(2\text{x}-1)\\-
\frac{1}{2}\Big(\frac{1}{\text{x}-3}\Big)\frac{\text{d}}{\text{dx}}(\text{x}-3)-\frac{1}{2}\frac{1}{(4\text{x}-1)}\frac{\text{d}}{\text{dx}}(4\text{x}-1)
\Rightarrow\frac{1}{\text{y}}\frac{\text{dy}}{\text{dx}}=3\Big(\frac{1}{\text{x}^2-1}\Big)(2\text{x})+\frac{1}{2\text{x}-1}(2)-\frac{1}{2}\Big(\frac{1}{\text{x}-3}\Big)(1)-\frac{1}{2}\Big(\frac{1}
{4\text{x}-1}\Big)(4)
\Rightarrow\frac{1}{\text{y}}\frac{\text{dy}}{\text{dx}}=\Big[\frac{6\text{x}}{\text{x}^2-1}+\frac{2}{2\text{x}-1}-\frac{1}{2(\text{x}-3)}-\frac{2}{4\text{x}-1}\Big]
\Rightarrow\frac{\text{dy}}{\text{dx}}=\text{y}\Big[\frac{6\text{x}}{\text{x}^2-1}+\frac{2}{2\text{x}-1}-\frac{1}{2(\text{x}-3)}-\frac{2}{4\text{x}-1}\Big]
\Rightarrow\frac{(\text{x}^2-1)^3(2\text{x}-1)}{\sqrt{(\text{x}-3)(4\text{x}-1)}}\Big[\frac{6\text{x}}{\text{x}^2-1}+\frac{2}{2\text{x}-1}-\frac{1}{2(\text{x}-3)}-\frac{2}{4\text{x}-1}\Big]
[Using equation (i)]
Q85. Finde the value of a and b, if the function f(x) defined by \text{f(x)}\begin{cases}\text{x}^2+3\text{x}+\text{a}, &\text{x}\leq1\\\text{bx}+2, & \text{x}>1\end{cases}is differentiable at x = 1. 6 Marks

Ans: Given that f(x) is differentiable at x = 1, Therefore, f(x) is countinuous at x = 1.


\lim\limits_{\text{x}\rightarrow1^{-}}\text{f(x)}=\lim\limits_{\text{x}\rightarrow1^{+}}\text{f(x)}=\text{f(1)}
\Rightarrow\lim\limits_{\text{x}\rightarrow1}(\text{x}^2+3\text{x}+\text{a})=\lim\limits_{\text{x}\rightarrow1}(\text{bx}+2)=1+3+\text{a}
\Rightarrow1+3+\text{a}=\text{b}+2
\Rightarrow\text{a}-\text{b}+2=0\dots(1)
Again, f(x) is differentiable at x = 1. So,
(LHL at x = 1) = (RHL at x = 1)
\Rightarrow\lim\limits_{\text{x}\rightarrow1^{-}}\frac{\text{f(z)}-\text{f}(1)}{\text{z}-1}=\lim\limits_{\text{x}\rightarrow1^{+}}\frac{\text{f(z)}-\text{f}(1)}{\text{z}-1}
\Rightarrow\lim\limits_{\text{x}\rightarrow1}\frac{(\text{x}^2+3\text{x}+\text{a})-(4+\text{a})}{\text{x}-1}=\lim\limits_{\text{x}\rightarrow0}\frac{(\text{bx}+2)-(4+\text{a})}{\text{x}-1}
\Rightarrow\lim\limits_{\text{x}\rightarrow1}\frac{\text{x}^2+3\text{x}-4}{\text{x}-1}=\lim\limits_{\text{x}\rightarrow1}\frac{(\text{bx}-2-\text{a})}{\text{x}-1}
\Rightarrow\lim\limits_{\text{x}\rightarrow1}(\text{x}-4)=\lim\limits_{\text{x}\rightarrow1}\frac{\text{b}(\text{x}-1)}{\text{x}-1}
\Rightarrow5=\text{b}
Hence, a = 3 and b = 5.
Q86. Differentiate \tan^{-1}\Big(\frac{1+\text{ax}}{1-\text{ax}}\Big) with respect to \sqrt{1+\text{a}^2\text{x}^2} 6 Marks

Ans: Let, \text{u}=\tan^{-1}\Big(\frac{1+\text{ax}}{1-\text{ax}}\Big)


Put \text{ax}= \tan\theta
\Rightarrow\text{u}=\tan^{-1}\Big(\frac{1+\tan\theta}{1-\tan\theta}\Big)
\Rightarrow\text{u}=\tan^{-1}\bigg(\frac{\tan\frac{\pi}{4}+\tan\theta}{1-\tan\frac{\pi}{4}\tan\theta}\bigg)
\Rightarrow\text{u}=\tan^{-1}\Big[\tan\Big(\frac{\pi}{4}+\theta\Big)\Big]

https://bls.smartstudies.co.in/#/exam/pdf-preview/c59cb220-8e86-4716-9ff7-82aec16b1ade/1 26/158
5/26/24, 6:19 PM Exam Automation
\Rightarrow\text{u}=\frac{\pi}{4}+\theta
\Rightarrow\text{u}=\frac{\pi}{4}+\tan^{-1}(\text{ax}) [\text{since,}\tan\theta=\text{ax}]
Differentiating it with respect to x,
\frac{\text{du}}{\text{dx}}=0+\frac{1}{1+(\text{ax}^2)}\frac{\text{d}}{\text{dx}}(\text{ax})
\Rightarrow\frac{\text{du}}{\text{dx}}=\frac{\text{a}}{1+\text{a}^2\text{x}^2}\ .....(\text{i})
Now,
Let, \text{v}=\sqrt{1+\text{a}^2\text{x}^2}
Differentiating it with respect to x,
\frac{\text{dv}}{\text{dx}}=\frac{1}{2\sqrt{1+\text{a}^2\text{x}^2}}\frac{\text{d}}{\text{dx}}(1+\text{a}^2\text{x}^2)
\Rightarrow\frac{\text{dv}}{\text{dx}}=\frac{1}{2\sqrt{1+\text{a}^2\text{x}^2}}(2\text{a}^2\text{x})
\Rightarrow\frac{\text{dv}}{\text{dx}}=\frac{\text{a}^2\text{x}}{\sqrt{1+\text{a}^2\text{x}^2}}\ .....(\text{ii})
Dividing equation (i) by (ii),
\frac{\frac{\text{du}}{\text{dx}}}{\frac{\text{dv}}{\text{dx}}}=\frac{\text{a}}{1+\text{a}^2\text{x}^2}\times\frac{\sqrt{1+\text{a}^2\text{x}^2}}{\text{a}^2\text{x}}
\frac{\text{du}}{\text{dv}}=\frac{1}{\text{ax}\sqrt{1+\text{a}^2\text{x}^2}}
Q87. Show that the function \text{f(x)}\begin{cases}\text{x}^\text{m}\sin(\frac{1}{\text{x}}), &\text{x}\neq0 \\0 ,& \text{x}=0\end{cases} 6 Marks
Continuous but not diffierentiable at x = 0, if 0 < m < 1

Ans: \text{LHL}=\lim_\limits{\text{x}\rightarrow0^{-}}\text{f(x)}
=\lim_\limits{\text{h}\rightarrow0}\text{f}(0-\text{h})
=\lim_\limits{\text{h}\rightarrow0}(-\text{h})^\text{m}\sin\Big(-\frac{1}{\text{h}}\Big)
=\lim_\limits{\text{h}\rightarrow0}(-\text{h})^\text{m}\sin\Big(\frac{1}{\text{h}}\Big)
=0\times\text{k}\ [\text{Where}-1\leq\text{k}\leq1]
=0
\text{RHL }=\lim_\limits{\text{x}\rightarrow0^{+}}\text{f(x)}
=\lim_\limits{\text{h}\rightarrow0^+}\text{f}(0+\text{h})
=\lim_\limits{\text{h}\rightarrow0}(+\text{h})^\text{m}\sin\Big(\frac{1}{0+\text{h}}\Big)
=\lim_\limits{\text{h}\rightarrow0^-}(-\text{h})^\text{m}\sin\Big(\frac{1}{\text{h}}\Big)
=0\times\text{k'}\ [\text{When}-1\leq\text{k}'\leq1]
=0
LHL = f(0) = RHL
\therefore f(x) is continuous at x = 0
For differentiable at x = 0
(LHL at x = 0) =\lim_\limits{\text{x}\rightarrow0^{-}}\frac{\text{f(x)}-\text{f}(0)}{\text{x}-0}
=\lim_\limits{\text{h}\rightarrow0}\frac{(0-\text{h})-\text{f}(0)}{(0-\text{h})-0}
=\lim_\limits{\text{h}\rightarrow0}\frac{(-\text{h})^\text{m}\sin\Big(-\frac{1}{\text{h}}\Big)}{-\text{h}}
=\lim_\limits{\text{h}\rightarrow0}(-\text{h})^\text{m-1}\sin\Big(-\frac{1}{\text{h}}\Big)
= Not definded [Since 0 < m < 1]
(RHL at x = 0) =\lim_\limits{\text{x}\rightarrow0^{+}}\frac{\text{f(x)}-\text{f}(0)}{\text{x}-0}
=\lim_\limits{\text{h}\rightarrow0}\frac{\text{f}(0+\text{h})-\text{f}(0)}{(0+\text{h})-0}
=\lim_\limits{\text{h}\rightarrow0}\frac{\text{h}^\text{m}\sin\Big(\frac{1}{\text{h}}\Big)}{\text{h}}
=\lim_\limits{\text{h}\rightarrow0}(\text{h})^\text{m-1}\sin\Big(\frac{1}{\text{h}}\Big)
= Not defined [as 0, m < 1]
\therefore (LHL at x = 0) and (RHL at x = 0) are not defined, so f(x) is continuous but not differentiable at x = 0, when 0 < m < 1.
Q88. If \text{y}=\text{x}^\text{n}\{\text{a}\cos(\log\text{x})+\text{b}\sin(\log\text{x})\}, prove that \text{x}^2\frac{\text{d}^2\text{y}}{\text{dx}^2}+(1-2\text{n})\frac{\text{dy}}{\text{dx}}+ 6 Marks
(1+\text{n}^2)\text{y}=0.

Ans: We have,
\text{y}=\text{x}^\text{n}\{\text{a}\cos(\log\text{x})+\text{b}\sin(\log\text{x})\},...(1)
Differentiating y with respect to x, we get
\frac{\text{dy}}{\text{dx}}=\text{nx}^{\text{n}-1}\{\text{a}\cos(\log\text{x})+\text{b}\sin(\log\text{x})\}+\text{x}^\text{n}\\\Big\{-\text{a}\sin(\log\text{x})\times\frac{1}
{\text{x}}+\text{b}\cos(\log\text{x})\times\frac{1}{\text{x}}\Big\}
=\frac{\text{n}}{\text{x}}\text{x}^\text{n}\{\text{a}\cos(\log\text{x})+\text{b}\sin(\log\text {x})\}+\text{x}^{\text{n}-1}\{-\text{a}\sin(\log\text{x})+\text{b}\cos(\log\text{x})\}
\Rightarrow\frac{\text{n}}{\text{x}}\text{y}+\text{x}^{\text{n}-1}\{-\text{a}\sin(\log\text{x})+\text{b}\cos(\log\text{x})\}\ [\text{from}(1)]
\Rightarrow\text{x}^{\text{n}-1}\{-\text{a}\sin(\log\text{x})+\text{b}\cos(\log\text{x})\}=\frac{\text{dy}}{\text{dx}}-\frac{\text{n}}{\text{x}}\text{y}...(2)
Differentiating \frac{\text{dy}}{\text{dx}} with respect to x, we get
\frac{\text{d}^2\text{y}}{\text{dx}^2}=\frac{\text{n}}{\text{x}}\frac{\text{dy}}{\text{dx}}=\frac{\text{ny}}{\text{x}^2}+(\text{n}-1)\text{x}^{\text{n}-2}\{-
\text{a}\sin(\log\text{x})+\text{b}\cos(\log\text{x})\}\\+\text{x}^{\text{n}-1}\Big\{-\text{a}\cos(\log\text{x})\times\frac{1}{\text{x}}-\text{b}\sin(\log\text{x})\times\frac{1}{\text{x}}\Big\}
=\frac{\text{n}}{\text{x}}\frac{\text{dx}}{\text{dy}}-\frac{\text{ny}}{\text{x}^2}+(\text{n}-1)\frac{\text{x}^{\text{n}-1}}{\text{x}}\{-\text{a}\sin(\log\text{x})+\text{b}\cos(\log\text{x})\}\\-
\frac{\text{x}^\text{n}}{\text{x}^2}\{\text{a}\cos(\log\text{x})+\text{b}\sin(\log\text{x})\}
\frac{\text{n}}{\text{x}}\frac{\text{dy}}{\text{dx}}-\frac{\text{ny}}{\text{x}^2}+\Big(\frac{\text{n}-1}{\text{x}}\Big)\Big(\frac{\text{dy}}{\text{dx}}-\frac{\text{n}}{\text{x}}\text{y}\Big)-
\frac{\text{y}}{\text{x}^2}
=\frac{\text{dy}}{\text{dx}}\Big(\frac{\text{n}+\text{n}-1}{\text{x}}\Big)-\frac{(\text{n}+\text{n}^2+\text{n}+1)\text{y}}{\text{x}^2}
=\Big(\frac{2\text{n}-1}{\text{x}}\Big)\frac{\text{dy}}{\text{dx}}=\frac{(\text{n}^2+1)\text{y}}{\text{x}^2}
\Rightarrow\text{x}^2\frac{\text{d}^2\text{y}}{\text{dx}^2}-\text{x}(2\text{n}-1)\frac{\text{dy}}{\text{dx}}+(\text{n}^2+1)\text{y}=0
Hence, \text{x}^2\frac{\text{d}^2\text{y}}{\text{dx}^2}+(1-2\text{n})\text{x}\frac{\text{dy}}{\text{dx}}+(1+\text{n}^2)\text{y}=0
Q89. Differentiate \cos^{-1}(4\text{x}^3-3\text{x}) with respect to \tan^{-1}\Big(\frac{\sqrt{1-\text{x}^2}}{\text{x}}\Big), if \frac{1}{2}<\text{x}<1 6 Marks

Ans: Let, \text{u}=\cos^{-1}(4\text{x}^3-3\text{x})


Put, \text{x}=\cos\theta
\Rightarrow\theta=\cos^{-1}\text{x}
Now, \text{u}=\cos^{-1}(4\cos^3\theta-3\cos\theta)
\Rightarrow \text{u}=\cos^{}-1(\cos3\theta)\ .....(\text{i})
Let, \text{v}=\tan^{-1}\Big(\frac{\sqrt{1-\text{x}^2}}{\text{x}}\Big)
\Rightarrow\text{v}=\tan^{-1}\bigg(\frac{\sqrt{-1\cos^2\theta}}{\cos\theta}\bigg)
\Rightarrow\text{v}=\tan^{-1}\Big(\frac{\sin\theta}{\cos\theta}\Big)
\Rightarrow\text{v}=\tan^{-1}(\tan\theta)\ .....(\text{ii})
Here,
\frac{1}{2}<\text{x}<1
\Rightarrow\frac{1}{2}<\cos<1
\Rightarrow0<\theta<\frac{\pi}{3}
So, from equation (i),
\text{u}=3\theta\big[\text{Since,} \cos^{-1}(\cos\theta)=\theta,\text{if }\theta\in[0,\pi]\big]
\Rightarrow\text{u}=3\cos^{-1}\text{x}
Differenting it with respect to x,
\frac{\text{dv}}{\text{dx}}=\frac{-3}{\sqrt{1-\text{x}^2}}\ .....(\text{iii})
From equation (ii),
\text{v}=\theta \Big[\text{since,}\tan^{-1}(\tan\theta)=\theta,\text{if }\in\Big(-\frac{\pi}{2},\frac{\pi}{2}\Big)\Big]

https://bls.smartstudies.co.in/#/exam/pdf-preview/c59cb220-8e86-4716-9ff7-82aec16b1ade/1 27/158
5/26/24, 6:19 PM Exam Automation
\Rightarrow\text{v}=\cos^{-1}\text{x}
Differentiating it with respect to x,
\frac{\text{dv}}{\text{dx}}=\frac{-1}{\sqrt{1-\text{x}^2}}\ .....(\text{iv})
Dividing equation (iii) by (iv),
\frac{\frac{\text{du}}{\text{dx}}}{\frac{\text{dv}}{\text{dx}}}=\Big(\frac{-3}{\sqrt{1-\text{x}^2}}\Big)\Big(\frac{-\sqrt{1-\text{x}^2}}{1}\Big)
\therefore\frac{\text{du}}{\text{dv}}=3
Q90. Differentiate the following functions with respect to x: 6 Marks
\tan^{-1}\Big(\frac{2\text{a}^{\text{x}}}{1-\text{a}^{2\text{x}}}\Big),\text{a}>1, -\infty<\text{x}<0

Ans: Let \text{y}=\tan^{-1}\Big(\frac{2\text{a}^{\text{x}}}{1-\text{a}^{2\text{x}}}\Big)


Put \text{a}^{\text{x}}=\tan\theta
\Rightarrow\text{y}=\tan^{-1}\Big\{\frac{2\times\text{a}^\text{x}}{1-(\text{a}^{\text{x}})^2}\Big\}
\Rightarrow\text{y}=\tan^{-1}\Big(\frac{2\tan\theta}{1-\tan^2\theta}\Big)
\Rightarrow\text{y}=\tan^{-1}(\tan2\theta)\ .....(\text{i})
Here, -\infty<\text{x}<0
\Rightarrow\text{a}^{-\infty}<\text{a}^{\text{x}}<2^{0}
\Rightarrow 0<\tan\theta<1
\Rightarrow 0<\theta<\frac{\pi}{4}
\Rightarrow 0<2\theta<\frac{\pi}{2}
So, from equation (i),
\text{y}=2\theta\Big[\text{Since},\tan^{-1}(\tan\theta)=\theta,\text{if }\theta\in\big(-\frac{\pi}{2},\frac{\pi}{2}\big)\Big]
\Rightarrow\text{y}=2\tan^{-1}(\text{a}^{\text{x}})
Differentiating it with respect to x,
\frac{\text{dy}}{\text{dx}}=\frac{2}{1+(\text{a}^{\text{x}})^2}\frac{\text{d}}{\text{dx}}(\text{a}^{\text{x}})
\Rightarrow\frac{\text{dy}}{\text{dx}}=\frac{2\times\text{a}^{\text{x}}\log_\text{e}\text{a}}{1+\text{a}^{2\text{x}}}
\therefore \frac{\text{dy}}{\text{dx}}=\frac{2\text{a}^{\text{x}}\log_\text{e}\text{a}}{1+\text{a}^{2\text{x}}}
Q91. Differentiate the following functions with respect to x: 6 Marks
\text{x}^{\text{x}^2-3}+(\text{x}-3)^{\text{x}^2}

Ans: Let \text{y}=\text{x}^{\text{x}^2-3}+(\text{x}-3)^{\text{x}^2}


Also, let \text{u}=\text{x}^{\text{x}^2-3}\text{ and v}=(\text{x}-3)^{\text{x}^2}
\therefore \text{y}=\text{u}+\text{v}
Differentiating both sides with respect to x, we obtain
\frac{\text{dy}}{\text{dx}}=\frac{\text{du}}{\text{dx}}+\frac{\text{dv}}{\text{dx}}\ .....(\text{i})
\text{u}=\text{x}^{\text{x}^2-3}
\log\text{u}=(\text{x}^2-3)\log\text{x}
Differentiating with respect to x, we obtain
\frac{1}{\text{u}}\frac{\text{du}}{\text{dx}}=\log\text{x}\times\frac{\text{d}}{\text{dx}}\big(\text{x}^2-3\big)+\big(\text{x}^2-3\big)\times\frac{\text{d}}{\text{dx}}(\log\text{x})
\Rightarrow\ \frac{1}{\text{u}}\frac{\text{du}}{\text{dx}}=\log\text{x}\times2\text{x}+(\text{x}^2-3)\times\frac{1}{\text{x}}
\Rightarrow\frac{\text{du}}{\text{dx}}=\text{x}^{\text{x}^2-3}\times\Big[\frac{\text{x}^2-3}{\text{x}}+2\text{x}\log\text{x}\Big]
Also,
\text{v}=(\text{x}-3)^{\text{x}^2}
\therefore\log\text{v}=\log(\text{x}-3)^{\text{x}^2}
\Rightarrow\log\text{v}=\text{x}^2\log(\text{x}-3)
Differentaiting both sides with respect to x, we obtain
\frac{1}{\text{v}}\times\frac{\text{dv}}{\text{dx}}=\log(\text{x}-3)\times\frac{\text{d}}{\text{dx}}(\text{x}^2)+\text{x}^2\times\frac{\text{d}}{\text{dx}}[\log(\text{x}-3)]
\Rightarrow\frac{1}{\text{v}}\frac{\text{dv}}{\text{dx}}=\log(\text{x}-3)\times2\text{x}+\text{x}^2\times\frac{1}{\text{x}-3}\times\frac{\text{d}}{\text{dx}}(\text{x}-3)
\Rightarrow\frac{\text{dv}}{\text{dx}}=\text{v}\Big[2\text{x}\log(\text{x}-3)+\frac{\text{x}^2}{\text{x}-3}\times1\Big]
\Rightarrow\frac{\text{dv}}{\text{dx}}=(\text{x}-3)^{\text{x}^2}\Big[\frac{\text{x}^2}{\text{x}-3}+2\text{x}\log(\text{x}-3)\Big]
Substituting the expressions of \frac{\text{du}}{\text{dx}} and \frac{\text{dv}}{\text{dx}} in equation (1), we obtain
\frac{\text{du}}{\text{dx}}=\text{x}^{\text{x}^2-3}\Big[\frac{\text{x}^2-3}{\text{x}}+2\text{x}\log\text{x}\Big] \\ +(\text{x}-3)^{\text{x}^2}\Big[\frac{\text{x}^2}
{\text{x}-3}+2\text{x}\log(\text{x}-3)\Big]
Q92. If \text{x}=3\cot-2\cos^3\text{t},\text{y}=3\sin\text{t}-2\sin^3\text{t} find \frac{\text{d}^2\text{y}}{\text{dx}^2}. 6 Marks

Ans: Given,
\text{x}=3\cot-2\cos^3\text{t},
\text{y}=3\sin\text{t}-2\sin^3\text{t}
Differentiating both w.r.t. t,
\frac{\text{dx}}{\text{dt}}=-3\sin\text{t}-6\cos^2\text{t}(-\sin\text{t})
\frac{\text{dx}}{\text{dt}}=-3\sin\text{t}+6\cos^2\text{t}\sin\text{t}
And \text{y}=3\sin\text{t}-2\sin^2\text{t}
Differentiating both w.r.t. t,
\frac{\text{dy}}{\text{dt}}=3\cos\text{t}-6\sin^2\text{t}\cos\text{t}
Now,
\frac{\text{dy}}{\text{dx}}=\frac{\frac{\text{dy}}{\text{dt}}}{\frac{\text{dx}}{\text{dt}}}
\Rightarrow\frac{\text{dy}}{\text{dx}}=\frac{\cot-2\sin^2\text{t}\cos\text{t}}{-\sin\text{t}+2\cos^2\text{t}\sin\text{t}}
\Rightarrow\frac{\text{dy}}{\text{dx}}=\frac{\cot[1-2\sin^2\text{t]}}{\sin\text{t}[2\cos^2\text{t}-1]}
\Rightarrow\frac{\text{dy}}{\text{dx}}=\cot\text{t}
Differentiating both w.r.t. x,
\frac{\text{d}^2\text{y}}{\text{dx}^2}=\frac{\text{d}(\cot\text{x})}{\text{dx}}=-\text{cosec}^2\text{x}
Q93. Find a point on the curve y = x3 + 1 where the tangent is parallel to the chord joining (1, 2) and (3, 28). 6 Marks

Ans: Let,
f(x) = x2 + 1
The tangent to the curve is parallel to the chord joining the points (1, 2) and (3, 28).
Assume that the chord joins the points (a, f(a)) and (b, f(b)).
\therefore a = 1, b = 3
The polynomial function is everywhere continuous and differentiable.
So, f(x) = x2 + 1 is continuous on [1, 3] and differentiable on (1, 3).
Thus, both the conditions of Lagrange's theorem are satisfied.
Concequently, there exists \text{c}\in(1,3) such that \text{f}'(\text{c})=\frac{\text{f}(3)-\text{f}(1)}{3-1}
Now,
f(x) = x2 + 1
⇒ f'(x) = 3x2, f(1) = 2, f(3) = 28
\therefore\ \text{f}'(\text{x})=\frac{\text{f}(3)-\text{f}(1)}{3-1}
\Rightarrow3\text{x}^2=\frac{26}{2}
\Rightarrow3\text{x}^2=13
\Rightarrow\text{x}=\pm\sqrt{\frac{13}{3}}

https://bls.smartstudies.co.in/#/exam/pdf-preview/c59cb220-8e86-4716-9ff7-82aec16b1ade/1 28/158
5/26/24, 6:19 PM Exam Automation
Thus, \text{c}=\sqrt{\frac{13}{3}} such that \text{f}'(\text{c})=\frac{\text{f}(3)-\text{f}(1)}{3-1}
Clearly,
\text{f}(\text{c})=\bigg[\Big(\frac{13}{3}\Big)^{\frac{3}{2}}+1\bigg]
Thus, c, f(c), i.e. \bigg(\sqrt{\frac{13}{3}},1+\Big(\frac{13}{3}\Big)^{\frac{3}{2}}\bigg), is a point on the given curve where the tangent is parallel to the chord joining the points (1, 2) and (3,
28).
Q94. If \sqrt{1-\text{x}^2}+\sqrt{1-\text{y}^2}=\text{a}(\text{x}-\text{y}), prove that \frac{\text{dy}}{\text{dx}}=\frac{\sqrt{1-\text{y}^2}}{1-\text{x}^2} 6 Marks

Ans: We have, \sqrt{1-\text{x}^2}+\sqrt{1-\text{y}^2}=\text{a}\big(\text{x}-\text{y}\big)


Let \text{x}=\sin\text{A},\text{y}=\sin\text{B}
\Rightarrow\sqrt{1-\sin^2\text{A}}+\sqrt{1-\sin^2\text{B}}=\text{a}\big(\sin\text{A}-\sin\text{B}\big)
\Rightarrow\cos\text{A}+\cos\text{B}=\text{a}\big(\sin\text{A}-\sin\text{B}\big)
\Rightarrow\text{a}=\frac{\cos\text{A}+\cos\text{B}}{\sin\text{A}-\sin\text{B}}
\Rightarrow\text{a}=\frac{2\cos\frac{\text{A}+\cos\text{B}}{2}\cos\frac{\text{A}-\text{B}}{2}}{2\cos\frac{\text{A}+\text{B}}{2}\sin\frac{\text{A}-\text{B}}{2}}
\begin{bmatrix}\because\sin\text{A}-\sin\text{B}=2\cos\frac{\text{A}+\text{B}}{2}\sin\frac{\text{A}-\text{B}}{2} \\ \because\cos \text{A}+\cos\text{B}=2\cos\frac{\text{A}+\text{B}}
{2}\cos\frac{\text{A}-\text{B}}{2}\end{bmatrix}
\Rightarrow\text{a}=\cot\Big(\frac{\text{A}-\text{B}}{2}\Big)
\Rightarrow\cot^{-1}\text{a}=\frac{\text{A}-\text{B}}{2}
\Rightarrow2\cot^{-1}\text{a}={\text{A}-\text{B}}
\Rightarrow2\cot^{-1}\text{a}=\sin^{-1}\text{x}-\sin^{-1}\text{y}
\Big[\because \text{x}=\sin\text{A},\text{y}=\sin\text{B}\Big]
Differentiating with respect to x, we get
\frac{\text{d}}{\text{dx}}\big(2\cot^{-1}\text{a}\big)=\frac{\text{d}}{\text{dx}}\big(\sin^{-1}\text{x}\big)-\frac{\text{d}}{\text{dx}}\big(\sin^{-1}\text{y}\big)
\Rightarrow0=\frac{1}{\sqrt{1-\text{x}^2}}-\frac{1}{\sqrt{1-\text{y}^2}}\frac{\text{dy}}{\text{dx}}
\Rightarrow\frac{1}{\sqrt{1-\text{y}^2}}\frac{\text{dy}}{\text{dx}}=\frac{1}{\sqrt{1-\text{x}^2}}
\Rightarrow\frac{\text{dy}}{\text{dx}}=\frac{\sqrt{1-\text{y}^2}}{\sqrt{1-\text{x}^2}}
\Rightarrow\frac{\text{dy}}{\text{dx}}=\sqrt\frac{1-\text{y}^2}{1-\text{x}^2}
Q95. If \text{y}=\frac{\text{e}^\text{x}-\text{e}^{-\text{x}}}{\text{e}^{\text{x}}+\text{e}^{-\text{x}}}, prove that \frac{\text{dy}}{\text{dx}}=1-\text{y}^2 6 Marks

Ans: Givne, \text{y}=\frac{\text{e}^{\text{x}}-\text{e}^{-\text{x}}}{\text{e}^{\text{x}}+\text{e}^{-\text{x}}}


Differentiate with respect to x,
\frac{\text{d}}{\text{dx}}=\frac{\text{d}}{\text{dx}}\Big(\frac{\text{e}^{\text{x}}-\text{e}^{-\text{x}}}{\text{e}^{\text{x}}+\text{e}^{-\text{x}}}\Big)
=\Bigg[\frac{\big(\text{e}^{\text{x}}+\text{e}^{-\text{x}}\big)\frac{\text{d}}{\text{dx}}\big(\text{e}^{\text{x}}-\text{e}^{-\text{x}}\big)\frac{\text{d}}
{\text{dx}}\big(\text{e}^{\text{x}}+\text{e}^{-\text{x}}\big)}{\big(\text{e}^{\text{x}}+\text{e}^{-\text{x}}\big)^2}\Bigg]
[Using quotient rule and chain rule]
=\begin{bmatrix} \frac{\big(\text{e}^{\text{x}}+\text{e}^{-\text{x}}\big)\Big[\text{e}^{\text{x}}-\text{e}^{-\text{x}}\frac{\text{d}}{\text{dx}}(-\text{x})-\big(\text{e}^{\text{x}}-\text{e}^{-
\text{x}}\big)\Big(\text{e}^{\text{x}}+\text{e}^{-\text{x}}\frac{\text{d}}{\text{dx}}(-\text{x})\Big)\Big]}{\big(\text{e}^{\text{x}}+\text{e}^{-\text{x}}\big)^2} \end{bmatrix}
=\begin{bmatrix} \frac{\big(\text{e}^{\text{x}}+\text{e}^{-\text{x}}\big)\big(\text{e}^{\text{x}}+\text{e}^{-\text{x}}\big)-\big(\text{e}^{\text{x}}-\text{e}^{-
\text{x}}\big)\big(\text{e}^{\text{x}}-\text{e}^{-\text{x}}\big)}{\big(\text{e}^{\text{x}}+\text{e}^{-\text{x}}\big)^2} \end{bmatrix}
=\bigg[\frac{\text{e}^{2\text{x}}+\text{e}^{-2\text{x}}+2\text{e}^{\text{x}}\times\text{e}^{-\text{x}}-\text{e}^{2\text{x}}-\text{e}^{-2\text{x}}+2\text{e}^{\text{x}}\text{e}^{-\text{x}}}
{\big(\text{e}^{\text{x}}+\text{e}^{-\text{x}}\big)^2}\bigg]
\frac{\text{dy}}{\text{dx}}\bigg[\frac{4}{(\text{e}^{\text{x}}+\text{e}^{-\text{x}})^2}\bigg]\ .....(\text{i})
Now,
1-\text{y}^2=1-\Big(\frac{\text{e}^{\text{x}}-\text{e}^{-\text{x}}}{\text{e}^{\text{x}}+\text{e}^{-\text{x}}}\Big)^2
=1-\frac{(\text{e}^{\text{x}}-\text{e}^{-\text{x}})^2}{(\text{e}^{\text{x}}+\text{e}^{-\text{x}})^2}
=\frac{(\text{e}^{\text{x}}+\text{e}^{-\text{x}})^2-(\text{e}^{\text{x}}-\text{e}^{-\text{x}})^2}{(\text{e}^{\text{x}}+\text{e}^{-\text{x}})^2}
=\frac{4}{(\text{e}^{\text{x}}+\text{e}^{-\text{x}})^2}
Q96. Differentiate the following functions from first principles: 6 Marks
\text{e}^{\sqrt{\cot\text{x}}}

Ans: Let \text{f(x)} =\text{e}^{\sqrt{\cot\text{x}}}


\Rightarrow\ \text{f}(\text{x}+\text{h})=\text{e}^{\sqrt{\cot\text{x}}}
\therefore \frac{\text{d}}{\text{dx}}\{\text{f(x)}\}=\lim\limits_{\text{h}\rightarrow0}\frac{\text{f}(\text{x}+\text{h})=\text{f(x)}}{\text{h}}
=\lim\limits_{\text{h}\rightarrow0}\frac{\text{e}^{\sqrt{\cot(\text{x}+\text{h})}}-\text{e}^{\sqrt{\cot\text{x}}}}{\text{h}}
=\lim\limits_{\text{h}\rightarrow0}\frac{\text{e}^{\sqrt{\cot\text{x}}}\Big(\text{e}^{\sqrt{\cot(\text{x}+\text{h})}-\sqrt{\cot\text{x}}}-1\Big)}{\text{h}}
=\text{e}^{\sqrt{\cot\text{x}}}\lim\limits_{\text{h}\rightarrow0}\bigg(\frac{\text{e}^{\sqrt{\cot(\text{x}+\text{h})}-\sqrt{\cot\text{x}}}-1}{{\sqrt{\cot(\text{x}+\text{h})}}-
\sqrt{\cot\text{x}}}\bigg)\times\bigg(\frac{\sqrt{\cot(\text{x}+\text{h})}-\sqrt{\cot\text{x}}}{\text{h}}\bigg)
=\text{e}^{\sqrt{\cot\text{x}}}\lim\limits_{\text{h}\rightarrow0}\frac{\sqrt{\cot(\text{x}+\text{h})}-\sqrt{\cot\text{x}}}{\text{h}}\times\frac{\sqrt{\cot(\text{x}+\text{h})}+\sqrt{\cot\text{x}}}
{\sqrt{\cot(\text{x}+\text{h})}+\sqrt{\cot\text{x}}}
\Big[\text{Since, } \lim\limits_{\text{h}\rightarrow0}\frac{\text{e}^\text{x}-1}{\text{x}}=1\text{ and rationalizing numerator}\Big]
\text{e}^{\sqrt{\cot\text{x}}}\lim\limits_{\text{h}\rightarrow0}\frac{\cot(\text{x}+\text{h})-\cot\text{x}}{\text{h}\big(\sqrt{\cot(\text{x}+\text{h})}+\sqrt{\cot\text{x}}\big)}
=\text{e}^{\sqrt{\cot\text{x}}}\lim\limits_{\text{h}\rightarrow0}\frac{\frac{\cot(\text{x}+\text{h})\cot\text{x}+1}{\cot(\text{x}+\text{h}-\text{x})}}
{\text{x}\big(\sqrt{\cot(\text{x}+\text{h})}+\sqrt{\cot\text{x}}\big)}
\Big[\text{Since,} \cot(\text{A}-\text{B})=\frac{\cot\text{A}\cot\text{B}+1}{\cot\text{A}-\cot\text{B}}\Big]
=\text{e}^{\sqrt{\cot\text{x}}}\lim\limits_{\text{h}\rightarrow0}\frac{\cot(\text{x}+\text{h})\cot\text{x}+1}{\cot(-
\text{h})\times\text{h}\big(\sqrt{\cot(\text{x}+\text{h})}+\sqrt{\cot\text{x}}\big)}
=\text{e}^{\sqrt{\cot\text{x}}}\lim\limits_{\text{h}\rightarrow0}\frac{\cot(\text{x}+\text{h})\cot\text{x}+1}{\Big(\frac{\text{h}}
{\cot\text{h}}\Big)\big(\sqrt{\cot(\text{x}+\text{h})}+\sqrt{\cot\text{x}}\big)}
=\frac{\text{e}^\sqrt{\cot\text{x}}\times(\cot^2\text{x}+1)}{2\sqrt{\cot\text{x}}}\ \Big[\because\ \lim\limits_{\text{x}\rightarrow0}\frac{\tan\text{x}}{\text{x}}=1\Big]
=-\frac{\text{e}^\sqrt{\cot\text{x}}\times\text{cosec}^2\text{x}}{2\sqrt{\cot\text{x}}}\ \big[\because(1+\cot^2\text{x})=\text{cosec}^2\text{x}\big]
\therefore\ \frac{\text{d}}{\text{dx}}\Big(\text{e}^\sqrt{\cot\text{x}}\Big)=-\frac{\text{e}^\sqrt{\cot\text{x}}\times\text{cosec}^2\text{x}}{2\sqrt{\cot}\text{x}}
Q97. If x and y are connected parametrically by the equations given in Exercise without eliminating the parameter, Find \frac{\text{dy}}{\text{dx}}. 6 Marks
\text{x}=\frac{\sin^3\text{t}}{\sqrt{\cos2\text{t}}},\text{y}\frac{\cos^3\text{t}}{\sqrt{\cos2\text{t}}}

Ans: The given equations are \text{x}=\frac{\sin^3\text{t}}{\sqrt{\cos2\text{t}}}\text{ and y}=\frac{\cos^3\text{t}}{\sqrt{\cos2\text{t}}}


Then, \frac{\text{dx}}{\text{dt}}= \frac{\text{d}}{\text{dt}}\Big[\frac{\sin^3\text{t}}{\sqrt{\cos2\text{t}}}\Big]
=\frac{\sqrt{\cos2\text{t}}.\frac{\text{d}}{\text{dt}}(\sin^3\text{t})-\sin^3\text{t}.\frac{\text{d}}{\text{dt}}\sqrt{\cos2\text{t}}}{\cos2\text{t}}
=\frac{\sqrt{\cos2\text{t}}.3\sin^2\text{t}.\frac{\text{d}}{\text{dt}}(\sin\text{t})-\sin^3\text{t}\times\frac{1}{2\sqrt{\cos2\text{t}}}.\frac{\text{d}}{\text{dt}}(\cos2\text{t})}{\cos2\text{t}}
=\frac{3\sqrt{\cos2\text{t}}.\sin^2\text{t}\cos\text{t}-\frac{\sin^3\text{t}}{2\sqrt{\cos2\text{t}}}.(-2\sin2\text{t})}{\cos2\text{t}}
=\frac{3\cos2\text{t}\sin^2\text{t}\cos\text{t}+\sin^3\text{t}\sin2\text{t}}{\cos2\text{t}\sqrt{\cos2\text{t}}}
\frac{\text{dy}}{\text{dt}}=\frac{\text{d}}{\text{dt}}\Big[\frac{\cos^3\text{t}}{\sqrt{\cos2\text{t}}}\Big]
=\frac{\sqrt{\cos2\text{t}}.\frac{\text{d}}{\text{dt}}(\cos^3\text{t})-\cos^3\text{t}.\frac{\text{d}}{\text{dt}}(\sqrt{\cos2\text{t}})}{\cos2\text{t}}
=\frac{\sqrt{\cos2\text{t}}\cos^2\text{t}.\frac{\text{d}}{\text{dt}}(\cos\text{t})-\cos^3\text{t}.\frac{1}{2\sqrt{\cos2\text{t}}}.\frac{\text{d}}{\text{dt}}(\cos2\text{t})}{\cos2\text{t}}
=\frac{3\sqrt{\cos2\text{t}}.\cos^2\text{t}(-\sin\text{t})-\cos^3\text{t}.\frac{1}{2\sqrt{\cos2\text{t}}}.(-2\sin2\text{t})}{\cos2\text{t}}
=\frac{-3\cos2\text{t}\cdot\cos^2\text{t}\cdot\sin\text{t}+\cos^3\text{t}.\sin2\text{t}}{\cos2\text{t}\cdot\sqrt{\cos2\text{t}}}
\therefore\ \frac{\text{dy}}{\text{dx}}=\frac{\Big(\frac{\text{dy}}{\text{dt}}\Big)}{\Big(\frac{\text{dx}}
{\text{dt}}\Big)}=\frac{-3\cos2\text{t}.\cos^2\text{t}.\sin\text{t}+\cos^3\text{t}\sin2\text{t}}{3\cos2\text{t}\sin^2\text{t}\cos\text{t}+\sin^3\text{t}\sin2\text{t}}
=\frac{-3\cos2\text{t}.\cos^2\text{t}.\sin\text{t}+\cos^3\text{t}(2\sin\text{t}\cos\text{t})}{3\cos2\text{t}\sin^2\text{t}\cos\text{t}+\sin^3\text{t}(2\sin\text{t}\cos\text{t})}
=\frac{\sin\text{t}\cos\text{t}[-3\cos2\text{t}.\cos\text{t}+2\cos^3\text{t}]}{\sin\text{t}\cos\text{t}[3\cos2\text{t}\sin\text{t}+2\sin^3\text{t}]}

https://bls.smartstudies.co.in/#/exam/pdf-preview/c59cb220-8e86-4716-9ff7-82aec16b1ade/1 29/158
5/26/24, 6:19 PM Exam Automation
=\frac{[-3(2\cos^2\text{t}-1)\cos\text{t}+2\cos^3\text{t}]}{[3(1-2\sin^2\text{t})\sin\text{t}+2\sin^3\text{t}]} \begin{bmatrix}\cos2\text{t}=(2\cos^2\text{t}-1). \\\cos2\text{t}=(1-
2\sin^2\text{t}) \end{bmatrix}
=\frac{-4\cos^3\text{t}+3\cos\text{t}}{3\sin\text{t}-4\sin^3\text{t}}
=\frac{-\cos3\text{t}}{\sin3\text{t}} \begin{bmatrix}\cos3\text{t}=4\cos^3\text{t}-3\cos\text{t}. \\\sin3\text{t}=3\sin\text{t}-4\sin^3\text{t} \end{bmatrix}
=-\cot3\text{t}
Q98. Differentiate the following functions from first principles: 6 Marks
\text{e}^{\sqrt{2\text{x}}}

Ans: Let \text{f(x)}=\text{e}^{\sqrt{2\text{x}}}


\Rightarrow\text{f}(\text{x}+\text{h})=\text{e}^{\sqrt{2(\text{x}+\text{h})}}
\therefore\frac{\text{d}}{\text{dx}}(\text{f(x)})=\lim\limits_{\text{h}\rightarrow0}\frac{\text{f}(\text{x}+\text{h})-\text{f}(\text{x})}{\text{h}}
=\lim\limits_{\text{h}\rightarrow0 }\frac{\text{e}^{\sqrt{2(\text{x}+\text{h})}}-\text{e}^{\sqrt{2\text{x}}}}{\text{h}}
=\lim\limits_{\text{h}\rightarrow0}\text{e}^{\sqrt{2\text{x}}}\frac{\text{e}^{\sqrt{2(\text{x}+\text{h})}-\sqrt{2\text{x}}}-1}{\text{h}}
=\text{e}^{\sqrt{2\text{x}}}\lim\limits_{\text{h}\rightarrow0}\Bigg(\frac{\big(\text{e}^{2(\text{x}+\text{h})-\sqrt{2\text{x}}}-1\big)}{\sqrt{2(\text{x}+\text{h})}-
\sqrt{2\text{x}}}\Bigg)\bigg(\frac{\sqrt{2(\text{x}+\text{h})}-\sqrt{2\text{x}}}{\text{h}}\bigg)
=\text{e}^{\sqrt{2\text{x}}}\lim\limits_{\text{h}\rightarrow0}\frac{\sqrt{2}(\text{x}+\text{h})-\sqrt{2\text{x}}}{\text{h}}\
\Big[\text{Since},\lim\limits_{\text{h}\rightarrow0}\frac{\text{e}^\text{h}-1}{\text{h}}=1\Big]
=\text{e}^{\sqrt{2\text{x}}}\lim\limits_{\text{h}\rightarrow0}\frac{\sqrt{2(\text{x}+\text{h})}-\sqrt{2\text{x}}}{\text{h}}\times\frac{\sqrt{2(\text{x}+\text{h})}+\sqrt{2\text{x}}}
{\sqrt{2(\text{x}+\text{x})}+\sqrt{2\text{x}}}
[\text{Rationalizing the numerator]}
=\text{de}^{\sqrt{2\text{x}}}\lim\limits_{\text{h}\rightarrow0}\frac{2(\text{x}+\text{h})-2\text{x}}{\text{h}\big(\sqrt{2(\text{x}+\text{h})}+\sqrt{2\text{x}}\big)}
=\text{e}^{\sqrt{2\text{x}}}\lim\limits_{\text{h}\rightarrow0}\frac{2\text{x}+2\text{h}-2\text{x}}{\text{h}\big(\sqrt{2}(\text{x}+\text{h})+\sqrt{2\text{x}}\big)}
=\text{e}^{\sqrt{2\text{x}}}\lim\limits_{\text{h}\rightarrow0}\frac{2\text{h}}{\text{h}\big(\sqrt{2(\text{x}+\text{h})}+\sqrt{2\text{x}}\big)}
=\text{e}^{\sqrt{2\text{x}}}\lim\limits_{\text{h}\rightarrow0}\frac{2\text{h}}{\big(\sqrt{2(\text{x}+\text{h})}+\sqrt{2\text{x}}\big)}
=\frac{\text{e}^{\sqrt{2\text{x}}}}{\sqrt{2\text{x}}}
So,
\frac{\text{d}}{\text{dx}}\big(\text{e}^{\sqrt{2\text{x}}}\big)=\frac{\text{e}^{2\text{x}}}{\sqrt{2\text{x}}}
Q99. \text{If y}=\text{e}^{\text{a}\cos^{-1}\text{x}},-1\leq\text{x}\leq1,\ \text{show that}(1-\text{x}^2)\frac{\text{d}^2\text{y}}{\text{dx}^2}-\text{x}\frac{\text{dy}}{\text{dx}}- 6 Marks
\text{a}^2\text{y}=0.

Ans: it is given that, \text{y}=\text{e}^{\text{a}\cos^{-1}\text{x}}


Taking logarithm on both the sides we obtain
\log\text{y}=\text{a}\cos^{-1}\text{x}\log\text{e}
\log\text{y}=\text{a}\cos^{-1}\text{x}
Differentiating both sides with respect to x, we obtain
\frac{1}{\text{y}}\frac{\text{dy}}{\text{dx}}=\text{a}\times\frac{-1}{\sqrt{1-\text{x}^2}}
\Rightarrow\ \frac{\text{dy}}{\text{dx}}=\frac{-\text{ay}}{\sqrt{1-\text{x}^2}}
By squaring both the sides, we obtain
\Big(\frac{\text{dy}}{\text{dx}}\Big)^2=\frac{\text{a}^2\text{y}^2}{1-\text{x}^2}
\Rightarrow\ (1-\text{x}^2)\Big(\frac{\text{dy}}{\text{dx}}\Big)^2=\text{a}^2\text{y}^2
(1-\text{x}^2)\Big(\frac{\text{dy}}{\text{dx}}\Big)^2=\text{a}^2\text{y}^2
Again differentiating both sides with respect to x, we obtain
\Big(\frac{\text{dy}}{\text{dx}}\Big)^2\frac{\text{d}}{\text{dx}}(1-\text{x}^2)+(1-\text{x}^2)\times\frac{\text{d}}{\text{dx}}\Big[\Big(\frac{\text{dy}}
{\text{dx}}\Big)^2\Big]=\text{a}^2\frac{\text{d}}{\text{dx}}(\text{y}^2)
\Rightarrow\ \Big(\frac{\text{dy}}{\text{dx}}\Big)^2(-2\text{x})+(1-\text{x}^2)\times2\frac{\text{dy}}{\text{dx}}.\frac{\text{d}^2\text{y}}{\text{dx}^2}=\text{a}^2.2\text{y}.\frac{\text{dy}}
{\text{dx}}
\Rightarrow\ -\text{x}\frac{\text{dy}}{\text{dx}}+(1-\text{x}^2)\frac{\text{d}^2\text{y}}{\text{dx}^2}=\text{a}^2.\text{y}\ \Big[\frac{\text{dy}}{\text{dx}}\neq0\Big]
\Rightarrow\ (1-\text{x}^2)\frac{\text{d}^2\text{y}}{\text{dx}^2}-\text{x}\frac{\text{dy}}{\text{dx}}-\text{a}^2\text{y}=0
Hence, proved.
Q100. Differentiate \sin^{-1}\Big(2\text{ax}\sqrt{1-\text{a}^2\text{x}^2}\Big) with respect to \sqrt{1-\text{a}^2\text{x}^2}, if -\frac{1}{\sqrt{2}}<\text{ax}<\frac{1}{\sqrt{2}}. 6 Marks

Ans: Let \text{u}=\sin^{-1}\Big(2\text{ax}\sqrt{1-\text{a}^2\text{x}^2}\Big)


Put \text{ax} =\sin\theta\Rightarrow\theta=\sin^{-1}(\text{ax})
\therefore\text{u}=\sin^{-1}\Big(2\sin\theta\sqrt{1-\sin^{2}\theta}\Big)
\Rightarrow\text{u}=\sin^{-1}(2\sin\theta\cos\theta)
\Rightarrow\text{u}=\sin^{-1}(\sin2\theta)\ .....(\text{i})
And
Let, \text{v}=\sqrt{1-\text{a}^2\text{x}^2}
Differentiating it with respect to x,
\frac{\text{dv}}{\text{dx}}=\frac{1}{2\sqrt{1-\text{a}^2\text{x}^2}}\times\frac{\text{d}}{\text{dx}}\big(1-\text{a}^2\text{x}^2\big)
\Rightarrow\frac{\text{dv}}{\text{dx}}=\Big(\frac{0-2\text{a}^2\text{x}}{2\sqrt{1-\text{a}^2\text{x}^2}}\Big)
\Rightarrow\frac{\text{dv}}{\text{dx}}=\frac{-\text{a}^2\text{x}}{\sqrt{1-\text{a}^2\text{x}^2}}\ .....(\text{ii})
Here,
-\frac{1}{\sqrt{2}}<\text{ax}<\frac{1}{\sqrt{2}}
\Rightarrow-\frac{1}{\sqrt{2}}<\sin\theta<\frac{1}{\sqrt{2}}
\Rightarrow-\frac{\pi}{4}<\theta<\frac{\pi}{4}
So, from equation (i),
\text{u}=2\theta\Big[\text{Since},\sin^{-1}(\sin\theta)=\theta,\text{if }\theta\in\Big[-\frac{\pi}{2},\frac{\pi}{2}\Big]\Big]
\Rightarrow\text{u}=2\sin^{-1}\text{x}
Differentiating it with respect to x,
\frac{\text{du}}{\text{dx}}=2\times\frac{1}{\sqrt{1-(\text{ax})^2}}\frac{\text{d}}{\text{dx}}(\text{ax})
\Rightarrow\frac{\text{du}}{\text{dx}}=\frac{2}{1-\text{a}^2\text{x}^2}(\text{a})
\Rightarrow\frac{\text{du}}{\text{dx}}=\frac{2\text{a}}{1-\text{a}^2\text{x}^2}\ .....(\text{iii})
Dividing equation (iii) by (ii),
\frac{\frac{\text{du}}{\text{dx}}}{\frac{\text{dv}}{\text{dx}}}=\Big(\frac{2\text{a}}{\sqrt{1-\text{a}^2\text{x}^2}}\Big)\Big(\frac{\sqrt{1-\text{a}^2\text{x}^2}}{\text{-a}^2\text{x}}\Big)
\therefore\frac{\text{du}}{\text{dv}}=-\frac{2}{\text{ax}}
Q101. Differentiate the following functions with respect to x: 6 Marks
\text{x}^{(\sin\text{x}-\cos\text{x})}+\frac{\text{x}^2-1}{\text{x}^2+1}

Ans: Let \text{y}=\text{x}^{(\sin\text{x}-\cos\text{x})}+\Big(\frac{\text{x}^2-1}{\text{x}^2+1}\Big)


\text{y}=\text{e}^{\log\text{x}^{\sin\text{c}-\cos\text{x}}}+\Big(\frac{\text{x}^2-1}{\text{x}^2+1}\Big)
\text{y}=\text{e}^{(\sin\text{c}-\cos\text{x})\log\text{x}}+\Big(\frac{\text{x}^2-1}{\text{x}^2+1}\Big)
\big[\text{Since},\text{e}^{\log\text{a}}=\text{a},\log\text{a}^\text{b}=\text{b}\log\text{a}\big]
Differentiating it with respect to x using chain rule and quotient rule,
\frac{\text{dy}}{\text{dx}}=\frac{\text{d}}{\text{dx}}\Big[\text{e}^{(\sin\text{x}-\cos\text{x})\log\text{x}}\Big]+\frac{\text{d}}{\text{dx}}\Big[\frac{\text{x}^2-1}{\text{x}^2+1}\Big]
=\text{e}^{(\sin\text{x}-\cos\text{x})\log\text{x}}+\frac{\text{d}}{\text{dx}}\big\{(\sin\text{x}-\cos\text{x})\log\text{x}\big\} \\ +\bigg[\frac{(\text{x}^2+1)\frac{\text{d}}{\text{dx}}(\text{x}^2-
1)-(\text{x}^2-1)\frac{\text{d}}{\text{dx}}(\text{x}^2+1)}{(\text{x}^2+1)^2}\bigg]

https://bls.smartstudies.co.in/#/exam/pdf-preview/c59cb220-8e86-4716-9ff7-82aec16b1ade/1 30/158
5/26/24, 6:19 PM Exam Automation
=\text{e}^{\log\text{x}^{(\sin\text{x}-\cos\text{x})}}\Big[(\sin\text{x}-\cos\text{x})\frac{\text{d}}{\text{dx}}(\log\text{x})+(\log\text{x})\frac{\text{d}}{\text{dx}}(\sin\text{x}-\cos\text{x})\Big]
\\+\Big[\frac{(\text{x}^2+1)(2\text{x})-(\text{x}^2-1)(2\text{x})}{(\text{x}^2+1)^2}\Big]
=\text{e}^{(\sin\text{x}-\cos\text{x})}\Big[(\sin\text{x}-\cos\text{x}\big(\frac{1}{\text{x}}\big)+\log\text{x}(\sin\text{x}+\cos\text{x})\Big] \\
+\Big[\frac{2\text{x}^3+2\text{x}-2\text{x}^3+2\text{x}}{(\text{x}^2+1)^2}\Big]
\frac{\text{dy}}{\text{dx}}=\text{x}^{\sin\text{x}-\cos\text{x}}\Big[\frac{(\sin\text{x}-\cos\text{x})}{\text{x}}+\log\text{x}(\sin\text{x}+\cos\text{x})\Big]+\frac{4\text{x}}{(\text{x}^2+1)^2}
Q102. If \text{f(x)}=\begin{cases}\text{ax}^2-\text{b}, & \text{if |x|}<1\\\frac{1}{|\text{x}|}, & \text{if |x|}\geq1\end{cases} is differentiable at x = 1, find a, b. 6 Marks

Ans: Here,
\text{f(x)}=\begin{cases}\text{ax}^2-\text{b}, & \text{if |x|}<1\\\frac{1}{|\text{x}|}, & \text{if |x|}\geq1\end{cases}
=\begin{cases}-\frac{1}{\text{x}}, & \text{if |x|}\leq-1\\\text{ax}^2-\text{b}, & \text{if}-1<\text{x}<1\\\frac{1}{\text{x}},&\text{if x}\geq1\end{cases}
\text{LHL }=\lim\limits_{\text{x}\rightarrow1^{-}}\text{f(x)}
=\lim\limits_{\text{h}\rightarrow0}\text{f}(1-\text{h})
=\lim\limits_{\text{h}\rightarrow0}\text{a}(1-\text{h})^2-\text{b}
= \text{a}- \text{b}
\text{RHL }=\lim\limits_{\text{x}\rightarrow1^{+}}\text{f(x)}
=\lim\limits_{\text{h}\rightarrow0}\text{f}(1+\text{h})
=\lim\limits_{\text{h}\rightarrow0}\frac{1}{1+\text{h}}
Since, f(x) is continuous, so
LHL = RHL
a - b = 1 .......(1)
(LHL at x = 1) =\lim\limits_{\text{x}\rightarrow1^{-}}\frac{\text{f(x)}-\text{f}(1)}{\text{x}-1}
\lim\limits_{\text{h}\rightarrow0}\frac{\text{f}(1-\text{h})-1}{1-\text{h}-1}
=\lim\limits_{\text{h}\rightarrow0}\frac{\text{a}(1-\text{h})^2-\text{b}-1}{-\text{h}}
=\lim\limits_{\text{h}\rightarrow0}\frac{\text{a}(1-\text{h})^2-(\text{a}-1)-1}{-\text{h}}
Using equation (1),
=\lim\limits_{\text{h}\rightarrow0}\frac{\text{a}+\text{ah}^2-2\text{ah}-\text{a}+1-1}{-\text{h}}
=\lim\limits_{\text{h}\rightarrow0}\frac{\text{ah}^2-2\text{ah}}{-\text{h}}
=\lim\limits_{\text{h}\rightarrow0}(2\text{a}-\text{ah})
=2\text{a}
RHL at x = 1 =\lim_\limits{\text{x}\rightarrow1^{+}}\frac{\text{f(x)}-\text{f}(1)}{\text{x}-1}
\lim_\limits{\text{h}\rightarrow0}\frac{\text{f}(1+\text{h})-{\text{f}(1)}}{1+\text{h}-1}
\lim_\limits{\text{h}\rightarrow0}\frac{\frac{1}{1+\text{h}}-1}{\text{h}}
\lim_\limits{\text{h}\rightarrow0}\frac{1-1-\text{h}}{(1+\text{h})\text{h}}
= -1
Since f(x) is differentiable at x = 1,
(LHL at x = 1) = (RHL at x = 1)
2a = -1
\text{a}=\frac{-1}{2}
Put \text{a}=\frac{-1}{2} in equation (1),
a-b=1
\Big(\frac{-1}{2}\Big)-\text{b}=1
\text{b}=\frac{-1}{2}-1
\text{b}=\frac{-3}{2}
\text{a}=\frac{-1}{2}
Q103. If \text{x}=\text{a}\Big(\text{t}+\frac{1}{\text{t}}\Big)\text{ and y}=\text{a}\Big(\text{t}-\frac{1}{\text{t}}\Big), prove that \frac{\text{dy}}{\text{dx}}=\frac{\text{x}}{\text{y}} 5 Marks

Ans: Here, \text{x}=\text{a}\Big(\text{t}+\frac{1}{\text{t}}\Big)


Differentiating it with respect to t,
\frac{\text{dx}}{\text{dt}}=\text{a}\frac{\text{d}}{\text{dt}}\Big(\text{t}+\frac{1}{\text{t}}\Big)
=\text{a}\Big(\text{t}-\frac{1}{\text{t}^{2}}\Big)
\frac{\text{dx}}{\text{dt}}=\text{a}\Big(\frac{\text{t}^{2}-1}{\text{t}^{2}}\Big)\ .....(\text{i})
And, \text{y}=\text{a}\Big(\text{t}-\frac{1}{\text{t}}\Big)
Differetiating it with respect to t,
\frac{\text{dy}}{\text{dt}}=\text{a}\frac{\text{d}}{\text{dt}}\Big(\text{t}-\frac{1}{\text{t}}\Big)
=\text{a}\Big(1+\frac{1}{\text{t}^{2}}\Big)
\frac{\text{dy}}{\text{dt}}=\text{a}\Big(\frac{\text{t}^{2}+1}{\text{t}^{2}}\Big)\ .....(\text{ii})
Dividing equation (ii) by (i),
\frac{\frac{\text{dy}}{\text{dt}}}{\frac{\text{dx}}{\text{dt}}}=\text{a}\frac{(\text{t}^{2}+1)}{\text{t}^{2}}\times\frac{\text{t}^{2}}{\text{a}(\text{t}^{2}-1)}
\frac{\text{dy}}{\text{dx}}=\frac{\text{t}^{2}+1}{\text{t}^{2}-1}
\frac{\text{dy}}{\text{dx}}=\frac{\text{x}}{\text{y}}
\Big[\text{Since},\frac{\text{x}}{\text{y}}=\frac{\text{a}(\text{t}^{2}+1)}{\text{t}}\times\frac{\text{t}}{\text{a}(\text{t}^{2}-1)}=\Big(\frac{\text{t}^{2}+1}{\text{t}^{2}-1}\Big)\Big]
Q104. Find \frac{\text{dy}}{\text{dx}} 5 Marks
y = xn + nx + xx + nn

Ans: We have, y = xn + nx + xx + nx
\Rightarrow\text{y}=\text{x}^\text{n}+\text{n}^\text{x}+\text{e}^{\log\text{x}^\text{x}}+\text{n}^\text{n}
\Rightarrow\text{y}=\text{x}^\text{n}+\text{n}^\text{x}+\text{e}^{\text{x}\log\text{x}}+\text{n}^\text{n}
Differentiate with respect to x,
\frac{\text{dy}}{\text{dx}}=\frac{\text{d}}{\text{dx}}(\text{x}^\text{n})+\frac{\text{d}}{\text{dx}}(\text{n}^\text{x})+\frac{\text{d}}{\text{dx}}(\text{e}^{\text{x}\log\text{x}})+\frac{\text{d}}
{\text{dx}}(\text{n}^\text{n})
=\text{nx}^{\text{n}-1}+\text{n}^\text{x}\log\text{n}=\text{e}^{\log\text{x}^\text{x}}\Big[\text{x}\frac{\text{d}}{\text{dx}}\log\text{x}+\log\text{x}\frac{\text{d}}{\text{dx}}(\text{x})\Big]
=\text{nx}^{\text{n}-1}+\text{n}^\text{x}\log\text{n}=\text{x}^{\text{x}}\Big[\text{x}\big(\frac{1}{\text{x}}\big)+\log\text{x}\Big]
=\text{nx}^{\text{n}-1}+\text{n}^\text{x}\log\text{n}=\text{x}^{\text{x}}\big[1+\log\text{x}\big]
=\text{nx}^{\text{n}-1}+\text{n}^\text{x}\log\text{n}=\text{x}^{\text{x}}\big[\log\text{e}+\log\text{x}\big] \\ \big[\because\log_\text{e}\text{e}=1\text{ and
}\log\text{A}+\log\text{B}=\log(\text{AB})\big]
=\text{nx}^{\text{n}-1}+\text{n}^\text{x}\log\text{n}=\text{x}^{\text{x}}\log\big(\text{ex}\big)
Q105. If the value of c prescribed in Roll's theorem for the function 5 Marks
\text{f}(\text{x})=2\text{x}(\text{x}-3)^{\text{n}} on the interval \big[0,2\sqrt3\big] is \frac{3}{4}, write the value of n (a positive integers).

Ans: We have,
\text{f}(\text{x})=2\text{x}(\text{x}-3)^{\text{n}}
Differentiating the given function with respect to x, we get
\text{f}'(\text{x})=2\big[\text{xn}(\text{x}-3)^{\text{n}-1}+(\text{x}-3)^{\text{n}}\big]
\Rightarrow\text{f}'(\text{x})=2(\text{x}-3)^{\text{n}}\Big[\frac{\text{xn}}{(\text{x}-3)}+1\Big]
\Rightarrow\text{f}'(\text{c})=2(\text{c}-3)^{\text{n}}\Big[\frac{\text{cn}}{(\text{c}-3)}+1\Big]
Given:
\text{f}'\Big(\frac{3}{4}\Big)=0
\therefore\ 2-\Big(\frac{9}{4}\Big)^{\text{n}}\Bigg[\frac{\frac{3}{4}\text{n}}{\big(\frac{-9}{4}\big)}+1\Bigg]=0

https://bls.smartstudies.co.in/#/exam/pdf-preview/c59cb220-8e86-4716-9ff7-82aec16b1ade/1 31/158
5/26/24, 6:19 PM Exam Automation
\Rightarrow2-\Big(\frac{9}{4}\Big)^{\text{n}}\Big[\frac{-\text{n}}{3}+1\Big]=0
\Rightarrow\Big[\frac{-\text{n}}{3}+1\Big]=0
\Rightarrow-\text{n}+3=0
\Rightarrow\text{n}=3
Q106. If ey= yx, prove that \frac{\text{dy}}{\text{dx}}=\frac{(\log\text{y})^2}{\log\text{y}-1} 5 Marks

Ans: We have, ey = yx
Taking log on both sides,
\log\text{e}^{\text{y}}=\log\text{y}^\text{x}
\Rightarrow\text{y}\log\text{e}=\text{x}\log\text{y}
\Rightarrow\text{y}=\text{x}\log\text{y}\ .....(\text{i})
Differentiating with respect to x,
\frac{\text{dy}}{\text{dx}}=\frac{\text{d}}{\text{dx}}(\text{x}\log\text{y})
\Rightarrow\frac{\text{dy}}{\text{dx}}=\text{x}\frac{\text{dy}}{\text{dx}}(\log\text{y})+\log\text{y}\frac{\text{d}}{\text{dx}}(\text{x})
\Rightarrow\frac{\text{dy}}{\text{dx}}=\frac{\text{x}}{\text{y}}\frac{\text{dy}}{\text{dx}}+\log\text{y}
\Rightarrow\frac{\text{dy}}{\text{dx}}\Big(1-\frac{\text{x}}{\text{y}}\Big)=\log\text{y}
\Rightarrow\frac{\text{dy}}{\text{dx}}\big(\frac{\text{y}-\text{x}}{\text{y}}\big)=\log\text{y}
\Rightarrow\frac{\text{dy}}{\text{dx}}=\frac{\text{y}\log\text{y}}{\text{y}-\text{x}}
\Rightarrow\frac{\text{y}\log\text{y}}{\Big(\text{y}-\frac{\text{y}}{\log\text{y}}\Big)}
[Using equation (i)]
\Rightarrow\frac{\text{dy}}{\text{dx}}=\frac{\text{y}\log\text{y}(\log\text{y})}{\text{y}\log\text{y}-\text{y}}
\Rightarrow\frac{\text{dy}}{\text{dx}}=\frac{\text{y}(\log\text{y})^2}{\text{y}(\log\text{y}-1)}
\Rightarrow\frac{\text{dy}}{\text{dx}}=\frac{(\log\text{y})^2}{(\log\text{y}-1)}
Q107. If \text{x}=\text{e}^{\cos2\text{t}} and \text{y}=\text{e}^{\sin2\text{t}}, prove that \frac{\text{dy}}{\text{dx}}=-\frac{\text{y}\log\text{x}}{\text{x}\log\text{y}} 5 Marks

Ans: We have, \text{x}=(\text{e}^{\cos2\text{t}}) and \text{y}=\text{e}^{\sin2\text{t}}


\Rightarrow\frac{\text{dx}}{\text{dt}}=\frac{\text{d}}{\text{dt}}\big(\text{e}^{\cos2\text{t}}\big) and \frac{\text{dy}}{\text{dt}}=\frac{\text{d}}{\text{dt}}\big(\text{e}^{\sin2\text{t}}\big)
\Rightarrow\frac{\text{dx}}{\text{dt}}=\text{e}^{\cos2\text{t}}\frac{\text{d}}{\text{dt}}(\cos2\text{t}) and \frac{\text{dx}}{\text{dt}}=\text{e}^{\cos2\text{t}}\frac{\text{d}}{\text{dt}}
(\cos2\text{t})
\Rightarrow\frac{\text{dx}}{\text{dt}}=\text{e}^{\cos2\text{t}}(-\sin2\text{t})\frac{\text{d}}{\text{dt}}(2\text{t}) and \frac{\text{dy}}{\text{dt}}=\text{e}^{\sin2\text{t}}
(\cos2\text{t})\frac{\text{d}}{\text{dt}}(2\text{t})
\Rightarrow\frac{\text{dx}}{\text{dt}}=-2\sin 2\text{t}\text{e}^{\cos2\text{t}} and \frac{\text{dy}}{\text{dt}}=2\cos 2\text{t}\text{e}^{\sin2\text{t}}
\because\frac{\frac{\text{dy}}{\text{dt}}}{\frac{\text{dx}}{\text{dt}}}=\frac{2\cos2\text{te}^{\sin2\text{t}}}{-2\sin2\text{te}^{\cos2\text{t}}}
\begin{bmatrix} \because\text{x}=\text{e}^{\cos2\text{t}}\Rightarrow\log\text{x}=\cos2\text{t} \\ \text{y}=\text{e}^{\sin2\text{t}}\Rightarrow\log\text{y}=\sin2\text{t} \end{bmatrix}
\Rightarrow\frac{\text{dy}}{\text{dx}}=-\frac{\text{y}\log\text{x}}{\text{x}\log\text{y}}
Q108. Verify Rolle's theorem of the following function on the indicated interval 5 Marks
\text{f}(\text{x})=2\sin\text{x}+\sin2\text{x}\text{ on }[0,\pi]

Ans: Here,
\text{f}(\text{x})=2\sin\text{x}+\sin2\text{x}\text{ on }[0,\pi]
We know that sine function is continuous and differentiable every where, so f(x) is continuous is [0,\pi] and differentiable is (0,\pi).
Now,
\text{f}(0)=2\sin0+\sin0=0
\text{f}(\pi)=2\sin\pi+\sin2\pi=0
\Rightarrow\text{f}(0)=\text{f}(\pi)
So, Rolle's theorem is applicable, so there must exist a point \text{c}\in(0,\pi) such that f'(c) = 0.
Now,
\text{f}(\text{x})=2\sin\text{x}+\sin2\text{x}
\text{f}'(\text{x})=2\cos\text{x}+2\cos2\text{x}
Now,
\text{f}'(\text{c})=0
2\cos\text{c}+2\cos2\text{c}=0
\Rightarrow2(\cos\text{c}+2\cos^2\text{c}-1)=0
\Rightarrow(2\cos^2+2\cos\text{c}-\cos\text{c}-1)=0
\Rightarrow(2\cos\text{c}-1)(\cos\text{c}+1)=0
\Rightarrow\cos\text{c}=\frac{1}{2},\cos\text{c}=-1
\Rightarrow\tan\text{c}=1
\text{c}=\frac{\pi}{3}\in(0,\pi),\text{c}=\pi
Hence, Rolle's theorem is verified.
Q109. Differentiate the following functions with respect to x: 5 Marks
\frac{\text{e}^{2\text{x}}+\text{e}^{-2\text{x}}}{\text{e}^{2\text{x}}-\text{e}^{-2\text{x}}}

Ans: Let, \text{y}=\frac{\text{e}^{2\text{x}}+\text{e}^{-2\text{x}}}{\text{e}^{2\text{x}}-\text{e}^{-2\text{x}}}


Differentiate with respect to x,
\frac{\text{dy}}{\text{dx}}=\frac{\text{d}}{\text{dx}}\Big[\frac{\text{e}^{2\text{x}}+\text{e}^{-2\text{x}}}{\text{e}^{2\text{x}}-\text{e}^{-2\text{x}}}\Big]
=\Bigg[\frac{\big(\text{e}^{2\text{x}}-\text{e}^{-2\text{x}}\big)\frac{\text{d}}{\text{dx}}\big(\text{e}^{2\text{x}}+\text{e}^{-2\text{x}}\big)-
\big(\text{e}^{2\text{x}}+\text{e}^{-2\text{x}}\big)\frac{\text{d}}{\text{dx}}\big(\text{e}^{2\text{x}}-\text{e}^{-2\text{x}}\big)}{\big(\text{e}^{2\text{x}}-\text{e}^{4\text{x}}\big)^2}\Bigg]
[Using quotient rule and chain rule]
=\frac{(\text{e}^{2\text{x}}-\text{e}^{-2\text{x}})\Big[\text{e}^{2\text{x}}\frac{\text{d}}{\text{dx}}(2\text{x})+\text{e}^{-2\text{x}}\frac{\text{d}}{\text{dx}}(-2\text{x})\Big]-
\big(\text{e}^{2\text{x}}+\text{e}^{-2\text{x}}\big)\Big[\text{e}^{2\text{x}}\frac{\text{d}}{\text{dx}}(2\text{x})-\text{e}^{-2\text{x}}\frac{\text{d}}{\text{dx}}(-2\text{x})\Big]}
{\big(\text{e}^{2\text{x}}-\text{e}^{2\text{x}}\big)^2}
=\frac{\big(\text{e}^{2\text{x}}-\text{e}^{-2\text{x}}\big)\big(2\text{e}^{2\text{x}}-2\text{e}^{-2\text{e}}\big)-
\big(\text{e}^{2\text{x}}+\text{e}^{-2\text{x}}\big)\big(2\text{e}^{2\text{e}}+2\text{e}^{-2\text{x}}\big)}{\big(\text{e}^{2\text{e}}-\text{e}^{-2\text{x}}\big)^2}
=\frac{2\big(\text{e}^{2\text{x}}-\text{e}^{-2\text{x}}\big)^2-2\big(\text{e}^{2\text{x}}+\text{e}^{-2\text{x}}\big)^2}{\big(\text{e}^{2\text{x}}-\text{e}^{-2\text{x}}\big)^2}
=\frac{2\big[\text{e}^{4\text{x}}+\text{e}^{-4\text{x}}-2\text{e}^{2\text{x}}\text{e}^{-2\text{x}}-\text{e}^{4\text{x}}-\text{e}^{-4\text{x}}-2\text{e}^{2\text{x}}\text{e}^{-2\text{x}}\big]}
{\big(\text{e}^{2\text{x}}-\text{e}^{-2\text{x}}\big)^2}
=\frac{-8}{\big(\text{e}^{2\text{x}}-\text{e}^{-2\text{x}}\big)^2}
So,
\frac{\text{d}}{\text{dx}}\Big(\frac{\text{e}^{2\text{x}}+\text{e}^{-2\text{x}}}{\text{e}^{2\text{x}}-\text{e}^{-2\text{x}}}\Big)=\frac{-8}{\big(\text{e}^{2\text{x}}-\text{e}^{-2\text{x}}\big)^2}
Q110. If \text{y}=(\sin\text{x}-\cos\text{x})^{\sin\text{x}-\cos\text{x}},\frac{\pi}{4}<\text{x}<\frac{3\pi}{4}, find \frac{\text{dy}}{\text{dx}} 5 Marks

Ans: We have, \text{y}=(\sin\text{x}-\cos\text{x})^{\sin\text{x}-\cos\text{x}}\ .....(\text{i})


Taking log on both sides,
\log\text{y}=\log(\sin\text{x}-\cos\text{x})^{\sin\text{x}-\cos\text{x}}
\Rightarrow\log\text{y}=(\sin\text{x}-\cos\text{x})^{\sin\text{x}-\cos\text{x}}
\Rightarrow\frac{1}{\text{y}}\frac{\text{dy}}{\text{dx}}=\log(\sin\text{x}-\cos\text{x})\frac{\text{d}}{\text{dx}}(\sin\text{x}-\cos\text{x}) \\ +(\sin\text{x}-\cos\text{x})\frac{\text{d}}
{\text{dx}}\log(\sin\text{x}-\cos\text{x})

https://bls.smartstudies.co.in/#/exam/pdf-preview/c59cb220-8e86-4716-9ff7-82aec16b1ade/1 32/158
5/26/24, 6:19 PM Exam Automation
\Rightarrow\frac{1}{\text{y}}\frac{\text{dy}}{\text{dx}}=(\sin\text{x}-\cos\text{x})(\sin\text{x}-\cos\text{x}) \\ +\frac{(\sin\text{x}-\cos\text{x})}{(\sin\text{x}-\cos\text{x})}\frac{\text{d}}
{\text{dx}}(\sin\text{x}-\cos\text{x})
\Rightarrow\frac{1}{\text{y}}\frac{\text{dy}}{\text{dx}}=(\cos\text{x}+\sin\text{x})\log(\sin\text{x}-\cos\text{x})+(\cos\text{x}+\sin\text{x})
\Rightarrow\frac{1}{\text{y}}\frac{\text{dy}}{\text{dx}}=(\cos\text{x}+\sin\text{x})[1+\log(\sin\text{x}-\cos\text{x})]
\Rightarrow\frac{\text{dy}}{\text{dx}}=\text{y}\big[(\cos\text{x}+\sin\text{x})\big\{1+\log(\sin\text{x}-\cos\text{x})\big\}\big]
\Rightarrow\frac{\text{dy}}{\text{dx}}=(\sin\text{x}-\cos\text{x})^{(\sin\text{x}-\cos\text{x})} \\ \big[(\cos\text{x}+\sin\text{x})\big\{1+\log(\sin\text{x}-\cos\text{x})\big\}\big]
Q111. If (\sin\text{x})^{\text{y}}=\text{x}+\text{y}, prove that \frac{\text{dy}}{\text{dx}}=\frac{1-(\text{x}+\text{y})\text{y}\cot\text{x}}{(\text{x}+\text{y})\log\sin\text{x}-1} 5 Marks

Ans: Here,
(\sin\text{x})^{\text{y}}=\text{x}+\text{y}
Taking log on both the sides,
\log(\sin\text{x})^\text{y}=\log(\text{x}+\text{y})
\text{y}\log(\sin\text{x})=\log(\text{x}+\text{y})\ \big[\text{Since},\log\text{a}^\text{b}=\text{b}\log\text{a}\big]
Differentiating it with respect to x using chain rule, product rule,
\frac{\text{d}}{\text{dx}}(\text{y}\log(\sin\text{x}))=\frac{\text{d}}{\text{dx}}\log(\text{x}+\text{y})
\text{y}\frac{\text{d}}{\text{dx}}\log\sin\text{x}+\log\sin\text{x}\frac{\text{dy}}{\text{dx}}=\frac{1}{\text{x}+\text{y}}\frac{\text{d}}{\text{dx}}(\text{x}+\text{y})
\frac{\text{y}}{\sin\text{x}}\frac{\text{d}}{\text{dx}}(\sin\text{x})+\log\sin\text{x}\frac{\text{dy}}{\text{dx}}=\frac{1}{(\text{x}+\text{y})}\Big[1+\frac{\text{dy}}{\text{dx}}\Big]
\frac{\text{y}(\cos\text{x})}{(\sin\text{x})}+\log\sin\text{x}\frac{\text{dy}}{\text{dx}}=\frac{1}{(\text{x}+\text{y})}+\frac{1}{(\text{x}+\text{y})}\frac{\text{dy}}{\text{dx}}
\frac{\text{dy}}{\text{dx}}\Big(\log\sin\text{x}-\frac{1}{\text{x}+\text{y}}\Big)=\frac{1}{(\text{x}+\text{y})}-\text{y}\cot\text{x}
\frac{\text{dy}}{\text{dx}}\Big(\frac{(\text{x}+\text{y})\log\sin\text{x}-1}{(\text{x}+\text{y})}\Big)=\Big(\frac{1-\text{y}(\text{x}+\text{y})\cot\text{x}}{\text{x}+\text{y}}\Big)
\frac{\text{dy}}{\text{dx}}=\Big(\frac{1-\text{y}(\text{x}+\text{y})\cot\text{x}}{(\text{x}+\text{y})\log\sin\text{x}-1}\Big)
Q112. Differentiate the following functions with respect to x: 5 Marks
(\log\text{x})^\text{x}

Ans: Let \text{y}=(\log\text{x})^\text{x}\ .....(\text{i})


Taking log on both the sides,
\log\text{y}=\log(\log\text{x})^\text{x}
\Rightarrow\log\text{y}=\text{x}\log(\log\text{x})\ \big[\text{Since}, \log\text{a}^\text{b}=\text{b}\log\text{a}\big]
Differentiating with respect to x, using product rule, chain rule,
\frac{1}{\text{y}}\frac{\text{dy}}{\text{dx}}=\text{x}\frac{\text{d}}{\text{dx}}\log(\log\text{x})+\log\log\text{x}\frac{\text{d}}{\text{dx}}(\text{x})
=\text{x}\frac{1}{\log\text{x}}\frac{\text{d}}{\text{dx}}(\log\text{x})+\log\log\text{x}(1)
=\frac{\text{x}}{\log\text{x}}\Big(\frac{1}{\text{x}}\Big)+\log\log\text{x}
\frac{1}{\text{y}}\frac{\text{dy}}{\text{dx}}=\frac{1}{\log\text{x}}+\log\log\text{x}
\frac{\text{dy}}{\text{dx}}=\text{y}\Big[\frac{1}{\log\text{x}}+\log\log\text{x}\Big]
\frac{\text{dy}}{\text{dx}}=(\log\text{x})^\text{x}\Big[\frac{1}{\log\text{x}}+\log\log\text{x}\Big]
[Using equation (i)]
Q113. If \text{xy}\log(\text{x}+\text{y})=1, prove that \frac{\text{dy}}{\text{dx}}=\frac{\text{y}(\text{x}^2\text{y}+\text{x}+\text{y})}{\text{x}(\text{xy}^2+\text{x}+\text{y})} 5 Marks

Ans: Here,
\text{xy}\log(\text{x}+\text{y})=1\ .....(\text{i})
Differentiaitng with respect to x using chain rula, product rule,
\frac{\text{dy}}{\text{dx}}(\text{xy}\log(\text{x}+\text{y}))=\frac{\text{d}}{\text{dx}}(1)
\text{xy}\frac{\text{d}}{\text{dx}}\log(\text{x}+\text{y})+\text{x}\log(\text{x}+\text{y})\frac{\text{dy}}{\text{dx}}+\text{y}\log(\text{x}+\text{y})\frac{\text{d}}{\text{dx}}(\text{x})=0
\frac{\text{xy}}{(\text{x}+\text{y})}\Big(1+\frac{\text{dy}}{\text{dx}}\Big)+\text{x}\log(\text{x}+\text{y})\frac{\text{dy}}{\text{dx}}+\text{y}\log(\text{x}+\text{y})(1)=0
\Big(\frac{\text{xy}}{\text{x}+\text{y}}\Big)\Big(1+\frac{\text{dy}}{\text{dx}}\Big)+\text{x}\log(\text{x}+\text{y})\frac{\text{dy}}{\text{dx}}+\text{x}\log(\text{x}+\text{y})=0
\Big(\frac{\text{xy}}{\text{x}+\text{y}}\Big)\frac{\text{dy}}{\text{dx}}+\frac{\text{xy}}{\text{x}+\text{y}}+\text{x}\Big(\frac{1}{\text{xy}}\Big)\frac{\text{dy}}{\text{dx}}+\text{y}\Big(\frac{1}
{\text{xy}}\Big)=0
[Using equation (i)]
\frac{\text{dy}}{\text{dx}}\Big[\frac{\text{xy}}{\text{x}+\text{y}}+\frac{1}{\text{y}}\Big]=-\Big[\frac{1}{\text{x}}+\frac{\text{xy}}{\text{x}+\text{y}}\Big]
\frac{\text{dy}}{\text{dx}}\Big[\frac{\text{xy}^2+\text{x}+\text{y}}{(\text{x}+\text{y})\text{y}}\Big]=-\Big[\frac{\text{x}+\text{y}+\text{x}^2\text{y}}{\text{x}(\text{x}+\text{y})}\Big]
\frac{\text{dy}}{\text{dx}}=-\frac{\text{y}}{\text{x}}\Big(\frac{\text{x}+\text{y}+\text{x}^2\text{y}}{\text{x}+\text{y}+\text{xy}^2}\Big)
Q114. Differentiate the following functions with respect to x: 5 Marks
(\sin\text{x})^{\log\text{x}}

Ans: Let \text{y}=(\sin\text{x})^{\log\text{x}}\ .....(\text{i})


Taking log on both sides,
\log\text{y}=\log(\sin\text{x})^{\log\text{x}}
\Rightarrow\log\text{y}=\log\text{x}\log\sin\text{x}
Differentiating with respect to x,
\frac{1}{\text{y}}\frac{\text{dy}}{\text{dx}}=\log\text{x}\frac{\text{d}}{\text{dx}}(\sin\text{x})+\log\sin\text{x}\frac{\text{d}}{\text{dx}}(\log\text{x})
\Rightarrow \frac{1}{\text{y}}\frac{\text{dy}}{\text{dx}}=\log\text{x}\frac{1}{\sin\text{x}}\frac{\text{d}}{\text{dx}}(\sin\text{x})+\log\sin\text{x}\Big(\frac{1}{\text{x}}\Big)
\Rightarrow\frac{1}{\text{y}}\frac{\text{dy}}{\text{dx}}=\frac{\log\text{x}}{\sin\text{x}}(\cos\text{x})+\frac{\log\sin\text{x}}{\text{x}}
\Rightarrow\frac{\text{dy}}{\text{dx}}=\text{y}\Big[\log\text{x}\cot\text{x}+\frac{\log\sin\text{x}}{\text{x}}\Big]
\Rightarrow\frac{\text{dy}}{\text{dx}}=(\sin\text{x})^{\log\text{x}}\Big[\log\text{x}\cot\text{x}+\frac{\log\sin\text{x}}{\text{x}}\Big]
[Using equation (i)]
Q115. Differentiate the following functions with respect to x: 5 Marks
\tan^{-1}\Big(\frac{\text{a}+\text{b}\tan\text{x}}{\text{b}-\text{a}\tan\text{x}}\Big)

Ans: Let, \text{y}=\tan^{-1}\Big[\frac{\text{a}+\text{b}\tan\text{x}}{\text{b}-\text{a}\tan\text{x}}\Big]


\Rightarrow\text{y}=\tan^{-1}\bigg[\frac{\frac{\text{a}+\text{b}\tan\text{x}}{\text{b}}}{\frac{\text{b}-\text{a}\tan\text{x}}{\text{b}}}\bigg]
\Rightarrow\text{y}=\tan^{-1}\bigg[\frac{\frac{\text{a}}{\text{b}}+\tan\text{x}}{1-\frac{\text{a}}{\text{b}}\tan\text{x}}\bigg]
\Rightarrow\text{y}=\tan^{-1}\Bigg[\frac{\tan\Big(\tan^{-1}\frac{\text{a}}{\text{b}}\Big)+\tan\text{x}}{1-\tan\Big(\tan^{-1}\frac{\text{a}}{\text{b}}\Big)\times\tan\text{x}}\Bigg]
\Rightarrow\text{y}=\tan^{-1}\Big[\tan\Big(\tan^{-1}\frac{\text{a}}{\text{b}}+\text{x}\Big)\Big]
\Rightarrow\text{y}=\tan^{-1}\big(\frac{\text{a}}{\text{b}}\big)+\text{x}
Differentiate it with respect to x,
\frac{\text{dy}}{\text{dx}}=0+1
\therefore\ \frac{\text{dy}}{\text{dx}}=1
Q116. Find \frac{\text{dy}}{\text{dx}} 5 Marks
\text{y}=(\tan\text{x})^{\log\text{x}}+\cos^2\big(\frac{\pi}{4}\big)

Ans: Here,
\text{y}=(\tan\text{x})^{\log\text{x}}+\cos^2\big(\frac{\pi}{4}\big)
\text{y}=\text{e}^{\log(\tan\text{x})^{\log\text{x}}}+\cos^2\big(\frac{\pi}{2}\big)
\text{y}=\text{e}^{\log\text{x}\log\tan\text{x}}+\cos\text{x}^2\big(\frac{\pi}{4}\big)
\big[\text{Since, e}^{\log\text{a}}=\text{a and}\log\text{a}^\text{b}=\text{b}\log\text{a}\big]
Differentiating ti using chain rule and product rule,
\frac{\text{dy}}{\text{dx}}=\frac{\text{d}}{\text{dx}}\big(\text{e}^{\log\text{x}\log\tan\text{x}})+\frac{\text{d}}{\text{dx}}\cos^2\big(\frac{\pi}{4}\big)
=\text{e}^{\log\text{x}\log\tan\text{x}}\frac{\text{d}}{\text{dx}}(\log\text{x}\log\tan\text{x})+0
=\text{e}^{\log(\tan\text{x})^{\log\text{x}}}\Big[\log\times\frac{\text{d}}{\text{dx}}(\log\tan\text{x})+\log\tan\text{x}\frac{\text{d}}{\text{dx}}(\log\text{x})\Big]

https://bls.smartstudies.co.in/#/exam/pdf-preview/c59cb220-8e86-4716-9ff7-82aec16b1ade/1 33/158
5/26/24, 6:19 PM Exam Automation
=(\tan\text{x})^{\log\text{x}}\Big[\log\times\Big(\frac{1}{\tan\text{x}}\Big)\frac{\text{d}}{\text{dx}}(\tan\text{x})+\log\tan\text{x}\Big(\frac{1}{\text{x}}\Big)\Big]
=(\tan\text{x})^{\log\text{x}}\Big[\log\times\Big(\frac{1}{\tan\text{x}}\Big)\big(\sec^2\text{x}\big)+\frac{\log\tan\text{x}}{\text{x}}\Big]
\frac{\text{dy}}{\text{dx}}=(\tan\text{x})^{\log\text{x}}\Big[\log\times\Big(\frac{\sec^2\text{x}}{\tan\text{x}}\Big)+\frac{\log\tan\text{x}}{\text{x}}\Big]
Q117. Derivative of y = f(x) w.r.t. x (if exists) is denoted by \frac{\text{dy}}{\text{dx}} or f'(x) and is called the first order derivative of y. If we take derivative of \frac{\text{dy}}{\text{dx}} again, then 5 Marks
we get \frac{\text{d}}{\text{dx}}\Big(\frac{\text{dy}}{\text{dx}}\Big)=\frac{\text{d}^2\text{y}}{\text{dx}^2} or f''(x) and is called the second order derivative of y. Similarly, \frac{\text{d}}
{\text{dx}}\Big(\frac{\text{d}^2\text{y}}{\text{dx}^2}\Big) is denoted and defined as \frac{\text{d}^3\text{y}}{\text{dx}^3} or f'''(x) and is known as third order derivative of y and so on.
Based on the above information, answer the following questions.
1. If \text{y}=\tan^{-1}\Big(\frac{\log(\frac{\text{e}}{\text{x}^2})}{\log(\text{ex}^2)}\Big)+\tan^{-1}\Big(\frac{3+2\log\text{x}}{1-6\log\text{x}}\Big), then \frac{\text{d}^2\text{y}}
{\text{dx}^2} is equal to:
1. 2
2. 1
3. 0
4. -1
2. If u = x2 + y2 and x = s + 3t, y = 2s - t, then \frac{\text{d}^2\text{u}}{\text{ds}^2} is equal to:
1. 12
2. 32
3. 36
4. 10
3. If \text{f}(\text{x})=2\log\sin\text{x}, then f''(x) is equal to:
1. 2\text{cosec}^3\text{x}
2. 2\cot^2\text{x}-4\text{x}^2\text{cosec}^2\text{x}^2
3. 2\text{x}\cot\text{x}^2
4. -2\text{cosec}^2\text{x}
4. If \text{f}(\text{x})=\text{e}^\text{x}\sin\text{x}, then f'''(x) =
1. 2\text{e}^\text{x}(\sin\text{x}+\cos\text{x})
2. 2\text{e}^\text{x}(\cos\text{x}-\sin\text{x})
3. 2\text{e}^\text{x}(\sin\text{x}-\cos\text{x})
4. 2\text{e}^\text{x}\cos\text{x}
5. If \text{y}^2=\text{ax}^2+\text{bx}+\text{c}, then \frac{\text{d}}{\text{dx}}(\text{y}^3\text{y}_2)=
1. 1
2. -1
3. \frac{4\text{ac}-\text{b}^2}{\text{a}^2}
4. 0

Ans: 1. (c) 0
Solution:
Given, \text{y}=\tan^{-1}\Bigg(\frac{\log\big(\frac{\text{e}}{\text{x}^2}\big)}{\log\text{ex}^2}\Bigg)+\tan^{-1}\Big(\frac{3+2\log\text{x}}{1-6\log\text{x}}\Big)
=\tan^{-1}\Big(\frac{1-\log\text{x}^2}{1+\log\text{x}^2}\Big)+\tan^{-1}\Big(\frac{3+2\log\text{x}}{1-6\log\text{x}}\Big)
=\tan^{-1}(1)-\tan^{-1}(2\log\text{x})+\tan^{-1}(3)+\tan^{-1}(2\log\text{x})
\Rightarrow\text{y}=\tan^{-1}(1)+\tan^{-1}(3)
\Rightarrow\frac{\text{dy}}{\text{dx}}=0\Rightarrow\frac{\text{d}^2\text{y}}{\text{dx}^2}=0
2. (d) 10
Solution:
Given, \text{x}=\text{s}+3\text{t},\text{y}=2\text{s}-\text{t}\Rightarrow\frac{\text{dx}}{\text{ds}}=1,\frac{\text{dy}}{\text{ds}}=2
Now, \text{u}=\text{x}^2+\text{y}^2\Rightarrow\frac{\text{du}}{\text{ds}}=2\text{x}\frac{\text{dx}}{\text{ds}}+2\text{y}\frac{\text{dy}}{\text{ds}}=2\text{x}+4\text{y}
\Rightarrow\frac{\text{d}^2\text{u}}{\text{ds}^2}=2\Big(\frac{\text{dx}}{\text{ds}}\Big)+4\Big(\frac{\text{dy}}{\text{ds}}\Big)\Rightarrow\frac{\text{d}^2\text{u}}{\text{ds}^2}=2(1)+4(2)=10
3. (d) -2\text{cosec}^2\text{x}
Solution:
We have, \text{f}(\text{x})=2\log\sin\text{x}
\Rightarrow\text{f}'(\text{x})=2\cdot\frac{1}{\sin\text{x}}\cdot\cos\text{x}=2\cot\text{x}\Rightarrow\text{f}''(\text{x})=-2\text{cosec}^2\text{x}
4. (b) 2\text{e}^\text{x}(\cos\text{x}-\sin\text{x})
Solution:
We have, \text{f}(\text{x})=\text{e}^\text{x}\sin\text{x}
\Rightarrow\text{f}'(\text{x})=\text{e}^\text{x}\cos\text{x}+\text{e}^\text{x}\sin\text{x}=\text{e}^\text{x}(\cos\text{x}+\sin\text{x})
\Rightarrow\text{f}''(\text{x})=\text{e}^\text{x}(\cos\text{x}-\sin\text{x})+\text{e}^\text{x}(\cos\text{x}+\sin\text{x})=2\text{e}^\text{x}\cos\text{x}
\Rightarrow\text{f}'''(\text{x})=2[\text{e}^\text{x}\cos\text{x}-\text{e}^\text{x}\sin\text{x}]=2\text{e}^\text{x}[\cos\text{x}-\sin\text{x}]
5. (d) 0
Solution:
Given, y2 = ax2 + bx + c
⇒ 2yy1 = 2ax + b
⇒ 2yy1 + y1(2y1) = 2a
\Rightarrow\text{yy}_2=\text{a}-\text{y}^2_1\Rightarrow\text{yy}_2=\text{a}-\Big(\frac{2\text{ax}+\text{b}}{2\text{y}}\Big)^2
=\frac{4\text{y}^2\text{a}-(4\text{a}^2\text{x}^2+\text{b}^2+4\text{abx})}{4\text{y}^2}
\Rightarrow\text{y}^3\text{y}_2=\frac{4\text{a}(\text{ax}^2+\text{bx}+\text{c})-(4\text{a}^2\text{x}^2+\text{b}^2+4\text{abx})}{4}
=\frac{4\text{ac}-\text{b}^2}{4}
\Rightarrow\frac{\text{d}}{\text{dx}}(\text{y}^3\text{y}_2)=0
Q118. Verify Rolle's theorem for the following function on the indicated intervals 5 Marks
f(x) = x2 + 5 x + 6 on the interval [-3, -2]

Ans: Here, f(x) = x2 + 5 x + 6 on [-3, -2]


f(x) is continuous is [-3, -2] and f(x) is differentiable is (-3, -2) since it is a polynomial function.
Now,
f(x) = x2 + 5x + 6
f(-3) = (-3)2 +5(-3) + 6
= 9 - 15 + 6
f(-3) = 0 ....(i)
f(-2) = (-2)2 + 5(-2) + 6
= 4 - 10 + 6
f(-2) = 20 ....(ii)
From equation (i) and (ii),
f(-3) = f(-2)
So, Rolle's theorem is applicable is [-3, -2], we have to show that
f'(c) = 0 as \text{c}\in (-3,-2)
Now,
f(x) = x2 + 5x + 6
f'(x) = 2x + 5
⇒ f'(c) = 0

https://bls.smartstudies.co.in/#/exam/pdf-preview/c59cb220-8e86-4716-9ff7-82aec16b1ade/1 34/158
5/26/24, 6:19 PM Exam Automation
2c + 5 = 0
\text{c}=\frac{-5}{2}\in(-3,-2)
So, Rolle's theorem is verified.
Q119. If (\cos\text{x})^{\text{y}}=(\tan\text{y})^{\text{x}}, Prove that \frac{\text{dy}}{\text{dx}}=\frac{\log\tan\text{y}-\text{y}\tan\text{x}}{\log\cos\text{x}-\text{x}\sec\text{y cosec y}} 5 Marks

Ans: Here,
(\cos\text{x})^{\text{y}}=(\tan\text{y})^{\text{x}}
Taking log on both sides,
\log(\cos\text{x})^{\text{y}}=\log(\tan\text{y})^{\text{x}}
\text{y}\log(\cos\text{x})=\text{x}\log(\tan\text{y})
\big[\text{Since}, \log\text{e}^{\text{b}}=\text{b}\log\text{a}\big]
Differentiating it with respect to x using chain rule and product rule,
\frac{\text{d}}{\text{dx}}(\text{y}\log\cos\text{x})=\frac{\text{d}}{\text{dx}}(\text{x}\log\tan\text{y})
\Big(\text{y}\frac{\text{d}}{\text{dx}}\log\cos\text{x}+\log\cos\text{x}\frac{\text{dy}}{\text{dx}}\Big) \\ =\Big(\text{x}\frac{\text{d}}{\text{dx}}\log\tan\text{y}+\log\tan\text{y}\frac{\text{d}}
{\text{dx}}(\text{x})\Big)
\Big(\text{y}\big(\frac{1}{\cos\text{x}}\big)\frac{\text{d}}{\text{dx}}(\cos\text{x})+\log\cos\text{x}\frac{\text{dy}}{\text{dx}}\Big) \\ =\Big(\text{x}\frac{1}{\tan\text{y}}\frac{\text{d}}
{\text{dx}}(\tan\text{y})+\log\tan\text{y}(1)\Big)
\Big(\frac{\text{y}}{\cos\text{x}}(-\sin\text{x})+\log\cos\text{x}\frac{\text{dy}}{\text{dx}}\Big)\\ =\Big(\frac{\text{x}}{\tan\text{y}}(\sec^2\text{y})\Big)\frac{\text{dy}}
{\text{dx}}+\log\tan\text{y}-\text{y}\tan\text{x}+\log\cos\text{x}\frac{\text{dy}}{\text{dx}} \\ =\Big(\sec\text{y cosec y}\times\text{y}\frac{\text{dy}}{\text{dx}}+\log\tan\text{y}\Big)
\frac{\text{dy}}{\text{dx}}\big[\log\cos\text{x}-\text{x}\sec\text{y cosec y}\big] \\ =\log\tan\text{y}+\text{y}\tan\text{x}
\frac{\text{dy}}{\text{dx}}=\Big[\frac{\log\tan\text{x}+\text{y}\tan\text{x}}{\log\cos\text{x}-\text{x}\sec\text{y cosec y}}\Big]
Q120. If \text{x}=\text{a}(\cos\theta+\theta\sin\theta),\text{y}=\text{a}(\sin\theta-\theta\cos\theta) prove that \frac{\text{d}^2\text{x}}{\text{d}\theta^2}=\text{a}(\cos\theta- 5 Marks
\theta\sin\theta),\frac{\text{d}^2}{\text{d}\theta^2} =\text{a}(\sin\theta-\theta\cos\theta)\ \text{and}\ \frac{\text{d}^2\text{y}}{\text{dx}^2}=\frac{\sec^3\theta}{\text{a}\theta}

Ans: It is given that, \text{x}=\text{a}(\cos\text{t}+\text{t}\sin\text{t}),\text{y}=\text{a}(\sin\text{t}-\text{t}\cos\text{t})


\therefore\frac{\text{dx}}{\text{dt}}=\text{a}.\frac{\text{d}}{\text{dt}}(\cos\text{t}+\text{t}\sin\text{t})
=\text{a}\Big[-\sin\text{t}+\sin\text{t}.\frac{\text{d}}{\text{dt}.}(\text{t})+\frac{\text{d}}{\text{dt}}(\sin\text{t})\Big]
=\text{a}[-\sin\text{t}+\sin\text{t}+t\cos\text{t}]=\text{at}\cos\text{t}
\frac{\text{dy}}{\text{dt}}=\text{a}.\frac{\text{d}}{\text{dt}}(\sin\text{t}-\text{t}\cos\text{t})
=\text{a}\Big[\cos\text{t}-\Big\{\cos\text{t}\frac{\text{d}}{\text{dt}}(\text{t})+\text{t}.\frac{\text{d}}{\text{dt}}(\cos\text{t})\Big\}\Big]
=\text{a}[\cos\text{t}-\{\cos\text{t}-\text{t}\sin\text{t}\}]=\text{at}\sin\text{t}
then, \therefore\frac{\text{dy}}{\text{dx}}=\frac{\Big(\frac{\text{dy}}{\text{dt}}\Big)}{\Big(\frac{\text{dx}}{\text{dt}}\Big)}=\frac{\text{at}\sin\text{t}}{\text{at}\cos\text{t}}=\tan\text{t}
\frac{\text{d}^2\text{y}}{\text{dx}^2}=\frac{\text{d}}{\text{dx}}\Big(\frac{\text{dy}}{\text{dx}}\Big)=\frac{\text{d}}{\text{dx}}(\tan\text{t})=\sec^2\text{t}.\frac{\text{dt}}{\text{dx}}
=\sec^2\text{t}.\frac{1}{\text{at}\cos\text{t}}\ \Big[\frac{\text{dx}}{\text{dt}}=\text{at}\cos\text{t}\Rightarrow\frac{\text{dt}}{\text{dx}}=\frac{1}{\text{at}\cos\text{t}}\Big]
=\frac{\sec^3\text{t}}{\text{at}},0<\text{t}<\frac{\pi}{2}
Q121. If \text{y}=\cot^{-1}\Big\{\frac{\sqrt{1+\sin\text{x}}+\sqrt{1-\sin\text{x}}}{\sqrt{1+\sin\text{x}}-\sqrt{1-\sin\theta}}\Big\}, show that \frac{\text{dy}}{\text{dx}} is independent of x. 5 Marks

Ans: Let \text{y}=\cot^{-1}\Big[\frac{\sqrt{1+\sin\text{x}}+\sqrt{1-\sin\text{x}}}{\sqrt{1+\sin\text{x}}-\sqrt{1-\sin\theta}}\Big] \ .....(\text{i})


Then, \frac{\sqrt{1+\sin\theta}+\sqrt{1-\sin\text{x}}}{\sqrt{1+\sin\text{x}}-\sqrt{1-\sin\text{x}}}
=\frac{\big(\sqrt{1+\sin\text{x}}+\sqrt{1-\sin\text{x}}\big)^2}{\big(\sqrt{1+\sin\text{x}}-\sqrt{1-\sin\text{x}}\big)\big(\sqrt{1+\sin\text{x}}+\sqrt{1-\sin\text{x}}\big)}
=\frac{(1+\sin\text{x})+(1-\sin\text{x})+2\sqrt{(1-\sin\text{x})(1+\sin\text{x})}}{(1+\sin\text{x})-(1-\sin\text{x})}
=\frac{2+2\sqrt{1-\sin^2\text{c}}}{2\sin\text{x}}
=\frac{1+\cos\text{x}}{\sin\text{x}}
=\frac{2\cos^2\frac{\text{x}}{2}}{2\sin\frac{\text{x}}{2}\cos\frac{\text{x}}{2}}
=\cot\frac{\text{x}}{2}
Therefore, equation (i) becomes
\text{y}=\cot^{-1}\Big(\cot\frac{\text{x}}{2}\Big)
\Rightarrow\ \text{y}=\frac{\text{x}}{2}
\therefore\ \frac{\text{dy}}{\text{dx}}=\frac{1}{2}\frac{\text{d}}{\text{dx}}(\text{x})
\Rightarrow \frac{\text{dy}}{\text{dx}}=\frac{1}{2}
Q122. Differentiate the following functions with respect to x: 5 Marks
\text{e}^{\sin\text{x}}+(\tan\text{x})^\text{x}

Ans: Let \text{y}=\text{e}^{\sin\text{x}}+(\tan\text{x})^\text{x}


\Rightarrow\text{y}=\text{e}^{\sin\text{x}}+\text{e}^{\log(\tan\text{x})^\text{x}}
\Rightarrow\text{y}=\text{e}^{\sin\text{x}}+\text{e}^{\text{x}\log(\tan\text{x})}
Differentiating with resepect to x,
\frac{\text{dy}}{\text{dx}}=\frac{\text{d}}{\text{dx}}\big(\text{e}^{\sin\text{x}}\big)+\frac{\text{d}}{\text{dx}}\big\{\text{e}^{\text{x}\log(\tan\text{x})}\big\}
=\text{e}^{\sin\text{x}}\frac{\text{d}}{\text{dx}}(\sin\text{x})+\text{e}^{\text{x}\log(\tan\text{x})}\frac{\text{d}}{\text{dx}}(\text{x}\log\tan\text{x})
=\text{e}^{\sin\text{x}}(\cos\text{x})+\text{e}^{\log(\tan\text{x})^\text{x}}\Big[\text{x}\frac{\text{d}}{\text{dx}}(\log\tan\text{x})+\log\tan\text{x}\frac{\text{d}}{\text{dx}}(\text{x})\Big]
=\text{e}^{\sin\text{x}}(\cos\text{x})+(\tan\text{x})^\text{x}\Big[\frac{\text{x}}{\tan\text{x}}(\sec^2\text{x})+\log\tan\text{x}\Big]
=\text{e}^{\sin\text{x}}(\cos\text{x})+(\tan\text{x})^\text{x}\big[\text{x}\sec\text{x cosec x}+\log\tan\text{x}\big]
Q123. If \tan^{-1}\Big(\frac{\text{x}^2-\text{y}^2}{\text{x}^2+\text{y}^2}\Big)=\text{a} prove that \frac{\text{dx}}{\text{dx}}=\frac{\text{y}}{\text{x}}\frac{(1-\tan\text{a})}{(1+\tan\text{a})} 5 Marks

Ans: We have, \tan^{-1}\Big(\frac{\text{x}^2-\text{y}^2}{\text{x}^2+\text{y}^2}\Big)=\text{a}


\Rightarrow\frac{\text{x}^2-\text{y}^2}{\text{x}^2+\text{y}^2}=\tan\text{a}
\Rightarrow\text{x}^2-\text{y}^2=\tan\text{a}(\text{x}^2+\text{y}^2)
Differentiating with respect to x, we get
\Rightarrow\frac{\text{d}}{\text{dx}}(\text{x}^2-\text{y}^2)=\tan\text{a}\frac{\text{d}}{\text{dx}}(\text{x}^2+\text{y}^2)
\Rightarrow\Big(2\text{x}-2\text{y}\frac{\text{dy}}{\text{dx}}\Big)=\tan\text{a}\Big(2\text{x}+2\text{y}\frac{\text{dy}}{\text{dx}}\Big)
\Rightarrow2\text{x}-2\text{y}\frac{\text{dy}}{\text{dx}}=2\text{x}\tan\text{a}+2\text{y}\tan\text{a}\frac{\text{dy}}{\text{dx}}
\Rightarrow2\text{y}\tan\text{a}\frac{\text{dy}}{\text{dx}}+2\text{y}\frac{\text{dy}}{\text{dx}}=2\text{x}-2\text{x}\tan\text{a}
\Rightarrow2\text{y}(1+\tan\text{a})\frac{\text{dy}}{\text{dx}}=2\text{x}(1-\tan\text{a})
\Rightarrow\frac{\text{dy}}{\text{dx}}=\frac{\text{x}}{\text{y}}\Big(\frac{1-\tan\text{a}}{1+\tan\text{a}}\Big)
Q124. Find whether the function is differentiable at x = 1 and x = 2 5 Marks
\text{f(x)}=\begin{cases}\text{x} & \text{x}\leq1\\2-\text{x} & 1\leq\text{x}\leq2\\-2+3\text{x}&\text{x}>2\end{cases}

Ans: \text{f(x)}=\begin{cases}\text{x} & \text{x}\leq1\\2-\text{x} & 1\leq\text{x}\leq2\\-2+3\text{x}&\text{x}>2\end{cases}


\text{f(x)}=\begin{cases}\text{x} & \text{x}\leq1\\-1 & 1\leq\text{x}\leq2\\3-2\text{x}&\text{x}>2\end{cases}
Now,
LHL =\lim_\limits{\text{x}\rightarrow1^{-}}\text{f}'(\text{x})=\lim_\limits{\text{x}\rightarrow1^{-}}1=1
RHL =\lim_\limits{\text{x}\rightarrow1^{+}}\text{f}'(\text{x})=\lim_\limits{\text{x}\rightarrow1^{+}}-1=-1
Since, at \text{x}=1,\text{LHL}\neq\text{RHL}
Hence, f(x) is not differentiable at x = 1
Again,
LHL =\lim_\limits{\text{x}\rightarrow2^{-}}\text{f}'(\text{x})=\lim_\limits{\text{x}\rightarrow2^{-}}-1=-1
RHL =\lim_\limits{\text{x}\rightarrow2^{+}}\text{f}'(\text{x})=\lim_\limits{\text{x}\rightarrow2^{+}}3-2\text{x}=3-4=-1
Since, at x = 2, LHL = RHL
Hence, f(x) is differentiable at x = 2.

https://bls.smartstudies.co.in/#/exam/pdf-preview/c59cb220-8e86-4716-9ff7-82aec16b1ade/1 35/158
5/26/24, 6:19 PM Exam Automation
Q125. Find the values of a and b such that the function f defined by \text{f(x)}=\begin{cases}\frac{\text{x}-4}{|\text{x}-4|}+\text{a},&\text{if x}<4\\\text{a+}\text{b},&\text{if x}=4\\\frac{\text{x}-4} 5 Marks
{|\text{x}-4|}+\text{b},&\text{if x}>4\end{cases} is a continuous function at x = 4.

Ans: Consider, \text{f(x)}=\begin{cases}\frac{\text{x}-4}{|\text{x}-4|}+\text{a},&\text{if x}<4\\\text{a+}\text{b},&\text{if x}=4\\\frac{\text{x}-4}{|\text{x}-4|}+\text{b},&\text{if x}>4\end{cases}


At x = 4 \text{L.H.L}=\lim\limits_{\text{x}\rightarrow4^-}\frac{\text{x}-4}{|\text{x}-4|}+\text{a}
=\lim\limits_{\text{h}\rightarrow0}\frac{4-\text{h}-4}{|4-\text{h}-4|}+\text{a}=\lim\limits_{\text{h}\rightarrow0}\frac{-\text{h}}{\text{h}}+\text{a}
=-1+\text{a}
\text{R.H.L}=\lim\limits_{\text{x}\rightarrow4^+}\frac{\text{x}-4}{|\text{x}-4|}+\text{b}
=\lim\limits_{\text{h}\rightarrow0}\frac{4+\text{h}-4}{|4+\text{h}-4|}+\text{b}=\lim\limits_{\text{h}\rightarrow0}\frac{\text{h}}{\text{h}}+\text{b}=1+\text{b}
\text{f}(4)=\text{a}+\text{b}
Since, f(x) is continuous at x = 0
\therefore L.H.L = R.H.L = f(0)
⇒ -1 + a = 1 + b = a + b
⇒ -1 + a = a + b and 1 + b = a + b
\therefore b = -1 and a = 1
Q126. Verify Lagrange's mean value theorem for the following function on the indicated intervals. find a point 'c' in the indicated interval as stated by the Lagrange's mean value theorem. 5 Marks
\text{f}(\text{x})=\tan^{-1}\text{x}\text{ on }[0,1]

Ans: We have,
\text{f}(\text{x})=\tan^{-1}\text{x}
Clearly, f(x) is continuous on 0, 1 and derivable on 0, 1
Thus, both the conditions of Lagrange's theorem are satisfied.
Concequently, there exist some \text{c}\in-3,4 such that
\text{f}'(\text{c})=\frac{\text{f}(1)-\text{f}(0)}{1-0}=\frac{\text{f}(1)-\text{f}(0)}{1}
Now,
\text{f}(\text{x})=\tan^{-1}\text{x}
\text{f}'(\text{x})=\frac{1}{1+\text{x}^2},\text{f}(1)=\frac{\pi}{4},\text{f}(0)=0
\therefore\ \text{f}'(\text{x})=\frac{\text{f}(1)-\text{f}(0)}{1-0}
\Rightarrow\frac{1}{1+\text{x}^2}=\frac{\pi}{4}-0
\Rightarrow49\Big(\frac{\pi}{4}-1\Big)=\text{x}^2
\Rightarrow\text{x}=\pm\sqrt{\frac{4-\pi}{\pi}}
Thus, \text{c}=\sqrt{\frac{4-\pi}{\pi}}\in(0,1) such that \text{f}'(\text{c})=\frac{\text{f}(1)-\text{f}(0)}{1-0}
Hence, Lagrange's mean value theorem is verified.
Q127. If x13 y7 = (x + y)20, prove that \frac{\text{dy}}{\text{dx}}=\frac{\text{y}}{\text{x}} 5 Marks

Ans: Here,
x13y7 = (x + y)20
Taking log on both the sides,
\log(\text{x}^{13}\text{y}^7)=\log(\text{x}+\text{y})^{20}
13\log\text{x}+7\log\text{y}=20\log(\text{x}+\text{y})
\big[\text{Since},\log(\text{AB})=\log\text{A}+\log\text{B},\log\text{a}^\text{b}=\text{b}\log\text{a}\big]
Differentiating it with respect to x using chain rule,
13\frac{\text{d}}{\text{dx}}(\log\text{x})+7\frac{\text{d}}{\text{dx}}(\log\text{y})=20\frac{\text{d}}{\text{dx}}\log(\text{x}+\text{y})
\frac{13}{\text{x}}+\frac{7}{\text{y}}\frac{\text{dy}}{\text{dx}}=\frac{20}{\text{x}+\text{y}}\frac{\text{d}}{\text{dx}}(\text{x}+\text{y})
\frac{13}{\text{x}}+\frac{7}{\text{y}}\frac{\text{dy}}{\text{dx}}=\frac{20}{(\text{x}+\text{y})}\Big[1+\frac{\text{dy}}{\text{dx}}\Big]
\frac{7}{\text{y}}\frac{\text{dy}}{\text{dx}}-\frac{20}{(\text{x}+\text{y})}=\frac{20}{(\text{x}+\text{y})}-\frac{13}{\text{x}}
\frac{\text{dy}}{\text{dx}}\Big[\frac{\text{7}}{\text{y}}-\frac{20}{(\text{x}+\text{y})}\Big]=\frac{20}{(\text{x}+\text{y})}-\frac{13}{\text{x}}
\frac{\text{dy}}{\text{dx}}\Big[\frac{2(\text{x}+\text{y})-20\text{y}}{\text{y}(\text{x}+\text{y})}\Big]=\Big[\frac{20\text{x}-13(\text{x}+\text{y})}{\text{x}(\text{x}+\text{y})}\Big]
\frac{\text{dy}}{\text{dx}}=\Big[\frac{20\text{x}-13\text{x}-13\text{y}}{\text{x}(\text{x}+\text{y})}\Big]\Big(\frac{\text{y}(\text{x}+\text{y})}{7\text{x}+7\text{y}-20\text{y}}\Big)
=\frac{\text{y}}{\text{x}}\Big(\frac{7\text{x}-13\text{y}}{7\text{x}-13\text{y}}\Big)
\frac{\text{dy}}{\text{dx}}=\frac{\text{y}}{\text{x}}
Q128. Find \frac{\text{dy}}{\text{dx}} 5 Marks
\text{y}=\frac{\text{e}^{\text{ax}}\sec\text{x}\times\log\text{x}}{\sqrt{1-2\text{x}}}

Ans: Here,
\text{y}=\frac{\text{e}^{\text{ax}}\sec\text{x}\times\log\text{x}}{\sqrt{1-2\text{x}}}\ .....(\text{i})
\Rightarrow\text{y}=\frac{\text{e}^{\text{ax}}\times\sec^\text{x}\times\log\text{x}}{(1-2\text{x})^\frac{1}{2}}
Taking log on both the sides,
\log\text{y}=\log\text{e}^{\text{ax}}+\log{\sec\text{x}}+\log\log\text{x}-\frac{1}{2}\log(1-2\text{x}) \\ \begin{bmatrix} \text{Since}, \log\Big(\frac{\text{A}}{\text{B}}\Big)=\log\text{A}-
\log\text{B},\\ \log(\text{AB})=\log\text{A}+\log\text{B} \end{bmatrix}
\log\text{y}=\text{ax}+\log{\sec\text{x}}+\log\log\text{x}-\frac{1}{2}\log(1-2\text{x}) \\ \big[\text{Since}, \log\text{a}^\text{b}=\text{b}\log\text{a and }\log_\text{e}\text{e}=1\big]
Differentiating it with respect to x using chain rule,
\frac{1}{\text{y}}\frac{\text{dy}}{\text{dx}}=\frac{\text{d}}{\text{dx}}(\text{ax})+\frac{\text{d}}{\text{dx}}(\log\sec\text{x})+\frac{\text{d}}{\text{dx}}(\log\log\text{x})-\frac{1}{2}\log(1-
2\text{x})
\frac{1}{\text{y}}\frac{\text{dy}}{\text{dx}}=\text{a}+\frac{1}{\sec\text{x}}\frac{\text{d}}{\text{dx}}(\sec\text{x})+\frac{1}{\log\text{x}}\frac{\text{d}}{\text{dx}}-\frac{1}{2}\Big(\frac{1}{1-
2\text{x}}\Big)\frac{\text{d}}{\text{dx}}(1-2\text{x})
\frac{1}{\text{y}}\frac{\text{dy}}{\text{dx}}=\text{a}+\frac{\sec\text{x}\tan\text{x}}{\sec\text{x}}+\frac{1}{(\log\text{x})}\Big(\frac{1}{\text{x}}\Big)-\frac{1}{2}\Big(\frac{1}{1-2\text{x}}\Big)
(-2)
\frac{\text{dy}}{\text{dx}}=\text{y}\Big[\text{a}+\tan\text{x}+\frac{1}{\text{x}\log\text{x}}+\frac{1}{1-2\text{x}}\Big]
\frac{\text{dy}}{\text{dx}}=\frac{\text{e}^{\text{ax}}\sec\text{x}\log\text{x}}{\sqrt{1-2\text{x}}}\Big[\text{a}+\tan\text{x}+\frac{1}{\text{x}\log\text{x}}+\frac{1}{1-2\text{x}}\Big]
[Using equation (i)]
Q129. Verify Rolle's theorem for the following function on the indicated intervals 5 Marks
f(x) = x(x - 2)2 on the interval [0, 2]

Ans: Given function is f(x) = x(x - 2)2. Which can be rewritten as f(x) = x3 - 4x2 + 4x.
We know that a polynomial function is everywhere derivable and hence continuous.
So, being a polynomial function f(x) is continuous and derivable on [0, 2].
Also,
f(0) = f(2) = 0
Thus, all the conditions of Rolle's theorem are satisfied.
Now, we have to show that there exists \text{c}\in[0,2] such that f'(c) = 0.
We have
f(x) = x3 - 4x2 + 4x
⇒ f'(x) = 3x2 - 8x + 4
When, f'(x) = 0
3x2 - 8x + 4 = 0
⇒ 3x2 - 6x - 2x + 4 = 0
⇒ 3x(x - 2) - 2(x - 2) = 0

https://bls.smartstudies.co.in/#/exam/pdf-preview/c59cb220-8e86-4716-9ff7-82aec16b1ade/1 36/158
5/26/24, 6:19 PM Exam Automation
⇒ (x - 2)(3x - 2)
\Rightarrow\text{x}=2,\frac{2}{3}
Thus, \text{c}=\frac{2}{3}\in(0,2) such that f'(c) = 0.
Hence, Rolle's theorem is verified.
Q130. If \text{x}=\Big(\text{t}+\frac{1}{\text{t}}\Big)^\text{a},\text{y}=\text{a}^{\text{t}+\frac{1}{\text{t}}}, find \frac{\text{dy}}{\text{dx}} 5 Marks

Ans: Here, \text{x}=\Big(\text{t}+\frac{1}{\text{t}}\Big)^{\text{a}}


Differentiating it with respect to t using chain rule,
\frac{\text{dx}}{\text{dt}}=\frac{\text{d}}{\text{dt}}\left[\Big(\text{t}+\frac{1}{\text{t}}\Big)^{\text{a}}\right]
=\text{a}\Big(\text{t}+\frac{1}{\text{t}}\Big)^{\text{a-1}}\frac{\text{d}}{\text{dt}}\Big(\text{t}+\frac{1}{\text{t}}\Big)
\frac{\text{dx}}{\text{dt}}=\text{a}\Big(\text{t}+\frac{1}{\text{t}}\Big)^{\text{a-1}}\Big(\text{1}-\frac{1}{\text{t}^{2}}\Big)\ .....(\text{i})
And, \text{y}=\text{a}^{(\text{t}+\frac{1}{\text{t}})}
Differentiating it with respect to t using chain rule,
\frac{\text{dy}}{\text{dt}}=\frac{\text{d}}{\text{dt}}\bigg[\text{a}^{\big(\text{t}+\frac{1}{\text{t}}\big)}\bigg]
=\frac{\text{d}}{\text{dt}}\text{a}^{\big(\text{t}+\frac{1}{\text{t}}\big)}\times\log\frac{\text{d}}{\text{dt}}\left(\text{t}+\frac{1}{\text{t}}\right)
\frac{\text{dy}}{\text{dt}}=\text{a}^{\big(\text{t}+\frac{1}{\text{t}}\big)}\times\log\text{a}\Big(\text{t}-\frac{1}{\text{t}^{2}}\Big)\ .....(\text{ii})
Dividing equation (ii) by (i),
\frac{\frac{\text{dy}}{\text{dt}}}{\frac{\text{dx}}{\text{dt}}}=\frac{\text{a}^{\big(\text{t}+\frac{1}{\text{t}}\big)}\times\log\text{a}\left(1-\frac{1}{\text{t}^{2}}\right)}
{\text{a}\big(\text{t}+\frac{1}{\text{t}}\big)^{\text{a}-1}\big(1-\frac{1}{\text{t}}^{2}\big)}
\frac{\text{dy}}{\text{dx}}=\frac{\text{a}^{\big(\text{t}+\frac{1}{\text{t}}\big)}\times\log\text{a}}{\text{a}\big(\text{t}+\frac{1}{\text{t}}\big)^{\text{a}-1}}
Q131. Find the points of discontinuity, if any of the following function: 5 Marks
\text{f(x)}=\begin{cases}\frac{\text{x}^4+\text{x}^3+2\text{x}^2}{\tan^{-1}\text{x}},&\text{if }\text{ x}\neq0\\10,&\text{if }\text{ x}=0\end{cases}

Ans: When \text{x}\neq0, we have \text{f(x)}=\frac{\text{x}^4+\text{x}^3+2\text{x}^2}{\tan^{-1}\text{x}}


We know that a polynomial is continuous for x < 0 and x > 0, Also the inverse trignometric function is continuous in its domain.
Here, X4 + x3 + 2x2 is polnomial, so is continuous for x < 0 and x > 0 and \tan^{-1}\text{x} is also continuous for x < 0 and x > 0
So, the quotient function \text{f(x)}=\frac{\text{x}^4+\text{x}^3+2\text{x}^2}{\tan^{-1}\text{x}} is continuous for each x < 0 and x > 0
Now, consider the point x = 0
\text{LHL}=\lim_\limits{\text{x}\rightarrow0^-}\text{f(x)}=\lim_\limits{\text{h}\rightarrow0}\text{f}(0-\text{h})
=\lim_\limits{\text{h}\rightarrow0}\frac{(-\text{h})^4+(-\text{h})^3+2(-\text{h})^2}{\tan^{-1}(-\text{h})}=\lim_\limits{\text{h}\rightarrow0}\frac{\text{h}^4-\text{h}^3+2\text{h}^2}
{\tan^{-1}\text{h}}=0
\text{RHL}=\lim_\limits{\text{x}\rightarrow0^+}\text{f(x)}=\lim_\limits{\text{h}\rightarrow0}\text{f}(0+\text{h})
=\lim_\limits{\text{h}\rightarrow0}\frac{\text{h}^4+\text{h}^3+2\text{h}^2}{\tan^{-1}\text{h}}=0
\text{f}(0)=10
Thus, \text{LHL}=\text{RHL}\neq\text{f}(0)
Hence, the function is not continuous at x = 0
Q132. A function f : R → R satisfies the equation f(x + y) = f(x)f(y) fot all x, y ∈ R, f(x) ≠ 0. Suppose that the function is differentiable at x = 0 and f'(0) = 2. Prove that f'(x) = 2 f(x). 5 Marks

Ans: Let f : R → R satisfies the equation \text{f(x+y})=\text{f(x).f(y)},\forall\text{ x, y}\in\text{R, f(x)} \neq0.


Let f(x) is differentiable at x = 0 and f'(0) = 2.
\Rightarrow\ \text{f}'(0)=\lim\limits_{\text{x}\rightarrow0}\frac{\text{f(x)}-\text{f}(0)}{\text{x}-0}
\Rightarrow\ 2=\lim\limits_{\text{x}\rightarrow0}\frac{\text{f(x)}-\text{f}(0)}{\text{x}}
\Rightarrow\ 2=\lim\limits_{\text{h}\rightarrow0}\frac{\text{f}(0+\text{h})-\text{f}(0)}{0+\text{h}}
\Rightarrow\ 2=\lim\limits_{\text{h}\rightarrow0}\frac{\text{f}(0).\text{f(h)}-\text{f}(0)}{\text{h}}
\Rightarrow\ 2=\lim\limits_{\text{h}\rightarrow0}\frac{\text{f}(0)[\text{f(h)-1]}}{\text{h}} [\because\text{f}(0)=\text{f(h)}]\ \dots(\text{i})
Also, \text{f}'(\text{x})=\lim\limits_{\text{h}\rightarrow0}\frac{\text{f(x}+\text{h)}-\text{f(x)}}{\text{h}}
=\lim\limits_{\text{h}\rightarrow0}\frac{\text{f(x)}.\text{f(h)}-\text{f(h)}}{\text{h}} [\because\text{f(x+y)}=\text{f(x).f(y)}]
=\lim\limits_{\text{h}\rightarrow0}\frac{\text{f(x)}[\text{f(h)}-1]}{\text{h}}=2 \text{f(x) }[\text{using Eq.(i)}]
\therefore\ \text{f}'(\text{x})=2\text{f(x)}
Q133. Extend the definition of the following by continuity \text{f(x)}=\frac{1-\cos7(\text{x}-\pi)}{5(\text{x}-\pi)^2} at the point \text{x}=\pi. 5 Marks

Ans: Given,
\text{f(x)}=\frac{1-\cos7(\text{x}-\pi)}{5(\text{x}-\pi)^2},\text{ x}=\pi
If f(x) is continuous at \text{x}=\pi, then
\lim_\limits{\text{x}\rightarrow\pi}\text{f(x)}=\text{f}(\pi)
\Rightarrow\lim_\limits{\text{x}\rightarrow\pi}\frac{1-\cos7(\text{x}-\pi)}{5(\text{x}-\pi)^2}=\text{f}(\pi)
\Rightarrow\frac{2}{5}\lim_\limits{\text{x}\rightarrow\pi}\frac{\sin^2\Big(\frac{7(\text{x}-\pi)}{2}\Big)}{(\text{x}-\pi)^2}=\text{f}(\pi)
\Rightarrow\frac{2}{5}\times\frac{49}{4}\lim_\limits{\text{x}\rightarrow\pi}\frac{\sin^2\Big(\frac{7(\text{x}-\pi)}{2}\Big)}{\frac{49}{4}(\text{x}-\pi)^2}=\text{f}(\pi)
\Rightarrow\frac{2}{5}\times\frac{49}{4}\lim_\limits{\text{x}\rightarrow\pi}\frac{\sin^2\Big(\frac{7(\text{x}-\pi)}{2}\Big)}{\Big(\frac{7}{2}(\text{x}-\pi)\Big)^2}=\text{f}(\pi)
\Rightarrow\begin{bmatrix}\frac{2}{5}\times\frac{49}{4}\lim_\limits{\text{x}\rightarrow\pi}\frac{\sin\Big(\frac{7(\text{x}-\pi)}{2}\Big)}{\Big(\frac{7}{2}(\text{x}-
\pi)\Big)}\end{bmatrix}^2=\text{f}(\pi)
\Rightarrow\frac{2}{5}\times\frac{49}{4}\times1=\text{f}(\pi)
\Rightarrow\frac{1}{5}\times\frac{49}{2}\times1=\text{f}(\pi)
\Rightarrow\frac{49}{10}=\text{f}(\pi)
Hence, the given function will be continuous at \text{x}=\pi, if \text{f}(\pi)=\frac{49}{10}
Q134. Examine the differentiability of f, where f is defined by: 5 Marks
\text{f(x)}=\begin{cases}1+\text{x},&\text{if x}\leq2\\5-\text{x},&\text{if x}>2\end{cases}
at x = 2.

Ans: We have, \text{f(x)}=\begin{cases}1+\text{x},&\text{if x}\leq2\\5-\text{x},&\text{if x}>2\end{cases} at x = 2.


Let's first check continuity at x = 2
For continuity at x = 0,
\text{L.H.L}=\lim\limits_{\text{x}\rightarrow2^-}(1+\text{x})=3
\text{R.H.L}=\lim\limits_{\text{x}\rightarrow2^+}(5-\text{x})=3
\text{f}(2)=1+2=3
So, f(x) is continuous at x = 2
For differentiability at x = 2,
\text{Lf}'(2)=\lim\limits_{\text{x}\rightarrow2^-}\frac{\text{f(x)}-\text{f}(2)}{\text{x}-2}
=\lim\limits_{\text{h}\rightarrow0}\frac{\text{f}(2-\text{h})-\text{f}(2)}{2-\text{h}-2}=\lim\limits_{\text{h}\rightarrow0}\frac{(1+2-\text{h})-3}{2-
\text{h}-2}=\lim\limits_{\text{h}\rightarrow0}\frac{-\text{h}}{-\text{h}}=1
\text{Rf}'(2)=\lim\limits_{\text{x}\rightarrow2^+}\frac{\text{f(x)}-\text{f}(2)}{\text{x}-2}
=\lim\limits_{\text{h}\rightarrow0}\frac{\text{f}(2+\text{h})-\text{f}(2)}{2+\text{h}-2}=\lim\limits_{\text{h}\rightarrow0}\frac{5-(2+\text{h})-3}
{2+\text{h}-2}=\lim\limits_{\text{h}\rightarrow0}\frac{-\text{h}}{\text{h}}=-1
Since, Lf'(2) ≠ Rf'(2)
So, f(x) is not differentiable at x = 2.
Q135. If for function \phi(\text{x})=\lambda\text{x}^2+7\text{x}-4, \phi(5)=97, find \lambda. 5 Marks

Ans: Given: \phi(\text{x})=\lambda\text{x}^2+7\text{x}-4


Clearly, being a polynomial function, is differentiable everywhere. Therefore the derivative of \phi at x is given by:

https://bls.smartstudies.co.in/#/exam/pdf-preview/c59cb220-8e86-4716-9ff7-82aec16b1ade/1 37/158
5/26/24, 6:19 PM Exam Automation
\phi'(\text{x})=\lim_\limits{\text{h}\rightarrow0}\frac{\phi(\text{x}+\text{h})-\phi(\text{x})}{\text{h}}
\Rightarrow\phi'(\text{x})=\lim_\limits{\text{h}\rightarrow0}\frac{\lambda(\text{x}+\text{h})^2+7(\text{x}+\text{h})-4-\lambda\text{x}^2-7\text{x}+4}{\text{h}}
\Rightarrow\phi'(\text{x})=\lim_\limits{\text{h}\rightarrow0}\frac{\lambda(\text{x}+\text{h})^2+7(\text{x}+\text{h})-4-\lambda\text{x}^2-7\text{x}+4}{\text{h}}
\Rightarrow\phi'(\text{x})=\lim_\limits{\text{h}\rightarrow0}\frac{\lambda\text{h}^2+2\lambda\text{xh}+7\text{h}}{\text{h}}
\Rightarrow\phi'(\text{x})=\lim_\limits{\text{h}\rightarrow\infty}\frac{\text{h}(\lambda\text{h}+2\lambda\text{x}+7)}{\text{h}}
\Rightarrow\phi'(\text{x})=2\lambda\text{x}+7
It is given \phi'(5)=97
Thus, \phi'(5)=10\lambda+7=97
\Rightarrow10\lambda+7=97
\Rightarrow10\lambda=90
\Rightarrow\lambda=9
Q136. If a relation between x and y is such that y cannot be expressed in terms of x, then y is called an implicit function of x. When a given relation expresses y as an implicit function of x and we 5 Marks
want to find \frac{\text{dy}}{\text{dx}}, then we differentiate every term of the given relation w.r.t. x, remembering that a tenn in y is first differentiated w.r.t. y and then multiplied
by \frac{\text{dy}}{\text{dx}}.
Based on the ab:ve information, find the value of \frac{\text{dy}}{\text{dx}} in each of the following questions.
1. x3 + x2y + xy2 + y3 = 81
1. \frac{(3\text{x}^2+2\text{xy}+\text{y}^2)}{\text{x}^2+2\text{xy}+3\text{y}^2}
2. \frac{-(3\text{x}^2+2\text{xy}+\text{y}^2)}{\text{x}^2+2\text{xy}+3\text{y}^2}
3. \frac{(3\text{x}^2+2\text{xy}-\text{y}^2)}{\text{x}^2-2\text{xy}+3\text{y}^2}
4. \frac{3\text{x}^2+\text{xy}+\text{y}^2}{\text{x}^2+\text{xy}+3\text{y}^2}
2. xy = ex-y
1. \frac{\text{x}-\text{y}}{(1+\log\text{x})}
2. \frac{\text{x}+\text{y}}{(1+\log\text{x})}
3. \frac{\text{x}-\text{y}}{\text{x}(1+\log\text{x})}
4. \frac{\text{x}+\text{y}}{\text{x}(1+\log\text{x})}
3. \text{e}^{\sin\text{y}}=\text{xy}
1. \frac{-\text{y}}{\text{x}(\text{y}\cos\text{y}-1)}
2. \frac{\text{y}}{\text{y}\cos\text{y}-1}
3. \frac{\text{y}}{\text{y}\cos\text{y}+1}
4. \frac{\text{y}}{\text{x}(\text{y}\cos\text{y}-1)}
4. \sin^2\text{x}+\cos^2\text{y}=1
1. \frac{\sin2\text{y}}{\sin2\text{x}}
2. -\frac{\sin2\text{x}}{\sin2\text{y}}
3. -\frac{\sin2\text{y}}{\sin2\text{x}}
4. \frac{\sin2\text{x}}{\sin2\text{y}}
5. \text{y}=(\sqrt{\text{x}})^{\sqrt{\text{x}}^\sqrt{\text{x}}...\infty}
1. \frac{-\text{y}^2}{\text{x}(2-\text{y}\log\text{x})}
2. \frac{\text{y}^2}{2+\text{y}\log\text{x}}
3. \frac{\text{y}^2}{\text{x}(2+\text{y}\log\text{x})}
4. \frac{\text{y}^2}{\text{x}(2-\text{y}\log\text{x})}

Ans: 1. (b) \frac{-(3\text{x}^2+2\text{xy}+\text{y}^2)}{\text{x}^2+2\text{xy}+3\text{y}^2}


Solution:
\text{x}^3+\text{x}^2\text{y}+\text{xy}^2+\text{y}^3=81
\Rightarrow3\text{}^2+\text{x}^2\frac{\text{dy}}{\text{dx}}+2\text{xy}+2\text{xy}\frac{\text{dy}}{\text{dx}}+\text{y}^2+3\text{y}^2\frac{\text{dy}}{\text{dx}}=0
\Rightarrow(\text{x}^2+2\text{xy}+3\text{y}^2)\frac{\text{dy}}{\text{dx}}=-3\text{x}^2-2\text{xy}-\text{y}^2
\Rightarrow\frac{\text{dy}}{\text{dx}}=\frac{-(3\text{x}^2+2\text{xy}+\text{y}^2)}{\text{x}^2+2\text{xy}+3\text{y}^2}
2. (c) \frac{\text{x}-\text{y}}{\text{x}(1+\log\text{x})}
Solution:
\text{x}^\text{y}=\text{e}^{\text{x}-\text{y}}\Rightarrow\text{y}\log\text{x}=\text{x}-\text{y}
\text{y}\times\frac{1}{\text{x}}+\log\text{x}\cdot\frac{\text{dy}}{\text{dx}}=1-\frac{\text{dy}}{\text{dx}}
\Rightarrow\frac{\text{dy}}{\text{dx}}[\log\text{x}+1]=1-\frac{\text{y}}{\text{x}}
\Rightarrow\frac{\text{dy}}{\text{dx}}=\frac{\text{x}-\text{y}}{\text{x}[1+\log\text{x}]}
3. (d) \frac{\text{y}}{\text{x}(\text{y}\cos\text{y}-1)}
Solution:
\text{e}^{\sin\text{y}}=\text{xy}\Rightarrow\sin\text{y}=\log\text{x}+\log\text{y}
\Rightarrow\cos\text{y}\frac{\text{dy}}{\text{dx}}=\frac{1}{\text{x}}+\frac{1}{\text{y}}\frac{\text{dy}}{\text{dx}}\Rightarrow\frac{\text{dy}}{\text{dx}}\Big[\cos\text{y}-\frac{1}
{\text{y}}\Big]=\frac{1}{\text{x}}
\Rightarrow\frac{\text{dy}}{\text{dx}}=\frac{\text{y}}{\text{x}(\text{y}\cos\text{y}-1)}
4. (d) \frac{\sin2\text{x}}{\sin2\text{y}}
Solution:
\sin^2\text{x}+\cos^2\text{y}=1
\Rightarrow2\sin\text{x}\cos\text{x}+2\cos\text{y}\Big(-\sin\text{y}\frac{\text{dy}}{\text{dx}}\Big)=0
\Rightarrow\frac{\text{dy}}{\text{dx}}=\frac{-\sin2\text{x}}{-\sin2\text{y}}=\frac{\sin2\text{x}}{\sin2\text{y}}
5. (d) \frac{\text{y}^2}{\text{x}(2-\text{y}\log\text{x})}
Solution:
\text{y}=(\sqrt{\text{x}})^{\sqrt{\text{x}}^\sqrt{\text{x}}...\infty}\Rightarrow\text{y}=(\sqrt{\text{x}})^\text{y}
\Rightarrow\text{y}=\text{y}(\log\sqrt{\text{x}})\Rightarrow\log\text{y}=\frac{1}{2}(\text{y}\log\text{x})
\Rightarrow\frac{1}{\text{y}}\frac{\text{dy}}{\text{dx}}=\frac{1}{2}\Big[\text{y}\times\frac{1}{\text{x}}+\log\text{x}\Big(\frac{\text{dy}}{\text{dx}}\Big)\Big]
\Rightarrow\frac{\text{dy}}{\text{dx}}\Big\{\frac{1}{\text{y}}-\frac{1}{2}\log\text{x}\Big\}=\frac{1}{2}\frac{\text{y}}{\text{x}}
\Rightarrow\frac{\text{dy}}{\text{dx}}=\frac{\text{y}}{2\text{x}}\times\frac{2\text{y}}{(2-\text{y}\log\text{x})}=\frac{\text{y}^2}{\text{x}(2-\text{y}\log\text{x})}
Q137. Find \frac{\text{dy}}{\text{dx}} 5 Marks
y = ex + 10x + xx

Ans: Here, \text{y}=\text{e}^\text{x}+10^\text{x}+\text{x}^\text{x}


\text{e}^{\text{x}}+10^{\text{x}}+\text{e}^{\log\text{x}^\text{x}}
\big[\text{Since, e}^{{\log}_\text{a}^\text{b}}=\text{a},\log\text{a}^\text{b}=\text{b}\log\text{a}\big]
\text{y}=\text{e}^{\text{x}}+10^{\text{x}}+\text{e}^{\log\text{x}^\text{x}}
Differentiating it with respect to x using product rule, chain rule,
\frac{\text{dy}}{\text{dx}}=\frac{\text{d}}{\text{dx}}(\text{e}^\text{x})+\frac{\text{d}}{\text{dx}}(10^{\text{x}})+\frac{\text{d}}{\text{dx}}\big(\text{e}^{\text{x}\log\text{x}}\big)
=\text{e}^\text{x}+10^\text{x}\log10+\text{e}^{\text{x}\log\text{x}}\frac{\text{d}}{\text{dx}}(\text{x}\log\text{x})
=\text{e}^\text{x}+10^\text{x}\log10+\text{e}^{\text{x}\log\text{x}}\Big[\text{x}\times\frac{\text{d}}{\text{dx}}(\log\text{x})+\log\text{x}\frac{\text{d}}{\text{dx}}(\text{x})\Big]
=\text{e}^\text{x}+10^\text{x}\log10+\text{e}^{\text{x}\log\text{x}}\Big[\text{x}\Big(\frac{1}{\text{x}}\Big)+\log\text{x}(1)\Big]
=\text{e}^\text{x}+10^\text{x}\log10+\text{x}^{\text{x}}[1+\log\text{x}]
=\text{e}^\text{x}+10^\text{x}\log10+\text{x}^{\text{x}}[\log\text{e}+\log\text{x}] \big[\text{Since}, \log_\text{e}\text{e}=1\big]
\frac{\text{dy}}{\text{dx}}=\text{e}^\text{x}+10^\text{x}\log10+\text{x}^\text{x}(\log\text{ex})\ \big[\text{Since}\log\text{A}+\log\text{B}=\log\text{AB}]
Q138. 5 Marks

https://bls.smartstudies.co.in/#/exam/pdf-preview/c59cb220-8e86-4716-9ff7-82aec16b1ade/1 38/158
5/26/24, 6:19 PM Exam Automation
Let C be a curve defined parametrically as \text{x}=\text{a}\cos^3\theta,\text{y}=\text{a}\sin^3\theta,0\leq\theta\leq\frac{\pi}{2}. Determine a point P on C, where the tangent to C is
parallel to the chord joining the points (a, 0) and (0, a).

Ans: As, \text{x}=\text{a}\cos^3\theta


\Rightarrow\frac{\text{dx}}{\text{d}\theta}=-3\text{a}\cos^2\theta\sin\theta
And, \text{y}=\text{a}\sin^3\theta
\Rightarrow\frac{\text{dy}}{\text{d}\theta}=3\text{a}\sin^2\theta\cos\theta
So, \frac{\text{dy}}{\text{dx}}=\frac{\Big(\frac{\text{dy}}{\text{d}\theta}\Big)}{\Big(\frac{\text{dx}}{\text{d}\theta}\Big)}=\frac{3\text{a}\sin^2\theta\cos\theta}
{-3\text{a}\cos^2\theta\sin\theta}=-\tan\theta
For the tangent to be parallel to the chord joining the points (a, 0) and (0, a).
\frac{\text{dy}}{\text{dx}}=\frac{0-\text{a}}{\text{a}-0}
\Rightarrow-\tan\theta=-1
\Rightarrow\tan\theta=1
\Rightarrow\theta=\frac{\pi}{4}
Now,
\text{x}=\text{a}\cos^3\frac{\pi}{4}=\text{a}\Big(\frac{1}{\sqrt{2}}\Big)^3=\frac{\text{a}}{2\sqrt2} and
\text{y}=\text{a}\sin^3\frac{\pi}{4}=\text{a}\Big(\frac{1}{\sqrt{2}}\Big)^3=\frac{\text{a}}{2\sqrt2}
So, the point P on the curve C is \Big(\frac{\text{a}}{2\sqrt2},\frac{\text{a}}{2\sqrt2}\Big).
Q139. A function f(x) is said to be continuous in an open interval (a, b), if it is continuous at every point in this interval. 5 Marks
A function f(x) is said to be continuous in the closed interval [a, b), if f(x) is continuous in (a, b) and \lim\limits_{\text{x}\rightarrow0}\text{f}(\text{a}+\text{h})=\text{f}
(\text{a}) and \lim\limits_{\text{x}\rightarrow0}\text{f}(\text{b}-\text{h})=\text{f}(\text{b})
If function \text{f}(\text{x})=\begin{cases}\frac{\sin(\text{a}+1)\text{x}+\sin\text{x}}{\text{x}}&,\text{x}<0\\\text{c}&,\text{x}=0\\\frac{\sqrt{\text{x}+\text{bx}^2}-\sqrt{\text{x}}}
{\text{bx}^{\frac{3}{2}}}&,\text{x}>0\end{cases} is continuous at x = 0, then answer the following questions.
1. The value of a is:
1. -\frac{3}{2}
2. 0
3. \frac{1}{2}
4. -\frac{1}{2}
2. The value of b is:
1. 1
2. -1
3. 0
4. Any real number.
3. The value of c is:
1. 1
2. \frac{1}{2}
3. -1
4. -\frac{1}{2}
4. The value of a + c is:
1. 1
2. 0
3. -1
4. -2
5. The value of c - a is:
1. 1
2. 0
3. -1
4. 2

Ans: \text{L.H.L.}(\text{at x})=\lim\limits_{\text{x}\rightarrow0}\frac{\sin(\text{a}+1)\text{x}+\sin\text{x}}{\text{x}}\Big(\frac{0}{0}\text{ form}\Big)


Using L' Hospital rule, we get
\text{L.H.L.} (\text{at x} = 0)
=\lim\limits_{\text{x}\rightarrow0}(\text{a}+1)\cos(\text{a}+1)\text{x}+\cos\text{x}=\text{a}+2
\text{R.H.L.} (\text{at x} = 0)=\lim\limits_{\text{x}\rightarrow0}\frac{\sqrt{\text{x}+\text{bx}^2}-\sqrt{\text{x}}}{\text{bx}^\frac{3}
{2}}=\lim\limits_{\text{x}\rightarrow0}\frac{\sqrt{1+\text{bx}}-1}{\text{bx}}
=\lim\limits_{\text{x}\rightarrow0}\frac{1}{\sqrt{1+\text{bx}}+1}=\frac{1}{2}
Since,f(x) is continuous at x = 0.
\therefore From (i) and (ii), we get
\text{a}+2=\text{c}=\frac{1}{2}\Rightarrow\text{a}=-\frac{3}{2},\text{c}=\frac{1}{2}
Also, value of b does not affect the continuity of f(x), so b can be any real number.
1. (a) -\frac{3}{2}
2. (d) Any real number.
3. (b) \frac{1}{2}
4. (c) -1
Solution:
\text{a}+\text{c}=-\frac{3}{2}+\frac{1}{2}=-1
5. (d) 2
Solution:
\text{c}-\text{a}=\frac{1}{2}+\frac{3}{2}=2
Q140. Find \frac{\text{dy}}{\text{dx}}, if \text{y}=\text{x}^{\tan\text{x}}+\sqrt{\frac{\text{x}^2+1}{2}}. 5 Marks

Ans: We have, \text{y}=\text{x}^{\tan\text{x}}+\sqrt{\frac{\text{x}^2+1}{2}}


Let \text{u}=\text{x}^{\tan\text{x}} and \text{v}=\sqrt{\frac{\text{x}^2+1}{2}}
\therefore\ \log\text{u}\log\text{x}^{\tan\text{x}}=\tan\text{x}\log\text{x}
Differentiating w.r.t. x, we get
\frac{1}{\text{u}}\cdot\frac{\text{du}}{\text{dx}}=\tan\text{x}\cdot\frac{1}{\text{x}}+\log\text{x}\cdot\sec^2\text{x}
\Rightarrow\ \frac{\text{du}}{\text{dx}}=\text{u}\Big[\frac{\tan\text{x}}{\text{x}}+\log\text{x}\cdot\sec^2\text{x}\Big]
=\text{x}^{\tan\text{x}}\Big[\frac{\tan\text{x}}{\text{x}}+\log\text{x}\cdot\sec^2\text{x}\Big]
Now, \text{v}=\sqrt{\frac{\text{x}^2+1}{2}}
Differentiating w.r.t. x, we get
\frac{\text{dv}}{\text{dx}}=\frac{1}{2}\Big(\frac{\text{x}^2+1}{2}\Big)^{\frac{-1}{2}}\frac{2\text{x}}{2}=\frac{\text{x}}{2}\sqrt{\frac{2}{\text{x}^2+1}}
\Rightarrow\ \frac{\text{dv}}{\text{dx}}=\frac{\text{x}}{\sqrt{2(\text{x}^2+1)}}
Now, \text{y}=\text{u}+\text{v}
\therefore\ \frac{\text{dy}}{\text{dx}}=\frac{\text{du}}{\text{dx}}+\frac{\text{dv}}{\text{dx}} =\text{x}^{\tan\text{x}}\Big[\frac{\tan\text{x}}
{\text{x}}+\log\text{x}\cdot\sec^2\text{x}\Big]+\frac{\text{x}}{\sqrt{2(\text{x}^2+1)}}
Q141. If \text{y}=\text{x}\sin(\text{a}+\text{y}), prove that \frac{\text{dx}}{\text{dx}}=\frac{\sin^2(\text{a}+\text{y})}{\sin(\text{a}+\text{y})-\text{y}\cos(\text{a}+\text{y})} 5 Marks

Ans: We have, \text{y}=\text{x}\sin(\text{a}+\text{y})

https://bls.smartstudies.co.in/#/exam/pdf-preview/c59cb220-8e86-4716-9ff7-82aec16b1ade/1 39/158
5/26/24, 6:19 PM Exam Automation
Differentiate with respect to y,
\frac{\text{dy}}{\text{dx}}=\frac{\text{d}}{\text{dx}}\big[\text{x}\sin(\text{a}+\text{y})\big]
\Rightarrow\frac{\text{dy}}{\text{dx}}=\text{x}\frac{\text{d}}{\text{dx}}\{\sin(\text{a}+\text{y})\}+\sin(\text{a}+\text{y})\frac{\text{d}}{\text{dx}}(\text{x})
[Using product rule and chain rule]
\Rightarrow\frac{\text{dy}}{\text{dx}}=\text{x}\cos(\text{a}+\text{y})\frac{\text{d}}{\text{dx}}+\sin(\text{a}+\text{y})(1)
\Rightarrow\frac{\text{dy}}{\text{dx}}\{1-\text{x}\cos(\text{a}+\text{y})\}=\sin(\text{a}+\text{y})
\Rightarrow\frac{\text{dy}}{\text{dx}}=\frac{\sin(\text{a}+\text{y})}{1-\text{x}\cos(\text{a}+\text{y})}
\Rightarrow\frac{\text{dy}}{\text{dx}}=\frac{\sin(\text{a}+\text{y})}{1-\frac{\text{y}}{\sin(\text{a}+\text{y})}\cos(\text{a}+\text{y})}
\Rightarrow\frac{\text{dy}}{\text{dx}}=\frac{\sin^2(\text{a}+\text{y})}{\sin(\text{a}+\text{y}){-\text{y}\cos(\text{a}+\text{y})}}
Hence, proved.
Q142. If \text{x}=\text{a}\sin2\text{t}(1+\cos 2\text{t}) and \text{y}=\text{b}\cos\text{t}(1-\cos2\text{t}), show that at \text{t}=\frac{\pi}{4},\frac{\text{dy}}{\text{dx}}=\frac{\text{b}}{\text{a}}\text{ 5 Marks
t}=\frac{\pi}{4},\frac{\text{dy}}{\text{dx}}=\frac{\text{b}}{\text{a}}

Ans: Consider the given functions,


\text{x}=\text{a}\sin(2\text{t})(1+\cos2\text{t})\text{ and y}=\text{b}\cos2\text{t}(1-\cos2\text{t})
Rewriting the above function, we have,
\text{x}=\text{a}\sin2\text{t}+\frac{\text{a}}{2}\sin4\text{t}
Differentiating the above function w.r.t. 't', we have,
\frac{\text{dx}}{\text{dx}}=2\text{a}\cos2\text{t}+2\text{a}\cos4\text{t}\ .....(\text{i})
\text{y}=\text{b}\cos2\text{t}(1-\cos2\text{t})
\text{y}=\text{b}\cos2\text{t}-\text{b}\cos^22\text{t}
\frac{\text{dy}}{\text{dt}}=-2\text{b}\sin2\text{t}+2\text{b}\cos2\text{t}\sin2\text{t} \\ =-2\text{b}\sin2\text{t}+\text{b}\sin4\text{t}\ .....(\text{ii})
From (1) and (2),
\frac{\text{dy}}{\text{dx}}=\frac{\frac{\text{dy}}{\text{dt}}}{\frac{\text{dx}}{\text{dt}}}=\frac{-2\text{b}\sin2\text{t}+\text{b}\sin4\text{t}}{2\text{a}\cos2\text{t}+2\text{a}\cos4\text{t}}
\therefore\frac{\text{dy}}{\text{dx}}\Big|_{\frac{\pi}{4}}=\frac{\frac{\text{dy}}{\text{dt}}}{\frac{\text{dx}}{\text{dt}}}\Bigg|_{\text{t}=\frac{\pi}{4}}=\frac{-2\text{b}}
{-2\text{a}}=\frac{\text{b}}{\text{a}}
Q143. If xx + yx = 1, prove that \frac{\text{dy}}{\text{dx}}=-\frac{\text{y}(\text{y}+\text{x}\log\text{y})}{\text{x}(\text{y}\log\text{x}+\text{x})} 5 Marks

Ans: Here,
xx + y x = 1
Taking on bith sides,
\log(\text{x}^\text{y}\times\text{y}^\text{x})=\log(1)
\text{y}=\log\text{x}+\text{x}\log\text{y}=\log1
\big[\text{Since}, \log(\text{AB})=\log\text{A}+\log\text{B},\log\text{a}^\text{b}=\text{b}\log\text{a}\big]
Differentiating it with respect to x using product rule,
\frac{\text{d}}{\text{dx}}(\text{y}\log\text{x})+\frac{\text{d}}{\text{dx}}(\text{x}\log\text{y})=\frac{\text{d}}{\text{dx}}(\log1)
\Big[\text{y}\frac{\text{d}}{\text{dx}}(\log\text{x})+\log\text{x}\frac{\text{dy}}{\text{dx}}\Big]+\Big[\text{x}\frac{\text{d}}{\text{dx}}(\log\text{y})+\log\text{y}\frac{\text{d}}
{\text{dx}}\text{(x)}\Big]=0
\Big[\text{y}\Big(\frac{1}{\text{x}}\Big)\log\text{x}\frac{\text{dy}}{\text{dx}}\Big]+\Big[\text{x}\Big(\frac{1}{\text{y}}\frac{\text{dy}}{\text{dx}}\Big)+\log\text{y}(1)\Big]=0
\frac{\text{y}}{\text{x}}+\log\text{x}\log\frac{\text{dy}}{\text{dx}}+\frac{\text{x}}{\text{y}}\frac{\text{dy}}{\text{dx}}+\log\text{y}=0
\frac{\text{dy}}{\text{dx}}\Big(\log\text{x}+\frac{\text{x}}{\text{y}}\Big)=-\Big[\log\text{y}+\frac{\text{y}}{\text{x}}\Big]
\frac{\text{dy}}{\text{dx}}\Big[\frac{\text{y}\log\text{x}+\text{x}}{\text{y}}\Big]=-\Big[\frac{\text{x}\log\text{y}+\text{y}}{\text{x}}\Big]
\frac{\text{dy}}{\text{dx}}=-\frac{\text{y}}{\text{x}}\Big[\frac{\text{x}\log\text{y}+\text{y}}{\text{y}\log\text{x}+\text{x}}\Big]
Q144. Differentiate the following functions with respect to x: 5 Marks
\log(3\text{x}+2)-\text{x}^2\log(2\text{x}-1)

Ans: Let \text{y}=\log(3\text{x}+2)-\text{x}^2\log(2\text{x}-1)


Differentiate it with respect to x,
\frac{\text{dy}}{\text{dx}}=\frac{\text{d}}{\text{dx}}\big[\log(3\text{x}+2)-\text{x}^2\log(2\text{x}-1)\big]
\frac{\text{d}}{\text{dx}}\log(3\text{x}+2)-\frac{\text{d}}{\text{dx}}\big(\text{x}^2\log(2\text{x}-1)\big)
=\frac{1}{3\text{x}+2}\frac{\text{d}}{\text{dx}}(3\text{x}+2)-\Big[\text{x}^2\frac{\text{d}}{\text{dx}}\log(2\text{x}-1)+\log(2\text{x}-1)\frac{\text{d}}{\text{dx}}\big(\text{x}^2\big)\Big]
[Using product rule and chain rule]
=\frac{3}{3\text{x}+2}\Big[\text{x}^2\times\frac{1}{2\text{x}-1}\frac{\text{d}}{\text{dx}}(2\text{x}-1)+\log(2\text{x}-1)\times2\text{x}\Big]
=\frac{3}{3\text{x}+2}-\frac{2\text{x}^2}{2\text{x}-1}-2\text{x}\log(2\text{x}-1)
So,
\frac{\text{d}}{\text{dx}}\big(\log(3\text{x}+2)-\text{x}^2\log(2\text{x}-1)\big) \\ =\frac{3}{3\text{x}+2}-\frac{2\text{x}^2}{2\text{x}-1}-2\text{x}\log(2\text{x}-1)
Q145. Show that the function f defined as follows, 5 Marks
\text{f(x)}=\begin{cases}3\text{x}-2, & 0<\text{x}\leq1\\2\text{x}^2-\text{x,} & 1<\text{x}\leq2\\5\text{x}-4,&\text{x}>2\end{cases}
is countinuous at x = 2, but not differentiable there at x = 2.

Ans: Given:
\text{f(x)}=\begin{cases}3\text{x}-2, & 0<\text{x}\leq1\\2\text{x}^2-\text{x,} & 1<\text{x}\leq2\\5\text{x}-4,&\text{x}>2\end{cases}
First, we will show that f(x) is continuos at x = 2.
We have,
(LHL at x = 2)
=\lim_\limits{\text{x}\rightarrow2^{-}}\text{f(x)}
=\lim_\limits{\text{x}\rightarrow0}\text{f}(2-\text{h)}
=\lim_\limits{\text{x}\rightarrow0}2(2-\text{h)}^2-(2-\text{h})
=\lim_\limits{\text{x}\rightarrow0}(8+2\text{h}^2-8\text{h}-2+\text{h})
=6
(RHL at x = 2)
=\lim_\limits{\text{x}\rightarrow2^{+}}\text{f(x)}
=\lim_\limits{\text{x}\rightarrow0}\text{f}(2+\text{h)}
=\lim_\limits{\text{x}\rightarrow0}5(2+\text{h)}-4
=\lim_\limits{\text{x}\rightarrow0}(10+5\text{h}-4)
=6
and \text{f}(2)=2\times4-2=6
Thus, =\lim_\limits{\text{x}\rightarrow2^{-}}\text{f(x)}=\lim_\limits{\text{x}\rightarrow2^{+}}\text{f(x)}=\text{f}(2)
Hence the function is continuous at x = 2.
Now, we will check whether the given function is differerentiable at x = 2.
We have,
(LHL at x = 2)
\lim_\limits{\text{x}\rightarrow2^{-}}\frac{\text{f(x)}-\text{f}(2)}{\text{x}-2}
=\lim_\limits{\text{x}\rightarrow0}\frac{\text{f}(2-\text{h})-\text{f}(2)}{-\text{h}}
=\lim_\limits{\text{x}\rightarrow0}\frac{2\text{h}^2-7\text{h}+6-6}{-\text{h}}
=\lim_\limits{\text{x}\rightarrow0}-2\text{h}+7
=7
(RHL at x = 2)

https://bls.smartstudies.co.in/#/exam/pdf-preview/c59cb220-8e86-4716-9ff7-82aec16b1ade/1 40/158
5/26/24, 6:19 PM Exam Automation
\lim_\limits{\text{x}\rightarrow2^{+}}\frac{\text{f(x)}-\text{f}(2)}{\text{x}-2}
=\lim_\limits{\text{x}\rightarrow0}\frac{\text{f}(2+\text{h})-\text{f}(2)}{\text{h}}
=\lim_\limits{\text{x}\rightarrow0}\frac{10+5\text{h}-4-6}{\text{h}}
=5
Thus, LHL at x = 2 \neq RHL at x = 2.
Hence, function is no differentiable at x = 2.
Q146. Show that the function \text{f(x)}\begin{cases}\text{x}^\text{m}\sin(\frac{1}{\text{x}}), &\text{x}\neq0 \\0 ,& \text{x}=0\end{cases} 5 Marks
Neirher continuous but not diffierentiable, if \text{m}\leq0

Ans: \text{LHL }=\lim_\limits{\text{x}\rightarrow0^{-}}\text{f(x)}


=\lim_\limits{\text{h}\rightarrow0}\text{f}(0-\text{h})
=\lim_\limits{\text{h}\rightarrow0}(-\text{h})^\text{m}\sin\Big(-\frac{1}{\text{h}}\Big)
= Not defined as \text{m}\leq0
\text{RHL }=\lim_\limits{\text{x}\rightarrow0^{+}}\text{f(x)}
=\lim_\limits{\text{h}\rightarrow0}\text{f}(0+\text{h})
=\lim_\limits{\text{h}\rightarrow0}(+\text{h})^\text{m}\sin\Big(\frac{1}{0+\text{h}}\Big)
=Not defined, as \text{m}\leq0
Since RHL and LHL are not difined, so f(x) is not continuous
Let x = 0 for \text{m}\leq0.
(LHL at x = 0) =\lim_\limits{\text{x}\rightarrow0^{-1}}\frac{\text{f(x)}-\text{f}(0)}{\text{x}-0}
=\lim_\limits{\text{h}\rightarrow0}\frac{\text{f}(0-\text{h})-(0)}{0-\text{h}-0}
=\lim_\limits{\text{h}\rightarrow0}\frac{(-\text{h})^\text{m}\sin\Big(-\frac{1}{\text{h}}\Big)}{-\text{h}}
=\lim_\limits{\text{h}\rightarrow0}(-\text{h})^\text{m-1}\sin\Big(-\frac{1}{\text{h}}\Big)
= Not definded, as \text{m}\leq0
(RHL at x = 0) =\lim_\limits{\text{x}\rightarrow0^{+}}\frac{\text{f(x)}-\text{f}(0)}{\text{x}-0}
=\lim_\limits{\text{h}\rightarrow0}\frac{\text{f}(0+\text{h})-\text{f}(0)}{0+\text{h}-0}
=\lim_\limits{\text{h}\rightarrow0}\frac{\text{h}^\text{m}\sin\Big(\frac{1}{\text{h}}\Big)}{\text{h}}
=\lim_\limits{\text{h}\rightarrow0}(\text{h})^{\text{m}^{-1}}\sin\Big(\frac{1}{\text{h}}\Big)
= Not defined \text{m}\leq0
Thus,
f(x) is neither continuous not differentiable at x = 0 for \text{m}\leq0.
Q147. If yx = ey-x, Prove that \frac{\text{dy}}{\text{dx}}=\frac{(1+\log\text{y})^2}{\log\text{y}} 5 Marks

Ans: Here,
yx = ey-x
Taking log on both the sides,
\log\text{y}=\log\text{e}^{(\text{y}-\text{x})}
\big[\text{Since},\log\text{a}^{\text{b}}=\text{b}\log\text{a and}\log_\text{e}\text{e}=1\big]
\text{x}\log\text{y}=(\text{y}-\text{x})\log\text{e}
\text{x}\log\text{y}=\text{y}-\text{x}\ .....(\text{i})
Differentiating it with respect to x using product rule,
\frac{\text{d}}{\text{dx}}(\text{x}\log\text{y})=\frac{\text{d}}{\text{dx}}(\text{y}-\text{x})
\Big[\text{x}\frac{\text{d}}{\text{dx}}(\log\text{y})+\log\text{y}\frac{\text{d}}{\text{dx}}(\text{x})\Big]=\frac{\text{dy}}{\text{dx}}-1
\text{x}\Big(\frac{\text{x}}{\text{y}}\Big)\frac{\text{dy}}{\text{dx}}+\log\text{y}(1)=\frac{\text{dy}}{\text{dx}}-1
\frac{\text{dx}}{\text{dx}}\Big(\frac{\text{x}}{\text{y}}-1\Big)=-1-\log\text{y}
\frac{\text{dy}}{\text{dx}}\Big(\frac{\text{y}}{(1+\log\text{y})\text{y}}\Big)=-(1+\log\text{y})
\Big[\text{Since, from equation (i), x}=\frac{\text{y}}{(1+\log\text{y})}\Big]
\frac{\text{dy}}{\text{dx}}\Big[\frac{1-1-\log\text{y}}{(1+\log\text{y})}\Big]=-(1+\log\text{y})
\frac{\text{dy}}{\text{dx}}=-\frac{(1+\log\text{y})^2}{-\log\text{y}}
\frac{\text{dy}}{\text{dx}}=\frac{(1+\log\text{y})^2}{\log\text{y}}
Q148. If \text{y}\sqrt{\text{x}^2+1}=\log\Big(\sqrt{\text{x}^2+1}-\text{x}\Big), prove that \big(\text{x}^2+1\big)\frac{\text{dx}}{\text{dx}}+\text{xy}+1=0 5 Marks

Ans: We have, \text{y}\sqrt{\text{x}^2+1}=\log\Big(\sqrt{\text{x}^2+1}-\text{x}\Big)


Differentiating with respect to x, we get,
\Rightarrow\frac{\text{d}}{\text{dx}}\Big(\text{y}\sqrt{\text{x}^2+1}\Big)=\frac{\text{d}}{\text{dx}}\log\Big(\sqrt{\text{x}^2+1}-\text{x}\Big)
[Using Product rule and chain rule]
\Rightarrow\text{y}\frac{\text{d}}{\text{dx}}\big(\sqrt{\text{x}^2+1}\big)+\sqrt{\text{x}^2+1}\frac{\text{dy}}{\text{dx}} \\ =\frac{1}{\big(\sqrt{\text{x}^2+1}-
\text{x}\big)}\times\frac{\text{d}}{\text{dx}}\big(\sqrt{\text{x}^2+1}-\text{x}\big)
\Rightarrow\frac{\text{y}}{2\sqrt{\text{x}^2+1}}\times\frac{\text{d}}{\text{dx}}(\text{x}^2+1)+\sqrt{\text{x}^2+1}\frac{\text{d}}{\text{dx}} \\ =\frac{1}{\big(\sqrt{\text{x}^2+1}-
\text{x}\big)}\times\Big[\frac{1}{2\sqrt{\text{x}^2+1}}\frac{\text{d}}{\text{dx}}(\text{x}^2+1)-1\Big]
\Rightarrow\ \frac{2\text{xy}}{2\sqrt{\text{x}^2+1}}+\sqrt{\text{x}^2+1}\frac{\text{dy}}{\text{dx}} \\ =\frac{1}{\big(\sqrt{\text{x}^2+1}-\text{x}\big)}\Big[\frac{2\text{x}}
{2\sqrt{\text{x}^2+1}}-1\Big]
\Rightarrow\sqrt{\text{x}^2+1}\frac{\text{dy}}{\text{dx}} \\ =\Big[\frac{1}{\sqrt{\text{x}^2+1}-\text{x}}\Big]\Big[\frac{\text{x}-\sqrt{\text{x}^2+1}}{\sqrt{\text{x}^2+1}}\Big]-\frac{\text{xy}}
{\sqrt{\text{x}^2+1}}
\Rightarrow\sqrt{\text{x}^2+1}\frac{\text{dy}}{\text{dx}}=\frac{-(1+\text{xy})}{\sqrt{\text{x}^2+1}}
\Rightarrow\big(\text{x}^2+1\big)\frac{\text{dy}}{\text{dx}}=-(1+\text{xy})
\Rightarrow(\text{x}^2+1)\frac{\text{dy}}{\text{dx}}+1+\text{xy}=0
Q149. Discuss the continuity of the following functions at the indicated point: 5 Marks
\text{f}\text{(x)}=\begin{cases}|\text{x}-\text{a|}\sin(\frac{1}{\text{x}-\text{a}}), &\text{for} \text{ x} \neq\text{a}\\0,&\text{for} \text{ x} = \text{a}\end{cases}\text{ at x}=0

Ans: \text{f}\text{(x)}=\begin{cases}|\text{x}-\text{a|}\sin\Big(\frac{1}{\text{x}-\text{a}}\Big),&\text{for }\text{x }\neq \text{a}\\0, &\text{for x } = \text{a}\end{cases}


(\text{LHL at x}=\text{a})\lim\limits_{\text{x} \rightarrow \text{a}^-}\text{f}\text{(x)}=\lim\limits_{\text{h} \rightarrow \text{0}}\text{f}\text{(a}-\text{h)}
=\lim\limits_{\text{h} \rightarrow0}\text{|a}-\text{h}-\text{a|}\sin\Big(\frac{1}{\text{a}-\text{h}-\text{a}}\Big)=\lim\limits_{\text{h} \rightarrow0}\text{h}\sin\Big(\frac{1}{-\text{h}}\Big)
= 0 × (an oscillating number between -1 and 1)
=0
(\text{RHL at x}=\text{a})\lim\limits_{\text{x} \rightarrow \text{a}^+}\text{f}\text{(x)}=\lim\limits_{\text{h} \rightarrow \text{0}}\text{f}\text{(a}+\text{h)}
=\lim\limits_{\text{h} \rightarrow0}\text{|a}+\text{h}-\text{a|}\sin\Big(\frac{1}{\text{a}+\text{h}-\text{a}}\Big)=\lim\limits_{\text{h} \rightarrow0}\text{h}\sin\Big(\frac{1}{\text{h}}\Big)
= 0 × (an oscillating number between -1 and 1)
Thus, we obtian \lim\limits_{\text{x} \rightarrow\text{a}^-}\text{f}\text{(x)}=\lim\limits_{\text{x} \rightarrow\text{a}^+}\text{f}\text{(x)}=\text{f}\text{(a)}
\therefore f(x) is continuous at x = a.
Q150. Differentiate the following functions with respect to x: 5 Marks
\text{e}^\text{x}\log\sin2\text{x}

Ans: Let \text{y}=\text{e}^\text{x}\log\sin2\text{x}


Differentiate it with respect to x,
\frac{\text{dy}}{\text{dx}}=\frac{\text{d}}{\text{dx}}\big[\text{e}^\text{x}\log\sin2\text{x}\big]
=\text{e}^\text{x}\frac{\text{d}}{\text{dx}}\log\sin2\text{x}+\log\sin2\text{x}\frac{\text{d}}{\text{dx}}\big(\text{e}^\text{x}\big)
[Using product rule and chain rule]

https://bls.smartstudies.co.in/#/exam/pdf-preview/c59cb220-8e86-4716-9ff7-82aec16b1ade/1 41/158
5/26/24, 6:19 PM Exam Automation
=\text{e}^\text{x}\frac{1}{\sin2\text{x}}\frac{\text{d}}{\text{dx}}(\sin2\text{x})+\log\sin2\text{x}\big(\text{e}^\text{x}\big)
=\frac{\text{e}^\text{x}}{\sin2\text{x}}\cos2\text{x}\frac{\text{d}}{\text{dx}}(2\text{x})+\text{e}^\text{x}\log\sin2\text{x}
=\frac{2\cos2\text{xe}^\text{x}}{\sin2\text{x}}+\text{e}^\text{x}\log\sin2\text{x}
=\text{e}^\text{x}(2\cot2\text{x}+\log\sin2\text{x})
So,
\frac{\text{d}}{\text{dx}}\big(\text{e}^\text{x}\log\sin2\text{x}\big)=\text{e}^\text{x}(2\cot2\text{x}+\log\sin2\text{x})
Q151. Let f : A → B and g : B → C be two functions defined on non-empty sets A, B, C, then gof : A → C be is called the composition of f and g defined as, \text{gof}(\text{x})=\text{g}\ 5 Marks
{\text{f(x)}\}\forall\text{ x }\epsilon\text{ A}.
Consider the functions \text{f}(\text{x})=\begin{cases}\sin\text{x},&\text{x}\geq0\\1-\cos\text{x},&\text{x}\leq0\end{cases},\text{g}(\text{x})=\text{e}^\text{x} and then answer the
following questions.
1. The function gof(x) is defined as:
1. \text{gof}(\text{x})=\begin{cases}\text{e}^\text{x}&,\text{x}\geq0\\1-\text{e}^{\cos\text{x}}&,\text{x}\leq0\end{cases}
2. \text{gof}(\text{x})=\begin{cases}\text{e}^{\sin\text{x}}&,\text{x}\leq0\\\text{e}^{1-\cos\text{x}}&,\text{x}\geq0\end{cases}
3. \text{gof}(\text{x})=\begin{cases}\text{e}^{\sin\text{x}}&,\text{x}\leq0\\1-\text{e}^{\cos\text{x}}&,\text{x}\geq0\end{cases}
4. \text{gof}(\text{x})=\begin{cases}\text{e}^{\sin\text{x}}&,\text{x}\geq0\\\text{e}^{1-\cos\text{x}}&,\text{x}\leq0\end{cases}
2. \frac{\text{d}}{\text{dx}}\{\text{gof}(\text{x})\}=
1. [\text{gof}(\text{x})]'=\begin{cases}\cos\text{x}\cdot\text{e}^{\sin\text{x}}&,\text{x}\geq0\\\text{e}^{1-\cos\text{x}}\cdot\sin\text{x}&,\text{x}\leq0\end{cases}
2. [\text{gof}(\text{x})]'=\begin{cases}\cos\text{x}\cdot\text{e}^{\sin\text{x}}&,\text{x}\geq0\\-\sin\text{x}\cdot\text{e}^{1-\cos\text{x}}&,\text{x}\leq0\end{cases}
3. [\text{gof}(\text{x})]'=\begin{cases}\cos\text{x}\cdot\text{e}^{\sin\text{x}}&,\text{x}\geq0\\\sin\text{x}\cdot({1-\cos\text{x}})&,\text{x}\leq0\end{cases}
4. [\text{gof}(\text{x})]'=\begin{cases}\cos\text{x}\cdot\text{e}^{\sin\text{x}}&,\text{x}\geq0\\(1-{\sin\text{x}})\cdot\text{e}^{1-\cos\text{x}}&,\text{x}\leq0\end{cases}
3. R.H.D. of gof(x) at x = 0 is:
1. 0
2. 1
3. -1
4. 2
4. L.H.D. of gof(x) at x = 0 is:
1. 0
2. 1
3. -1
4. 2
5. The value of f'(x) at \text{x}=\frac{\pi}{4} is:
1. \frac{1}{9}
2. \frac{1}{\sqrt2}
3. \frac{1}{2}
4. Not defined.

Ans: 1. (d) \text{gof}(\text{x})=\begin{cases}\text{e}^{\sin\text{x}}&,\text{x}\geq0\\\text{e}^{1-\cos\text{x}}&,\text{x}\leq0\end{cases}


2. (a) [\text{gof}(\text{x})]'=\begin{cases}\cos\text{x}\cdot\text{e}^{\sin\text{x}}&,\text{x}\geq0\\\text{e}^{1-\cos\text{x}}\cdot\sin\text{x}&,\text{x}\leq0\end{cases}
3. (b) 1
4. (a) 0
5. (b) \frac{1}{\sqrt2}
Q152. Differentiate the following functions with respect to x: 5 Marks
(\cos\text{x})^\text{x}+(\sin\text{x})^\frac{1}{\text{x}}

Ans: Let \text{y}=(\cos\text{x})^\text{x}+(\sin\text{x})^\frac{1}{\text{x}}


\Rightarrow\text{y}=\text{e}^{\log(\cos\text{x})^\text{x}}+\text{e}^{\log(\sin\text{x})^\frac{1}{\text{x}}}
\Rightarrow\text{y}=\text{e}^{\text{x}\log(\cos\text{x})}+\text{e}^{\frac{1}{\text{x}}\log\sin\text{x}}
Differentiating with respect to x,
\frac{\text{dy}}{\text{dx}}=\frac{\text{d}}{\text{dx}}\big(\text{e}^{\text{x}\log\cos\text{x}}\big)+\frac{\text{d}}{\text{dx}}\big(\text{e}^{\frac{1}{\text{x}}\log\sin\text{x}}\big)
=\text{e}^{\log\cos\text{x}}\times\frac{\text{d}}{\text{dx}}(\text{x}\log\cos\text{x})+\text{e}^{\frac{1}{\text{x}}\log\sin}\frac{\text{d}}{\text{dx}}\big(\frac{1}{\text{x}}\log\sin\text{x}\big)
=\text{e}^{\log(\cos\text{x})^\text{x}}\times\Big[\text{x}\frac{\text{d}}{\text{dx}}(\log\cos\text{x})+\log\cos\text{x}\times\frac{\text{d}}{\text{dx}}(\text{x})\Big] \\
+\text{e}^{\log(\sin\text{x})^\frac{1}{\text{x}}}\times\Big[\frac{1}{\text{x}}\frac{\text{d}}{\text{dx}}(\log\sin\text{x})+\log\sin\text{x}\frac{\text{d}}{\text{dx}}\big(\frac{1}{\text{x}}\big)\Big]
=(\cos\text{x})^\text{x}\Big[\text{x}\big(\frac{1}{\cos\text{x}}\big)\frac{\text{d}}{\text{dx}}(\cos\text{x})+\log\cos\text{x}+\log\cos\text{x}(1)\Big] \\ +(\sin)^\frac{1}{\text{x}}\Big[\frac{1}
{\text{x}}\times\frac{1}{\sin\text{x}}\times\frac{\text{d}}{\text{dx}}(\sin\text{x})+\log\sin\text{x}\Big(-\frac{1}{\text{x}^2}\Big)\Big]
=(\cos\text{x})^\text{x}\Big[\text{x}\Big(\frac{1}{\cos\text{x}}\Big)(-\sin\text{x})+\log\cos\text{x}\Big] \\ +(\sin\text{x})^\frac{1}{\text{x}}\Big[\frac{1}{\text{x}}\times\frac{1}{\sin\text{x}}
(\cos\text{x})-\frac{1}{\text{x}^2}\log\sin\text{x}\Big]
=(\cos\text{x})^\text{x}\big[\log\cos\text{x}-\text{x}\tan\text{x}\big](\sin\text{x})^\frac{1}{\text{x}} \\ \Big[\frac{\cot\text{x}}{\text{x}}-\frac{1}{\text{x}^2}\log\sin\text{x}\Big]
Q153. Find \text{y}=\text{Ae}^{-\text{kt}}\cos\text({pt}+\text{c})prove that \frac{\text{d}^2\text{y}}{\text{dt}^2}+2\text{k}\frac{\text{dy}} 5 Marks
{\text{dt}}+\text{n}^2\text{y}=0,Where \text{n}^2=\text{p}^2+\text{k}^2.

Ans: We have,
\text{y}=\text{Ae}^{-\text{kt}}\cos\text({pt}+\text{c})...(1)
Differentiating y with respect to t, we get
\frac{\text{dy}}{\text{dt}}=-\text{KAe}^{-\text{kt}}\cos(\text{pt}+\text{c})-\text{PAe}^{-\text{kt}}\sin(\text{pt}+\text{c})
=-\text{ky}-\text{PAe}^{-\text{kt}}\sin(\text{pt}+\text{c})\ [\text{from}(1)]
\Rightarrow\text{pAe}^{-\text{kt}}\sin(\text{pt}+\text{c})=-\text{ky}-\frac{\text{dy}}{\text{dt}}...(2)
Differentiating \frac{\text{dy}}{\text{dt}} with respect to t, we get
\frac{\text{d}^2\text{y}}{\text{dt}}=-\text{k}\frac{\text{dy}}{\text{dt}}+\text{pkAe}^{-\text{kt}}\sin(\text{pt}+\text{c})-\text{p}^2\text{Ae}^{-\text{kt}}\cos(\text{pt}+\text{c})
=-\text{k}\frac{\text{dy}}{\text{dt}}+\text{k}\Big(-\text{ky}-\frac{\text{dy}}{\text{dt}}\Big)-\text{p}^2\text{y}\ [\text{from}(1)\ \text{and}\ (2)]
=-\text{k}\frac{\text{dy}}{\text{dt}}-\text{k}^2\text{y}-\text{k}\frac{\text{dy}}{\text{dt}}-\text{p}^2\text{y}
=-2\text{k}\frac{\text{dy}}{\text{dt}}-(\text{k}^2+\text{p}^2)\text{y}
\Rightarrow\frac{\text{d}^2\text{y}}{\text{dt}^2}+2\text{k}\frac{\text{dy}}{\text{dt}}+(\text{k}^2+\text{p}^2)\text{y}=0
\Rightarrow\frac{\text{d}^2\text{y}}{\text{dt}^2}+2\text{k}\frac{\text{dy}}{\text{dt}} \text{n}^2\text{y}=0, where \text{n}^2=\text{p}^2+\text{k}^2.
Hence,
\Rightarrow\frac{\text{d}^2\text{y}}{\text{dt}^2}+2\text{k}\frac{\text{dy}}{\text{dt}} \text{n}^2\text{y}=0, where \text{n}^2=\text{p}^2+\text{k}^2.
Q154. If y = f(u) is a differentiable function of u and u = g(x) is a differentiable function of x, then y = f(g(x)] is a differentiable function of x and \frac{\text{dy}}{\text{dx}}=\frac{\text{dy}} 5 Marks
{\text{du}}\times\frac{\text{du}}{\text{dx}}. This rule is also known as CHAIN RULE.
Based on the above information, find the derivative of functions w.r.t. x in the following questions.
1. \cos\sqrt{\text{x}}
1. \frac{-\sin\sqrt{\text{x}}}{2\sqrt{\text{x}}}
2. \frac{\sin\sqrt{\text{x}}}{2\sqrt{\text{x}}}
3. \sin\sqrt{\text{x}}
4. -\sin\sqrt{\text{x}}
2. 7^{\text{x}+\frac{1}{\text{x}}}
1. \Big(\frac{\text{x}^2-1}{\text{x}^2}\Big)\cdot7^{\text{x}+\frac{1}{\text{x}}}\cdot\log7
2. \Big(\frac{\text{x}^2+1}{\text{x}^2}\Big)\cdot7^{\text{x}+\frac{1}{\text{x}}}\cdot\log7
3. \Big(\frac{\text{x}^2-1}{\text{x}^2}\Big)\cdot7^{\text{x}-\frac{1}{\text{x}}}\cdot\log7

https://bls.smartstudies.co.in/#/exam/pdf-preview/c59cb220-8e86-4716-9ff7-82aec16b1ade/1 42/158
5/26/24, 6:19 PM Exam Automation
4. \Big(\frac{\text{x}^2+1}{\text{x}^2}\Big)\cdot7^{\text{x}-\frac{1}{\text{x}}}\cdot\log7
3. \sqrt\frac{{1-\cos\text{x}}}{1+\cos\text{x}}
1. \frac{1}{2}\sec^2\frac{\text{x}}{2}
2. -\frac{1}{2}\sec^2\frac{\text{x}}{2}
3. \sec^2\frac{\text{x}}{2}
4. -\sec^2\frac{\text{x}}{2}
4. \frac{1}{\text{b}}\tan^{-1}\Big(\frac{\text{x}}{\text{b}}\Big)+\frac{1}{\text{a}}\tan^{-1}\Big(\frac{\text{x}}{\text{a}}\Big)
1. \frac{-1}{\text{x}^2+\text{b}^2}+\frac{1}{\text{x}^2+\text{a}^2}
2. \frac{1}{\text{x}^2+\text{b}^2}+\frac{1}{\text{x}^2+\text{a}^2}
3. \frac{1}{\text{x}^2+\text{b}^2}-\frac{1}{\text{x}^2+\text{a}^2}
4. None of these.
5. \sec^{-1}\text{x}+\text{cosec}^{-1}\frac{\text{x}}{\sqrt{\text{x}^2-1}}
1. \frac{2}{\sqrt{\text{x}^2-1}}
2. \frac{-2}{\sqrt{\text{x}^2-1}}
3. \frac{1}{|\text{x}|\sqrt{\text{x}^2-1}}
4. \frac{2}{|\text{x}|\sqrt{\text{x}^2-1}}

Ans: 1. (a) \frac{-\sin\sqrt{\text{x}}}{2\sqrt{\text{x}}}


Solution:
Let \text{y}=\cos\sqrt{\text{x}}
\therefore\frac{\text{dy}}{\text{dx}}=\frac{\text{d}}{\text{dx}}(\cos\sqrt{\text{x}})=-\sin\sqrt{\text{x}}\cdot\frac{\text{d}}{\text{dx}}(\sqrt{\text{x}})
=-\sin\sqrt{\text{x}}\times\frac{1}{2\sqrt{\text{x}}}=\frac{-\sin\sqrt{\text{x}}}{2\sqrt{\text{x}}}
2. (a) \Big(\frac{\text{x}^2-1}{\text{x}^2}\Big)\cdot7^{\text{x}+\frac{1}{\text{x}}}\cdot\log7
Solution:
Let \text{y}=7^{\text{x}+\frac{1}{\text{x}}}\therefore\frac{\text{dy}}{\text{dx}}=\frac{\text{d}}{\text{dx}}\Big(7^{\text{x}+\frac{1}{\text{x}}}\Big)
=7^{\text{x}+\frac{1}{\text{x}}}\cdot\log7\cdot\frac{\text{d}}{\text{dx}}\Big(\text{x}+\frac{1}{\text{x}}\Big)=7^{\text{x}+\frac{1}{\text{x}}}\cdot\log7\cdot\Big(1-\frac{1}{\text{x}^2}\Big)
=\Big(\frac{\text{x}^2-1}{\text{x}^2}\Big)\cdot7^{\text{x}+\frac{1}{\text{x}}}\cdot\log7
3. (a) \frac{1}{2}\sec^2\frac{\text{x}}{2}
Solution:
Let \text{y}=\sqrt\frac{{1-\cos\text{x}}}{1+\cos\text{x}}=\sqrt{\frac{1-1+2\sin^2\frac{\text{x}}{2}}{2\cos^2\frac{\text{x}}{2}-1+1}}=\tan\Big(\frac{\text{x}}{2}\Big)
\therefore\frac{\text{dy}}{\text{dx}}=\sec^2\frac{\text{x}}{2}\cdot\frac{1}{2}=\frac{1}{2}\sec^2\frac{\text{x}}{2}
4. (b) \frac{1}{\text{x}^2+\text{b}^2}+\frac{1}{\text{x}^2+\text{a}^2}
Solution:
Let \text{y}=\frac{1}{\text{b}}\tan^{-1}\Big(\frac{\text{x}}{\text{b}}\Big)+\frac{1}{\text{a}}\tan^{-1}\Big(\frac{\text{x}}{\text{a}}\Big)
\therefore\frac{\text{dy}}{\text{dx}}=\frac{1}{\text{b}}\times\frac{1}{1+\frac{\text{x}^2}{\text{b}^2}}\times\frac{1}{\text{b}}+\frac{1}{\text{a}}\times\frac{1}{1+\frac{\text{x}^2}
{\text{a}^2}}\times\frac{1}{\text{a}}
=\frac{1}{\text{x}^2+\text{b}^2}+\frac{1}{\text{x}^2+\text{a}^2}
5. (d) \frac{2}{|\text{x}|\sqrt{\text{x}^2-1}}
Solution:
Let \text{y}=\sec^{-1}\text{x}+\text{cosec}^{-1}\frac{\text{x}}{\sqrt{\text{x}^2-1}}
Put \text{x}=\sec\theta\Rightarrow\theta=\sec^{-1}\text{x}
\therefore\text{y}=\sec^{-1}(\sec\theta)+\text{cosec}^{-1}\Big(\frac{\sec\theta}{\sqrt{\sec^2\theta-1}}\Big)
=\theta+\sin^{-1}\Big[\sqrt{1-\cos^2\theta}\Big]
=\theta+\sin^{-1}(\sin\theta)=\theta+\theta=2\theta=2\sec^{-1}\text{x}
\therefore\frac{\text{dy}}{\text{dx}}=2\frac{\text{d}}{\text{dx}}(\sec^{-1}\text{x})=2\times\frac{1}{|\text{x}|\sqrt{\text{x}^2-1}}
=\frac{2}{|\text{x}|\sqrt{\text{x}^2-1}}
Q155. If \log\sqrt{\text{x}^2+\text{y}^2}=\tan^{-1}\Big(\frac{\text{y}}{\text{x}}\Big), prove that \frac{\text{dx}}{\text{dx}}=\frac{\text{x}+\text{y}}{\text{x}-\text{y}} 5 Marks

Ans: Here,
\log\sqrt{\text{x}^2+\text{y}^2}=\tan^{-1}\Big(\frac{\text{x}}{\text{y}}\Big)
\Rightarrow\log(\text{x}^2+\text{y}^2)^{\frac{1}{2}}=\tan^{-1}\Big(\frac{\text{x}}{\text{y}}\Big)
\Rightarrow\frac{1}{2}\log(\text{x}^2+\text{y}^2)=\tan^{-1}\Big(\frac{\text{y}}{\text{x}}\Big)
Differentiating with respect to x,
\Rightarrow\frac{1}{2}\frac{\text{d}}{\text{dx}}\log(\text{x}^2+\text{y}^2)=\frac{\text{d}}{\text{dx}}\tan^{-1}\Big(\frac{\text{y}}{\text{x}}\Big)
\Rightarrow\frac{1}{2}\times\Big(\frac{1}{\text{x}^2+\text{y}^2}\Big)\frac{\text{d}}{\text{dx}}(\text{x}^2+\text{y}^2)=\frac{1}{1+\Big(\frac{\text{y}}{\text{x}}\Big)^2}\frac{\text{d}}
{\text{dx}}\Big(\frac{\text{y}}{\text{x}}\Big)
\Rightarrow\frac{1}{2}\Big(\frac{1}{\text{x}^2+\text{y}^2}\Big)\Big[2\text{x}+2\text{y}\frac{\text{dy}}{\text{dx}}\Big]=\frac{\text{x}^2}
{(\text{x}^2+\text{y}^2)}\bigg[\frac{\text{x}\frac{\text{dy}}{\text{dx}}-\text{y}\frac{\text{d}}{\text{dx}}(\text{x})}{\text{x}^2}\bigg]
\Rightarrow\frac{1}{2}\Big(\frac{1}{\text{x}^2+\text{y}^2}\Big)\times2\Big(\text{x}+\text{y}\frac{\text{dy}}{\text{dx}}\Big)=\frac{\text{x}^2}
{(\text{x}^2+\text{y}^2)}\bigg[\frac{\text{x}\frac{\text{dy}}{\text{dx}}-\text{y}(1)}{\text{x}^2}\bigg]
\Rightarrow\text{x}+\text{y}\frac{\text{dy}}{\text{dx}}=\text{x}\frac{\text{dy}}{\text{dx}}-\text{y}
\Rightarrow\text{y}\frac{\text{dy}}{\text{dx}}-\text{x}\frac{\text{dy}}{\text{dx}}=-\text{y}-\text{x}
\Rightarrow\frac{\text{dy}}{\text{dx}}(\text{y}-\text{x})=-(\text{y}+\text{x})
Q156. Differentiate the following functions from first principles: 5 Marks
x2 e x .

Ans: Let f(x) = x2ex


⇒ f(x + h) = (x + h)2 e(x+h)
\therefore \frac{\text{d}}{\text{dx}}\{\text{f(x)}\}=\lim\limits_{\text{h}\rightarrow0}\frac{\text{f}(\text{x}+\text{h})-\text{f(x)}}{\text{h}}
=\lim\limits_{\text{h}\rightarrow0}\frac{(\text{x}+\text{h})^2\text{e}^{(\text{x}+\text{h})}-\text{x}^2\text{e}^{\text{x}}}{\text{h}}
=\lim\limits_{\text{h}\rightarrow0}\Big(\frac{\text{x}^2\text{e}^{(\text{x}+\text{h})}-\text{x}^2\text{e}^\text{x}}{\text{h}}+\frac{2\text{xhe}^{(\text{x}+\text{h})}}
{\text{h}}+\frac{\text{h}^2\text{e}^{(\text{x}+\text{h})}}{\text{h}}\Big)
=\lim\limits_{\text{x}\rightarrow0}\bigg(\frac{\text{x}^2\text{e}^\text{x}\big(\text{e}^{(\text{x}+\text{h}-\text{x})}-1\big)}
{\text{x}}+2\text{xe}^{(\text{x}+\text{h})}+\text{he}^{(\text{x}+\text{h})}\bigg)
=\lim\limits_{\text{h}\rightarrow0}\bigg[\text{x}^2\text{e}^{\text{x}}\frac{\big(\text{e}^\text{h}-1\big)}{\text{h}}+2\text{xe}^{(\text{x}+\text{h})}+\text{h}^{\text{e}}(\text{x}+\text{h})\bigg]
=\text{x}^2\text{e}^\text{x}+2\text{xe}^\text{x}+0\times\text{e}^\text{x}\ \Big[\text{Since,}\lim\limits_{\text{x}\rightarrow0}\frac{\text{e}^\text{x}-1}{\text{x}}=1\Big]
So,
\frac{\text{d}}{\text{dx}}(\text{x}^2\text{e}^\text{x})=\text{e}^\text{x}(\text{x}^2+2\text{x})
Q157. Differentiate the following functions with respect to x: 5 Marks
\log\sqrt{\frac{\text{x}-1}{\text{x}+1}}

Ans: Let \text{y}=\log\sqrt{\frac{\text{x}-1}{\text{x}+1}}


\Rightarrow\text{y}=\log\Big(\frac{\text{x}-1}{\text{x}+1}\Big)^\frac{1}{2}
\Rightarrow\text{y}=\frac{1}{2}\log\Big(\frac{\text{x}-1}{\text{x}+1}\Big)
\Rightarrow\text{y}=\frac{1}{2}\big[\log(\text{x}-1)-\log(\text{x}+1)\big]
Differentiate it with respect to x,
\frac{\text{dy}}{\text{dx}}=\frac{1}{2}\Big[\frac{\text{d}}{\text{dx}}\big\{\log(\text{x}-1)\big\}-\frac{\text{d}}{\text{dx}}\big\{\log(\text{x}+1)\big\}\Big]
=\frac{1}{2}\Big(\frac{1}{\text{x}-1}-\frac{1}{\text{x}+1}\Big)

https://bls.smartstudies.co.in/#/exam/pdf-preview/c59cb220-8e86-4716-9ff7-82aec16b1ade/1 43/158
5/26/24, 6:19 PM Exam Automation
=\frac{1}{2}\Big(\frac{2}{\text{x}^2-1}\Big)
=\frac{2}{\text{x}^2-1}
So,
\frac{\text{dy}}{\text{dx}}=\frac{2}{\text{x}^2-1}
Q158. Differentiate the following functions with respect to x: 5 Marks
\tan^{-1}\Big(\frac{\sqrt{1+\text{a}^2\text{x}^2-1}}{\text{ax}}\Big),\text{x}\neq0

Ans: Let \text{y}=\tan^{-1}\Big(\frac{\sqrt{1+\text{a}^2\text{x}^2}-1}{\text{ax}}\Big)


Put \text{ax}=\tan\theta
\text{y}=\tan^{-1}\Big(\frac{\sqrt{1+\text{a}^2\text{x}^2}-1}{\text{ax}}\Big)
=\tan^{-1}\Big(\frac{\sec\theta-1}{\tan\theta}\Big)
=\tan^{-1}\Big(\frac{1-\cos\theta}{\sin\theta}\Big)
=\tan^{-1}\bigg(\frac{\frac{2\sin^2\theta}{2}}{\frac{2\sin\theta}{2}\frac{\cos\theta}{2}}\bigg)
\text{y}=\tan^{-1}\Big(\frac{\tan\theta}{2}\Big)
=\frac{\theta}{2}
\text{y}=\frac{1}{2}\tan^{-1}(\text{ax})
Differentiating it with respect to x using chain rule,
\frac{\text{dy}}{\text{dx}}=\frac{1}{2}\times\Big(\frac{1}{1+(\text{ax})^2}\Big)\frac{\text{d}}{\text{dx}}(\text{ax})
\frac{\text{dy}}{\text{dx}}=\frac{1}{2(1+\text{a}^2\text{x}^2)}(\text{a})
\frac{\text{dy}}{\text{dx}}=\frac{\text{a}}{2(1+\text{a}^2\text{x}^2)}
Q159. Differentiate the following functions with respect to x: 5 Marks
(\sin^{-1}\text{x})^\text{x}

Ans: Let \text{y}=(\sin^{-1}\text{x})^\text{x}\ .....(\text{i})


Taking log on both the sides,
\log\text{y}=\log(\sin^{-1}\text{x})^\text{x}
\log\text{y}=\text{x}\log(\sin^{-1}\text{x})\ \big[\text{Since},\log\text{a}^\text{b}=\text{b}\log\text{a}\big]
Differentiating it with respect to x using product rule and chain rule,
\frac{1}{\text{y}}\frac{\text{dy}}{\text{dx}}=\text{x}\frac{\text{d}}{\text{dx}}(\log\sin^{-1}\text{x})+\log\sin(-1)\times\frac{\text{d}}{\text{dx}}(\text{x})
=\text{x}\times\frac{1}{\sin^{-1}\text{x}}\frac{\text{d}}{\text{dx}}\big(\sin^{-1}\text{x}\big)+\log\sin^{-1}\text{x}(1)
\frac{1}{\text{y}}\frac{\text{dy}}{\text{dx}}=\frac{\text{x}}{\sin^{-1}\text{x}}\Big(\frac{1}{\sqrt{1-\text{x}^2}}\Big)+\log\sin^{-1}\text{x}
\frac{\text{dy}}{\text{dx}}=\text{y}\bigg[\log\sin^{-1}\text{x}+\frac{\text{x}}{\sin^{-1}\text{x}\big(\sqrt{1-\text{x}^2}\big)}\bigg]
\frac{\text{dy}}{\text{dx}}=(\sin^{-1}\text{x})^2\Big[\log\sin^{-1}\text{x}+\frac{\text{x}}{\sin^{-1}\text{x}\sqrt{1-\text{x}^2}}\Big]
[Using equation (i)]
Q160. If \text{x}=3\sin\text{t}-\sin3\text{t}, \text{y}=3\cos-\cos3\text{t} find \frac{\text{dy}}{\text{dx}} at \text{t}=\frac{\pi}{3}. 5 Marks

Ans: \text{x}=3\sin\text{t}-\sin3\text{t}, \text{y}=3\cos\text{t}-\cos3\text{t},


\therefore\ \frac{\text{dx}}{\text{dt}}=3.\frac{\text{d}}{\text{dt}}\sin\text{t}-\frac{\text{d}}{\text{dx}}\sin3\text{t}
=3\cos\text{t}-\cos3\text{t}.\frac{\text{d}}{\text{dt}}3\text{t}=\cos\text{t}-3\cos3\text{t}\ \ \dots(\text{i})
and \frac{\text{dy}}{\text{dt}}=3.\frac{\text{d}}{\text{dt}}\cos\text{t}-\frac{\text{d}}{\text{dt}}\cos3\text{t}
=3\sin\text{t}+\sin3\text{t}.\frac{\text{d}}{\text{dt}}3\text{t}
\frac{\text{dy}}{\text{dt}}=3\sin3\text{t}-3\text{t}\sin\text{t}\ \ \dots(\text{ii})
\therefore\ \frac{\text{dy}}{\text{dt}}=\frac{\frac{\text{dy}}{\text{dt}}}{\frac{\text{dx}}{\text{dt}}}=\frac{3(\sin3\text{t}-\sin\text{t})}{3(\cos\text{t}-\cos3\text{t})}
Now, \Big(\frac{\text{dy}}{\text{dx}}\Big)_{\text{t}=\frac{\pi}{3}}=\frac{\sin\frac{3\pi}{3}-\sin\frac{\pi}{3}}{\Big(\cos\frac{\pi}{3}-\cos3\frac{\pi}{3}\Big)} =\frac{0-\frac{\sqrt{3}}{2}}{\frac{1}
{2}-(-1)}
=\frac{-\frac{\sqrt{3}}{2}}{\frac{3}{2}}=\frac{-\sqrt{3}}{3}=\frac{-1}{\sqrt{3}}
Q161. If (\sin\text{x})^{\text{y}}=(\cos\text{y})^{\text{x}}, Prove that \frac{\text{dy}}{\text{dx}}=\frac{\log\cos\text{y}-\text{y}\cot\text{x}}{\log\sin\text{x}+\text{x}\tan\text{y}} 5 Marks

Ans: We have, (\sin\text{x})^{\text{y}}=(\cos\text{y})^{\text{x}}


Taking log on both sides,
\log(\sin\text{x})^\text{y}=\log(\cos\text{y})^{\text{x}}
\Rightarrow\text{y}\log(\sin\text{x})=\text{x}\log(\cos\text{y})
Differentiating with respect to x,
\frac{\text{d}}{\text{dx}}\big[\text{y}\log\sin\text{x}\big]=\frac{\text{d}}{\text{dx}}\big[\text{x}\log\cos\text{y}\big]
\Rightarrow\text{y}\frac{\text{d}}{\text{dx}}(\log\sin\text{x})+\log\sin\text{x}\frac{\text{dy}}{\text{dx}} \\ =\text{x}\frac{\text{dy}}{\text{dx}}(\log\cos\text{y})+\log\cos\text{y}\frac{\text{d}}
{\text{dx}}(\text{x})
\Rightarrow\text{y}\Big(\frac{1}{\sin\text{x}}\Big)\frac{\text{d}}{\text{dx}}(\sin\text{x})+\log\sin\text{x}\frac{\text{dy}}{\text{dx}} \\ =\frac{\text{x}}{\cos\text{y}}\frac{\text{x}}{\text{dx}}
(\cos\text{y})+\log\cos\text{y}(1)
\Rightarrow\frac{\text{y}}{\sin\text{x}}(\cos\text{x})+\log\sin\text{x}\frac{\text{dy}}{\text{dx}} \\ =\frac{\text{x}}{\cos\text{y}}(-\sin\text{y})\frac{\text{dy}}{\text{dx}}+\log\cos\text{y}
\Rightarrow \text{y}\cot\text{x}+\log\sin\text{x}\frac{\text{dy}}{\text{dx}} \\ =-\text{x}\tan\text{y}\frac{\text{dy}}{\text{dx}}+\log\cos\text{y}
\Rightarrow\frac{\text{dy}}{\text{dx}}(\log\sin\text{x}+\text{x}\tan\text{y}) \\ =\log\cos\text{y}-\text{y}\cot\text{x}
\Rightarrow\frac{\text{dy}}{\text{dx}}=\frac{\log\cos\text{y}-\text{y}\cot\text{x}}{\log\sin\text{x}+\text{x}\tan\text{y}}
Q162. If \text{x}=\text{a}\sin\text{t}\ \text{and}\ \text{y}=\text{a}(\cos\text{t}+\log\tan\frac{\text{t}}{2}), find \frac{\text{d}^2\text{y}}{\text{dx}^2} 5 Marks

Ans: \text{x}=\text{a}\sin\text{t}\ \text{and}\ \text{y}=\text{a}(\cos\text{t}+\log\tan\frac{\text{t}}{2}),


\frac{\text{dx}}{\text{dt}}=\text{a}\cos\text{t}
\frac{\text{d}^2\text{x}}{\text{dt}^2}=-\text{a}\sin\text{t}
\frac{\text{dy}}{\text{dt}}=-\text{a}\sin\text{t}+\text{a}\frac{1}{\tan\frac{\text{t}}{2}}\times\sec^2\frac{\text{t}}{2}\times\frac{1}{2}
=-\text{a}\sin\text{t}+\text{a}\frac{1}{2\sin\frac{\text{t}}{2}\cos\frac{\text{t}}{2}}
=-\text{a}\sin\text{t}+\text{a}\ \text{cosec}\ \text{t}
\frac{\text{d}^2\text{y}}{\text{dt}^2}=-\text{a}\cos\text{t}-\text{a cosec t}\cot\text{t}
\frac{\text{d}^2\text{y}}{\text{dx}^2}=\frac{\frac{\text{dx}}{\text{dt}}\frac{\text{d}^2\text{y}}{\text{dt}^2}-\frac{\text{dy}}{\text{dt}}\frac{\text{d}^2\text{x}}{\text{dt}^2}}
{\Big(\frac{\text{dx}}{\text{dt}}\Big)^3}
=\frac{\text{a}\cos\text{t}(-\text{a}\cos\text{t}-\text{a}\ \text{cosec t}\cot\text{t})-(-\text{a}\sin\text{t}+\text{a}\text{cosec t})(-\text{a}\sin\text{t})}{(\text{a}\cos\text{t})^3}
=\frac{-\text{a}^2\cos^2\text{t}-\text{a}^2\cot^2\text{t}-\text{a}^2\sin^2\text{t}+\text{a}^2}{\text{a}^3\cos^3\text{t}}
=\frac{-\text{a}^2\cos^2\text{t}-\text{a}^2\sin^2\text{t}-\text{a}^2\cot^2\text{t}+\text{a}^2}{\text{a}^3\cos^3\text{t}}
=\frac{-\text{a}^2(\cos^2\text{t}+\sin^2\text{t})-\text{a}^2\cot^2\text{t}+\text{a}^2}{\text{a}^3\cos^3\text{t}}
=-\frac{1}{\text{a}\sin^2\text{t}\cos\text{t}}
Q163. Logarithmic differentiation is a powerful technique to differentiate functions of the form \text{f}(\text{x})=[\text{u}(\text{x})]^{\text{v}(\text{x})}, where both u(x) and v(x) are differentiable 5 Marks
functions and f and u need to be positive functions.
Let function \text{y}=\text{f}(\text{x})=(\text{u}(\text{x}))^{\text{v}(\text{x})}, then \text{y}'=\text{y}\Big[\frac{\text{v}(\text{x})}{\text{u}
(\text{x})}\text{u}'(\text{x})+\text{v}'(\text{x})\cdot\log[\text{u}(\text{x})]\Big]
On the basis of above information, answer the following questions.
1. Differentiate xx w.r.t. x.
1. \text{x}^\text{x}(1+\log\text{x})
2. \text{x}^\text{x}(1-\log\text{x})
3. -\text{x}^\text{x}(1+\log\text{x})
4. \text{x}^\text{x}\log\text{x}

https://bls.smartstudies.co.in/#/exam/pdf-preview/c59cb220-8e86-4716-9ff7-82aec16b1ade/1 44/158
5/26/24, 6:19 PM Exam Automation
x x a a
2. Differentiate x + a + x + a w.r.t. x.
1. (1+\log\text{x})+(\text{a}^\text{x}\log\text{a}+\text{ax}^{\text{a}-1})
2. \text{x}^\text{x}(1+\log\text{x})+\log\text{a}+\text{ax}^{\text{a}-1}
3. \text{x}^\text{x}(1+\log\text{x})+\text{x}^\text{a}\log\text{x}+\text{ax}^{\text{a}-1}
4. \text{x}^\text{x}(1+\log\text{x})+\text{a}^\text{x}\log\text{a}+\text{ax}^{\text{a}-1}
3. If \text{x}=\text{e}^\frac{\text{x}}{\text{y}}, then find \frac{\text{dy}}{\text{dx}}.
1. -\frac{(\text{x}+\text{y})}{\text{x}\log\text{x}}
2. -\frac{(\text{x}-\text{y})}{\text{x}\log\text{x}}
3. \frac{(\text{x}+\text{y})}{\text{x}\log\text{x}}
4. \frac{\text{x}-\text{y}}{\text{x}\log\text{x}}
4. If y = (2 - x)3(3 + 2x)5, then find \frac{\text{dy}}{\text{dx}}.
1. (2-\text{x})^3(3+2\text{x})^5\Big[\frac{15}{3+2\text{x}}-\frac{8}{2-\text{x}}\Big]
2. (2-\text{x})^3(3+2\text{x})^5\Big[\frac{15}{3+2\text{x}}+\frac{3}{2-\text{x}}\Big]
3. (2-\text{x})^3(3+2\text{x})^5\Big[\frac{10}{3+2\text{x}}-\frac{3}{2-\text{x}}\Big]
4. (2-\text{x})^3(3+2\text{x})^5\cdot\Big[\frac{10}{3+2\text{x}}+\frac{3}{2-\text{x}}\Big]
5. If \text{y}=\text{x}^\text{x}\cdot\text{e}^{(2\text{x}+5)}, then find \frac{\text{dy}}{\text{dx}}.
1. \text{x}^\text{x}\text{e}^{2\text{x}+5}
2. \text{x}^\text{x}\text{e}^{2\text{x}+5}(3-\log\text{x})
3. \text{x}^\text{x}\text{e}^{2\text{x}+5}(1-\log\text{x})
4. \text{x}^\text{x}\text{e}^{2\text{x}+5}\cdot(3+\log\text{x})

Ans: 1. (a) \text{x}^\text{x}(1+\log\text{x})


Solution:
Let \text{y}=\text{x}^\text{x}\Rightarrow\log\text{y}=\text{x}\log\text{x}
\Rightarrow\frac{1}{\text{y}}\frac{\text{dy}}{\text{dx}}=\frac{\text{d}}{\text{dx}}(\text{x}\log\text{x})
\Rightarrow\frac{\text{dy}}{\text{dx}}=\text{x}^\text{x}[1\times\log\text{x}+\text{x}\times\frac{1}{\text{x}}]
=\text{x}^\text{x}[1+\log\text{x}]
2. (d) \text{x}^\text{x}(1+\log\text{x})+\text{a}^\text{x}\log\text{a}+\text{ax}^{\text{a}-1}
3. (d) \frac{\text{x}-\text{y}}{\text{x}\log\text{x}}
Solution:
Given \text{x}=\text{e}^\frac{\text{x}}{\text{y}}\Rightarrow\log\text{x}=\frac{\text{x}}{\text{y}}\log\text{e}\Rightarrow\text{y}\log\text{x}=\text{x}
\Rightarrow\text{y}\frac{1}{\text{x}}+(\log\text{x})\frac{\text{dy}}{\text{dx}}=1
\Rightarrow\frac{\text{dy}}{\text{dx}}=\Big(1-\frac{\text{y}}{\text{x}}\Big)\frac{1}{\log\text{x}}\Rightarrow\frac{1}{\text{x}\log\text{x}}(\text{x}-\text{y})
4. (c) (2-\text{x})^3(3+2\text{x})^5\Big[\frac{10}{3+2\text{x}}-\frac{3}{2-\text{x}}\Big]
Solution:
\text{y}=(2-\text{x})^3(3+2\text{x})^5
\Rightarrow\log\text{y}=\log(2-\text{x})^3+\log(3+2\text{x})^5
=3\log(2-\text{x})+5\log(3+2\text{x})
\Rightarrow\frac{1}{\text{y}}\frac{\text{dy}}{\text{dx}}=\frac{3\times(-1)}{2-\text{x}}+\frac{5}{3+2\text{x}}\times(2)
\Rightarrow\frac{\text{dy}}{\text{dx}}=(2-\text{x})^3(3+2\text{x})^5\Big[\frac{10}{3+2\text{x}}-\frac{3}{2-\text{x}}\Big]
5. (d) \text{x}^\text{x}\text{e}^{2\text{x}+5}\cdot(3+\log\text{x})
Solution:
\text{y}=\text{x}^\text{x}\cdot\text{e}^{(2\text{x}+5)}
\Rightarrow\log\text{y}=\text{x}\log\text{x}+(2\text{x}+5)
\Rightarrow\frac{1}{\text{y}}\cdot\frac{\text{dy}}{\text{dx}}=\Big(\text{x}\cdot\frac{1}{\text{x}}+\log\text{x}\Big)+2
\Rightarrow\frac{\text{dy}}{\text{dx}}=\text{x}^\text{x}\cdot\text{e}^{2\text{x}+5}\cdot(3+\log\text{x})
Q164. Differentiate the following functions with respect to x: 5 Marks
(\log\text{x})^{\log\text{x}}

Ans: Let \text{y}=(\log\text{x})^{\log\text{x}}\ .....(\text{i})


Taking logarithm on both the sides, we obtain
\log\text{y}=\log\text{x}.\log(\log\text{x})
Differentiating both sides with resepect to x, we obtain
\frac{1}{\text{y}}\frac{\text{dy}}{\text{dx}}=\frac{\text{d}}{\text{dx}}\big[\log\text{x}.\log(\log\text{x})\big]
\Rightarrow\ \frac{1}{\text{y}}\frac{\text{dy}}{\text{dx}}=\log(\log\text{x}).\frac{\text{d}}{\text{dx}}(\log\text{x})+\log\text{x}.\frac{\text{d}}{\text{dx}}[\log(\log\text{x})]
\Rightarrow\ \frac{\text{dy}}{\text{dx}}=\text{y}\Big[\log(\log\text{x}).\frac{1}{\text{x}}+\log\text{x}.\frac{1}{\log\text{x}}.\frac{\text{d}}{\text{dx}}(\log\text{x})\Big]
\Rightarrow\ \frac{\text{dy}}{\text{dx}}=\text{y}\Big[\frac{1}{\text{x}}\log(\log\text{x})+\frac{1}{\text{x}}\Big]
\therefore\ \frac{\text{dy}}{\text{dx}}=(\log\text{x})^{\log\text{x}}\Big[\frac{1}{\text{x}}+\frac{\log(\log\text{x})}{\text{x}}\Big]
Q165. If \text{y}=(\text{x}-1)\log(\text{x}-1)-(\text{x}+1)\log(\text{x}+1) prove that \frac{\text{dy}}{\text{dx}}=\log\Big(\frac{\text{x}-1}{1+\text{x}}\Big) 5 Marks

Ans: We have, \text{y}=(\text{x}-1)\log(\text{x}-1)-(\text{x}+1)\log(\text{x}+1)


Differentiating with respect to x,
\frac{\text{dy}}{\text{dx}}=\frac{\text{d}}{\text{dx}}\big[(\text{x}-1)\log(\text{x}-1)-(\text{x}+1)\log(\text{x}+1)\big]
=\Big[(\text{x}-1)\frac{\text{d}}{\text{dx}}\log(\text{x}-1)+\log(\text{x}-1)\frac{\text{d}}{\text{dx}}(\text{x}-1)\Big] \\ -\Big[(\text{x}+1)\frac{\text{d}}
{\text{dx}}\log(\text{x}+1)+\log(\text{x}+1)\frac{\text{d}}{\text{dx}}(\text{x}+1)\Big]
=\Big[(\text{x}-1)\times\frac{1}{(\text{x}-1)}\frac{\text{d}}{\text{dx}}(\text{x}-1)+\log(\text{x}-1)\times(1)\Big] \\ -\Big[(\text{x}+1)\times\frac{1}{(\text{x}+1)}\times\frac{\text{d}}{\text{dx}}
(\text{x}+1)+\log(\text{x}+1)(1)\Big]
=\big[1+\log(\text{x}-1)\big]-\big[1+\log(\text{x}+1)\big]
=\log(\text{x}-1)-\log(\text{x}+1)
=\log\frac{(\text{x}-1)}{(\text{x}+1)}
So,
\frac{\text{dy}}{\text{dx}}=\log\frac{(\text{x}-1)}{(\text{x}+1)}
Q166. Examine the differentiability of f, where f is defined by: 5 Marks
\text{f(x)}=\begin{cases}\text{x}^2\sin\frac{1}{\text{x}},&\text{if x}\neq0\\0,&\text{if x}=0\end{cases}
at x = 0.

Ans: We have, \text{f(x)}=\begin{cases}\text{x}^2\sin\frac{1}{\text{x}},&\text{if x}\neq0\\0,&\text{if x}=0\end{cases} at x = 0.


For differentiability at x = 0,
\text{Lf}'(0)=\lim\limits_{\text{x}\rightarrow0^-}\frac{\text{f(x)}-\text{f}(0)}{\text{x}-0}=\lim\limits_{\text{x}\rightarrow0^-}\frac{\text{x}^2\sin\frac{1}{\text{x}}-0}{\text{x}-0}
=\lim\limits_{\text{h}\rightarrow0}\frac{(0-\text{h})^2\sin\Big(\frac{1}{0-\text{h}}\Big)}{0-\text{h}}=\lim\limits_{\text{h}\rightarrow0}\frac{\text{h}^2\sin\Big(\frac{-1}{\text{h}}\Big)}{-
\text{h}}
=\lim\limits_{\text{h}\rightarrow0}+\text{h}\sin\Big(\frac{1}{\text{h}}\Big)\ [\because\sin(-\theta)=-\sin\theta]
= 0 × [an oscillating number between -1 and 1] = 0
\text{Rf}'(0)=\lim\limits_{\text{x}\rightarrow0^+}\frac{\text{f(x)}-\text{f}(0)}{\text{x}-0}=\lim\limits_{\text{x}\rightarrow0^+}\frac{\text{x}^2\sin\frac{1}{\text{x}}-0}{\text{h}}
=\lim\limits_{\text{h}\rightarrow0}\frac{(0+\text{h})^2\sin\Big(\frac{1}{0+\text{h}}\Big)}{0+\text{h}}=\lim\limits_{\text{h}\rightarrow0}\frac{\text{h}^2\sin\big(\frac{1}{\text{h}}\big)}
{\text{h}}
=\lim\limits_{\text{h}\rightarrow0}\text{h}\sin\Big(\frac{1}{\text{h}}\Big)
= 0 × [an oscillating number between -1 and 1] = 0

https://bls.smartstudies.co.in/#/exam/pdf-preview/c59cb220-8e86-4716-9ff7-82aec16b1ade/1 45/158
5/26/24, 6:19 PM Exam Automation
\because Lf'(0) = Rf'(0)
So, f(x) is differentiable at x = 0.
Q167. Differentiate the following functions from first principles: 5 Marks
\log\text{cosec x}

Ans: Let \text{f(x)}=\log\text{cosec x}


\Rightarrow\ \text{f}(\text{x}+\text{h})=\log\text{cosec x}
\therefore \frac{\text{d}}{\text{dx}}\{\text{f(x)}\}=\lim\limits_{\text{h}\rightarrow0}\frac{\text{f}(\text{x}+\text{h})-\text{f(x)}}{\text{h}}
=\lim\limits_{\text{h}\rightarrow0}\frac{\log\text{cosec}(\text{x}+\text{h})-\log\text{cosec x}}{\text{h}}
=\lim\limits_{\text{h}\rightarrow0}\frac{\log\Big\{\frac{\text{cosec}(\text{x}+\text{h})}{\text{cosec x}}\Big\}}{\text{h}}
=\lim\limits_{\text{h}\rightarrow0}\frac{\log\Big\{1+\Big(\frac{\sin\text{x}}{\sin(\text{x}+\text{h})}-1\Big)\Big\}}{\text{h}}
=\lim\limits_{\text{h}\rightarrow0}\begin{Bmatrix}\frac{\log\Big\{1+\Big(\frac{\sin\text{x}-\sin(\text{x}+\text{h})}{\sin(\text{x}+\text{h})}\Big)\Big\}}{\Big\{\frac{\sin\text{x}-
\sin(\text{x}+\text{h})}{\sin(\text{x}+\text{h})}\Big\}} \end{Bmatrix}\frac{\Big\{\frac{\sin\text{x}-\sin(\text{x}+\text{h})}{\sin(\text{x}+\text{h})}\Big\}}{\text{h}}
=\lim\limits_{\text{h}\rightarrow0}\frac{2\cos\Big(\frac{\text{x}+\text{x}+\text{h}}{2}\Big)\sin\Big(\frac{\text{x}-\text{x}-\text{h}}{2}\Big)}{\sin(\text{x}+\text{h})\text{h}}
\Big[\because\ \lim\limits_{\text{x}\rightarrow0}\frac{\log(1+\text{x})}{\text{x}}=1\text{ and }\sin\text{A}-\sin\text{B} \\ =2\cos\Big(\frac{\text{A}+\text{B}}{2}\Big)\sin\Big(\frac{\text{A}-
\text{B}}{2}\Big)\Big]
=\lim\limits_{\text{h}\rightarrow0}\frac{2\cos\Big(\frac{2\text{x}+\text{h}}{2}\Big)}{\sin(\text{x}+\text{h})(-2)}\bigg\{\frac{\sin\big(-\frac{\text{h}}{2}\big)}{-\frac{\text{h}}{2}}\bigg\}
=-\cot\text{x}
\therefore\ \frac{\text{d}}{\text{dx}}(\log\text{cosec x})=-\cot\text{x}
Q168. Differentiate the following functions with respect to x: 5 Marks
\log\Big(\frac{\sin\text{x}}{1+\cos\text{x}}\Big)

Ans: Let, \log\Big(\frac{\sin\text{x}}{1+\cos\text{x}}\Big)


Differentiate with respect to x,
\frac{\text{dy}}{\text{dx}}=\frac{\text{d}}{\text{dx}}\log\Big(\frac{\sin\text{x}}{1+\cos\text{x}}\Big)
=\frac{1}{\Big(\frac{\sin\text{x}}{1+\cos\text{x}}\Big)}\times\frac{\text{d}}{\text{dx}}\Big(\frac{\sin\text{x}}{1+\cos\text{x}}\Big)
[Using chain rule]
=\Big(\frac{1+\cos\text{x}}{\sin\text{x}}\Big)\Bigg[\frac{(1+\cos\text{x})\frac{\text{d}}{\text{dx}}(\sin\text{x})-\sin\text{x}\frac{\text{d}}{\text{dx}}(1+\cos\text{x})}{(1+\cos\text{x})^2}\Bigg]
[Using quotient rule]
=\frac{(1+\cos\text{x})}{\sin\text{x}}\bigg[\frac{(1+\cos\text{x})(\cos\text{x})-\sin\text{x}(-\sin\text{x})}{(1+\cos\text{x})^2}\bigg]
=\frac{(1+\cos\text{x})}{\sin\text{x}}\Big[\frac{\cos\text{x}+\cos^2\text{x}+\sin^2\text{x}}{(1+\cos\text{x})^2}\Big]
=\frac{(1+\cos\text{x})}{\sin\text{x}}\Big[\frac{(1+\cos\text{x})}{(1+\cos\text{x})^2}\Big]
=\frac{1}{\sin\text{x}}
=\text{cosec x}
So,
\frac{\text{d}}{\text{dx}}\Big(\log\Big(\frac{\sin\text{x}}{1+\cos\text{x}}\Big)\Big)=\text{cosec x}
Q169. Find \frac{\text{dy}}{\text{dx}} 5 Marks
\text{y}=(\sin\text{x})^\text{x}+\sin^{-1}\sqrt{\text{x}}

Ans: Here,
\text{y}=(\sin\text{x})^{\text{x}}+\sin^{-1}\sqrt{\text{x}}
=\text{e}^{\log(\sin\text{x})^\text{x}}+\sin^{-1}\sqrt{\text{x}}
\text{y}=\text{e}^{\text{x}\log\sin\text{x}}+\sin^{-1}\sqrt{\text{x}}
\big[\text{Since},\log_\text{e}^\text{e}=1,\log\text{a}^\text{b}=\text{b}\log\text{a}\big]
Differentitating it with respect to x using chain rule and product rule,
\frac{\text{dy}}{\text{dx}}=\frac{\text{d}}{\text{dx}}\big(\text{e}^{\text{x}\log\sin\text{x}}\big)+\frac{\text{d}}{\text{dx}}\sin^{-1}\big(\sqrt{\text{x}}\big)
=\text{e}^{\text{x}\log\sin\text{x}}\frac{\text{d}}{\text{dx}}(\text{x}\log\sin\text{x})+\frac{1}{\sqrt{1-\big(\sqrt{\text{x}}\big)^2}}\frac{\text{d}}{\text{dx}}\big(\sqrt{\text{x}}\big)
=\text{e}^{\log(\sin\text{x})^\text{x}}\Big[\text{x}\frac{\text{d}}{\text{dx}}\log\sin\text{x}+\log\sin\text{x}\frac{\text{d}}{\text{dx}}(\text{x})+\frac{1}{\sqrt{1-\text{x}}}\times\frac{1}
{2\sqrt{\text{x}}}\Big]
=(\sin\text{x})^\text{x}\Big[\text{x}\times\frac{1}{\sin\text{x}}\frac{\text{d}}{\text{dx}}(\sin\text{x})+\log\sin\text{x}(1)\Big]+\frac{1}{2\sqrt{\text{x}-\text{x}^2}}
=(\sin\text{x})^\text{x}\Big[\frac{\text{x}}{\sin\text{x}}(\cos\text{x})+\log\sin\text{x}\Big]+\frac{1}{2\sqrt{\text{x}-\text{x}^2}}
\frac{\text{dy}}{\text{dx}}=(\sin\text{x})^\text{x}\Big[\text{x}\cot\text{x}+\log\sin\text{x}\Big]+\frac{1}{2\sqrt{\text{x}-\text{x}^2}}
Q170. If \text{x}=10(\text{t}-\sin\text{t}),\text{y}=12(1-\cos\text{t}), find \frac{\text{dy}}{\text{dx}}. 5 Marks

Ans: Here, \text{x}=10(\text{t}-\sin\text{t}),\text{y}=12(1-\cos\text{t})


\Rightarrow\frac{\text{dx}}{\text{dt}}=10(1-\cos\text{t})\ ...(\text{i})
\Rightarrow\frac{\text{dx}}{\text{dt}}=12(\sin\text{t})\ ...(\text{ii})
\Rightarrow\frac{\text{dy}}{\text{dx}}=\frac{\frac{\text{dy}}{\text{dt}}}{\frac{\text{dx}}{\text{dt}}}=\frac{12(\sin\text{t})}{10(1-\cos\text{t})} From equation (i) and (ii)
\Rightarrow\frac{\text{dy}}{\text{dx}}=\frac{12\sin\frac{\text{t}}{2}\cdot\cos\frac{\text{t}}{2}}{10\sin^2\frac{\text{t}}{2}}
\Rightarrow\frac{\text{dy}}{\text{dx}}=\frac{6}{5}\cot\frac{\text{t}}{2}
Q171. If x16y9 = (x + y)17, prove that \text{x}\frac{\text{dy}}{\text{dx}}=2\text{y} 5 Marks

Ans: Here,
x16y9 = (x + y)20
Taking log on both the siede,
\log(\text{x}^{16}\times\text{y}^{19})=\log(\text{x}^2+\text{y})^{17}
\big[\text{since}, \log(\text{AB})=\log\text{A}+\log\text{B},\log\text{a}^\text{b}=\text{b}\log\text{a}\big]
16\log\text{x}+9\log\text{y}=17\log(\text{x}^2+\text{y})
Differentiating it with respect to x using chain rule
16\frac{\text{d}}{\text{dx}}(\log\text{x})+9\frac{\text{d}}{\text{dx}}(\log\text{y})=17\frac{\text{d}}{\text{dx}}\log(\text{x}^2+\text{y})
\frac{16}{\text{x}}+\frac{9}{\text{y}}\frac{\text{dy}}{\text{dx}}=17\frac{1}{(\text{x}^2+\text{y})}\frac{\text{d}}{\text{dx}}(\text{x}^2+\text{y})
\frac{16}{\text{x}}+\frac{9}{\text{y}}\frac{\text{dy}}{\text{dx}}=\frac{17}{\text{x}^2+\text{y}}\Big[2\text{x}+\frac{\text{dy}}{\text{dx}}\Big]
\frac{9}{\text{y}}\frac{\text{dy}}{\text{dx}}-\frac{17}{(\text{x}^2+\text{y})}\frac{\text{dy}}{\text{dx}}=\Big(\frac{34\text{x}}{\text{x}^2+\text{y}}\Big)-\frac{16}{\text{x}}
\frac{\text{dy}}{\text{dx}}\Big[\frac{9}{\text{y}}-\frac{17}{(\text{x}^2+\text{y})}\Big]=\frac{34\text{x}^2-16\text{x}^2-16\text{y}}{\text{x}(\text{x}^2+\text{y})}
\frac{\text{dy}}{\text{dx}}\Big[\frac{9\text{x}^2+9\text{y}-17\text{y}}{\text{y}(\text{x}^2+\text{y})}\Big]=\frac{18\text{x}^2-16\text{y}}{\text{x}(\text{x}^2+\text{y})}
\frac{\text{dy}}{\text{dx}}=\frac{\text{y}}{\text{x}}\Big(\frac{2(9\text{x}^2-8\text{y})}{9\text{x}^2-8\text{y}}\Big)
\frac{\text{dy}}{\text{dx}}=\frac{2\text{y}}{\text{x}}
\text{x}\frac{\text{dy}}{\text{dx}}=2\text{y}
Q172. The function f(x) will be discontinuous at x = a if f(x) has 5 Marks
Discontinuity of first kind : \lim\limits_{\text{h}\rightarrow0}\text{f}(\text{a}-\text{h}) and \lim\limits_{\text{h}\rightarrow0}\text{f}(\text{a}+\text{h}) both exist but are not equal. If is
also known as irremovable discontinuity.
Discontinuity of second kind : If none of the limits \lim\limits_{\text{h}\rightarrow0}\text{f}(\text{a}-\text{h}) and \lim\limits_{\text{h}\rightarrow0}\text{f}(\text{a}+\text{h}) exist.
Removable discontinuity : \lim\limits_{\text{h}\rightarrow0}\text{f}(\text{a}-\text{h}) and \lim\limits_{\text{h}\rightarrow0}\text{f}(\text{a}+\text{h}) both exist and equal but not
equal to f(a).
Based on the above information, answer the following questions.
1. If \text{f}(\text{x})=\begin{cases}\frac{\text{x}^2-9}{\text{x}-3},&\text{for x}\neq3\\4,&\text{for x}=3\end{cases}, then at x = 3
1. f has removable discontinuity.
2. f is continuous.

https://bls.smartstudies.co.in/#/exam/pdf-preview/c59cb220-8e86-4716-9ff7-82aec16b1ade/1 46/158
5/26/24, 6:19 PM Exam Automation
3. f has irremovable discontinuity.
4. None of these.
2. Let \text{f}(\text{x})=\begin{cases}\text{x}+2,&\text{if x}\leq4\\\text{x}+4,&\text{if x}\geq4\end{cases} then at x = 4
1. f is continuous.
2. f has removable discontinuit.
3. f has irremovable discontinuit.
4. None of thesee.
3. Consider the function f(x) defined as \text{f}(\text{x})=\begin{cases}\frac{\text{x}^2-4}{\text{x}-2},&\text{for x}\neq2\\5,&\text{for x}=2\end{cases}, then at x = 2
1. f has removable discontinuity.
2. f has irremovable discontinuity.
3. f is continuous.
4. f is continuous if f(2) = 3
4. If \text{f}(\text{x})=\begin{cases}\frac{\text{x}-|\text{x}|}{\text{x}},&\text{x}\neq0\\2,&\text{x}=0\end{cases}, then at x = 0
1. f is continuous.
2. f has removable discontinuity.
3. f has irremovable discontinuity.
4. None of these.
5. If \text{f}(\text{x})=\begin{cases}\frac{\text{e}^\text{x}-1}{\log(1+2\text{x})},&\text{if x}\neq0\\7,&\text{if x}=0\end{cases}, then at x = 0
1. fis continuous if f(0) = 2
2. f is continuous
3. f has irremovable discontinuity.
4. f has removable discontinuity.

Ans: 1. (a) f has removable discontinuity.


Solution:
f(3) = 4
\lim\limits_{\text{x}\rightarrow3}\text{f}(\text{x})=\lim\limits_{\text{x}\rightarrow3}\frac{\text{x}^2-9}{\text{x}-3}=\lim\limits_{\text{x}\rightarrow3}\frac{(\text{x}+3)(\text{x}-3)}
{(\text{x}-3)}
=\lim\limits_{\text{x}\rightarrow3}(\text{x}+3)=6\because\lim\limits_{\text{x}\rightarrow3}\text{f}(\text{x})\neq\text{f}(3)
\therefore f(x) has removable discontinuity at x = 3.
2. (c) f has irremovable discontinuit.
Solution:
\lim\limits_{\text{x}\rightarrow4^-}\text{f}(\text{x})=\lim\limits_{\text{x}\rightarrow4}(\text{x}+2)=4+2=6
\lim\limits_{\text{x}\rightarrow4^+}\text{f}(\text{x})=\lim\limits_{\text{x}\rightarrow4}(\text{x}+4)=4+4=8
\therefore\lim\limits_{\text{x}\rightarrow4^-}\text{f}(\text{x})\neq\lim\limits_{\text{x}\rightarrow4^+}\text{f}(\text{x})
\therefore f(x) has an irremovable discontinuity at x = 4.
3. (a) f has removable discontinuity.
Solution:
\lim\limits_{\text{x}\rightarrow2}\text{f}(\text{x})=\lim\limits_{\text{x}\rightarrow2}\frac{(\text{x}^2-4)}{(\text{x}-2)}=\lim\limits_{\text{x}\rightarrow2}(\text{x}+2)=4
and f(2) = 5 (given) \therefore\lim\limits_{\text{x}\rightarrow2}\text{f}(\text{x})\neq\text{f}(2)
\therefore f(x) has removable discontinuity at x = 2.
4. (c) f has irremovable discontinuity.
Solution:
f(0) = 2
\lim\limits_{\text{x}\rightarrow0^-}\text{f}(\text{x})=\lim\limits_{\text{x}\rightarrow0}\frac{\text{x}+\text{x}}{\text{x}}=2
\lim\limits_{\text{x}\rightarrow0^+}\text{f}(\text{x})=\lim\limits_{\text{x}\rightarrow0}\frac{\text{x}-\text{x}}{\text{x}}=0
\because\lim\limits_{\text{x}\rightarrow0^-}\text{f}(\text{x})\neq\lim\limits_{\text{x}\rightarrow0^+}\text{f}(\text{x})
\therefore f(x) has an irremovable discontinuity at x = 0.
5. (d) f has removable discontinuity.
Solution:
f(0) = 7
\lim\limits_{\text{x}\rightarrow0}\text{f}(\text{x})=\lim\limits_{\text{x}\rightarrow0}\frac{\text{e}^\text{x}-1}
{\log(1+2\text{x})}=\lim\limits_{\text{x}\rightarrow0}\frac{\Big(\frac{\text{e}^\text{x}-1}{\text{x}}\Big)}{\frac{\log(1+2\text{x})}{2\text{x}}\cdot2}=\frac{1}{2}
\because\lim\limits_{\text{x}\rightarrow0}\text{f}(\text{x})\neq\text{f}(0)
\therefore f(x) has removable discontinuity at x = 0.
Q173. Let f(x) be a real valued function, then its 5 Marks
Left Hand Derivative (L.H.D.) : \text{Lf}'(\text{a})=\lim_\limits{\text{h}\rightarrow0}\frac{\text{f}(\text{a}-\text{h})-\text{f}(\text{a})}{-\text{h}}
Right Hand Derivative (R.H.D.) : \text{Rf}'(\text{a})=\lim_\limits{\text{h}\rightarrow0}\frac{\text{f}(\text{a}+\text{h})-\text{f}(\text{a})}{\text{h}}
Also, a function f(x) is said to be differentiable at x = a if its L.H.D. and R.H.D. at x = a exist and are equal.
For the function \text{f}(\text{x})=\begin{cases}|\text{x}-3|,\text{x}\geq1\\\\\frac{\text{x}^2}{4}-\frac{3\text{x}}{2}+\frac{13}{4},\text{x}<1\end{cases}, answer the following questions.
1. R.H.D. of f(x) at x = 1 is:
1. 1
2. -1
3. 0
4. 2
2. L.H.D. of f(x) at x = 1 is:
1. 1
2. -1
3. 0
4. 2
3. f(x) is non-differentiable at:
1. x = 1
2. x = 2
3. x = 3
4. x = 4
4. Find the value of f'(2).
1. 1
2. 2
3. 3
4. -1
5. The value of f'(-1) is:
1. 2
2. 1
3. -2
4. -1

Ans: We have, \text{f}(\text{x})=\begin{cases}\text{x}-3&,\text{x}\geq3\\3-\text{x}&,1\leq\text{x}<3\\\\\frac{\text{x}^2}{4}-\frac{3\text{x}}{2}+\frac{13}{4}&,\text{x}<1\end{cases}

https://bls.smartstudies.co.in/#/exam/pdf-preview/c59cb220-8e86-4716-9ff7-82aec16b1ade/1 47/158
5/26/24, 6:19 PM Exam Automation
1. (b) -1
Solution:
\text{Rf}'(1)=\lim\limits_{\text{h}\rightarrow0}\frac{\text{f}(1+\text{h})-\text{f}(1)}{\text{h}}
\lim\limits_{\text{h}\rightarrow0}\frac{3-(1+\text{h})-2}{\text{h}}=\lim\limits_{\text{h}\rightarrow0}-\frac{\text{h}}{\text{h}}=-1
2. (b) -1
Solution:
\text{Lf}'(1)=\lim\limits_{\text{h}\rightarrow0}\frac{\text{f}(1-\text{h})-\text{f}(1)}{-\text{h}}
=\lim\limits_{\text{h}\rightarrow0}\frac{-1}{\text{h}}\Big[\frac{(1-\text{h})^2}{4}-\frac{3(1-\text{h})}{2}+\frac{13}{4}-2\Big]
=\lim\limits_{\text{h}\rightarrow0}\frac{-1}{\text{h}}\Big(\frac{1+\text{h}^2-2\text{h}-6+6\text{h}+13-8}{-4\text{h}}\Big)
=\lim\limits_{\text{h}\rightarrow0}\Big(\frac{\text{h}^2+4\text{h}}{-4\text{h}}\Big)=-1
3. (c) x = 3
Solution:
Since, R.H.D. at x = 3 is 1
and L.H.D. at x = 3 is - 1
\therefore f(x) is non-differentiable at x = 3.
4. (d) -1
5. (c) -2
Solution:
From above, we have
\text{f}'(\text{x})=\frac{\text{x}}{2}-\frac{3}{2},\text{x}<1
\therefore\text{f}'(-1)=\frac{-1}{2}-\frac{3}{2}=-2
Q174. Differentiate the following functions with respect to x: 5 Marks
\sin^{-1}\Big\{\frac{\sin\text{x}+\cos\text{x}}{\sqrt{2}}\Big\},-\frac{3\pi}{4}<\text{x}<\frac{\pi}{4}

Ans: Let \text{y}=\sin^{-1}\Big\{\frac{\sin\text{x}+\cos\text{x}}{\sqrt{2}}\Big\}


=\sin^{-1}\bigg\{\sin\text{x}\Big(\frac{1}{\sqrt{2}}\Big)+\cos\text{x}\Big(\frac{1}{\sqrt{2}}\Big)\bigg\}
=\sin^{-1}\Big\{\sin{\text{x}}\cos\frac{\pi}{4}+\cos\text{x}\times\sin\frac{\pi}{4}\Big\}
\text{y}=\sin^{-1}\Big\{\sin\Big(\text{x}+\frac{\pi}{4}\Big)\Big\}
Here, \frac{-3\pi}{4}<\text{x}<\frac{\pi}{4}
\Rightarrow\Big(\frac{-3\pi}{4}+\frac{\pi}{4}\Big)
\Big[\text{Since},\sin^{-1}(\sin\theta)=\theta, \text{ if }\theta\in\Big[\frac{-\pi}{2},\frac{\pi}{2}\Big]\Big]
Differentiating it with respect to x,
\frac{\text{dy}}{\text{dx}}=1+0
\frac{\text{dy}}{\text{dx}}=1
Q175. Verify Rolle's theorem for the following function on the indicated intervals 5 Marks
f(x) = (x2 - 1)(x - 2) on [-1, 2]

Ans: Here,
f(x) = (x2 - 1)(x - 2) on [-1, 2]
f(x) is continuous is [-1, 2] and differentiable in (-1, 2) as it is a polynomial functions.
Now,
f(-1) = (1-1)(-1-2) = 0
f(2) = (4-1)(2-2) = 0
⇒ f(-1) = f(2)
So, Rolle's theorem is applicable on f(x) is [-1, 2] therefore, we have to show that there exist a \text{c}\in(-1,2) such that f'(c) = 0
Now,
f(x) = (x2 - 1)(x - 2)
f'(x) = 2x(x - 2) + (x2 - 1)
= 2x2 - 4 + x2 - 1
f'(x) = 3x2 - 5
Now,
f'(c) = 0
⇒ 3x2 - 5 = 0
\Rightarrow\text{x}=-\sqrt{\frac{5}{3}} or \text{x}=\sqrt{\frac{5}{3}}\in(-1,2)
Thus, Rolle's theorem is verified.
Q176. If \text{x}=\text{a}(\theta-\sin\theta),\text{y}=\text{a}(1+\cos\theta) find \frac{\text{d}^2\text{y}}{\text{dx}^2} 5 Marks

Ans: \text{x} =\text{a} (\theta -\sin\theta)\dots\text{ eq. } 1


\text{y}=\text{a}(1+\cos\theta)\dots\text{ eq. 2}
To find: \frac{\text{d}^2\text{y}}{\text{dx}^2}
As, \frac{\text{d}^2\text{y}}{\text{dx}^2}=\frac{\text{d}}{\text{dx}}\Big(\frac{\text{dy}}{\text{dx}}\Big)
So, lets first find \frac{\text{dy}}{\text{dx}} using parametric form and differentiate it again.
\frac{\text{dx}\text{}}{\text{d}\theta}=\frac{\text{d}}{\text{d}\theta}\text{a}(\theta-\sin\theta)=\text{a}(1-\cos\theta)\dots\ \text{eq. 3}
Similarly,
\frac{\text{dy}}{\text{d}\theta}=\frac{\text{d}}{\text{d}\theta}\text{a}(1+\cos\theta)=-\text{a}\sin\theta\dots\text{ eq. }4
\Big[\because\frac{\text{d}}{\text{dx}}\cos\text{x}=-\sin\text{x},\frac{\text{d}}{\text{dx}}\sin\text{x}=\cos\text{x}\Big]
\therefore\frac{\text{dy}}{\text{dx}}=\frac{\frac{\text{dy}}{\text{d}\theta}}{\frac{\text{dx}}{\text{d}\theta}}=\frac{-\text{a}\sin\theta}{\text{a}(1-\cos\theta)}=\frac{-\sin\theta}{(1-
\cos\theta)}\dots\ \text{eq. }5
Differentiating again w.r.t. x:
\frac{\text{d}}{\text{dx}}\Big(\frac{\text{dy}}{\text{dx}}\Big)=-\frac{\text{d}}{\text{dx}}\Big(\frac{\sin\theta}{1-\cos\theta}\Big)
Using product rule and chain rule of differentiation together:
\frac{\text{d}^2\text{y}}{\text{dx}^2}=\Big\{-\frac{1}{1-\cos\theta}\frac{\text{d}}{\text{d}\theta}\sin\theta-\sin\theta\frac{\text{d}}{\text{d}\theta}\frac{1}{(1-
\cos\theta)}\Big\}\frac{\text{d}\theta}{\text{dx}}
Apply chain rule to determine \frac{\text{d}}{\text{d}\theta}\frac{1}{(1-\cos\theta)}
\frac{\text{d}^2\text{y}}{\text{dx}}=\Big\{\frac{-\cos\theta}{1-\cos\theta}+\frac{\sin^2\theta}{(1-\cos\theta)^2}\Big\}\frac{1}{\text{a}(1-\cos\theta)} [using eq.3]
\frac{\text{d}^2\text{y}}{\text{dx}^2}=\bigg\{\frac{-\cos\theta(1-\cos\theta)+\sin^2\theta}{(1-\cos\theta)^2}\bigg\}\frac{1}{\text{a}(1-\cos\theta)}
\frac{\text{d}^2\text{y}}{\text{dx}^2}=\bigg\{\frac{-\cos\theta+\cos^2\theta+\sin^2\theta}{(1-\cos\theta)^2}\bigg\}\frac{1}{\text{a}(1-\cos\theta)}
\frac{\text{d}^2\text{y}}{\text{dx}^2}=\Big\{\frac{1-\cos\theta}{(1-\cos\theta)^2}\Big\}\frac{1}{\text{a}(1-\cos\theta)}[\because\cos^2\theta+\sin^2\theta=1]
\frac{\text{d}^2\text{y}}{\text{dx}^2}=\frac{1}{\text{a(1}-\cos\theta)^2}
\frac{\text{d}^2\text{y}}{\text{dx}^2}=\frac{1}{\text{a}\Big(2\sin^2\frac{\theta}{2}\Big)^2}\big[\because-\cos\theta=2\sin^2\frac{\theta}{2}\big]
\therefore\frac{\text{d}^2\text{y}}{\text{dx}^2}=\frac{1}{\text{4a}}\text{cosec}^4\frac{\theta}{2}
Q177. Differentiate the following functions with respect to x: 5 Marks
\log\Big(\frac{\text{x}^2+\text{x}+1}{\text{x}^3-\text{x}+1}\Big)

Ans: Let, \text{y}=\log\Big(\frac{\text{x}^2+\text{x}+1}{\text{x}^3-\text{x}+1}\Big)


Differentiate with respect to x we get,
\frac{\text{dy}}{\text{dx}}=\frac{\text{d}}{\text{dx}}\Big[\log\Big(\frac{\text{x}^2+\text{x}+1}{\text{x}^3-\text{x}+1}\Big)\Big]
=\frac{1}{\Big(\frac{\text{x}^2+\text{x}+1}{\text{x}^2-\text{x}+1}\Big)}\frac{\text{d}}{\text{dx}}\Big(\frac{\text{x}^2+\text{x}+1}{\text{x}^2-\text{x}+1}\Big)

https://bls.smartstudies.co.in/#/exam/pdf-preview/c59cb220-8e86-4716-9ff7-82aec16b1ade/1 48/158
5/26/24, 6:19 PM Exam Automation
[Using chain rule and quotient rule]
=\Big(\frac{\text{x}^2+\text{x}+1}{\text{x}^3-\text{x}+1}\Big)\bigg[\frac{\text{x}^2-\text{x}+1\frac{\text{d}}{\text{dx}}(\text{x}^2+\text{x}+1)-(\text{x}^2+\text{x}+1)\frac{\text{d}}{\text{dx}}
(\text{x}^2-\text{x}+1)}{(\text{x}^2-\text{x}+1)^2}\bigg]
=\Big(\frac{\text{x}^2+\text{x}+1}{\text{x}^3-\text{x}+1}\Big)\bigg[\frac{(\text{x}^2-\text{x}+1)(2\text{x}+1)-(\text{x}^2+\text{x}+1)(2\text{x}-1)}{(\text{x}^2-\text{x}+1)^2}\bigg]
=\Big(\frac{\text{x}^2+\text{x}+1}{\text{x}^3-\text{x}+1}\Big)\bigg[\frac{2\text{x}^3-2\text{x}^2+2\text{x}+\text{x}^3-\text{x}+1-2\text{x}^3-2\text{x}^2-2\text{x}+\text{x}^2+\text{x}+1}
{(\text{x}^2-\text{x}+1)^2}\bigg]
=\frac{-4\text{x}^2+2\text{x}^3+2}{(\text{x}^2+\text{x}+1)(\text{x}^2+\text{x}+1)}
=\frac{-4\text{x}^2+2\text{x}^3+2}{(\text{x}^2+1)^2-(\text{x})^2}
=\frac{-2(\text{x}^2-1)}{\text{x}^4+1+2\text{x}^2-\text{x}^2}
=\frac{-2(\text{x}^2-1)}{\text{x}^4+\text{x}^2+1}
So,
\frac{\text{d}}{\text{dx}}\Big\{\log\Big(\frac{\text{x}^2+\text{x}+1}{\text{x}^2-\text{x}+1}\Big)\Big\}=\frac{-2(\text{x}^2-1)}{\text{x}^4+\text{x}^2+1}
Q178. Differentiate the following functions with respect to x: 5 Marks
\tan^{-1}\Big(\frac{\cos\text{x}+\sin\text{x}}{\cos\text{x}-\sin\text{x}}\Big), \frac{\pi}{4}<\text{x}<\frac{\pi}{4}

Ans: Let \text{y}=\tan^{-1}\Big[\frac{\cos\text{x}+\sin\text{x}}{\cos\text{x}-\sin\text{x}}\Big]


=\tan\bigg[\frac{\frac{\cos\text{x}+\sin\text{x}}{\cos\text{x}}}{\frac{\cos\text{x}-\sin\text{x}}{\cos\text{x}}}\bigg]
=\tan^{-1}\bigg[\frac{\frac{\cos\text{x}}{\cos\text{x}}+\frac{\sin\text{x}}{\cos\text{x}}}{\frac{\cos\text{x}}{\cos\text{x}}-\frac{\sin\text{x}}{\cos\text{x}}}\bigg]
=\tan^{-1}\Big[\frac{1+\tan\text{x}}{1-\tan\text{x}}\Big]
=\tan^{-1}\bigg[\frac{\frac{\tan\pi}{4}+\tan\text{x}}{1-\frac{\tan\pi}{4}\tan\text{x}}\bigg]
=\tan^{-1}\Big[\tan\Big(\frac{\pi}{4}+\text{x}\Big)\Big]
\text{y}=\frac{\pi}{4}+\text{x}
Differentiating it with respect to x,
\frac{\text{dy}}{\text{dx}}=0+1
\frac{\text{dy}}{\text{dx}}=1
Q179. If a real valued function f{x) is finitely derivable at any point of its domain, it is necessarily continuous at that point. But its converse need not be true. 5 Marks
For example, every polynomial, constant function are both continuous as well as differentiable and inverse trigonometric functions are continuous and differentiable in its domains etc.
Based on the above information, answer the following questions.
1. If \text{f}(\text{x})=\begin{cases}\text{x},\text{for x}\leq0\\0,\text{for x}>0\end{cases}, then at x = 0
1. f(x) is differentiable and continuous.
2. f(x) is neither continuous nor differentiable.
3. f(x) is continuous but not differentiable.
4. None of these.
2. If \text{f}(\text{x})=|\text{x}-1|,\text{x }\epsilon\text{ R}, then at x = 1
1. f(x) is not continuous.
2. f(x) is continuous but not differentiable.
3. f(x) is continuous and differentiable.
4. None of these.
3. f(x) = x3 is:
1. Continuous but not differentiable at x = 3
2. Continuous but not differentiable at x = 3
3. Neither continuous nor differentiable at x = 3
4. None of these.
4. If \text{f}(\text{x})=[\sin\text{x}], then which of the following is true?
1. f(x) is continuous and differentiable at x = 0.
2. f(x) is discontinuous at x = 0.
3. f(x) is continuous at x = 0 but not differentiable.
4. f(x) is differentiable but not continuous at \text{x}=\frac{\pi}{2}.
5. If \text{f}(\text{x})=\sin^{-1}\text{x},-1\leq\text{x}\leq1, then:
1. f(x) is both continuous and differentiable.
2. f(x) is neither continuous nor differentiable.
3. f(x) is continuous but not differentiable.
4. None of these.

Ans: 1. (c) f(x) is continuous but not differentiable.


2. (b) f(x) is continuous but not differentiable.
3. (b) Continuous but not differentiable at x = 3
4. (b) f(x) is discontinuous at x = 0.
5. (a) f(x) is both continuous and differentiable.
Q180. Differentiate the following functions with respect to x: 5 Marks
(\text{x}^\text{x})\sqrt{\text{x}}

Ans: Let \text{y}=(\text{x}^\text{x})\sqrt{\text{x}}\ .....(\text{i})


Taking log on both the sides,
\log\text{y}=\log(\text{x}^\text{x}\sqrt{\text{x}})
\log\text{y}=\text{x}\log\text{x}+\frac{1}{2}\log\text{x}
Differentiating it with respect to x,
\frac{1}{\text{y}}\frac{\text{dy}}{\text{dx}}=\text{x}\frac{\text{d}}{\text{dx}}(\log\text{x})+\log\text{x}\frac{\text{d}}{\text{dx}}(\text{x})+\frac{1}{2}\frac{\text{d}}{\text{dx}}(\log\text{x})
\Rightarrow\frac{1}{\text{y}}\frac{\text{dy}}{\text{dx}}=\text{x}\Big(\frac{1}{\text{x}}\Big)+\log\text{x}(1)+\frac{1}{2}\Big(\frac{1}{\text{x}}\Big)
\Rightarrow\frac{1}{\text{y}}\frac{\text{dy}}{\text{dx}}=1+\log\text{x}+\frac{1}{2\text{x}}
\Rightarrow\ \frac{\text{dy}}{\text{dx}}=\text{y}\Big[1+\log\text{x}+\frac{1}{2\text{x}}\Big]
\Rightarrow\ \frac{\text{dy}}{\text{dx}}=\text{x}^\text{x}\sqrt{\text{x}}\Big[1+\log\text{x}+\frac{1}{2\text{x}}\Big]
[Using equation (i)]
\Rightarrow\frac{\text{dy}}{\text{dx}}=\text{x}^{\text{x}+\frac{1}{2}}\Big[\Big(\frac{2\text{x}+1}{2\text{x}}\Big)+\log\text{x}\Big]
Q181. Differentiate the following functions with respect to x: 5 Marks
\tan^{-1}\Big(\frac{\text{x}-\text{a}}{\text{x}+\text{a}}\Big)

Ans: Let \text{y}=\tan^{-1}\Big(\frac{\text{x}-\text{a}}{\text{x}+\text{a}}\Big)


=\tan^{-1}\bigg(\frac{\frac{\text{x}-\text{a}}{\text{x}}}{\frac{\text{x}+\text{a}}{\text{x}}}\bigg)
=\tan^{-1}\bigg(\frac{\frac{\text{x}}{\text{x}}-\frac{\text{x}}{\text{x}}}{\frac{\text{x}}{\text{x}}+\frac{\text{a}}{\text{x}}}\bigg)
=\tan^{-1}\bigg(\frac{1-\frac{\text{x}}{\text{x}}}{1+1\times\frac{\text{a}}{\text{x}}}\bigg)
\text{y}=\tan^{-1}(1)-\tan^{-1}\big(\frac{\text{a}}{\text{x}}\big)
Differentiating it with respect to x using chain rule,
\frac{\text{dy}}{\text{dx}}=0-\frac{1}{1+\big(\frac{\text{a}}{\text{x}}\big)^2}\frac{\text{d}}{\text{dx}}\big(\frac{\text{a}}{\text{x}}\big)
=-\frac{\text{x}^2}{\text{x}^2+\text{a}^2}\Big(\frac{-\text{a}}{\text{x}^2}\Big)
\frac{\text{dy}}{\text{dx}}=\frac{\text{a}}{\text{a}^2+\text{x}^2}
Q182. Find A and B so that \text{y}=\text{A}\sin3\text{x}+\text{B}\cos3\text{x} satisfy the equation \frac{\text{d}^2\text{y}}{\text{dx}^2}+4\frac{\text{dy}}{\text{dx}}+3\text{y}=10\cos3\text{x}. 5 Marks

https://bls.smartstudies.co.in/#/exam/pdf-preview/c59cb220-8e86-4716-9ff7-82aec16b1ade/1 49/158
5/26/24, 6:19 PM Exam Automation
Ans: \text{y}=\text{Ae}^{-kt}\cos(\text{pt}+\text{c})
Differentiating w.r.t.x,
\Rightarrow\frac{\text{dy}}{\text{dt}}=\text{A}\Big\{\text{e}^{-\text{kt}}(-\sin(\text{pt}+\text{c})\times\text{p})+(\cos(\text{pt}+{c}))(-\text{re}^{-\text{kt}})\Big\}
\Rightarrow-\text{Ape}^{-\text{kt}}\sin(\text{pt}+\text{c})-\text{KAe}^{-\text{kt}}\cos(\text{pt}+\text{c})
\Rightarrow\frac{\text{dy}}{\text{dt}}=-\text{Ape}^{-\text{kt}}\sin(\text{pt}+\text{c})-\text{ky}
Differentiating w.r.t.x,
\Rightarrow\frac{\text{d}^2\text{y}}{\text{dt}^2}=-\text{Ap}\Big\{\text{e}^{-\text{kt}}(\cos(\text{pt}+\text{c})\times\text{p})+(\sin(\text{pt}+\text{c}))(\text{e}^{-\text{kt}}\times-\text{R})-
\text{ky}^1\Big\}
=-\text{p}^2\text{y}+\text{Apke}^{-\text{kt}}\sin(\text{pt}+\text{c})-\text{ky}^1
Adding & substracting ky1 on RHS
\Rightarrow\frac{\text{d}^2\text{y}}{\text{dt}^2}=+\text{Apke}^{-\text{kt}}\sin(\text{pt}+\text{c})-\text{p}^2\text{y}-2\text{ky}^1+\text{ky}^1
\frac{\text{d}^2\text{y}}{\text{dt}^2}=\text{Apke}^{-\text{kt}}\sin(\text{pt}+\text{c})-\text{p}^2\text{y}-2\text{ky}^1-\text{k}\text{ape}^{-\text{kt}}\sin(\text{pt}+\text{c})-\text{k}^2\text{y}
\Rightarrow\frac{\text{d}^2\text{y}}{\text{dt}^2}=-(\text{p}+\text{k}^2)\text{y}-2\text{k}\frac{\text{dy}}{\text{dx}}
\Rightarrow\frac{\text{d}^2\text{y}}{\text{dt}^2}+2\text{k}\frac{\text{dy}}{\text{dt}}+\text{n}^2\text{y}=0
Q183. Find the points on the curve \text{y}=(\cos\text{x}-1) in [0, 2\pi], where the tangent is parallel to x-axis. 5 Marks

Ans: The equation of the curve is \text{y}=\cos\text{x}-1.


Now, we have to find a point on the curve in [0,2\pi].
where the tangent is parallel to X-axis i.e., the tangent to the curve at x = c has a slope o,
where \text{c}\in[0,2\pi].
Let us apply Rolle’s theorem to get the point.
1. \text{y}=\cos\text{x}-1 is a continuous function in [0,2\pi].
[since it is a combination of cosine function and a constant function]
2. \text{y}'=-\sin\text{x}, which exists in (0,2\pi).
Hence, y is differentiable in (0,2\pi).
3. \text{y}(0)=\cos0-1=0 and \text{y}(2\pi)=\cos2\pi-1=0
\therefore\ \text{y}(0)=\text{y}(2\pi)
Since, conditions of Rolle’s theorem are satisfied.
Hence, there exists a real number c such that
\text{f}'(\text{c})=0
\Rightarrow-\sin\text{c}=0
\Rightarrow\ \text{c}=\pi or 0, where \pi\in(0,2\pi)
\Rightarrow\ \text{x}=\pi
\therefore\ \text{y}=\cos\pi-1=-2
Hence, the required point on the curve, where the tangent drawn is parallel to the X-axis is (\pi,2).
Q184. Find \frac{\text{dy}}{\text{dx}} 5 Marks
\text{y}=(\sin\text{x})^{\cos\text{x}}+(\cos\text{x})^{\sin\text{x}}

Ans: We have, \text{y}=(\sin\text{x})^{\cos\text{x}}+(\cos\text{x})^{\sin\text{x}}


\Rightarrow\text{y}=\text{e}^{\log(\sin\text{x})^{\cos\text{x}}}+\text{e}^{\log(\cos\text{x})^{\sin\text{x}}}
\Rightarrow\text{y}=\text{e}^{\cos\text{x}\log\sin\text{x}}+\text{e}^{\sin\text{x}\log\cos\text{x}}
Differentiating with respect to x,
\frac{\text{dy}}{\text{dx}}=\frac{\text{d}}{\text{dx}}\big(\text{e}^{\cos\text{x}\log\sin\text{x}}\big)+\frac{\text{d}}{\text{dx}}\big(\text{e}^{\sin\text{x}\log\cos\text{x}}\big)
=\text{e}^{\cos\text{x}\log\sin\text{x}}\frac{\text{d}}{\text{dx}}\big(\cos\text{x}\log\sin\text{x})+\text{e}^{\sin\text{x}\log\cos\text{x}}\frac{\text{d}}{\text{dx}}(\sin\text{x}\log\cos\text{x})
=\text{e}^{\log(\sin\text{x})^{\cos\text{x}}}\Big[\cos\text{x}\frac{\text{d}}{\text{dx}}\log\sin\text{x}+\log\sin\text{x}\frac{\text{d}}{\text{dx}}(\cos\text{x})\Big] \\
+\text{e}^{\log(\cos\text{x})^{\sin\text{x}}}\Big[\sin\text{x}\frac{\text{d}}{\text{dx}}\log\cos\text{x}+\log\cos\text{x}\frac{\text{d}}{\text{dx}}(\sin\text{x})\Big]
=(\sin\text{x})^{\cos\text{x}}\Big[\cos\text{x}\frac{1}{\sin\text{x}}\frac{\text{d}}{\text{dx}}(\sin\text{x})+\log\sin\text{x}\times(-\sin\text{x})\Big] \\ +
(\cos\text{x})^{\sin\text{x}}\big[\sin\text{x}\frac{1}{\cos\text{x}}\frac{\text{d}}{\text{dx}}(\cos\text{x})+\log\cos\text{x}\times(\cos\text{x})\Big]
=(\sin\text{x})^{\cos\text{x}}\big[\cot\text{x}\cos\text{x}-\sin\text{x}\log\sin\text{x}\big] \\ +(\cos\text{x})^{\sin\text{x}}\big[\tan\text{x}(-\sin\text{x})+\cos\text{x}\log\cos\text{x}\big]
=(\sin\text{x})^{\cos\text{x}}\big[\cot\text{x}\cos\text{x}-\sin\text{x}\log\sin\text{x}\big] \\ +(\cos\text{x})^{\sin\text{x}}\big[\cos\text{x}\log\cos\text{x}-\sin\text{x}\tan\text{x}\big]
Q185. Let x = f(t) and y = g(t) be parametric forms with t as a parameter, then 5 Marks
\frac{\text{dy}}{\text{dx}}=\frac{\text{dy}}{\text{dt}}\times\frac{\text{dt}}{\text{dx}}=\frac{\text{g}'(\text{t})}{\text{f}'(\text{t})}, where \text{f}'(\text{t})\neq0.
On the basis of above information, answer the following questions.
1. The derivative of \text{f}(\tan\text{x})\text{w.r.t.}\text{ g}(\sec\text{x})\text{ at}\text{ x}=\frac{\pi}{4}, where f'(1) = 2 and \text{g}'(\sqrt{2})=4, is:
1. \frac{1}{\sqrt{2}}
2. {\sqrt{2}}
3. 1
4. 0
2. The derivative of \sin^{-1}\Big(\frac{2\text{x}}{1+\text{x}^2}\Big) with respect to \cos^{-1}\Big(\frac{1-\text{x}^2}{1+\text{x}^2}\Big) is:
1. -1
2. 1
3. 2
4. 4
3. The derivative of \text{e}^{\text{x}^3} with respect to log x is:
1. \text{e}^{\text{x}^3}
2. 3\text{x}^22\text{e}^{\text{x}^3}
3. 3\text{x}^3\text{e}^{\text{x}^3}
4. 3\text{x}^2\text{e}^{\text{x}^3}+3\text{x}
4. The derivative of \cos^{-1}(2\text{x}^2-1)\text{w.r.t.}\cos^{-1}\text{x} is:
1. 2
2. \frac{-1}{2\sqrt{1-\text{x}^2}}
3. \frac{2}{\text{x}}
4. 1-\text{x}^2
5. If \text{y}=\frac{1}{4}\mu^4 and \mu=\frac{2}{3}\text{x}^3+5, then \frac{\text{dy}}{\text{dx}}=
1. \frac{2}{27}\text{x}^2(2\text{x}^3+15)^3
2. \frac{2}{7}\text{x}^2(2\text{x}^3+15)^3
3. \frac{2}{27}\text{x}(2\text{x}^3+5)^3
4. \frac{2}{7}(2\text{x}^3+15)^3

Ans: 1. (a) \frac{1}{\sqrt{2}}


Solution:
Now, \frac{\text{df}(\tan\text{x})}{\text{dg}(\sec\text{x})}=\frac{\text{f}'(\tan \text{x})\sec^2\text{x}}{\text{g}'(\sec\text{x})\sec\text{x}\tan \text{x}}
=\frac{\text{f}'(\tan \text{x})\sec\text{x}}{\text{g}'(\sec\text{x})\tan \text{x}}
\therefore\Big[\frac{\text{df}(\tan\text{x})}{\text{dg}(\sec\text{x})}\Big]_{\text{x}=\frac{\pi}{4}}=\frac{\text{f}'(1)\sqrt{2}}{\text{g}'(\sqrt{2})\cdot1}=\frac{2\sqrt{2}}{4\cdot1}=\frac{1}
{\sqrt{2}}
2. (b) 1
3. (c) 3\text{x}^3\text{e}^{\text{x}^3}

https://bls.smartstudies.co.in/#/exam/pdf-preview/c59cb220-8e86-4716-9ff7-82aec16b1ade/1 50/158
5/26/24, 6:19 PM Exam Automation
Solution:
Let \text{y}=\text{e}^{\text{x}^3},\text{z}=\log\text{x}
Differentiating w.r.t. x, we get
\frac{\text{dy}}{\text{dx}}=\text{e}^{\text{x}^3}(3\text{x}^2)=3\text{x}^2\text{e}^{\text{x}^3} and \therefore\frac{\text{dy}}{\text{dz}}=\frac{\frac{\text{dy}}{\text{dx}}}{\frac{\text{dz}}
{\text{dx}}}=\frac{3\text{x}^2\text{e}^{\text{x}^3}}{\Big(\frac{1}{\text{x}}\Big)}=3\text{x}^3\text{e}^{\text{x}^3}
4. (a) 2
Solution:
Let \text{y}=\cos^{-1}(2\text{x}^2-1)=2\cos^{-1}\text{x}
Differentiating w.r.t. \cos^{-1}\text{x}, we get
\frac{\text{dy}}{\text{d}(\cos^{-1}\text{x})}=\frac{2\text{d}(\cos^{-1}\text{x})}{\text{d}(\cos^{-1}\text{x})}=2
5. (a) \frac{2}{27}\text{x}^2(2\text{x}^3+15)^3
Solution:
We have, \text{y}=\frac{1}{4}\text{u}^4\Rightarrow\frac{\text{dy}}{\text{du}}=\frac{1}{4}\cdot4\text{u}^3=\text{u}^3
and \text{u}=\frac{2}{3}\text{x}^3+5\Rightarrow\frac{\text{du}}{\text{dx}}=\frac{2}{3}\cdot3\text{x}^2=2\text{x}^2
\therefore\frac{\text{dy}}{\text{dx}}=\frac{\text{dy}}{\text{du}}\cdot\frac{\text{du}}{\text{dx}}=\text{u}^3\cdot2\text{x}^2=\Big(\frac{2}{3}\text{x}^3+5\Big)^3(2\text{x})^2
=\frac{2}{27}\text{x}^2(2\text{x}^3+15)^3
Q186. If \text{y}=\tan^{-1}\Big(\frac{2\text{x}}{1-\text{x}^2}\Big)+\sec^{-1}\Big(\frac{1+\text{x}^2}{1-\text{x}^2}\Big),\text{x}>0, prove that \frac{\text{dy}}{\text{dx}}=\frac{4}{1+\text{x}^2} 5 Marks

Ans: Here, \text{y}=\tan^{-1}\Big(\frac{2\text{x}}{1-\text{x}^2}\Big)+\sec^{-1}\Big(\frac{1+\text{x}^2}{1-\text{x}^2}\Big)


\Rightarrow \text{y}=\tan^{-1}\Big(\frac{2\text{x}}{1-\text{x}^2}\Big)+\cos^{-1}\Big(\frac{1-\text{x}^2}{1+\text{x}^2}\Big)
Put \text{x}=\tan\theta
\therefore \text{y}=\tan^{-1}\Big(\frac{2\tan\theta}{1-\tan^2\theta}\Big)+\cos^{-1}\Big(\frac{1-\tan^{2}\theta}{1+\tan^{2}\theta}\Big)
\Rightarrow \text{y}=\tan^{-1}(\tan2\theta)+\cos^{-1}(\cos2\theta)
\Rightarrow \text{y}=2\theta+2\theta
\Rightarrow \text{y}=4\theta
\Rightarrow \text{y}=4\tan^{-1} \text{x} \big[\text{using, x}=\tan\theta\big]
Differentiate it with respect to x,
\therefore \frac{\text{dy}}{\text{dx}}=\frac{4}{1+\text{x}^2}
Q187. Find \frac{\text{dy}}{\text{dx}} 5 Marks
\text{y}=\text{x}^{\text{x}}+\text{x}^\frac{1}{\text{x}}

Ans: Here,
\text{y}=\text{x}^{\text{x}}+\text{x}^\frac{1}{\text{x}}
=\text{e}^{\log\text{x}^\text{x}}+\text{e}^{\log\text{x}^\frac{1}{\text{x}}}
\text{y}=\text{e}^{\text{x}\log\text{x}}+\text{e}^{\big(\frac{1}{\text{x}}\log\text{x}\big)}
\big[\text{Since, e}^{\log\text{a}}=\text{a},\log\text{a}^\text{b}=\text{b}\log\text{a}\big]
Differentiating it with respect to x using chain rule and product rule,
\frac{\text{dy}}{\text{dx}}=\frac{\text{d}}{\text{dx}}\big(\text{e}^{\text{x}\log\text{x}}\big)+\frac{\text{d}}{\text{dx}}\Big(\text{e}^{\frac{1}{\text{x}}\log\text{x}}\Big)
=\text{e}^{\text{x}\log\text{x}}+\frac{\text{d}}{\text{dx}}(\text{x}\log\text{x})+\text{e}^{\frac{1}{\text{x}}\log\text{x}}\frac{\text{d}}{\text{dx}}\Big(\frac{1}{\text{x}}\log\text{x}\Big)
=\text{e}^{\text{x}\log\text{x}}\Big[\text{x}\frac{\text{d}}{\text{dx}}(\log\text{x})+\log\text{x}\frac{\text{d}}{\text{dx}}(\text{x})\Big] \\ +\text{e}^{\log\text{x}^\frac{1}{\text{x}}}\Big[\frac{1}
{\text{x}}\frac{\text{d}}{\text{dx}}(\log\text{x})+\log\text{x}\frac{\text{d}}{\text{dx}}\big(\frac{1}{\text{x}}\big)\Big]
=\text{x}^{\text{x}}\Big[\text{x}\Big(\frac{1}{\text{x}}\Big)+\log\text{x}(1)\Big] \\ +\text{x}^\frac{1}{\text{x}}\Big[\Big(\frac{1}{\text{x}}\Big)\Big(\frac{1}{\text{x}}\Big)+\log\text{x}\Big(-
\frac{1}{\text{x}^2}\Big)\Big]
=\text{x}^\text{x}[1+\log\text{x}]+\text{x}^{\frac{1}{\text{x}}}\Big(\frac{1}{\text{x}^2}-\frac{1}{\text{x}^2}\log\text{x}\Big)
\frac{\text{dy}}{\text{dx}}=\text{x}^\text{x}[1+\log\text{x}]+\text{x}^{\frac{1}{\text{x}}}\frac{(1-\log\text{x})}{\text{x}^2}
Q188. Differentiate the following functions with respect to x: 4 Marks
\cos^{-1}\Big\{\sqrt{\frac{1+\text{x}}{2}}\Big\},-1<\text{x}<1

Ans: Let \text{y}=\cos^{-1}\Big\{\sqrt{\frac{1+\text{x}}{2}}\Big\}


Put \text{x}=\cos2\theta
\text{y}=\cos^{-1}\Big\{\sqrt{\frac{1+\cos2\theta}{2}}\Big\}
=\cos^{-1}\Big\{\sqrt{\frac{2\cos^2\theta}{2}}\Big\}
\text{y}=\cos^{-1}\{\cos\theta\}
Here, -1<\text{x}<1
\Rightarrow\ -1<\cos2\theta<1
\Rightarrow\ 0<2\theta<\pi
\Rightarrow\ 0<\theta<\frac{\pi}{2}
So, equation (i),
\text{y}=\theta
\big[\text{Since}, \cos^{-1}(\cos\theta)=\theta\text{ if }\theta\in[0,\pi]\big]
\text{y}=\frac{1}{2}\cos^{-1}\text{x}\ \big[\text{Since x}=\cos2\theta\big]
Differentiating it with respect to x,
\frac{\text{dy}}{\text{dx}}=-\frac{1}{2\sqrt{1-\text{x}^2}}
Q189. If \text{f}\text{(x)}=\begin{cases}\text{e}^\frac{1}{\text {x}}, & \text{if} \text{ x}\neq 0\\1, & \text{if}\text{x} = 0\end{cases} find whethe f is continuous at x = 0. 4 Marks

Ans: Given,
\text{f}\text{(x)}=\begin{cases}\text{e}^\frac{1}{\text {x}}, & \text{if} \text{ x}\neq 0\\1, & \text{if}\text{x} = 0\end{cases}
We observe
(\text{LHL at x}=0)=\lim\limits_{\text{x} \rightarrow 0^-}\text{f}\text{(x)}=\lim\limits_{\text{h} \rightarrow 0}\text{f}(0-\text{x)}=\lim\limits_{\text{h} \rightarrow 0}\text{f}\text{(-h)}
=\lim\limits_{\text{h} \rightarrow 0}\text{e}^\frac{-1}{\text{h}}=\lim\limits_{\text{h} \rightarrow 0}\bigg(\frac{1}{\text{e}^{\frac{1}{\text{h}}}}\bigg)=\frac{1}{\lim\limits_{\text{h}
\rightarrow 0}\text{e}^{\frac{1}{\text{h}}}}=0
(\text{RHL at x}=0)=\lim\limits_{\text{h} \rightarrow 0^+}\text{f}(\text{x})=\lim\limits_{\text{h} \rightarrow 0}\text{f}(\text{h})
=\lim\limits_{\text{h} \rightarrow 0^+}\text{e}^\frac{1}{\text{h}}=\infty
Given,
\text{f}(0)=1
It is known for a function f(x) to be continuous at x = a,
\lim\limits_{\text{x} \rightarrow \text{a}^-}\text{f}\text{(x)}=\lim\limits_{\text{x} \rightarrow \text{a}^+}\text{f}\text{(x)}=\text{f}\text{(a)}
But here,
\lim\limits_{\text{x} \rightarrow \text{a}^-}\text{f}\text{(x)}\neq\lim\limits_{\text{x} \rightarrow \text{0}^+}\text{f}\text{(x)}
Hence, f(x) is discontinuous at x = 0.
Q190. If \text{y}=\log\sqrt{\text{x}+1}+\sqrt{\text{x}-1}, prove that \sqrt{\text{x}^2-1}\frac{\text{dy}}{\text{dx}}=\frac{1}{2}\text{y}. 4 Marks

Ans: Consider \text{y}=\cos(\log\text{ x})^2


Differentiating it with respect to x and applying the chain and the product rule, we get
\frac{\text{dy}}{\text{dx}}=\frac{\text{d}}{\text{dx}}\sqrt{\text{x}+1}+\frac{\text{d}}{\text{dx}}\sqrt{\text{x}-1}
=\frac{1}{2}(\text{x}+1)^\frac{-1}{2}+\frac{1}{2}(\text{x}-1)^\frac{-1}{2}
=\frac{1}{2}\Big(\frac{1}{\sqrt{\text{x}+1}}\frac{1}{\sqrt{\text{x}-1}}\Big)
=\frac{1}{2}\bigg(\frac{\sqrt{\text{x}-1}+\sqrt{\text{x}+1}}{\big(\sqrt{\text{x}+1}\big)\big(\sqrt{\text{x}-1}\big)}\bigg)

https://bls.smartstudies.co.in/#/exam/pdf-preview/c59cb220-8e86-4716-9ff7-82aec16b1ade/1 51/158
5/26/24, 6:19 PM Exam Automation
\frac{\text{dy}}{\text{dx}}=\frac{1}{2}\Big(\frac{\text{y}}{\sqrt{\text{x}^2-1}}\Big)
So,
\sqrt{\text{x}^2}-\frac{\text{dy}}{\text{dx}}=\frac{1}{2}\text{y}
Q191. Differentiate \sin^{-1}\Big\{\frac{2^{\text{x}+1}\times3^\text{x}}{1+(36)^\text{x}}\Big\} with respect to x: 4 Marks

Ans: We have \text{y}=\sin^{-1}\Big\{\frac{2^{\text{x}+1}\times3^\text{x}}{1+(36)^\text{x}}\Big\}


\Rightarrow \text{y}=\sin^{-1}\Big\{\frac{2\times6^\text{x}}{1+6^{2\text{x}}}\Big\}
Put 6^\text{x}=\tan\theta
\Rightarrow \theta=\tan^{-1}(6^\text{x})
Now, \text{y}=\sin^{-1}\Big\{\frac{2\tan\theta}{1+\tan^2\theta}\Big\}
\Rightarrow \text{y}=\sin^{-1}\big\{\sin2\theta\big\}
\Rightarrow \text{y}=2\theta
\Rightarrow \text{y}=2\tan^{-1}(6^\text{x})
\Rightarrow \frac{\text{dy}}{\text{dx}}=2\times\frac{1}{(6^\text{x})^2}\times6^\text{x}\log6
\therefore\frac{\text{dy}}{\text{dx}}=\frac{2(\log6)6^\text{x}}{36^\text{x}}
Q192. Find \frac{\text{dy}}{\text{dx}} in the following cases: 4 Marks
x5 + y5 = 5xy

Ans: We Heve, x5 + y5 = 5xy


Differentiating with respect to x, we get,
\frac{\text{d}}{\text{dx}}\big(\text{x}^5\big)+\frac{\text{d}}{\text{dx}}\big(\text{y}^5\big)=\frac{\text{d}}{\text{dx}}\big(5\text{xy}\big)
\Rightarrow5\text{x}^4+5\text{y}^4\frac{\text{dy}}{\text{dx}}=5\Big[\text{x}\frac{\text{dy}}{\text{dx}}+\text{y}\frac{\text{dy}}{\text{dx}}\big(\text{x}\big)\Big]
\Rightarrow5\text{x}^4+5\text{y}^4\frac{\text{dy}}{\text{dx}}=5\Big[\text{x}\frac{\text{dy}}{\text{dx}}+\text{y}\big(1\big)\Big]
\Rightarrow5\text{x}^4+5\text{y}^4\frac{\text{dy}}{\text{dx}}=5\text{x}\frac{\text{dy}}{\text{dx}}+5\text{y}
\Rightarrow5\text{y}^4\frac{\text{dy}}{\text{dx}}-5\text{x}\frac{\text{dy}}{\text{dx}}=5\text{y}-5\text{x}^4
\Rightarrow5\frac{\text{dy}}{\text{dx}}\big(\text{y}^4-\text{x}\big)=5\big(\text{y}-\text{x}^4\big)
\Rightarrow\frac{\text{dy}}{\text{dx}}=\frac{5(\text{y}-\text{x}^4)}{5(\text{y}^4-\text{x})}
\Rightarrow\frac{\text{dy}}{\text{dx}}=\frac{\text{y}-\text{x}^4}{\text{y}^4-\text{x}}
Q193. Is the function f defined by 4 Marks
\text{f(x)} = \begin{cases}\text{x}, \text{if}\ \text{x}\leq1\\5, \text{if}\ \text{x} > 1\end{cases}
continuous at x = 0? At x = 1? At x = 2?

Ans: Here \text{f(x)} = \begin{cases}\text{x}, \text{if}\ \text{x}\leq1\\5, \text{if}\ \text{x} > 1\end{cases}
At x = 0
^{\ \ \text{Lt}}_{\text{x}\rightarrow\text{0}^{-}}\text{f(x)} = ^{\ \ \text{Lt}}_{\text{x}\rightarrow\text{0}}\text{(x)}\ [ \text{Put}\ \text{x} = 0 - \text{h}, \text{h}>0\ \text{so that}\
\text{h}\rightarrow 0\ \text{as}\ \text{x}\rightarrow 0^-]
=\ ^{\ \ \text{Lt}}_{\text{h}\rightarrow\text{0}}(0-\text{h})
=\ ^{\ \ \text{Lt}}_{\text{h}\rightarrow\text{0}}\text{(-h)} = (0) = 0
^{\ \ \text{Lt}}_{\text{x}\rightarrow\text{0}^{+}}\text{f(x)} = ^{\ \ \text{Lt}}_{\text{x}\rightarrow\text{0}^{+}}\text{(x)}\ [ \text{Put}\ \text{x} = 0 + \text{h}, \text{h}>0\ \text{so that}\
\text{h}\rightarrow 0\ \text{as}\ \text{x}\rightarrow 0^+]
=\ ^{\ \ \text{Lt}}_{\text{h}\rightarrow\text{0}}(0 + \text{h})
=0+0=0
\therefore\ ^{\ \ \text{Lt}}_{\text{x}\rightarrow\text{0}^{-}}\text{f(x)} = ^{\ \ \text{Lt}}_{\text{x}\rightarrow\text{0}^{+}}\text{f(x)} = 0
\therefore \ ^{\ \ \text{Lt}}_{\text{x}\rightarrow\text{0}}\text{f(x)} = 0
Also f(0) = value of x at x = 0
=0
\therefore f is continous at x = 0.
At x = 1
^{\ \ \text{Lt}}_{\text{x}\rightarrow\text{1}^{-}}\text{f(x)} = ^{\ \ \text{Lt}}_{\text{x}\rightarrow\text{1}^{-}}\text{x}\ [ \text{Put}\ \text{x} = 1 - \text{h}, \text{h}>0,\ \text{so that}\
\text{h}\rightarrow 0\ \text{on}\ \text{x}\rightarrow 1^-]
^{\ \ \text{Lt}}_{\text{h}\rightarrow\text{0}}(1 - \text{h}) = 1 - 0 = 1
^{\ \ \text{Lt}}_{\text{x}\rightarrow\text{1}^{+}}\text{f(x)} = ^{\ \ \text{Lt}}_{\text{x}\rightarrow\text{1}^{+}}(5) = 5
\therefore\ ^{\ \ \text{Lt}}_{\text{x}\rightarrow\text{1}^{-}}\text{f(x)}\neq\ ^{\ \ \text{Lt}}_{\text{x}\rightarrow\text{1}^{+}}\text{f(x)}
\therefore \ ^{\ \ \text{Lt}}_{\text{x}\rightarrow\text{1}^{-}}\text{f(x)} does not exist
\therefore f is discontinuous at x =1.
At x = 2
^{\ \ \text{Lt}}_{\text{x}\rightarrow\text{2}}\text{f(x)} = ^{\ \ \text{Lt}}_{\text{x}\rightarrow\text{2}}(5) = 5
Also f(2) = 5
\therefore\ ^{\ \ \text{Lt}}_{\text{x}\rightarrow\text{2}}\text{f(x)} = \text{f(2)} = 5
\therefore f is continous at x = 2.
Q194. If \text{x}=\cos\text{t}(3-2\cos^2\text{t}),\text{y}\sin\text{t}(3-2\sin^2\text{t}) find the value of \frac{\text{dy}}{\text{dx}}\text{ at t}=\frac{\pi}{4} 4 Marks

Ans: \text{x}=\cos\text{t}(3-2\cos^2\text{t})\text{ and }\text{y}\sin\text{t}(3-2\sin^2\text{t})


\Rightarrow\frac{\text{dx}}{\text{dt}}=-\sin\text{t}(3-2\cos^2\text{t})+\cos\text{t}(4\cos\text{t}\sin\text{t}) and \frac{\text{dy}}{\text{dx}}=\cos\text{t}(3-2\sin^2\text{t})+\sin\text{t}
(-4\sin\text{t}\cos\text{t})
\Rightarrow\frac{\text{dy}}{\text{dx}}=-\sin\text{t}+6\sin\text{t}\cos^2\text{t} and \frac{\text{dy}}{\text{dx}}=3\cos\text{t}-6\sin^2\text{t}\cos\text{t}
\Rightarrow\frac{\text{dy}}{\text{dx}}=-\sin\text{t}+(1-2\cos^2\text{t}) and \frac{\text{dy}}{\text{dt}}=3\cos\text{t}(1-2\sin^2\text{t})
\Rightarrow\frac{\text{dx}}{\text{dt}}=3\sin\text{t}\cos2\text{t}\cos2\text{t} and \frac{\text{dy}}{\text{dt}}=3\cos\text{t}(1-2\sin^2\text{t})
\therefore\frac{\text{dy}}{\text{dx}}=\frac{\frac{\text{dy}}{\text{dx}}}{\frac{\text{dx}}{\text{dt}}}=\frac{3\cos\text{t}\cos2\text{t}}{3\sin\text{t}\cos2\text{t}}=\cot\text{t}
Now, \Big(\frac{\text{dy}}{\text{dt}}\Big)_{\text{t}=\frac{\pi}{4}}=\cot\frac{\pi}{4}=1
Q195. If \text{f}\text{(x)}=\begin{cases}\frac{1-\cos\text{kx}}{\text{x}\sin\text{x}}, & \text{x} \neq 0\\\frac{1}{2}, & \text{x}= 0\end{cases} is continuous at x = 0. find k. 4 Marks

Ans: Given,
\text{f}\text{(x)}=\begin{cases}\frac{1-\cos\text{kx}}{\text{x}\sin\text{x}}, & \text{x} \neq 0\\\frac{1}{2}, & \text{x}= 0\end{cases}
If f(x) is continuous at x = 0, then
\lim\limits_{\text{x} \rightarrow 0}\text{f}\text{(x)}=\text{f}(0)\dots(\text{i})
Consider:
\lim\limits_{\text{x} \rightarrow 0}\text{f}\text{(x)}=\lim\limits_{\text{x} \rightarrow 0}\Bigg(\frac{1-\cos\text{kx}}{\text{x}\sin\text{x}}\Bigg)=\lim\limits_{\text{x} \rightarrow
0}\Bigg(\frac{2\sin^2\frac{\text{kx}}{2}}{\text{x}\sin\text{x}}\Bigg)
\Rightarrow\lim\limits_{\text{x} \rightarrow 0}\text{f}\text{(x)}=\lim\limits_{\text{x} \rightarrow 0}\begin{pmatrix}\frac{2\sin^2\frac{\text{kx}}{2}}{\text{x}^2\Big(\frac{\sin\text{x}}
{\text{x}}\Big)}\end{pmatrix}
\Rightarrow\lim\limits_{\text{x} \rightarrow 0}\text{f}\text{(x)}=\lim\limits_{\text{x} \rightarrow 0}\begin{pmatrix}\frac{\frac{2\text{k}^2}{4}\Big(\sin\frac{\text{kx}}{\text{x}}\Big)^2}
{\Big(\frac{\text{kx}}{2}\Big)^2\Big(\frac{\sin\text{x}}{\text{x}}\Big)}\end{pmatrix}
\Rightarrow\lim\limits_{\text{x} \rightarrow 0}\text{f}\text{(x)}=\frac{2\text{k}^2}{4}\begin{pmatrix}\frac{\lim\limits_{\text{x} \rightarrow 0}\frac{\Big(\sin\frac{\text{kx}}{2}\Big)^2}
{\Big(\frac{\text{kx}}{2}\Big)^2}}{\lim\limits_{\text{x} \rightarrow 0}\frac{\sin\text{x}}{\text{x}}}\end{pmatrix}
\Rightarrow\lim\limits_{\text{x} \rightarrow 0}\text{f}\text{(x)}=\frac{2\text{k}^2}{4}\times1=\frac{\text{k}^2}{2}
From equation (i), we have
\frac{\text{k}^2}{2}=\text{f}(0)

https://bls.smartstudies.co.in/#/exam/pdf-preview/c59cb220-8e86-4716-9ff7-82aec16b1ade/1 52/158
5/26/24, 6:19 PM Exam Automation
\Rightarrow\frac{\text{k}^2}{2}=\frac{1}{2}
\Rightarrow\text{k}=\pm1
Q196. Find the points of discontinuity, if any of the following function: 4 Marks
\text{f(x)}=\begin{cases}2\text{x},&\text{ if}\text{ x}<0\\0,&\text{if }0\leq\text{x}\leq1\\4\text{x},&\text{if }\text{ x}>1\end{cases}

Ans: The given function is


\text{f(x)}=\begin{cases}2\text{x},&\text{ if}\text{ x}<0\\0,&\text{if }0\leq\text{x}\leq1\\4\text{x},&\text{if }\text{ x}>1\end{cases}
The given function is defined at all points of the real line.
Let c be a point on the real line.
Case I:
If c < 0 then f(c) = 2c
\lim_\limits{\text{x}\rightarrow\text{c}}\text{f(x)}=\lim_\limits{\text{x}\rightarrow\text{c}}(2\text{x})=2\text{c}
\therefore\ \lim_\limits{\text{x}\rightarrow\text{c}}\text{f(x)}=\text{f(c)}
Therefore, f is continuous at all points x, such that x < 0
Case II:
If c = 0, then f(c) = f(0) = 0
The left hand limit of f at x = 0 is,
\lim_\limits{\text{x}\rightarrow0^-}\text{f(x)}=\lim_\limits{\text{x}\rightarrow0^-}(2\text{x})=2\times0=0
The right hand limit of at x = 0 is,
\lim_\limits{\text{x}\rightarrow0^+}\text{f(x)}=\lim_\limits{\text{x}\rightarrow0^+}(0)=0
\therefore\ \lim_\limits{\text{x}\rightarrow0^-}\text{f(x)}=\text{f}(0)
Therefore, f is continuous at x = 0
Case III:
If 0 < c < 1, then f(x) = 0 and \lim_\limits{\text{x}\rightarrow\text{c}}\text{f(x)}=\lim_\limits{\text{x}\rightarrow\text{c}}(0)=0
\therefore\ \lim_\limits{\text{x}\rightarrow\text{c}}\text{f(x)}=\text{f}(\text{c})
Therefore, if is continuous at all points of the interval (0, 1)
Case IV:
If c = 1, then f(c) = f(1) = 0
The left hand limit of at x = 1 is,
\lim_\limits{\text{x}\rightarrow1^-}\text{f(x)}=\lim_\limits{\text{x}\rightarrow1^-}(0)=0
The right hand limit of f at x = 1 is,
\lim_\limits{\text{x}\rightarrow1^-}\text{f(x)}=\lim_\limits{\text{x}\rightarrow1^-}(4\text{x})=4\times1=4
It is observes that the left and right hand limits of f at x = 1 do not coincide.
Therefore, f is not continuous at x = 1
Case V:
If c < 1, then f(c) = 4c and \lim_\limits{\text{x}\rightarrow\text{c}}\text{f(x)}=\lim_\limits{\text{x}\rightarrow\text{c}}\text{(4x)}=4\text{c}
\therefore\ \lim_\limits{\text{x}\rightarrow\text{c}}\text{f(x)}=\text{f(c)}
Therefore, f is continuouse at all points x such that x > 1
Hence, f is not continuous only at x = 1
Q197. Verify the Rolle’s theorem for each of the functions: 4 Marks
f(x) = x(x - 1)2 in [0, 1].

Ans: Consider, f(x) = x(x - 1)2 in [0, 1].


1. Since, f(x) = x(x - 1)2 is a polynomial function
So, it is continuous in [0, 1].
2. Now, f(0) = 0 and f(1) = 0 ⇒ f(0) = 1
f satisfies the above conditions of Rolle’s theorem.
Hence, by Rolle’s theorem \exists\text{ c}\in(0,1) such that
f(c) = 0
⇒ 3c2 - 4c + 1 = 0
⇒ 3c2 - 3c - c + 1 = 0
⇒ 3c(c - 1) -1(c - 1) = 0
⇒ (3c - 1)(c - 1) = 0
\Rightarrow\ \text{c}=\frac{1}{3},1\Rightarrow\frac{1}{3}\in(0,1)
Thus, we see that there exists a real number c in the open interval (0, 1).
Hence, Rolle’s theorem has been verified.
Q198. Find \frac{\text{dy}}{\text{ dx}} in the following: 4 Marks
\text{y}=\sin^{-1}\Bigg(\frac{{1}-\text{x}^{2}}{1+\text{x}^{2}}\Bigg), 0 <\text{x}<1

Ans: The given relationship is,\text{y}=\sin^{-1}\Bigg(\frac{{1}-\text{x}^{2}}{1+\text{x}^{2}}\Bigg)


\text{y}=\sin^{-1}\Bigg(\frac{{1}-\text{x}^{2}}{1+\text{x}^{2}}\Bigg)
\Rightarrow\sin\text{y}=\frac{{1}-\text{x}^{2}}{1+\text{x}^{2}}
Differentiating this relationship with respect to x, we obtain
\frac{\text{d}}{\text{dx}}\sin\text{y}=\frac{\text{d}}{\text{dx}}\Big[\frac{1-\text{x}^{2}}{1+\text{x}^{2}}\Big]
\cos\text{y}.\frac{\text{dy}}{\text{dx}}=\frac{(1+\text{x}^2)(-2\text{x})-(1-\text{x}^2)\cdot2\text{x}}{(1+\text{x}^2)^2}
\Rightarrow\sqrt{1-\sin^2\text{y}}\frac{\text{dy}}{\text{dx}}=\frac{-2\text{x}-2\text{x}^3-2\text{x}+2\text{x}^3}{(1+\text{x}^2)^2}
\Rightarrow\sqrt{1-\Big(\frac{1-\text{x}^2}{1+\text{x}^2}\Big)^2}\frac{\text{dy}}{\text{dx}}=\frac{-4\text{x}^2}{(1+\text{x}^2)^2}
\Rightarrow\sqrt{1-\frac{(1-\text{x}^2)^2}{(1+\text{x}^2)^2}}\frac{\text{dy}}{\text{dx}}=\frac{-4\text{x}^2}{(1+\text{x}^2)^2}
\Rightarrow\sqrt{1-\frac{(1-\text{x}^2)^2-(1-\text{x}^2)^2}{(1+\text{x}^2)^2}}\frac{\text{dy}}{\text{dx}}=\frac{-4\text{x}^2}{(1+\text{x}^2)^2}
\Rightarrow\sqrt{\frac{1+\text{x}^4+2\text{x}^2-1-\text{x}^4+2\text{x}^2}{(1+\text{x}^2)^2}}\frac{\text{dy}}{\text{dx}}=\frac{-4\text{x}^2}{(1+\text{x}^2)^2}
\Rightarrow\sqrt{\frac{4\text{x}^2}{(1+\text{x}^2)^2}}\frac{\text{dy}}{\text{dx}}=\frac{-4\text{x}^2}{(1+\text{x}^2)^2}
\Rightarrow\frac{2\text{x}}{1+\text{x}^2}\frac{\text{dy}}{\text{dx}}=\frac{-4\text{x}^2}{(1+\text{x}^2)^2}
\Rightarrow\frac{\text{dy}}{\text{dx}}=\frac{-4\text{x}}{(1+\text{x}^2)2\text{x}}
\Rightarrow\frac{\text{dy}}{\text{dx}}=\frac{-2}{1+\text{x}^2}
Q199. If \text{x}=\text{a}(\theta+\sin\theta),\text{y}=\text{a}(1+\cos\theta), find \frac{\text{dy}}{\text{dx}}. 4 Marks

Ans: Here,
\text{x}=\text{a}(\theta+\sin\theta)
Differentiating it with respect to \theta,
\frac{\text{dx}}{\text{d}\theta}=\text{a}\Big(\frac{\text{d}}{\text{d}\theta}(\theta)+\frac{\text{d}}{\text{d}\theta}(\sin\theta)\Big)
\frac{\text{dx}}{\text{d}\theta}=\text{a}(1+\cos\theta)\ .....(\text{i})
And, \text{y}=\text{a}(1+\cos\theta)
Differentiating it with respect to \theta,
\frac{\text{dx}}{\text{d}\theta}=\text{a}(0-\sin\theta)
\frac{\text{dx}}{\text{d}\theta}=\text{a}\sin\theta\ .....(\text{ii})
Dividing equation (ii) by (i),
\frac{\frac{\text{dy}}{\text{d}\theta}}{\frac{\text{dx}}{\text{d}\theta}}=\frac{-\text{a}\sin\theta}{\text{a}(1+\cos\theta)}
=\frac{-\frac{2\sin\theta}{2}\frac{\cos\theta}{2}}{\frac{2\cos^2\theta}{2}}

https://bls.smartstudies.co.in/#/exam/pdf-preview/c59cb220-8e86-4716-9ff7-82aec16b1ade/1 53/158
5/26/24, 6:19 PM Exam Automation
\frac{\text{dy}}{\text{dx}}=-\frac{\tan\theta}{2}
Q200. If \log(\text{x}^2+\text{y}^2)=2\tan^{-1}\Big(\frac{\text{y}}{\text{x}}\Big), show that =\frac{\text{dy}}{\text{dx}}=\frac{\text{x}+\text{y}}{\text{x}-\text{y}}. 4 Marks

Ans: We have, \log(\text{x}^2+\text{y}^2)=2\tan^{-1}\Big(\frac{\text{y}}{\text{x}}\Big)


\Rightarrow\frac{1}{2}.\frac{\text{d}}{\text{dx}}\log(\text{x}^2+\text{y}^2)=\frac{\text{d}}{\text{dx}}\tan^{-1}\Big(\frac{\text{y}}{\text{x}}\Big)
\Rightarrow\frac{1}{2}\Big(\frac{1}{\text{x}^2+\text{y}^2}\Big)\frac{\text{d}}{\text{dx}}\big(\text{x}^2+\text{y}^2\big)=\frac{1}{1+\Big(\frac{\text{y}}{\text{x}}\Big)^2}\frac{\text{d}}
{\text{dx}}\Big(\frac{\text{y}}{\text{x}}\Big)
\Rightarrow\frac{1}{2}\Big(\frac{1}{\text{x}^2+\text{y}^2}\Big)\Big[2\text{x}+2\text{y}\frac{\text{dy}}{\text{dx}}\Big]=\frac{\text{x}^2}
{(\text{x}^2+\text{y}^2)}\bigg[\frac{\text{x}\frac{\text{dy}}{\text{dx}}-\text{y}\frac{\text{d}}{\text{dx}}(\text{x})}{\text{x}^2}\bigg]
\Rightarrow\Big(\frac{1}{\text{x}^2+\text{y}^2}\Big)\Big(\text{x}+\text{y}\frac{\text{dy}}{\text{dx}}\Big)=\frac{\text{x}^2}{(\text{x}^2+\text{y}^2)}\bigg[\frac{\text{x}\frac{\text{dy}}
{\text{dx}}-\text{y}\frac{\text{d}}{\text{dx}}(\text{x})}{\text{x}^2}\bigg]
\Rightarrow\Big(\frac{1}{\text{x}^2+\text{y}^2}\Big)\Big(\text{x}+\text{y}\frac{\text{dy}}{\text{dx}}\Big)=\frac{\text{x}^2}{(\text{x}^2+\text{y}^2)}\bigg[\frac{\text{x}\frac{\text{dy}}
{\text{dx}}-\text{y}(1)}{\text{x}^2}\bigg]
\Rightarrow\text{x}+\text{y}\frac{\text{dy}}{\text{dx}}=\text{x}\frac{\text{dy}}{\text{dx}}-\text{y}
\Rightarrow\text{y}\frac{\text{dy}}{\text{dx}}-\text{x}\frac{\text{dy}}{\text{dx}}=-\text{y}-\text{x}
\Rightarrow\frac{\text{dy}}{\text{dx}}(\text{y}-\text{x})=-(\text{y}+\text{x})
\Rightarrow\frac{\text{dy}}{\text{dx}}=\frac{-(\text{y}+\text{x})}{\text{y}-\text{x}}
\Rightarrow\frac{\text{dy}}{\text{dx}}=\frac{\text{x}+\text{y}}{\text{x}-\text{y}}
Q201. \text{If (cos x)}^{\text{y}}=\text{(cos y)}^{\text{x}},\text{find }\frac{\text{dy}}{\text{dx}}. 4 Marks

Ans: \text{(cos x)}^{\text{y}}=\text{(cos y)}^{\text{x}}\Rightarrow y log cos x = x log cos y.


\therefore\text{y}.\frac{\text{(-sin x)}}{\text{cos x}}+\text{log cos x.}\frac{\text{dy}}{\text{dx}}=\text{x}.\frac{\text{(-sin y)}}{\text{cos y}}\frac{\text{dy}}{\text{dx}}+\text{log cos y.}
(log cos x + x tan y) \frac{\text{dy}}{\text{dx}} = log cos y + y tan x
\therefore\frac{\text{dy}}{\text{dx}}= \frac{\text{log cos y + y tan x}}{\text{log cos x + x tan y}}.
Q202. Find all points of discontinuity of f, where f is defined by: 4 Marks
\text{f(x)}= \begin{cases}\text{x} + 1,\ \ \text{if x}\geq 1 \\\text{x}^2 + 1,\ \text{if x}<1\end{cases}

Ans: Here \text{f(x)}= \begin{cases}\text{x} + 1,\ \ \text{if x}\geq 1 \\\text{x}^2 + 1,\ \text{if x}<1\end{cases}
Function f is defined at all points of the real line.
Let c be any real number.
Three cases arise:
Case I: c < 1
^{\ \ \text{Lt}}_{\text{x}\rightarrow\text{c}}\text{f(x)} = ^{\ \ \text{Lt}}_{\text{x}\rightarrow\text{c}}\text{f(x}^2 + 1) = \text{c}^2 + 1
f(x) = c2 + 1
\therefore\ ^{\ \ \text{Lt}}_{\text{x}\rightarrow\text{c}}\text{f(x)} = \text{f(c)}
\therefore f is continuous at all points x < 1.
Case II: c > 1
^{\ \ \text{Lt}}_{\text{x}\rightarrow\text{c}}\text{f(x)} = ^{\ \ \text{Lt}}_{\text{x}\rightarrow\text{c}}\text{f(x} + 1) = \text{c} + 1
f(c) = c + 1
\therefore\ ^{\ \ \text{Lt}}_{\text{x}\rightarrow\text{c}}\text{f(x)} = \text{f(c)}
\therefore f is continuous at all points x > 1.
Case III: c = 1
^{\ \ \text{Lt}}_{\text{x}\rightarrow\text{1}^{-}}\text{f(x)} = ^{\ \ \text{Lt}}_{\text{x}\rightarrow\text{1}^{-}}\text{(x}^2 + 1) = 1 + 1 = 2
^{\ \ \text{Lt}}_{\text{x}\rightarrow\text{1}^{+}}\text{f(x)} = ^{\ \ \text{Lt}}_{\text{x}\rightarrow\text{1}^{+}}\text{(x}^2 + 1) = 1 + 1 = 2
f(1) = 1 + 1 = 2
\therefore\ ^{\ \ \text{Lt}}_{\text{x}\rightarrow\text{1}^{-}}\text{f(x)} = ^{\ \ \text{Lt}}_{\text{x}\rightarrow\text{1}^{+}}\text{f(x)} = \text{f(1)}
\therefore f is continuous at all points x < 1.
\therefore f is continuous at all point of domain.
Q203. Differentiate the following functions with respect to x: 4 Marks
\tan^{-1}\Big(\frac{5\text{x}}{1+6\text{x}^3}\Big), -\frac{1}{\sqrt{6}}<\text{x}<\frac{1}{\sqrt{6}}

Ans: Let \text{y}=\tan^{-1}\Big(\frac{5\text{x}}{1+6\text{x}^3}\Big)


=\tan^{-1}\Big(\frac{3\text{x}+2\text{x}}{1-(3\text{x})(2\text{x})}\Big)
\text{y}=\tan^{-1}(3\text{x})+\tan^{-1}(2\text{x})
\Big[\text{Since},\tan^{-1}\text{x}+\tan^{-1}\text{y}=\tan^{-1}\Big(\frac{\text{x}+\text{y}}{1-\text{xy}}\Big)\Big]
Differentiating it with respect to x using chain rule,
\frac{\text{dy}}{\text{dx}}=\frac{1}{1+(3\text{x})^2}\frac{\text{d}}{\text{dx}}(3\text{x})+\frac{1}{1+(2\text{x})^2}\frac{\text{d}}{\text{dx}}(2\text{x})
=\frac{1}{1+9\text{x}^2}(3)+\frac{1}{1+4\text{x}^2}(2)
\frac{\text{dy}}{\text{dx}}=\frac{3}{1+9\text{x}^2}+\frac{2}{1+4\text{x}^2}
Q204. Differentiate the following functions with respect to x: 4 Marks
\tan^{-1}\Big\{\frac{\text{x}}{\sqrt{\text{a}^2-\text{x}^2}}\Big\},-\text{a}<\text{x}<\text{a}

Ans: Let \text{y}=\tan^{-1}\Big\{\frac{\text{x}}{\sqrt{\text{a}^2-\text{x}^2}}\Big\}


Put \text{x}=\text{a}\sin\theta
\text{y}=\tan^{-1}\Big\{\frac{\text{a}\sin\theta}{\sqrt{\text{a}^2-\text{a}^2\sin^2\theta}}\Big\}
\text{y}=\tan^{-1}\bigg\{\frac{\text{a}\sin\theta}{\sqrt{\text{a}^2(1-\sin^2\theta)}}\bigg\}
\text{y}=\tan^{-1}\Big\{\frac{\text{a}\sin\theta}{\text{a}\cos\theta}\Big\}
\text{y}=\tan^{-1}(\tan\theta)\ .....(\text{i})
Here, -\text{a}<\text{x}<\text{a}
\Rightarrow-1<\frac{\text{x}}{\text{a}}<1
\Rightarrow -\frac{\pi}{2}<\theta<\frac{\pi}{2}
From equation (i),
\text{y}=\theta
\Big[\text{Since},\tan^{-1}(\tan\theta)=\theta,\text{ if }\theta \in\Big[-\frac{\pi}{2},\frac{\pi}{2}\Big]\Big]
\text{y}=\sin^{-1}\Big(\frac{\text{x}}{\text{a}}\Big)\big[\text{Since x}=\text{a }\sin \theta\big]
Differentiating it with respect to x,
\frac{\text{dy}}{\text{dx}}=\frac{1}{\sqrt{1-\big(\frac{\text{x}}{\text{a}}\big)^2}}\frac{\text{d}}{\text{dx}}\Big(\frac{\text{x}}{\text{a}}\Big)
=\frac{\text{a}}{\sqrt{\text{a}^2-\text{x}^2}}\times\big(\frac{1}{\text{a}}\big)
\frac{\text{dy}}{\text{dx}}=\frac{1}{\sqrt{\text{a}^2-\text{x}^2}}
Q205. Differentiate \sin^{-1}\Big(4\text{x}\sqrt{1-4\text{x}^2}\Big) with respect to \sqrt{1-4\text{x}^2}, if: 4 Marks
\text{x}\in\Big(-\frac{1}{2\sqrt{2}},\frac{1}{2}\Big)

Ans: Let \text{u}=\sin^{-1}\Big(4\text{x}\sqrt{1-4\text{x}^2}\Big)


Put 2\text{x}=\cos\theta \text{ So},
\Rightarrow\text{u}=\sin^{-1}\Big(2\times\cos\theta\sqrt{1-\cos^2\theta}\Big)
\Rightarrow\text{u}=\sin^{-1}(2\cos\theta\sin\theta)
\Rightarrow\text{u}=\sin^{-1}(\sin2\theta)\ .....(\text{i})
Let, \text{v}=\sqrt{1-4\text{x}^2}\ .....(\text{ii})
Here,

https://bls.smartstudies.co.in/#/exam/pdf-preview/c59cb220-8e86-4716-9ff7-82aec16b1ade/1 54/158
5/26/24, 6:19 PM Exam Automation
\text{x}\in\Big(-\frac{1}{2\sqrt{2}},\frac{1}{2}\Big)
\Rightarrow2\text{x}\in\Big(-\frac{1}{\sqrt{2}},1\Big)
\Rightarrow\cos\theta\in\Big(\frac{1}{\sqrt{2}},1\Big)
\Rightarrow\theta\in\Big(0,\frac{\pi}{4}\Big)
So, from equation (i),
\text{u}=2\theta
\Big[\text{Since},\sin^{-1}(\sin\theta)=\theta,\text{ if }\theta\in\Big[-\frac{\pi}{2},\frac{\pi}{2}\Big]\Big]
\Rightarrow\text{u}=2\cos^{-1}(2\text{x})\big[\text{Since},2\text{x}=\cos\theta\big]
Differentiating it with respect to x using chain rule,
\frac{\text{dv}}{\text{dx}}=2\bigg(\frac{-1}{\sqrt{1-(2\text{x})^2}}\bigg)\frac{\text{d}}{\text{dx}}(2\text{x})
\Rightarrow\frac{\text{dv}}{\text{dx}}=\Big(\frac{-2}{\sqrt{1-4\text{x}^2}}(2)\Big)
\Rightarrow\frac{\text{dv}}{\text{dx}}=\frac{-4}{\sqrt{1-4\text{x}^2}}\ .....(\text{v})
Dividing equation (v) by (iv)
\frac{\frac{\text{du}}{\text{dx}}}{\frac{\text{dv}}{\text{dx}}}=\frac{-4}{\sqrt{1-4\text{x}^2}}\times\frac{\sqrt{1-4\text{x}^2}}{-4\text{x}}
\frac{\text{du}}{\text{dv}}=\frac{1}{\text{x}}
Q206. Differentiate the following functions with respect to x: 4 Marks
\text{x}^{\tan^{-1}\text{x}}

Ans: Let \text{y}=\text{x}^{\tan^{-1}\text{x}}\ .....(\text{i})


Taking log on both the sides,
\log\text{y}=\log\text{x}^{\tan^{-1}\text{x}}
\log\text{y}=\tan^{-1}\text{x}\log\text{x}\ \big[\text{Since},\log\text{a}^\text{b}=\text{b}\log\text{a}\big]
Differentiating it with respect to x using product rule,
\frac{1}{\text{y}}\frac{\text{dy}}{\text{dx}}=\tan^{-1}\text{x}\frac{\text{d}}{\text{dx}}(\log\text{x})+\log\text{x}\frac{\text{d}}{\text{dx}}(\tan^{-1}\text{x})
\frac{1}{\text{y}}\frac{\text{dy}}{\text{dx}}=\tan^{-1}\text{x}\Big(\frac{1}{\text{x}}\Big)+\log\text{x}\Big(\frac{1}{1+\text{x}^2}\Big)
\frac{\text{dy}}{\text{dx}}=\text{y}\Big[\frac{\tan^{-1}\text{x}}{\text{x}}+\frac{\log\text{x}}{1+\text{x}^2}\Big]
\frac{\text{dy}}{\text{dx}}=\text{x}^{\tan^{-1}\text{x}}\Big[\frac{\tan^{-1}\text{x}}{\text{x}}+\frac{\log\text{x}}{1+\text{x}^2}\Big]
[Using equation (i)]
Q207. Explain if Rolle's theorem is applicable to any one of the following functions. 4 Marks
1. \text{f}(\text{x})=[\text{x}]\text{ on }\text{x}\in[5,9]
2. \text{f}(\text{x})=[\text{x}]\text{ on }\text{x}\in[-2,2]
Can you say something about the converse of Rolle's Theorem from these functions?

Ans: By Rolle’s theorem, for a function \text{f}:[\text{a},\text{b}]\rightarrow\text{R}, if


1. f is continuous on [a, b],
2. f is differentiable on (a, b) and
3. f(a) = f(b)
Then there exists some \text{c}\in(\text{a},\text{b}) such that f'(c) = 0
Therefore, Rolle’s theorem is not applicable to those functions that do not satisfy any of the three conditions of the hypothesis.
1. \text{f}(\text{x})=[\text{x}]\text{ on }\text{x}\in[5,9]
It is evident that the given function f(x) is not continuous at every integral point.
In particular, f(x) is not continuous at x = 5 and x = 9.
Thus, f(x) is not continuous on [5, 9].
Also, f(5) = [5] = 5 and f(9) = [9] = 9
\therefore\ \text{f}(5)\neq\text{f}(9)
The differentiability of f on (5, 9) is checked in the following way.
Let n be an integer such that \text{n}\in(5,9).
The left hand limit of f at x = n is,
\lim_\limits{\text{h}\rightarrow0^-}\frac{\text{f}(\text{n}+\text{h})-\text{f}(\text{n})}{\text{h}}=\lim_\limits{\text{h}\rightarrow0^-}\frac{[\text{n}+\text{h}]-[\text{n}]}{\text{h}}
=\lim_\limits{\text{h}\rightarrow0^-}\frac{\text{n}-1-\text{n}}{\text{h}}=\lim_\limits{\text{h}\rightarrow0^-}\frac{-1}{\text{h}}=\infty
The right hand limit of f at x = n is,
\lim_\limits{\text{h}\rightarrow0^+}\frac{\text{f}(\text{n}+\text{h})-\text{f}(\text{n})}{\text{h}}=\lim_\limits{\text{h}\rightarrow0^+}\frac{[\text{n}+\text{h}]-[\text{n}]}{\text{h}}
=\lim_\limits{\text{h}\rightarrow0^+}\frac{\text{n}-\text{n}}{\text{h}}=\lim_\limits{\text{h}\rightarrow0^+}0=0
Since the left and the right hand limits of f at x = n are not equal, f is not differentiable at x = n.
Thus, f is not differentiable on (5, 9).
It is observed that f does not satisfy all the conditions of the hypothesis of Rolle’s theorem.
Hence, Rolle’s theorem is not applicable on \text{f}(\text{x})=[\text{x}]\text{ on }\text{x}\in[5,9]
2. ​\text{f}(\text{x})=[\text{x}]\text{ on }\text{x}\in[-2,2]
It is evident that the given function f(x) is not continuous at every integral point.
In particular, f(x) is not continuous at x = −2 and x = 2.
Thus, f (x) is not continuous on [−2, 2].
Also, f(-2) = [-2] = -2 and f(2) = [2] = 2
\therefore\ \text{f}(-2)\neq\text{f}(2)
The differentiability of f on (-2, 2) is checked in the following way.
Let n be an integer such that \text{n}\in(-2,2).
The left hand limit of f at x = n is,
\lim_\limits{\text{h}\rightarrow0^-}\frac{\text{f}(\text{n}+\text{h})-\text{f}(\text{n})}{\text{h}}=\lim_\limits{\text{h}\rightarrow0^-}\frac{[\text{n}+\text{h}]-[\text{n}]}{\text{h}}
=\lim_\limits{\text{h}\rightarrow0^-}\frac{\text{n}-1-\text{n}}{\text{h}}=\lim_\limits{\text{h}\rightarrow0^-}\frac{-1}{\text{h}}=\infty
The right hand limit of f at x = n is,
\lim_\limits{\text{h}\rightarrow0^+}\frac{\text{f}(\text{n}+\text{h})-\text{f}(\text{n})}{\text{h}}=\lim_\limits{\text{h}\rightarrow0^+}\frac{[\text{n}+\text{h}]-[\text{n}]}{\text{h}}
=\lim_\limits{\text{h}\rightarrow0^+}\frac{\text{n}-\text{n}}{\text{h}}=\lim_\limits{\text{h}\rightarrow0^+}0=0
Since the left and the right hand limits of f at x = n are not equal, f is not differentiable at x = n.
Thus, f is not differentiable on (-2, 2).
It is observed that f does not satisfy all the conditions of the hypothesis of Rolle’s theorem.
Hence, Rolle’s theorem is not applicable on \text{f}(\text{x})=[\text{x}]\text{ on }\text{x}\in[-2,2]​​
Q208. Verify Rolle's theorem of the following function on the indicated interval 4 Marks
\text{f}(\text{x})=\frac{\sin\text{x}}{\text{e}^{\text{x}}}\text{ on }0\leq\text{x}\leq\pi

Ans: The given function is \text{f}(\text{x})=\frac{\sin\text{x}}{\text{e}^{\text{x}}}.


Since \cos\text{x} and \text{e}^\text{x} are everywhere continuous and differentiable, being a quotient of these two, f(x) is continuous on [0,\pi] and differentiable on (0,\pi).
Also,
\text{f}(\pi)=\text{f}(0)=0
Thus, f(x) satisfies all the conditions of Rolle's theorem.
Now, we have to show that there exists \text{c}\in(0,\pi) such that f'(c) = 0.
We have
\text{f}(\text{x})=\frac{\sin\text{x}}{\text{e}^{\text{x}}}
\Rightarrow \text{f}'(\text{x})=\frac{\cos\text{x}-\sin\text{x}}{\text{e}^{\text{x}}}

https://bls.smartstudies.co.in/#/exam/pdf-preview/c59cb220-8e86-4716-9ff7-82aec16b1ade/1 55/158
5/26/24, 6:19 PM Exam Automation
\therefore\ \text{f}'(\text{x})=0
\Rightarrow\frac{\cos\text{x}-\sin\text{x}}{\text{e}^{\text{x}}}=0
\Rightarrow\cos\text{x}-\sin\text{x}=0
\Rightarrow\tan\text{x}=1
\Rightarrow\text{x}=\frac{\pi}{4}
Thus, \text{c}=\frac{\pi}{4}\in(0,\pi) such that f'(c) = 0
Hence, Rolle's theorem is verified.
Q209. If ey (x + 1) = 1, then show that \frac{\text{d}^{2}\text{y}}{\text{dx}^{2}} = \bigg(\frac{\text{dy}}{\text{dx}}\bigg)^{2}. 4 Marks

Ans: \text{e}^{\text{y}.} \text{(x + 1)} = 1 \Rightarrow \text{e}^{\text{y}}. \text{1 + (x + 1)}.\text{e}^{\text{y}}.\frac{\text{dy}}{\text{dx}} = 0


\Rightarrow \frac{\text{dy}}{\text{dx}} = -\frac{1}{\text{(x + 1)}}
\frac{\text{d}^{2}\text{y}}{\text{dx}^{2}} = + \frac{1}{\text{(x + 1)}^{2}} = \big(\frac{\text{dy}}{\text{dx}}\big)^{2}
Q210. Differentiate the following functions with respect to x: 4 Marks
(\sin\text{x})^{\cos\text{x}}

Ans: Let \text{y}=(\sin\text{x})^{\cos\text{x}}\ .....(\text{i})


Taking log on both the sides,
\log\text{y}=(\sin\text{x})^{\cos\text{x}}
\Rightarrow\log\text{y}=\cos\text{x}\log(\sin\text{x})
Differentiating with respect to x,
\frac{1}{\text{y}}\frac{\text{dy}}{\text{dx}}=\cos\text{x}\frac{\text{d}}{\text{dx}}(\log\sin\text{x})+\log\sin\text{x}\frac{\text{d}}{\text{dx}}(\cos\text{x})
\Rightarrow\frac{1}{\text{y}}\frac{\text{dy}}{\text{dx}}=\cos\text{x}\frac{1}{\sin\text{x}}\frac{\text{d}}{\text{dx}}(\sin\text{x})+\log\sin\text{x}(-\sin\text{x})
\Rightarrow\frac{1}{\text{y}}\frac{\text{dy}}{\text{dx}}=\frac{\cos\text{x}}{\sin\text{x}}(\cos\text{x})-\sin\text{x}\log\sin\text{x}
\Rightarrow\frac{\text{dy}}{\text{dx}}=\text{y}[\cos\text{x}\cot\text{x}-\sin\text{x}\log\sin\text{x}]
\Rightarrow\frac{\text{dy}}{\text{dx}}=(\sin\text{x})^{\cos\text{x}}[\cos\text{x}\cot\text{x}-\sin\text{x}\log\sin\text{x}]
Q211. Discuss the continuity of the f(x) at the indicated points f(x) = |x| + |x - 1| at x = 0, 1. 4 Marks

Ans: Given,
\text{f(x)}=|\text{x}|+|\text{x}-1|
We have,
(\text{LHL at x}= 0)=\lim_\limits{\text{x}\rightarrow0^-}\text{f(x)}=\lim_\limits{\text{h}\rightarrow0}\text{f}(0-\text{h})
=\lim_\limits{\text{h}\rightarrow0}\big[|0-\text{h}|+|0-\text{h}-1|\big]=1
(\text{RHL at x}= 0)=\lim_\limits{\text{x}\rightarrow0^+}\text{f(x)}=\lim_\limits{\text{h}\rightarrow0}\text{f}(0+\text{h})
=\lim_\limits{\text{h}\rightarrow0}\big[|0+\text{h}|+|0+\text{h}-1|\big]=1
Also, \text{f}(1)=|1|+|1-1|=1+0=1
\therefore\ \lim_\limits{\text{x}\rightarrow0^-}\text{f(x)}=\lim_\limits{\text{x}\rightarrow0^+}\text{f(x)}=\text{f}
(0) and \lim_\limits{\text{x}\rightarrow1^-}\text{f(x)}=\lim_\limits{\text{x}\rightarrow1^+}\text{f(x)}=\text{f}=1
Hence, f(x) is continuous at x = 0, 1
Q212. If xm + yn = 1, Prove that \frac{\text{dy}}{\text{dx}}=-\frac{\text{my}}{\text{nx}} 4 Marks

Ans: We have, xm + yn = 1
Taking log on both side,
\log(\text{x}^\text{m}\text{y}^{\text{n}})=\log(1)
Differentiating with respect to x,
\frac{\text{dy}}{\text{dx}}(\text{m}\log\text{x})+\frac{\text{d}}{\text{dx}}(\text{n}\log\text{y})=\frac{\text{d}}{\text{dx}}\big\{\log(1)\big\}
\Rightarrow\frac{\text{m}}{\text{n}}+\frac{\text{n}}{\text{y}}\frac{\text{dy}}{\text{dx}}=0
\Rightarrow\frac{\text{dy}}{\text{dx}}=-\frac{\text{m}}{\text{x}}\times\frac{\text{y}}{\text{n}}
\Rightarrow\frac{\text{dy}}{\text{dx}}=-\frac{\text{my}}{\text{nx}}
Q213. If \text{y}=\log\sqrt{\frac{1+\tan\text{x}}{1-\tan\text{x}}}, prove that \frac{\text{dy}}{\text{dx}}=\sec2\text{x} 4 Marks

Ans: We have, \text{y}=\sqrt{\frac{1+\text{e}^\text{x}}{1-\text{e}^\text{x}}}


Differentiating with respect to x,
\frac{\text{dy}}{\text{dx}}=\frac{\text{d}}{\text{dx}}\Big(\sqrt{\frac{1+\text{e}^\text{x}}{1-\text{e}^\text{x}}}\Big)
=\frac{1}{\sqrt[2]{\Big(\frac{1+\text{e}^\text{x}}{1-\text{e}^\text{x}}\Big)}}\times\frac{\text{d}}{\text{dx}}\Big(\frac{1+\text{e}^\text{x}}{1-\text{e}^\text{x}}\Big)
[Using chain rule]
=\frac{1}{2}\times\sqrt{\frac{1-\text{e}^\text{x}}{1+\text{e}^\text{x}}}\Bigg[\frac{\big(1-\text{e}^\text{x}\frac{\text{d}}{\text{dx}}(1+\text{e}^\text{x})-(1+\text{e}^\text{x})\frac{\text{d}}
{\text{dx}}(1-\text{e}^\text{x})\big)}{(1-\text{e}^\text{x})^2}\Bigg]
=\frac{1}{2}\sqrt{\frac{1-\text{e}^\text{x}}{1+\text{e}^\text{x}}}\bigg[\frac{(1-\text{e}^\text{x})\text{e}^\text{x}+(1+\text{e}^\text{x})\text{e}^\text{x}}{(1-\text{e}^\text{x})^2}\bigg]
=\frac{1}{2}\sqrt{\frac{1-\text{e}^\text{x}}{1+\text{e}^\text{x}}}\Big[\frac{2\text{e}^\text{x}}{(1-\text{e}^\text{x})^2}\Big]
=\frac{\text{e}^\text{x}}{\sqrt{(1+\text{e}^\text{x})\sqrt{(1+\text{e}^\text{x})}}}\frac{1}{(1-\text{e}^\text{x})}
\frac{\text{dy}}{\text{dx}}=\frac{\text{e}^\text{x}}{(1-\text{e}^\text{x})\sqrt{1-\text{e}^{2\text{x}}}}
Q214. Find the values of a and b, if the function f defined by 4 Marks
\text{f}(x) = \begin{cases} x^{2} + 3x + \text{a} \text{ }, & x \leq 1\ bx + 2 \text{ }\text{ }\text{ }\text{ }\text{ }\text{ }\text{ }\text{ }\text{ }\text{ },& x > 1\ \end{cases}
is differentiable at x = 1.

Ans: \text{f}'_{1-} = \text{2x + 3 = 5}


\text{f}'_{1+} = \text{b}
\text{f}_{1-} = \text{f}'_{1+} \Rightarrow \text{b = 5}
\lim\limits_{\text{x} \rightarrow 1^{-}} \text{f(x)} = \text{f(1)} = \lim\limits_{\text{x} \rightarrow 1^{+}} \text{f(x)}
\Rightarrow \text{4 + a = b + 2}
\Rightarrow \text{a = 3}
Q215. If \text{xy}=4, prove that \text{x}\Big(\frac{\text{dy}}{\text{dx}}+\text{y}^2\Big)=3\text{y} 4 Marks

Ans: We have, \text{xy}=4


\Rightarrow\text{y}=\frac{4}{\text{x}}
Differentiating with respect to x,
\frac{\text{dy}}{\text{dx}}=\frac{\text{d}}{\text{dx}}\big(\frac{4}{\text{x}}\big)
\Rightarrow\frac{\text{dy}}{\text{dx}}=4\frac{\text{d}}{\text{dx}}\big(\text{x}^{-1}\big)
\Rightarrow\frac{\text{dy}}{\text{dx}}=4(-1\times\text{x}^{-1-1})
\Rightarrow\frac{\text{dy}}{\text{dx}}=4\Big(-\frac{1}{\text{x}^2}\Big)
\Rightarrow\frac{\text{dy}}{\text{dx}}=\frac{-4}{\text{x}^2}
\Rightarrow\frac{\text{dy}}{\text{dx}}=-\frac{4}{\big(\frac{4}{\text{y}}\big)^2}\ \Big[\because\text{x}=\frac{4}{\text{y}}\Big]
\Rightarrow\frac{\text{dy}}{\text{dx}}=-\frac{4\text{y}^2}{16}
\Rightarrow\frac{\text{dy}}{\text{dx}}=-\frac{\text{y}^2}{4}
\Rightarrow4\frac{\text{dy}}{\text{dx}}=-\text{y}^2
\Rightarrow4\frac{\text{dy}}{\text{dx}}+4\text{y}^2=3\text{y}^2
\Rightarrow4\Big(\frac{\text{dy}}{\text{dx}}+\text{y}^2\Big)=3\text{y}^2
Dividing both side by x,
\Rightarrow\frac{4}{\text{x}}\Big(\frac{\text{dy}}{\text{dx}}+\text{y}^2\Big)=\frac{3\text{y}^2}{\text{x}}

https://bls.smartstudies.co.in/#/exam/pdf-preview/c59cb220-8e86-4716-9ff7-82aec16b1ade/1 56/158
5/26/24, 6:19 PM Exam Automation
\Rightarrow\text{y}\Big(\frac{\text{dy}}{\text{dx}}+\text{y}^2\Big)=\frac{3\text{y}^2}{\text{y}}
\Rightarrow\text{x}\Big(\frac{\text{dy}}{\text{dx}}+\text{y}^2\Big)=\frac{3\text{y}^2}{\text{y}}
\Rightarrow\text{x}\Big(\frac{\text{dy}}{\text{dx}}+\text{y}^2\Big)=3\text{y}
Q216. Find the value of k in this question, so that the function f is continuous at the indicated point: 4 Marks
\text{f(x)}=\begin{cases}\frac{\sqrt{1+\text{kx}}-\sqrt{1-\text{kx}}}{\text{x}},&\text{if}-1\leq0\\\frac{2\text{x}+1}{\text{x}-1},&\text{if }0\leq\text{x}\leq1\end{cases} at x = 0.

Ans: We have, \text{f(x)}=\begin{cases}\frac{\sqrt{1+\text{kx}}-\sqrt{1-\text{kx}}}{\text{x}},&\text{if}-1\leq\text{x<0}\\\frac{2\text{x}+1}{\text{x}-1},&\text{if }0\leq\text{x}\leq1\end{cases} at x =


0
\text{L.H.L}=\lim\limits_{\text{h}\rightarrow0^-}\frac{\sqrt{1+\text{kx}}-\sqrt{1-\text{kx}}}{\text{x}}
=\lim\limits_{\text{h}\rightarrow0^-}\bigg(\frac{\sqrt{1+\text{kx}}-\sqrt{1-\text{kx}}}{\text{x}}\bigg)\cdot\bigg(\frac{\sqrt{1+\text{kx}}+\sqrt{1-\text{kx}}}{\sqrt{1+\text{kx}}+\sqrt{1-
\text{kx}}}\bigg)
=\lim\limits_{\text{h}\rightarrow0^-}\frac{1+\text{kx}-1+\text{kx}}{\text{x}\sqrt{1+\text{kx}}+\sqrt{1-\text{kx}}}
=\lim\limits_{\text{h}\rightarrow0^-}\frac{2\text{kx}}{\text{x}\sqrt{1+\text{kx}}+\sqrt{1-\text{kx}}}
=\lim\limits_{\text{h}\rightarrow0}\frac{2\text{k}}{\sqrt{1+\text{k}(0-\text{h})}+\sqrt{1-\text{k}(0-\text{h})}}
=\lim\limits_{\text{h}\rightarrow0}\frac{2\text{k}}{\sqrt{1-\text{kh}}+\sqrt{1+\text{kh}}}
=\frac{2\text{k}}{2}=\text{k}
\text{R.H.L}=\lim\limits_{\text{h}\rightarrow0^+}\frac{2\text{x}+1}{\text{x}-1}=\lim\limits_{\text{h}\rightarrow0}\frac{2(0+\text{h})+1}
{(0+\text{h})-1} =\lim\limits_{\text{h}\rightarrow0}\frac{2\text{h}+1}{\text{h}-1}=-1
Also \text{f}(0)=\frac{2\times0+1}{0-1}=-1
We must have L.H.L = R.H.L = f(0)
⇒ k = -1
Q217. If f is defined by \text{f(x)}=\text{x}^2-4\text{x}+7, show that \text{f}'(5)=2\text{f}'\Big(\frac{7}{2}\Big). 4 Marks

Ans: f(x) = x2 - 4x + 7 is a polynomial function, So it is differentiable everywhere.


\text{f}'(5)=\lim_\limits{\text{h}\rightarrow0}\frac{\text{f}(5+\text{h})-\text{f}(5)}{\text{h}}
=\lim_\limits{\text{h}\rightarrow0}\frac{\big\{(5+\text{h})^2-4(5+\text{h})+7\big\}-[25-20+7]}{\text{h}}
=\lim_\limits{\text{h}\rightarrow0}\frac{\text{h}^2+25+10\text{h}-20-4\text{h}+7-12}{\text{h}}
=\lim_\limits{\text{h}\rightarrow0}\frac{\text{h}^2+6\text{h}}{\text{h}}
=\lim_\limits{\text{h}\rightarrow0}(\text{h}+6)
=6
\text{f}'\Big(\frac{7}{2}\Big)=\lim_\limits{\text{h}\rightarrow0}\frac{\text{f}\Big(\frac{7}{2}+\text{h}\Big)^2-\text{f}\Big(\frac{7}{2}\Big)}{\text{h}}
=\lim_\limits{\text{h}\rightarrow0}\frac{\Big[\Big(\frac{7}{2}+\text{h}\Big)^2-4\Big(\frac{7}{2}+\text{h}\Big)+7\Big]-\Big[\Big(\frac{7}{2}\Big)^2-4\Big(\frac{7}{2}\Big)+7\Big]}{\text{h}}
=\lim_\limits{\text{h}\rightarrow0}\frac{\Big[\frac{49}{2}+\text{h}^2+7\text{h}-14-4\text{h}+7\Big]-\Big[\frac{49}{2}-14+7\Big]}{\text{h}}
=\lim_\limits{\text{h}\rightarrow0}\frac{\frac{49}{2}+\text{h}^2+7\text{h}-14-4\text{h}+7-\frac{49}{2}-14+7}{\text{h}}
=\lim_\limits{\text{h}\rightarrow0}\frac{\text{h}^2+3\text{h}}{\text{h}}
=\lim_\limits{\text{h}\rightarrow0}(\text{h}+3)
=3
\text{f}'(5)=6
=2(3)
=\text{f}'(5)=2\text{f}'\Big(\frac{7}{2}\Big)
Q218. If \text{y}=\text{e}^{\tan^{-1}\text{x}}, prove that (1+\text{x}^2)\text{y}_2+(2\text{x}-1)\text{y}_1=0 4 Marks

Ans: Given,
\text{y}=\text{e}^{\tan^{-1}\text{x}}\dots\text{ eq. }1
To prove: (1+\text{x}^2)\text{y}_2+(2\text{x}-1)\text{y}_1=0
Let's find \frac{\text{d}^2\text{y}}{\text{dx}^2}
As, \frac{\text{d}^2\text{y}}{\text{dx}^2}=\frac{\text{d}}{\text{dx}}\Big(\frac{\text{dy}}{\text{dx}}\Big)
So, let’s first find \frac{\text{dy}}{\text{dx}}
\frac{\text{dy}}{\text{dx}}=\frac{\text{d}}{\text{dx}}\text{e}^{\tan^{-1}\text{x}}
Using chain rule we will differentiate the above expression:
Let \text{t}=\tan^{-1}\text{x}
\Rightarrow\frac{\text{dt}}{\text{dx}}=\frac{1}{1+\text{x}^2}\Big[\frac{\text{d}}{\text{dx}}\tan^{-1}\text{x}=\frac{1}{1+\text{x}^2}\Big]
And y = et
\frac{\text{dy}}{\text{dx}}=\frac{\text{dy}}{\text{dt}}\frac{\text{dt}}{\text{dx}}
\frac{\text{dy}}{\text{dx}}=\text{e}^{\text{t}}\frac{1}{1+\text{x}^2}=\frac{\text{e}^{\tan^{-1}\text{x}}}{1+\text{x}^2}\dots\text{ eq. 2}
Again differentiating with respect to x applying product rule:
\frac{\text{d}^2\text{y}}{\text{dx}^2}=\text{e}^{\tan^{-1}\text{x}}\Big(\frac{1}{1+\text{x}}\Big)+\frac{1}{1+\text{x}^2}\frac{\text{d}}{\text{dx}}\text{e}^{\tan^{-1}\text{x}}
Using chain rule we will differentiate the above expression:
\frac{\text{d}^2\text{y}}{\text{dx}}=\Big(\frac{\text{e}^{\tan^{-1}\text{x}}}{(1+\text{x}^2)^2}\Big)\frac{2\text{xe}^{\tan^{-1}\text{x}}}{(1+\text{x}^2)^2} \Big[\text{using eq. 2;}\frac{\text{d}}
{\text{dx}}(\text{x}^\text{n})=\text{nx}^{\text{x}-1}\text{ and }\frac{\text{d}}{\text{dx}}\tan^{-1}\text{x}=\frac{1}{1+\text{x}^2}\Big]
(1+\text{x}^2)\frac{\text{d}^2\text{y}}{\text{dx}^2}=\frac{\text{e}^{\tan^{-1}\text{x}}}{1+\text{x}^2}-\frac{2\text{xe}^{\tan^{-1}\text{x}}}{1+\text{x}^2}
(1+\text{x}^2)\frac{\text{d}^2\text{y}}{\text{dx}^2}=\frac{\text{e}^{\tan^{-1}\text{x}}}{1+\text{x}^2}(1-2\text{x})
Using equation 2:
(1+\text{x}^2)\frac{\text{d}^2\text{y}}{\text{dx}^2}=\frac{\text{dy}}{\text{dx}}(1-\text{2x})
\therefore(1+\text{x}^2)\text{y}_2+(2\text{x}-1)\text{y}_1=0\dots\text{ proved.}
Q219. Using differentials, find the approximate value of \sqrt{49.5} 4 Marks

Ans: Let y = \sqrt{x}\therefore\text{ y }+\Delta\text{y}=\sqrt{\text{x}+\Delta\text{x}}


\Rightarrow\text{y}\frac{\text{dy}}{\text{dx}}.\Delta\text{x}\simeq\sqrt{\text{x}+\Delta\text{x}}
\Rightarrow\sqrt{\text{x}}+\frac{\text{1}}{\text{2}\sqrt{x}}.\Delta\text{x}\simeq\sqrt{\text{x}+\Delta\text{x}}
Putting x = 49 and Δx = 0.5 we get
\Rightarrow\sqrt{\text{49}}+\frac{\text{1}}{\text{2}\sqrt{49}}(0.5)\simeq\sqrt{\text{49.5}}
\Rightarrow\sqrt{\text{49}}=7+\frac{\text{1}}{\text{28}}=\text{7.0357}.
Q220. If log y = tan–1 x, then show that \text{(1 + x}^{2}) \frac{\text{d}^{2}\text{y}}{\text{dx}^{2}} + \text{(2x + 1)} \frac{\text{dy}}{\text{dx}} = 0. 4 Marks

Ans: Differentiating the given expression w.r.t. x, we get


\frac{1}{\text{y}} \frac{\text{dy}}{\text{dx}} = \frac{1}{\text{1 + x}^{2}}
\Rightarrow \text{(1 + x}^{2}) \frac{\text{dy}}{\text{dx}} = \text{y}
diff. again w.r.t. x,
\text{(1 + x}^{2}) \frac{\text{d}^2\text{y}}{\text{dx}^2}+ \frac{\text{dy}}{\text{dx}} \text{(2x)} = \frac{\text{dy}}{\text{dx}}
\Rightarrow \text{(1 + x}^{2}) \frac{\text{d}^{2}\text{y}}{\text{dx}^{2}} + \text{(2x - 1)} \frac{\text{dy}}{\text{dx}} = 0
Q221. If \text{f}\text{(x)}=\begin{cases}\frac{2^\text{z+2}-16}{4^\text{x}-16}, &\text{if x} \neq 2\\\text{k}, & \text{x} = 2\end{cases} 4 Marks
is continuous at x = 2, Find k.

Ans: We are given that the function is continuous at x = 2


\therefore LHL = RHL = f(2)...(i)
Now,
f(2) = k...(A)
\text{LHL}=\lim\limits_{\text{x} \rightarrow 2^-}\text{f}\text{(x)}=\lim\limits_{\text{h} \rightarrow 0}\text{f}(2-\text{h)}

https://bls.smartstudies.co.in/#/exam/pdf-preview/c59cb220-8e86-4716-9ff7-82aec16b1ade/1 57/158
5/26/24, 6:19 PM Exam Automation
=\lim\limits_{\text{h} \rightarrow 0}\frac{2^{(2-\text{h)+2}}-16}{4^{(2-\text{h)}}-16}=\lim\limits_{\text{h} \rightarrow 0}\frac{2^{2-\text{h}}-16}{4^{2-\text{h}}-16}
=\lim\limits_{\text{h} \rightarrow 0}\frac{2^4-2^\text{-h}-16}{4^2.4^\text{-h}-16}
=\lim\limits_{\text{h} \rightarrow 0}\frac{16.2^\text{-h}-16}{16.4^\text{-h}-16}
=\lim\limits_{\text{h} \rightarrow 0}\frac{16\Big(2^\text{-h}-1\Big)}{16\Big(4^\text{-h}-1\Big)}
=\lim\limits_{\text{h} \rightarrow 0}\frac{2^\text{-h}-1}{\Big(2^\text{-h}\Big)-1^2} \Big[\because2^{-2\text{h}}=\Big(2^{-\text{h}}\Big)^2=4^{-\text{h}}\Big]
=\lim\limits_{\text{h} \rightarrow 0}\frac{2^\text{-h}-1}{\Big(2^\text{-h}-1\Big)\Big(2^\text{-h}+1\Big)}=\frac{1}{2}\dots(\text{B})
\therefore Using (i) from (A) & (B)
\text{k}=\frac{1}{2}
Q222. It is given that the Rolle's theorem holds for the function f(x) = x3 + bx2 + cx, \text{x}\in[1,2] at the point \text{x}=\frac{4}{3}, the values of b and c. 4 Marks

Ans: So, f(1) = f(2)


⇒ (1)3 + b(1)2 + c(1) = (2)3 + b(2)2 + c(2)
⇒ 1 + b + c = 8 + 4b + 2c
⇒ 3b + c + 7 = 0 ....(i)
And \text{f}'\Big(\frac{4}{3}\Big)=0
\Rightarrow3\Big(\frac{4}{3}\Big)^2+2\text{b}\Big(\frac{4}{3}\Big)+\text{c}=0 [As, f'(x) = 3x2 + 2bx + c]
\Rightarrow\frac{16}{3}+\frac{8\text{b}}{3}+\text{c}=0
\Rightarrow8\text{b}+3\text{c}+16=0\ ....(\text{ii})
(ii) - (i) × 3, we get
8b - 9b + 16 - 21 = 0
⇒ -b - 5 = 0
⇒ b = -5
Substituting b = -5 in (i), we get
3(-5) + c + 7 = 0
⇒ -15 + c + 7 = 0
⇒c=8
Q223. Find \frac{\text{dy}}{\text{dx}}, when 4 Marks
\text{x}=\frac{\text{e}^\text{t}+\text{e}^{-\text{t}}}{2}\text{ and y}=\frac{\text{e}^\text{t}-\text{e}^\text{-t}}{2}

Ans: We have, \text{x}=\frac{\text{e}^{\text{t}}+\text{e}^{\text{-t}}}{2} and \text{y}=\frac{\text{e}^{\text{t}}+\text{e}^{\text{-t}}}{2}


\Rightarrow\frac{\text{dx}}{\text{dt}}=\frac{1}{2}\bigg[\frac{\text{d}}{\text{dt}}(\text{e}^{\text{t}})+\frac{\text{d}}{\text{dt}}(\text{e}^{\text{-t}})\bigg] and \frac{\text{dy}}{\text{dt}}=\frac{1}
{2}\bigg[\frac{\text{d}}{\text{dt}}(\text{e}^{\text{t}})-\frac{\text{d}}{\text{dt}}(\text{e}^{\text{-t}})\bigg]
\Rightarrow\frac{\text{dx}}{\text{dt}}=\frac{1}{2}\bigg[\text{e}^{\text{t}}+\text{e}^{\text{-t} \frac{\text{d}}{\text{dt}}}(\text{-t})\bigg] and \frac{\text{dy}}{\text{dt}}=\frac{1}
{2}\bigg[\text{e}^{\text{t}}-\text{e}^{\text{-t}}\frac{\text{d}}{\text{dt}}({\text{e}^{\text{-t}}})\bigg]
\Rightarrow\frac{1}{2}(\text{e}^{\text{t}}-\text{e}^\text{-t})=\text{y} and \frac{\text{dy}}{\text{dt}}=\frac{1}{2}(\text{e}^\text{t}+\text{e}^{\text{-t}})=\text{x}
\therefore\frac{\text{dy}}{\text{dt}}=\frac{\frac{\text{dy}}{\text{dt}}}{\frac{\text{dx}}{\text{dt}}}=\frac{\text{x}}{\text{y}}
Q224. If x = a (cos t + t sin t) and y = a (sin t – t cos t), 0 < t < \frac{\pi}{2}, 4 Marks
\text{find }\frac{\text{d}^{2}\text{x}}{\text{dt}^{2}},\frac{\text{d}^{2}\text{y}}{\text{dt}^{2}}\text{ and }\frac{\text{d}^{2}\text{y}}{\text{dx}^{2}}.

Ans: x = a [cos t + t sin t] \Rightarrow \frac{\text{dx}}{\text{dt}}=\text{a}[-\sin\text{t}+\sin\text{t}+\text{t cos t}]=\text{at cos t}


y = a [sin t - t cos t] \Rightarrow \frac{\text{dy}}{\text{dt}}=\text{a}[\cos\text{t}-\cos\text{t}+\text{t sin t}]=\text{at sin t}
\frac{\text{d}^{2}\text{x}}{\text{dt}^{2}}=\text{a[cos t - t sin t]},\Rightarrow\frac{\text{d}^{2}\text{y}}{\text{dt}^{2}}\text{a[sin t - t cos t]}
\frac{\text{dy}}{\text{dx}}=\tan\text{t}\Rightarrow\frac{\text{d}^{2}\text{y}}{\text{dx}^{2}}=\sec^{2}\text{t}\cdot\frac{\text{dt}}{\text{dx}}=\frac{\text{sec}^{2}\text{t}}{\text{at cos
t}}=\frac{\text{sec}^{3}\text{t}}{\text{at}}.
Q225. Find \frac{\text{dy}}{\text{dx}} of the functions expressed in parametric: 4 Marks
\text{x}=3\cos\theta-2\cos^3\theta,\text{ y}=3\sin\theta-2\sin^3\theta.

Ans: \because\ \text{x}=3\cos\theta-2\cos^3\theta and \text{y}=3\sin\theta-2\sin^3\theta


\therefore\ \frac{\text{dx}}{\text{d}\theta}=\frac{\text{d}}{\text{d}\theta}(3\cos\theta)-\frac{\text{d}}{\text{d}\theta}(2\cos^3\theta)
=3.(-\sin\theta)-2.3\cos^2\theta.\frac{\text{d}}{\text{d}\theta}.\cos\theta
=-3\sin\theta+6\cos^2\theta\sin\theta
and \frac{\text{dy}}{\text{d}\theta}=3\cos\text{A}-2.3\sin^2\theta.\frac{\text{d}}{\text{d}\theta}.\sin\theta
=3\cos\theta-6\sin^2\theta.\cos\theta
Now, \frac{\text{dy}}{\text{dx}}=\frac{\frac{\text{dy}}{\text{d}\theta}}{\frac{\text{dx}}{\text{d}\theta}}=\frac{3\cos\theta-6\sin^2\theta\cos\theta}{-3\sin\theta+6\cos^2\theta\sin\theta}
=\frac{3\cos\theta(1-2\sin^2\theta)}{3\sin\theta(-1+2\cos^2\theta)} =\cot\theta.\frac{\cos2\theta}{\cos2\theta}=\cot\theta
Q226. If \text{f(x)}=\frac{\tan\big(\frac{\pi}{4}-\text{x}\big)}{\cot2\text{x}} for \text{x}\neq\frac{\pi}{4}, find the value which can be assigned to f(x) at \text{x}=\frac{\pi}{4} so that the function f(x) 4 Marks
becomes continuous every where in \Big[0,\frac{\pi}{2}\Big]

Ans: Where \text{x}\neq\frac{\pi}{4},\ \tan\big(\frac{\pi}{4}-\text{x}\big) and \cot2\text{x} are continuous in \Big[0,\frac{\pi}{2}\Big]


Thus, the quotient function \frac{\tan\Big(\frac{\pi}{4}-\text{x}\Big)}{\cot2\text{x}} is continuous in \Big[0,\frac{\pi}{2}\Big] for each \text{x}\neq\frac{\pi}{4}
So, if f(x) is continuous at \text{x}=\frac{\pi}{4}, then it will be everywhere continuous in \Big[0,\frac{\pi}{2}\Big]
Now,
Let us consider the point \text{x}=\frac{\pi}{4}
Given, \text{f(x)}=\frac{\tan\big(\frac{\pi}{4}-\text{x}\big)}{\cot2\text{x}},\text{x}\neq\frac{\pi}{4}
We have
\Big(\text{LHL at x}=\frac{\pi}{4}\Big)=\lim\limits_{\text{x}\rightarrow\frac{\pi}{4}^-}\text{f(x)}=\lim\limits_{\text{h}\rightarrow0}\Big(\frac{\pi}{4}-
\text{h}\Big)\\=\lim\limits_{\text{h}\rightarrow0}\Bigg(\frac{\tan\big(\frac{\pi}{4}-\frac{\pi}{4}+\text{h}\big)}{\cot\big(\frac{\pi}
{2}-2\text{h}\big)}\Bigg)=\lim\limits_{\text{h}\rightarrow0}\Big(\frac{\tan(\text{h})}{\tan(2\text{h})}\Big)\\=\lim\limits_{\text{h}\rightarrow0}\Bigg(\frac{\frac{\tan(\text{h})}{\text{h}}}
{\frac{2\tan(2\text{h})}{2\text{h}}}\Bigg)=\frac{1}{2}\Bigg(\frac{\lim\limits_{\text{h}\rightarrow0}\frac{\tan(\text{h})}{\text{h}}}{\lim\limits_{\text{h}\rightarrow0}\frac{\tan(2\text{h})}
{2\text{h}}}\Bigg)=\frac{1}{2}
\Big(\text{RHL at x}=\frac{\pi}{4}\Big)=\lim\limits_{\text{x}\rightarrow\frac{\pi}{4}^+}\text{f(x)}=\lim\limits_{\text{h}\rightarrow0}\text{f}\Big(\frac{\pi}
{4}+\text{h}\Big)\\=\lim\limits_{\text{h}\rightarrow0}\Bigg(\frac{\tan\big(\frac{\pi}{4}-\frac{\pi}{4}-\text{h}\big)}{\cot\big(\frac{\pi}
{2}+2\text{h}\big)}\Bigg)=\lim\limits_{\text{h}\rightarrow0}\Big(\frac{\tan(-\text{h})}{-\tan(2\text{h})}\Big)\\=\lim\limits_{\text{h}\rightarrow0}\Big(\frac{\tan(\text{h})}
{\tan(2\text{h})}\Big)=\lim\limits_{\text{h}\rightarrow0}\Bigg(\frac{\frac{\tan(\text{h})}{\text{h}}}{\frac{2\tan(2\text{h})}{2\text{h}}}\Bigg)\\=\frac{1}
{2}\Bigg(\frac{\lim\limits_{\text{h}\rightarrow0}\frac{\tan(\text{h})}{\text{h}}}{\lim\limits_{\text{h}\rightarrow0}\frac{\tan(2\text{h})}{2\text{h}}}\Bigg)=\frac{1}{2}
If f(x) is continuous at \text{x}=\frac{\pi}{4}, then
\lim\limits_{\text{x}\rightarrow\frac{\pi}{4}^-}\text{f(x)}=\lim\limits_{\text{x}\rightarrow\frac{\pi}{4}^-}\text{f(x)}=\text{f}\Big(\frac{\pi}{4}\Big)
\therefore\ \text{f}\Big(\frac{\pi}{4}\Big)=\frac{1}{2}
Hence, for \text{f}\Big(\frac{\pi}{4}\Big)=\frac{1}{2}, the function f(x) will be everywhere continuous in \Big[0,\frac{\pi}{2}\Big]
Q227. Find \frac{\text{dy}}{\text{dx}}, when 4 Marks
\text{x}=\text{ae}^{\theta}(\sin\theta-\cos\theta),\text{y}=\text{ae}^\theta(\sin\theta+\cos\theta)

Ans: We have, \text{x}=\text{ae}^{\theta}(\sin\theta-\cos\theta) and \text{y}=\text{ae}^{\theta}(\sin\theta+\cos\theta)


\Rightarrow\frac{\text{dx}}{\text{d}\theta}=\text{a}\Big[\text{e}^\theta\frac{\text{d}}{\text{d}\theta}(\sin\theta-\cos\theta)+(\sin\theta-\cos\theta)\frac{\text{d}}{\text{d}\theta}
(\text{e}^\theta)\Big] and
\frac{\text{dy}}{\text{d}\theta}=\text{a}\Big[\text{e}^\theta\frac{\text{d}}{\text{d}\theta}(\sin\theta+\cos\theta)+(\sin\theta+\cos\theta)\frac{\text{d}}{\text{d}\theta}(\text{e}^\theta)\Big]
\Rightarrow\frac{\text{dx}}{\text{d}\theta}=\text{a}\Big[\text{e}^\theta(\cos\theta+\sin\theta)+(\sin\theta-\cos\theta)(\text{e}^\theta)\Big] and \frac{\text{dy}}
{\text{d}\theta}=\text{a}\Big[\text{e}^\theta(\cos\theta-\sin\theta)+(\sin\theta+\cos\theta)(\text{e}^\theta)\Big]
\Rightarrow\frac{\text{dx}}{\text{d}\theta}=\text{a}\big[2\text{e}^\theta(\sin\theta)\big] and \frac{\text{dy}}{\text{d}\theta}=\text{a}\big[2\text{e}^\theta(\cos\theta)\big]

https://bls.smartstudies.co.in/#/exam/pdf-preview/c59cb220-8e86-4716-9ff7-82aec16b1ade/1 58/158
5/26/24, 6:19 PM Exam Automation
\therefore\frac{\frac{\text{dy}}{\text{d}\theta}}{\frac{\text{dx}}{\text{d}\theta}}=\frac{\text{a}(2\text{e}^\theta\cos\theta)}{\text{a}(2\text{e}^\theta\sin\theta)}=\cot\theta
Q228. Find the points of discontinuity, if any of the following function: 4 Marks
\text{f(x)}=\begin{cases}\frac{\sin\text{x}}{\text{x}},&\text{if }\text{ x}<0\\2\text{x}+3,&\text{ x}\geq0\end{cases}

Ans: When x < 0, we have \text{f(x)}=\frac{\sin\text{x}}{\text{x}}


We know that \sin\text{x} and the identity function continuous for x < 0, so the quotient function
\text{f(x)}=\frac{\sin\text{x}}{\text{x}} is continuous for x < 0
When x > 0 f(x) = 2x + 3, which is a polynomial of degree 1. So, f(x) 2x + 3 is continuous for x > 0
Now, consider the point x = 0
\text{LHL}=\lim_\limits{\text{x}\rightarrow0^-}\text{f(x)}=\lim_\limits{\text{h}\rightarrow0}\text{f}(0-\text{h})
=\lim_\limits{\text{h}\rightarrow0}\frac{\sin(-\text{h})}{-\text{h}}=\lim_\limits{\text{h}\rightarrow0}\frac{-\sin\text{h}}{-\text{h}}=1
\text{RHL}=\lim_\limits{\text{x}\rightarrow0^+}\text{f(x)}=\lim_\limits{\text{h}\rightarrow0}\text{f}(0+\text{h})
=\lim_\limits{\text{h}\rightarrow0}\frac{\sin(\text{h})}{\text{h}}=1
\text{f}(0)=2\times0+3=3
Thus, \text{LHL}=\text{RHL}\neq\text{f}(0)
Hence, f(x) is discontinuous at x = 0
Q229. Find the relationship between 'a' and 'b' so that the function 'f' defined by: 4 Marks
\text{f(x)}=\begin{cases}\text{ax + 1,} &\text{if x}\leq3\\\text{bx + 3,} & \text{if x > 3}\end{cases}\text{is continuous at x = 3.}

Ans: L.H.L. = 3a + 1
f (3) = 3a + 1
RHL = 3b + 3
since f(x) is continuous at x = 3, \therefore 3a + 1 = 3b + 3.
OR 3a – 3b = 2, which is the required relation.
Q230. Find the values of p and q, for which 4 Marks

is continuous at

Ans:
Q231. Find \frac{\text{dy}}{\text{dx}} when x and y are connected by the relation: 4 Marks
\sin(\text{xy})+\frac{\text{x}}{\text{y}}=\text{x}^2-\text{y}

Ans: Consider, \sin(\text{xy})+\frac{\text{x}}{\text{y}}=\text{x}^2-\text{y}


\Rightarrow\ \frac{\text{d}}{\text{dx}}(\sin\text{xy})+\frac{\text{d}}{\text{dx}}\Big(\frac{\text{x}}{\text{y}}\Big)=\frac{\text{d}}{\text{dx}}\text{x}^2-\frac{\text{d}}{\text{dx}}\text{y}
\Rightarrow\ \cos\text{xy}\cdot\frac{\text{d}}{\text{dx}}(\text{xy})+\frac{\text{y}\frac{\text{d}}{\text{dx}}\text{x}-\text{x}\cdot\frac{\text{d}}{\text{dx}}\text{y}}
{\text{y}^2}=2\text{x}\frac{\text{dy}}{\text{dx}}
\Rightarrow\ \cos\text{xy}\cdot\Big[\text{x}\cdot\frac{\text{d}}{\text{dx}}\text{y}+\text{y}\cdot\frac{\text{d}}{\text{dx}}\cdot\text{x}\Big]+\frac{\text{y}-\text{x}\frac{\text{dy}}{\text{dx}}}
{\text{y}^2}=2\text{x}-\frac{\text{dy}}{\text{dx}}
\Rightarrow\ \text{x}\cos\text{xy}\cdot\frac{\text{dy}}{\text{dx}}+\text{y}\cos\text{xy}+\frac{\text{y}}{\text{y}^2}\frac{\text{x}}{\text{y}^2}\frac{\text{dy}}
{\text{dx}}=2\text{x}\frac{\text{dy}}{\text{dx}}
\Rightarrow\ \text{x}\cos\text{xy}\cdot\frac{\text{dy}}{\text{dx}}-\frac{\text{x}}{\text{y}^2}\frac{\text{dy}}{\text{dx}}+\frac{\text{dy}}{\text{dx}}=2\text{x}-\text{y}\cos\text{xy}-\frac{1}
{\text{y}}
\Rightarrow\ \frac{\text{dy}}{\text{dx}}\Big[\text{x}\cos\text{xy}-\frac{\text{x}}{\text{y}^2}+1\Big]=2\text{x}-\text{y}\cos\text{xy}-\frac{1}{\text{y}}
\therefore\ \frac{\text{dy}}{\text{dx}}=\Big[\frac{2\text{xy}-\text{y}^2\cos\text{xy}-1}{\text{y}}\Big]\Big[\frac{\text{y}^2}{\text{xy}^2\cos\text{xy}-\text{x}+\text{y}^2}\Big]
=\frac{2\text{xy}^2-\text{y}^3\cos\text{xy}-\text{y}}{\text{xy}^2\cos\text{xy}-\text{x}+\text{y}^2}
Q232. If x = a (cos θ + log tan \frac{\theta}{2}) and y = a sin θ, find the value of \frac{\text{dy}}{\text{dx}} at θ = \frac{\pi}{4}. 4 Marks

Ans: x = a (cos θ + log tan \frac{\theta}{2}), y = a sin θ.


\frac{\text{dx}}{\text{d}\theta}=\text{a}\Bigg[-\sin\theta+\frac{\frac{1}{2}\sec^{2}\theta/2}{\tan\frac{\theta}{2}}\Bigg],
=\text{a}\Bigg[-\sin\theta+\frac{1}{2}\frac{1}{\cos^{2}\theta/2}\frac{\cos\theta/2}{\sin\theta/2}\Bigg],
=\text{a}\Bigg[\frac{1}{\sin\theta}-\sin\theta\Bigg]=\frac{\text{a cos}^{2}\theta}{\sin\theta},\frac{\text{dy}}{\text{d}\theta}=\text{a cos}\theta
\frac{\text{dy}}{\text{dx}}=\frac{\text{a cos}\theta\times\sin\theta}{\text{a cos}^{2}\theta}=\tan\theta
\Bigg(\frac{\text{dy}}{\text{dx}}\Bigg)_{\text{at }\theta=\pi/4}=\text{1.}
Q233. Find \frac{\text{dy}}{\text{dx}} of the functions expressed in parametric: 4 Marks
\text{x}=\frac{1+\log\text{t}}{\text{t}^2},\text{ y}=\frac{3+2\log\text{t}}{\text{t}}.

Ans: Consider, \text{x}=\frac{1+\log\text{t}}{\text{t}^2} and \text{y}=\frac{3+2\log\text{t}}{\text{t}}


\Rightarrow\ \frac{\text{dx}}{\text{dt}}=\frac{\text{t}^2\cdot\frac{\text{d}}{\text{dt}}(1+\log\text{t})-(1+\log\text{t})\cdot\frac{\text{d}}{\text{dt}}\text{t}^2}{(\text{t}^2)^2}
=\frac{\text{t}^2\cdot\frac{1}{\text{t}}-(1+\log\text{t})\cdot2\text{t}}{\text{t}^4}
=\frac{\text{t}-(1+\log\text{t})\cdot2\text{t}}{\text{t}^4}
=\frac{-1-2\log\text{t}}{\text{t}^3}
and \frac{\text{dy}}{\text{dt}}=\frac{\text{t}\cdot\frac{\text{d}}{\text{dt}}(3+2\log\text{t})-(3+2\log\text{t})\cdot\frac{\text{d}}{\text{dt}}\text{t}}{\text{t}^2}
=\frac{\text{t}\cdot2\cdot\frac{1}{\text{t}}-(3+2\log\text{t})\cdot1}{\text{t}^2}a
=\frac{-1-2\log\text{t}}{\text{t}^2}
\frac{\text{dy}}{\text{dx}}=\frac{\frac{\text{dy}}{\text{dt}}}{\frac{\text{dx}}{\text{dt}}}=\frac{-1-2\log\frac{\text{t}}{\text{t}^2}}{-1-2\log\frac{\text{t}}{\text{t}^3}}=\text{t}
Q234. Differentiate the following functions with respect to x: 4 Marks
\tan^{-1}\Big\{\frac{\text{x}}{1+\sqrt{1-\text{x}^3}}\Big\},-1<\text{x}<1

Ans: Let \text{y}=\tan^{-1}\Big\{\frac{\text{x}}{1+\sqrt{1-\text{x}^3}}\Big\}


Put \text{x}=\sin\theta
\text{y}=\tan^{-1}\Big\{\frac{\sin\theta}{1+\sqrt{1-\sin^2\theta}}\Big\}
\text{y}=\tan^{-1}\Big(\frac{\sin\theta}{1+\cos\theta}\Big)
\text{y}=\tan^{-1}\bigg\{\frac{2\sin\frac{\theta}{2}\cos\frac{\theta}{2}}{2\cos^2\frac{\theta}{2}}\bigg\}
\text{y}=\tan^{-1}\Big(\tan\frac{\theta}{2}\Big)\ .....(\text{i})
Here, -1<\text{x}<1
\Rightarrow\ -1<\sin\theta<1
\Rightarrow -\frac{\pi}{2}<\theta<\frac{\pi}{2}
\Rightarrow -\frac{\pi}{4}<\frac{\theta}{2}<\frac{\pi}{4}
So, from equation (i),
\text{y}=\frac{\theta}{2}\ \Big[\text{Since}, \tan^{-1}(\tan\theta)=\theta,\text{ if }\theta\in\Big[-\frac{\pi}{2},\frac{\pi}{2}\Big]\Big]
\text{y}=\frac{1}{2}\sin^{-1}\text{x}\big[\text{Since, x}=\sin\theta\big]
Differentiating it with respect to x,
\frac{\text{dy}}{\text{dx}}=\frac{1}{2\sqrt{1-\text{x}^2}}
Q235. Find the values of p and q so that \text{f(x)}=\begin{cases}\text{x}^2+3\text{x}+\text{p},&\text{if x}\leq1\\\text{qx}+2,&\text{if x}>1\end{cases} is differentiable at x = 1. 4 Marks

Ans: We have, \text{f(x)}=\begin{cases}\text{x}^2+3\text{x}+\text{p},&\text{if x}\leq1\\\text{qx}+2,&\text{if x}>1\end{cases} is differentiable at x = 1.


\text{Lf}'(1)=\lim\limits_{\text{x}\rightarrow1^-}\frac{\text{f(x)}-\text{f}(1)}{\text{x}-1}
=\lim\limits_{\text{x}\rightarrow1^-}\frac{\big(\text{x}^2+3\text{x}+\text{p}\big)-\big(1+3+\text{p}\big)}{\text{x}-1}
=\lim\limits_{\text{h}\rightarrow0}\frac{\big[(1-\text{h})^2+3(1-\text{h})+\text{p}\big]-[4+\text{p}]}{(1-\text{h})-1}
=\lim\limits_{\text{h}\rightarrow0}\frac{\big[1+\text{h}^2-2\text{h}+3-3\text{h}+\text{p}\big]-[4+\text{p}]}{-\text{h}}

https://bls.smartstudies.co.in/#/exam/pdf-preview/c59cb220-8e86-4716-9ff7-82aec16b1ade/1 59/158
5/26/24, 6:19 PM Exam Automation
=\lim\limits_{\text{h}\rightarrow0}\frac{\big[\text{h}^2-5\text{h}+\text{p}+4-4-\text{p}\big]}{-\text{h}}=\lim\limits_{\text{h}\rightarrow0}\frac{\text{h}[\text{h}-5]}{-\text{h}}
=\lim\limits_{\text{h}\rightarrow0}-[\text{h}-5]=5
\text{Rf}'(1)=\lim\limits_{\text{x}\rightarrow1^+}\frac{\text{f(x)}-\text{f}(1)}{\text{x}-1}=\lim\limits_{\text{x}\rightarrow1^+}\frac{(\text{qx}+2)-(\text{q}+2)}{\text{x}-1}
\lim\limits_{\text{h}\rightarrow0}\frac{\big[\text{q}+\text{qh}+2-2-\text{q}\big]}{\text{h}}=\lim\limits_{\text{h}\rightarrow0}\frac{\text{qh}}{\text{h}}
\Rightarrow\ \text{q}
Since, f(x) is differentiable at x = 1
\therefore\ \text{Lf}'(1)=\text{Rf}'(1),
\therefore\ 5=\text{q}
Substituting q = 5, we get
\text{p}=3
\therefore\ \text{p}=3\text{ and q}=5
Q236. Verify Lagrange's mean value theorem for the following function on the indicated intervals. find a point 'c' in the indicated interval as stated by the Lagrange's mean value theorem. 4 Marks
f(x) = x3 - 2x2 - x + 3 on [0, 1]

Ans: Here,
f(x) = x3 - 2x2 - x + 3
Since f(x) is polynomial function. So, f(x) is continuous in [0, 1] and differentiable in (0, 1).
Thus, both conditions of Lagrange's mean value theorem is appplicable.
Thus, there exist a point \text{c}\in(0,1) such that
\text{f}'(\text{c})=\frac{\text{f}(1)-\text{f}(0)}{1-0}
\Rightarrow3\text{c}^2-4\text{c}-1=\frac{[(1)^3-2(1)^2-(1)+3]-3}{1}
⇒ 3c2 - 4c - 1 = 1-2
⇒ 3c2 - 4c + 1 = 0
⇒ 3c2 - 3c - c + 1 = 0
⇒ 3c(c - 1) - 1(c - 1) = 0
⇒ (3c - 1)(c - 1) = 0
\Rightarrow\text{c}=\frac{1}{3}\in(0,1)
Hence, Lagrange's mean value theorem is verified.
Q237. Verify Rolle's theorem of the following function on the indicated interval 4 Marks
\text{f}(\text{x})=\text{e}^{\text{x}}\cos{\text{x}}\text{ on }\Big[\frac{-\pi}{2},\frac{\pi}{2}\Big]

Ans: The given function is \text{f}(\text{x})=\text{e}^{\text{x}}\cos{\text{x}}


Since \cos\text{x}\ \&\ \text{e}^{\text{x}} are everywhere continuous and differentiable.
Therefore, f(x) being a product of these two, is continuous on \Big[\frac{-\pi}{2},\frac{\pi}{2}\Big] and differentiable on \Big(\frac{-\pi}{2},\frac{\pi}{2}\Big)
Also,
\text{f}\Big(\frac{-\pi}{2}\Big)=\text{f}\Big(\frac{\pi}{2}\Big)=0
Thus, f(x) satisfies all the conditions of Rolle's theorem.
Now, we have to show that there exists \text{c}\in\Big(\frac{-\pi}{2},\frac{\pi}{2}\Big) such that f'(c) = 0.
We have
\text{f}(\text{x})=\text{e}^{\text{x}}\cos{\text{x}}
\Rightarrow\text{f}'(\text{x})=\text{e}^{\text{x}}(\cos\text{x}+\sin\text{x})
\therefore\ \text{f}'(\text{x})=0
\Rightarrow\text{e}^{\text{x}}(\cos\text{x}+\sin\text{x})=0
\Rightarrow\sin\text{x}+\cos\text{x}=0
\Rightarrow\tan\text{x}=-1
\Rightarrow\text{x}=\frac{\pi}{4}
Since, \text{c}=\frac{\pi}{4}\in\Big(\frac{\pi}{4},\frac{\pi}{2}\Big) such that f'(c) = 0
Hence, Rolle's theorem verified.
Q238. Examine the differentiability of f, where f is defined by: 4 Marks
\text{f(x)}=\begin{cases}\text{x[x]},&\text{if }0\leq\text{x}<2\\(\text{x}-1)\text{x},&\text{if }2\leq\text{x}<3\end{cases}
at x = 2.

Ans: We have, \text{f(x)}=\begin{cases}\text{x[x]},&\text{if }0\leq\text{x}<2\\(\text{x}-1)\text{x},&\text{if }2\leq\text{x}<3\end{cases} at x = 2


Let us first check continuity at x = 2
\text{L.H.L}=\lim\limits_{\text{x}\rightarrow2^-}\text{x[x]} =\lim\limits_{\text{h}\rightarrow0}(2-\text{h})[2-\text{h}]=2\times1=2
\text{R.H.L}=\lim\limits_{\text{x}\rightarrow2^+}(\text{x}-1)\text{x} =\lim\limits_{\text{h}\rightarrow0}(2+\text{h}-1)(2+\text{h})=1\times2=2
Also, f(2) = (2 - 1)2 = 2
Thus, f(x) is continuous at x = 2.
Now \text{Lf}'(2)=\lim\limits_{\text{h}\rightarrow0}\frac{\text{f}(2-\text{h})-\text{f}(2)}{-\text{h}} =\lim\limits_{\text{h}\rightarrow0}\frac{(2-\text{h})[2-\text{h}]-2}{-\text{h}}
=\lim\limits_{\text{h}\rightarrow0}\frac{(2-\text{h}).1-2}{-\text{h}}=\lim\limits_{\text{h}\rightarrow0}\frac{-\text{h}}{-\text{h}}=1
\text{Rf}'(2)=\lim\limits_{\text{h}\rightarrow0}\frac{\text{f}(2+\text{h})-\text{f}(2)}{\text{h}} =\lim\limits_{\text{h}\rightarrow0}\frac{(2+\text{h}-1)(2+\text{h})-2}{\text{h}}
=\lim\limits_{\text{h}\rightarrow0}\frac{\text{h}^2+3\text{h}}{\text{h}}=3
Clearly, Lf'(2) ≠ Rf'(2)
Thus, f(x) is not diffcrentiable at x = 2.
Q239. If \text{f(x)} = \begin{cases} \frac{\text{x}^{2}-25}{\text{x - 5}},\\\text{ }\text{ }\text{ }\text{ }\text{ }k,&\\ \end{cases}\quad \text{when x}\neq5\\\\\quad \text{when x = 5 }is continuous at x 4 Marks
= 5, find the value of k.

Ans: For \DeclareMathOperator*{\median}{\text{lim}} \median_{\text{x}\rightarrow5}\frac{\text{x}^{2}-25}{\text{x - 5}}=\DeclareMathOperator*{\median}{\text{lim}}


\median_{\text{x}\rightarrow5}(\text{x + 5})=10
\therefore\text{k}=10.
Q240. Find the points of discontinuity, if any of the following function: 4 Marks
\text{f(x)}=​​\begin{cases}\sin\text{x}-\cos\text{x},&\text{if }\text{ x}\neq0\\-1,&\text{if }\text{ x}=0\end{cases}

Ans: The given function f is \text{f(x)}=​​\begin{cases}\sin\text{x}-\cos\text{x},&\text{if }\text{ x}\neq0\\-1,&\text{if }\text{ x}=0\end{cases}


It is evident that f is defind at all points of the real line.
Let c be a real number.
Case I:
If \text{c}\neq0, then \text{f(c)}=\sin\text{c}-\cos\text{c}
\lim_\limits{\text{x}\rightarrow\text{c}}\text{f(x)}=\lim_\limits{\text{x}\rightarrow\text{c}}(\sin\text{x}-\cos\text{x})=\sin\text{c}-\cos\text{c}
\therefore\ \lim_\limits{\text{x}\rightarrow\text{c}}\text{f(x)}=\text{f(c)}
Therefore, f is continuous at all points x, such that \text{x}\neq0
Case II:
If c = 0, then f(0) = -1
\lim_\limits{\text{x}\rightarrow0^-}\text{f(x)}=\lim_\limits{\text{x}\rightarrow0}(\sin\text{x}-\cos\text{x})\\=\sin0-\cos0=0-1=-1
\lim_\limits{\text{x}\rightarrow0^+}\text{f(x)}=\lim_\limits{\text{x}\rightarrow0}(\sin\text{x}-\cos\text{x})\\=\sin0-\cos0=0-1=-1
Therefore, f is continuousb at x = 0
From tha above observations, it can be continuous that f is continuous at every point of the real line.
Thus, f is a continuous function.

https://bls.smartstudies.co.in/#/exam/pdf-preview/c59cb220-8e86-4716-9ff7-82aec16b1ade/1 60/158
5/26/24, 6:19 PM Exam Automation
Q241. Differentiate the following functions with respect to x: 4 Marks
\text{x}^{\cos^{-1}\text{x}}

Ans: Let \text{y}=\text{x}^{\cos^{-1}\text{x}}\ .....(\text{i})


Taking log on both the sides,
\log\text{y}=\log\text{x}^{\cos^{-1}\text{x}}
\Rightarrow\log\text{y}=\cos^{-1}\text{x}\log\text{x}
Differentiating with respect to x,
\frac{1}{\text{y}}\frac{\text{dy}}{\text{dx}}=\cos^{-1}\text{x}\frac{\text{d}}{\text{dx}}(\log\text{x})+\log\text{x}\frac{\text{d}}{\text{dx}}\cos^{-1}\text{x}
\Rightarrow\frac{1}{\text{y}}\frac{\text{dy}}{\text{dx}}=\cos^{-1}\text{x}\Big(\frac{1}{\text{x}}\Big)+\log\text{x}\Big(\frac{-1}{\sqrt{1-\text{x}^2}}\Big)
\Rightarrow\frac{1}{\text{y}}\frac{\text{dy}}{\text{dx}}=\frac{\cos^{-1}}{\text{x}}-\frac{\log\text{x}}{\sqrt{1-\text{x}^2}}
\Rightarrow\frac{\text{dy}}{\text{dx}}=\text{y}\Big[\frac{\cos^{-1}\text{x}}{\text{x}}-\frac{\log\text{x}}{\sqrt{1-\text{x}^2}}\Big]
\Rightarrow\frac{\text{dy}}{\text{dx}}=\text{x}^{\cos^{-1}\text{x}}\Big[\frac{\cos^{-1}\text{x}}{\text{x}}-\frac{\log\text{x}}{\sqrt{1-\text{x}^2}}\Big]
[Using equation (i)]
Q242. Differentiate the following functions with respect to x: 4 Marks
\sin^{-1}\Big\{\frac{\text{x}}{\sqrt{\text{x}^2+\text{a}^2}}\Big\}

Ans: Let \text{y}=\sin^{-1}\Big\{\frac{\text{x}}{\sqrt{\text{x}^2+\text{a}^2}}\Big\}


Put \text{x}=\text{a}\tan\theta
\Rightarrow\text{y}=\sin^{-1}\Big\{\frac{\text{a}\tan\theta}{\sqrt{\text{a}^2\tan^2\theta+\text{a}^2}}\Big\}
\Rightarrow\text{y}=\sin^{-1}\bigg\{\frac{\text{a}\tan\theta}{\sqrt{\text{a}^2(\tan^2\theta+1)}}\bigg\}
\Rightarrow\text{y}=\sin^{-1}\Big(\frac{\text{a}\tan\theta}{\text{a}\sec\theta}\Big)
\Rightarrow\text{y}=\sin^{-1}(\sin\theta)
\Rightarrow\text{y}=\theta
\Rightarrow\text{y}=\tan^{-1}\Big(\frac{\text{x}}{\text{a}}\Big)\big[\text{Since, x} = \text{a}\tan\theta\big]
Differentiating it with respect to x using chain rule,
\frac{\text{dy}}{\text{dx}}=\frac{1}{1+\big(\frac{\text{x}}{\text{a}}\big)^2}\frac{\text{d}}{\text{dx}}\Big(\frac{\text{x}}{\text{a}}\Big)
\Rightarrow\frac{\text{dy}}{\text{dx}}=\frac{\text{a}^2}{\text{a}^2+\text{x}^2}\times\big(\frac{1}{\text{a}}\big)
\therefore\frac{\text{dy}}{\text{dx}}=\frac{\text{a}}{\text{a}^2+\text{x}^2}
Q243. If \text{xy}=1, prove that \frac{\text{dy}}{\text{dx}}+\text{y}^2=0 4 Marks

Ans: Here, xy = 1 .....(i)


Differentiating with respect to x,
\frac{\text{d}}{\text{dx}}(\text{xy})=\frac{\text{d}}{\text{dx}}(1)
\Rightarrow\text{x}\frac{\text{dy}}{\text{dx}}+\text{y}\frac{\text{d}}{\text{dx}}(\text{x})=0
[Using product rule]
\Rightarrow\text{x}\frac{\text{dy}}{\text{dx}}+\text{y}(1)=0
\Big[\text{Put x}=\frac{1}{\text{y}}\text{ from equation (i)}\Big]
\Rightarrow\frac{\text{dy}}{\text{dx}}=-\frac{\text{y}}{\text{x}}
\Rightarrow\frac{\text{dy}}{\text{dx}}=-\frac{\text{y}}{\frac{1}{\text{y}}}
\Rightarrow\frac{\text{dy}}{\text{dx}}=-\text{y}^2
\Rightarrow\frac{\text{dy}}{\text{dx}}+\text{y}^2=0
Q244. Verify Rolle's theorem of the following function on the indicated interval 4 Marks
\text{f}(\text{x})=\text{e}^{1-\text{x}^2}\text{ on }[-1,1]

Ans: Here,
\text{f}(\text{x})=\text{e}^{1-\text{x}^2}\text{ on }[-1,1]
We know that, exponantial function is continuous and differentiable everywhere. So, f(x) is continuous IS [-1,1] and differentiable is (-1,1).
Now,
\text{f}(-1)=\text{e}^{1-1}=1
\text{f}(1)=\text{e}^{1-1}=1
\Rightarrow\text{f}(-1)=1
So, Rolle's theorem is applicable, so there must exist a point \text{c}\in(-1,1) such that f'(c) = 0.
Now,
\text{f}(\text{x})=\text{e}^{1-\text{x}^2}
\text{f}'(\text{x})=\text{e}^{1-\text{x}^2}(-2\text{x})
Now,
\text{f}'(\text{c})=0
-2\text{ce}^{1-\text{c}^2}=0
\Rightarrow\text{c}=0 or \text{e}^{1-\text{c}^2}=0
\Rightarrow\text{c}=0\in(-1,1)
Hence, Rolle's theorem is verified.
Q245. Find \frac{\text{dy}}{\text{ dx}} in the following: 4 Marks
\text{y}=\sin^{-1}\big(2\text{x}\sqrt{1-\text{x}^{2}}\big), -\frac{1}{\sqrt{2}}<\text{x}<\frac{1}{\sqrt{2}}

Ans: The given relationship is,\text{y}=\sin^{-1}\big(2\text{x}\sqrt{1-\text{x}^{2}}\big)


\text{y}=\sin^{-1}\big(2\text{x}\sqrt{1-\text{x}^{2}}\big)
\Rightarrow\sin\text{y}=2\text{x}\sqrt{1-\text{x}^{2}}
Differentiating this relationship with respect to x, we obtain
\cos\text{y}\frac{\text{dy}}{\text{dx}}=2\bigg[\text{x}\frac{\text{d}}{\text{dx}}\big(\sqrt{1-\text{x}^{2}}\big)+\sqrt{1-\text{x}^{2}}\frac{\text{d}}{\text{dx}}\text{x}\bigg]
\Rightarrow\sqrt{1-\sin^{2}\text{y}\frac{\text{dy}}{\text{dx}}}=2\bigg[\frac{\text{x}}{2}.\frac{-2\text{x}}{\sqrt{1-\text{x}^{2}}}+\sqrt{1-\text{x}^{2}}\bigg]
\Rightarrow\sqrt{1-\big(2\text{x}\sqrt{1-\text{x}^{2}}\big)^{2}}\frac{\text{dy}}{\text{dx}}=2\bigg[\frac{-\text{x}^{2}+1-\text{x}^{2}}{\sqrt{1-\text{x}^{2}}}\bigg]
\Rightarrow\sqrt{1-4\text{x}^{2}\big({1-\text{x}^{2}}\big)}\frac{\text{dy}}{\text{dx}}=2\bigg[\frac{1-2\text{x}^{2}}{\sqrt{1-\text{x}^{2}}}\bigg]
\sqrt{1-4x^2+4x^4}\frac{\text{dy}}{\text{dx}}=2\bigg[\frac{1-2\text{x}^{2}}{\sqrt{1-\text{x}^{2}}}\bigg]
\Rightarrow\sqrt{(1-2\text{x}^{2}})^{2}\frac{\text{dy}}{\text{dx}}=2\bigg[\frac{1-2\text{x}^{2}}{\sqrt{1-\text{x}^{2}}}\bigg]
\Rightarrow\big(1-2\text{x}^{2}\big)\frac{\text{dy}}{\text{dx}}=2\bigg[\frac{1-2\text{x}^{2}}{\sqrt{1-\text{x}^{2}}}\bigg]
\Rightarrow\frac{\text{dy}}{\text{dx}}=\frac{2}{\sqrt{1-\text{x}^2}}
Q246. Examine the applicability of Mean Value Theorem for all three functions given in the above exercise 2. 4 Marks
\text{f(x)}=[\text{x}]\text{ for x}\in[5,\ 9]

Ans: By Rolle's Theorem, for a function \text{f}:[\text{a},\ \text{b}]\rightarrow\text{R},\text{if}


1. f is continuous on [a, b]
2. f is differentiabie on (a, b)
3. f(a) = f(b)
then, there exists some \text{c}\in(\text{a},\ \text{b}) such that f'(c) = 0
Therefore, Rolle's Theorem is not applicable to those functions that do not satisfy any of the three two conditions of the hypothesis.
\text{f(x)}=[\text{x}]\text{for x}\in[5,\ 9]
It is evident that the given function f(x) is not continuous at every integral point.
In particular, f(x) is not continuous at x = 5 and x = 9

https://bls.smartstudies.co.in/#/exam/pdf-preview/c59cb220-8e86-4716-9ff7-82aec16b1ade/1 61/158
5/26/24, 6:19 PM Exam Automation
\Rightarrow\ f(x) is not continuous in [5, 9].
The differentiability of f in (5, 9) is checked as follows.
Let n be an integer such that n \in(5, 9).
The left hand limit of f at x = n is,
^{\ \ \text{lim}}_{\text{h}\rightarrow0^-}\frac{\text{f(n}+\text{h})-\text{f(n)}}{\text{h}}=^{\ \ \text{lim}}_{\text{h}\rightarrow0^-}\frac{[\text{n}+\text{h}]-[\text{n}]}{\text{h}} =^{\ \
\text{lim}}_{\text{h}\rightarrow0^-}\frac{\text{n}-1-\text{n}}{\text{h}}=^{\ \ \text{lim}}_{\text{h}\rightarrow0^-}\frac{-1}{\text{h}}=\infty
The right hand limit of f at x = n is,
^{\ \ \text{lim}}_{\text{h}\rightarrow0^+}\frac{\text{f(n}+\text{h})-\text{f(n)}}{\text{h}}=^{\ \ \text{lim}}_{\text{h}\rightarrow0^+}\frac{[\text{n}+\text{h}]-[\text{n}]}{\text{h}} =^{\ \
\text{lim}}_{\text{h}\rightarrow0^+}\frac{\text{n}-\text{n}}{\text{h}}=^{\ \ \text{lim}}_{\text{h}\rightarrow0^+}0=0
Since the left and right hand limits of f at x = n are not equal, f is not differentiable at x = n
\therefore\ f is not differentiable in (5, 9).
It is observed that f dose not satisfy all the conditions of the hypothesis of Mean value Theorem.
Hence, Mean value Theorem is not applicable for \text{f(x)}=[\text{x}]\text{ for x}\in[5,\ 9].
Q247. Differentiate the following function with respect to x: 4 Marks
(\log\text{x})^{x} + \text{x}^{\log\text{x}}.

Ans: Let y =(log x)x + x logx


\Rightarrow y =u+v where u=(log x)x , v = xlog x
\Rightarrow\frac{\text{dy}}{\text{dx}} = \frac{\text{du}}{\text{dx}} + \frac{\text{dv}}{\text{dx}} - - - - (i)
Now u=(log x)x
Taking logarithm of both sides, we get
logu= x. log(log x)
Differentiating both sides w.r.t.x, we get
\frac{1}{\text{u}}.\frac{\text{du}}{\text{dx}} =\text{x}.\frac{1}{\log\text{x}}.\frac{1}{\text{x}} + \log(\log\text{x} )\Rightarrow\frac{\text{du}}{\text{dx}} = \text{u}\left\{\frac{1}{\log\text{x}} +
\log(\log\text{x})\right\}
\Rightarrow\frac{\text{du}}{\text{dx}} = (\log\text{x})^{x}\left\{\frac{1}{\log\text{x}} + \log(\log\text{x})\right\} - - - - (ii)
Again v = xlog x
Taking logarithm of both sides , we get
log v = log xlog x
\Rightarrow\log\text{v} = \log\text{x}.\log\text{x}\Rightarrow\log v = ( \log\text{x})^{2}
Differentiating both sides w.r.t.x, we get
\frac{1}{v}\frac{\text{dv}}{\text{dx}} = 2 \log\text{x}.\frac{1}{\text{x}}
\Rightarrow\frac{\text{dv}}{\text{dx}} = 2 \text{x}^{log\text{ x}}.\frac{\log\text{x}}{\text{x}} - - - - - (iii)
Putting \frac{\text{du}}{\text{dx}}\text{ and } \frac{\text{dv}}{\text{dx}} from (ii) and (iii) in (i) we get
\frac{\text{dy}}{\text{dx}} = ( \log\text{x})^{x}\left\{\frac{1}{\log\text{x}} + \log(\log\text{x})\right\} + 2 \frac{\log\text{x.x}^{\log\text{x}}}{\text{x}}.
Q248. If \text{x}=\sin\Big(\frac{1}{\text{a}}\log\text{y}\Big), show that (1-\text{x}^2)\text{y}_2-\text{xy}_1-\text{a}^2\text{y}=0 4 Marks

Ans: Here,
\text{x}=\sin\Big(\frac{1}{\text{a}}\log\text{y}\Big),
\Rightarrow\frac{1}{\text{a}}\log\text{y}=\sin^{-1}\text{x}
\Rightarrow\text{y}=\text{e}^\text{a}\sin^{-1}\text{x}
Differentiating w.r.t.x, we get
\text{y}_1=\text{e}^{\text{a} \sin^{-1}\text{x}}\times\frac{\text{a}}{\sqrt{1-\text{x}^2}}
\Rightarrow\text{y}_1=\frac{\text{ay}}{\sqrt{1-\text{x}^2}}
Differentiating w.r.t.x, we get
\text{y}_2=\frac{\text{ay}_1\sqrt{1-\text{x}^2}+\frac{\text{x}\text{ay}}{\sqrt{1-\text{x}^2}}}{(1-\text{x}^2)}
\Rightarrow\text{y}_2=\frac{\text{ay}_1(1-\text{x}^2)+\text{xay}}{(1-\text{x}^2)\sqrt{1-\text{x}^2}}
\Rightarrow\text{y}_2=\frac{\text{ay}_1}{\sqrt{1-\text{x}^2}}+\frac{\text{xay}}{(1-\text{x}^2)\sqrt{1-\text{x}^2}}
\Rightarrow\text{y}_2=\frac{\text{a}^2\text{y}}{1-\text{x}^2}+{\frac{\text{xy}_1}{(1-\text{x}^2)}}
\Rightarrow(1-\text{x}^2)\text{y}_2-\text{xy}_1-\text{a}^2\text{y}=0
Q249. If \text{x}-\text{e}^{\tan\text{x}}+\sqrt{\frac{\text{x}^2+1}{2}}, find \frac{\text{dy}}{\text{dx}} 4 Marks

Ans: \text{y}=\text{x}^{\tan\text{x}}+\sqrt{\frac{\text{x}^2+1}{2}}
\text{y}=\text{e}^{\tan\text{x}\log\text{x}}+\text{e}^{\frac{1}{2}\log\big(\frac{\text{x}^2+1}{2}\big)}
\frac{\text{dy}}{\text{dx}}=\text{e}^{\tan\text{x}\log\text{x}}\frac{\text{d}}{\text{dx}}(\tan\text{x}\log\text{x})+\text{e}^{\frac{1}{2}\log\big(\frac{\text{x}^2+1}{2}\big)}\frac{\text{d}}
{\text{dx}}\Big(\frac{1}{2}\log\Big(\frac{\text{x}^2+1}{2}\Big)\Big)
\frac{\text{dy}}{\text{dx}}=\text{e}^{\tan\text{x}}\Big[\frac{\tan\text{x}}{\text{x}}+\sec^3\text{x}\log\text{x}\Big]+\sqrt{\frac{\text{x}^2+1}{2}}\Big(\frac{1}{2}\times\frac{2}
{\text{x}^2+1}\times(\text{x})\Big)
\frac{\text{dy}}{\text{dx}}=\text{e}^{\tan\text{x}}\Big[\frac{\tan\text{x}}{\text{x}}+\sec^3\text{x}\log\text{x}\Big]+\sqrt{\frac{\text{x}^2+1}{2}}\Big(\frac{\text{x}}{\text{x}^2+1}\Big)
\frac{\text{dy}}{\text{dx}}=\text{e}^{\tan\text{x}}\Big[\frac{\tan\text{x}}{\text{x}}+\sec^3\text{x}\log\text{x}\Big]+\frac{\text{x}}{\sqrt{2(\text{x}^2+1)}}
Q250. Verify Lagrange's mean value theorem for the following function on the indicated intervals. find a point 'c' in the indicated interval as stated by the Lagrange's mean value theorem. 4 Marks
f(x) = x(x - 1) on [1, 2]

Ans: We have
f(x) = x(x - 1) which can be rewritten as f(x) = x2 - x
Since a polynomial function is everywhere continuous and differentiable.
Therefore, f(x) is continuous on [1, 2] and differentiable on (1, 2).
Thus, both conditions of Lagrange's mean value theorem is satisfied.
So, there must exist at least one real number \text{c}\in(1,2) such that
\text{f}'(\text{c})=\frac{\text{f}(2)-\text{f}(1)}{2-1}
Now, f(x) = x2 - x
⇒ f'(x) = 2x - 1,
⇒ f(2) = 2, f(1) = 0
\therefore\ \text{f}'(\text{x})=\frac{\text{f}(2)-\text{f}(1)}{2-1}
\Rightarrow2\text{x}-1=\frac{2-0}{2-1}
\Rightarrow2\text{x}-1-2=0
\Rightarrow2\text{x}=3
\Rightarrow\text{x}=\frac{3}{2}
Thus, \text{c}=\frac{3}{2}\in(1,2) such that \text{f}'(\text{c})=\frac{\text{f}(2)-\text{f}(1)}{2-1}
Hence, Lagrange's mean value theorem is verified.
Q251. If \text{x}=\text{a}\sec^3\theta,\text{y}=\text{a}\tan^3\theta, then find \frac{\text{d}^2\text{y}}{\text{dx}^2} at \theta=\frac{\pi}{4}. 4 Marks

Ans: \frac{\text{dy}}{\text{dx}}=\frac{\frac{\text{dy}}{\text{d}\theta}}{\frac{\text{dx}}{\text{d}\theta}}=\frac{3\text{a}\tan^2\theta\sec^2\theta}{3\text{a}\sec^3\theta\tan\theta}=\sin\theta
\frac{\text{d}^2\text{y}}{\text{dx}^2}=\cos\theta\cdot\frac{\text{d}\theta}{\text{dx}}=\frac{1}{3\text{a}\sec^4\theta\tan\theta}
\frac{\text{d}^2\text{y}}{\text{dx}^2}\Bigg|_{\theta=\frac{\pi}{4}}=\frac{1}{12\text{a}}
Q252. Discuss the continuity of the function \text{f(x)}=\begin{cases}2\text{x}-1,&\text{if }\text{ x}<2\\\frac{3\text{x}}{2},&\text{if }\text{ x}\geq2\end{cases} 4 Marks

Ans: When x < 2, we have


f(x) = 2x - 1

https://bls.smartstudies.co.in/#/exam/pdf-preview/c59cb220-8e86-4716-9ff7-82aec16b1ade/1 62/158
5/26/24, 6:19 PM Exam Automation
We know that a polynomial function is everywhere continuous.
So, f(x) is continuous for each x < 2
When x > 2, we have
\text{f(x)}=\frac{3\text{x}}{2}
The functions 3x and 2 are continuous being the polynomial and constant function, respectively.
Thus, the quotient function \frac{3\text{x}}{2} is continuous at each x > 2
Now,
Let us consider the point x = 2
\text{f(x)}=\begin{cases}2\text{x}-1,&\text{if }\text{ x}<2\\\frac{3\text{x}}{2},&\text{if }\text{ x}\geq2\end{cases}
We have
(\text{LHL at x}= 2)=\lim\limits_{\text{x}\rightarrow2^-}\text{f(x)}=\lim\limits_{\text{h}\rightarrow0}\text{f}(2-\text{h})\\=\lim\limits_{\text{h}\rightarrow0}(2(2-\text{h})-1)=4-1=3
(\text{LHL at x}= 2)=\lim\limits_{\text{x}\rightarrow2^+}\text{f(x)}=\lim\limits_{\text{h}\rightarrow0}\text{f}(2+\text{h})\\=\lim\limits_{\text{h}\rightarrow0}\frac{3(\text{h}+2)}{2}=3
Also,
\text{f}(2)=\frac{3(2)}{2}=3
\therefore\ \lim\limits_{\text{x}\rightarrow2^-}\text{f(x)}=\lim\limits_{\text{x}\rightarrow2^+}\text{f(x)}=\text{f}(2)
Thus, f(x) is continuous at x = 2
Hence, f(x) is everywhere continuous.
Q253. Find all points of discontinuity of f, where f is defined by: 4 Marks
\text{f(x)}= \begin{cases}\text{x}^{10} - 1,\ \ \text{if x}\leq 1 \\\text{x}^2,\ \ \ \ \ \ \ \ \ \ \text{if x}>1\end{cases}

Ans: Here \text{f(x)}= \begin{cases}\text{x}^{10} - 1,\ \ \text{if x}\leq 1 \\\text{x}^2,\ \ \ \ \ \ \ \ \ \ \text{if x}>1\end{cases}
Function f is defined at all points of the real line.
Let c be any real number.
Three cases arise:
Case I: c < 1
^{\ \ \text{Lt}}_{\text{x}\rightarrow\text{c}}\text{f(x)} = ^{\ \ \text{Lt}}_{\text{x}\rightarrow\text{c}}\text{(x}^{10} - 1) = \text{c}^{10}- 1
f(c) = c10 - 1
\therefore\ ^{\ \ \text{Lt}}_{\text{x}\rightarrow\text{c}}\text{f(x)} = \text{f(c)}
\therefore f is continuous at all points x < 1.
Case II: c > 1
^{\ \ \text{Lt}}_{\text{x}\rightarrow\text{c}}\text{f(x)} = ^{\ \ \text{Lt}}_{\text{x}\rightarrow\text{c}}\text{x}^{2} = \text{c}^{2}
f(c) = c2
\therefore\ ^{\ \ \text{Lt}}_{\text{x}\rightarrow\text{c}}\text{f(x)} = \text{f(c)}
\therefore f is continuous at all points x > 1.
Case III: c = 1
^{\ \ \text{Lt}}_{\text{x}\rightarrow\text{1}^{-}}\text{f(x)} = ^{\ \ \text{Lt}}_{\text{x}\rightarrow\text{1}^{-}}\text{(x}^{10} - 1) = 1^{10}- 1 = 1 - 1 =0
^{\ \ \text{Lt}}_{\text{x}\rightarrow\text{1}^{+}}\text{f(x)} = ^{\ \ \text{Lt}}_{\text{x}\rightarrow\text{1}^{+}}\text{x}^2 = (1)^2 = 1
\therefore\ ^{\ \ \text{Lt}}_{\text{x}\rightarrow\text{1}^{-}}\text{f(x)} \neq ^{\ \ \text{Lt}}_{\text{x}\rightarrow\text{1}^{+}}\text{f(x)}
\therefore f is discontinuous at x = 1.
Q254. In the following, find the value of the constant k so that the given function is continuous at the indicated point: 4 Marks
\text{f(x)}=\begin{cases}\text{k}+1,&\text{if}\text{ x}\leq\pi\\\cos\text{x},&\text{if}\text{ x}>\pi\end{cases}\text{at x} = \pi

Ans: Given,
\text{f(x)}=\begin{cases}\text{k}+1,&\text{if}\text{ x}\leq\pi\\\cos\text{x},&\text{if}\text{ x}>\pi\end{cases}
We have,
(\text{LHL at x}= \pi)=\lim_\limits{\text{x}\rightarrow\pi^-}\text{f(x)}=\lim_\limits{\text{h}\rightarrow0}\text{f}(\pi-\text{h})
=\lim_\limits{\text{h}\rightarrow0}\text{k}(\pi-\text{h})+1=\text{k}\pi+1
(\text{RHL at x}= \pi)=\lim_\limits{\text{x}\rightarrow\pi^+}\text{f(x)}=\lim_\limits{\text{h}\rightarrow0}\text{f}(\pi+\text{h})
=\lim_\limits{\text{h}\rightarrow0}\cos(\pi+\text{h})=\cos\pi=-1
If f(x) is continuous at \text{x}=\pi, then
\lim_\limits{\text{x}\rightarrow\pi^-}\text{f(x)}=\lim_\limits{\text{x}\rightarrow\pi^+}\text{f(x)}
\Rightarrow\text{k}\pi+1=-1
\Rightarrow\text{k}=\frac{-2}{\pi}
Q255. If \text{y}=\sqrt{\text{a}^2-\text{x}^2}, prove that \text{y}\frac{\text{dy}}{\text{dx}}+\text{x}=0 4 Marks

Ans: DIfferentiating with respect to x,


\frac{\text{dy}}{\text{dx}}=\frac{\text{d}}{\text{dx}}\big(\sqrt{\text{a}^2-\text{x}^2}\big)
=\frac{1}{2\sqrt{\text{a}^2-\text{x}^2}}\frac{\text{d}}{\text{dx}}\big(\text{a}^2-\text{x}^2\big)
[Using chain rule]
=\frac{1}{2\sqrt{\text{a}^2-\text{x}^2}}(-2\text{x})
=\frac{-\text{x}}{\sqrt{\text{a}^2-\text{x}^2}}
\Rightarrow\frac{\text{dy}}{\text{dx}}=\frac{-\text{x}}{\text{y}}
\big[\text{Since},\sqrt{\text{a}^2-\text{x}^2}=\text{y}\big]
\Rightarrow\text{y}\frac{\text{dy}}{\text{dx}}=-\text{x}
Hence, the solution is, \text{y}\frac{\text{dy}}{\text{dx}}+\text{x}=0
Q256. Verify Rolle's theorem of the following function on the indicated interval 4 Marks
\text{f}(\text{x})=\cos2\text{x}\text{ on }\Big[\frac{-\pi}{4},\frac{\pi}{4}\Big]

Ans: Here
\text{f}(\text{x})=\cos2\text{x}\text{ on }\Big[\frac{-\pi}{4},\frac{\pi}{4}\Big]
We know that \cos\text{x} is continuous and differentiable everywhere. So, f(x) is continuous in \Big[\frac{-\pi}{4},\frac{\pi}{4}\Big] and differentiable is \Big(\frac{-\pi}{4},\frac{\pi}{4}\Big).
Now,
\text{f}\Big(-\frac{\pi}{4}\Big)=\cos2\Big(-\frac{\pi}{4}\Big)=\cos\Big(-\frac{\pi}{2}\Big)=0
\text{f}\Big(\frac{\pi}{4}\Big)=\cos2\Big(\frac{\pi}{4}\Big)=\cos\Big(\frac{\pi}{2}\Big)=0
\Rightarrow\text{f}\Big(-\frac{\pi}{4}\Big)=\text{f}\Big(\frac{\pi}{4}\Big)
So, Rolle's theorem is applicable, so, there must exist a \text{c}\in\Big(0,\frac{\pi}{2}\Big) such that f'(c) = 0
Now,
\text{f}'(\text{x})=2\sin2\text{x}
\text{f}'(\text{c})=2\sin2\text{c}=0
\Rightarrow\sin2\text{c}=0
\Rightarrow2\text{c}=0
\Rightarrow\text{c}=0\in\Big(\frac{-\pi}{4},\frac{\pi}{4}\Big)
Thus, Rolle's theorem verified.
Q257. If \text{y}=\sin^{-1}\big(6\text{x}\sqrt{1-9\text{x}^2}\big), -\frac{1}{3\sqrt{2}}<\text{x}<\frac{1}{3\sqrt{2}}, then find \frac{\text{dy}}{\text{dx}}. 4 Marks

Ans: We have, \text{y}=\sin^{-1}\big(6\text{x}\sqrt{1-9\text{x}^2}\big), -\frac{1}{3\sqrt{2}}<\text{x}<\frac{1}{3\sqrt{2}}


So, \frac{\text{dy}}{\text{dx}}=\frac{\text{d}}{\text{dx}}\big[\sin^{-1}\big(6\text{x}\sqrt{1-9\text{x}^2}\big)\big]

https://bls.smartstudies.co.in/#/exam/pdf-preview/c59cb220-8e86-4716-9ff7-82aec16b1ade/1 63/158
5/26/24, 6:19 PM Exam Automation
=\frac{\text{d}}{\text{dx}}\Big[\sin^{-1}\big(6\text{x}\sqrt{1-9\text{x}^2}\big)\Big]
=\frac{1}{\sqrt{1-\big(6\text{x}\sqrt{1-9\text{x}^2}\big)^2}}\times\frac{\text{d}}{\text{dx}}\big(6\text{x}\sqrt{1-9\text{x}^2}\big)
=\frac{1}{\sqrt{1-[36\text{x}^2(1-9\text{x}^2)]}}\times\Big(6\text{x}\frac{\text{d}}{\text{dx}}\sqrt{1-9\text{x}^2}+\sqrt{1-9\text{x}^2}\frac{\text{d}}{\text{dx}}(6\text{x})\Big)
=\frac{1}{\sqrt{1-36\text{x}^2-324\text{x}^4}}\times\Big(6\text{x}\times\frac{1}{2\sqrt{1-9\text{x}^2}}\frac{\text{d}}{\text{dx}}(1-9\text{x}^2)+\sqrt{1-9\text{x}^2}(6)\Big)
=\frac{1}{\sqrt{1-36\text{x}^2-324\text{x}^4}}\times\Big(6\text{x}\times\frac{1}{2\sqrt{1-9\text{x}^2}}(-18\text{x}^2)+6\sqrt{1-9\text{x}^2}\Big)
=\frac{1}{\sqrt{1-36\text{x}^2-324\text{x}^4}}\times\Big(\frac{-54\text{x}^2}{\sqrt{1-9\text{x}^2}}+6\sqrt{1-9\text{x}^2}\Big)
=\frac{1}{\sqrt{1-36\text{x}^2-324\text{x}^4}}\times\Big(\frac{-54\text{x}^2+6\sqrt{1-9\text{x}^2}}{\sqrt{1-9\text{x}^2}}\Big)
=\frac{-54\text{x}^2+6-54\text{x}}{\sqrt{1-9\text{x}^2}\sqrt{1-36\text{x}^2-324\text{x}^4}}
=\frac{6-108\text{x}}{\sqrt{1-9\text{x}^2}\sqrt{1-36\text{x}^2-324\text{x}^4}}
Q258. Given the function \text{f(x)}=\frac{1}{\text{x}+2}. Find the points of discontinuity of the composite function y = f(f(x)). 4 Marks

Ans: We have, \text{f(x)}=\frac{1}{\text{x}+2}


\therefore\ \text{y}=\text{f}\big\{\text{f(x)}\big\}
=\text{f}\Big(\frac{1}{\text{x}+2}\Big)=\frac{1}{\frac{1}{\text{x}+2}+2}
=\frac{1}{1+2\text{x}+4}\cdot(\text{x}+2)=\frac{(\text{x}+2)}{(2\text{x}+5)}
So, the function y will not be continuous at those points, where it is not defined as it is a rational function.
Therefore, \text{y}=\frac{(\text{x}+2)}{(2\text{x}+5)} is not defined, when 2x + 5 = 0
\therefore\ \text{x}=\frac{-5}{2}
Hence, y is discontinuous at \text{x}=\frac{-5}{2}
Q259. Find \frac{\text{dy}}{\text{dx}} of the functions expressed in parametric: 4 Marks
\text{x}=\text{e}^\theta\Big(\theta+\frac{1}{\theta}\Big),\text{ y}=\text{e}^{-\theta}\Big(\theta-\frac{1}{\theta}\Big)

Ans: We have, \text{x}=\text{e}^\theta\Big(\theta+\frac{1}{\theta}\Big) and \text{y}=\text{e}^{-\theta}\Big(\theta-\frac{1}{\theta}\Big)


\therefore\ \frac{\text{dx}}{\text{d}}=\frac{\text{d}}{\text{d}\theta}\bigg[\text{e}^\theta\Big(\theta+\frac{1}{\theta}\Big)\bigg]
=\text{e}^\theta\frac{\text{d}}{\text{d}\theta}\Big(\theta+\frac{1}{\theta}\Big)+\Big(\theta+\frac{1}{\theta}\Big)\frac{\text{d}}{\text{d}\theta}\text{e}^\theta =\text{e}^\theta\Big(1-\frac{1}
{\theta^2}\Big)+\Big(\theta+\frac{1}{\theta}\Big)\text{e}^\theta
=\text{e}^\theta\Big(1-\frac{1}{\theta^2}+\theta+\frac{1}{\theta}\Big) =\text{e}^\theta\Big(\frac{\theta^2-1+\theta^3+\theta}{\theta^2}\Big)\ \ \dots(\text{i})
and \frac{\text{dy}}{\text{d}\theta}=\frac{\text{d}}{\text{d}\theta}\bigg[\text{e}^{-\theta}\cdot\Big(\theta-\frac{1}{\theta}\Big)\bigg] =\text{e}^{-\theta}\frac{\text{d}}
{\text{d}\theta}\Big(\theta-\frac{1}{\theta}\Big)+\Big(\theta-\frac{1}{\theta}\Big)\frac{\text{d}}{\text{d}\theta}\text{e}^{-\theta}
=\text{e}^{-\theta}\Big(1+\frac{1}{\theta^2}\Big)-\Big(1-\frac{1}{\theta}\Big)\text{e}^{-\theta} =\text{e}^{\theta}\Big[\frac{\theta^2+1-\theta^3+\theta}{\theta^2}\Big]\ \ \dots(\text{ii})
\therefore\ \frac{\text{dy}}{\text{dx}}=\frac{\frac{\text{dy}}{\text{d}\theta}}{\frac{\text{dx}}{\text{d}\theta}}=\frac{\text{e}^{-\text{e}\Big(\frac{\theta^2+1-\theta^3+\theta}
{\theta^2}\Big)}}{\text{e}^\theta\Big(\frac{\theta^2-1+\theta^3+\theta}{\theta^2}\Big)} =\text{e}^{-2\theta}\Big(\frac{-\theta^3+\theta^2+\theta+1}{\theta^3+\theta^2+\theta-1}\Big)
Q260. Differentiate the following w.r.t. x: 4 Marks
\cos^{-1}\Big(\frac{\sin\text{x}+\cos\text{x}}{\sqrt{2}}\Big),\frac{-\pi}{4}<\text{x}<\frac{\pi}{4}

Ans: Let \text{y}=\cos^{-1}\Big(\frac{\sin\text{x}+\cos\text{x}}{\sqrt{2}}\Big)


\therefore\ \frac{\text{dy}}{\text{dx}}=\frac{\text{d}}{\text{dx}}\cos^{-1}\Big(\frac{\sin\text{x}+\cos\text{x}}{\sqrt{2}}\Big)
=\frac{-1}{\sqrt{1-\Big(\frac{\sin\text{x}+\cos\text{x}}{\sqrt{2}}\Big)^2}}\cdot\frac{\text{d}}{\text{dx}}\Big(\frac{\sin\text{x}+\cos\text{x}}{\sqrt{2}}\Big)
\Big[\because\ \frac{\text{d}}{\text{dx}}(\cos\text{x})=-\frac{1}{\sqrt{1-\text{x}^2}}\Big]
=\frac{-1}{\sqrt{4-\frac{(\sin^2\text{x}+\cos^2\text{x}+2\sin\text{x}\cdot\cos\text{x})}{2}}}\cdot\frac{1}{\sqrt{2}}(\cos\text{x}-\sin\text{x})
=\frac{-1\cdot\sqrt{2}}{\sqrt{1-\sin2\text{x}}}\cdot\frac{1}{\sqrt{2}}(\cos\text{x}-\sin\text{x})
\big[\because1-\sin2\text{x}=(\cos\text{x}-\sin\text{x})^2=\cos^2\text{x}+\sin^2\text{x}-2\sin\text{x}\cos\text{x}\big]
=\frac{-1(\cos\text{x}-\sin\text{x})}{(\cos\text{x}-\sin\text{x})}=-1
Q261. Let f: N \rightarrow N be defined by 4 Marks
\text{f(n)} = \begin{cases} \frac{\text{n + 1}}{2}, & \text{if n is odd}\\ \frac{\text{n}}{2},& \text{if n is even}\\ \end{cases}for all n \in N.
Find whether the function f is bijective.

Ans: For n = 1, we have


\text{f(1)}=\frac{1+1}{2}=\frac{2}{2}=1
For n = 2, we have
\text{f(2)}=\frac{2}{2}=1
Thus, f(1) = f(2) for 1 ≠ 2
So, f is not one-one as two distinct elements in the domain have the same image under function f.
Suppose n be an arbitrary element of N.
If n is odd natural number, then 2n –1 is also an odd natural number.
\therefore{\text{f (2n - 1)}}=\frac{\text{2n - 1 + 1}}{2}=\frac{\text{2n}}{2}=\text{n}
If n is even natural number, then 2n is also an even natural number.
\therefore{\text{f (2n)}}=\frac{\text{2n}}{2}=\text{n}
Thus, for every n in the codomain of f, there exists its pre image in the domain N. Hence, the range of f is equal to the codomain of f. So, fis onto.
The function is not one-one but onto. Hence, f is not bijective.
Q262. If (\cos\text{x})^\text{y}=(\cos\text{y})^\text{x}, find \frac{\text{dy}}{\text{dx}} 4 Marks

Ans: (\cos\text{x})^\text{y}=(\cos\text{y})^\text{x}
Taking log on both sides we get
\text{y}\log\cos\text{x}=\text{x}\log\cos\text{y}
\Rightarrow\frac{\text{dy}}{\text{dx}}\log\cos\text{x}-\text{y}\tan\text{x}=\log\cos\text{y}-\text{x}\tan\text{y}\frac{\text{dy}}{\text{dx}}
\Rightarrow\frac{\text{dy}}{\text{dx}}\log\cos\text{x}+\text{x}\tan\text{y}\frac{\text{dy}}{\text{dx}}=\log\cos\text{y}+\text{y}\tan\text{x}
\Rightarrow\frac{\text{dy}}{\text{dx}}(\log\cos\text{x}+\text{x}\tan\text{y})=\log\cos\text{y}+\text{y}\tan\text{x}
\Rightarrow\frac{\text{dy}}{\text{dx}}=\frac{\log\cos\text{y}+\text{y}\tan\text{x}}{\log\cos\text{x}+\text{x}\tan\text{y}}
Q263. Differentiate w.r.t. x the function in Exercise: 4 Marks
(\sin\text{x}-\cos\text{x)}^{(\sin\text{x}-\cos\text{x})},\ \frac{\pi}{4}<\text{x}<\frac{3\pi}{4}

Ans: Let \text{y}=(\sin\text{x}-\cos\text{x)}^{(\sin\text{x}-\cos\text{x})}


Tanking logarithm on both the sides, we obtain
\log\text{y}=\log\big[(\sin\text{x}-\cos\text{x)}^{(\sin\text{x}-\cos\text{x})}\big]
\Rightarrow\ \log\text{y}=(\sin\text{x}-\cos\text{x)}.\log(\sin\text{x}-\cos\text{x})
Differentiating both sides with respect to x, we obtain
\frac{1}{\text{y}}\frac{\text{dy}}{\text{dx}}=\frac{\text{d}}{\text{dx}}[(\sin\text{x}-\cos\text{x})\log(\sin\text{x}-\cos\text{x})]
\Rightarrow\ \frac{1}{\text{y}}\frac{\text{dy}}{\text{dx}}=\log(\sin\text{x}-\cos\text{x}).\frac{\text{d}}{\text{dx}}(\sin\text{x}-\cos\text{x})+(\sin\text{x}-\cos\text{x}).\frac{\text{d}}
{\text{dx}}\log(\sin\text{x}-\cos\text{x})
\Rightarrow\ \ \frac{1}{\text{y}}\frac{\text{dy}}{\text{dx}}=\log(\sin\text{x}-\cos\text{x}).(\cos\text{x}+\sin\text{x})+(\sin\text{x}-\cos\text{x}).\frac{1}{(\sin\text{x}-
\cos\text{x})}.\frac{\text{d}}{\text{dx}}(\sin\text{x}-\cos\text{x)}
\Rightarrow\ \frac{\text{dy}}{\text{dx}}=(\sin\text{x}-\cos\text{x)}^{(\sin\text{x}-\cos\text{x})}[(\cos\text{x}+\sin\text{x}).\log(\sin\text{x}-\cos\text{x})+(\cos\text{x}+\sin\text{x})]
\therefore\ \frac{\text{dy}}{\text{dx}}=(\sin\text{x}-\cos\text{x})^{(\sin\text{x}-\cos\text{x})}(\cos\text{x}+\sin\text{x})[1+\log(\sin\text{x}-\cos\text{x})]
Q264. Differentiate the following functions with respect to x: 4 Marks
\text{e}^{\tan^{-1}\sqrt{\text{x}}}

Ans: Let, \text{y}=\text{e}^{\tan^{-1}\sqrt{\text{x}}}


Differentiate it with respect to x we get,

https://bls.smartstudies.co.in/#/exam/pdf-preview/c59cb220-8e86-4716-9ff7-82aec16b1ade/1 64/158
5/26/24, 6:19 PM Exam Automation
\frac{\text{dy}}{\text{dx}}=\frac{\text{d}}{\text{dx}}\big(\text{e}^{\tan^{-1}\sqrt{\text{x}}}\big)
=\text{e}^{\tan^{-1}\sqrt{\text{x}}}\frac{\text{d}}{\text{dx}}\big(\tan^{-1}\sqrt{\text{x}}\big)
[Using chain rule]
=\text{e}^{\tan^{-1}\sqrt{\text{x}}}\times\frac{1}{1+(\sqrt{\text{x}})^2}\frac{\text{d}}{\text{dx}}\big(\sqrt{\text{x}}\big)
=\frac{\text{e}^{\tan^{-1}\sqrt{\text{x}}}}{1+\text{x}}\times\frac{1}{2\sqrt{\text{x}}}
=\frac{\text{e}^{\tan^{-1}\sqrt{\text{x}}}}{2\sqrt{\text{x}}(1+\text{x})}
So,
\frac{\text{d}}{\text{dx}}\Big(\text{e}^{\tan^{-1}\sqrt{\text{x}}}\Big)=\frac{\text{e}^{\tan^{-1}\sqrt{\text{x}}}}{2\sqrt{\text{x}}(1+\text{x})}
Q265. Verify the Rolle’s theorem for each of the functions: 4 Marks
\text{f(x)}=\text{x}(\text{x}+3)\text{e}^{-\frac{\text{x}}{2}}\text{ in }[-3,0].

Ans: We have, \text{f(x)}=\text{x}(\text{x}+3)\text{e}^{-\frac{\text{x}}{2}}


Since polynomial function x(x + 3) and exponential funxtion \text{e}^{-\frac{\text{x}}{2}} are continuous and diffferentiable in R, given function f(x) is also continuous and differentiable in R
Also f(0) = f(-3) = 0
So, conditions of Rolle's theorem are satisfied.
Hence, there exists a real number \text{c}\in(-3,0) such that f'(c) = 0
Now \text{f(x)}=(\text{x}^2+3\text{x})\text{e}^{-\frac{\text{x}}{2}}
\therefore\ \text{f}'(\text{x})=(2\text{x}+3)\text{e}^{-\frac{\text{x}}{2}}-\frac{1}{2}\text{e}^{-\frac{\text{x}}{2}}(\text{x}^2+3\text{x})
=-\frac{1}{2}\text{e}^{-\frac{\text{x}}{2}}(\text{x}^2+3\text{x}-4\text{x}-6)=-\frac{1}{2}\text{e}^{-\frac{\text{x}}{2}}(\text{x}^2-\text{x}-6)
So, f'(c) = 0
\Rightarrow\ -\frac{1}{2}\text{e}^{-\frac{\text{x}}{2}}(\text{c}+2)(\text{c}-3)=0
\Rightarrow\ \text{c}=-2\in(-3,0)
Therefore, Rolle's theorem has been verified.
Q266. If \text{y}=(\text{x})^{\cos\text{x}}+(\cos\text{x})^{\sin\text{x}}, find \frac{\text{dy}}{\text{dx}}. 4 Marks

Ans: Let \text{u}=\text{x}^\text{cos x},\text{v}=(\cos\text{x})^\text{sin x}


\text{y}=\text{u}+\text{v}
\Rightarrow\frac{\text{dy}}{\text{dx}}=\frac{\text{du}}{\text{dx}}+\frac{\text{dv}}{\text{dx}}\ \dots(1)
\log\text{u}=\cos \text{x}\log \text{x}
\Rightarrow \frac{1}{\text{u}}\frac{\text{du}}{\text{dx}}=\frac{\cos \text{x}}{\text{x}}-\sin\text{x}\log \text{x}
\frac{\text{du}}{\text{dx}}=\text{x}^\text{cos x}\Big(\frac{\cos\text{x}}{\text{x}}-\sin\text{x}\log\text{x}\Big)
\log \text{v}=\sin \text{x}\log \cos\text{x}
\Rightarrow\frac{1}{\text{v}}\frac{\text{dv}}{\text{dx}}=-\sin\text {x}+\tan\text{x} +\cos\text{x}\ \log \cos \text{x}
\Rightarrow \frac{\text{dv}}{\text{dx}}=(\cos\text{x})^\text{sin x}(-\sin\text{x}\tan\text{x}+\cos\text{x}\log\cos\text{x})
So,
\frac{\text{dy}}{\text{dx}}=\text{ x}^\text{cos x}\Big(\frac{\cos\text{x}}{\text{x}}-\sin\text{x}\log\text{x}\Big)+(\cos\text{x})^{\sin\text{x}}\\\ \ \ \ \ \ \ \ \ (-\sin\text{
x}\tan\text{x}+\cos\text{x}\log\cos\text{x})
Q267. Differentiate the following functions with respect to x: 4 Marks
\text{e}^{3\text{x}}\cos2\text{x}

Ans: Consider \text{y}=\text{e}^{3\text{x}}\cos2\text{x}


Differentiate with respect to x,
\frac{\text{dy}}{\text{dx}}=\frac{\text{d}}{\text{dx}}\text{e}^{3\text{x}}\cos2\text{x}
=\text{e}^{3\text{x}}\times\frac{\text{d}}{\text{dx}}(\cos2\text{x})+\cos2\text{x}\frac{\text{d}}{\text{dx}}(\text{e}^{3\text{x}})
[Using chain rule]
=\text{e}^{3\text{x}}\times(-\sin2\text{x})\frac{\text{d}}{\text{dx}}(2\text{x})+\cos2\text{xe}^{3\text{x}}\frac{\text{d}}{\text{dx}}(3\text{x})
[Using chain rule]
=-2\text{e}^{3\text{x}}\sin2\text{x}+3\text{e}^{3\text{x}}\cos2\text{x}
=\text{e}^{3\text{x}}(3\cos2\text{x}-2\sin2\text{x})
Hence, the solution is, \frac{\text{d}}{\text{dx}}(\text{e}^{3\text{x}}\cos2\text{x})=\text{e}^{3\text{x}}(3\cos2\text{x}-2\sin2\text{x})
Q268. If \text{y}=\text{x}\sin^{-1}\text{x}+\sqrt{1-\text{x}^2}, prove that \frac{\text{dy}}{\text{dx}}=\sin^{-1}\text{x} 4 Marks

Ans: We have, \text{y}=\text{x}\sin^{-1}\text{x}+\sqrt{1-\text{x}^2}


Differentiate it with respect to x,
\frac{\text{dy}}{\text{dx}}=\frac{\text{d}}{\text{dx}}\Big[\text{x}\sin^{-1}\text{x}+\sqrt{1-\text{x}^2}\Big]
=\frac{\text{d}}{\text{dx}}\big(\text{x}\sin^{-1}\text{x}\big)+\frac{\text{d}}{\text{dx}}\big(\sqrt{1-\text{x}^2}\big)
=\Big[\text{x}\frac{\text{d}}{\text{dx}}\sin^{-1}\text{x}+\sin^{-1}\text{x}\frac{\text{d}}{\text{dx}}(\text{x})\Big]+\frac{1}{2\sqrt{1-\text{x}^2}}\frac{\text{d}}{\text{dx}}(1-\text{x}^2)
=\Big[\frac{\text{x}}{\sqrt{1-\text{x}^2}}+\sin^{-1}\text{x}\Big]-\frac{2\text{x}}{2\sqrt{1-\text{x}^2}}
=\frac{\text{x}}{\sqrt{1-\text{x}^2}}+\sin^{-1}\text{x}-\frac{\text{x}}{\sqrt{1-\text{x}^2}}
=\sin^{-1}\text{x}
Q269. If \text{x}=\text{a}\Big(\frac{1+\text{t}^2}{1-\text{t}^2}\Big)\text{ and y}=\frac{2\text{t}}{1-\text{t}^2}, find \frac{\text{dy}}{\text{dx}} 4 Marks

Ans: We have, \text{x}=\text{a}\Big(\frac{1+\text{t}^{2}}{1-\text{t}^{2}}\Big)


\Rightarrow\frac{\text{dx}}{\text{dt}}=\text{a}\bigg[\frac{(1-\text{t}^{2})\frac{\text{d}}{\text{dt}}(1+\text{t}^{2})-(1+\text{t}^{2})\frac{\text{d}}{\text{dt}}(1-\text{t}^{2})}{(1-
\text{t}^{2})}\bigg]
[Using quotient rule]
\Rightarrow\frac{\text{dx}}{\text{dt}}=\text{a}\bigg[\frac{(1-\text{t}^{2})(2\text{t)}-(1+\text{t}^{2})(-2\text{t})}{(1-\text{t}^{2})^{2}}\bigg]
\Rightarrow\frac{\text{dx}}{\text{dt}}=\text{a}\bigg[\frac{2\text{t}-2\text{t}^{3}+2\text{t}+2\text{t}^{3}}{(1-\text{t}^{2})^{2}}\bigg]
\Rightarrow\frac{\text{dx}}{\text{dt}}=\frac{4\text{a}\text{t}}{(1-\text{t}^{2})^{2}}\ .....(\text{i})
and, \text{y}=\frac{2\text{t}}{1-\text{t}^{2}}
\Rightarrow\frac{\text{dx}}{\text{dt}}=2\bigg[\frac{(1-\text{t}^{2})\frac{\text{d}}{\text{dt}}(\text{t})-\text{t}\frac{\text{d}}{\text{dt}}(1-\text{t}^{2})}{(1-\text{t}^{2})^{2}}\bigg]
\Rightarrow\frac{\text{dx}}{\text{dt}}=2\bigg[\frac{(1-\text{t}^{2})(1)-\text{t}(-2\text{t})}{(1-\text{t}^{2})^{2}}\bigg]
\Rightarrow\frac{\text{dx}}{\text{dt}}=2\bigg[\frac{(1-\text{t}^{2})+2\text{t}^{2}}{(1-\text{t}^{2})^{2}}\bigg]
\Rightarrow\frac{\text{dy}}{\text{dt}}=\frac{2(1-\text{t}^{2})}{(1-\text{t}^{2})^{2}}\ .....(\text{ii})
Dividing equation (ii) by (i),
\frac{\frac{\text{dy}}{\text{dt}}}{\frac{\text{dx}}{\text{dt}}}=\frac{2(1+\text{t}^{2})}{(1-\text{t}^{2})^{2}}\times\frac{(1-\text{t}^{2})^{2}}{4\text{a}\text{t}}
\Rightarrow\frac{\text{dy}}{\text{dx}}=\frac{(1+\text{t}^{2})}{2\text{a}\text{t}}
Q270. Show that the function f defined as follows, is continuous at x = 2, but not differentiable thereat: 4 Marks
f(x) = \begin{matrix} \text{3x - 2}, && 0<x\leq1 \\ \text{ 2x}^{2}-\text{x}, && 1<x\leq2 \\ \text{5x - 4}, && x>2 \end{matrix}.

Ans: \DeclareMathOperator*{\median}{\text{lim}} \median_{\text{h}\rightarrow0}\text{ f(2 - h)}=\DeclareMathOperator*{\median}{\text{lim}} \median_{\text{h}\rightarrow0}[2(2 -


\text{h})^{2}-(2 - \text{h})]=6\ .......\text{(i)}
\DeclareMathOperator*{\median}{\text{lim}} \median_{\text{h}\rightarrow0}\text{ f(2 + h)}=\DeclareMathOperator*{\median}{\text{lim}} \median_{\text{h}\rightarrow0}[5(2 + \text{h})-4
]=6\ ............\text{(ii)}
f(2) = 8 – 2 = 6 ...........(iii)
From (i), (ii), and (iii), f(x) is continuous at x = 2
\text{RHD}=\DeclareMathOperator*{\median}{\text{lim}} \median_{\text{h}\rightarrow0}\Bigg[\frac{\left\{5{(2+h)-4}-(6)\right\}}{h}\Bigg]\neq\text{LHD}=\DeclareMathOperator*
{\median}{\text{lim}} \median_{\text{h}\rightarrow0}\Bigg[\frac{\left\{(2h - 3)(h - 2) - 6\right\}}{-h}\Bigg]\text{as 5}\neq7
\therefore f(x) is not differentiable there at.

https://bls.smartstudies.co.in/#/exam/pdf-preview/c59cb220-8e86-4716-9ff7-82aec16b1ade/1 65/158
5/26/24, 6:19 PM Exam Automation
Q271. If x = a sin 2t (1 + cos 2t) and y = b cos 2t (1 – cos 2t), show that at \text{t}=\frac{\pi}{4},\big(\frac{\text{dy}}{\text{dx}}\big)=\frac{\text{b}}{\text{a}} 4 Marks

Ans: \frac{\text{dx}}{\text{dt}}= 2a cos 2t (1 + cos 2t) – 2a sin22t


\frac{\text{dy}}{\text{dt}}= 2b cos 2t sin 2t – 2b sin 2t (1 – cos 2t)
\therefore\ \frac{\text{dy}}{\text{dx}}=\frac{\text{b}}{\text{a}}\Bigg(\frac{\sin2\text{t}\cos2\text{t}-\sin2\text{t}(1-\cos2\text{t})}{\cos2\text{t}(1+\cos2\text{t})-\sin^22\text{t}}\Bigg)
At t = \frac{\pi}{4} sin 2t = 1 and cos 2t = 0
\therefore\ \frac{\text{dy}}{\text{dx}}\big(\text{at t}=\frac{\pi}{4}\big)=\frac{\text{b}}{\text{a}}\Big(\frac{0-1}{0-1}\Big)
Q272. Differentiate the following functions with respect to x: 4 Marks
\sin(2\sin^{-1}\text{x})

Ans: Let, \text{y}=\sin(2\sin^{-1}\text{x})


Differentiate it with respect to x,
\frac{\text{dy}}{\text{dx}}=\frac{\text{d}}{\text{dx}}\Big(\sin(2\sin^{-1}\text{x})\Big)
=\cos\big(2\sin^{-1}\text{x}\big)\frac{\text{d}}{\text{dx}}\big(2\sin^{-1}\text{x}\big)
[Using chain rule]
=\cos\big(2\sin^{-1}\text{x}\big)\times2\frac{1}{\sqrt{1-\text{x}^2}}
=\frac{2\cos\big(2\sin^{-1}\text{x}\big)}{\sqrt{1-\text{x}^2}}
So,
\frac{\text{d}}{\text{dx}}\Big(\sin\big(2\sin^{-1}\text{x}\big)\Big)=\frac{2\cos\big(2\sin^{-1}\text{x}\big)}{\sqrt{1-\text{x}^2}}
Q273. \text{If x = a}\sin 2\text{t} (1 + \cos\text{2t) and y = b}\cos\text{2t (1} - \cos \text{2t)}, find the values of \frac{\text{dy}}{\text{dx}} \text{at t} = \frac{\pi}{4} \text{and t} \frac{\pi}{3}. 4 Marks

Ans: \text{Here x = a} \bigg(\sin \text{2t} + \frac{1}{2}\sin \text{4t }\bigg).\text{y = b}(\cos \text{2t} - \cos^{2}\text{2t})
\frac{\text{dx}}{\text{dt}} = 2\text{a}[\cos\text{2t}+ \cos\text{4t}], \frac{\text{dy}}{\text{dt}} = \text{2b}[-\sin \text{2t} + 2\cos\text{2t}\sin \text{2t}] = \text{2b}[\sin\text{4t} - \sin\text{2t}]
\frac{\text{dy}}{\text{dx}} = \frac{\text{b}}{\text{a}} \bigg[\frac{\sin\text{4t}- \sin\text{2t}}{\cos\text{4t} + \cos \text{2t}}\bigg]
\frac{\text{dy}}{\text{dx}}\bigg]_{\text{t} = \frac{\pi}{4}} = \frac{\text{b}}{\text{a}}
\text{and} \frac{\text{dy}}{\text{dx}}\bigg]_{\text{t} = \frac{\pi}{3}} = \sqrt{3}\frac{\text{b}}{\text{a}}
Q274. If y = 2 \cos (\log x) + 3 \sin (\log x), Prove that x^{2} \frac{d^{2}y}{dx}^{2} + x\frac{dy}{dx} + y = 0. 4 Marks

Ans: \frac{\text{dy}}{\text{dx}} = \frac{-2\sin(\log \text{x})}{\text{x}} + \frac{3\cos(\log \text{x})}{\text{x}}


\Rightarrow\text{x}\frac{\text{dy}}{\text{dx}} = -2\sin(\log\text{x}) + 3 \cos(\log\text{x}), \text{differentiate w.r.t 'x'}
\Rightarrow\text{x}\frac{\text{d}^{2}\text{y}}{\text{dx}^{2}} + \frac{\text{dy}}{\text{dx}} = \frac{-2\cos(\log \text{x})}{\text{x}} - \frac{3\sin(\log\text{x})}{\text{x}}
\Rightarrow\text{x}^2\frac{\text{d}^{2}\text{y}}{\text{dx}^{2}} + \text{x}\frac{\text{dy}}{\text{dx}} = -\text{y} \Rightarrow\text{x}^{2}\frac{\text{d}^{2}\text{y}}{\text{dx}^{2}} +
\text{x}\frac{\text{dy}}{\text{dx}} +\text{y} = 0
Q275. If \text{x}=\text{a}\cos\theta,\text{y}=\text{b}\sin\theta Show that \frac{\text{d}^2\text{y}}{\text{dx}^2}=-\frac{\text{b}^4}{\text{a}^2\text{y}^3} 4 Marks

Ans: Given,
\text{x}=\text{a}\cos\theta\dots\text{ eq. }1
\text{y}=\text{b}=\sin\theta\dots\text{ eq. }2
To prove: \frac{\text{d}^2\text{y}}{\text{dx}^2}=-\frac{\text{b}^4}{\text{a}^2\text{y}^3}
Let's find \frac{\text{d}^2\text{y}}{\text{dx}^2}
As \frac{\text{d}^2\text{y}}{\text{dx}^2}=\frac{\text{d}}{\text{dx}}\Big(\frac{\text{dy}}{\text{dx}}\Big)
So, let's first find \frac{\text{dy}}{\text{dx}} using parametric form and defferentiate it again.
\frac{\text{dx}}{\text{d}\theta}=\frac{\text{d}}{\text{d}\theta}\text{a}\cos\theta=-\text{a}\sin\theta\dots\text{ eq. 3}
Similarly, \frac{\text{dy}}{\text{d}\theta}=\text{b}\cos\theta\dots\text{ eq. 4}
\Big[\because\frac{\text{d}}{\text{dx}}\cos\text{x}=-\sin\text{x}\tan\text{x},\frac{\text{d}}{\text{dx}}\sin\text{x}=\cos\text{x}\Big]
\therefore\frac{\text{dy}}{\text{dx}}=\frac{\frac{\text{dy}}{\text{d}\theta}}{\frac{\text{dx}}{\text{d}\theta}}=-\frac{\text{b}\cos\theta}{\text{a}\sin\theta}=-\frac{\text{b}}{\text{a}}\cot\theta
Differentiating again w.r.t. x:
\frac{\text{d}}{\text{dx}}\Big(\frac{\text{dy}}{\text{dx}}\Big)=\frac{\text{d}}{\text{dx}}\Big(-\frac{\text{b}}{\text{a}}\cot\theta\Big)
\frac{\text{d}^2\text{y}}{\text{dx}^2}=\frac{\text{b}}{\text{a}}\text{cosec}^2\theta\frac{\text{d}\theta}{\text{dx}}\dots\text{ eq. 5}
\Big[Using chain rule and \frac{\text{d}}{\text{dx}}\cot\text{x}=-\text{cosec}^2\text{x}\Big]
From equation 3:
\frac{\text{dx}}{\text{d}\theta}=-\text{a}\sin\theta
\therefore\frac{\text{d}\theta}{\text{dx}}=\frac{-1}{\text{a}\sin\theta}
Putting the value in equation 5:
\frac{\text{d}^2\text{y}}{\text{dx}^2}=-\frac{\text{b}}{a}\text{cosec}^2\theta\frac{1}{\text{a}\sin\theta}
\frac{\text{d}^2\text{y}}{\text{dx}^2}=\frac{-\text{b}}{\text{a}^2\sin^3\theta}
From equation 1:
\text{y}=\text{b}\sin\theta
\therefore\frac{\text{d}^2\text{y}}{\text{dx}^2}=\frac{-\text{b}}{\frac{\text{a}^2\text{y}^3}{\text{b}^3}}=-\frac{\text{b}^4}{\text{a}^2\text{y}^3}\dots\text{proved.}
Q276. Find \frac{\text{dy}}{\text{dx}} 4 Marks
\text{y}=\text{e}^{3\text{x}}\sin4\text{x}\times2^\text{x}

Ans: We have, \text{y}=\text{e}^{3\text{x}}\sin4\text{x}\times2^\text{x}\ .....(\text{i})


Taking log on both sides,
\log\text{y}=\log\text{e}^{3\text{x}}+\log\sin4\text{x}+\log2^\text{x}
\Rightarrow\log\text{y}=3\text{x}\log\text{e}+\log\sin4\text{x}+\text{x}\log2
\Rightarrow\log\text{y}=3\text{x}+\log\sin4\text{x}+\text{x}\log2
Differentiating with resepect to x,
\frac{1}{\text{y}}\frac{\text{dy}}{\text{dx}}=\frac{\text{d}}{\text{dx}}(3\text{x})+\frac{\text{d}}{\text{dx}}(\sin4\text{x})+\frac{\text{d}}{\text{dx}}(\text{x}\log2)
\Rightarrow\frac{1}{\text{y}}\frac{\text{dy}}{\text{dx}}=3+\frac{1}{\sin4\text{x}}\frac{\text{d}}{\text{dx}}(\sin4\text{x})+\log2(1)
\Rightarrow\frac{1}{\text{y}}\frac{\text{dy}}{\text{dx}}=3+\frac{1}{\sin4\text{x}}(\cos4\text{x})\frac{\text{d}}{\text{dx}}(4\text{x})+\log2
\Rightarrow\frac{1}{\text{y}}\frac{\text{dy}}{\text{dx}}=3+\cot4\text{x}(4)+\log2
\Rightarrow\frac{1}{\text{y}}\frac{\text{dy}}{\text{dx}}=3+4\cot4\text{x}+\log2
\Rightarrow\frac{\text{dy}}{\text{dx}}=\text{y}\big[3+4\cot4\text{x}+\log2\big]
\Rightarrow\frac{\text{dy}}{\text{dx}}=\text{e}^{3\text{x}}\sin4\text{x}2^\text{x}\big[3+4\cot4\text{x}+\log2\big]
[Using equation (i)]
Q277. Differentiate the following functions with respect to x: 4 Marks
\cos^{-1}\big\{2\text{x}\sqrt{1-\text{x}^2}\big\},\frac{1}{\sqrt{2}}<\text{x}<1

Ans: Let \text{y}=\cos^{-1}\Big\{2\text{x}\sqrt{1-\text{x}^2}\Big\}


Put \text{x}=\cos\theta
\text{y}=\cos^{-1}\Big\{2\cos\sqrt{1-\cos^2\theta}\Big\}
=\cos^{-1}\big\{2\cos\theta\sin\theta\big\}
\text{y}=\cos^{-1}\big\{\sin2\theta\big\}
\big[\text{Since}, \sin2\theta=2\sin\theta\cos\theta,\sin^2\theta+\cos^2\theta=1\big]
\text{y}=\cos^{-1}\Big[\cos\Big(\frac{\pi}{2}-\theta\Big)\Big]\ .....(\text{i})
Now,
\frac{1}{\sqrt{2}}<\text{x}<1
\Rightarrow\frac{1}{\sqrt{2}}<\cos\theta<1

https://bls.smartstudies.co.in/#/exam/pdf-preview/c59cb220-8e86-4716-9ff7-82aec16b1ade/1 66/158
5/26/24, 6:19 PM Exam Automation
\Rightarrow 0<\theta<\frac{\pi}{4}
\Rightarrow 0<2\theta<\frac{\pi}{2}
\Rightarrow 0>-2\theta>-\frac{\pi}{2}
\Rightarrow\frac{\pi}{2}>\Big(\frac{\pi}{2}-2\theta\Big)>0
Hence, from equation (i),
\text{y}=\frac{\pi}{2}-2\theta
\big[\text{Sicne}, \cos^{-1}(\cos\theta)=\theta,\text{ if }\theta\in[0,\pi]\big]
\text{y}=\frac{\pi}{2}-2\cos^{-1}\text{x}\ \big[\text{Since x}=\cos\theta\big]
Differentiating it with respect to x,
\frac{\text{dy}}{\text{dx}}=\frac{\text{d}}{\text{dx}}\Big(\frac{\pi}{2}\Big)-2\frac{\text{d}}{\text{dx}}\big(\cos^{-1}\text{x}\big)
=0-2\Big(\frac{-1}{\sqrt{1-\text{x}^2}}\Big)
\frac{\text{dy}}{\text{dx}}=\frac{2}{\sqrt{1-\text{x}^2}}
Q278. Find the values of p and q, for which 4 Marks
\text{f(x)} = \begin{matrix} \frac{1- sin^{3}x}{3\cos^{2}x} & , & \text{if x} < \frac{\pi}{2} \\ \text{p} & , & \text{if x} = \frac{\pi}{2} \\ \frac{\text{q}(1 - \sin x)}{(\pi - 2x)^{2}} & , & \text{if x}>
\frac{\pi}{2} \end{matrix}
is continuous at x = \frac{\pi}{2}.

Ans: \text{L.H.S} = \lim\limits_{x\rightarrow\frac{\pi}{2}^{-}} \frac{(1 - \sin\text{x)}(1 + \sin \text{x} + \sin^{2}\text{x})}{3(1 - \sin\text{x)}(1 + \sin\text{x})}
= \frac{1}{2}
\therefore \text{p} = \frac{1}{2}
\text{R.H.S} = \lim\limits_{\text{x}\rightarrow\frac{\pi}{2}^{+}} \frac{\text{q}(1 - \sin\text{x)}}{(\pi - 2\text{x)}^{2}} =\lim\limits_{h\rightarrow 0} \frac{\text{q}(1 - \cos\text{h)}}
{(2\text{h})^{2}}, \text{where x -}\frac{\pi}{2} = \text{h}
= \lim\limits_{\text{h}\rightarrow{0}} \frac{2\text{q} \sin^{2}\frac{\text{h}}{2}}{4.4.\frac{\text{h}^{2}}{4}}= \frac{\text{q}}{8}
\therefore \frac{\text{q}}{8} = \frac{1}{2} \Rightarrow \text{q} = 4
Q279. Show that \text{f}(\text{x})=\text{x}^\frac{1}{3} is not differentible at x = 0. 4 Marks

Ans: \text{f}(\text{x})=\text{x}^\frac{1}{3}
(LHL at x = 0) =\lim_\limits{\text{x}\rightarrow0^{-}}\frac{\text{f}(\text{x})-\text{f}(0)}{\text{x}-0}
=\lim_\limits{\text{x}\rightarrow0}\frac{\text{f}(0-\text{h})-\text{f}(0)}{0-\text{h}-0}
=\lim_\limits{\text{x}\rightarrow0^{-}}\frac{\text{f}(\text{x})-\text{f}(0)}{\text{x}-0}
=\lim_\limits{\text{x}\rightarrow0}\frac{(-\text{h})^\frac{1}{3}}{-\text{h}}
=\lim_\limits{\text{x}\rightarrow0}\frac{(-1)^\frac{1}{3}\text{h}^\frac{1}{3}}{(-1)-\text{h}}
=\lim_\limits{\text{x}\rightarrow0}(-1)^\frac{-2}{3}\text{h}^\frac{-2}{3}
= Not defined
(RHL at x = 0) =\lim_\limits{\text{x}\rightarrow0^{+}}\frac{\text{f}(\text{x})-\text{f}(0)}{\text{x}-0}
=\lim_\limits{\text{x}\rightarrow0}\frac{\text{f}(0+\text{h})-\text{f}(0)}{0+\text{h}-0}
=\lim_\limits{\text{x}\rightarrow0}\frac{(\text{h})^\frac{1}{3}-0}{\text{h}}
=\lim_\limits{\text{x}\rightarrow0}\text{h}^\frac{-2}{3}
= Not defined
Since,
LHL and RHL does not exists at x = 0
\therefore f(x) is not differentiable at x = 0
Q280. Find \frac{\text{dy}}{\text{dx}} in the following cases: 4 Marks
\text{x}^{\frac{2}{3}}+\text{y}^{\frac{2}{3}}=\text{a}^{\frac{2}{3}}

Ans: We have, \text{x}^{\frac{2}{3}}+\text{y}^{\frac{2}{3}}=\text{a}^{\frac{2}{3}}


Differentiating it with respect to x, we get,
\frac{\text{d}}{\text{dx}}\Big(\text{x}^{\frac{2}{3}}\Big)+\frac{\text{d}}{\text{dx}}\Big(\text{y}^{\frac{2}{3}}\Big)=\frac{\text{d}}{\text{dx}}\Big(\text{a}^{\frac{2}{3}}\Big)
\Rightarrow\frac{2}{3}\big(\text{x}\big)^{\frac{2}{3}-1}+\frac{2}{3}\big(\text{y}\big)^{\frac{2}{3}-1}\frac{\text{dy}}{\text{dx}}=0
\Rightarrow\frac{2}{3}\big(\text{x}\big)^{\frac{-1}{3}}+\frac{2}{3}\big(\text{y}\big)^{\frac{-1}{3}}\frac{\text{dy}}{\text{dx}}=0
\Rightarrow\frac{2}{3}\big(\text{y}\big)^{\frac{-1}{3}}\frac{\text{dy}}{\text{dx}}=-\frac{2}{3}\big(\text{x}\big)^{\frac{-1}{3}}
\Rightarrow\frac{\text{dy}}{\text{dx}}=-\frac{2}{3}\big(\text{x}\big)^{\frac{-1}{3}}\times\frac{3}{2\text{y}^{\frac{-1}{3}}}
\Rightarrow\frac{\text{dy}}{\text{dx}}=-\frac{\text{x}^{\frac{-1}{3}}}{\text{y}^{\frac{-1}{3}}}
\Rightarrow\frac{\text{dy}}{\text{dx}}=-\frac{\text{x}^{\frac{1}{3}}}{\text{y}^{\frac{1}{3}}}
\Rightarrow\frac{\text{dy}}{\text{dx}}=-\big(\frac{\text{x}}{\text{y}}\big)^{\frac{1}{3}}
Q281. \text{If y} = \tan^{-1} \bigg(\frac{\sqrt{1 + x^{2}}+{\sqrt{1 - x^{2}}}}{\sqrt{1 + x^{2}} - {\sqrt{1 - x^{2}}}}\bigg), x^{2}\leq 1, \text{then find} \frac{dy}{dx}. 4 Marks

Ans: \text{Putting x}^{2} = \cos\theta, \text{we get}


\text{y} = \tan^{-1} \bigg(\frac{\sqrt{1 +\cos \theta}+{\sqrt{1-\cos\theta}}}{\sqrt{1 +\cos\theta} - {\sqrt{1- \cos\theta }}}\bigg)
= \tan^{-1}\bigg(\frac{\cos\theta/2+\sin\theta/2}{\cos\theta/2-{\sin\theta/2}}\bigg)= \tan^{-1} \bigg(\frac{1 + \tan\theta/2}{1 - \tan\theta/2}\bigg)
\text{y} = \frac{\pi}{4} + {\theta}/{2} = \frac{\pi}{4} + \frac{1}{2} \cos^{-1}\text{x}^{2}
\frac{\text{dy}}{\text{dx}} = -\frac{1}{2} \frac{1}{\sqrt{1 - \text{x}^{4}}}. 2\text{x} = -\frac{\text{x}}{\sqrt{1 - \text{x}^{4}}}
Q282. Prove that \frac{\text{dy}}{\text{dx}}\Big\{\frac{\text{x}}{2}\sqrt{\text{a}^2-\text{x}^2}+\frac{\text{a}^2}{2}\sin^{-1}\frac{\text{x}}{\text{a}}\Big\}=\sqrt{\text{a}^2-\text{x}^2} 4 Marks

Ans: \frac{\text{dy}}{\text{dx}}\Big\{\frac{\text{x}}{2}\sqrt{\text{a}^2-\text{x}^2}+\frac{\text{a}^2}{2}\sin^{-1}\frac{\text{x}}{\text{a}}\Big\}=\sqrt{\text{a}^2-\text{x}^2}
\text{L.H.S}=\frac{\text{dy}}{\text{dx}}\Big\{\frac{\text{x}}{2}\sqrt{\text{a}^2-\text{x}^2}+\frac{\text{a}^2}{2}\sin^{-1}\frac{\text{x}}{\text{a}}\Big\}
=\frac{\text{dy}}{\text{dx}}\Big(\frac{\text{x}}{2}\sqrt{\text{a}^2-\text{x}^2}\Big)+\frac{\text{d}}{\text{dx}}\Big(\frac{\text{a}^2}{2}\sin^{-1}\frac{\text{x}}{\text{a}}\Big)
=\frac{1}{2}\Big[\text{x}\frac{\text{d}}{\text{dx}}\sqrt{\text{a}^2-\text{x}^2}+\sqrt{\text{a}^2-\text{x}^2}\frac{\text{d}}{\text{dx}}(\text{x})\Big] \\ +\frac{\text{a}^2}{2}\times\frac{1}{\sqrt{1-
\Big(\frac{\text{x}}{\text{x}}\Big)^2}}\times\frac{\text{d}}{\text{dx}}\Big(\frac{\text{x}}{\text{a}}\Big)
[Using product rule, chain rule]
=\frac{1}{2}\bigg[\text{x}\times\frac{1}{2\sqrt{\text{a}^2-\text{x}^2}}\frac{\text{d}}{\text{dx}}\big(\text{a}^2-\text{x}^2\big)+\sqrt{\text{a}^2-\text{x}^2}\Big] \\ +\Big(\frac{\text{a}^2}
{2}\Big)\times\frac{1}{\sqrt{\frac{\text{a}^2-\text{x}^2}{\text{a}^2}}}\times\Big(\frac{1}{\text{a}}\Big)
=\frac{1}{2}\Big[\frac{\text{x}(-2\text{x})}{2\sqrt{\text{a}^2-\text{x}^2}}+\sqrt{\text{a}^2-\text{x}^2}\Big]+\Big(\frac{\text{a}^2}{2}\Big)\times\frac{1}{\sqrt{\text{a}^2-
\text{x}^2}}\times\Big(\frac{1}{\text{a}}\Big)
=\frac{1}{2}\bigg[\frac{-2\text{x}^2+2\big(\text{a}^2-\text{x}^2\big)}{2\sqrt{\text{a}^2-\text{x}^2}}\bigg]+\frac{\text{a}^2}{2\sqrt{\text{a}^2-\text{x}^2}}
=\frac{1}{2}\bigg[\frac{2\big(\text{a}^2-2\text{x}^2\big)}{2\sqrt{\text{a}^2-\text{x}^2}}\bigg]+\frac{\text{a}^2}{2\sqrt{\text{a}^2-\text{x}^2}}
=\frac{\text{a}^2-2\text{x}^2}{2\sqrt{\text{a}^2-\text{x}^2}}+\frac{\text{a}^2}{2\sqrt{\text{a}^2-\text{x}^2}}
=\frac{\text{a}^2-2\text{x}^2+\text{a}^2}{2\sqrt{\text{a}^2-\text{x}^2}}
=\frac{2\text{a}^2-2\text{x}^2}{2\sqrt{\text{a}^2-\text{x}^2}}
=\frac{2\big(\text{a}^2-\text{a}^2\big)}{2\sqrt{\text{a}^2-\text{x}^2}}
=\frac{(\text{a}^2-\text{x}^2)}{\sqrt{\text{a}^2-\text{x}^2}}
=\sqrt{\text{a}^2-\text{x}^2}
=\text{R.H.S}
Q283. If \text{y}=\tan^{-1}\text{x}, find \frac{\text{d}^2\text{y}}{\text{dx}^2} in terms of y alone. 4 Marks

Ans: We have, \text{y}=\tan^{-1}\text{x} [ondifferentiating w.r.t. x]


\frac{\text{dy}}{\text{dx}}=\frac{1}{1+\text{x}^2} [again differentiating w.r.t. x]
Now, \frac{\text{d}^2\text{y}}{\text{dx}^2}=\frac{\text{d}}{\text{dx}}(1+\text{x}^2)^{-1}

https://bls.smartstudies.co.in/#/exam/pdf-preview/c59cb220-8e86-4716-9ff7-82aec16b1ade/1 67/158
5/26/24, 6:19 PM Exam Automation
=-1(1+\text{x}^2)^{-2}\cdot\frac{\text{d}}{\text{dx}}(1+\text{x}^2)
=-\frac{1}{(1+\text{x}^2)^2}\cdot2\text{x}
=\frac{-2\tan\text{y}}{(1+\tan^2\text{y})^2} [\because\text{y}=\tan^{-1}\text{x}\Rightarrow\tan\text{y}=\text{x}]
=\frac{-2\tan\text{y}}{(\sec^2\text{y})^2}
=-2\frac{\sin\text{y}}{\cos\text{y}}\cdot\cos^2\text{y}\cdot\cos^2\text{y}
=-\sin2\text{y}\cdot\cos^2\text{y} [\because\sin2\text{x}=2\sin\text{x}\cos\text{x}]
Q284. Differentiate tan-1 \Bigg[\frac{\sqrt{\text{1+x}^{2}-1}}{\text{x}}\Bigg] with respect to x. 4 Marks

Ans: Let x = tan θ


\therefore Given expression becomes y = tan-1 \Bigg(\frac{\sec\theta-1}{\tan\theta}\Bigg)=\tan^{-1}\Bigg(\frac{1-\cos\theta}{\sin\theta}\Bigg)
\therefore\text{y = tan}^{-1}\Bigg(\tan\frac{\theta}{2}\Bigg)=\frac{1}{2}\theta=\frac{1}{2}\tan^{-1}\text{x}
\therefore\frac{\text{dy}}{\text{dx}}=\frac{1}{\text{2(1+x}^{2})}.
Q285. \text{If y = 3} \cos (\log\text{x}) + 4\sin (\log \text{x}), \text{then show that x}^{2} .\frac{\text{d}^{2}{\text{y}}}{\text{dx}^{2}} + \text{y} = 0 4 Marks

Ans: \text{y} = 3\cos(\log\text{x}) + 4\sin (\log\text{x}) \Rightarrow \frac{\text{dy}}{\text{dx}} = - \frac{3\sin(\log\text{x})}{\text{x}} +\frac{4\cos(\log\text{x})}{\text{x}}


\text{x} \frac{\text{dy}}{\text{dx}} = -3\sin (\log\text{x}) + 4\cos(\log\text{x})
\Rightarrow \text{x} \frac{\text{d}^{2}\text{y}}{\text{dx}^{2}} = - \frac{3\cos(\log\text{x})}{\text{x}} -\frac{4\sin(\log\text{x})}{\text{x}}
\Rightarrow \text{x} \frac{\text{d}^{2}\text{y}}{\text{dx}^{2}} + \text{x} \frac{\text{dy}}{\text{dx}} = -[3\cos (\log\text{x}) + 4\sin (\log{\text{x}})] = \text{-y}
\text{or x}^{2} \frac{\text{d}^{2}\text{y}}{\text{dx}^{2}} +\text{y = o}
Q286. Differentiate the following functions with respect to x: 4 Marks
\frac{\text{x}^2+2}{\sqrt{\cos\text{x}}}

Ans: Let \text{y}=\frac{\text{x}^2+2}{\sqrt{\cos\text{x}}}


Differentiate it with respect to x we get,
\frac{\text{dy}}{\text{dx}}=\frac{\sqrt{\cos\text{x}}\frac{\text{d}}{\text{dx}}(\text{x}^2+2)-(\text{x}^2+2)\frac{\text{d}}{\text{dx}}(\sqrt{\cos\text{x}})}{(\sqrt{\cos\text{x}})^2}
[Using quotient rule and chain rule]
=\frac{2\text{x}\sqrt{\cos\text{x}}-(\text{x}^2+2)\Big(-\frac{1}{2}\frac{\sin\text{x}}{\sqrt{\cos\text{x}}}\Big)}{\cos\text{x}}
=\frac{2\text{x}\sqrt{\cos\text{x}}+\frac{(\text{x}^2+2)\sin\text{x}}{2\sqrt{\cos\text{x}}}}{\cos\text{x}}
=\frac{4\text{x}\cos\text{x}+(\text{x}^2+2)\sin\text{x}}{2(\cos\text{x})^\frac{3}{2}}
=\frac{2\text{x}}{\sqrt{\cos\text{x}}}+\frac{1}{2}\frac{(\text{x}^2+2)\sin\text{x}}{(\cos\text{x})^\frac{3}{2}}
=\frac{1}{\sqrt{\cos\text{x}}}\Big\{2\text{x}+\frac{1}{2}\frac{(\text{x}^2+2)\sin\text{x}}{\cos\text{x}}\Big\}
=\frac{1}{\sqrt{\cos\text{x}}}\Big\{2\text{x}+\frac{(\text{x}^2+2)\tan\text{x}}{2}\Big\}
So,
\frac{\text{d}}{\text{dx}}\Big(\frac{\text{x}^2+2}{\sqrt{\cos\text{x}}}\Big)=\frac{1}{\sqrt{\cos\text{x}}}\Big\{2\text{x}+\frac{(\text{x}^2+2)\sin\text{x}}{2}\Big\}
Q287. If \text{y}=(\cot^{-1}\text{x})^2, show that 4 Marks
(\text{x}^2+1)^2\frac{\text{d}^\text{2}\text{y}}{\text{dx}^2}+2\text{x}(\text{x}^2+1)\frac{\text{dy}}{\text{dx}}=2.

Ans: \text{y}=(\cot^{-1}\text{x})^2\Rightarrow\frac{\text{dy}}{\text{dx}}+2\cot^{-1}\text{x}.\Big(\frac{-1}{1+\text{x}^2}\Big)
\Rightarrow(1+\text{x}^2)\frac{\text{dy}}{\text{dx}}=-2\cot^{-1}\text{x}=-2\sqrt{\text{y}}
Squaring both sides,
we get (1+\text{x}^2)^2.\Big(\frac{\text{dy}}{\text{dx}}\Big)^2=4\text{y}
differentiating, w.r.t. x,
Given,
(1+\text{x}^2)^2\Big(\frac{\text{dy}}{\text{dx}}\Big)^2=4\text{y}
On defferentiation,
2(1+​​\text{x}^2)^2\frac{\text{d}^2\text{y}}{\text{dx}^2}+2(1+\text{x}^2).2\text{x}\Big(\frac{\text{dy}}{\text{dx}}\Big)^2=4\Big(\frac{\text{dy}}{\text{dx}}\Big)
2\Big[(1+\text{x}^2)^2\frac{\text{d}^2\text{y}}{\text{dx}^2}+2\text{x}(1+\text{x}^2)\frac{\text{dy}}{\text{dx}}\Big]=4
\Rightarrow(1+\text{x}^2)\frac{\text{d}^2\text{y}}{\text{dx}^2}+2\text{x}(1+\text{x}^2)\frac{\text{dy}}{\text{dx}}=2
Q288. If \text{y}=\frac{\text{x}\sin^{-1}\text{x}}{\sqrt{1-\text{x}^2}}, prove that (1-\text{x}^2)\frac{\text{dy}}{\text{dx}}=\text{x}+\frac{\text{y}}{\text{x}} 4 Marks

Ans: Givne, \text{y}=\frac{\text{x}\sin^{-1}\text{x}}{\sqrt{1-\text{x}^2}}


Differentiate with respect to x,
\frac{\text{d}}{\text{dx}}=\frac{\text{d}}{\text{dx}}\Big(\frac{\text{x}\sin^{-1}\text{x}}{\sqrt{1-\text{x}^2}}\Big)
=\bigg[\frac{\sqrt{1-\text{x}^2}\frac{\text{d}}{\text{dx}}(\text{x}\sin^{-1}\text{x})-(\text{x}\sin^{-1}\text{x})\frac{\text{d}}{\text{dx}}(\sqrt{1-\text{x}^2})}{(\sqrt{1-\text{x}^2})^2}\bigg]
[Using quotient rule, product rule, chain rule]
=\begin{bmatrix}\frac{\sqrt{1-\text{x}^2}\Big\{\text{x}\frac{\text{d}}{\text{dx}}\sin^{-1}\text{x}+\sin^{-1}\text{x}\frac{\text{d}}{\text{dx}}(\text{x})\Big\}-
\big(\text{x}\sin^{-1}\text{x}\big)\frac{1}{2\sqrt{1-\text{x}^2}}\frac{\text{d}}{\text{dx}}\big(1-\text{x}^2\big)}{\big(1-\text{x}^2\big)} \end{bmatrix}
=\begin{bmatrix}\frac{\sqrt{1-\text{x}^2}\Big\{\frac{\text{x}}{\sqrt{1-\text{x}^2}}+\sin^{-1}\text{x}\Big\}-\frac{\text{x}\sin{-1}\text{x}(-2\text{x})}{2\sqrt{1-\text{x}^2}}}{\big(1-
\text{x}^2\big)} \end{bmatrix}
=\begin{bmatrix}\frac{\text{x}+\sqrt{1-\text{x}^2}\sin^{-1}\text{x}+\frac{\text{x}^2\sin^{-1}\text{x}}{\sqrt{1-\text{x}^2}}}{\big(1-\text{x}^2\big)} \end{bmatrix}
\Rightarrow\big(1-\text{x}^2\big)\frac{\text{dy}}{\text{dx}}=\text{x}+\frac{\sqrt{1-\text{x}^2}\sin^{-1}}{1}+\frac{\text{x}^2\sin^{-1}\text{x}}{\sqrt{1-\text{x}^2}}
\Rightarrow\big(1-\text{x}^2\big)\frac{\text{dy}}{\text{dx}}=\text{x}+\bigg(\frac{(1-\text{x}^2)\sin^{-1}\text{x}+\text{x}^2\sin^{-1}\text{x}}{\sqrt{1-\text{x}^2}}\bigg)
\Rightarrow\big(1-\text{x}^2\big)\frac{\text{dy}}{\text{dx}}=\text{x}+\bigg(\frac{\sin^{-1}\text{x}-\text{x}^2\sin^{-1}\text{x}+\text{x}^2\sin^{-1}\text{x}}{\sqrt{1-\text{x}^2}}\bigg)
\Rightarrow\big(1-\text{x}^2\big)\frac{\text{dy}}{\text{dx}}=\text{x}+\bigg(\frac{\sin^{-1}\text{x}}{\sqrt{1-\text{x}^2}}\bigg)
\Rightarrow(1-\text{x}^2)\frac{\text{dy}}{\text{dx}}=\text{x}+\frac{\text{y}}{\text{x}}\ \Big\{\text{Since, given y}=\frac{\text{x}\sin^{-1}\text{x}}{\sqrt{1-\text{x}^2}}\Big\}
Q289. If \text{x}=\cos\theta,\text{y}=\sin^3 prove that \text{y}\frac{\text{d}^2\text{y}}{\text{dx}^2}+\Big(\frac{\text{dy}}{\text{dx}^2}\Big)=3\sin^2\theta(5\cos^2\theta-1) 4 Marks

Ans: Here
\text{x}=\cos\theta,\text{y}=\sin^3
Differentiating w.r.t.x, we get
\frac{\text{dx}}{\text{d}\theta}=-\sin\theta\ \text{and}\ \frac{\text{dy}}{\text{d}\theta}=3\sin^2\theta\cos\theta
\therefore\frac{\text{dy}}{\text{dx}}=\frac{3\sin^2\theta\cos\theta}{-\sin\theta}=-3\sin\theta\cos\theta
Differentiating w.r.t.x, we get
\frac{\text{d}^2\text{y}}{\text{dx}^2}=(-3\cos^2\theta+3\sin^2\theta)\frac{\text{d}\theta}{\text{dx}}\frac{)-3\cos^2\theta+3\sin^2\theta)}{-\sin\theta}
Now,
\text{LHS}=\text{y}\frac{\text{d}^2\text{y}}{\text{dx}^2}+\Big(\frac{\text{dy}}{\text{dx}}\Big)^2
=\sin^3\theta\times\frac{(-3\cos^2\theta+3\sin^2\theta)}{\sin\theta}+(-3\sin\theta\cos\theta)^2
=3\sin^2\theta\cos^2\theta-3\sin^4\theta+9\sin^2\theta\cos^2\theta
=12\sin^2\theta\cos^2\theta-3\sin^4\theta
=3\sin^2\theta(4\cos^2\theta-\sin^2\theta)
=3\sin^2\theta(5\cos^2\theta-1)
=\text{RHS}
Q290. If \cos\text{y}=\text{x}\cos(\text{a}+\text{y}), with \cos\text{a}\neq\pm1, prove that \frac{\text{dy}}{\text{dx}}=\frac{\cos^2(\text{a}+\text{y})}{\sin\text{a}} 4 Marks

Ans: We have, \cos\text{y}=\text{x}\cos(\text{a}+\text{y})


Differentiating with respect to x, we get,
\frac{\text{d}}{\text{dx}}(\cos\text{y})=\frac{\text{d}}{\text{dx}}\big\{\text{x}\cos(\text{a}+\text{y})\big\}
\Rightarrow -\sin\text{y}\frac{\text{dy}}{\text{dx}}=\cos(\text{a}+\text{y})\frac{\text{d}}{\text{dx}}(\text{x})+\text{x}\frac{\text{d}}{\text{dx}}\cos(\text{a}+\text{y})
\Rightarrow -\sin\frac{\text{dy}}{\text{dx}}=\cos(\text{a}+\text{y})+\text{x}\big[-\sin(\text{a}+\text{y})\big]\frac{\text{dy}}{\text{dx}}

https://bls.smartstudies.co.in/#/exam/pdf-preview/c59cb220-8e86-4716-9ff7-82aec16b1ade/1 68/158
5/26/24, 6:19 PM Exam Automation
\Rightarrow\big[\text{x}\sin(\text{a}+\text{y})-\sin\text{y}\big]\frac{\text{dy}}{\text{dx}}=\cos(\text{a}+\text{y})
\Rightarrow\Big[\frac{\cos\text{y}}{\cos(\text{a}+\text{y})}\sin(\text{a}+\text{y})-\sin\text{y}\Big]\frac{\text{dy}}{\text{dx}}=\cos(\text{a}+\text{y}) \\ \Big[\because
\cos\text{y}=\text{x}\cos(\text{a}+\text{y})\Rightarrow\text{x}=\frac{\cos\text{y}}{\cos(\text{a}+\text{y})}\Big]
\Rightarrow\big[\cos\text{y}\sin(\text{a}+\text{y})-\sin\text{y}\cos(\text{a}+\text{y})\big]\frac{\text{dy}}{\text{dx}}=\cos^2(\text{a}+\text{y})
\Rightarrow \sin(\text{a}+\text{y}-\text{y})\frac{\text{dy}}{\text{dx}}=\cos^2(\text{a}+\text{y})
\Rightarrow \frac{\text{dy}}{\text{dx}}=\frac{\cos^2(\text{a}+\text{y})}{\sin\text{a}}
Q291. Verify the Rolle’s theorem for each of the functions: 4 Marks
\text{f(x)}=\sqrt{4-\text{x}^2}\text{ in }[-2,2].

Ans: We have, \text{f(x)}=\sqrt{4-\text{x}^2}=(4-\text{x}^2)^{\frac{1}{2}}


1. \text{f(x)}=\sqrt{4-\text{x}^2} is continuous function.
[since every polynomial function is a continuous function]
Hence, f(x) is continuous in [-2, 2]
2. \text{f}'(\text{x})=\frac{1}{2}(4-\text{x}^2)^{\frac{-1}{2}}\cdot(-2\text{x})
=-\text{x}\cdot\frac{1}{\sqrt{4-\text{x}^2}}, which exists everywhere except at \text{x}=\pm2.
Hence, f(x) is differentiable in (-2, 2).
3. \text{f}(-2)=\sqrt{(4-4)}=0 and \text{f}(2)=\sqrt{(4-4)}=0
\Rightarrow\ \text{f}(-2)=\text{f}(2)
Conditions of Rolle’s theorem are satisfied.
Hence, there exists a real number c such that f'(c) = 0
\Rightarrow\ -\text{c}\frac{1}{\sqrt{4-\text{c}^2}}=0
\Rightarrow\ \text{c}=0\in(-2,2)
Hence, Rolle’s theorem has been verified.
Q292. Differentiate \tan^{-1}\Big(\frac{\sqrt{1+\text{x}^2}-1}{\text{x}}\Big) w.r.t. \tan^{-1}\text{x} when \text{x}\neq0. 4 Marks

Ans: Let \text{u}=\tan^{-1}\frac{\sqrt{1+\text{x}^2}-1}{\text{x}} and \text{y}=\tan^{-1}\text{x}


Put \text{x}=\tan\theta
\Rightarrow\ \text{u}=\tan^{-1}\frac{\sqrt{1+\tan^2\theta}-1}{\tan\theta}
=\tan^{-1}\frac{\sec\theta-1}{\tan\theta}=\tan^{-1}\Big(\frac{1-\cos\theta}{\sin\theta}\Big)
=\tan^{-1}\bigg[\frac{2\sin^2\frac{\theta}{2}}{2\sin\frac{\theta}{2}\cos\frac{\theta}{2}}\bigg] =\tan^{-1}\Big[\tan\frac{\theta}{2}\Big]=\frac{\theta}{2}=\frac{1}{2}\tan^{-1}\text{x}
\therefore\ \frac{\text{du}}{\text{dx}}=\frac{1}{2}\cdot\frac{1}{1+\text{x}^2}\ \ \dots(\text{i})
and \frac{\text{dv}}{\text{dx}}=\frac{\text{d}}{\text{dx}}\tan^{-1}\text{x}=\frac{1}{1+\text{x}^2}\ \ \dots(\text{ii})
\therefore\ \frac{\text{du}}{\text{dv}}=\frac{\frac{\text{du}}{\text{dx}}}{\frac{\text{dv}}{\text{dx}}}=\frac{1}{2}
Q293. If x and y are connected parametrically by the equations given in Exercise without eliminating the parameter, Find \frac{\text{dy}}{\text{dx}}. 4 Marks
\text{If x}=\sqrt{\text{a}^{\sin^{-1}}\text{t}},\text{y}=\sqrt{\text{a}^{\cos^{-1}}\text{t}},\text{ Show that}\frac{\text{dy}}{\text{dx}}=-\frac{\text{y}}{\text{x}}

Ans: The given equations are \text{x}=\sqrt{\text{a}^{\sin{-1}}\text{t}}\text{ and y}=\sqrt{\text{a}^{\cos^{-1}}\text{t}}


\text{x}=\sqrt{\text{a}^{\sin^{-1}}\text{t}}\text{ and y}=\sqrt{\text{a}^{\cos^{-1}}\text{t}}
\Rightarrow\ \text{x}=\Big({\text{a}^{\sin^{-1}}\text{t}\Big)}^{\frac{1}{2}}\text{ and y}=\Big({\text{a}^{\cos^{-1}}\text{t}\Big)^{\frac{1}{2}}}
\Rightarrow\ \text{x}=\text{a}^{\frac{1}{2}\sin^{-1}\text{t}}\text{ and y}=\text{a}^{\frac{1}{2}\cos^{-1}\text{t}}
Consider \text{x}=\text{a}^{\frac{1}{2}\sin^{-1}\text{t}}
Taking logarithm on both the sides, we obtain
\log\text{x}=\frac{1}{2}\sin^{-1}\text{t}\log\text{a}
\therefore \frac{1}{\text{x}}.\frac{\text{dx}}{\text{dt}}=\frac{1}{2}\log\text{a}.\frac{\text{d}}{\text{dt}}(\sin^{-1}\text{t})
\Rightarrow\ \frac{\text{dx}}{\text{dt}}=\frac{\text{x}}{2}\log\text{a}.\frac{1}{\sqrt{1-\text{t}^2}}
\Rightarrow\ \frac{\text{dx}}{\text{dt}}=\frac{\text{x}\log\text{a}}{2\sqrt{1-\text{t}^2}}
Then, consider \text{a}^{\frac{1}{2}\cos^{-1}\text{t}}
Taking logarithm on both the sides, we obtain
\log\text{y}=\frac{1}{2}\cos^{-1}\text{t}\log\text{a}
\therefore \frac{1}{\text{y}}.\frac{\text{dy}}{\text{dx}}=\frac{1}{2}\log\text{a}.\frac{\text{d}}{\text{dt}}(\cos^{-1}\text{t})
\Rightarrow\ \frac{\text{dy}}{\text{dt}}=\frac{-\text{y}\log\text{a}}{2}.\Big(\frac{1}{\sqrt{1-\text{t}^2}}\Big)
\Rightarrow\ \frac{\text{dy}}{\text{dt}}=\frac{-\text{y}\log\text{a}}{2\sqrt{1-\text{t}^2}}
\therefore\ \frac{\text{dy}}{\text{dx}}=\frac{\Big(\frac{\text{dy}}{\text{dt}}\Big)}{\Big(\frac{\text{dx}}{\text{dx}}\Big)}=\frac{\Big(\frac{-\text{y}\log\text{a}}{2\sqrt{1-\text{t}^2}}\Big)}
{\Big(\frac{\text{x}\log\text{a}}{2\sqrt{1-\text{t}^2}}\Big)}=-\frac{\text{y}}{\text{x}}.
Hence, proved.
Q294. Differentiate the following with respect to x: \tan^{-1} \bigg(\frac{\sqrt{1 + x} - \sqrt{1 - x}}{\sqrt{1 + x + \sqrt{ 1 - x}}}\bigg) 4 Marks

Ans: \text{Let x} = \cos 2 \theta , \sqrt{1 + x} = \sqrt{2} \cos\theta, \sqrt{1 - x} = \sqrt{2} \sin\theta
\text{Let y} = \bigg[\frac{\sqrt{1 + x} - \sqrt{1 - x}}{\sqrt{1 + x + \sqrt{ 1 - x}}}\bigg] = \tan^{-1} \bigg[\frac{1 - \tan \theta}{1 + \tan \theta}\bigg] = \tan^{-1} \tan \bigg(\frac{\pi}{4} -
\theta\bigg)
= \frac{\pi}{4} - \theta = \frac{\pi}{4} - \frac{1}{2} \cos^{-1} \text{x}
\therefore \frac{dy}{dx} = \frac{-1}{2} \bigg(\frac{-1}{\sqrt{1 - x^{2}}}\bigg) = \frac{1}{2\sqrt{1 - x^{2}}}
Q295. If \text{y}=\sqrt{\text{x}}+\frac{1}{\sqrt{\text{x}}}, prove that 2\text{x}\frac{\text{dy}}{\text{dx}}=\sqrt{\text{x}}-\frac{1}{\sqrt{\text{x}}} 4 Marks

Ans: \text{y}=\sqrt{\text{x}}+\frac{1}{\sqrt{\text{x}}}
Differentiate with respect to x,
\frac{\text{dy}}{\text{dx}}=\frac{\text{d}}{\text{dx}}\Big(\sqrt{\text{x}}+\frac{1}{\sqrt{\text{x}}}\Big)
=\frac{\text{d}}{\text{dx}}\big(\sqrt{\text{x}}\big)+\frac{\text{d}}{\text{dx}}\Big(\text{x}^{-1\frac{1}{2}}\Big)
=\frac{1}{2\sqrt{\text{x}}}+\Big(-\frac{1}{2}\times\text{x}^{-\frac{1}{2}-1}\Big)
=\frac{2}{2\sqrt{\text{x}}}-\frac{1}{2\sqrt[\text{x}]{\text{x}}}
\frac{\text{dy}}{\text{dx}}=\frac{\text{x}-1}{2\text{x}\sqrt{\text{x}}}
\Rightarrow 2\text{x}\frac{\text{dy}}{\text{dx}}=\frac{\text{x}}{\sqrt{\text{x}}}-\frac{1}{\sqrt{\text{x}}}
Hence, the solution is, 2\text{x}\frac{\text{dy}}{\text{dx}}=\frac{\text{x}}{\sqrt{\text{x}}}-\frac{1}{\sqrt{\text{x}}}
Q296. Find the value of k if f(x) is continuous at \text{x}=\frac{\pi}{2}, where 4 Marks
\text{f}\text{(x)}=\begin{cases}\frac{\text{k}\cos\text{x}}{\pi-2\text{x}}, &\text{ x}\neq\frac{\pi}{2}\\3, &\text{ x}=\frac{\pi}{2}\end{cases}

Ans: Since f(x) is continuous at \text{x}=\frac{\pi}{2}, L.H.Limit = R.H.Limit.


\Rightarrow\lim\limits_{\text{x} \rightarrow \frac{\pi^-}{2}}\text{f}\text{(x)}=\lim\limits_{\text{x} \rightarrow \frac{\pi^+}{2}}\text{f}\text{(x)}=\lim\limits_{\text{x} \rightarrow \frac{\pi}
{2}}\text{f}\text{(x)}=\text{f}\Big(\frac{\pi}{2}\Big)
\Rightarrow\lim\limits_{\text{x} \rightarrow \frac{\pi^-}{2}}\frac{\text{k}\cos\text{x}}{\pi-2\text{x}}=3
\Rightarrow\text{k}\lim\limits_{\text{x} \rightarrow \frac{\pi}{2}}\frac{\sin\Big(\frac{\pi}{2}-\text{x}\Big)}{2\Big(\frac{\pi}{2}-\text{x}\Big)}=3
\Rightarrow\frac{\text{k}}{2}\lim\limits_{\text{x} \rightarrow \frac{\pi}{2}}\frac{\sin\Big(\frac{\pi}{2}-\text{x}\Big)}{2\Big(\frac{\pi}{2}-\text{x}\Big)}=3
\Rightarrow\frac{\text{k}}{2}=3
\Rightarrow\text{k}=6
Q297. \text{If y}=3\cos(\log \text{x})+4\sin(\log\text{x}),\text{ show that }\text{x}^2\text{y}_2+\text{xy}_1+\text{y}=0 4 Marks

Ans: Given: \text{y}=3\cos(\log\text{x})+4\sin(\log\text{x})\ \dots\text{(i)}


\therefore\ \frac{\text{dy}}{\text{dx}}=\text{y}_1=-3\sin(\log\text{x})\frac{\text{d}}{\text{dx}}\log\text{x}+4\cos(\log\text{x})\frac{\text{d}}{\text{dx}}\log\text{x}
\Rightarrow\ \text{y}_1=-3\sin(\log\text{x})\frac{1}{\text{x}}+4\cos(\log\text{x})\frac{1}{\text{x}} =\frac{1}{\text{x}}[-3\sin(\log\text{x})+4\cos(\log\text{x})]

https://bls.smartstudies.co.in/#/exam/pdf-preview/c59cb220-8e86-4716-9ff7-82aec16b1ade/1 69/158
5/26/24, 6:19 PM Exam Automation
\Rightarrow\ \text{xy}_1=-3\sin(\log\text{x})+4\cos(\log\text{x})
Now \frac{\text{d}}{\text{dx}}(\text{xy}_1)=-3\cos(\log\text{x})\frac{\text{d}}{\text{dx}}\log\text{x}-4\sin(\log\text{x})\frac{\text{d}}{\text{dx}}\log\text{x}
\Rightarrow\ \text{x}\frac{\text{d}}{\text{dx}}(\text{y}_1)+\text{y}_1\frac{\text{d}}{\text{dx}}\text{x} =-3\cos(\log\text{x})\frac{1}{\text{x}}-4\sin(\log\text{x})\frac{1}{\text{x}}
\Rightarrow\ \text{xy}_2+\text{y}_1=-\frac{[3\cos(\log\text{x})+4\sin(\log\text{x})]}{\text{x}}
\Rightarrow\ \text{x}(\text{xy}_2+\text{y}_1)=-[3\cos(\log\text{x})+4\sin(\log\text{x})]
\Rightarrow\ \text{x}(\text{xy}_2+\text{y}_1)=-\text{y}\ \ [\text{From eq.(i)}]
\Rightarrow\ \text{x}^2\text{y}_2+\text{xy}_1+\text{y}=0 \ \text{ Hence proved}.
Q298. Differentiate sin (2x + 3) w.r.t. x from first principle. 4 Marks

Ans: y = \sin(2x + 3)
y = \triangle y = \sin (2x +2\triangle x + 3)
\therefore \triangle y \sin (2x + 2\triangle x + 3) -\sin (2x + 3)
= 2\cos (2x + 3 + \triangle x) \sin \triangle x
\therefore \lim\limits_{\triangle x\rightarrow 0} \frac{\triangle y}{\triangle x} = \lim\limits_{\triangle x \rightarrow 0} 2\cos (2x +3 +\triangle x) \lim\limits_{\triangle x\rightarrow 0} \frac
{\sin\triangle x}{\triangle x}
OR
\frac{dy}{dx} = \text2\cos (2x + 3) 1 = 2\cos (2x + 3)
Q299. Find the value of 'a' for which the function f defined as 4 Marks
\begin{matrix} & \text{a sin}\frac{\pi}{2}\text{(x + 1)}, & x\leq0 \\ \text{f(x)} \\ & \frac{\text{tan x - sin x}}{\text{x}^{3}}, & x<0 \\ \end{matrix}
is continuous at X = 0.

Ans: L.H.L = \DeclareMathOperator*{\median}{\text{lim}} \median_{\text{x}\rightarrow0^{-}} f(x) = a


f(0) = a.sin\pi/\text{r} = a
R.H.L. = \DeclareMathOperator*{\median}{\text{lim}} \median_{\text{x}\rightarrow0^{-}} \frac{\text{tan x}}{\text{x}}\cdot\frac{\text{(1 - cos x)}}{\text{x}^{2}}
= \DeclareMathOperator*{\median}{\text{lim}} \median_{\text{x}\rightarrow0^{-}}\frac{\tan \text{x}}{\text{x}}\cdot2\Bigg(\frac{\sin\text{x/2}}{2\text{x/2}}\Bigg)^{2}=\frac{1}{2}
\therefore\text{a}=\frac{1}{2}
Q300. Verify Rolle's theorem for the following function on the indicated intervals 4 Marks
\text{f}(\text{x})=\cos2\Big(\text{x}-\frac{\pi}{4}\Big)\text{ on }\Big[0,\frac{\pi}{2}\Big]

Ans: The given function is \text{f}(\text{x})=\cos2\Big(\text{x}-\frac{\pi}{4}\Big)


=\cos\Big(2\text{x}-\frac{\pi}{2}\Big)=\sin2\text{x}.
Thus, we have to show that there exists \text{c}\in\Big(0,\frac{\pi}{2}\Big) such that f'(c) = 0.
We have
\text{f}(\text{x})=\sin2\text{x}
\Rightarrow\text{f}'(\text{x})=2\cos2\text{x}
\Rightarrow\text{f}'(\text{x})=0
\Rightarrow2\cos2\text{x}=0
\Rightarrow\cos2\text{x}=0
\Rightarrow\text{x}=\frac{\pi}{4}
Thus, \text{c}=\frac{\pi}{4}\in\Big(0,\frac{\pi}{2}\Big) such that \text{f}'(\text{c})=0.
Hence, Rolle's theorem is verified .
Q301. \text{if y } = \sqrt{\frac{(x - 3)(x^{2} + 4)}{3x^{2} + 4x + 5}}, \text{find} \frac{dy}{dx} 4 Marks

Ans: y = \sqrt{\frac{(x - 3)(x^{2} + 4)}{3x^{2} + 4x + 5}} \dots\dots\dots \text{(i})


Talking log of both sides of (i), we get
\text{\log y} = \frac{1}{2} \bigg[\log (x - 3) + \log (x^{2} + 4) - \log (3x^{2} + 4x +5)\bigg]
\therefore \frac{1}{y} \frac{dy}{dx} = \frac{1}{2}\bigg[ \frac{1}{x-3} + \frac{2x}{x^{2} + 4} - \frac{6x + 4}{3x^{2} + 4x + 5}\bigg]
\therefore \frac{dy}{dx} = \frac{y}{2}\bigg[ \frac{1}{x-3} + \frac{2x}{x^{2} + 4} - \frac{6x + 4}{3x^{2} + 4x + 5}\bigg]
OR
\frac{1}{2} \sqrt{\frac{x - 3)(x ^{2} + 4)}{3x^{2} + 4x +5}} \bigg[ \frac{1}{x - 3} + \frac{2x}{x^{2} + 4} - \frac{6x + 4}{3x^{2} + 4x + 5}\bigg]
Q302. If \text{e}^{\text{x}}+\text{e}^{\text{y}}=\text{e}^{\text{x}+\text{y}}, prove that \frac{\text{dy}}{\text{dx}}=-\frac{\text{e}^{\text{x}}(\text{e}^\text{y}-1)}{\text{e}^{\text{y}} 4 Marks
(\text{e}^{\text{x}}-1)} or \frac{\text{dy}}{\text{dx}}+\text{e}^{\text{y}-\text{x}}=0

Ans: \text{e}^\text{x}+\text{e}^\text{y}=\text{e}^{\text{x}+\text{y}}
\Rightarrow\text{e}^\text{x}+\text{e}^\text{y}\frac{\text{dy}}{\text{dx}}=\text{e}^{\text{x}+\text{y}}\Big(1+\frac{\text{dy}}{\text{dx}}\Big)
\Rightarrow\text{e}^\text{x}+\text{e}^{\text{y}}\frac{\text{dy}}{\text{dx}}=\text{e}^{\text{x}+\text{y}}+\text{e}^{\text{x}+\text{y}}\frac{\text{dy}}{\text{dx}}
\Rightarrow\text{e}^\text{y}\frac{\text{dy}}{\text{dx}}-\text{e}^{\text{x}+\text{y}}\frac{\text{dy}}{\text{dx}}=\text{e}^{\text{x}+\text{y}}-\text{e}^{\text{x}}
\Rightarrow\frac{\text{dx}}{\text{dy}}(\text{e}^\text{y}-\text{e}^{\text{x}+\text{y}})=\text{e}^{\text{x}+\text{y}}-\text{e}^{\text{x}}
\Rightarrow\frac{\text{dy}}{\text{dx}}=\frac{\text{e}^{\text{x}+\text{y}}-\text{e}^{\text{x}}}{\text{x}^\text{y}-\text{e}^{\text{x}+\text{y}}}
=\frac{\text{e}^\text{x}(\text{e}^\text{y}-1)}{\text{e}^\text{y}({1-\text{e}}^\text{x})}
=-\frac{\text{e}^\text{x}(\text{e}^\text{y}-1)}{\text{e}^\text{y}(\text{e}^\text{x}-1)}
Q303. Differentiate xx cos x + \frac{\text{x}^{2}+1}{\text{x}^{2}-1} w.r.t.x. 4 Marks

Ans: ​\text{y}=​\text{x}^{\text{x cos x}}+​\frac{\text{x}^{2}+1}{\text{x}^{2}-1}=\text{u + v}


\text{u}=\text{x}^{\text{x cos x}}\Rightarrow\log\text{u}=\text{x cos x}\cdot{\text{log x}}
\therefore\frac{1}{\text{u}}\cdot\frac{\text{du}}{\text{dx}}=\frac{\text{x cos x}}{\text{x}}+\text{cos x}\cdot{\text{log x - sin x log x}}
\Rightarrow\frac{\text{du}}{\text{dx}}=\text{x}^{\text{x cos x}}(\text{cos x+cos x log x}\text{ - sin x log x})
\text{v}=\frac{\text{x}^{2}+1}{\text{x}^{2}-1},\frac{\text{dv}}{\text{dx}}=\frac{(\text{x}^{2}-1)\text{2x}-\text{(x}^{2}+1)\text{2x}}{\text{(x}^{2}-1)^{2}}=-\frac{\text{4x}}{\text{(x}^{2}-1)^{2}}
\therefore\frac{\text{dy}}{\text{dx}}=\text{x}^{\text{x cos x}}\text{(cos x + log x}\cdot\text{cos x - sin x log x})-\frac{\text{4x}}{(\text{x}^{2}-1)^{2}}
Q304. If \text{y}=(\cot^{-1}\text{x})^2 prove that \text{y}^2(\text{x}^2+1)^2+2\text{x}(\text{x}^2+1)\text{y}_1=2. 4 Marks

Ans: \text{y}=(\cot^{-1}\text{x})^2
Differentiating w.r.t.x,
\Rightarrow\frac{\text{dy}}{\text{dx}}=\text{y}_1=\frac{-2\cot^{-1}\text{x}}{1+\text{x}^2}
=\frac{-2\cot^{-1}\text{x}}{1+\text{x}^2}\ (\text{chain rule})
\Rightarrow(1+\text{x}^2)\frac{\text{dy}}{\text{dx}}=-2\cot^{-1}\text{x}
Differentiating w.r.t.x,
\Rightarrow(1+\text{x}^2)\text{y}^2+2\text{xy}_1=+2\Big(\frac{+1}{1+\text{x}^2}\Big)
(Multiplication rule on LHS)
\Rightarrow(1+\text{x}^2)^2\text{y}_2+2\text{x}(1+\text{x}^2)\text{y}_1=2
Hence proved
Q305. Discuss the applicability of Rolle’s theorem on the function given by. 4 Marks
\text{f(x)}=\begin{cases}\text{x}^2+1,&\text{if }0\leq\text{x}\leq1\\3-\text{x},&\text{if }1\leq\text{x}\leq2\end{cases}

Ans: Consider, \text{f(x)}=\begin{cases}\text{x}^2+1,&\text{if }0\leq\text{x}\leq1\\3-\text{x},&\text{if }1\leq\text{x}\leq2\end{cases}


We know that, polynomial function is everywhere continuous and differentiability.
So, f(x) is continuous and differentiable at all points except possibly at x = 1.
Now, check the differentiability at x = 1,
At x = 1 \text{L.D.H}=\lim\limits_{\text{x}\rightarrow1^-}\frac{\text{f(x)}-\text{f}(1)}{\text{x}-1}
=\lim\limits_{\text{x}\rightarrow1}\frac{(\text{x}^2+1)-(1+1)}{\text{x}-1} [\because\ \text{f(x)}=\text{x}^2+1,\forall\ 0\leq\text{x}\leq1]

https://bls.smartstudies.co.in/#/exam/pdf-preview/c59cb220-8e86-4716-9ff7-82aec16b1ade/1 70/158
5/26/24, 6:19 PM Exam Automation
=\lim\limits_{\text{x}\rightarrow1}\frac{\text{x}^2-1}{\text{x}-1}=\lim\limits_{\text{x}\rightarrow1}\frac{(\text{x}+1)(\text{x}-1)}{\text{x}-1}
and \text{R.D.H}=\lim\limits_{\text{x}\rightarrow1^+}\frac{\text{f(x)}-\text{f}(1)}{\text{x}-1}=\lim\limits_{\text{x}\rightarrow1}\frac{(3-\text{x})\text{f}(1+1)}{(\text{x}-1)}
=\lim\limits_{\text{x}\rightarrow1}\frac{3-\text{x}-2}{\text{x}-1}=\lim\limits_{\text{x}\rightarrow1}\frac{-(\text{x}-1)}{\text{x}-1}=-1
\therefore L.H.D ≠ R.H.D
So, f(x) is not differentiable at x = 1.
Hence, polle’s theorem is not applicable on the interval [0, 2]
Q306. Differentiate \tan^{-1}\Big(\frac{\cos\text{x}}{1+\sin\text{x}}\Big) with respect to \sec^{-1}\text{x} 4 Marks

Ans: Let, \text{u}=\tan^{-1}\Big(\frac{\cos\text{x}}{1+\sin\text{x}}\Big)


\Rightarrow\text{u}=\tan^{-1}\Big[\tan\Big(\frac{\pi}{4}-\frac{\text{x}}{2}\Big)\Big]
\Rightarrow\text{u}=\frac{\pi}{4}-\frac{\text{x}}{2}
Differentiating it with respect to x,
\frac{\text{du}}{\text{dx}}=0-\big(\frac{1}{2}\big)
\frac{\text{du}}{\text{dx}}=-\frac{1}{2}\ .....(\text{i})
Let, \text{v}=\sec^{-1}\text{x}
Differentiating it with respect to x,
\frac{\text{dv}}{\text{dx}}=\frac{1}{\text{x}\sqrt{\text{x}^2-1}}\ .....\text{(ii)}
Differentiating equation (i) by (ii),
\frac{\frac{\text{du}}{\text{dx}}}{\frac{\text{dv}}{\text{dx}}}=-\frac{1}{2}\times\frac{\text{x}\sqrt{\text{x}^2-1}}{1}
\frac{\text{du}}{\text{dv}}=\frac{-\text{x}\sqrt{\text{x}^2-1}}{2}
Q307. If \text{x}=\text{a}(\theta+\sin\theta)\ \text{and}\ \text{y}=\text{a}(1+\cos\theta) prove that \frac{\text{d}^2\text{y}}{\text{dx}^2}=-\frac{\text{a}}{\text{y}^2}. 4 Marks

Ans: Here
\text{x}=\text{a}(\theta+\sin\theta)\ \text{and}\ \text{y}=\text{a}(1+\cos\theta)
Differentiating w.r.t.\theta, we get
\frac{\text{dx}}{\text{d}\theta}=\text{a}+\text{a}\cos\theta\ \text{and}\ \frac{\text{dy}}{\text{d}\theta}=-\text{a}\sin\theta
\therefore\frac{\text{dy}}{\text{dx}}=\frac{-\text{a}\sin\theta}{\text{a}+\text{a}\cos\theta}=\frac{-\sin\theta}{1+\cos\theta}
Differentiating w.r.t.\theta, we get
\frac{\text{d}^2\text{y}}{\text{dx}^2}=-\Big\{\frac{(1+\cos\theta)\cos\theta+\sin^2\theta}{(1+\cos\theta)^2}\Big\}\frac{\text{d}\theta}{\text{dx}}
=\frac{-\cos\theta-\cos^2\theta-\sin^2\theta}{(1+\cos\theta)^2}\times\frac{1}{\text{a}+\text{a}\cos\theta}
=\frac{-(1+\cos\theta)}{\text{a}(1+\cos\theta)^3}
=\frac{-1}{\text{a}(1+\cos\theta)^2}
=\frac{-\text{a}}{\text{y}^2}\ [\because\text{y}=\text{a}(1+\cos\theta)]
Hence proved
Q308. Discuss the continuity of the following functions at the indicated point: 4 Marks
\text{f}\text{(x)}=\begin{cases}\frac{{1}-\text{x}^\text{n}}{1-\text{x}}, & \text{x} \neq1\\\text{n}-1, & \text{ x} = 1\end{cases}\text{ n }\in\ \text{N at x}=1

Ans: We want, to check the continuity at x = 1


\text{LHL}=\lim\limits_{\text{x} \rightarrow 1^-}\text{(x)}=\lim\limits_{\text{h} \rightarrow 0} \text{f}(1-\text{h)}
=\lim\limits_{\text{h} \rightarrow 0}\frac{1-(1-\text{h})^\text{n}}{1-(1-\text{h})}=\lim\limits_{\text{h} \rightarrow 0}\frac{1-\Big[1-\text{nh}+\frac{\text{n}(\text{n}-1)}
{2}\text{h}^2+\dots\Big]}{\text{h}}
=\lim\limits_{\text{h} \rightarrow 0}\text{n}-\frac{\text{n(n-1)}}{2!}\text{h}+\dots
=\text{n}
\text{RHL}=\lim\limits_{\text{x} \rightarrow 1^+}\text{(x)}=\lim\limits_{\text{h} \rightarrow 0} \text{f}\text{(1+h)}
=\lim\limits_{\text{h} \rightarrow 0}\frac{1-(1+\text{h})^\text{n}}{1-(1+\text{h})}=\lim\limits_{\text{h} \rightarrow 0}\frac{1-\Big[1-\text{nh}+\frac{\text{n}(\text{n}-1)}
{2}\text{h}^2+\dots\Big]}{\text{h}}
=\lim\limits_{\text{h} \rightarrow 0}\text{n}+\frac{\text{n}(\text{n}-1)}{2!}\text{h}+\dots
=\text{n}
\text{f}(1)=\text{n}-1
Thus, \text{LHL}=\text{RHL}\neq\text{f}( 1)
Hence, funtion is discontinuous at x = 1
This is removable discotinuity.
Q309. Differentiate the following w.r.t. x: 4 Marks
\tan^{-1}\Big(\frac{\text{a}\cos\text{x}-\text{b}\sin\text{x}}{\text{b}\cos\text{x}+\text{a}\sin\text{x}}\Big),-\frac{\pi}{2}<\text{x}<\frac{\pi}{2}\text{ and }\frac{\text{a}}
{\text{b}}\tan\text{x}>-1

Ans: Let \text{y}=\tan^{-1}\Big(\frac{\text{a}\cos\text{x}-\text{b}\sin\text{x}}{\text{b}\cos\text{x}+\text{a}\sin\text{x}}\Big)


=\tan^{-1}\Bigg[\frac{\frac{\text{a}\cos\text{x}}{\text{b}\cos\text{x}}-\frac{\text{b}\sin\text{x}}{\text{b}\cos\text{x}}}{\frac{\text{b}\cos\text{x}}
{\text{b}\cos\text{x}}+\frac{\text{a}\sin\text{x}}{\text{b}\cos\text{x}}}\Bigg]
=\tan^{-1}\Bigg[\frac{\frac{\text{a}}{\text{b}}-\tan\text{x}}{1+\frac{\text{a}}{\text{b}}\tan\text{x}}\Bigg]
=\tan^{-1}\frac{\text{a}}{\text{b}}-\tan^{-1}\tan\text{x} \bigg[\because\tan^{-1}\text{y}=\tan^{-1}\Big(\frac{\text{x}-\text{y}}{1+\text{xy}}\Big)\bigg]
\therefore\ \text{y}=\tan^{-1}\frac{\text{a}}{\text{b}}-\text{x}
\Rightarrow\ \frac{\text{dy}}{\text{dx}}=\frac{\text{d}}{\text{dx}}\Big(\tan^{-1}\frac{\text{a}}{\text{b}}\Big)-\frac{\text{d}}{\text{dx}}(\text{x})
=0-1\bigg[\because\ \frac{\text{d}}{\text{dx}}\Big(\frac{\text{a}}{\text{b}}\Big)=0\bigg]
=-1
Q310. Differentiate \sin^{-1}\sqrt{1-\text{x}^2} with respect to \cos^{-1}\text{x}, if 4 Marks
\text{x}\in(0, 1)

Ans: Let \text{u}=\sin^{-1}\sqrt{1-\text{x}^2}


Put \text{x}=\cos\theta
\Rightarrow\text{u}=\sin^{-1}\sqrt{1-\cos^2\theta}
\Rightarrow\text{u}=\sin^{-1}(\sin\theta)\ .....(\text{i})
And, \text{v}=\cos^{-1}\text{x}\ .....(\text{ii})
Now, \text{x}\in(0,1)
\Rightarrow\cos\theta\in(0,1)
\Rightarrow\theta\in\Big(0,\frac{\pi}{2}\Big)
So, from equation (i),
\text{u}=\theta
\Big[\text{Since},\sin^{-1}(\sin\theta)=\theta\text{ if }\theta\in\Big(-\frac{\pi}{2},\frac{\pi}{2}\Big)\Big]
\Rightarrow\text{u}=\cos^{-1}\text{x}\big[\text{Since},\cos\theta=\text{x}\big]
Differentiating it with respect to x,
\frac{\text{du}}{\text{dx}}=\frac{-1}{\sqrt{1-\text{x}^2}}\ .....(\text{iii})
From equation (ii),
\text{v}=\cos^{-1}\text{x}
Differentaiting it with respect to x,
\frac{\text{dv}}{\text{dx}}=\frac{-1}{\sqrt{1-\text{x}^2}}\ .....(\text{iv})
Dividing equation (iii) by (iv),
\frac{\frac{\text{du}}{\text{dx}}}{\frac{\text{dv}}{\text{dx}}}=\frac{-1}{\sqrt{1-\text{x}^2}}\times\frac{\sqrt{1-\text{x}^2}}{-1}

https://bls.smartstudies.co.in/#/exam/pdf-preview/c59cb220-8e86-4716-9ff7-82aec16b1ade/1 71/158
5/26/24, 6:19 PM Exam Automation
\therefore\frac{\text{du}}{\text{dx}}=1
Q311. Prove that the function f defined by \text{f(x)}=\begin{cases}\frac{\text{x}}{|\text{x}|+2\text{x}^2},&\text{if x}\neq0\\\text{k},&\text{ if x}=0\end{cases} remains discontinuous at x = 0, 4 Marks
regardless the choice of k.

Ans: We have, \text{f(x)}=\begin{cases}\frac{\text{x}}{|\text{x}|+2\text{x}^2},&\text{if x}\neq0\\\text{k},&\text{ if x}=0\end{cases}


At x = 0, \text{L.H.L}=\lim\limits_{\text{x}\rightarrow0^-}\frac{\text{x}}{|\text{x}|+2\text{x}^2}=\lim\limits_{\text{h}\rightarrow0}\frac{(0-\text{h})}{|0-\text{h}|+2(0-\text{h})^2}
=\lim\limits_{\text{h}\rightarrow0}\frac{-\text{h}}{\text{h}+2\text{h}^2}=\lim\limits_{\text{h}\rightarrow0}\frac{-\text{h}}{\text{h}(1+2\text{h})}=-1
\text{R.H.L}=\lim\limits_{\text{x}\rightarrow0^+}\frac{\text{x}}{|\text{x}|+2\text{x}^2}=\lim\limits_{\text{h}\rightarrow0}\frac{0+\text{h}}{|0+\text{h}|+2(0+\text{h})^2}
=\lim\limits_{\text{h}\rightarrow0}\frac{\text{h}}{\text{h}+2\text{h}^2}=\lim\limits_{\text{h}\rightarrow0}\frac{\text{h}}{\text{h}(1+2\text{h})}=1
And f(0) = k
Since, L.H.L ≠ R.H.L for any value of k.
Hence, f(x) is discontinuous at x = 0 regardless the choice of k.
Q312. If f(x), defined by the following, is continuous at x = 0, find the values of a, b and c. 4 Marks
\text{f(x)} = \begin{cases} \frac{\text{sin (a + 1)x + sin x}}{\text{x}},\quad&\text{if x < 0}\\ \text{c}, \quad &\text{if x = 0}\\ \frac{\sqrt{\text{x + bx}^{2}}-\sqrt{\text{x}}}
{\text{bx}^{3/2}},\quad&\text{if x > 0} \end{cases}.

Ans: \text{L.H.L.}=\lim\limits_{\text{x} \to 0^{-}}\Bigg(\frac{\sin\text{(a+1)x}}{\text{(a+1)x}}\text{(a+1)}\frac{\sin\text{x}}{\text{x}}\Bigg)=\text{a+2}


\text{R.H.L.}=\lim\limits_{\text{x} \to 0^{+}}\Bigg(\frac{\sqrt{1+\text{bx}}-1}{\text{bx}}\Bigg)=\lim\limits_{\text{x} \to 0^{+}}\frac{1}{\sqrt{1+\text{bx}}+1}=\frac{1}{2}.
f(0) = c
As f(x) is continuous at x = 0,
\therefore\text{c}=\frac{1}{2},\text{a+2}=\frac{1}{2}\Rightarrow\text{a}=-3/2, b can take any arbitrary value.
Q313. Find the points of discontinuity, if any of the following function: 4 Marks
\text{f(x)}=\begin{cases}\frac{\sin\text{x}}{\text{x}}+\cos\text{x},&\text{if }\text{ x}\neq0\\5,&\text{if }\text{ x}=0\end{cases}

Ans: When \text{x}\neq0, then


\text{f(x)}=\frac{\sin\text{x}}{\text{x}}+\cos\text{x}
We know that \sin\text{x} as well as the identity function x both are everwhere continuous.
So, the quotient function \frac{\sin\text{x}}{\text{x}} is continuous at each \text{x}\neq0
Also, \cos\text{x} is everwhere continuous.
Therefore, \text{f(x)}=\frac{\sin\text{x}}{\text{x}}+\cos\text{x} is continuous at each \text{x}\neq0
Let us consider the point x = 0
Given, \text{f(x)}=\begin{cases}\frac{\sin\text{x}}{\text{x}}+\cos\text{x},&\text{if }\text{ x}\neq0\\5,&\text{if }\text{ x}=0\end{cases}
We have
(\text{LHL at x}=0)=\lim_\limits{\text{x}\rightarrow0^-}\text{f(x)}=\lim_\limits{\text{h}\rightarrow0}\text{f}(0-\text{h})
=\lim_\limits{\text{h}\rightarrow0}\text{f}(-\text{h})=\lim_\limits{\text{h}\rightarrow0}\Big(\frac{\sin(-\text{h})}{-\text{h}}+\cos(-\text{h})\Big)
=\lim_\limits{\text{h}\rightarrow0}\Big(\frac{\sin(-\text{h})}{-\text{h}}\Big)+\lim_\limits{\text{h}\rightarrow0}\cos(-\text{h})=1+1=2
(\text{RHL at x}=0)=\lim_\limits{\text{x}\rightarrow0^+}\text{f(x)}=\lim_\limits{\text{h}\rightarrow0}\text{f}(0+\text{h})
=\lim_\limits{\text{h}\rightarrow0}\text{f}(\text{h})=\lim_\limits{\text{h}\rightarrow0}\Big(\frac{\sin(\text{h})}{\text{h}}+\cos(\text{h})\Big)
=\lim_\limits{\text{h}\rightarrow0}\Big(\frac{\sin(\text{h})}{\text{h}}\Big)+\lim_\limits{\text{h}\rightarrow0}\cos(\text{h})=1+1=2
Also, f(0)=5
\therefore\ \lim_\limits{\text{x}\rightarrow0^-}\text{f(x)}=\lim_\limits{\text{x}\rightarrow0^+}\text{f(x)}\neq\text{f}(0)
Thus, f(x) is discontinuous at x = 0
Hence, the only point of discontinuity for f(x) is x = 0
Q314. Show that the function g(x) = x - [x] is discontinuous at all integral points. Here [x] denotes the greatest integer function. 4 Marks

Ans: The given function is g(x) = x - [x]


It is evident that g is defined at all integral points.
Let n be an integer.
Then,
g(n) = n - [n] = n - n = 0
The left hand limit of f at x = n is,
\lim\limits_{{\text{x}}\rightarrow\text{n}^-}\text{g(x)}=\lim\limits_{{\text{x}}\rightarrow\text{n}^-}\big(\text{x}-[\text{x}]\big)=\lim\limits_{{\text{x}}\rightarrow\text{n}^-}(\text{x})-
\lim\limits_{{\text{x}}\rightarrow\text{n}^-}[\text{x}]\\=\text{n}-(\text{n}-1)=1
The right hand limit of f at x = n is,
\lim\limits_{{\text{x}}\rightarrow\text{n}^+}\text{g(x)}=\lim\limits_{{\text{x}}\rightarrow\text{n}^+}\big(\text{x}-[\text{x}]\big)\\=\lim\limits_{{\text{x}}\rightarrow\text{n}^+}(\text{x})-
\lim\limits_{{\text{x}}\rightarrow\text{n}^+}[\text{x}]=\text{n}-\text{n}=0
It is observed that the left and right hand limits of f at x = n do not coincide.
Therefore, f is not continuous at x = n
Hence, g is discontinuous at all integral points.
Q315. At what points on the following curves, is the tangent parallel to x-axis? 4 Marks
\text{y}=\text{e}^{1-\text{x}^2}\text{ on }[-1,1]

Ans: Let \text{f}(\text{x})=\text{e}^{1-\text{x}^2}


Since f(x) is a exponential function, which is continuous and differentiable on its domain.
Also,
f(1) = f(-1) = 1
Thus, all the conditions of Rolle's theorem are satisfied.
Consequently, there exists at least one point \text{c}\in(-1,1) for which f'(c) = 0.
But \text{f}'(\text{c})=0\Rightarrow2\text{c}\text{e}^ {1-\text{c}^2}=0\Rightarrow\text{c}=0 \Big(\because\ \text{e}^{1-\text{c}^2}\neq0\Big)
\therefore f(c) = f(0) = e
By the geometrical interpretetion of Rolle's theorem, (0, e) is the point on \text{y}=\text{e}^{1-\text{x}^2}where the tangent is parallel to the x-axis.
Q316. If \text{y}=\sin^{-1}\Big(\frac{\sqrt{1+\text{x}}+\sqrt{1-\text{x}}}{2}\Big), then show that \frac{\text{dy}}{\text{dx}}=\frac{-1}{2\sqrt{1-\text{x}^2}} 4 Marks

Ans: Put \text{x}\cos2\theta\Rightarrow\theta=\frac{1}{2}\cos^{-1}\text{x}


\cos2\theta=\cos^2\theta-\sin^2\theta
\cos2\theta=2\cos^2\theta-1
\cos2\theta=1-2\sin^2\theta
\therefore\text{y}=\sin^{-1}\bigg(\frac{\sqrt{2}\cos\theta+\sqrt{2}\sin\theta)}{2}\bigg)
=\sin^{-1}\Big(\sin\Big(\frac{\pi}{4}+0\Big)\Big)
\Rightarrow\text{y}=\frac{\pi}{4}+\theta=\frac{\pi}{4}+\frac{1}{2}\cos^{-1}\text{x}
\Rightarrow\frac{\text{dy}}{\text{dx}}=\frac{-1}{2\sqrt{1-\text{x}}^2}
Q317. In the following, determine the values of constants involved in the definition so that the given function is continuous: 4 Marks
\text{f(x)}=\begin{cases}2,&\text{if }\text{ x}\leq3\\\text{ax}+\text{b},&\text{if }3<\text{ x}<5\\9,&\text{if }\text{ x}\geq5\end{cases}

Ans: Given, \text{f(x)}=\begin{cases}2,&\text{if }\text{ x}\leq3\\\text{ax}+\text{b},&\text{if }3<\text{ x}<5\\9,&\text{if }\text{ x}\geq5\end{cases}


If f(x) is continuous at x = 3 and 5, then
\lim_\limits{\text{x}\rightarrow3^-}\text{f(x)}=\lim_\limits{\text{x}\rightarrow3^+}\text{f(x)} and \lim_\limits{\text{x}\rightarrow5^-}\text{f(x)}=\lim_\limits{\text{x}\rightarrow5^+}\text{f(x)}
\Rightarrow\lim_\limits{\text{h}\rightarrow0}\text{f}(3-\text{h})=\lim_\limits{\text{h}\rightarrow0}\text{f}(3+\text{h}) and \lim_\limits{\text{h}\rightarrow0}\text{f}(5-
\text{h})=\lim_\limits{\text{h}\rightarrow0}\text{f}(5+\text{h})

https://bls.smartstudies.co.in/#/exam/pdf-preview/c59cb220-8e86-4716-9ff7-82aec16b1ade/1 72/158
5/26/24, 6:19 PM Exam Automation
\Rightarrow\lim_\limits{\text{h}\rightarrow0}(2)=\lim_\limits{\text{h}\rightarrow0}\big(\text{a}(3+\text{h})+\text{b}\big) and \lim_\limits{\text{h}\rightarrow0}\big(\text{a}(5-
\text{h})+\text{b}\big)=\lim_\limits{\text{h}\rightarrow0}(9)
\Rightarrow2=3\text{a}+\text{b} and 5\text{a}+\text{b}=9
Q318. Let \text{f(x)}=\begin{cases}\frac{1-\sin^3\text{x}}{3\cos^2\text{x}},&\text{if }\text{ x}<\frac{\pi}{2}\\\text{a},&\text{if }\text{ x}=\frac{\pi}{2}\\\frac{\text{b}(1-\sin\text{x})}{(\pi- 4 Marks
2\text{x})}^2,&\text{x}>\frac{\pi}{2}\end{cases} if f(x) is continuous at \text{x}=\frac{\pi}{2}, find a and b.

Ans: It is given that the function is continuous at \text{x}=\frac{\pi}{2}


\therefore\ \text{LHL}=\text{RHL}=\text{f}\Big(\frac{\pi}{2}\Big)\ ...(\text{i})
Now, \text{f}\Big(\frac{\pi}{2}\Big)=\text{a}
\text{LHL}=\lim_\limits{\text{x}\rightarrow\frac{\pi^-}{2}}\text{f(x)}=\lim_\limits{\text{h}\rightarrow 0}\text{f}\Big(\frac{\pi}{2}-\text{h}\Big)
=\lim_\limits{\text{h}\rightarrow 0}\frac{1-\sin^3\Big(\frac{\pi}{2}-\text{h}\Big)}{3\cos^2\Big(\frac{\pi}{2}-\text{h}\Big)}=\lim_\limits{\text{h}\rightarrow 0}\frac{1-\cos^3\text{h}}
{3\sin^2\text{h}}
=\lim_\limits{\text{h}\rightarrow 0}\frac{(1-\cos\text{h})(1+\cos^2\text{h}+\cos\text{h})}{3\sin^2\text{h}}
=\lim_\limits{\text{h}\rightarrow 0}\frac{2\sin^2\frac{\text{h}}{2}(1+\cos^2\text{h}+\cos\text{h})}{3\sin^2\text{h}}
=\lim_\limits{\text{h}\rightarrow 0}\frac{2\Bigg(\frac{\sin\frac{\text{h}}{2}}{\frac{\text{h}}{2}}\Bigg)^2\times\frac{\text{h}^2}{4}\times(1+\cos^2\text{h}+\cos\text{h})}
{3\Big(\frac{\sin\text{h}}{\text{h}}\Big)^2\times\text{ h}^2}
=\lim_\limits{\text{h}\rightarrow 0}\frac{2\times\frac{1}{4}(1+\cos^2\text{h}+\cos\text{h})}{3}=\frac{1}{2}
\text{RHL}=\lim_\limits{\text{x}\rightarrow\frac{\pi^-}{2}}\text{f(x)}=\lim_\limits{\text{h}\rightarrow 0}\text{f}\Big(\frac{\pi}{2}+\text{h}\Big)
=\lim_\limits{\text{h}\rightarrow 0}\frac{\text{b}\Big(1-\sin\Big(\frac{\pi}{2}+\text{h}\Big)\Big)}{\Big(\pi-2\Big(\frac{\pi}{2}+\text{h}\Big)\Big)^2}=\lim_\limits{\text{h}\rightarrow
0}\frac{\text{b}(1-\cos\text{h})}{(\pi-\pi-2\text{h})^2}
=\lim_\limits{\text{h}\rightarrow 0}\frac{\text{b }\times\ 2\sin^2\frac{\text{h}}{2}}{(2\text{h})^2}
=\lim_\limits{\text{h}\rightarrow 0}\frac{\text{b}}{2}\Bigg(\frac{\sin\frac{\text{h}}{2}}{\frac{\text{h}}{2}}\Bigg)^2\times\frac{1}{4}
=\lim_\limits{\text{h}\rightarrow 0}\frac{\text{b}}{8}=\frac{\text{b}}{8}
Thus, using (i) we get,
\text{a}=\frac{1}{2}
And \frac{\text{b}}{8}=\frac{1}{2}
\Rightarrow\text{b}=4
Thus, \text{a}=\frac{1}{2} and \text{b}=4
Q319. Prove that \text{f(x)}=\begin{cases}\frac{\sin\text{x}}{\text{x}},&\text{x}<0\\\text{x}+1,&\text{x}\geq0\end{cases} is everywhere continuous. 4 Marks

Ans: When x < 0, we have


\text{f(x)}=\frac{\sin\text{x}}{\text{x}}
We know that \sin\text{x} as well as the identity function x are everywhere continuous.
So, the quotient function \frac{\sin\text{x}}{\text{x}} is continuous at each x < 0
When x > 0, we have
f(x) = x + 1, which is a polynomial function.
Therefore, f(x) is continuous at each x > 0
Now,
Let us consider the point x = 0
Given, \text{f(x)}=\begin{cases}\frac{\sin\text{x}}{\text{x}},&\text{x}<0\\\text{x}+1,&\text{x}\geq0\end{cases}
We have,
(\text{LHL at x = 0})=\lim\limits_{\text{x}\rightarrow0^-}\text{f(x)}=\lim\limits_{\text{h}\rightarrow0}\text{f}(0-\text{h})\\=\lim\limits_{\text{h}\rightarrow0}\text{f}(-
\text{h})=\lim\limits_{\text{h}\rightarrow0}\Big(\frac{\sin(-\text{h})}{-\text{h}}\Big)=\lim\limits_{\text{h}\rightarrow0}\Big(\frac{\sin(\text{h})}{\text{h}}\Big)=1
(\text{RHL at x = 0})=\lim\limits_{\text{x}\rightarrow0^+}\text{f(x)}\\=\lim\limits_{\text{h}\rightarrow0}\text{f}
(0+\text{h})=\lim\limits_{\text{h}\rightarrow0}\text{f(h)}=\lim\limits_{\text{h}\rightarrow0}(\text{h}+1)=1
Also, \text{f}(0)=0+1=1
\therefore\ \lim\limits_{\text{x}\rightarrow0^-}\text{f(x)}=\lim\limits_{\text{x}\rightarrow0^+}\text{f(x)}=\text{f}(0)
Thus, f(x) is continuous at x = 0
Hence, f(x) is everywherefore continuous.
Q320. Differentiate (\log\text{x})^\text{x} with respect to x. 4 Marks

Ans: Let \text{u}=(\log1+\text{x})^\text{x}


Taking log on both sides,
\log\text{u}=\log(\log\text{x})^\text{x}
\Rightarrow\log\text{u}=\text{x}\log(\log\text{x})
\Rightarrow\frac{1}{\text{u}}\frac{\text{du}}{\text{dx}}=\text{x}\frac{\text{d}}{\text{dx}}\big\{\log(\log\text{x})\big\}+\log(\log\text{x})\frac{\text{d}}{\text{dx}}(\text{x})
\Rightarrow\frac{1}{\text{u}}\frac{\text{du}}{\text{dx}}=\text{x}\Big(\frac{1}{\log\text{x}}\Big)\frac{\text{d}}{\text{dx}}(\log\text{x})+\log\log\text{x}(1)
\Rightarrow\frac{\text{du}}{\text{dx}}=\text{u}\Big[\frac{\text{x}}{\log\text{x}}\big(\frac{1}{\text{x}}\big)+\log\log\text{x}\Big]
\Rightarrow\frac{\text{du}}{\text{dx}}=(\log\text{x})^\text{x}\Big[\frac{1}{\log\text{x}}+\log\log\text{x}\Big]\ .....(\text{i})
Again, let \text{v}=\log\text{x}
\Rightarrow\frac{\text{dv}}{\text{dx}}=\frac{1}{\text{x}}\ .....(\text{ii})
Dividing equation (i) by (ii), we get
\frac{\frac{\text{du}}{\text{dx}}}{\frac{\text{dv}}{\text{dx}}}=\frac{(\log\text{x})^\text{x}\Big[\frac{1}{\log\text{x}}+\log\log\text{x}\Big]}{\frac{1}{\text{x}}}
\Rightarrow\frac{\text{du}}{\text{dv}}=\frac{(\log\text{x})^\text{x}\Big[\frac{1+\log\text{x}(\log\log\text{x})}{\log\text{x}}\Big]}{\frac{1}{\text{x}}}
\Rightarrow\frac{\text{du}}{\text{dv}}=\text{x}(\log\text{x})^{\text{x}{-1}}(1+\log\text{x}\times\log\log\text{x})
Q321. If ex + xy = ex+y, prove that \frac{\text{dy}}{\text{dx}}+\text{e}^{\text{y}-\text{x}}=0 4 Marks

Ans: Here,
ex + ey = ex+y ......(i)
Differentiating both the sides using chain rule,
\frac{\text{d}}{\text{dx}}(\text{e}^\text{x})+\frac{\text{d}}{\text{dx}}(\text{e}^\text{y})=\frac{\text{d}}{\text{dx}}(\text{e}^{\text{x}+\text{y}})
\text{e}^{\text{x}}+\text{e}^{\text{y}}\frac{\text{dy}}{\text{dx}}=\text{e}^{\text{x}+\text{y}}\frac{\text{d}}{\text{dx}}(\text{x}+\text{y})
\text{e}^{\text{x}}+\text{e}^{\text{y}}\frac{\text{dy}}{\text{dx}}=\text{e}^{\text{x}+\text{y}}\Big[1+\frac{\text{d}}{\text{dx}}\Big]
\text{e}^{\text{y}}\frac{\text{dy}}{\text{dx}}-\text{e}^{\text{x}+\text{y}}\frac{\text{dy}}{\text{dx}}=\text{e}^{\text{x}+\text{y}}-\text{e}^\text{x}
\frac{\text{dy}}{\text{dx}}=\frac{\text{e}^{\text{x}+\text{y}}-\text{e}^\text{x}}{\text{e}^\text{y}-\text{e}^{\text{x}+\text{y}}}
=\Big(\frac{\text{e}^\text{x}+\text{e}^\text{x}-\text{e}^\text{x}}{\text{e}^\text{y}-\text{e}^\text{x}-\text{e}^\text{y}}\Big)
[Using equation (i)]
\frac{\text{dy}}{\text{dx}}=-\text{e}^{\text{y}-\text{x}}
\frac{\text{dy}}{\text{dx}}+\text{e}^{\text{y}-\text{x}}=0
Q322. Evaluate: 4 Marks
\lim\limits_{X\rightarrow \frac{\pi}{6}} \bigg[ \frac{\sqrt{3}\sin x - \cos x}{x- \frac{\pi}{6}}\bigg]

Ans: \lim\limits_{X\rightarrow \frac{\pi}{6}} \Bigg[ \frac{\sqrt{3}\sin x - \cos x}{x- \frac{\pi}{6}}\Bigg] \lim\limits_{X\rightarrow \frac{\pi}{6}} \Bigg[2\frac{\Big( \frac{\sqrt{3}}{2}.\sin x-\frac{1}
{2}\cos x\Big)}{x-\frac{\pi}{6}}\Bigg]
\lim\limits_{X\rightarrow \frac{\pi}{6}} \Bigg[\frac{2\big(\cos \frac{\pi}{6}.\sin x-\sin\frac{\pi}{6}.\cos x\big)}{x-\frac{\pi}{6}}\Bigg]
2\lim\limits_{X\rightarrow\frac{\pi}{6}}\frac{\sin\big(x-\frac{\pi}{6}\big)}{\big(x - \frac{\pi}{6}\big)}
Q323. Differentiate the following functions with respect to x: 4 Marks
\frac{\text{x}^2(1-\text{x}^2)}{\cos2\text{x}}

https://bls.smartstudies.co.in/#/exam/pdf-preview/c59cb220-8e86-4716-9ff7-82aec16b1ade/1 73/158
5/26/24, 6:19 PM Exam Automation
Ans: Consider \text{y}=\frac{\text{x}^2+2}{\sqrt{\cos\text{x}}}
Differentiate it with respect to x and applying the chain and product rule, we get
\frac{\text{dy}}{\text{dx}}=\frac{\cos2\text{x}\frac{\text{d}}{\text{dx}}\text{x}^2(1-\text{x}^2)^3-\text{x}^2(1-\text{x}^2)^3\frac{\text{d}}{\text{dx}}\cos2\text{x}}{\cos^2 2\text{x}}
=\frac{\cos2\text{x}\Big[\text{x}^2\frac{\text{d}}{\text{dx}}(1-\text{x}^2)^3+(1-\text{x}^2)^3\frac{\text{d}}{\text{dx}}\text{x}^2-\text{x}^2(1-\text{x}^2)^3(-2\sin2\text{x})\Big]}{\cos^2
2\text{x}}
=\frac{\cos2\text{x}\Big[-6\text{x}^2(1-\text{x}^2)^2+(1-\text{x}^2)^32\text{x}+2\text{x}^2(1-\text{x}^2)^3\sin2\text{x}\Big]}{\cos^2 2\text{x}}
=\frac{2\text{x}(1-\text{x}^2)^2}{\cos2\text{x}}-\frac{6\text{x}^3(1-\text{x}^2)^2}{\cos2\text{x}}+\frac{2\text{x}^2(1-\text{x}^2)^3\sin2\text{x}}{\cos^2 2\text{x}}
=2\text{x}(1-\text{x}^2)\sec2\text{x}\big\{1-4\text{x}^2+\text{x}(1-\text{x}^2)\tan2\text{x}\big\}
Therefore,
\frac{\text{dy}}{\text{dx}}=2\text{x}(1-\text{x}^2)\sec2\text{x}\big\{1-4\text{x}^2+\text{x}(1-\text{x}^2)\tan2\text{x}\big\}
Q324. If \sqrt{1-\text{x}^2}+\sqrt{1-\text{y}^2}=\text{a}(\text{x}-\text{y}),\ |\text{x}|<1,\ |\text{y}|<1, show that \frac{\text{dy}}{\text{dx}}=\sqrt{\frac{1-\text{y}^2}{{1-\text{x}^2}}}. 4 Marks

Ans: \sqrt{1-\text{x}^2}+\sqrt{1-\text{y}^2}=\text{a}(\text{x}-\text{y}), put \text{x}=\sin\theta, \text{y}= \sin\phi


\sqrt{1-\sin^2\theta}+\sqrt{1-\sin^2\phi}=\text{a}(\sin\theta-\sin\theta)
\Rightarrow\cos \theta =\cos \phi = \text{a}( \sin\theta -\sin\phi)
\Rightarrow 2\cos \Big(\frac{\theta+\phi}{2}\Big)\cos\Big( \frac{\theta+\phi}{2}\Big)=2\text{a}\cos\Big(\frac{\theta+\phi}{2}\Big)\sin\Big( \frac{\theta-\theta}{2}\Big)
\Rightarrow \tan\Big( \frac{ \theta+\phi}{ 2}\Big)=\frac{1}{\text{a}}
\Rightarrow\frac{\theta-\phi}{2}=\tan^{-1}\Big(\frac{1}{\text{a}}\Big)
\Rightarrow \sin^{-1}\text{x}-\sin ^{-1}\text{ y}=2\tan^{-1}\Big(\frac{1}{\text{a}}\Big)
Differentating both sides w.e.r. x
\frac{1}{\sqrt{1-\text{x}^2}}-\frac{1}{\sqrt{1-\text{x}^2}}\frac{\text {dy}}{\text{dx}}=0
\frac{1}{\sqrt{1-\text{y}}^2}\frac{\text{dy}}{\text{dx}}=\frac{1}{\sqrt{1-\text{x}^2}} or \frac{\text{dy}}{\text{dx}}=\frac{\sqrt{1-\text{y}^2}}{\sqrt{1-\text{x}^2}}
Q325. Verify Rolle's theorem of the following function on the indicated interval 4 Marks
\text{f}(\text{x})=\sin3\text{x}\text{ on }[0,\pi]

Ans: The given function is \text{f}(\text{x})=\sin3\text{x}


Since \sin3\text{x} everywhere continuous and differentiable,
\sin3\text{x} is continuous on [0,\pi] and differentiable on (0,\pi).
Also,
\text{f}(\pi)=\text{f}(0)=0
Thus, f(x) satisfies all the conditions of Rolle's theorem.
Now, we have to show that there exists \text{c}\in(0,\pi) such that f'(c) = 0.
We have
\text{f}(\text{x})=\sin3\text{x}
\Rightarrow \text{f}'(\text{x})=3\cos3\text{x}
\therefore\ \text{f}'(\text{x})=0
\Rightarrow3\cos3\text{x}=0
\Rightarrow\cos3\text{x}=0
\Rightarrow3\text{x}=\frac{\pi}{2},\frac{3\pi}{2},....
\Rightarrow\text{x}=\frac{\pi}{6},\frac{\pi}{2},\frac{5\pi}{6}
Since, \text{c}=\frac{\pi}{4}\in(0,\pi) such that f'(c) = 0
Hence, Rolle's theorem is verified.
Q326. If \text{y}=\cos^{-1}\Big\{\frac{2\text{x}-3\sqrt{1-\text{x}^2}}{\sqrt{13}}\Big\}, find \frac{\text{dy}}{\text{dx}}. 4 Marks

Ans: Here, \text{y}=\cos^{-1}\Big\{\frac{2\text{x}-3\sqrt{1-\text{x}^2}}{\sqrt{13}}\Big\}


Let \text{x}=\cos\theta, \text{So},
\text{y}=\cos^{-1}\Big\{\frac{2\cos\theta-3\sqrt{1-\cos^2\theta}}{\sqrt{13}}\Big\}
=\cos^{-1}\Big\{\frac{2}{\sqrt{13}}\cos\theta-\frac{3}{13}\sin\theta\Big\}
Let \cos\phi=\frac{2}{\sqrt{13}}
\Rightarrow\sin\phi=\sqrt{1-\cos^2\phi}
=\sqrt{1-\Big(\frac{2}{\sqrt{13}}\Big)^2}
=\sqrt{\frac{13-4}{13}}
=\sqrt{\frac{9}{13}}
\sin\phi=\frac{3}{\sqrt{13}}
So,
\text{y}=\cos^{-1}\big\{\cos\phi\cos\theta-\sin\phi\sin\theta\big\}
=\cos^{-1}\big[\cos(\theta+\phi)\big]
\text{y}=\phi+\theta
\text{y}=\cos^{-1}\Big(\frac{2}{\sqrt{13}}\Big)+\cos^{-1}\text{x}
\Big[\text{Since, x}=\cos\theta,\cos\phi=\frac{2}{\sqrt{13}}\Big]
Differentiating it with respect to x,
\frac{\text{dy}}{\text{dx}}=0+\Big(-\frac{1}{\sqrt{1-\text{x}^2}}\Big)
\frac{\text{dy}}{\text{dx}}=-\frac{1}{\sqrt{1-\text{x}^2}}
Q327. Find \frac{\text{dy}}{\text{dx}} in the following cases: 4 Marks
(x2 + y2)2 = xy

Ans: Given, (x2 + y2)2 = xy


Differentiating with respect to x,
\frac{\text{d}}{\text{dx}}\Big(\big(\text{x}^2+\text{y}^2\big)^2\Big)=\frac{\text{d}}{\text{dx}}(\text{xy})
\Rightarrow2(\text{x}^2+\text{y}^2\big)\frac{\text{d}}{\text{dx}}(\text{x}^2+\text{y}^2\big)=\text{x}\frac{\text{dy}}{\text{dx}}+\text{y}\frac{\text{d}}{\text{dx}}\big(\text{x}\big)
[Using chain rule]
\Rightarrow2(\text{x}^2+\text{y}^2\big)\Big(2\text{x}+2\text{y}\frac{\text{dy}}{\text{dx}}\Big)=\text{x}\frac{\text{dy}}{\text{dx}}+\text{y}(1)
\Rightarrow4\text{x}(\text{x}^2+\text{y}^2\big)+4\text{y}\big(\text{x}^2+\text{y}^2\big)\frac{\text{dy}}{\text{dx}}=\text{x}\frac{\text{dy}}{\text{dx}}+\text{y}
\Rightarrow4\text{y}\big(\text{x}^2+\text{y}^2\big)\frac{\text{dy}}{\text{dx}}-\text{x}\frac{\text{dy}}{\text{dx}}=\text{y}-4\text{x}(\text{x}^2+\text{y}^2\big)
\Rightarrow\frac{\text{dy}}{\text{dx}}\big[4\text{y}\text{x}^2+4\text{y}^3-\text{x}\big]=\text{y}-4\text{x}^3-4\text{x}\text{y}^2
\Rightarrow\frac{\text{dy}}{\text{dx}}=\Big(\frac{\text{y}-4\text{x}^3-4\text{x}\text{y}^2}{4\text{y}\text{x}^2+4\text{y}^3-\text{x}}\Big)
Q328. \text{If y} = \sin (\log x) , \text{prove that} 4 Marks
x^{2} \frac{d^{2}y}{dx^{2}} + x \frac{dy}{dx} + y =0

Ans: y = \sin(\log x)
\therefore \frac{dy}{dx} = \cos (\log x) . \frac{1}{x} \Rightarrow x \frac{dy}{dx} = \cos (\log x)
\therefore x \frac{d^{2}y}{dx^{2}} + \frac{dy}{dx} = \frac{-1}{x} \sin (\log x) = -\frac{y}{x}
\Rightarrow x^{2} \frac{d^{2}y}{dx^{2}} + x \frac{dy}{dx} + y = 0
Q329. If xy = ex-y, find \frac{\text{dy}}{\text{dx}} 4 Marks

Ans: The given function is xy = ex-y


Taking logarithm on both the sides, we obtain
\log(\text{xy})=\log\big(\text{e}^{\text{x}-\text{y}})
\Rightarrow\log\text{x}+\log\text{y}=(\text{x}-\text{y})\log\text{e}

https://bls.smartstudies.co.in/#/exam/pdf-preview/c59cb220-8e86-4716-9ff7-82aec16b1ade/1 74/158
5/26/24, 6:19 PM Exam Automation
\Rightarrow\log\text{x}=\log\text{y}=(\text{x}-\text{y})\times1
\Rightarrow\log\text{x}=\log\text{y}=\text{x}-\text{y}
Differentiating both sides with respect to x, we obtain
\frac{\text{d}}{\text{dx}}(\log\text{x})+\frac{\text{d}}{\text{dx}}(\log\text{y})=\frac{\text{d}}{\text{dx}}(\text{x})-\frac{\text{dy}}{\text{dx}}
\Rightarrow\frac{1}{\text{x}}+\frac{1}{\text{y}}\frac{\text{dy}}{\text{dx}}=1-\frac{\text{dy}}{\text{dx}}
\Rightarrow\big(1+\frac{1}{\text{y}}\big)\frac{\text{dy}}{\text{dx}}=1-\frac{1}{\text{x}}
\Rightarrow\big(\frac{\text{y}+1}{\text{y}}\big)\frac{\text{dy}}{\text{dx}}=\frac{\text{x}-1}{\text{x}}
\therefore\frac{\text{dy}}{\text{dx}}=\frac{\text{y}(\text{x}-1)}{\text{x}(\text{y}+1)}
Q330. Differentiate the following functions with respect to x: 4 Marks
\log_\text{x}3

Ans: Let, \text{y}=\log_\text{x}3


\Rightarrow\ \text{y}=\frac{\log3}{\log\text{x}}\ \Big[\because\ \log_\text{a}\text{b}=\frac{\log\text{b}}{\log\text{a}}\Big]
Differentiate it with respect to x we get,
\frac{\text{dy}}{\text{dx}}=\frac{\text{d}}{\text{dx}}\Big(\frac{\log3}{\log\text{x}}\Big)
=\log3\frac{\text{d}}{\text{dx}}(\log\text{x})^{-1}
=\log3\times\Big[-1(\log\text{x})^{-2}\Big]\frac{\text{d}}{\text{dx}}(\log\text{x})
[Using chain rule]
=-\frac{\log 3}{(\log\text{x})^2}\times\frac{1}{\text{x}}
=-\Big(\frac{\log 3}{\log\text{x}}\Big)^2\times\frac{1}{\text{x}}\times\frac{1}{\log3}
=-\frac{1}{\text{x}\log3(\log_3\text{x})^2} \Big[\because \frac{\log\text{b}}{\log\text{a}}=\log_\text{a}\text{b}\Big]
So,
\frac{\text{d}}{\text{dx}}(\log_\text{x}3)=-\frac{1}{\text{x}\log3(\log_3\text{x})^2}
Q331. If x and y are connected parametrically by the equations given in Exercise without eliminating the parameter, Find0 \frac{\text{dy}}{\text{dx}}. 4 Marks
\text{x}=\text{a}\Big(\cos\text{t}+\log\tan\frac{\text{t}}{2}\Big)\text{y}=\text{a}\sin\text{t}

Ans: The given equations are \text{x}=\text{a}\Big(\cos\text{t}+\log\tan\frac{\text{t}}{2}\Big)\text{ and y}=\text{a}\sin\text{t}


Then, \frac{\text{dx}}{\text{dt}}= \text{a}.\Big[\frac{\text{d}}{\text{dt}}(\cos\text{t})+\frac{\text{d}}{\text{dt}}\Big(\log\tan\frac{\text{t}}{2}\Big)\Big]
=\text{a}\Bigg[-\sin\text{t}+\frac{1}{\tan\frac{\text{t}}{2}}.\frac{\text{d}}{\text{dt}}\Big(\tan\frac{\text{t}}{2}\Big)\Bigg]
=\text{a}\Big[-\sin\text{t}+\cot\frac{\text{t}}{2}.\sec^2\frac{\text{t}}{2}.\frac{\text{d}}{\text{dt}}\Big(\frac{\text{t}}{2}\Big)\Big]
=\text{a}\Bigg[-\sin\text{t}+\frac{\cos\frac{\text{t}}{2}}{\sin\frac{\text{t}}{2}}\times\frac{1}{\cos^2\frac{\text{t}}{2}}\times\frac{1}{2}\Bigg]
=\text{a}\Bigg[-\sin\text{t}+\frac{1}{2\sin\frac{\text{t}}{2}\cos\frac{\text{t}}{2}}\Bigg]
=\text{a}\Big(-\sin\text{t}+\frac{1}{\sin\text{t}}\Big)
=\text{a}\Big(\frac{-\sin^2\text{t}+1}{\sin\text{t}}\Big)
=\text{a}\frac{\cos^2\text{t}}{\sin\text{t}}
\frac{\text{dy}}{\text{dt}}=\text{a}\frac{\text{d}}{\text{dt}}(\sin\text{t})=\text{a}\cos\text{t}
\therefore\ \frac{\text{dy}}{\text{dx}}=\frac{\Big(\frac{\text{dy}}{\text{dt}}\Big)}{\Big(\frac{\text{dx}}{\text{dt}}\Big)}=\frac{\text{a}\cos\text{t}}{\Big(\text{a}\frac{\cos^2\text{t}}
{\sin\text{t}}\Big)}=\frac{\sin\text{t}}{\cos\text{t}}=\tan\text{t}
Q332. Verify mean value theorem for the function: 4 Marks
\text{f(x)}=\sqrt{25-\text{x}^2}\text{ in }[1,5].

Ans: We have, \text{f(x)}=\sqrt{25-\text{x}^2}\text{ in }[1,5]


Since 25 - x2 and square root function are continuous and differentiable in their domain, given function f(x) is also continuous and differentiable.
So, conditions of mean value thecorem are satisfied.
Hence, there exists atleast one \text{c}\in(1,5) such that,
\text{f}'(\text{c})=\frac{\text{f}(5)-\text{f}(1)}{5-1}
\Rightarrow\ \frac{-\text{c}}{\sqrt{25-\text{c}^2}}=\frac{0-\sqrt{24}}{4}
\Rightarrow\ 16\text{c}^2=24(25-\text{c}^2)
\Rightarrow\ 40\text{c}^2=600
\Rightarrow\ \text{c}^2=15
\Rightarrow\ \text{c}=\sqrt{15}\in(1,5)
Hence, mean value theorem has been verified.
Q333. \text{If}\cos\text{y}=\text{x}\cos(\text{a}+\text{y}),\ \text{with}\cos\text{a}\neq\pm1,\ \text{prove that}\frac{\text{dy}}{\text{dx}}=\frac{\cos^2(\text{a}+\text{y})}{\sin\text{a}} 4 Marks

Ans: It is given that, \cos\text{y}=\text{x}\cos(\text{a}+\text{y})


\therefore\ \frac{\text{d}}{\text{dx}}[\cos\text{y}]=\frac{\text{d}}{\text{dx}}[\text{x}\cos(\text{a}+\text{y})]
\Rightarrow\ -\sin\frac{\text{dy}}{\text{dx}}=\cos(\text{a}+\text{y}).\frac{\text{d}}{\text{dx}}(\text{x})+\text{x}.\frac{\text{d}}{\text{dx}}[\cos(\text{a}+\text{y})]
\Rightarrow\ -\sin\text{y}\frac{\text{dy}}{\text{dt}}=\cos(\text{a}+\text{y})+\text{x}.[-\sin(\text{a}+\text{y})]\frac{\text{dy}}{\text{dx}}
\Rightarrow\ [\text{x}\sin(\text{a}+\text{y})-\sin\text{y}]\frac{\text{dy}}{\text{dx}}=\cos(\text{a}+\text{y})\ \dots(1)
Since \cos\text{y}=\text{x}\cos(\text{a}+\text{y}),\text{x}=\frac{\cos\text{y}}{\cos(\text{a}+\text{y})}
Then, equation (1) reduces to
\Big[\frac{\cos\text{y}}{\cos(\text{a}+\text{y})}.\sin(\text{a}+\text{y})-\sin\text{y}\Big]\frac{\text{dy}}{\text{dx}}=\cos(\text{a}+\text{y})
\Rightarrow\ \ [\cos\text{y}.\sin(\text{a}+\text{y})-\sin\text{y}.\cos(\text{a}+\text{y})].\frac{\text{dy}}{\text{dx}}=\cos^2(\text{a}+\text{y})
\Rightarrow\ \sin(\text{a}+\text{y}-\text{y})\frac{\text{dy}}{\text{dx}}=\cos^2(\text{a}+\text{b})
\Rightarrow\ \frac{\text{dy}}{\text{dx}}=\frac{\cos^2(\text{a}+\text{b})}{\sin\text{a}}
Hence, proved.
Q334. Find which of the function in Exercises 2 to 10 is continuous or discontinuos at the indicated point: 4 Marks
\text{f(x)}=\begin{cases}\frac{\text{e}^{\frac{1}{\text{x}}}}{1+\text{e}^{\frac{1}{\text{x}}}},&\text{if x}\neq0\\0,&\text{if x }=\text{a}\end{cases}
at x = 0

Ans: We have, \text{f(x)}=\begin{cases}\frac{\text{e}^{\frac{1}{\text{x}}}}{1+\text{e}^{\frac{1}{\text{x}}}},&\text{if x}\neq0\\0,&\text{if x }=\text{a}\end{cases} at x = 0.


At x = 0 \text{L.H.L}=\lim\limits_{\text{h}\rightarrow0^-}\frac{\text{e}^{\frac{1}{\text{x}}}}{1+\text{e}^{\frac{1}{\text{x}}}}
=\lim\limits_{\text{h}\rightarrow0}\frac{\text{e}^\frac{1}{0-\text{h}}}{1+\text{e}^{\frac{1}{0-\text{h}}}}=\lim\limits_{\text{h}\rightarrow0}\frac{\text{e}^{\frac{1}{\text{h}}}}{1+\text{e}^{-
\frac{1}{\text{h}}}} =\frac{\text{e}^{-\infty}}{1+\text{e}^{-\infty}}=\frac{0}{1+0}=0
\text{R.H.L}=\lim\limits_{\text{h}\rightarrow0^+}\frac{\text{e}^{\frac{1}{\text{x}}}}{1+\text{e}^{\frac{1}{\text{x}}}}
=\lim\limits_{\text{h}\rightarrow0}\frac{\text{e}^{\frac{1}{0+\text{h}}}}{1+\text{e}^{\frac{1}{0+\text{h}}}}
=\lim\limits_{\text{h}\rightarrow0}\frac{\text{e}^{\frac{1}{\text{h}}}}{1+\text{e}^{\frac{1}{\text{x}}}}=\lim\limits_{\text{h}\rightarrow0}\frac{1}{\text{e}^{-\frac{1}{\text{h}}}+1} =\frac{1}
{\text{e}^{-\infty}+1}=\frac{1}{0+1}=1
Thus, L.H.L ≠ R.H.L at x = 0.
So, f(x) is discontinuous at x = 0.
Q335. Verify Rolle's Theorem for the following function: 4 Marks
f (x) = (x - 1) (x - 2)^{2}, [1,2]

Ans: f (x) = (x - 1) (x - 2)^{2}, [1,2]


The function f(x) is differentiable on [1, 2] and so it is continuous on [1, 2]
\therefore Al conditions of Rolles Theorem are satisfied
\therefore f' (x) = (x - 1) 2 (x -2) + (x - 2)^{2}
= (x - 2)[2x - 2 + x- 2] = (x - 2) (3x - 4)
\therefore f' (c) = 0 \Rightarrow c = 2 \text{ and}\ {c}\frac{4}{3}

https://bls.smartstudies.co.in/#/exam/pdf-preview/c59cb220-8e86-4716-9ff7-82aec16b1ade/1 75/158
5/26/24, 6:19 PM Exam Automation
As 1<\frac{4}{3}<2, \text{the Roll's theorem is verfied}
Q336. If \text{y}=(\sin^{-1}\text{x})^2, prove that (1-\text{x}^2)\text{y}_2-\text{xy}_1-2=0 4 Marks

Ans: Given,
\text{y}=(\sin^{-1}\text{x})^2\dots\text{ eq.1}
To prove: (1-\text{x}^2)\text{y}_2-\text{xy}_1-2=0
Let's find \frac{\text{d}^2\text{y}}{\text{dx}^2}
As, \frac{\text{d}^2\text{y}}{\text{dx}^2}=\frac{\text{d}}{\text{dx}}\Big(\frac{\text{dy}}{\text{dx}}\Big)
So, lets first find \frac{\text{dy}}{\text{dx}}
\frac{\text{dy}}{\text{dx}}=\frac{\text{d}}{\text{dx}}(\sin^{-1}\text{x})^2
Using chain rule we will differentiate the above expression:
Let \text{t}=\sin^{-1}\text{x}
\Rightarrow\frac{\text{dt}}{\text{dx}}=\frac{1}{\sqrt{(1-\text{x}^2)}} [using formula for derivative of \sin^{-1}\text{x}]
And y = t2
\frac{\text{dy}}{\text{dx}}=\frac{\text{dy}}{\text{dt}}\frac{\text{dt}}{\text{dx}}
\frac{\text{dy}}{\text{dx}}=2\text{t}\frac{1}{\sqrt{(1-\text{x}^2)}}=2\sin^{-1}\text{x}\frac{1}{\sqrt{(1-\text{x}^2)}}\dots\text{ eq. 2}
Again differentiating with respect to x applying product rule:
\frac{\text{d}^2\text{y}}{\text{dx}^2}=2\sin^{-1}\text{x}\frac{\text{d}}{\text{dx}}\Big(\frac{1}{\sqrt{1-\text{x}^2}}\Big)+\frac{2}{\sqrt{(1-\text{x}^2)}}\frac{\text{d}}{\text{dx}}\sin^{-1}\text{x}
\frac{\text{d}^2\text{y}}{\text{dx}^2}=-\frac{2\sin^{-1}\text{x}}{2(1-\text{x}^2)\sqrt{1-\text{x}^2}}(-2\text{x})+\frac{2}{(1-\text{x}^2)} \bigg[\text{using }\frac{\text{d}}{\text{dx}}
(\text{x}^\text{n})=\text{nx}^{\text{n}-1}\frac{\text{d}}{\text{dx}}\sin^{-1}\text{x}=\frac{1}{\sqrt{(1-\text{x}^2)}}\bigg]
\frac{\text{d}^2\text{y}}{\text{dx}^2}=\frac{2\text{x}\sin^{-1}\text{x}}{(1-\text{x}^2)\sqrt{1-\text{x}^2}}+\frac{2}{(1-\text{x}^2)}
(1-\text{x}^2)\frac{\text{d}^2\text{y}}{\text{dx}^2}=2+\frac{2\text{x}\sin^{-1}\text{x}}{\sqrt{1-\text{x}^2}}
Using eq. 2:
(1-\text{x}^2)\frac{\text{d}^2\text{y}}{\text{dx}^2}=2+\frac{\text{dy}}{\text{dx}}
\therefore(1-\text{x}^2)\text{y}_2-\text{xy}_1-2=0\dots\text{ proved.}
Q337. If sin y = x sin (a + y), prove that \frac{\text{dy}}{\text{dx}}=\frac{\text{sin}^{2}\text{(a+y)}}{\text{sin a}}. 4 Marks

Ans: sin y = x sin (a + y) \Rightarrow cos y \frac{\text{dy}}{\text{dx}} = x cos (a + y) \frac{\text{dy}}{\text{dx}} + sin (a + y)


\therefore\frac{\text{dy}}{\text{dx}}= \frac{\text{sin (a + y)}}{\text{cos y - x cos (a + y)}}
\text{x} = \frac{\text{sin y}}{\text{sin (a+y)}}\Rightarrow \frac{\text{dy}}{\text{dx}}=\frac{\text{sin (a+y)}}{\text{cos y -}\frac{\text{sin y}}{\text{sin (a+y)}}.\text{cos (a+y)}}
\therefore \frac{\text{dy}}{\text{dx}}= \frac{\text{sin}^{2}\text{(a+y)}}{\text{sin (a+y) cos y - cos (a + y) sin y}} = \frac{\text{sin}^{2}\text{(a + y)}}{\text{sin a}}.
Q338. Evaluate: 4 Marks
\lim\limits_{\text{y|} \to \infty}\Big(\sqrt{\text{x}^{2}+\text{x + 1}}-\text{x}\Big).

Ans: \lim\limits_{x \to \infty}\Big(\sqrt{\text{x}^{2}+\text{x + 1}}-\text{x}\Big)=\lim\limits_{\text{x}\rightarrow\infty}\frac{\Big(\sqrt{\text{x}^{2}+\text{x + 1}}-


\text{x}\Big)\Big(\sqrt{\text{x}^{2}+\text{x + 1}}+\text{x}\Big)}{\Big(\sqrt{\text{x}^{2}+\text{x + 1}}+\text{x}\Big)}
=\lim\limits_{\text{x}\rightarrow\infty}\frac{\text{x + 1}}{\sqrt{\text{x}^{2}+\text{x + 1}}+\text{x}}=\lim\limits_{\text{x}\rightarrow\infty}\frac{1+\frac{1}{\text{x}}}{\sqrt{1+\frac{1}
{\text{x}}+\frac{1}{\text{x}^{2}}+1}}
\frac{1}{1+1}=\frac{1}{2}.
Q339. If f(x) = Ax2 + Bx + C is such that f(a) = f(b), then write the value of c in Rolle's theorem. 4 Marks

Ans: We have,
f(x) = Ax2 + Bx + C
Differentiating the given function with respect to x, we get
f'(x) = 2Ax + B
⇒ f'(c) = 2Ac + B
\therefore f'(c) = 0
⇒ 2Ac + B = 0
\Rightarrow\text{c}=\frac{-\text{b}}{2\text{A}}\ ...1
\because f(a) = f(b)
\therefore Aa2 + Ba + C = Ab2 + bB + C
⇒ Aa2 + Ba = Ab2 + bB
⇒ A(a2 - b2) + B(a - b) = 0
⇒ A(a - b)(a + b) + B(a - b) = 0
⇒ (a - b)[A(a + b) + B] = 0
\Rightarrow\text{a}=\text{b},\text{A}=\frac{-\text{B}}{\text{a}+\text{b}}
\Rightarrow(\text{a}+\text{b})=\frac{-\text{B}}{\text{A}} (\because\ \text{a}\neq\text{b})
From (1) we have,
\text{c}=\frac{\text{a}+\text{b}}{2}
Q340. If \text{y}=3\cos(\log\text{x})+4\sin(\log\text{x}), prove that \text{x}^2\text{y}_2+\text{xy}_1+\text{y}=0 4 Marks

Ans: Here,
\text{y}=3\cos(\log\text{x})+4\sin(\log\text{x}),
Differentiating w.r.t.x, we get
\text{y}_1=-3\sin(\log\text{x})\times\frac{1}{\text{x}}+4\cos(\log\text{x})\times\frac{1}{\text{x}}
=\frac{-3\sin(\log\text{x})+4\cos(\log\text{x})}{\text{x}}
Differentiating w.r.t.x, we get
\text{y}_2=\frac{\Big(\frac{-3\cos(\log\text{x})}{\text{x}}-\frac{4\sin(\log\text{x})}{\text{x}}\Big)\times \text{ x}-\{-3\sin(\log\text{x})+4\cos(\log\text{x})\}}{\text{x}^2}
\Rightarrow\text{y}_2=\frac{-3\cos(\log\text{x})-4\sin(\log\text{x})-\{-3\sin(\log\text{x})+4\cos(\log\text{x})\}}{\text{x}^2}
\Rightarrow\text{y}_2=\frac{-3\cos(\log\text{x})-4\sin(\log\text{x})}{\text{x}^2}-\frac{\{-3\sin(\log\text{x})+4\cos(\log\text{x})\}}{\text{x}^2}
\Rightarrow\text{y}_2=\frac{-3\cos(\log\text{x})+4\sin(\log\text{x})}{\text{x}^2}-\frac{\{-3\sin(\log\text{x})+4\cos(\log\text{x})\}}{\text{x}^2}
\Rightarrow\text{y}_2=\frac{-\text{y}}{\text{x}^2}-\frac{\text{y}_1}{\text{x}}
\Rightarrow\text{x}^2\text{y}_2=-\text{y}-\text{xy}_1
\Rightarrow\text{x}^2\text{y}_2+\text{y}+\text{xy}_1=0
Hence proved
Q341. Differentiate the following functions with respect to x: 4 Marks
\log(\text{x}+\sqrt{\text{x}^2+1})

Ans: Let \text{y}=\log(\text{x}+\sqrt{\text{x}^2+1})


Differentiate with respect to x,
\frac{\text{dy}}{\text{dt}}=\frac{\text{d}}{\text{dy}}\log\big(\text{x}+\sqrt{\text{x}^2+1}\big)
=\frac{1}{\text{x}+\sqrt{\text{x}^2+1}}\frac{\text{d}}{\text{dx}}\Big(\text{x}+\big(\text{x}^2+1\big)^\frac{1}{2}\Big)
[Using chain rule]
=\frac{1}{\text{x}+\sqrt{\text{x}^2+1}}\Big[1+\frac{1}{2}\big(\text{x}^2+1\big)^{\frac{1}{2}-1}\frac{\text{d}}{\text{dx}}\big(\text{x}^2+1\big)\Big]
=\frac{1}{\text{x}+\sqrt{\text{x}^2+1}}\Big[1+\frac{1}{2\sqrt{\text{x}^2+1}}\times2\text{x}\Big]
=\frac{1}{\text{x}+\sqrt{\text{x}^2+1}}\Big[\frac{\sqrt{\text{x}^2+1}+\text{x}}{\sqrt{\text{x}^2+1}}\Big]
=\frac{1}{\sqrt{\text{x}^2+1}}
So,
\frac{\text{d}}{\text{dx}}\Big(\log\big(\text{x}+\sqrt{\text{x}^2+1}\big)\Big)=\frac{1}{\sqrt{\text{x}^2+1}}

https://bls.smartstudies.co.in/#/exam/pdf-preview/c59cb220-8e86-4716-9ff7-82aec16b1ade/1 76/158
5/26/24, 6:19 PM Exam Automation
Q342. Differentiate the following function w.r.t. x: 4 Marks
X^{\sin x} + (\sin x)^{\cos x}

Ans: \text{Let x}^{\sin x} = \text{u, and} ( \sin \text{x)}^{\cos \text{x}} = \text{v} \therefore \text{y = u + v} \Rightarrow \frac{\text {dy}}{\text{dx}} = \frac{\text{du}}{\text{dx}} + \frac{\text{dv}}
{\text{dx}}
\text{Getting} \frac{\text{du}}{\text{dx}} = \text{x}^{\sin{\text{x}}} \bigg[\frac{\sin\text{x}}{\text{x}} + \log \text{x} . \cos{\text{x}} \bigg]
\text{and} \frac{\text{dv}}{\text{dx}} = ( \sin \text{x})^{\cos\text{x}}[\cos\text{x} .\cot\text{x} - \sin\text{x} \log \sin \text{x}]
\therefore \frac{\text{dy}}{\text{dx}} = \text{x}^{\sin \text{x}} \bigg[ \frac{\sin \text{x}}{\text{x}} + \log\text{x}.\cos\text{x}\bigg] + (\sin\text{x}) ^{\cos{\text{x}}} [\cos\text{x}. \cot\text{x} -
\sin\text{x} \log\sin\text{x}]
Q343. Differentiate the following functions with respect to x: 4 Marks
\tan^{-1}\Big(\frac{\text{a}+\text{bx}}{\text{b}-\text{ax}}\Big)

Ans: Let \text{y}=\tan^{-1}\Big(\frac{\text{a}+\text{bx}}{\text{b}-\text{ax}}\Big)


=\tan^{-1}\bigg(\frac{\frac{\text{a}+\text{bx}}{\text{b}}}{\frac{\text{b}-\text{ax}}{\text{b}}}\bigg)
=\tan^{-1}\bigg(\frac{\frac{\text{a}}{\text{b}}+\frac{\text{bx}}{\text{b}}}{\frac{\text{b}}{\text{a}}-\frac{\text{ax}}{\text{b}}}\bigg)
=\tan^{-1}\bigg(\frac{\frac{\text{a}}{\text{b}}+\text{x}}{1-\big(\frac{\text{a}}{\text{b}}\big)\text{x}}\bigg)
\text{y}=\tan^{-1}\big(\frac{\text{a}}{\text{b}}\big)+\tan^{-1}\text{x}
\Big[\text{Since},\tan^{-1}\text{x}+\tan^{-1}\text{y}=\tan^{-1}\Big(\frac{\text{x}+\text{y}}{1-\text{xy}}\Big)\Big]
Differentiating it with respect to x,
\frac{\text{dy}}{\text{dx}}=0+\frac{1}{1+\text{x}^2}
\frac{\text{dy}}{\text{dx}}=\frac{1}{1+\text{x}^2}
Q344. Differentiate the following functions with respect to x: 4 Marks
\sin^{-1}\big(2\text{x}^2-1\big),0<\text{x}<1

Ans: Let \text{y}=\sin^{-1}\big\{2\text{x}^2-1\big\}


Let \text{x}=\cos\theta
\text{y}=\sin^{-1}\big(2\cos^2\theta-1\big)
=\sin^{-1}(\cos2\theta)
\text{y}=\sin^{-1}\Big\{\sin\Big(\frac{\pi}{2}-2\theta\Big)\Big\}\ .....(\text{i})
Here, 0<\text{x}<1
\Rightarrow\ 0<\cos\theta<1
\Rightarrow\ 0<2\theta<\frac{\pi}{2}
\Rightarrow\ 0> -2\theta>-\pi
\Rightarrow\ \frac{\pi}{2}>\Big(\frac{\pi}{2}-2\theta\Big)>-\frac{\pi}{2}
So, from equation (i),
\text{y}=\frac{\pi}{2}-2\theta
\Big[\text{Sicne}, \sin^{-1}(\cos\theta)=\theta,\text{ if }\theta\in\Big[-\frac{\pi}{2},\frac{\pi}{2}\Big]\Big]
\text{y}=\frac{\pi}{2}-2\cos^{-1}\text{x}\ \big[\text{Since x}=\cos\theta\big]
\frac{\text{dy}}{\text{dx}}=0-2\frac{\text{d}}{\text{dx}}\big(\cos^{-1}\text{x}\big)
=-2\Big(-\frac{1}{\sqrt{1-\text{x}^2}}\Big)
\frac{\text{dy}}{\text{dx}}=\frac{2}{\sqrt{1-\text{x}^2}}
Q345. Find \frac{\text{dy}}{\text{dx}} in the following cases: 4 Marks
y3 - 3xy2 = x3 + 3x2y

Ans: Here, y3 - 3xy2 = x3 + 3x2y


Differentiating with respect to x,
\Rightarrow\frac{\text{d}}{\text{dy}}(\text{y}^3)-\frac{\text{d}}{\text{dx}}(3\text{xy}^2)=\frac{\text{d}}{\text{dx}}(\text{x}^3)+\frac{\text{d}}{\text{dx}}(3\text{x}^2\text{y})
\Rightarrow 3\text{y}^2\frac{\text{dy}}{\text{dx}}-3\Big[\text{x}\frac{\text{d}}{\text{dx}}\text{y}^2\frac{\text{d}}{\text{dx}}(\text{x})\Big]=3\text{x}^2+3\Big[\text{x}^2\frac{\text{d}}
{\text{dx}}(\text{y})+\text{y}\frac{\text{d}}{\text{dx}}(\text{x}^2)\Big]
\Rightarrow 3\text{y}^2\frac{\text{dy}}{\text{dx}}-3\Big[\text{x}(2\text{y})\frac{\text{dy}}{\text{dx}}+\text{y}^2\Big]=3\text{x}^2+3\Big[\text{x}^2\frac{\text{dy}}{\text{dx}}+\text{y}
(2\text{x})\Big]
\Rightarrow 3\text{y}^2\frac{\text{dy}}{\text{dx}}-6\text{xy}\frac{\text{dy}}{\text{dx}}-3\text{y}^2+3\text{x}^2+3\text{x}^2\frac{\text{dy}}{\text{dx}}+6\text{xy}
\Rightarrow 3\text{y}^2\frac{\text{dy}}{\text{dx}}-6\text{xy}\frac{\text{dy}}{\text{dx}}-3\text{x}^2\frac{\text{dy}}{\text{dx}}=3\text{x}^2+6\text{xy}+3\text{y}^2
=3\frac{\text{dy}}{\text{dx}}(\text{y}^2-2\text{xy}-\text{x}^2)=3(\text{x}^2+2\text{xy}+\text{y}^2)
\Rightarrow\frac{\text{dy}}{\text{dx}}=\frac{3(\text{x}+\text{y})^2}{3(\text{y}^2-2\text{xy}-\text{x}^2)}
\Rightarrow\frac{\text{dy}}{\text{dx}}=\frac{(\text{x}+\text{y})^2}{\text{y}^2-2\text{xy}-\text{x}^2)}
Q346. Differentiate \tan^{-1}\Big(\frac{\sqrt{1 + \text{x}^{2}}-1}{\text{x}}\Big)with respect to \sin^{-1}\Big(\frac{2\text{x}}{1+\text{x}^{2}}\Big), when \text{x}\neq0. 4 Marks

Ans: let x = \tan\theta\ \ \therefore\ \ \theta=\tan^{-1}\text{x}


\text{u}=\tan^{-1}\Bigg(\frac{\sqrt{1+\tan^2\theta}-1}{\tan\theta}\Bigg)=\tan^{-1}\bigg(\tan\frac{\theta}{2}\bigg)=\frac{\theta}{2}=\frac{1}{2}\tan^{-1}\text{x}
\text{v}=\sin^{-1}\Bigg(\frac{2\tan\theta}{1+\tan^2\theta}\Bigg)=\sin^{-1}(\sin2\theta)=2\theta={2}\tan^{-1}\text{x}
\frac{\text{du}}{\text{dx}}=\frac{1}{2(1+\text{x}^2)};\frac{\text{dv}}{\text{dx}}=\frac{2}{1+\text{x}^2}
\frac{\text{du}}{\text{dv}}=\frac{1}{2(1+\text{x}^2)}\times\frac{1+\text{x}^2}{2}=\frac{1}{4}
\text{In case, if x}= \cot\theta\text{ answer is}-\frac{1}{4}
Q347. \text{ If }\text{ x } =\cos\text{t} (3-2\cos^2\text{t)}\ \text{and}\ \text{y}=\sin\text{t}(3-2\sin\text{ t}),\text{ find the value of}\ \frac{\text{dy}}{\text{dx}} = \text{ at t}=\frac{\pi}{\text{4}}. 4 Marks

Ans: x = 3 cos t – 2 cos3t \therefore\frac{\text{dy}}{\text{dx}}=-3\sin\text{t}+6\cos^2\text{t}\sin\text{t}


{\text{y}}=3\sin\text{t}-2\sin^3\text{t} \therefore\ \frac{\text{dy}}{\text{dx}}=-3\cos\text{t}-6\sin^2\text{t}\cos\text{t}
\frac{\text{dy}}{\text{dx}} =\frac{3\cos \text{t}(1-2\sin^2\text{t})}{3\sin\text{t}(-1+2\cos^2\text{t})} = \cot\text{t}
\text{at t}=\frac{\pi}{4},\frac{\text{dy}}{\text{dx}}=1
Q348. Differentiate the following functions with respect to x: 4 Marks
\sqrt{\frac{1-\text{x}^2}{1+\text{x}^2}}

Ans: Let \text{y}=\sqrt{\frac{1-\text{x}^2}{1+\text{x}^2}}


\text{y}=\Big(\frac{1-\text{x}^2}{1+\text{x}^2}\Big)^\frac{1}{2}
Differentiate it with respect to x,
\frac{\text{dy}}{\text{dx}}=\frac{\text{d}}{\text{dx}}\Big(\frac{1-\text{x}^2}{1+\text{x}^2}\Big)^\frac{1}{2}
=\frac{1}{2}\Big(\frac{1-\text{x}^2}{1+\text{x}^2}\Big)^{\frac{1}{2}-1}\frac{\text{d}}{\text{dx}}\Big(\frac{1-\text{x}^2}{1+\text{x}^2}\Big)
[Using chain rule]
=\frac{1}{2}\Big(\frac{1-\text{x}^2}{1+\text{x}^2}\Big)^{\frac{-1}{2}}\bigg[\frac{(1+\text{x}^2)\frac{\text{d}}{\text{dx}}(1-\text{x}^2)-(1-\text{x}^2)\frac{\text{d}}{\text{dx}}(1+\text{x}^2)}
{\big(1+\text{x}^2\big)^2}\bigg]
[Using chain rule]
=\frac{1}{2}\Big(\frac{1-\text{x}^2}{1+\text{x}^2}\Big)^{\frac{1}{2}}\bigg[\frac{(1+\text{x}^2)(-2\text{x})-(1-\text{x}^2)(2\text{x})}{\big(1+\text{x}^2\big)^2}\bigg]
=\frac{1}{2}\Big(\frac{1-\text{x}^2}{1+\text{x}^2}\Big)^{\frac{1}{2}}\bigg[\frac{-2\text{x}-2\text{x}^3-2\text{x}+2\text{x}^3}{\big(1+\text{x}^2\big)^2}\bigg]
=\frac{1}{2}\frac{-4\text{x}}{\sqrt{1-\text{x}^2}\big(1+\text{x}^2)^\frac{3}{2}}
So,
\frac{\text{d}}{\text{dx}}\bigg(\sqrt{\frac{1-\text{x}^2}{1+\text{x}^2}}\bigg)=\frac{-4\text{x}}{\sqrt{1-\text{x}^2}\big(1+\text{x}^2)^\frac{3}{2}}
Q349. Differentiate the following functions with respect to x: 4 Marks
\frac{\text{e}^\text{x}\log\text{x}}{\text{x}^2}

https://bls.smartstudies.co.in/#/exam/pdf-preview/c59cb220-8e86-4716-9ff7-82aec16b1ade/1 77/158
5/26/24, 6:19 PM Exam Automation
Ans: Let \text{y}=\frac{\text{e}^\text{x}\log\text{x}}{\text{x}^2}
Differentiate with respect to x we get,
\frac{\text{dy}}{\text{dx}}=\frac{\text{x}^2\frac{\text{d}}{\text{dx}}(\text{e}^\text{x}\log\text{x})-(\text{e}^\text{x}\log\text{x})\frac{\text{d}}{\text{dx}}\text{x}^2}{\big(\text{x}^2\big)^2}
[Using quotient rule]
=\frac{\text{x}^2\Big\{\text{e}^\text{x}\frac{\text{d}}{\text{dx}}(\log\text{x})+\log\text{x}\frac{\text{d}}{\text{dx}}(\text{e}^\text{x})\Big\}-\text{e}^\text{x}\log\text{x}\times2\text{x}}
{\text{x}^4}
[Using product rule]
=\frac{\text{x}^2\Big[\frac{\text{e}^\text{x}}{\text{x}}+\text{e}^\text{x}\log\text{x}\Big]-2\text{xe}^\text{x}\log\text{x}}{\text{x}^4}
=\frac{\frac{\text{x}^2\text{e}^\text{z}(1+\text{x}\log\text{x})}{\text{x}}-2\text{xe}^\text{z}\log\text{x}}{\text{x}^4}
=\frac{\text{xe}^\text{x}[1+\text{x}\log\text{x}-2\log\text{x}]}{\text{x}^4}
=\frac{\text{xe}^\text{x}}{\text{x}^3}\Big[\frac{1}{\text{x}}+\frac{\text{x}\log\text{x}}{\text{x}}-\frac{2\log\text{x}}{\text{x}}\Big]
=\text{e}^\text{x}\text{x}^{-2}\Big[\frac{1}{\text{x}}+\log\text{x}-\frac{2}{\text{x}}\log\text{x}\Big]
So,
\frac{\text{d}}{\text{dx}}\Big[\frac{\text{e}^\text{x}\log\text{x}}{\text{x}^2}\Big]=\text{e}^\text{x}\text{x}^{-2}\Big[\frac{1}{\text{x}}+\log\text{x}-\frac{2}{\text{x}}\log\text{x}\Big]
Q350. Differentiate the following functions with respect to x: 4 Marks
\tan^{-1}\bigg[\frac{\text{x}^\frac{1}{3}+\text{a}^{\frac{1}{3}}}{1-(\text{ax})^\frac{1}{3}}\bigg]

Ans: Let \text{y}=\tan^{-1}\bigg[\frac{\text{x}^\frac{1}{3}+\text{a}^{\frac{1}{3}}}{1-(\text{ax})^\frac{1}{3}}\bigg]


\Rightarrow\text{y}=\tan^{-1}\big(\text{x}^\frac{1}{3}\big)+\tan^{-1}\big(\text{a}^\frac{1}{3}\big)
\Big[\text{Since},\tan^{-1}\text{x}+\tan^{-1}\text{y}=\tan^{-1}\Big(\frac{\text{x}+\text{y}}{1-\text{xy}}\Big)\Big]
Differentiate it with respect to x usign chain rule,
\frac{\text{dy}}{\text{dx}}=\frac{1}{1+\big(\text{x}^\frac{1}{3}\big)^2}\times\frac{\text{d}}{\text{dx}}\big(\text{x}^\frac{1}{2}\big)+0
\Rightarrow\frac{\text{dy}}{\text{dx}}=\frac{\bigg(\frac{1}{2}\times\text{x}^{\frac{1}{3}-1}\bigg)}{1+\text{x}^\frac{2}{3}}
\therefore\frac{\text{dy}}{\text{dx}}=\frac{1}{3\text{x}^\frac{2}{3}\Big(1+\text{x}^\frac{2}{3}\Big)}
Q351. Write the number of points where f(x) = |x| + |x − 1| is continuous but not differentiable. 4 Marks

Ans: Given:
f(x) = |x| + |x - 1|
\Rightarrow\text{f(x)}=\begin{cases}-\text{x}-(\text{x}-1),&\text{x}<0\\\text{x}-(\text{x}-1),&0\leq\text{x}<1\\\text{x}+(\text{x}-1),&\text{x}\geq1\end{cases}
\Rightarrow\text{f(x)}=\begin{cases}-2\text{x}+1,&\text{x}<0\\1,&0\leq\text{x}<12\\\text{x}-1,&\text{x}\geq1\end{cases}
When x < 0, we have:
f(x) = -2x + 1 which, being a polynomial function is continuous and differentiable.
When 0\leq\text{x}<1, we have:
f(x) = 1 which, being a polynimial function is continuous and differentiable on (0, 1).
When \text{x}\leq1, we have:
f(x) = 2x - 1 which, being a polynimial function is continuous and differentiable on x > 2.
Thus, the possible points of non differentiability of f(x) are 0 and 1.
Now,
(LHL at x = 0)
\lim\limits_{\text{x}\rightarrow0^{-}}\frac{\text{f(x)}-\text{f}(0)}{\text{x}-0}
\lim\limits_{\text{x}\rightarrow0^{-}}\frac{2\text{x}+1-1}{\text{x}-0}\ [\because\text{f(x)}=-2\text{x}+1,\text{x}<0]
\lim\limits_{\text{x}\rightarrow0}\frac{-2\text{x}}{\text{x}}=-2
(RHL at x = 0)
=\lim\limits_{\text{x}\rightarrow0^{+}}\text{f(x)}-\text{f}(0)\text{x}-0
=\lim\limits_{\text{x}\rightarrow\infty}\frac{1-1}{\text{x}-1}
=0\ [\because\text{f(x)}=1,0\leq\text{x}<1]
Thus, (LHL at x = 1) \neq (RHL at x = 1)
Hence f(x) is not differentiable at x = 1.
Therefore, 0, 1 are the points where f(x) is continuous but not differentiable.
Q352. \text{if x} = a \cos\theta + b \sin\theta, y = a\sin\theta - b\cos\theta, \text{show that y} ^{2} \frac{d^{2}y}{dx^{2}}- x \frac{dy}{dx} + y = 0. 4 Marks

Ans: \frac{\text{dx}}{\text{d}\theta} = - \text{a} \sin\theta + {b} \cos\theta


\frac{\text{dy}}{\text{d}\theta} = \text{a} \cos\theta + {b} \sin\theta
\therefore \frac{\text{dy}}{\text{dx}} = \frac{\text{a} \cos\theta + {b} \sin\theta}{\text{a} \sin\theta + {b} \cos\theta} = -\frac{\text{x}}{\text{y}}
\text{or y} \frac{\text{dy}}{\text{dx}} + \text{x} = 0
\therefore \text{y} \frac{\text{d}^{2}\text{y}}{\text{dx}^{2}} + \frac{\text{dy}}{\text{dx}}.\frac{\text{dy}}{\text{dx}} + 1 = 0
\text{Using (i) we get y} \frac{\text{d}^{2}\text{y}}{\text{dx}^{2}} - \frac{\text{x}}{\text{y}} \frac{\text{dy}}{\text{dx}} +1 = 0
\therefore \text{y}^{2} \frac{\text{d}^{2}\text{y}}{\text{dx}^{2}} - \text{x} \frac{\text{dy}}{\text{dx}} + \text{y} = 0
Q353. Show that \text{f(x)}=|\cos\text{x}| is a continuous function. 4 Marks

Ans: If f is a real function on a subset of the real numbers and c be a point in the domain off, then f is
continuous at c is \lim\limits_{{\text{x}}\rightarrow\text{c}}\text{f(x)}=\text{f(c)}
Step I:
Let \text{g(x)}=|\text{x}|
\text{h(x)}=\cos\text{x}
\text{f(x)}=(\text{goh})(\text{x})
=\text{g}(\text{h(x)})
=\text{g}(\cos\text{x})
={|\cos\text{x}|}
\text{g(x)}=|\text{x}| and \text{h(x)}=\cos\text{x}
Both are continuous for all values of \text{x}\in\text{R}
Step II:
(goh)(x) is also continuous
\text{f(x)}=(\text{goh})(\text{x})
={|\cos\text{x}|}
\text{f(x)}={|\cos\text{x}|} is continuous for all values of \text{x}\in\text{R}
Q354. Find the value of \sin\Big(\cos^{-1}\frac{4}{5}+\tan^{-1}\frac{2}{3}\Big). 4 Marks

Ans: \sin\Big[\cos^{-1}\big(\frac{4}{5}\big)+\tan^{-1}\big(\frac{2}{3}\big)\Big]

https://bls.smartstudies.co.in/#/exam/pdf-preview/c59cb220-8e86-4716-9ff7-82aec16b1ade/1 78/158
5/26/24, 6:19 PM Exam Automation
=\sin\Big[\cos^{-1}\big(\frac{3}{4}\big)+\tan^{-1}\big(\frac{2}{3}\big)\Big]
=\sin\Bigg[\tan^{-1}\bigg(\frac{\frac{3}{4}+\frac{2}{3}}{1-\frac{3}{4}.\frac{2}{3}}\bigg)\Bigg]=\sin\bigg[\tan^{-1}\Big(\frac{17}{6}\Big)\bigg]
=\sin\bigg[\sin^{-1}\Big(\frac{17}{\sqrt{325}}\Big)\bigg]=\frac{17}{\sqrt{325}}
Q355. If \text{y}=\text{x}^3\log\text{x}, Prove that \frac{\text{d}^4\text{y}}{\text{dx}^4}=\frac{6}{\text{x}} 4 Marks

Ans: here,
\text{y}=\text{x}^3\log\text{x},
Differentiating w.r.t.x, we get
\frac{\text{dy}}{\text{dx}}=3\text{x}^2\log{x}+\text{x}^3\times\frac{1}{\text{x}}
=3\text{x}^2\log{\text{x}}+\text{x}^2
Differentiating w.r.t.x, we get
\frac{\text{d}^2\text{y}}{\text{dx}^2}=6\text{x}\log\text{x}+3\text{x}^2\times\frac{1}{\text{x}}+2\text{x}
=6\text{x}\log\text{x}+5\text{x}
Differentiating w.r.t.x, we get
\frac{\text{d}^2\text{y}}{\text{dx}^2}=6\log\text{x}+6\text{x}\times\frac{1}{\text{x}}+5=6\log\text{x}+11
Differentiating w.r.t.x, we get
Q356. Verify mean value theorem for the function: 4 Marks
\text{f(x)}=\frac{1}{4\text{x}-1}\text{ in }[1,4].

Ans: We have, \text{f(x)}=\frac{1}{4\text{x}-1}\text{ in }[1,4]


f(x) is continuous in [1, 4].
Also, at \text{x}=\frac{1}{4}. f(x) is discontinuous.
Hence, f(x) is continuous in [1, 4].
\text{f}'(\text{x})=-\frac{4}{(4\text{x}-1)^2}, which exists in (1, 4).
Since, conditions of mean value theorem are satisfied.
Hence, there exists a real number \text{c}\in[1,4] such that
\text{f}'(\text{c})=\frac{\text{f}(4)-\text{f}(1)}{4-1}
\Rightarrow\ \frac{-4}{(4\text{c}-1)^2}=\frac{\frac{1}{16-1}-\frac{1}{4-1}}{4-1}=\frac{\frac{1}{15}-\frac{1}{3}}{3}
\Rightarrow\ \frac{-4}{(4\text{c}-1)^2}=\frac{1-5}{45}=\frac{-4}{45}
\Rightarrow\ (4\text{c}-1)^2=45
\Rightarrow\ 4\text{c}-1=\pm3\sqrt{5}
\Rightarrow\ \text{c}=\frac{3\sqrt{5}+1}{4}\in(1,4) [neglecting (-ve) value]
Hence, mean value theorem has been verified.
Q357. If \text{y}=\tan^{-1}\Big(\frac{1-\text{x}}{1+\text{x}}\Big),, find \frac{\text{dy}}{\text{dx}}. 4 Marks

Ans: Here,
\text{y}=\tan^{-1}\Big(\frac{1-\text{x}}{1+\text{x}}\Big)
Differentiating it with respect to x using chain rule and quotinet rule,
\frac{\text{dy}}{\text{dx}}=\frac{1}{1+\big(\frac{1-\text{x}}{1+\text{x}}\big)^2}\frac{\text{d}}{\text{dx}}\Big(\frac{1-\text{x}}{1+\text{x}}\Big)
=\frac{(1+\text{x})^2}{(1+\text{x}^2+2\text{x}+1+\text{x}^2-2\text{x})}\bigg[\frac{(1+\text{x})\frac{\text{d}}{\text{dx}}(1-\text{x})-(1-\text{x})\frac{\text{d}}{\text{dx}}(1+\text{x})}
{(1+\text{x})^2}\bigg]
=\frac{(1+\text{x})^2}{2\text{x}^2+2}\Big[\frac{(1+\text{x})(-1)-(1-\text{x})(1)}{(1+\text{x})^2}\Big]
=\frac{(1+\text{x})^2}{2(\text{x}^2+1)}\Big(\frac{-\text{x}-1-1+\text{x}}{(1+\text{x})^2}\Big)
=\frac{(1+\text{x})^2}{2(\text{x}^2+1)}\times\frac{-2}{(1+\text{x})^2}
\frac{\text{dy}}{\text{dx}}=-\frac{1}{\text{x}^2+1}
Q358. Differentiate the functions given in Exercise: 4 Marks
\text{x}^{\sin\text{x}}+(\sin\text{x})^{\cos\text{x}}

Ans: Let \text{y}=\text{x}^{\sin\text{x}}+(\sin\text{x})^{\cos\text{x}}


Putting \text{u}=\text{x}^{\sin\text{x}}\text{ and v }(\sin\text{x})^{\cos\text{x}},\text{we get }\text{ y}=\text{u}+\text{v}
\therefore\ \frac{\text{dy}}{\text{dx}}=\frac{\text{du}}{\text{dx}}+\frac{\text{dv}}{\text{dx}}\ \dots\text{(i)}
Now \text{u}=\text{x}^{\sin\text{x}}\ \Rightarrow\ \log\text{u}=\log\text{x}^{\sin\text{x}}=\sin\text{x}\log\text{x}
\Rightarrow\ \frac{\text{d}}{\text{dx}}\log\text{u}=\frac{\text{d}}{\text{dx}}(\sin\text{x}\log\text{x}) \Rightarrow\ \frac{1}{\text{u}}\frac{\text{du}}{\text{dx}}=\sin\text{x}\frac{\text{d}}
{\text{dx}}\log\text{x}+\log\text{x}\frac{\text{d}}{\text{dx}}\sin\text{x}
\Rightarrow\ \frac{1}{\text{u}}\frac{\text{du}}{\text{dx}}=\sin\text{x}\frac{1}{\text{x}}+\log\text{x}(\cos\text{x}) \Rightarrow\ \frac{1}{\text{u}}\frac{\text{du}}{\text{dx}}=\frac{\sin\text{x}}
{\text{x}}+\cos\text{x}\log\text{x}
\Rightarrow\ \frac{\text{du}}{\text{dx}}=\text{u}\Big(\frac{\sin\text{x}}{\text{x}}+\cos\text{x}\log\text{x}\Big) \Rightarrow\ \frac{\text{du}}
{\text{dx}}=\text{x}^{\sin\text{x}}\Big(\frac{\sin\text{x}}{\text{x}}+\cos\text{x}\log\text{x}\Big) \dots\text{(ii)}
Again \text{v}=(\sin\text{x})^{\cos\text{x}}\ \Rightarrow\ \log\text{v}=\log(\sin\text{x})^{\cos\text{x}}=\cos\text{x}\log\sin\text{x}
\Rightarrow\ \frac{\text{d}}{\text{dx}}\log\text{v}=\frac{\text{d}}{\text{dx}}[\cos\text{x}\log(\sin\text{x})] \Rightarrow\ \frac{1}{\text{v}}\frac{\text{dy}}{\text{dx}}=\cos\text{x}\frac{\text{d}}
{\text{dx}}\log\sin\text{x}+\log\sin\text{x}\frac{\text{d}}{\text{dx}}\cos\text{x}
\Rightarrow\ \frac{1}{\text{v}}\frac{\text{dv}}{\text{dx}}=\cos\text{x}\frac{1}{\sin\text{x}}\frac{\text{d}}{\text{dx}}\sin\text{x}+\log\sin\text{x}(-\sin\text{x})
\Rightarrow\ \frac{1}{\text{v}}\frac{\text{dv}}{\text{dx}}=\cot\text{x}.\cos\text{x}-\sin\text{x}\log\sin\text{x} \Rightarrow\ \frac{\text{dv}}{\text{dx}}=\text{v}(\cot\text{x}.\cos\text{x}-
\sin\text{x}\log\sin\text{x})
\Rightarrow\ \frac{\text{dv}}{\text{dx}}=(\sin\text{x})^{\cos\text{x}}(\cot\text{x}.\cos\text{x}-\sin\text{x}\log\sin\text{x})\ \dots\text{(iii)}
Putting values from eq. (ii) and (iii) in eq. (i),
\frac{\text{dy}}{\text{dx}}=\text{x}^{\sin\text{x}}\Big(\frac{\sin\text{x}}{\text{x}}+\cos\text{x}\log\text{x}\Big)+(\sin\text{x})^{\cos\text{x}}(\cot\text{x}.\cos\text{x}-
\sin\text{x}\log\sin\text{x})
Q359. If xy2 = 1, prove that 2\frac{\text{dy}}{\text{dx}}+\text{y}^3=0 4 Marks

Ans: We have xy2 = 1 .....(i)


Differentiating with respect to x, we get,
\frac{\text{d}}{\text{dx}}(\text{xy}^2)=\frac{\text{d}}{\text{dx}}(1)
\Rightarrow\text{x}\frac{\text{d}}{\text{dx}}(\text{y}^2)+\text{y}^2\frac{\text{d}}{\text{dx}}(\text{x})=0
\Rightarrow\text{x}(2\text{y})\frac{\text{d}}{\text{dx}}+\text{y}^2(1)=0
\Rightarrow2\text{xy}\frac{\text{dy}}{\text{dx}}=-\text{y}^2
\Rightarrow\frac{\text{dy}}{\text{dx}}=\frac{-\text{y}^2}{2\text{xy}}
\Rightarrow\frac{\text{dy}}{\text{dx}}=\frac{-\text{y}}{2\text{x}}
Put \text{x}=\frac{1}{\text{y}^2} from equation (i)
\Rightarrow\frac{\text{dy}}{\text{dx}}=\frac{-\text{y}}{2\Big(\frac{1}{\text{y}^2}\Big)}
\Rightarrow2\frac{\text{dy}}{\text{dx}}=-\text{y}^3
\Rightarrow2\frac{\text{dy}}{\text{dx}}+\text{y}^3=0
Q360. Verify Rolle's theorem of the following function on the indicated interval 4 Marks
\text{f}(\text{x})=\frac{6\text{x}}{\pi}-4\sin^{2}\text{x}\text{ on }\Big[0,\frac{\pi}{6}\Big]

Ans: The given function is \text{f}(\text{x})=\frac{6\text{x}}{\pi}-4\sin^{2}\text{x}


Since \sin\text{x}\ \&\ \text{x} are everywhere continuous and differentiable, f(x) is continuous on \Big[0,\frac{\pi}{6}\Big] and differentiable on \Big(0,\frac{\pi}{6}\Big)
Also,
\text{f}\Big(\frac{\pi}{6}\Big)=\text{f}(0)=0

https://bls.smartstudies.co.in/#/exam/pdf-preview/c59cb220-8e86-4716-9ff7-82aec16b1ade/1 79/158
5/26/24, 6:19 PM Exam Automation
Thus, f(x) satisfies all the conditions of Rolle's theorem.
Now, we have to show that there must exist a point \text{c}\in\Big(0,\frac{\pi}{6}\Big) such that f'(c) = 0.
We have
\text{f}(\text{x})=\frac{6\text{x}}{\pi}-4\sin^{2}\text{x}
\Rightarrow\text{f}'(\text{x})=\frac{6}{\pi}-8\sin\text{x}\cos\text{x}
\therefore\ \text{f}'(\text{x})=0
\Rightarrow\frac{6}{\pi}-8\sin\text{x}\cos\text{x}=0
\Rightarrow\sin2\text{x}=\frac{3}{2\pi}
\Rightarrow\text{x}=\frac{1}{2}\sin^{-1}\Big(\frac{3}{2\pi}\Big)
Thus, \text{c}=\frac{1}{2}\sin^{-1}\Big(\frac{3}{2\pi}\Big)\in\Big(0,\frac{\pi}{6}\Big) such that f'(c) = 0.
Hence, Rolle's theorem is verified.
Q361. Using properties of determinants, find the value of x for which, 4 Marks
\begin{vmatrix}4-\text{x}&4+\text{x}&4+\text{x}\\4+\text{x}&4-\text{ x}&4+ \text{x}\\4+\text{x}&4+\text{x}&4-\text{x}\end{vmatrix}=0.

Ans: \text{R}_1\rightarrow\text{R}_1+\text{R}_2+\text{R}_3
And taking 12 + x common
(12+\text{x})\begin{vmatrix}1&1&1\\4+\text{x}&4-\text{ x}&4+ \text{x}\\4+\text{x}&4+\text{x}&4-\text{x}\end{vmatrix}=0
\text{C}_2\rightarrow\text{C}_2-\text{C}_1,\ \text{C}_3\rightarrow\text{C}_3-\text{C}_1
(12+\text{x})\begin{vmatrix}1&0&0\\4+\text{x}&-2\text{x}&0\\4+\text{x}&0&-2\text{x}\end{vmatrix}=0
4\text{x}^2\ (12+\text{x})\ =0
\text{x}=0 Or \text{x}=-12
Q362. Differentiate the following functions with respect to x: 4 Marks
\text{x}^{\sin^{-1}\text{x}}

Ans: Let \text{y}=\text{x}^{\sin^{-1}\text{x}}\ .....(\text{i})


Taking log on both the sides,
\log\text{y}=\log\text{x}^{\sin^{-1}\text{x}}
\log\text{y}=\sin^{-1}\text{x}\log\text{x}
Differentiating it with respect to x,
\frac{1}{\text{y}}\frac{\text{dy}}{\text{dx}}=\sin^{-1}\text{x}\frac{\text{d}}{\text{dx}}(\log\text{x})+(\log\text{x})\frac{\text{d}}{\text{dx}}(\sin^{-1}\text{x})
\Rightarrow\frac{1}{\text{y}}\frac{\text{dy}}{\text{dx}}=\sin^{-1}\text{x}\Big(\frac{1}{\text{x}}\Big)+(\log\text{x})\Big(\frac{1}{\sqrt{1-\text{x}^2}}\Big)
\Rightarrow\frac{\text{dy}}{\text{dx}}=\text{y}\Big[\frac{\sin^{-1}\text{x}}{\text{x}}+\frac{\log\text{x}}{\sqrt{1-\text{x}^2}}\Big]
\Rightarrow\frac{\text{dy}}{\text{dx}}=\text{x}^{\sin^{-1}\text{x}}\Big[\frac{\sin^{-1}\text{x}}{\text{x}}+\frac{\log\text{x}}{\sqrt{1-\text{x}^2}}\Big]
[Using equation (i)]
Q363. Differentiate the following functions with respect to x: 4 Marks
\frac{2^\text{x}\cos\text{x}}{(\text{x}^2+3)^2}

Ans: Let \text{y}=\frac{\text{2}^\text{x}\cos\text{x}}{(\text{x}^2+3)^3}


Differentiate it with respect to x we get,
\frac{\text{dy}}{\text{dx}}=\frac{\text{d}}{\text{dx}}\Big[\frac{\text{2}^\text{x}\cos\text{x}}{(\text{x}^2+3)^3}\Big]
=\bigg[\frac{(\text{x}^2+3)^2\frac{\text{d}}{\text{dx}}(2^\text{x}\cos\text{x})-(2^\text{x}\cos\text{x})\frac{\text{d}}{\text{dx}}(\text{x}^2+3)^2}{\big[(\text{x}^2+3)\big]^2}\bigg]
[Using quotient rule]
=\Bigg[\frac{(\text{x}^2+3)^2\Big\{2^\text{x}\frac{\text{d}}{\text{dx}}\cos\text{x}+\cos\text{x}\frac{\text{d}}{\text{dx}}2^\text{x}\Big\}-(2^\text{x}+3)2(\text{x}^2+3)\frac{\text{d}}{\text{dx}}
(\text{x}^2+3)}{(\text{x}^2+3)^4}\Bigg]
[Using Product rule and chain rule]
=\bigg[\frac{(\text{x}^2+3)^2\big\{-2^\text{x}\sin\text{x}+\cos\text{x}2^\text{x}\log_\text{e}2\big\}-2(2^\text{x}\cos\text{x})(\text{x}^2+3)(2\text{x})}{(\text{x}^2+3)^4}\bigg]
=\bigg[\frac{2^\text{x}(\text{x}^2+3)\big\{(\text{x}^2+3)(\cos\text{x}\log_\text{e}2-\sin\text{x})-4\text{x}\cos\text{x}\big\}}{(\text{x}^2+3)^4}\bigg]
=\frac{2^\text{x}}{(\text{x}^2+3)^2}\bigg[\cos\text{x}\log_\text{e}2-\sin\text{x}-\frac{4\text{x}\cos\text{x}}{(\text{x}^2+3)}\bigg]
So,
\frac{\text{d}}{\text{dx}}\Big[\frac{2^\text{x}\cos\text{x}}{(\text{x}^2+3)^2}\Big]=\frac{2^\text{x}}{(\text{x}^2+3)^2}\bigg[\cos\text{x}\log_\text{e}2-\sin\text{x}-\frac{4\text{x}\cos\text{x}}
{(\text{x}^2+3)}\bigg]
Q364. Differentiate the following functions with respect to x: 4 Marks
\big(\sin^{-1}\text{x}^4\big)^4

Ans: Consider \text{y}=\big(\sin^{-1}\text{x}^4\big)^4


Differentiate it with respect to x,
\frac{\text{dy}}{\text{dx}}=\frac{\text{d}}{\text{dx}}\big(\sin^{-1}\text{x}^4\big)^4
=4\big(\sin^{-1}\text{x}^4\big)\frac{\text{d}}{\text{dx}}\big(\sin^{-1}\text{x}^4\big)
[Using chain rule]
=4\big(\sin^{-1}\text{x}^4\big)^3\frac{1}{\sqrt{1-\big(\text{x}^4\big)^2}}\frac{\text{d}}{\text{dx}}\big(\text{x}^4\big)
=4\big(\sin^{-1}\text{x}^4\big)^3\frac{4\text{x}^3}{\sqrt{1-\text{x}^8}}
=\frac{16\text{x}^3\big(\sin^{-1}\text{x}^4\big)^3}{\sqrt{1-\text{x}^8}}
Hence, the solution is, \frac{\text{d}}{\text{dx}}\big(\sin^{-1}\text{x}^4\big)=\frac{16\text{x}^3\big(\sin^{-1}\text{x}^4\big)^3}{\sqrt{1-\text{x}^8}}
Q365. Discuss the continuity of the following functions at the indicated point: 4 Marks
\text{f(x)}=\begin{cases}\frac{\text{x}^\text{x}-1}{\log(1+2\text{x})}&,\text{ if x}\neq0&\\ \ &&\text{at x}=0\\7&,\text{ if x}=0\end{cases}

Ans: In this problem we need to check continuity at x = 0


Given function is,
\text{f(x)}=\begin{cases}\frac{\text{x}^\text{x}-1}{\log(1+2\text{x})}&,\text{ if x}\neq0&\\ \ &&\text{at x}=0\\7&,\text{ if x}=0\end{cases}
\therefore We need to check L.H.L, R.H.L and value of function at x = 0
Idea of logarithmic limit and exponential limit,
\lim\limits_{\text{x} \rightarrow 0}\frac{\log(1+\text{x})}{\text{x}}=1\ ...(\text{i})
\lim\limits_{\text{x} \rightarrow 0}\frac{\text{e}^{\text{x}-1}}{\text{x}}=1\ ...(\text{ii})
You must have read such limits. You can verify these by expanding log(1 + x) and ex in its taylor form.
Numerator and denominator conditions also hold for this limit like sandwich theorem.
E.g: \lim\limits_{\text{x} \rightarrow 0}\frac{\log(1+2\text{x})}{2\text{x}}=1
But, \lim\limits_{\text{x} \rightarrow 0}\frac{\log(1+2\text{x})}{\text{x}}\neq1 as denominator does not have 2x
Now we are ready to solve the problem.
Given function is,
\text{f(x)}=\begin{cases}\frac{\text{e}^\text{x}-1}{\log(1+2\text{x})}&,\text{ if x}\neq0&\\ \ \ \ \ &&\text{at x}=0\\7&,\text{ if x}=0\end{cases}\ ...(\text{iii})
Clearly,
f(0) = 7 [from equation 2]
\text{L.H.L}=\lim\limits_{\text{h} \rightarrow 0}\text{f}(0-\text{h})=\lim\limits_{\text{h} \rightarrow 0}\text{f}(-\text{h}) [putting x = -h in equation iii]
=\lim\limits_{\text{h} \rightarrow 0}\frac{\text{e}^{-\text{h}}-1}{\log1+2(-\text{h})}=\lim\limits_{\text{h} \rightarrow 0}\frac{\text{e}^{-\text{h}}-1}{\log(1-2\text{h})}
Using logarithmic and exponential limit as explained above, we have,
\text{L.H.L}=\frac{1}{2}\lim\limits_{\text{h} \rightarrow 0}\frac{\frac{(\text{e}^{-\text{h}}-1)}{-\text{h}}}{\frac{\log(1-2\text{h})}{-2\text{h}}}=\frac{1}{2}
\text{RHL}=\lim\limits_{\text{h} \rightarrow 0}\text{f}(0+\text{h})=\lim\limits_{\text{h} \rightarrow 0}\text{f(h)} [putting x = h in equation iii]
=\lim\limits_{\text{h} \rightarrow 0}\frac{\text{e}^{\text{h}}-1}{\log1+2\text{h}}=\lim\limits_{\text{h} \rightarrow 0}\frac{\text{e}^{\text{h}}-1}{\log(1+2\text{h})}

https://bls.smartstudies.co.in/#/exam/pdf-preview/c59cb220-8e86-4716-9ff7-82aec16b1ade/1 80/158
5/26/24, 6:19 PM Exam Automation
Using logarithmic and exponential limit as explained above, we have,
\text{R.H.L}=\frac{1}{2}\lim\limits_{\text{h} \rightarrow 0}\frac{\frac{(\text{e}^\text{h}-1)}{\text{h}}}{\frac{\log(1+2\text{h})}{2\text{h}}}=\frac{1}{2}
Thus, \text{L.H.L}=\text{R.H.L}\neq\text{f(0)}
\therefore\ \text{f(x)} is discontinuous at x = 0
Q366. Verify Rolle's theorem of the following function on the indicated interval 4 Marks
\text{f}(\text{x})=\frac{\text{x}}{2}-\sin\frac{\pi\text{x}}{6}\text{ on }[-1,0]

Ans: The given function is \text{f}(\text{x})=\frac{\text{x}}{2}-\sin\frac{\pi\text{x}}{6}


Since \sin\text{x}\ \&\ \frac{\text{x}}{2} are everywhere continuous and differentiable, f(x) is continuous on [-1, 0] and differentiable on (-1, 0).
Also,
f(-1) - f(0) = 0
Thus, f(x) satisfies all the conditions of Rolle's theorem.
Now, we have to show that there must exist a point \text{c}\in(-1,0) such that f'(c) = 0.
We have
\text{f}(\text{x})=\frac{\text{x}}{2}-\sin\frac{\pi\text{x}}{6}
\Rightarrow\text{f}'(\text{x})=\frac{1}{2}-\frac{\pi}{6}\cos\frac{\pi\text{x}}{6}
\therefore\ \text{f}'(\text{x})=0
\Rightarrow\frac{1}{2}-\frac{\text{x}}{6}\cos\frac{\pi\text{x}}{6}=0
\Rightarrow\cos\frac{\pi\text{x}}{6}=\frac{3}{\pi}
\Rightarrow\text{x}=\frac{-6}{\pi}\cos^{-1}\Big(\frac{3}{\pi}\Big)
Thus, \text{c}=\frac{-6}{\pi}\cos^{-1}\Big(\frac{3}{\pi}\Big)\in(-1,0) such that f'(c) = 0.
Hence, Rolle's theorem is verified.
Q367. If \text{y}=(\tan\text{x})^{(\tan\text{x})^{(\tan\text{x})^{....\infty}}}, prove that \frac{\text{dy}}{\text{dx}}=2\text{ at x}=\frac{\pi}{4} 4 Marks

Ans: We have, \text{y}=(\tan\text{x})^{(\tan\text{x})^{(\tan\text{x})^{....\infty}}}


\Rightarrow\text{y}=(\tan\text{x})^{\text{y}}
Taking log on both sides,
\log\text{y}=\log(\tan\text{x})^\text{y}
\Rightarrow\log\text{y}=\text{y}\log\tan\text{x}
Differentaiting with respect to x using chain rule,
\frac{1}{\text{y}}\frac{\text{dy}}{\text{dx}}=\text{y}\frac{\text{d}}{\text{dx}}\big\{\log\tan\text{x}\big\}+\log\tan\frac{\text{dy}}{\text{dx}}
\Rightarrow\frac{1}{\text{y}}\frac{\text{dy}}{\text{dx}}=\frac{\text{y}}{\tan\text{x}}\frac{\text{d}}{\text{dx}}(\tan\text{x})+\log\tan\frac{\text{d}}{\text{dx}}
\Rightarrow\frac{\text{dy}}{\text{dx}}\Big(\frac{1}{\text{y}}-\log\tan\text{x}\Big)=\frac{\text{y}}{\tan\text{y}}\sec^2\text{x}
Now, \Big(\frac{\text{dy}}{\text{dx}}\Big)_{\text{x}=\frac{\pi}{4}}=\frac{\text{y}\sec^3\big(\frac{\pi}{4}\big)}{\tan\big(\frac{\pi}{4}\big)}\times\frac{\text{y}}{1-\text{y}\log\tan\big(\frac{\pi}
{4}\big)}
\Rightarrow\Big(\frac{\text{dy}}{\text{dx}}\Big)_{\text{x}=\frac{\pi}{4}}=\frac{\text{y}^2\big(\sqrt{2}\big)^2}{1(1-\text{y}\log\tan1)}
\Rightarrow \Big(\frac{\text{dy}}{\text{dx}}\Big)_{\text{x}=\frac{\pi}{4}}=\frac{2(1)^2}{(1-0)}\Bigg[\because(\text{y})_{\frac{\pi}{4}}=\big(\tan\frac{\pi}{4}\big)^{\big(\tan\frac{\pi}
{4}\big)^{\big(\tan\frac{\pi}{4}\big)^{\ .....\infty}}}=1\Bigg]
\Rightarrow \Big(\frac{\text{dy}}{\text{dx}}\Big)_{\text{x}=\frac{\pi}{4}}=2
Q368. Show that the derivative of the function f given by f(x) = 2x3 - 9x2 + 12x + 9, at x = 1 and x = 2 are equal. 4 Marks

Ans: Given: f(x) = 2x3 - 9x2 + 12x + 9


Clearly, being a polynomial function, is differentiable everywhere. Therefore the derivative of f at x is given by:
\text{f}'(\text{x})=\lim_\limits{\text{h}\rightarrow0}\frac{\text{f}(\text{x}+\text{h})-\text{f(x)}}{\text{h}}
\Rightarrow\text{f}'(\text{x})=\lim_\limits{\text{h}\rightarrow0}\frac{2(\text{x}+\text{h})^3-9(\text{x}+\text{h})^2+12(\text{x}+\text{h})+9-2\text{x}^3+9\text{x}^2-12\text{x}-9}{\text{h}}
\Rightarrow\text{f}'(\text{x})=\lim_\limits{\text{h}\rightarrow0}\frac{2\text{x}^3+2\text{h}^3+6\text{x}^2\text{h}+6\text{xh}^2-9\text{x}^2-9\text{h}^2-
18\text{xh}+12\text{x}+12\text{h}+9-2\text{x}^3+9\text{x}^2-12\text{x}-9}{\text{h}}
\Rightarrow\text{f}'(\text{x})=\lim_\limits{\text{h}\rightarrow0}\frac{2\text{h}^3+6\text{x}^2\text{h}+6\text{xh}^2-9\text{h}^2-18\text{xh}+12\text{h}}{\text{h}}
\Rightarrow\text{f}'(\text{x})=\lim_\limits{\text{h}\rightarrow0}\frac{\text{h}(\text{h}^2+6\text{x}^2+6\text{xh}-9\text{h}-18\text{x}+12)}{\text{h}}
=6\text{x}^2-18\text{x}+12
So,
\text{f}'(1)=6(\text{x}^2-3\text{x}+2)
=6\times(1-3+2)
=0
\text{f}'(2)=6(\text{x}^2-3\text{x}+2)
=6\times(4-6+2)
=0
Hence, the derivative at x = 1 and x = 2 are equal.
Q369. Differentiate w.r.t. x the function in Exercise: 4 Marks
\text{x}^\text{x}+\text{x}^\text{a}+\text{a}^\text{x}+\text{a}^\text{a}, for some fixed a > 0 and x > 0

Ans: Let \text{y}=\text{x}^\text{x}+\text{x}^\text{a}+\text{a}^\text{x}+\text{a}^\text{a}


Also, let xx = u, xa = v, ax = w, and aa = s
\therefore\ y = u + v + w + s
\Rightarrow\ \frac{\text{dy}}{\text{dx}}=\frac{\text{du}}{\text{dx}}+\frac{\text{dv}}{\text{dx}}+\frac{\text{dw}}{\text{dx}}+\frac{\text{ds}}{\text{dx}}
U = xx
\Rightarrow\ \log\text{u}=\log\text{x}^\text{x}
\Rightarrow\ \log\text{u}=\text{x}\log\text{x}
Differentiating both sides with respect to x, we obtain
\frac{1}{\text{u}}\frac{\text{du}}{\text{dx}}=\log\text{x}.\frac{\text{d}}{\text{dx}}(\text{x)}+\text{x}.\frac{\text{d}}{\text{dx}}(\log\text{x})
\Rightarrow\ \frac{\text{du}}{\text{dx}}=\text{u}\Big[\log\text{x}.1+\text{x}.\frac{1}{\text{x}}\Big]
V = xa
\therefore\ \frac{\text{dv}}{\text{dx}}=\frac{\text{d}}{\text{dx}}(\text{x}^\text{a})
\Rightarrow\ \frac{\text{dv}}{\text{dx}}=\text{ax}^{\text{a}-1}\ \dots(3)
W = ax
\Rightarrow\ \log\text{w}=\log\text{a}^\text{x}
\Rightarrow\ \log\text{w}=\text{x}\log\text{a}
Differentiating both sides with respect to x, we obtain
\Rightarrow\ \frac{\text{dw}}{\text{dx}}=\text{w}\log\text{a}
\Rightarrow\ \frac{\text{dw}}{\text{dx}}=\text{a}^\text{x}\log\text{a}\ \dots(4)
S = aa
Since a is constant, aa is also a constant.
\therefore\ \frac{\text{ds}}{\text{dx}}=0\ \dots(5)
From (1), (2), (3), (4), and (5), we obtain
\frac{\text{dy}}{\text{dx}}=\text{x}^\text{x}(1+\log\text{x})+\text{ax}^{\text{a}-1}+\text{a}^\text{x}\log\text{a}+0
Q370. Discuss the continuity of \text{f(x)}=\sin|\text{x}| 4 Marks

Ans: Let \text{f(x)}=\sin|\text{x}|

https://bls.smartstudies.co.in/#/exam/pdf-preview/c59cb220-8e86-4716-9ff7-82aec16b1ade/1 81/158
5/26/24, 6:19 PM Exam Automation
This function f is defined for every real number and f can be written as the composition of two functions as,
f = goh, where g(x) = |x| and \text{h(x)}=\sin\text{x}
\big[\because (\text{goh})(\text{x})=\text{g(h(x))}=\text{g}(\sin\text{x})=|\sin\text{x}|=\text{f(x)}]\Big]
It has to be proved first that g(x) = |x| and \text{h(x)}=\sin\text{x} are continuous functions.
g(x) = |x| can be written as
\text{g(x)}=\begin{cases}-\text{x},&\text{if }\text{ x}<0\\\text{x},&\text{if }\text{ x}\geq0\end{cases}
Clearly, g is defined for all real numbers.
Let c be real number.
Case I:
If c < 0, then g(c) = -c and \lim\limits_{{\text{x}}\rightarrow\text{c}}\text{g(x)}=\lim\limits_{{\text{x}}\rightarrow\text{c}}(-\text{x})=-\text{c}
\therefore\ \lim\limits_{{\text{x}}\rightarrow\text{c}}\text{g(x)}=\text{g(c)}
Therefore, g is continuous at all points x, such that x < 0
Case II:
If c > 0, then g(c) = c and \lim\limits_{{\text{x}}\rightarrow\text{c}}\text{g(x)}=\lim\limits_{{\text{x}}\rightarrow\text{c}}(\text{x})=\text{c}
\therefore\ \lim\limits_{{\text{x}}\rightarrow\text{c}}\text{g(x)}=\text{g(c)}
Therefore, g is continuous at all points x, such that x > 0
Case III:
If c = 0, then g(c) = g(0) = 0
\lim\limits_{\text{x}\rightarrow0^-}\text{g(x)}=\lim\limits_{\text{x}\rightarrow0^-}\text{g}(-\text{x})=0
\lim\limits_{\text{x}\rightarrow0^+}\text{g(x)}=\lim\limits_{\text{x}\rightarrow0^+}\text{g(x)}=0
\therefore\ \lim\limits_{\text{x}\rightarrow0^-}\text{g(x)}=\lim\limits_{\text{x}\rightarrow0^+}\text{g(x)}=\text{g}(0)
Therefore, g is continuous at x = 0
From the above three observations, it can be concluded that g is continuous at all points.
\text{h(x)}=\sin\text{x}
It is evident that \text{h(x)}=\sin\text{x} is defined for every real number.
Let c be a real number. Put x = c + k
If x → c, then k → 0
\text{h(c)}=\sin\text{c}
\lim\limits_{{\text{x}}\rightarrow\text{c}}\text{h(x)}=\lim\limits_{{\text{x}}\rightarrow\text{c}}\sin\text{x}
=\lim\limits_{{\text{x}}\rightarrow\text{c}}\sin(\text{c}+\text{k})
=\lim\limits_{{\text{x}}\rightarrow\text{c}}\big[\sin\text{c}\cos\text{k}+\cos\text{c}\sin\text{k}\big]
=\lim\limits_{{\text{x}}\rightarrow\text{c}}(\sin\text{c}\cos\text{k})+\lim\limits_{{\text{x}}\rightarrow\text{c}}(\cos\text{c}\sin\text{k})
=\sin\text{c}\cos0+\cos\text{c}\sin0
=\sin\text{c}+0
=\sin\text{c}
\therefore\ \lim\limits_{{\text{x}}\rightarrow\text{c}}\text{h(x)}=\text{g(c)}
Therefore, h is a continuous function.
It is know that for real valued functions g and h, such that (goh) is defind at c, if g is continuse at c and if f is continuous at g(c), then (fog) is continuous at c.
Therefore, \text{f(x)}=(\text{goh})(\text{x})=\text{g(h(x))}=\text{g}(\sin\text{x})=|\sin\text{x}| is a continuous function.
Q371. Differentiate the function (\sin x)^{x} + \sin^{-1} \sqrt{x} with respect to x. 4 Marks

Ans: \text{y} = (\sin \text{x})^{\text{x}} + \sin^{-1} \sqrt{\text{x}}


\text{y = u + v} \Rightarrow \frac{\text{dy}}{\text{dx}} = \frac{\text{du}}{\text{dx}} + \frac{\text{dv}}{\text{dx}}
\text{u} = (\sin \text{x})^{\text{x}}
\Rightarrow \log \text{u = x } \log \sin \text{x}
\Rightarrow \frac{\text{du}}{\text{dx}} = (\sin \text{x})^{\text{x}} [\text{x} \cot \text{x} + \log \sin \text{x}]
\text{v} = \sin^{-1} \sqrt{\text{x}}
\Rightarrow \frac{\text{dv}}{\text{dx}} = \frac{1}{2\sqrt{\text{x - x}^{2}}}
\therefore \frac{\text{dy}}{\text{dx}} = (\sin \text{x})^{\text{x}} [\text{x} \cot \text{x} + \log \sin \text{x}] + \frac{1}{2\sqrt{\text{x - x}^{2}}}
Q372. Find which of the function: 4 Marks
\text{f(x)}=\begin{cases}\frac{2\text{x}^2-3\text{x}-2}{\text{x}-2},&\text{if x}\neq2\\5,&\text{if x}=2\end{cases}
at x = 2

Ans: We have, \text{f(x)}=\begin{cases}\frac{2\text{x}^2-3\text{x}-2}{\text{x}-2},&\text{if x}\neq2\\5,&\text{if x}=2\end{cases} at x = 2.


At x = 2, \text{L.H.L}=\lim\limits_{\text{x}\rightarrow2^-}\frac{2\text{x}^2-3\text{x}-2}{\text{x}-2}
=\lim\limits_{\text{h}\rightarrow0}\frac{2(2-\text{h})^2-3(2-\text{h})-2}{(2-\text{h})-2}
=\lim\limits_{\text{h}\rightarrow0}\frac{8+2\text{h}^2-8\text{h}-6+3\text{h}-2}{-\text{h}}
=\lim\limits_{\text{h}\rightarrow0}\frac{2\text{h}^2-5\text{h}}{-\text{h}}=\lim\limits_{\text{h}\rightarrow0}\frac{\text{h}(2\text{h}-5)}{-\text{h}}=5
\text{R.H.L}=\lim\limits_{\text{h}\rightarrow2^+}\frac{2\text{x}^2-3\text{x}-2}{\text{x}-2}
=\lim\limits_{\text{h}\rightarrow0}\frac{2(2+\text{h})^2-3(2+\text{h})-2}{(2+\text{h})-2}
=\lim\limits_{\text{h}\rightarrow0}\frac{8+2\text{h}^2+8\text{h}-6-3\text{h}-2}{\text{h}}
=\lim\limits_{\text{h}\rightarrow0}\frac{2\text{h}^2+5\text{h}}{\text{h}}=\lim\limits_{\text{h}\rightarrow0}\frac{\text{h}(2\text{h}+5)}{\text{h}}=5
and f(2) = 5
\therefore L.H.L = R.H.L = f(2)
So, f(x) is continuous at x = 2.
Q373. If \text{y}=\sin(\sin\text{x}) prove that \frac{\text{d}^2\text{y}}{\text{dx}^2}+\tan\text{x}.\frac{\text{dy}}{\text{dx}}+\text{y}\cos^2\text{x}=0 4 Marks

Ans: Given,
\text{y} = \sin (\sin \text{x})\dots\text{ eq. } 1
To prove: \frac{\text{d}^2\text{y}}{\text{dx}^2}+\tan\text{x}.\frac{\text{dy}}{\text{dx}}+\text{y}\cos^2\text{x}=0
Let's find \frac{\text{d}^2\text{y}}{\text{dx}^2}
As, \frac{\text{d}^2\text{y}}{\text{dx}^2}=\frac{\text{d}}{\text{dx}}\Big(\frac{\text{dy}}{\text{dx}}\Big)
So, lets first find \frac{\text{dy}}{\text{dx}}
\frac{\text{dy}}{\text{dx}}=\frac{\text{d}}{\text{dx}}\sin(\sin\text{x})
Using chain rule, we will differentiate the above expression:
Let \text{t}=\sin\text{x}\Rightarrow\frac{\text{dt}}{\text{dx}}=\cos\text{x}
\frac{\text{dy}}{\text{dx}}=\frac{\text{dy}}{\text{dy}}\frac{\text{dt}}{\text{dx}}
\frac{\text{dy}}{\text{dx}}=\cos\text{t}\cos\text{x}=\cos(\sin\text{x})\cos\text{x}\dots\text{ eq. 2}
Again differentiating with respect to x applying product rule:
\frac{\text{d}^2\text{y}}{\text{dx}^2}=\cos\text{x}\frac{\text{d}}{\text{dx}}\cos(\sin\text{x})+\cos(\sin\text{x})\frac{\text{d}}{\text{dx}}\cos\text{x}
Using chain rule again in the next step:
\frac{\text{d}^2\text{y}}{\text{dx}^2}=-\cos\text{x}\cos\text{x}\sin(\sin\text{x})-\sin\text{x}\cos(\sin\text{x})
\frac{\text{d}^2\text{y}}{\text{dx}^2}=-\text{y}\cos^2\text{x}-\tan\text{x}\cos\text{x}\cos(\sin\text{x})
[using eq. 1: \text{y} = \sin (\sin \text{x})]
And using eq. 2, we have:
\frac{\text{d}^2\text{y}}{\text{dx}^2}=-\text{y}\cos^2\text{x}-\tan\text{x}\frac{\text{dy}}{\text{dx}}
\frac{\text{d}^2\text{y}}{\text{dx}^2}+\text{y}\cos^2\text{x}+\tan\text{x}\frac{\text{dy}}{\text{dx}}=0\dots\text{ proved.}

https://bls.smartstudies.co.in/#/exam/pdf-preview/c59cb220-8e86-4716-9ff7-82aec16b1ade/1 82/158
5/26/24, 6:19 PM Exam Automation
Q374. Differentiate the following functions with respect to x: 4 Marks
\sin^{-1}\big\{\sqrt{1-\text{x}^2}\big\},0<\text{x}<1

Ans: Let \text{y}=\sin^{-1}\big\{\sqrt{1-\text{x}^2}\big\}


Put \text{x}=\cos2\theta
\text{y}=\sin^{-1}\big\{\sqrt{1-\cos^2\theta}\big\}
\text{y}=\sin^{-1}(\sin\theta)\ .....(\text{i})
Here, 0<\text{x}<1
\Rightarrow\ 0<\cos2\theta<1
\Rightarrow\ 0<2\theta<\frac{\pi}{2}
From equation (i),
\text{y}=\theta
\Big[\text{Since, } \sin^{-1}(\sin\theta)=\theta\text{ if }\theta \in\Big[-\frac{\pi}{2},\frac{\pi}{2}\Big]\Big]
\text{y}=\cos^{-1}\text{x}\ \big[\text{Since x}=\cos\theta\big]
Differentiating it with respect to x,
\frac{\text{dy}}{\text{dx}}=-\frac{1}{\sqrt{1-\text{x}^2}}
Q375. If \text{y}=\Big[\log\Big(\text{x}+\sqrt{\text{x}^2+1}\Big)\Big]^2 show that (1+\text{x}^2)\frac{\text{d}^2\text{y}}{\text{dx}^2}+\text{x}\frac{\text{dy}}{\text{dx}}=2 4 Marks

Ans: Given:
\text{y}=\Big[\log\Big(\text{x}+\sqrt{1+\text{x}}\Big)\Big]^2
Differentiating w.r.t. x,
\frac{\text{dy}}{\text{dx}}=\frac{\text{d}\big[\log\big(\text{x}+\sqrt{1+\text{x}^2}\big)\big]^2}{\text{dx}}
Using formula (ii),
\Rightarrow\frac{\text{dy}}{\text{dx}}=2\log\big(\text{x}+\sqrt{1+\text{x}}\big).\frac{1}{(\text{x}+\sqrt{1+\text{x}^2})}.\Big(1+\frac{2\text{x}}{2\sqrt{1+\text{x}^2}}\Big)
Using formula (i),
\Rightarrow\text{y}_1=\frac{2\log(\text{x}+\sqrt{1+\text{x}^2}}{\text{x}+\sqrt{1+\text{x}^2}}.\frac{\text{x}+\sqrt{1+\text{x}^2}}{\sqrt{1+\text{x}^2}}
\Rightarrow\text{y}_1=\frac{2\log(\text{x}\sqrt{1+\text{x}^2})}{\sqrt{1+\text{x}^2}}
Squaring both sides:
\text{(y}_1)^2=\frac{4}{1+\text{x}^2}[\log\big(\text{x}\sqrt{1+\text{x}^2}\big)
Differentiating w.r.t. x,
\Rightarrow(1+\text{x}^2)\text{y}_2\text{y}_1+2\text{x}(\text{y}_1)^2=4\text{y}_1
Using formual (iii),
\Rightarrow(1+\text{x}^2)\text{y}_2+\text{xy}_1=2
Hence proved.
Q376. Find all points of discontinuity of f, where f is defined by: 4 Marks
\text{f(x)}= \begin{cases}\text{x}^3 - 3,\ \ \text{if x}\leq 2 \\\text{x}^2 + 1,\ \text{if x}>2\end{cases}

Ans: Here \text{f(x)}= \begin{cases}\text{x}^3 - 3,\ \ \text{if x}\leq 2 \\\text{x}^2 + 1,\ \text{if x}>2\end{cases}
Function f is defined at all points of the real line.
Let c be any real number.
Three cases arise:
Case I: c < 2
^{\ \ \text{Lt}}_{\text{x}\rightarrow\text{c}}\text{f(x)} = ^{\ \ \text{Lt}}_{\text{x}\rightarrow\text{c}}\text{f(x}^2 - 3) = \text{c}^3 - 3
Also f(c) = c3 - 3
\therefore\ ^{\ \ \text{Lt}}_{\text{x}\rightarrow\text{c}}\text{f(x)} = \text{f(c)}
\therefore f is continuous at all points x < 2.
Case II: c > 2
^{\ \ \text{Lt}}_{\text{x}\rightarrow\text{c}}\text{f(x)} = ^{\ \ \text{Lt}}_{\text{x}\rightarrow\text{c}}\text{f(x}^{2} + 1) = \text{c}^{2} + 1
Also f(c) = c2 + 1
\therefore\ ^{\ \ \text{Lt}}_{\text{x}\rightarrow\text{c}}\text{f(x)} = \text{f(c)}
\therefore f is continuous at all points x > 2.
Case III: c = 2
^{\ \ \text{Lt}}_{\text{x}\rightarrow\text{2}^{-}}\text{f(x)} = ^{\ \ \text{Lt}}_{\text{x}\rightarrow\text{2}^{-}}\text{(x}^3 - 3) = 8 - 3 = 5
^{\ \ \text{Lt}}_{\text{x}\rightarrow\text{2}^{+}}\text{f(x)} = ^{\ \ \text{Lt}}_{\text{x}\rightarrow\text{2}^{+}}\text{(x}^2 + 1) = 4 + 1 = 5
Also f(2) = (2)3 - 3 = 8 - 3 = 5
\therefore\ ^{\ \ \text{Lt}}_{\text{x}\rightarrow\text{2}^{-}}\text{f(x)} = ^{\ \ \text{Lt}}_{\text{x}\rightarrow\text{2}^{+}}\text{f(x)} = \text{f(2)}
\therefore f is continuous at x = 2.
Q377. Verify Lagrange's mean value theorem for the following function on the indicated intervals. find a point 'c' in the indicated interval as stated by the Lagrange's mean value theorem. 4 Marks
f(x) = x3 - 5x2 - 3x on [1, 3]

Ans: We have,
f(x) = x3 - 5x2 - 3x
Since, polynomial function is everywhere continuous and differentiable.
Therefore, f(x) is continuous on 1, 3 and differentiable on 1, 3
Thus, both the conditions of Lagrange's theorem is satisfied.
Concequently, there exist some \text{c}\in1,3 such that
\text{f}'(\text{c})=\frac{\text{f}(3)-\text{f}(1)}{3-1}=\frac{\text{f}(3)-\text{f}(1)}{2}
Now, f(x) = x3 - 5x2 - 3x
f'(x) = 3x2 - 10x - 3
⇒ f(3) = -27
⇒ f(1) = -7
\therefore\ \text{f}'(\text{x})=\frac{\text{f}(3)-\text{f}(1)}{2}
\Rightarrow3\text{x}^2-10\text{x}-3=\frac{-20}{2}
\Rightarrow3\text{x}^2-10\text{x}+7=0
\Rightarrow\text{x}=1,\frac{7}{3}
Thus, \text{c}=\frac{7}{3}\in(1,3) such that \text{f}'(\text{c})=\frac{\text{f}(3)-\text{f}(1)}{3-1}
Hence, Lagrange's theorem is verified.
Q378. Differentiate the following functions with respect to x: 4 Marks
\text{x}^{\text{x}\cos\text{x}}+\frac{\text{x}^2+1}{\text{x}^2-1}

Ans: Let \text{y}=\text{x}^{\text{x}\cos\text{x}}+\frac{\text{x}^2+1}{\text{x}^2-1}


Also, let \text{u}=\text{x}^{\text{x}\cos\text{x}}\text{ and v}=\frac{\text{x}^2+1}{\text{x}^2-1}
\therefore\ \text{y}=\text{u}+\text{v}
\Rightarrow\ \frac{\text{dy}}{\text{dx}}=\frac{\text{du}}{\text{dx}}+\frac{\text{dv}}{\text{dx}}\ .....(\text{i})
\text{u}=\text{x}^{\text{x}\cos\text{x}}
\Rightarrow\ \log\text{u}=\log(\text{x}^{\text{x}\cos\text{x}})
\Rightarrow\log\text{u}=\text{x}\cos\text{x}\log\text{x}
Diffrerentiating both sides with respect to x, we obtain

https://bls.smartstudies.co.in/#/exam/pdf-preview/c59cb220-8e86-4716-9ff7-82aec16b1ade/1 83/158
5/26/24, 6:19 PM Exam Automation
\frac{1}{\text{u}}\frac{\text{du}}{\text{dx}}=\frac{\text{d}}{\text{dx}}(\text{x}).\cos\text{x}\log\text{x}+\text{x}.\frac{\text{d}}{\text{dx}}
(\cos\text{x}).\log\text{x}+\text{x}\cos\text{x}.\frac{\text{d}}{\text{dx}}(\log\text{x})
\Rightarrow\frac{\text{du}}{\text{dx}}=\text{u}\Big[1.\cos\text{x}.\log\text{x}+\text{x}.(-\sin\text{x})\log\text{x}+\text{x}\cos\text{x}.\frac{1}{\text{x}}\Big]
\Rightarrow\frac{\text{du}}{\text{dx}}=\text{x}^{\text{x}\cos\text{x}}(\cos\text{x}\log\text{x}-\text{x}\sin\text{x}\log\text{x}+\cos\text{x})
\Rightarrow\frac{\text{du}}{\text{dx}}=\text{x}^{\text{x}\cos\text{x}}\big[\cos\text{x}(1+\cos\text{x})-\text{x}\sin\text{x}\log\text{x}\big]\ .....(\text{ii})
\text{v}=\frac{\text{x}^2+1}{\text{x}^2-1}
\Rightarrow\log\text{v}=\log(\text{x}^2+1)-\log(\text{x}^2-1)
Differentiating both sides with respect to x, we obtain
\frac{1}{\text{v}}\frac{\text{dv}}{\text{dx}}=\frac{2\text{x}}{\text{x}^2+1}-\frac{2\text{x}}{\text{x}^2-1}
\Rightarrow\frac{\text{dv}}{\text{dx}}=\text{v}\Big[\frac{2\text{x}(\text{x}^2-1)-2\text{x}(\text{x}^2+1)}{(\text{x}^2+1)(\text{x}^2-1)}\Big]
\Rightarrow\frac{\text{dv}}{\text{dx}}=\frac{\text{x}^2+1}{\text{x}^2-1}\times\Big[\frac{-4\text{x}}{(\text{x}^2+1)(\text{x}^2-1)}\Big]
\Rightarrow\frac{\text{dv}}{\text{dx}}=\frac{-4\text{x}}{(\text{x}^2-1)^2}\ .....(\text{iii})
From (1), (2) and (3), we obtain
\frac{\text{dy}}{\text{dx}}=\text{x}^{\text{x}\cos\text{x}}\big[\cos\text{x}(1+\log\text{x})-\text{x}\sin\text{x}\log\text{x}\big]-\frac{4\text{x}}{(\text{x}^2-1)^2}
Q379. Differentiate x^{sin x} + (\sin x)^{\cos x} with respect to x. 4 Marks

Ans: \text{let y = u + v, u = x}^{\sin\text{x}}, \text{v }={(\sin \text{x})}^{\cos \text{x}}


\log \text{u} = \sin \text{x}\log \text{x}\Rightarrow\frac{\text{du}}{\text{dx}} = \text{x}^{\sin \text{x}}.\left\{\cos\text{x}\log\text{x} + \frac{\sin\text{x}}{\text{x}}\right\}
\log \text{v} = \cos \text{x}.\log(\sin \text{x})\Rightarrow\frac{\text{dv}}{\text{dx}} = (\sin\text{x})^{\cos \text{x}}. \left\{\cos \text{x}. \cot\text{x} - \sin\text{x}. \log(\sin \text{x})\right\}
\frac{\text{dy}}{\text{dx}}= \frac{\text{du}}{\text{dx}} + \frac{\text{dv}}{\text{dx}} = \text{x}^{\sin \text{x}}.\left\{ \cos\text{x}.\log\text{x} + \frac{\sin\text{x}}{\text{x}} +
(\sin\text{x})^{\cos\text{x}}\right\}\left\{\cos \text{x}.\cot \text{x} - \sin\text{x}. \log (\sin \text{x})\right\}
Q380. Differentiate the following functions with respect to x: 4 Marks
(\log\text{x})^{\cos\text{x}}

Ans: Let \text{y}=(\log\text{x})^{\cos\text{x}}\ .....(\text{i})


Taking log on both the sides,
\log\text{y}=(\log\text{x})^{\cos\text{x}}
\Rightarrow\log\text{y}=\cos\text{x}\log(\log\text{x})
Differentiating with respect to x,
\Rightarrow\frac{1}{\text{y}}\frac{\text{dy}}{\text{dx}}=\cos\text{x}\frac{\text{d}}{\text{dx}}\log(\log\text{x})+\log(\log\text{x})\frac{\text{d}}{\text{dx}}(\cos\text{x})
\Rightarrow\frac{1}{\text{y}}\frac{\text{dy}}{\text{dx}}=\frac{\cos\text{x}}{\log\text{x}}\frac{\text{d}}{\text{dx}}(\log\text{x})+\log(\log\text{x})\times(-\sin\text{x})
\Rightarrow\frac{1}{\text{y}}\frac{\text{dy}}{\text{dx}}=\frac{\cos\text{x}}{\log\text{x}}\times\big(\frac{1}{\text{x}}\big)-\sin\text{x}\log(\log\text{x})
\Rightarrow\frac{\text{dy}}{\text{dx}}=\text{y}\Big[\frac{\cos\text{x}}{\text{x}\log\text{x}}-\sin\text{x}\log(\log\text{x})\Big]
\Rightarrow\frac{\text{dy}}{\text{dx}}=(\log\text{x}^{\cos\text{x}})\Big[\frac{\cos\text{x}}{\text{x}\log\text{x}}-\sin\text{x}\log(\log\text{x})\Big]
[Using equation (i)]
Q381. If \text{f}\text{(x)}=\begin{cases}\frac{\text{x}^2-1}{\text{x}-1},& \text{for }\text{ x}\neq1 \\2,&\text{for }\text{ x}=1\end{cases} Find whether f (x) is continuous at x = 1. 4 Marks

Ans: Given,
\text{f}\text{(x)}=\frac{\text{x}^2-1}{\text{x}-1},\text{ if}\text{ x}\neq1
\text{f}\text{(x)}=2,\text{ if}\text{ x}=1
We observe
\text{(LHL at x = 1)}
\lim\limits_{\text{x} \rightarrow 1^-}\text{f}\text{ (x)}=\lim\limits_{\text{x} \rightarrow 0}(1-\text{h})
\lim\limits_{\text{x} \rightarrow 0}(1-\text{h})=\lim\limits_{\text{x} \rightarrow 0}\frac{(1-\text{h})^2-1}{(1-\text{h})^2-1}
\lim\limits_{\text{x} \rightarrow 0}\frac{1-\text{h}^2-2\text{h}-1}{1-\text{h}-1}
\lim\limits_{\text{x} \rightarrow 0}\frac{\text{h}^2-2\text{h}}{-\text{h}}
\lim\limits_{\text{x} \rightarrow 0}2-\text{h}
=2
(\text{RHL at x}=1)
\lim\limits_{\text{x} \rightarrow 1^+}\text{f}\text{(x)}=\lim\limits_{\text{h} \rightarrow 0}(1+\text{h)}
\lim\limits_{\text{h} \rightarrow 0}(1-\text{h)}=\lim\limits_{\text{h} \rightarrow 0}\frac{(1+\text{h})^2-1}{(1+\text{h})-1}
\lim\limits_{\text{h} \rightarrow 0}\frac{1+\text{h}^2+2\text{h}-1}{1+\text{h}-1}
\lim\limits_{\text{h} \rightarrow 0}\frac{\text{h}^2+2\text{h}}{\text{h}}
\lim\limits_{\text{h} \rightarrow 0}\text{h}+2
=2
Also f(x) = 2
\lim\limits_{\text{x} \rightarrow 1^-}\text{f}\text{(x)}=\lim\limits_{\text{x} \rightarrow 1^+}\text{f}\text{(x)}=\text{f}(1)
Hence f(x) is continuous at x = 1.
Q382. Verify Rolle's theorem of the following function on the indicated interval 4 Marks
\text{f}(\text{x})=4^{\sin\text{x}}\text{ on }[0,\pi]

Ans: Here,
\text{f}(\text{x})=4^{\sin\text{x}}\text{ on }[0,\pi]
We know that exponential and \sin\text{x} both are continuous and differentiable, so f(x) is continuous is [0,\pi] and differentiable is (0,\pi).
Now,
\text{f}(0)=4^{\sin0}=4^0=1
\text{f}(\pi)=4^{\sin\pi}=4^0=1
\Rightarrow\text{f}(0)=\text{f}(\pi)
So, Rolle's theorem is applicable, there must exist a point \text{c}\in(0,\pi) such that f'(c) = 0.
Now,
\text{f}(\text{x})=4^{\sin\text{x}}
\text{f}'(\text{x})=4^{\sin\text{x}}\log4\times\cos\text{x}
Now,
\text{f}'(\text{c})=0
4^{\sin\text{c}}\times\cos\times\text{c}\log4=0
\Rightarrow\cos\text{c}=0
\Rightarrow\text{c}=\frac{\pi}{2}\in(0,\pi)
Hence, Rolle's theorem is verified.
Q383. Find the values of a so that the function 4 Marks
\text{f}\text{(x)}=\begin{cases}\text{ax}+5, &\text{if}\text{ x}\leq2\\\text{x}-1, &\text{if}\text{ x}>2\end{cases} is continuous at x = 2.

Ans: Given,
\text{f}\text{(x)}=\begin{cases}\text{ax}+5, &\text{if}\text{ x}\leq2\\\text{x}-1, &\text{if}\text{ x}>2\end{cases}
We observe
\text{(LHL at x}=2)=\lim\limits_{\text{x} \rightarrow 2^-}\text{f}\text{(x)}=\lim\limits_{\text{h} \rightarrow 0}\text{f}(2-\text{h)}
=\lim\limits_{\text{h} \rightarrow 0}\text{a}(2-\text{h)}+5=2\text{a}+5
\text{(RHL at x}=2)=\lim\limits_{\text{x} \rightarrow 2^+}\text{f}\text{(x)}=\lim\limits_{\text{h} \rightarrow 0}\text{f}(2+\text{h)}
=\lim\limits_{\text{h} \rightarrow 0}(2+\text{h}-1)=1

https://bls.smartstudies.co.in/#/exam/pdf-preview/c59cb220-8e86-4716-9ff7-82aec16b1ade/1 84/158
5/26/24, 6:19 PM Exam Automation
And, \text{f}(2)=\text{a}(2)+5=2\text{a}+5
Since f(x) is continuous at x = 2, we have
=\lim\limits_{\text{x} \rightarrow 2^-}\text{f}\text{(x)}=\lim\limits_{\text{x} \rightarrow 2^+}\text{f}\text{(x)}=\text{f}(2)
\Rightarrow2\text{a}+5=1
\Rightarrow2\text{a}=-4
\Rightarrow\text{a}=-2
Q384. Find all points of discontinuity of f, where 4 Marks
\text{f(x)}=\begin{cases}\frac{\sin\text{x}}{\text{x}},\text{if x}<0\\ \text{x}+ 1, \text{if} \text{x}\geq0\end{cases}

Ans: It is given that \text{f(x)}=\begin{cases}\frac{\sin\text{x}}{\text{x}},\text{if x}<0\\ \text{x}+ 1, \text{if} \text{x}\geq0\end{cases}


We know that f is defined at all point of the real line.
Let k be a real number.
Case I: k < 0,
Then \text{f(k)} = \frac{\sin\text{k}}{\text{k}}
^{\ \ \text{lim}}_{\text{x}\rightarrow\text{k}}\text{f(x)} = ^{\ \ \text{lim}}_{\text{x}\rightarrow\text{k}}\big(\frac{\sin\text{x}}{\text{x}}\big) = \frac{\sin \text{k}}{\text{k}}
\therefore\ ^{\ \ \text{lim}}_{\text{x}\rightarrow\text{k}}\text{f(x)} = \text{f(k)}
Thus, f is continuous at all points x that is x < 0.
Case II: k > 0
Then f(k) = c + 1
^{\ \ \text{lim}}_{\text{x}\rightarrow\text{k}}\text{f(x)} = ^{\ \ \text{lim}}_{\text{x}\rightarrow\text{k}}(\text{x} + 1) = {\text{k}+ 1}
\therefore\ ^{\ \ \text{lim}}_{\text{x}\rightarrow\text{k}}\text{f(x)} =\text{f(k)}
Thus, fi s continous at all points x that is x > 0.
Case III: k = 0
THen f(k) = f(0) = 0 + 1 = 1
^{\ \ \text{lim}}_{\text{x}\rightarrow\text{0}^{-}}\text{f(x)} = ^{\ \ \text{lim}}_{\text{x}\rightarrow\text{0}^{-}}\big(\frac{\sin\text{x}}{\text{x}}\big) = 1
^{\ \ \text{lim}}_{\text{x}\rightarrow\text{0}^{+}}\text{f(x)} = ^{\ \ \text{lim}}_{\text{x}\rightarrow\text{0}^{+}}(\text{x} + 1) = 1
\Rightarrow ^{\ \ \text{lim}}_{\text{x}\rightarrow\text{k}^{-}}\text{f(x)} = ^{\ \ \text{lim}}_{\text{x}\rightarrow\text{k}^{+}}\text{f(x)} = \text{f(x)}
Hence , f is continuous at x = 0.
Therefore, f is continuous at all points of the real line.
Q385. Find all points of discontinuity of f, where f is defined by: 4 Marks
\text f(\text x)=\begin{cases}\left|\text x\right|+3, \text{if x}\leq-3\\-2 \text{x},\text{if}-3 < \text x > 3\\6\text{x}+2,\text{if}\ \text{x}\geq3\end{cases}

Ans: Here \text f(\text x)=\begin{cases}\left|\text x\right|+3, \text{if x}\leq-3\\-2 \text{x},\text{if}-3 < \text x > 3\\6\text{x}+2,\text{if}\ \text{x}\geq3\end{cases}
Function f is defined for all points of thr real line.
Let c be any real number.
Five cases arise:
Case I: c < -3
^{\ \ \text{Lt}}_{\text{x}\rightarrow\text{c}}\text{f(x)} = ^{\ \ \text{Lt}}_{\text{x}\rightarrow\text{c}}(|\text{x}|+ 3) = |\text{c}| + 3
Also f(c) = |c| + 3
\therefore\ ^{\ \ \text{Lt}}_{\text{x}\rightarrow\text{c}}\text{f(x)} = \text{f(c)}
\therefore f is continuous at all point x < -3.
Case II: c = -3
^{\ \ \text{Lt}}_{\text{x}\rightarrow\text{c}{-}}\text{f(x)} = ^{\ \ \text{Lt}}_{\text{x}\rightarrow\text{c}{-}}(|\text{x}|+ 3) = |-3| + 3 = 3 +3 =6
^{\ \ \text{Lt}}_{\text{x}\rightarrow\text{c}{ +}}\text{f(x)} = ^{\ \ \text{Lt}}_{\text{x}\rightarrow\text{c}{ +}}\text{(-2x)} = 6
Also f(c) = f(-3) = |-3|+3 = 3 + 3 = 6
\therefore\ ^{\ \ \text{Lt}}_{\text{x}\rightarrow\text{c}^{-}}\text{f(x)} = ^{\ \ \text{Lt}}_{\text{x}\rightarrow\text{c}^{+}}\text{f(x)} =\text{f(c)}= 6​​
\therefore f is continuous at x= -3
Case III: -3 < c < 3
f(x) = -2 x is a continuous function as it is a polynomial.
Case IV: c = 3
^{\ \ \text{Lt}}_{\text{x}\rightarrow\text{c}^{-}}\text{f(x)} = ^{\ \ \text{Lt}}_{\text{x}\rightarrow\text{c}^{-}}\text{(-2x)} = -6​​
^{\ \ \text{Lt}}_{\text{x}\rightarrow\text{c}^{+}}\text{f(x)} = ^{\ \ \text{Lt}}_{\text{x}\rightarrow\text{c}^{+}}\text{(6x + 2)} = 18 + 2 = 20
\therefore\ ^{\ \ \text{Lt}}_{\text{x}\rightarrow\text{c}^{-}}\text{f(x)}\neq ^{\ \ \text{Lt}}_{\text{x}\rightarrow\text{c}^{+}}\text{f(x)}
\therefore f is discontinuous at x = 3.
Case V: c > 3
^{\ \ \text{Lt}}_{\text{x}\rightarrow\text{c}}\text{f(x)} = ^{\ \ \text{Lt}}_{\text{x}\rightarrow\text{c}}\text{(6x+2)}= \text{6c}+ 2
Also f(c) = 6c +2
\therefore\ ^{\ \ \text{Lt}}_{\text{x}\rightarrow\text{c}}\ \text{f(x)} = \text{f(c)}
\therefore f is continuous at all points x > 3.
Q386. The function f(x) is befined as follows: \text{f(x)}=\begin{cases}\text{x}^2+\text{ax}+\text{b},&0\leq\text{x} 4 Marks
<2\\3\text{x}+2,&2\leq\text{x}\leq4\\2\text{ax}+5\text{b},&4<\text{x}\leq8\end{cases} if is continuous on [0, 8] find the value of a and b.

Ans: It is given that the f(x) is continuous on [0, 8]


f(x) is continuous at x = 2 and x = 4
Now, At x = 2
LHL = RHL = f(2) ....(A)
f(2) = 3 × 2 + 2 = 8 ....(i)
\text{LHL}=\lim\limits_{\text{x}\rightarrow2^-}\text{f(x)}=\lim\limits_{\text{h}\rightarrow0}(2-\text{h})\\=\lim\limits_{\text{h}\rightarrow0}(2-\text{h}^2)+\text{a}(2-
\text{h})+\text{b}=4+2\text{a}+\text{b}
From (A)
4 + 2a + b = 8
2a + b = 4 ....(B)
Now, At x = 4
LHL = RHL = f(4) ....(C)
f(4) = 3 × 4 + 2 = 14 ....(ii)
\text{RHL}=\lim\limits_{\text{x}\rightarrow4^+}\text{f(x)}=\lim\limits_{\text{h}\rightarrow0}(4+\text{h})\\=\lim\limits_{\text{h}\rightarrow0}2\text{a}
(4+\text{h})+5\text{b}=8\text{a}+5\text{b}
From (c) we get
8a + 5b = 14 ....(D)
Solving (B) and (D) we get
a = 3 and b = -2
Q387. Discuss the applicability of Lagrange's mean value theorem for the function: 4 Marks
f(x) = |x| on [−1, 1]

Ans: Here,
f(x) = |x| on [−1, 1]
\text{f}(\text{x})=\begin{cases}-\text{x},&\text{x}<0\\\text{x},&\text{x}\geq0\end{cases}
For differentiability at x = 0

https://bls.smartstudies.co.in/#/exam/pdf-preview/c59cb220-8e86-4716-9ff7-82aec16b1ade/1 85/158
5/26/24, 6:19 PM Exam Automation
\text{LHD}=\lim_\limits{\text{x}\rightarrow0^-}\frac{\text{f}(0-\text{h})-\text{f}(0)}{-\text{h}}
=\lim_\limits{\text{h}\rightarrow0}\frac{-(0-\text{h})-0}{-\text{h}}
=\lim_\limits{\text{h}\rightarrow0}\frac{\text{h}}{-\text{h}}
\text{LHD}=-1
\text{RHD}=\lim_\limits{\text{x}\rightarrow0^+}\frac{\text{f}(0+\text{h})-\text{f}(0)}{\text{h}}
=\lim_\limits{\text{h}\rightarrow0}\frac{(0+\text{h})-0}{\text{h}}
=\lim_\limits{\text{h}\rightarrow0}\frac{\text{h}}{\text{h}}
=1
\text{LHD}\neq\text{RHD}
⇒ f(x) is not differentiable at \text{x}=0\in(-1,1)
Hence, Lagrange's mean value theorem is verified.
Q388. If \text{f}:[-5,5]\rightarrow\text{R} is differentiable and if f'(x) does not vanish anywhere, then prove that \text{f}(-5)\pm\text{f}(5). 4 Marks

Ans: It is given that \text{f}:[-5,5]\rightarrow\text{R} is a differentiable function.


Since every differentiable function is continuous function, we obtain
1. f is continuous on [-5, 5].
2. f is differentiable on (-5, 5).
Therefore, by the Mean Value Theorem, there exists \text{c}\in(-5,5) such that
\text{f}'(\text{c})=\frac{\text{f}(5)-\text{f}(-5)}{5-(-5)}
\Rightarrow10\text{f}'(\text{c})=\text{f}(5)-f(-5)
It is also given that f'(x) does not vanish anywhere.
\therefore\ \text{f}'(\text{c})\neq0
\Rightarrow10\text{f}'(\text{c})\neq0
\Rightarrow\text{f}(5)-\text{f}(-5)\neq0
\Rightarrow\text{f}(5)\neq\text{f}(-5)
Hence, proved.
Q389. Verify mean value theorem for the function: 4 Marks
\text{f(x)}=\text{x}^3-2\text{x}^2-\text{x}+3\text{ in }[0,1].

Ans: Consider, \text{f(x)}=\text{x}^3-2\text{x}^2-\text{x}+3\text{ in }[0,1]


Since, f(x) is a polynomial function
Hence, f(x) is continuous in [0,1]
\text{f(x)}=3\text{x}^2-4\text{x}-1 which ecists in (0, 1).
Hence, f(x) is differentiable in (0, 1).
Since, conditions of mean value theorem are satisfied.
Therefore, by mean value theorem \exists\text{ c}\in(0,1), such that
\text{f}'(\text{c})=\frac{\text{f}(1)-\text{f}(0)}{1-0}
\Rightarrow\ 3\text{c}^2-4\text{c}-1=\frac{[1-2-1+3]-[0+3]}{1-0}
\Rightarrow\ 3\text{c}^2-4\text{c}-1=\frac{-2}{1}
\Rightarrow\ 3\text{c}^2-4\text{c}+1=0
\Rightarrow\ 3\text{c}^2-3\text{c}-\text{c}+1=0
\Rightarrow\ 3\text{c}(\text{c}-1)-1(\text{c}-1)=0
\Rightarrow\ (3\text{c}-1)(\text{c}-1)=0
\Rightarrow\ \text{c}=\frac{1}{3},1, where \frac{1}{3}\in(0,1)
Hence, the mean value theorem has been verified.
Q390. If \text{x}=\text{a}(\theta-\sin\theta),\text{y}=\text{a}(1-\cos\theta),\text{a}>0, then find \frac{\text{d}^2\text{y}}{\text{dx}^2} at \theta=\frac{\pi}{3}. 4 Marks

Ans: \frac{\text{dy}}{\text{d}\theta}=\text{a}\sin\theta,\frac{\text{dx}}{\text{d}\theta}=\text{a}(1-\cos\theta)
\frac{\text{dy}}{\text{dx}}=\frac{\sin\theta}{1-\cos\theta}=\frac{2\sin\frac{\theta}{2}\cos\frac{\theta}{2}}{2\sin^2\frac{\theta}{2}}=\cot\frac{\theta}{2}
\frac{\text{d}^2\text{y}}{\text{dx}^2}=-\frac{1}{2}\cdot\text{cosec}^2\frac{\theta}{2}\cdot\frac{\text{d}\theta}{\text{dx}}=-\frac{\text{cosec}^2\frac{\theta}{2}}{2\text{a}(1-\cos\theta)}
\frac{\text{d}^2\text{y}}{\text{dx}^2}\Big]_{\theta=\frac{\pi}{3}}=-\frac{1}{2}\times\frac{4}{\text{a}\Big(1-\frac{1}{2}\Big)}=-\frac{4}{\text{a}}
Q391. Differentiate the functions given in Exercise: 4 Marks
(\sin\text{x})^{\text{x}}+\sin^{-1}\sqrt{\text{x}}

Ans: Let \text{y}=(\sin\text{x})^{\text{x}}+\sin^{-1}\sqrt{\text{x}}=\text{u}+\text{v }\text{ where u}=(\sin\text{x})^{\text{x}}\text{and v}\sin^{-1}\sqrt{\text{x}}


\therefore\ \frac{\text{dy}}{\text{dx}}=\frac{\text{du}}{\text{dx}}+\frac{\text{dv}}{\text{dx}}\ \dots\text{(i)}
Now \text{u}=(\sin\text{x})^\text{x}\ \Rightarrow\ \log\text{u}=\log(\sin\text{x})^\text{x}=\text{x}\log(\sin\text{x})
\Rightarrow\ \frac{\text{d}}{\text{dx}}\log\text{u}=\frac{\text{d}}{\text{dx}}[\text{x}\log(\sin\text{x}] \Rightarrow\ \frac{1}{\text{u}}\frac{\text{du}}{\text{dx}}=\text{x}\frac{\text{d}}
{\text{dx}}[\log(\sin\text{x})+\log(\sin\text{x)}\frac{\text{d}}{\text{dx}}\text{x}
\Rightarrow\ \frac{1}{\text{u}}\frac{\text{du}}{\text{dx}}=\text{x}\frac{1}{\sin\text{x}}\frac{\text{d}}{\text{dx}}\sin\text{x}+\log(\sin\text{x}).1
\Rightarrow\ \frac{1}{\text{u}}\frac{\text{du}}{\text{dx}}=\text{x}\frac{1}{\sin\text{x}}\cos\text{x}+\log(\sin)=\text{x}\cot\text{x}+\log\sin\text{x}
\Rightarrow\ \frac{\text{du}}{\text{dx}}=\text{u}[\text{x}\cot\text{x}+\log\sin\text{x}] \Rightarrow\ \frac{\text{du}}{\text{dx}}=(\sin\text{x})^\text{x}[\text{x}\cot\text{x}+\log\sin\text{x}]\
\dots\text{(ii)}
Again \text{v}=\sin^{-1}\sqrt{\text{x}}\ \Rightarrow\ \log\text{v}=\log\sin^{-1}\sqrt{\text{x}}
\Rightarrow\ \frac{\text{dv}}{\text{dx}}=\frac{1}{\sqrt{1-(\sqrt{\text{x})^2}}}\frac{\text{d}}{\text{dx}}\sqrt{\text{x}}\ \Big[\because\frac{\text{d}}{\text{dx}}\sin^{-1}\text{f(x)}=\frac{1}
{\sqrt{1-(\text{f(x))}^2}}\frac{\text{d}}{\text{dx}}\text{f(x)}\Big]
\Rightarrow\ \frac{\text{dv}}{\text{dx}}=\frac{1}{\sqrt{1-\text{x}}}\frac{1}{2\sqrt{\text{x}}}=\frac{1}{2\sqrt{\text{x}}\sqrt{1-\text{x}}}=\frac{1}{2\sqrt{\text{x}-\text{x}^2}}\ \dots\text{(iii)}
Putting the values from eq. (ii) and (iii) in eq. (i),
\frac{\text{dv}}{\text{dx}}=(\sin\text{x})^\text{x}[\text{x}\cot\text{x}+\log\sin\text{x}]+\frac{1}{2\sqrt{\text{x}-\text{x}^2}}
Q392. If y = \frac{\sin^{-1}\text{x}}{\sqrt{\text{1 - x}^{2}}} show that 4 Marks
(1-\text{x}^{2})\frac{\text{d}^{2}\text{y}}{\text{dx}}-\text{3x}\frac{\text{dy}}{\text{dx}}-\text{y}=0.

Ans: \sqrt{\text{1 - x}^{2}}\text{ y}=\sin^{-1}\text{x}


\sqrt{\text{1 - x}^{2}}\text{ }\frac{\text{dy}}{\text{dx}}-\frac{\text{xy}}{\sqrt{\text{1 - x}^{2}}}=\frac{1}{\sqrt{\text{1 - x}^{2}}}
\Rightarrow\text{(1 - x)}^{2}\text{ }\frac{\text{dy}}{\text{dx}}-\text{xy}=1
\Rightarrow\text{(1 - x)}^{2}\text{ }\frac{\text{d}^{2}\text{y}}{\text{dx}^{2}}-\text{ 2x }\frac{\text{dy}}{\text{dx}}-\text{ x }\frac{\text{dy}}{\text{dx}}-\text{y}=0
\Rightarrow\text{(1 - x)}^{2}\text{ }\frac{\text{d}^{2}\text{y}}{\text{dx}^{2}}-\text{ 3x }\frac{\text{dy}}{\text{dx}}-\text{ y }=0.
Q393. Differentiate the following functions with respect to x: 4 Marks
3\text{e}^{-3\text{x}}\log(1+\text{x})

Ans: Consider \text{y}=3\text{e}^{-3\text{x}}\log(1+\text{x})


Differentiating it with respect to x and applying the chain and product rule, we get
\frac{\text{dy}}{\text{dx}}=3\frac{\text{d}}{\text{dx}}\big[3\text{e}^{-3\text{x}}\log(1+\text{x})\big]
\frac{\text{dy}}{\text{dt}}=3\Big(\text{e}^{-3\text{x}}\frac{1}{1+\text{x}}+\log(1+\text{x})\big(-3\text{e}^{-3\text{x}}\big)\Big)
=3\Big(\frac{\text{e}^{-3\text{x}}}{1+\text{x}}-3\log(1+\text{x})\Big)
The solution is,
=3\text{e}^{-3\text{e}}\Big(\frac{1}{1+\text{x}}-3\log(1-\text{x})\Big)
Q394. If \text{y}=\text{e}^{\text{x}}\cos\text{x}, Prvoe that \frac{\text{dy}}{\text{dx}}=\sqrt{2}\text{e}^\text{x}.\cos\Big(\text{x}+\frac{\pi}{4}\Big) 4 Marks

https://bls.smartstudies.co.in/#/exam/pdf-preview/c59cb220-8e86-4716-9ff7-82aec16b1ade/1 86/158
5/26/24, 6:19 PM Exam Automation
Ans: Given, \text{y}=\text{e}^{\text{x}}\cos\text{x}
Differentiating with respect to x,
\frac{\text{dy}}{\text{dx}}=\frac{\text{d}}{\text{dx}}\big(\text{e}^\text{x}\cos\text{x}\big)
=\text{e}^\text{x}\frac{\text{d}}{\text{dx}}\cos\text{x}+\cos\text{x}\frac{\text{d}}{\text{dx}}\text{e}^{\text{x}} [Using product rule]
=\text{e}^\text{e}(-\sin\text{x})+\text{e}^\text{x}\cos\text{x}
=\text{e}^\text{x}(\cos\text{x}-\sin\text{x})
=\sqrt{2}\text{e}^\text{x}\Big(\frac{\cos\text{x}}{\sqrt{2}}-\frac{\sin\text{x}}{\sqrt{2}}\Big) \big[Multiplying and dividing by \sqrt{2}\big]
=\sqrt{2}\text{e}^\text{x}\Big(\cos\frac{\pi}{4}\cos\text{x}-\sin\frac{\pi}{4}\sin\text{x}\Big)
\frac{\text{dy}}{\text{dx}}=\sqrt{2}\text{e}^\text{x}\cos\Big(\text{x}+\frac{\pi}{4}\Big)
Q395. Discuss the continuity of the following functions: 4 Marks
1. \text{f(x)}=\sin\text{x}+\cos\text{x}
2. \text{f(x)}=\sin\text{x}-\cos\text{x}
3. \text{f(x)}=\sin\text{x}\cos\text{x}

Ans: It is know that if g and h are two continuous functions, then


g + h, g - h, and g, h are also continuous.
It has to proved first that \text{g(x)}=\sin\text{x} and \text{h(x)}=\cos\text{x} are continuous functions.
Let \text{g(x)}=\sin\text{x}
It is evident that \text{g(x)}=\sin\text{x} is defined for every real number.
Let c be a real number. Put x = c + h
If x → c, then h → 0
\text{g(c)}=\sin\text{c}
\lim\limits_{{\text{x}}\rightarrow\text{c}}\text{g(x)}=\lim\limits_{{\text{x}}\rightarrow\text{c}}\sin\text{x}
=\lim\limits_{\text{h}\rightarrow0}\sin(\text{c}+\text{h})
=\lim\limits_{\text{h}\rightarrow0}\big[\sin\text{c}\cos\text{h}+\cos\text{c}\sin\text{h}\big]
=\lim\limits_{\text{h}\rightarrow0}(\sin\text{c}\cos\text{h})+\lim\limits_{\text{h}\rightarrow0}(\cos\text{c}\sin\text{h})
=\sin\text{c}\cos0+\cos\text{c}\sin0
=\sin\text{c}+0
=\sin\text{c}
\therefore\ \lim\limits_{{\text{x}}\rightarrow\text{c}}\text{g(x)}=\text{g(c)}
Therefore, g is a continuous function.
Let \text{h(x)}=\cos\text{x}
It is evident that \text{h(x)}=\cos\text{x} is defined for every real number.
Let c be a real number. Put x = c + h
If x → c, then h → 0
\text{h(c)}=\cos\text{c}
\lim\limits_{{\text{x}}\rightarrow\text{c}}\text{h(x)}=\lim\limits_{{\text{x}}\rightarrow\text{c}}\cos\text{x}
=\lim\limits_{{\text{h}}\rightarrow0}\cos(\text{c}+\text{h})
=\lim\limits_{{\text{h}}\rightarrow0}\big[\cos\text{c}\cos\text{h}-\sin\text{c}\sin\text{h}\big]
=\lim\limits_{{\text{h}}\rightarrow0}\cos\text{c}\cos\text{h}-\lim\limits_{{\text{h}}\rightarrow0}\sin\text{c}\sin\text{h}
=\cos\text{c}\cos0-\sin\text{c}\sin0
=\cos\text{c}\times1-\sin\text{c}\times0
=\cos\text{c}
\therefore\ \lim\limits_{{\text{x}}\rightarrow\text{c}}\text{h(x)}=\text{h(c)}
Therefore, h is a continuous function.
Therefore, it can be concluded that
1. \text{f(x)}=\text{g(x)}+\text{h(x)}=\sin\text{x}+\cos\text{x} is a continuous function.
2. \text{f(x)}=\text{g(x)}-\text{h(x)}=\sin\text{x}-\cos\text{x} is a continuous function.
3. \text{f(x)}=\text{g(x)}\times\text{h(x)}=\sin\text{x}\times\cos\text{x} is a continuous function.
Q396. Find the interval in which the function f given by 4 Marks
\text{f(x)}=\sin2\text{x}+\cos2\text{x},0\leq\text{x}\leq\pi is strictly decreasing.

Ans: \text{f(x)}=\sin2\text{x}+\cos2\text{x}
\text{f}'(\text{x})=2\cos2\text{x}-2\sin2\text{x}
\text{f}'(\text{x})=0
\Rightarrow2\cos2\text{x}-2\sin2\text{x}=0
\Rightarrow\text{x}=\frac{\pi}{8},\frac{5\pi}{8}
Possible interval are \Big(0,\frac{\pi}{8}\Big),\Big(\frac{\pi}{8},\frac{5\pi}{8}\Big),\Big(\frac{5\pi}{8},2\pi\Big)
\text{f}'(\text{x})<0 in \Big(\frac{\pi}{8},\frac{5\pi}{8}\Big)
\Rightarrow\text{f(x)} is decreasing in \Big(\frac{\pi}{8},\frac{5\pi}{8}\Big)
Q397. In the following, determine the values of constants involved in the definition so that the given function is continuous: 4 Marks
\text{f(x)}=\begin{cases}5,&\text{if }\text{ x}\leq2\\\text{ax}+\text{b},&\text{if }2<\text{x}<10\\21,&\text{if }\text{ x}\geq10\end{cases}

Ans: Given,
\text{f(x)}=\begin{cases}5,&\text{if }\text{ x}\leq2\\\text{ax}+\text{b},&\text{if }2<\text{x}<10\\21,&\text{if }\text{ x}\geq10\end{cases}
If f(x) is continuous x = 2 and 10, then
\lim\limits_{\text{x}\rightarrow2^-}\text{f(x)}=\lim\limits_{\text{x}\rightarrow2^+}\text{f(x)} and \lim\limits_{\text{x}\rightarrow10^-}\text{f(x)}=\lim\limits_{\text{x}\rightarrow10^+}\text{f(x)}
\Rightarrow\lim_\limits{\text{h}\rightarrow0}\text{f}(2-\text{h})=\lim_\limits{\text{h}\rightarrow0}\text{f}(2+\text{h}) and \lim_\limits{\text{h}\rightarrow0}\text{f}(10-
\text{h})=\lim_\limits{\text{h}\rightarrow0}\text{f}(10+\text{h})
\Rightarrow\lim_\limits{\text{h}\rightarrow0}=\lim_\limits{\text{h}\rightarrow0}\big[\text{a}(2+\text{h})+\text{b}\big] and \lim_\limits{\text{h}\rightarrow0}\big[\text{a}(10-
\text{h})+\text{b}\big]=\lim_\limits{\text{h}\rightarrow0}(21)
\Rightarrow5=2\text{a}+\text{b}\ ....(\text{i}) and 10\text{a}+\text{b}=21\ ....(\text{ii})
On solving eqs. (i) and (ii) we get
\text{a}=2 and \text{b}=1
Q398. If \text{y}=\text{x}\sin\text{y}, prove that \frac{\text{dy}}{\text{dx}}=\frac{\text{y}}{\text{x}(1-\text{x}\cos\text{y})} 4 Marks

Ans: We have, \text{y}=\text{x}\sin\text{y}\ .....(\text{i})


Differentiating with respect to x,
\frac{\text{dy}}{\text{dx}}=\frac{\text{d}}{\text{dx}}(\text{x}\sin\text{y})
\Rightarrow\frac{\text{dy}}{\text{dx}}=\text{x}\frac{\text{d}}{\text{dx}}(\sin\text{y})+\sin\text{y}\frac{\text{d}}{\text{dx}}(\text{x})
\Rightarrow\frac{\text{dy}}{\text{dx}}=\text{x}\cos\text{y}\frac{\text{dy}}{\text{dx}}+\sin\text{y}(1)
\Rightarrow\frac{\text{dy}}{\text{dx}}-\text{x}\cos\text{y}\frac{\text{dy}}{\text{dx}}=\sin\text{y}
\Rightarrow\frac{\text{dy}}{\text{dx}}(1-\text{x}\cos\text{y})=\sin\text{y}
\Rightarrow\frac{\text{dy}}{\text{dx}}=\frac{\sin\text{y}}{(1-\text{x}\cos\text{y})}
\Rightarrow\frac{\text{dy}}{\text{dx}}=\frac{\text{y}}{\text{x}(1-\text{x}\cos\text{y})}\Big[\because\sin\text{y}=\frac{\text{y}}{\text{x}}\Big]
Q399. Using Lagrange's mean value theorem, prove that 4 Marks
(\text{b}-\text{a})\sec^2\text{a}<\tan\text{b}-\tan\text{a}<(\text{b}-\text{a})\sec^2\text{b}
where 0<\text{a}<\text{b}<\frac{\pi}{2}.

https://bls.smartstudies.co.in/#/exam/pdf-preview/c59cb220-8e86-4716-9ff7-82aec16b1ade/1 87/158
5/26/24, 6:19 PM Exam Automation

Ans: Consider the function as


\text{f}(\text{x})=\tan\text{x}, \Big\{\text{x}\in[\text{a},\text{b}]\text{ such that }0<\text{a}<\text{b}<\frac{\pi}{2}\Big\}
We know that \tan\text{x} is continuous and differentiable in \Big(0,\frac{\pi}{2}\Big), so, Lagrange's mean value theorem is applicable on (a, b), so there exist a point c such that,
\text{f}'(\text{c})=\frac{\text{f}(\text{b})-\text{f}(\text{a})}{\text{b}-\text{a}}
\Rightarrow\sec^2\text{c}=\frac{\tan\text{b}-\tan\text{a}}{\text{b}-\text{a}}\ ....(\text{i})
Now,
\text{c}\in(\text{a},\text{b})
\Rightarrow\text{a}<\text{c}<\text{b}
\Rightarrow\sec^2\text{a}<\sec^2\text{c}<\sec^2\text{b}
\Rightarrow\sec^2\text{a}<\Big(\frac{\tan\text{b}-\tan\text{a}}{\text{b}-\text{a}}\Big)<\sec^2\text{b}
Using equation (i),
(\text{b}-\text{a})\sec^2\text{a}<(\tan\text{b}-\tan\text{a})<(\text{b}-\text{a})\sec^2\text{b}
Q400. Differentiate \sin(x^{2} + 1)with respect to xfrom first principle. 4 Marks

Ans: \frac{\triangle y}{\triangle x} = \frac {\sin \bigg[( {x + \triangle x)^{2}}+ 1\bigg] -\sin ( x^{2} + 1)}{\triangle x}
\therefore \frac{dy}{dx} = \lim\limits_{\triangle x \rightarrow 0} \frac{2\cos\bigg[ \frac{( x + \triangle x)^{2} + x^{2}}{2}\bigg] .\sin \bigg[\frac{\triangle x( 2x + \triangle x}{2} \bigg]} {\triangle
x}
= 2.\cos ( x^{2} + 1) . \lim\limits_{\triangle x \rightarrow 0} \frac{\sin\bigg[\frac{(\triangle x+{2 x} +\triangle{x)}}{2}\bigg] \bigg[\frac{(2x +\triangle x)}{2}\bigg]}{\triangle x.\bigg(\frac{2x +
\triangle x}{2}\bigg)}
= \text{2 x} .\cos ( x^{2} + 1)

Q401. In the following, determine the values of constants involved in the definition so that the given function is continuous: 4 Marks
\text{f(x)}=\begin{cases}4,&\text{if }\text{ x}\leq-1\\\text{ax}^2+\text{b},&\text{if }-1<\text{ x}<0\\\cos\text{x},&\text{if }\text{ x}\geq0\end{cases}

Ans: It is given that the function is continuous


At x = -1
f(-1) = 4
\text{RHL}=\lim_\limits{\text{x}\rightarrow-1^+}\text{f(x)}=\lim_\limits{\text{h}\rightarrow0}\text{a}(-1+\text{h})^2+\text{b}=\text{a}+\text{b}
Since, f(x) is continuous at x = -1
\therefore a + b = 4 ...(i)
Now, at x = 0
\text{f}(0)=\cos0^\circ=1
\text{LHL}=\lim_\limits{\text{h}\rightarrow0^-}\text{f(x)}=\lim_\limits{\text{h}\rightarrow0}\text{f}(0-\text{h})=\lim_\limits{\text{h}\rightarrow0}\text{a}(-\text{h})^2+\text{b}=\text{b}
\therefore f(0) = LHL
b=1
\therefore from (i)
a=3
Thus, a = 3, b = 1
Q402. Prove that the function f(x) = xn is continuous at x = n, where n is a positive integer. 4 Marks

Ans: Here f(x) = xn, Where is a possitive integer.


^{\ \ \text{Lt}}_{\text{x}\rightarrow\text{n}}\text{f(x)} = ^{\ \ \text{Lt}}_{\text{x}\rightarrow\text{n}}(\text{x}^\text{n}) = \text{n}^\text{n}
Now f is defined at x = n
and f(n) = nn
\therefore\ ^{\ \ \text{Lt}}_{\text{x}\rightarrow\text{n}}\text{f(x)} = \text{f(n)}
\therefore f is continous at x = n.
Q403. Find the points of discontinuity, if any of the following function: 4 Marks
\text{f(x)}=\begin{cases}|\text{x}|+3,&\text{if }\text{ x}\geq-3\\-2\text{x},&\text{if }-3<\text{ x}<3\\6\text{x}+2,&\text{if }\text{ x}>3\end{cases}

Ans: When x < -3,


f(x) = |x| + 3
We know that |x| is continuous for x < -3
\therefore |x| + 3 is continuous for x < -3
When x > 3
f(x) = 6x + 2 which is a polynomial of degree 1, so f(x) = 6x + 2 is continuous for x > 3
When -3 < x < 3
f(x) = -2x which is again a polynomial so, it is continuous for -3 < x < 3
Now, consider the point x = -3
\text{LHL}=\lim_\limits{\text{x}\rightarrow3^-}\text{f(x)}=\lim_\limits{\text{h}\rightarrow0}\text{f}(-3-\text{h})
=\lim_\limits{\text{h}\rightarrow0}|-3-\text{h}|+3=\lim_\limits{\text{h}\rightarrow0}|3+\text{h}|+3=6
\text{RHL}=\lim_\limits{\text{x}\rightarrow3^+}\text{f(x)}=\lim_\limits{\text{h}\rightarrow0}\text{f}(-3+\text{h})
=\lim_\limits{\text{h}\rightarrow0}-2(-3+\text{h})=6
\text{f}(-3)=|-3|+3=6
Thus, LHL = RHL = f(-3) = 6
So, the function is continuous at x = 3
Now, consider the point x = 3
\text{LHL}=\lim_\limits{\text{x}\rightarrow3^-}\text{f(x)}=\lim_\limits{\text{h}\rightarrow0}\text{f}(-3-\text{h})
=\lim_\limits{\text{h}\rightarrow0}-2(3-\text{h})=-6
\text{RHL}=\lim_\limits{\text{x}\rightarrow3^+}\text{f(x)}=\lim_\limits{\text{h}\rightarrow0}\text{f}(3+\text{h})
=\lim_\limits{\text{h}\rightarrow0}6(3+\text{h})+2=20
Thus, \text{LHL}\neq\text{RHL}
Hence, f(x) is discontinuous at x = 3
Q404. Find: \int\frac{\sin2\text{x}}{(\text{sin}^2\text{x+1}(\sin^2\text{x}+3)}\text{dx} 4 Marks

Ans: Let \text{I} =\int\frac{\sin2\text{x}}{(\text{sin}^2\text{x+1}(\sin^2\text{x}+3)}\text{dx}


\Rightarrow\text{I}=\int\frac{2\sin\text{x}.\cos\text{x}}{(\ \sin^2\text{x+1)}(\sin^2\text{x+3)}}\text{dx}
Let \sin^2\text{x}+3=\text{t}
\Rightarrow2\sin\text{x}. \cos\text{xdx}=\text{dt}
Therefore,
\text{I}=\int\frac{\text{dt}}{(\text{t}-2)\text{t}}
\Rightarrow \text{I} =\frac{1}{2}\int \Big(\frac{1}{\text{t}-2}-\frac{1}{\text{t}}\Big)\text{dt}
\Rightarrow\text{I}=\frac{1}{2}\big[\text{ln(}\text{t}-2)-\text{ln}\big]+\text{c}
\Rightarrow \text{I}= \frac{1}{2} \text{ln}\Big(\frac{\text{t}-2}{\text{t}}\Big)+\text{c}
\Rightarrow \text{I}=\text{}\text{ln}\sqrt{\frac{\text{t}-2}{\text{t}}}+\text{c}
\Rightarrow \text{I}= \text{ln}\sqrt{\frac{\sin^2\text{x}+1}{\sin^2\text{x}+3}}+\text{c}
Q405. If \lim\limits_{\text{x}\rightarrow{\text{c}}}\frac{\text{f(x)}-\text{f(c)}}{\text{x}-\text{c}} exists finitely, write the value of \lim\limits_{\text{x}\rightarrow{\text{c}}}\text{f(x)}. 4 Marks

https://bls.smartstudies.co.in/#/exam/pdf-preview/c59cb220-8e86-4716-9ff7-82aec16b1ade/1 88/158
5/26/24, 6:19 PM Exam Automation
Ans: LHL = f(1) = RHL
So, f(x) is continuous at x = 1
Now,
(LHL at x = 1) =\lim\limits_{\text{x}\rightarrow1^{-}}\frac{\text{f(x)}-\text{f}(1)}{\text{x}-1}
=\lim\limits_{\text{h}\rightarrow0}\frac{\text{f}(1-\text{h})-\text{f}-(1)}{(1-\text{h})-1}
=\lim\limits_{\text{h}\rightarrow0}\frac{1-1}{-\text{h}}
= Not defined
(RHL at x = 1) =\lim\limits_{\text{x}\rightarrow1^{+}}\frac{\text{f(x)}-\text{f}(1)}{\text{x}-1}
=\lim\limits_{\text{h}\rightarrow0}\frac{\text{f}(1+\text{h})-\text{f}-(1)}{(1+\text{h})-1}
=\lim\limits_{\text{h}\rightarrow0}\frac{2(1+\text{h})-1-1}{\text{h}}
=\lim\limits_{\text{h}\rightarrow0}\frac{2\text{h}}{\text{h}}
=1
(LHL at x = 1) \neq (RHL at x = 1)
\therefore f(x) is continuous but not differentiable at x = 0 and 1.
\lim\limits_{\text{x}\rightarrow{\text{c}}}\frac{\text{f(x)}-\text{f(c)}}{\text{x}-\text{c}} exists finitely
So,
\text{f}'(\text{c})=\lim\limits_{\text{x}\rightarrow{\text{c}}}\frac{\text{f(x)}-\text{f(c)}}{\text{x}-\text{c}}
\text{f}'(\text{c})\lim\limits_{\text{x}\rightarrow{\text{c}}}(\text{x}-\text{c})=\lim\limits_{\text{x}\rightarrow{\text{c}}}(\text{x})-\text{f(c)}
\text{f}'(\text{c})(\text{c}-\text{c})=\lim\limits_{\text{x}\rightarrow{\text{c}}}(\text{x})-\text{f(c)}
0=\lim\limits_{\text{x}\rightarrow{\text{c}}}(\text{x})-\text{f(c)}
\lim\limits_{\text{x}\rightarrow{\text{c}}}\text{f(x})=\text{f(c)}
Q406. Write the points where f(x) = |loge x| is not differentiable. 4 Marks

Ans: Given: \text{f(x)}=|\log_\text{e}\text{x}|=\begin{cases}-\log_\text{e}\text{x}, & 0<\text{x}<1\\\log_\text{e}\text{x}, & \text{x}\geq1\end{cases}


Clearly f(x) is differentiable for all x > 1 and for all x < 1. So, we have to check the differentiability at x = 1.
(LHL at x = 1)
\lim_\limits{\text{x}\rightarrow1^{-}}\frac{\text{f(x)}-\text{f}(1)}{\text{x}-1}
=\lim_\limits{\text{x}\rightarrow1^{-}}\frac{-\log\text{x}-\log1}{\text{x}-1}
=-\lim_\limits{\text{x}\rightarrow1^{-}}\frac{\log\text{x}}{\text{x}-1}
=-\lim_\limits{\text{h}\rightarrow0^{-}}\frac{\log(1-\text{h})}{1-\text{h}-1}
=-\lim_\limits{\text{h}\rightarrow0^{-}}\frac{\log(1-\text{h})}{-\text{h}}
=-1
(RHL at x = 1)
\lim_\limits{\text{x}\rightarrow1^{+}}\frac{\text{f(x)}-\text{f}(1)}{\text{x}-1}
=\lim_\limits{\text{x}\rightarrow1^{+}}\frac{-\log\text{x}-\log1}{\text{x}-1}
=\lim_\limits{\text{x}\rightarrow1^{+}}\frac{\log\text{x}}{\text{x}-1}
=-\lim_\limits{\text{h}\rightarrow0^{+}}\frac{\log(1+\text{h})}{1+\text{h}-1}
=\lim_\limits{\text{h}\rightarrow0^{-}}\frac{\log(1+\text{h})}{\text{h}}
=1
Thus, (LHL at x = 1) \neq (RHL at x = 1)
So, f(x) is not differentiable at x = 1.
Q407. Verify Rolle's theorem for the function f(x) = x2 - 4x + 3 on [1, 3]. 4 Marks

Ans: f (x) being a polynomial is continuous in [1, 3] and differentiable in (1, 3).
Also f (a) = f (1) = 0 = f (b) = f (3) \therefore Roll s Theorem is applicable
\Rightarrow\text{f'(c)}=2\text{c} - 4 = 0 \Rightarrow\text{c}=2\in(1, 3)
Hence Rolle's theorem is verified.
Q408. If the value of c prescribed bye Lagrange's mean value theorem for the function 4 Marks
\text{f}(\text{x})=\sqrt{\text{x}^2-4} defined on [2, 3].

Ans: Here,
\text{f}(\text{x})=\sqrt{\text{x}^2-4} defined on [2, 3].
We have to find c prescribed by Lagrange's mean value theorem, so
\text{f}'(\text{c})=\frac{\text{f}(\text{b})-\text{f}(\text{a})}{\text{b}-\text{a}}
\Rightarrow\frac{2\text{c}}{2\sqrt{\text{c}^2-4}}=\frac{(\sqrt{9-4})-(\sqrt{4-4})}{3-2}
\Rightarrow\frac{\text{c}}{\sqrt{\text{c}^2-4}}=\frac{\sqrt5-0}{1}
\Rightarrow\frac{\text{c}}{\sqrt{\text{c}^2-4}}=\sqrt5
Squaring both sides,
⇒ c2 = (c2 - 4)5
⇒ 5c2 - c2 = 20
⇒ 4c2 = 20
⇒ c2 = 5
\Rightarrow\text{c}=\pm\sqrt5
but \text{c}=\sqrt5\text{ as }\sqrt5\in(2,3).
Q409. Verify Rolle's theorem for the following function on the indicated intervals 4 Marks
f(x) = x2 -4x + 3 on [1, 3]

Ans: The given function is f(x) = x2 -4x + 3


f, being a pollynomial function, is continuous in [1, 4] and is differentiable in (1, 4) whose derivative is 2x - 4.
f(1) = 12 - 4 × 1 + 3 = 0
f(4) = 42 - 4 × 4 + 3 = 3
\therefore\ \frac{\text{f}(\text{b})-\text{f}(\text{a})}{\text{b}-\text{a}}=\frac{\text{f}(4)-\text{f}(1)}{4-1}=\frac{3-(0)}{3}=\frac{3}{3}=1
Mean Value Theorem states that there is a point \text{c}\in(1,4) such that f'(c) = 1
f'(c) = 1
⇒ 2c - 4 = 1
\Rightarrow\text{c}=\frac{5}{2}, where \text{c}=\frac{5}{2}\in(1,4)
Hence, Mean Value Theorem is verified for the given function.
Q410. If \text{y}=\cos^{-1}(2\text{x})+2\cos^{-1}\sqrt{1-4\text{x}^2}, -\frac{1}{2}<\text{x}<0, find \frac{\text{dy}}{\text{dx}}. 4 Marks

Ans: Here, \text{y}=\cos^{-1}(2\text{x})+2\cos^{-1}\sqrt{1-4\text{x}^2}


Put 2\text{x}=\cos\theta, \text{So},
\text{y}=\cos^{-1}(\cos\theta)+2\cos^{-1}\sqrt{1-\cos^2\theta}
=\cos^2(\cos\theta)+2\cos^{-1}(\sin\theta)
\text{y}=\cos^{-1}(\cos\theta)+2\cos^{-1}\Big(\cos\Big(\frac{\pi}{2}-\theta\Big)\Big)\ .....(\text{i})
Now, -\frac{1}{2}<\text{x}<0
\Rightarrow -1<2\text{x}<0
\Rightarrow -1<\cos\theta<0
\Rightarrow \frac{\pi}{2}<\theta<\pi

https://bls.smartstudies.co.in/#/exam/pdf-preview/c59cb220-8e86-4716-9ff7-82aec16b1ade/1 89/158
5/26/24, 6:19 PM Exam Automation
And
\Rightarrow -\frac{\pi}{2}>-\theta>-\pi
\Rightarrow \Big(\frac{\pi}{2}-\frac{\pi}{2}\Big)>\Big(\frac{\pi}{2}-\theta\Big)>\Big(\frac{\pi}{2}-\pi\Big)
\Rightarrow 0>\Big(\frac{\pi}{2}-\theta\Big)>-\frac{\pi}{2}
So, from equation (i),
\text{y}=\theta+2\Big[-\Big(\frac{\pi}{2}-\theta\Big)\Big]
\begin{bmatrix} \text{Since}, \cos^{-1}\cos(\theta)=\theta, \text{if }\theta\in[0,\pi] \\ \cos^{-1}\cos(\theta)=-\theta, \text{if }\theta\in[-\pi,0] \end{bmatrix}
\text{y}=\theta-2\times\frac{\pi}{2}+2\theta
\text{y}=-\pi+3\theta
\text{y}=-\pi+3\cos^{-1}(2\text{x})\ \big[\text{Since}, 2\text{x}=\cos\theta\big]
Differentiating it with respect to x using chain rule,
\frac{\text{dy}}{\text{dx}}=0+3\Big(\frac{1}{\sqrt{1-(2\text{x})^2}}\Big)\frac{\text{d}}{\text{dx}}(2\text{x})
=\frac{-3}{\sqrt{1-4\text{x}^2}}(2)
\frac{\text{dy}}{\text{dx}}=-\frac{6}{\sqrt{1-4\text{x}^2}}
Q411. If \text{y}=\text{cosec}^{-1}\text{x},\text{x}>1 prove that \text{x}(\text{x}^2-1)\frac{\text{d}^2\text{y}}{\text{dx}^2}+(2\text{x}^2-1)\frac{\text{dy}}{\text{dx}}=0. 4 Marks

Ans: Here,
\text{y}=\text{cosec}^{-1}\text{x}
Differentiating w.r.t.x, we get
\frac{\text{dy}}{\text{dx}}=\frac{-1}{\text{x}\sqrt{{}\text{x}^2-1}}
Differentiating w.r.t.x, we get
\frac{\text{d}^2\text{y}}{\text{dx}^2}=\frac{\sqrt{\text{x}^2-1}+\frac{\text{x}^2}{\sqrt{\text{x}^2-1}}}{\text{x}^2(\text{x}^2-1)}
\Rightarrow\frac{\text{d}^2\text{y}}{\text{dx}^2}=\frac{\text{x}^2-1+\text{x}^2}{\text{x}^2(\text{x}^2-1)\sqrt{\text{x}^2-1}}
\Rightarrow\frac{\text{d}^2\text{y}}{\text{dx}^2}=\frac{2\text{x}^2-1}{\text{x}^2(\text{x}^2-1)\sqrt{\text{x}^2-1}}
\Rightarrow\frac{\text{d}^2\text{y}}{\text{dx}^2}=\frac{2\text{x}}{(\text{x}^2-1)\sqrt{\text{x}^2-1}}-\frac{1}{\text{x}^2(\text{x}^2-1)\sqrt{\text{x}^2-1}}
\Rightarrow(\text{x}^2-1)\frac{\text{d}^2\text{y}}{\text{dx}^2}=\frac{2\text{x}}{\sqrt{\text{x}^2-1}}-\frac{1}{\sqrt{\text{x}^2-1}}
\Rightarrow(\text{x}^2-1)\frac{\text{d}^2\text{y}}{\text{dx}^2}=-2\text{x}\frac{\text{dy}}{\text{dx}}+\frac{1}{\text{x}}\frac{\text{dy}}{\text{dx}}
\Rightarrow(\text{x}^2-1)\frac{\text{d}^2\text{y}}{\text{dx}^2}=-2\text{x}\frac{\text{dy}}{\text{dx}}+\frac{1}{\text{x}}\frac{\text{dy}}{\text{dx}}
\Rightarrow(\text{x}^2-1)\frac{\text{d}^2\text{y}}{\text{dx}^2}=-(2\text{x}^2-1)\frac{\text{dy}}{\text{dx}}
\Rightarrow(\text{x}^2-1)\frac{\text{d}^2\text{y}}{\text{dx}^2}=+(2\text{x}^2-1)\frac{\text{dy}}{\text{dx}}=0
Hence proved
Q412. Differentiate \tan^{-1}\Big(\frac{2\text{x}}{1-\text{x}^2}\Big) with respect to \cos^{-1}\Big(\frac{1-\text{x}^2}{1+\text{x}^2}\Big), if 0 < x < 1. 4 Marks

Ans: Let \text{u}=\tan^{-1}\Big(\frac{2\text{x}}{1-\text{x}^2}\Big)


Put \text{x}=\tan\theta,\text{so}
\text{u}=\tan^{-1}\Big(\frac{2\tan\theta}{1-\tan^2\theta}\Big)
\text{u}=\tan^{-1}(\tan2\theta)\ .....(\text{i})
Let \text{v}=\cos^{-1}\Big(\frac{1-\text{x}^2}{1+\text{x}^2}\Big)
=\cos^{-1}\Big(\frac{1-\tan^2\theta}{1+\tan^2\theta}\Big)
\text{v}=\cos^{-1}(\cos2\theta)\ .....(\text{ii})
Hrer, 0 < x < 1
\Rightarrow0< \tan\theta<1
\Rightarrow0<\theta<\frac{\pi}{4}
So, from equation (i),
\text{u}=2\theta\Big[\text{Since,}\tan^{-1}(\tan\theta)=\theta,\text{if }\theta\in\Big[-\frac{\pi}{2},\frac{\pi}{2}\Big]\Big]
\text{u}=2\tan^{-1}\text{x }[\text{Since,x}=\tan\theta]
Differentiating it with respect to x,
\frac{\text{du}}{\text{dx}}=\frac{2}{1+\text{x}^2}\ .....(\text{iii})
From equation (ii),
\text{v}=\theta \big[\text{Since,} \cos^{-1}(\cos\theta)=\theta,\text{if }\theta\epsilon[0,\pi]\big]
\text{v}=2\tan^{-1}\text{x }[\text{Since},\text{x}=\tan\theta]
Differentiating it with respect to x,
\frac{\text{dv}}{\text{dx}}=\frac{2}{1+\text{x}^2}\ .....(\text{iv})
Dividing equation (iii) by (iv)
\frac{\frac{\text{}du}{\text{dx}}}{\frac{\text{dv}}{\text{dx}}}=\frac{2}{1+\text{x}^2}\times\frac{1+\text{x}^2}{2}
\frac{\text{du}}{\text{dv}}=1
Q413. Find the points of discontinuity, if any of the following function: 4 Marks
\text{f(x)}=\begin{cases}\text{x}^{10}-1,&\text{if }\text{ x}\leq1\\\text{x}^2,&\text{if }\text{ x}>1\end{cases}

Ans: Given, \text{f(x)}=\begin{cases}\text{x}^{10}-1,&\text{if }\text{ x}\leq1\\\text{x}^2,&\text{if }\text{ x}>1\end{cases}


The given function f is defind at all the points of the relline.
Let c be a point on the real line.
Case I:
If c < 1, then \text{f(c)}=\text{c}^{10}-1 and \lim_\limits{\text{x}\rightarrow\text{c}}\text{f(x)}=\lim_\limits{\text{x}\rightarrow\text{c}}(\text{x}^{10}-1)=\text{c}^{10}-1
\therefore\lim_\limits{\text{x}\rightarrow\text{c}}\text{f(x)}=\text{f(c)}
Therefore, f is continuous at all points x such, that x < 1
Case II:
If c = 1, then the left hand limit of at x = 1 is,
\lim_\limits{\text{x}\rightarrow1^-}\text{f(x)}=\lim_\limits{\text{x}\rightarrow1^+}(\text{x}^{10}-1)=1^{10}-1=1-1=0
The right hand limit of f at x = 1 is,
\lim_\limits{\text{x}\rightarrow1^+}\text{f(x)}=\lim_\limits{\text{x}\rightarrow1^+}(\text{x}^2)=1^1=1
It is observed that the left and right hand limit of at x = 1 do not coincide.
Therefore, f is not continuouse at x = 1
Case III:
If c > 1, then f(c) = c2
\lim_\limits{\text{x}\rightarrow\text{c}}\text{f(x)}=\lim_\limits{\text{x}\rightarrow\text{c}}(\text{x}^2)=\text{c}^2
\therefore\ \lim_\limits{\text{x}\rightarrow\text{c}}\text{f(x)}=\text{f(c)}
Therefore, f is continuous at all points x, such that x > 1
Thus, from the above observation, if can be concluded that x = 1 is the only point of discontinuity of y.
Q414. If \text{x}=\text{a}(\cos\text{t}+\text{t}\sin\text{t})\ \text{and}\ \text{y}=\text{a}(\sin\text{t}-\text{t}\cos\text{t}), find the value of \frac{\text{d}^2\text{y}}{\text{dx}^2}\ \text{at}\ 4 Marks
\text{t}=\frac{\pi}{4}.

Ans: We have,
\text{x}=\text{a}(\cos\text{t}+\text{t}\sin\text{t})\ \text{and}\ \text{y}=\text{a}(\sin\text{t}-\text{t}\cos\text{t}),
On differentiating with respect to t, we get
\frac{\text{dx}}{\text{dt}}=\frac{\text{d}}{\text{dt}}[\text{a}(\cos\text{t}+\text{t}\sin\text{t})]=-\text{a}\sin\text{t}+\text{a}\sin\text{t}+\text{at}\cos\text{t}=\text{at}\cos\text{at}
and
\frac{\text{dy}}{\text{dt}}=\frac{\text{d}}{\text{dt}}[\text{a}(\sin\text{t}-\text{t}\cos\text{t})]=\text{a}\cos\text{t}-\text{a}\cos\text{t}+\text{at}\sin\text{t}=\text{at}\sin\text{t}

https://bls.smartstudies.co.in/#/exam/pdf-preview/c59cb220-8e86-4716-9ff7-82aec16b1ade/1 90/158
5/26/24, 6:19 PM Exam Automation
Now, \Big(\frac{\text{dy}}{\text{dx}}\Big)=\frac{\frac{\text{dy}}{\text{dt}}}{\frac{\text{dx}}{\text{dt}}}=\frac{\text{at}\sin\text{t}}{\text{at}\cos\text{t}}=\tan\text{t}
\frac{\text{d}^2\text{y}}{\text{dx}^2}=\frac{\text{d}}{\text{dx}}\Big(\frac{\text{dy}}{\text{dx}}\Big)=\frac{\text{d}}{\text{dx}}(\tan\text{t})
=\frac{\text{d}}{\text{dt}}(\tan\text{t})\times\frac{\text{dt}}{\text{dx}}=\sec^2\text{t}\times\frac{1}{\text{at}\cos\text{t}}
=\frac{1}{\text{at}\cos^3\text{t}}
\Big(\frac{\text{d}^2\text{y}}{\text{dx}^2}\Big)_{\text{t}=\frac{\pi}{4}}=\frac{1}{\text{a}\Big(\frac{\pi}{4}\Big)\cos^3\Big(\frac{\pi}{4}\Big)}=\frac{8\sqrt{2}}{\text{a}\pi}
Hence, \text{at}\ \text{t}=\frac{\pi}{4},\frac{\text{d}^2\text{y}}{\text{dx}^2}=\frac{8\sqrt{2}}{\text{a}\pi}
Q415. Verify Rolle's theorem for the function f ( x) = x^{2} - 5x + 4\text{ on} [ 1, 4]. 4 Marks

Ans: \text{f (x)} = x^{2} - 5x + 4 \text {( a polynomial function)}


\text{(i) the function is continuous on [1,4] }
\text{(ii) It is differentiable on (1,4)}
\text{(iii) f (1)} = \text{f (4)} = 0
\therefore All the conditions of Rolles' Theoremare satisfied.
\therefore \text{f' (c)} = 0 \Rightarrow \text{2 c - 5} = 0 \Rightarrow \text{c} = \frac{5}{2}
\text{As c} = \frac{5}{2} \in (1,4), the Rolle's theorem is verified.

Q416. Differentiate the following functions with respect to x: 4 Marks


\sin^{-1}\Big\{\sqrt{\frac{1-\text{x}}{2}}\Big\},0<\text{x}<1

Ans: Let \text{y}=\sin^{-1}\Big\{\sqrt{\frac{1-\text{x}}{2}}\Big\}


Put \text{x}=\cos2\theta
\text{y}=\sin^{-1}\Big\{\sqrt{\frac{1+\cos2\theta}{2}}\Big\}
=\sin^{-1}\Big\{\sqrt{\frac{2\sin^2\theta}{2}}\Big\}
\text{y}=\sin^{-1}(\sin\theta)\ .....(\text{i})
Here, 0<\text{x}<1
\Rightarrow\ 0<\cos2\theta<1
\Rightarrow\ 0<2\theta<\frac{\pi}{2}
\Rightarrow\ 0<\theta<\frac{\pi}{4}
So, from equation (i),
\text{y}=\theta
\Big[\text{Since, } \sin^{-1}(\sin\theta)=\theta\text{ if }\theta \in\Big[-\frac{\pi}{2},\frac{\pi}{2}\Big]\Big]
\text{y}=\frac{1}{2}\cos^{-1}\text{x}\ \big[\text{Since x}=\cos2\theta\big]
Differentiating it with respect to x,
\frac{\text{dy}}{\text{dx}}=-\frac{1}{2\sqrt{1-\text{x}^2}}
Q417. Differentiate the following functions with respect to x: 4 Marks
\text{e}^{\text{ax}}\sec\text{x}\tan2\text{x}

Ans: Let \text{y}=\text{e}^{\text{ax}}\sec\text{x}\tan2\text{x}


Differentiate it with respect to x,
\frac{\text{dy}}{\text{dx}}=\frac{\text{d}}{\text{dx}}(\text{e}^{\text{ax}}\sec\text{x}\tan2\text{x})
=\text{e}^{\text{ax}}\frac{\text{d}}{\text{dx}}\big\{\sec\text{x}\tan2\text{x}\big\}+\sec\text{x}\tan2\text{x}\frac{\text{d}}{\text{dx}}\big\{\text{e}^{\text{ax}}\big\}
=\text{e}^{\text{ax}}\big[\text{sec}\text{x}\tan\text{x}\tan2\text{x}+2\sec^2 2\text{x}\sec\text{x}\big]+\text{ae}^{\text{ax}}\sec\text{a}\tan^{2\text{x}}
=\text{ae}^{\text{ax}}\sec\text{x}\tan2\text{x}+\text{e}^{\text{ax }}\sec\text{x}\tan\text{x}\tan2\text{x}+2\text{e}^{\text{ax}}\sec\text{x}\sec^2 2\text{x}
=\text{e}^{\text{ax}}\sec\text{x}\big\{\text{a}\tan2\text{x}+\tan\text{x}\tan2\text{x}+2\sec^22\text{x}\big\}
So,
\frac{\text{d}}{\text{dx}}(\text{e}^{\text{ax}}\sec\text{x}\tan2\text{x})=\text{e}^{\text{ax}}\sec\text{x}\big\{\text{a}\tan2\text{x}+\tan\text{x}\tan2\text{x}+2\sec^22\text{x}\big\}
Q418. Find which of the function: 4 Marks
\text{f(x)}=\begin{cases}\frac{\text{x}^2}{2},&\text{if }0\leq\text{x}\leq1\\2\text{x}^2-3\text{x}+\frac{3}{2},&\text{if }1<\text{x}\leq2\end{cases}
at x = 1

Ans: We have, \text{f(x)}=\begin{cases}\frac{\text{x}^2}{2},&\text{if }0\leq\text{x}\leq1\\2\text{x}^2-3\text{x}+\frac{3}{2},&\text{if }1<\text{x}\leq2\end{cases} at x = 1.


At x = 1, \text{L.H.L}=\lim\limits_{\text{h}\rightarrow1^-}\frac{\text{x}^2}{2}=\lim\limits_{\text{h}\rightarrow0}\frac{(1-\text{h})^2}{2}
=\lim\limits_{\text{h}\rightarrow0}\frac{1+\text{h}^2-2\text{h}}{2}=\frac{1}{2}
\text{R.H.L}=\lim\limits_{\text{h}\rightarrow1^+}\Big(2\text{x}^2-3\text{x}+\frac{3}{2}\Big)
=\lim\limits_{\text{h}\rightarrow0}\Big[2(1+\text{h})^2-3(1+\text{h})+\frac{3}{2}\Big]
=\lim\limits_{\text{h}\rightarrow0}\Big(2+2\text{h}^2+4\text{h}-3-3\text{h}+\frac{3}{2}\Big) =-1+\frac{3}{2}=\frac{1}{2}
And \text{f}(1)=\frac{1^2}{2}=\frac{1}{2}
\therefore L.H.L = R.H.L = f(1)
Hence, f(x) is continuous at x = 1.
Q419. Verify Lagrange's mean value theorem for the following function on the indicated intervals. find a point 'c' in the indicated interval as stated by the Lagrange's mean value theorem. 4 Marks
f(x) = x(x + 4)2 on [0,4]

Ans: Here,
f(x) = x(x + 4)2
⇒ f(x) = x(x2 + 16 + 8x)
⇒ f(x) = x3 + 8x2 + 16x
Since f(x) is a polynomial function which is everywhere continuous and differentiable.
Therefore, f(x) is continuous on [0, 4] and derivable on (0, 4)
Thus, both the conditions of Lagrange's theorem is satisfied.
Consequently, there exists some \text{c}\in(0,4) such that
\text{f}'(\text{c})=\frac{\text{f}(4)-\text{f}(0)}{4-0}=\frac{\text{f}(4)-\text{f}(0)}{4}
Now, f(x) = x3 + 8x2 + 16x
⇒ f(x) = 3x2 + 16x + 16,
⇒ f(4) = 64+ 128 + 64 = 256,
⇒ f(0) = 0
\therefore\ \text{f}'(\text{x})=\frac{\text{f}(4)-\text{f}(0)}{4-0}
\Rightarrow3\text{x}^2+16\text{x}+16=\frac{256}{4}
\Rightarrow3\text{x}^2+16\text{x}-48=0
\Rightarrow\text{x}=-\frac{4}{3}\big(2+\sqrt{13}\big),\frac{4}{3}\big(\sqrt{13}-2\big)
Thus, \text{c}=\frac{-8+4\sqrt{13}}{3}\in(0,4) such that \text{f}'(\text{c})=\frac{\text{f}(4)-\text{f}(0)}{4-0}
Hence, Lagrange's mean value theorem is verified.
Q420. If \text{y}=1+\frac{\alpha}{\big(\frac{1}{\text{x}}-\alpha\big)}+\frac{\frac{\beta}{\text{x}}}{\big(\frac{1}{\text{x}}-\alpha\big)\big(\frac{1}{\text{x}}-\beta\big)}+\frac{\frac{\gamma} 4 Marks
{\text{x}^2}}{\big(\frac{1}{\text{x}}-\alpha\big)\big(\frac{1}{\text{x}}-\beta\big)\big(\frac{1}{\text{x}}-\gamma\big)}, find \frac{\text{dy}}{\text{dx}}

Ans: \text{y}=1+\frac{\alpha}{\big(\frac{1}{\text{x}}-\alpha\big)}+\frac{\frac{\beta}{\text{x}}}{\big(\frac{1}{\text{x}}-\alpha\big)\big(\frac{1}{\text{x}}-\beta\big)}+\frac{\frac{\gamma}{\text{x}^2}}
{\big(\frac{1}{\text{x}}-\alpha\big)\big(\frac{1}{\text{x}}-\beta\big)\big(\frac{1}{\text{x}}-\gamma\big)}
Using the theorem,
If \text{y}=1+\frac{\text{ax}^2}{(\text{x}-\text{a})(\text{x}-\text{b})(\text{x}-\text{c})}+\frac{\text{bx}}{(\text{x}-\text{b})(\text{x}-\text{c})}+\frac{\text{c}}{(\text{x}-\text{x})} then,
\frac{\text{dy}}{\text{dx}}=\frac{\text{y}}{\text{x}}\Big\{\frac{\text{a}}{\text{a}-\text{x}}+\frac{\text{b}}{\text{b}-\text{x}}+\frac{\text{c}}{\text{c}-\text{x}}\Big\}

https://bls.smartstudies.co.in/#/exam/pdf-preview/c59cb220-8e86-4716-9ff7-82aec16b1ade/1 91/158
5/26/24, 6:19 PM Exam Automation
Here, we have \frac{1}{\text{x}} instead of x,
So, using above theorem we get,
\frac{\text{dy}}{\text{dx}}=\frac{\alpha}{\big(\frac{1}{\text{a}}-\alpha\big)}+\frac{\beta}{\big(\frac{1}{\text{x}}-\beta\big)}+\frac{\gamma}{\big(\frac{1}{\text{x}}-\gamma\big)}
Q421. Differentiate the following functions with respect to x: 4 Marks
10^{\log\sin\text{x}}

Ans: Let \text{y}=10^{\log\sin\text{x}}\ .....(\text{i})


Taking log on both sides,
\log\text{y}=\log10^{\log\sin\text{x}}
\Rightarrow\log\text{y}=\log\sin\text{x}\log10
Differentiating with respect to x,
\Rightarrow\frac{1}{\text{y}}\frac{\text{dy}}{\text{dx}}=\log10\frac{\text{d}}{\text{dx}}\log\sin\text{x}
\Rightarrow\frac{1}{\text{y}}\frac{\text{dy}}{\text{dx}}=\log10\frac{1}{\sin\text{x}}\frac{\text{d}}{\text{dx}}(\sin\text{x})
\Rightarrow\frac{1}{\text{y}}\frac{\text{dy}}{\text{dx}}=\log10\Big(\frac{1}{\sin\text{x}}\Big)(\cos\text{x})
\Rightarrow\frac{\text{dy}}{\text{dx}}=\text{y}\big[\log10\times\cot\text{x}\big]
\Rightarrow\frac{\text{dy}}{\text{dx}}=10^{\log\sin\text{x}}\times\log10\times\cot\text{x}
[Using equation (i)]
Q422. The function \text{f(x)}=\begin{cases}\frac{\text{x}^2}{\text{a}},&\text{if }0\leq\text{ x}<1\\\text{a},&\text{if }1\leq\text{x}<\sqrt{2}\\\frac{2\text{b}^2-4\text{b}}{\text{x}^2},&\text{if 4 Marks
}\sqrt{2}\leq\text{x}<\infty\end{cases} is continuous on (0,\infty), then find the most suitable value of a and b.

Ans: Given, f is continuous on (0,\infty)


\therefore f is continuous at x = 1 and \sqrt{2}
Ar x = 1, we have
\lim\limits_{\text{x}\rightarrow1^-}\text{f(x)}=\lim\limits_{\text{h}\rightarrow0}(1-\text{h})\\=\lim\limits_{\text{h}\rightarrow0}\bigg[\frac{(1-\text{h})^2}{\text{a}}\bigg]=\frac{1}{\text{a}}
\lim\limits_{\text{x}\rightarrow1^+}\text{f(x)}=\lim\limits_{\text{h}\rightarrow0}(1+\text{h})\\=\lim\limits_{\text{h}\rightarrow0}(\text{a})=\text{a}
Also,
At \text{x}=\sqrt{2}, we have
\lim\limits_{\text{x}\rightarrow\sqrt{2}^-}\text{f(x)}=\lim\limits_{\text{h}\rightarrow0}\text{f}(\sqrt{2}-\text{h})\\=\lim\limits_{\text{h}\rightarrow0}(\text{a})=\text{a}
\lim\limits_{\text{x}\rightarrow\sqrt{2}^+}\text{f(x)}=\lim\limits_{\text{h}\rightarrow0}\text{f}(\sqrt{2}+\text{h})\\=\lim\limits_{\text{h}\rightarrow0}\bigg[\frac{2\text{b}^2-4\text{b}}
{(\sqrt{2}+\text{h})^2}\bigg]=\frac{2\text{b}^2-4\text{b}}{2}=\text{b}^2-2\text{b}
f is continuous at x = 1 and \sqrt{2}
\therefore\lim\limits_{\text{x}\rightarrow1^-}\text{f(x)}=\lim\limits_{\text{x}\rightarrow1^+}\text{f(x)} and \lim\limits_{\text{x}\rightarrow\sqrt{2}^-}\text{f(x)}=\lim\limits_{\text{x}\rightarrow\sqrt{2}^+}\text
\Rightarrow\frac{1}{\text{a}}=\text{a} and \text{b}^2-2\text{b}=\text{a}
\Rightarrow\text{a}^2=1 and \text{b}^2-2\text{b}=\text{a}
\Rightarrow\text{a}=\pm1 and \text{b}^2-2\text{b}=\text{a}\ ...(\text{i})
If a = 1, then
\text{b}^2-2\text{b}=\text{a} [From eq. (i)]
\Rightarrow\text{b}^2-2\text{b}-1=0
\Rightarrow\text{b}=\frac{2\pm\sqrt{4+4}}{2}=\frac{2\pm2\sqrt{2}}{2}\\=1\pm\sqrt{2}
If a = -1, then
\Rightarrow\text{b}^2-2\text{b}=-1 [From eq. (i)]
\Rightarrow\text{b}^2-2\text{b}+1=0
\Rightarrow(\text{b}-1)^2=0
\Rightarrow\text{b}=1
Hence, the most suitable value of a and b are
\text{a}=-1,\text{ b}=1 or \text{a}=1,\text{ b}=1\pm\sqrt{2}
Q423. If \text{a}(1-\cos\theta),\text{y}=\text{a}(\theta+\sin\theta), prove that, \frac{\text{d}^2\text{y}}{\text{dx}^2}=-\frac{1}{\text{a}} at \theta=\frac{\pi}{2}. 4 Marks

Ans: \text{a}(1-\cos\theta),\text{y}=\text{a}(\theta+\sin\theta),
Differentiating w.r.t.\theta,
\Rightarrow\frac{\text{dx}}{\text{d}\theta}=\text{a}(0+\sin\theta);\ ...\text{Eq}\ 1
\frac{\text{dy}}{\text{d}\theta}\ \text{a}(1+\cos\theta)\ ...\text{Eq}\ 2
Dividing (2) by (1)
\Rightarrow\frac{\text{dy}}{\text{dx}}=\frac{\text{dy}}{\text{d}\theta}\times\frac{\text{d}\theta}{\text{dx}}=\frac{\text{a}(1+\cos\theta)}{\text{a}\sin\theta}
Differentiating w.r.t.\theta,
\Rightarrow\frac{\text{d}\Big(\frac{\text{dy}}{\text{dx}}\Big)}{\text{d}\theta}=\frac{\sin\theta(0-\sin\theta)-(1+\cos\theta)\cos\theta}{\sin^2\theta}...(3)
=-\frac{\sin^2\theta-\cos\theta-\cos^2\theta}{\sin^2\theta}
=-\frac{(1+\cos\theta)}{\sin^2\theta}...(4 )
dividing (4) by (3)
\Rightarrow\frac{\text{d}^2\text{y}}{\text{dx}^2}=-\frac{(1+\cos\theta)}{\sin^2\theta\times\text{a}\sin\theta}
putting \theta=\frac{\pi}{2}
\Rightarrow\frac{\text{d}^2\text{y}}{\text{dx}^2}=-\frac{1}{\text{a}}
Hence proved
Q424. Differentiate w.r.t. x the function in Exercise: 4 Marks
\cot^{-1}\Big[\frac{\sqrt{1+\sin\text{x}}+\sqrt{1-\sin\text{x}}}{\sqrt{1+\sin\text{x}}-\sqrt{1-\sin\text{x}}}\Big],\ 0<\text{x}<\frac{\pi}{2}

Ans: Let \text{y}=\cot^{-1}\Big[\frac{\sqrt{1+\sin\text{x}}+\sqrt{1-\sin\text{x}}}{\sqrt{1+\sin\text{x}}-\sqrt{1-\sin\text{x}}}\Big]\ \dots(1)


Then, \frac{\sqrt{1+\sin\text{x}}+\sqrt{1-\sin\text{x}}}{\sqrt{1+\sin\text{x}}-\sqrt{1-\sin\text{x}}}
=\frac{(\sqrt{1+\sin\text{x}}+\sqrt{1-\sin\text{x}})^2}{(\sqrt{1+\sin\text{x}}-\sqrt{1-\sin\text{x}})(\sqrt{1+\sin\text{x}}+\sqrt{1-\sin\text{x}})}
=\frac{(1+\sin\text{x})+(1-\sin\text{x})+2\sqrt{(1-\sin\text{x})(1+\sin\text{x})}}{(1+\sin\text{x})-(1-\sin\text{x})}
=\frac{2+2\sqrt{1-\sin^2\text{x}}}{2\sin\text{x}}
=\frac{1+\cos\text{x}}{\sin\text{x}}
=\frac{2\cos^2\frac{\text{x}}{2}}{2\sin\frac{\text{x}}{2}\cos\frac{\text{x}}{2}}
=\cot\frac{\text{x}}{2}
Therefore, equation (1) becomes
\text{y}=\cot^{-1}\Big(\cot\frac{\text{x}}{2}\Big)
\Rightarrow\ \text{y}=\frac{\text{x}}{2}
\therefore\ \frac{\text{dy}}{\text{dx}}=\frac{1}{2}\frac{\text{d}}{\text{dx}}(\text{x)}
\Rightarrow\ \frac{\text{dy}}{\text{dx}}=\frac{1}{2}
Q425. If \text{f(x)}=\log\Big\{\frac{\text{u(x)}}{\text{v(x)}}\Big\},\text{u}(1)=\text{v}(1) and u'(1) = v'(1) = 2, then find the value of f(1). 4 Marks

Ans: We have, \text{f(x)}=\log\Big\{\frac{\text{u(x)}}{\text{v(x)}}\Big\}


And,
\text{u}(1)=\text{v}(1),\text{u}'(1)=\text{v}'(1)=2\ .....(\text{i})
\Rightarrow\text{f}'\text{(x)}=\frac{\text{d}}{\text{dx}}\Big[\text{f(x)}=\log\Big\{\frac{\text{u(x)}}{\text{v(x)}}\Big\}\Big]
\Rightarrow\text{f}'\text{(x)}=\frac{1}{\Big[\frac{\text{u(x)}}{\text{v(x)}}\Big]}\times\frac{\text{d}}{\text{dx}}\Big[\frac{\text{u(x)}}{\text{v(x)}}\Big]
\Rightarrow\text{f}'\text{(x)}=\frac{\text{v(x)}}{\text{u(x)}}\times\bigg[\frac{\text{v(x)}\frac{\text{d}}{\text{dx}}\{\text{u(x)}\}-\text{u(x)}\frac{\text{d}}{\text{dx}}\{\text{v(x)}\}}{\
{\text{v(x)}\}^2}\bigg]

https://bls.smartstudies.co.in/#/exam/pdf-preview/c59cb220-8e86-4716-9ff7-82aec16b1ade/1 92/158
5/26/24, 6:19 PM Exam Automation
\Rightarrow\text{f}'\text{(x)}=\frac{\text{v(x)}}{\text{u(x)}}\times\Big[\frac{\text{u(x)}\times\text{u}'\text{(x)}-\text{u(x)}\times\text{v}'\text{(x)}}{\{\text{v(x)}\}^2}\Big]
Putting x = 1, we get,
\text{f}'(1)=\frac{\text{v}(1)}{\text{u}(1)}\times\Big[\frac{\text{u}(1)\times\text{u}'(1)-\text{u}1\times\text{v}'(1)}{\{\text{v}(1)\}^2}\Big]
\Rightarrow\text{f}'(1)=1\times\Big[\frac{\text{u}(1)\times2-\text{u}(1)\times 2}{\{\text{u}(1)\}^2}\Big]
[Using eqn (1)]
\Rightarrow\text{f}'(1)=\Big[\frac{0}{\{\text{u}(1)\}^2}\Big]
\Rightarrow\text{f}'(1)=0
Q426. In the following, determine the values of constants involved in the definition so that the given function is continuous: 4 Marks
\text{f(x)}=\begin{cases}\frac{\sqrt{1+\text{px}}\sqrt{1-\text{px}}}{\text{x}},&\text{if }-1\leq\text{ x}\leq-0\\\frac{2\text{x}+1}{\text{x}-2},&\text{if }0\leq\text{ x}\leq1\end{cases}

Ans: Given, \text{f(x)}=\begin{cases}\frac{\sqrt{1+\text{px}}\sqrt{1-\text{px}}}{\text{x}},&\text{if }-1\leq\text{ x}\leq-0\\\frac{2\text{x}+1}{\text{x}-2},&\text{if }0\leq\text{ x}\leq1\end{cases}


If f(x) is continuous at x = 0, then
\lim_\limits{\text{x}\rightarrow0^-}\text{f(x)}=\lim_\limits{\text{x}\rightarrow0^+}\text{f(x)}
\Rightarrow\lim_\limits{\text{h}\rightarrow0}\text{f}(-\text{h})=\lim_\limits{\text{h}\rightarrow0}\text{f}(\text{h})
\Rightarrow\lim_\limits{\text{h}\rightarrow0}\bigg(\frac{\sqrt{1-\text{px}}\sqrt{1+\text{px}}}{-\text{h}}\bigg)=\lim_\limits{\text{h}\rightarrow0}\Big(\frac{2\text{h}+1}{\text{h}-2}\Big)
\Rightarrow\lim_\limits{\text{h}\rightarrow0}\Bigg(\frac{\big(\sqrt{1-\text{ph}}\big)-(\sqrt{1+\text{ph}})\big(\sqrt{1-\text{ph}}\big)+\big(1+\text{ph}\big)}{-\text{h}\big(\sqrt{1-
\text{ph}}+\sqrt{1+\text{ph}}\big)}\Bigg)=\lim_\limits{\text{h}\rightarrow0}\Big(\frac{2\text{h}+1}{\text{h}-2}\Big)
\Rightarrow\lim_\limits{\text{h}\rightarrow0}\Bigg(\frac{\big(1-\text{ph}-1-\text{ph}\big)}{-\text{h}\big(\sqrt{1-
\text{ph}}+\sqrt{1+\text{ph}}\big)}\Bigg)=\lim_\limits{\text{h}\rightarrow0}\Big(\frac{2\text{h}+1}{\text{h}-2}\Big)
\Rightarrow\lim_\limits{\text{h}\rightarrow0}\Bigg(\frac{\big(-2\text{ph}\big)}{-\text{h}\sqrt{1-
\text{ph}+\sqrt{1+\text{ph}}}}\Bigg)=\lim_\limits{\text{h}\rightarrow0}\Big(\frac{2\text{h}+1}{\text{h}-2}\Big)
\Rightarrow\lim_\limits{\text{h}\rightarrow0}\Bigg(\frac{\big(2\text{p}\big)}{\big(\sqrt{1-\text{ph}}+\sqrt{1+\text{ph}}\big)}\Bigg)=\lim_\limits{\text{h}\rightarrow0}\Big(\frac{2\text{h}+1}
{\text{h}-2}\Big)
\Rightarrow\Big(\frac{(2\text{p})}{(2)}\Big)=\Big(\frac{1}{-2}\Big)
\Rightarrow\text{p}=\frac{-1}{2}
Q427. Find all the points of discontinuity of f defined by f(x) = |x| – |x + 1|. 4 Marks

Ans: It is given function is \text{f(x)} =|\text{x}|-|\text{x} + 1|


The given function f is defined for real number and f can be written as the composition of two functions, as
f = goh, where, \text{g(x}) =| \text{x}|\ \text {and}\ \text{h(x)} = |\text{x} + 1|
Then, f = g - h
First we have to prove that \text{g(x}) =| \text{x}|\ \text {and}\ \text{h(x)} = |\text{x} + 1| are continuous functions.
g(x) = lxl can be written as
\text{g(x)}=\begin{cases}-\text{x},&\text{if}\ \text{x}<{0}\\\text{x},& \text{if}\ \text{x}\geq0\end{cases}
Now, g is defined for all real number.
Let k be a real number.
Case I: If k < 0,
Then g(k) = -k
And ^{\ \ \text{lim}}_{\text{x}\rightarrow\text{k}}\text{g(x)} = ^{\ \ \text{lim}}_{\text{x}\rightarrow\text{k}}(-\text{x}) = -\text{k}
Thus ^{\ \ \text{lim}}_{\text{x}\rightarrow\text{k}}\text{g(x)} =\text{g(k)}
Therefore, g is continuous at all points x, i.e. x > 0
Case II: If k > 0,
Then g(k) = k and
^{\ \ \text{lim}}_{\text{x}\rightarrow\text{k}}\text{g(x)} =^{\ \ \text{lim}}_{\text{x}\rightarrow\text{k}}\text{x}=\text{k}
Thus ^{\ \ \text{lim}}_{\text{x}\rightarrow\text{k}}\text{g(x)} =\text{g(k)}
Therefore, g is continuous at all points x, i.e. x < 0
Case III: If k = 0,
Then, g(k) = g(0) = 0
^{\ \ \text{lim}}_{\text{x}\rightarrow\text{0}^{-}}\text{g(x)} = ^{\ \ \text{lim}}_{\text{x}\rightarrow\text{0}^{-}}(-\text{x}) = 0
^{\ \ \text{lim}}_{\text{x}\rightarrow\text{0}^{+}}\text{g(x)} = ^{\ \ \text{lim}}_{\text{x}\rightarrow\text{0}^{+}}(\text{x}) = 0
\therefore^{\ \ \text{lim}}_{\text{x}\rightarrow\text{0}^{-}}\text{g(x)} = ^{\ \ \text{lim}}_{\text{x}\rightarrow\text{0}^{+}}\text g({\text x}) =\text{g}( 0)
Therefore, g is continuous at x = 0
From the above 3 cases, we get that g is continuous at all points.
g(x) = |x + 1| can be written as
\text{g(x)}=\begin{cases}-(\text{x} + 1),&\text{if}\ \text{x}<-{1}\\\text{x}+1,& \text{if}\ \text{x}\geq-1\end{cases}
Now, h is defined for all real number.
Let k be a real number.
Case I: If k < -1,
Then h(k) = -(k + 1)
And^{\ \ \text{lim}}_{\text{x}\rightarrow\text{k}}\text{h(x)} =^{\ \ \text{lim}}_{\text{x}\rightarrow\text{k}}[-(\text{x} + 1)] = - (\text{k} + 1)
= ^{\ \ \text{lim}}_{\text{x}\rightarrow\text{k}}\text{h(x)} = \text{h(k)}
Therefore, h is continuous at all points x, i.e., x > -1.
Case II: If k > -1,
Then h(k) = k + 1 and
^{\ \ \text{lim}}_{\text{x}\rightarrow\text{k}}\text{h(x)} =^{\ \ \text{lim}}_{\text{x}\rightarrow\text{k}}(\text{x} + 1) = \text{k} + 1
Thus = ^{\ \ \text{lim}}_{\text{x}\rightarrow\text{k}}\text{h(x)} = \text{h(k)}
Therefore, h is continuous at all points x, i.e., x > -1.
Case III: If k = -1,
Then, h(k) = h(-1) = -1 + 1 = 0
= ^{\ \ \text{lim}}_{\text{x}\rightarrow\text{1}^{-}}\text{h(x)} = ^{\ \ \text{lim}}_{\text{x}\rightarrow\text{1}^{-}}[-(\text{x} + 1)] = -(-1 + 1) = 0
= ^{\ \ \text{lim}}_{\text{x}\rightarrow\text{1}^{+}}\text{h(x)} = ^{\ \ \text{lim}}_{\text{x}\rightarrow\text{1}^{+}}(\text{x} + 1) = -(-1 + 1) = 0
\therefore\ ^{\ \ \text{lim}}_{\text{x}\rightarrow\text{1}^{-}}\text{h(x)} = ^{\ \ \text{lim}}_{\text{x}\rightarrow\text{1}^{+}}\text{h(x)} = \text{h}(-1)
Therefore, q is continuous at x = -1
From the above 3 cases, we get that h is continuous at all points.
Hence, g and h are continuous function.
Therefore, f = g - h is also a continuous function.
Q428. If \text{y}=\log\big\{\sqrt{\text{x}-1}-\sqrt{\text{x}+1}\big\}, show that \frac{\text{dy}}{\text{dt}}=\frac{-1}{2\sqrt{\text{x}^2-1}}. 4 Marks

Ans: Here \text{y}=\log\big\{\sqrt{\text{x}-1}-\sqrt{\text{x}+1}\big\}


Differentiating it with respect to x and applying the chain and product rule, we get
\frac{\text{dy}}{\text{dx}}=\frac{\text{d}}{\text{dx}} \log\big\{\sqrt{\text{x}-1}-\sqrt{\text{x}+1}\big\}
\frac{\text{dy}}{\text{dx}}=\frac{1}{\big\{\sqrt{\text{x}-1}-\sqrt{\text{x}+1}\big\}}\frac{\text{d}}{\text{dx}}(\sqrt{\text{x}-1}-\sqrt{\text{x}+1})
=\frac{1}{\big\{\sqrt{\text{x}-1}-\sqrt{\text{x}+1}\big\}}\Big[\frac{\text{d}}{\text{dx}}\sqrt{\text{x}-1}-\frac{\text{d}}{\text{dx}}\sqrt{\text{x}+1}\Big]
=\frac{1}{\big\{\sqrt{\text{x}-1}-\sqrt{\text{x}+1}\big\}}\Big[\frac{1}{2}(\text{x}-1)^{\frac{1}{2}}-\frac{1}{2}(\text{x}+1)^{\frac{1}{2}}\Big]
=\frac{1}{2}\frac{1}{\big\{\sqrt{\text{x}-1}-\sqrt{\text{x}+1}\big\}}\Big[\frac{1}{\sqrt{\text{x}-1}}-\frac{1}{\sqrt{\text{x}+1}}\Big]
=\frac{1}{2}\frac{1}{\big\{\sqrt{\text{x}-1}-\sqrt{\text{x}+1}\big\}}\Bigg(\frac{-\big\{\sqrt{\text{x}-1}-\sqrt{\text{x}-1}\big\}}{\big(\sqrt{\text{x}-1}\big)\big(\sqrt{\text{x}+1}\big)}\Bigg)
=\frac{1}{2}\bigg(\frac{1}{\big(\sqrt{\text{x}-1}\big)\big(\sqrt{\text{x}+1}\big)}\bigg)

https://bls.smartstudies.co.in/#/exam/pdf-preview/c59cb220-8e86-4716-9ff7-82aec16b1ade/1 93/158
5/26/24, 6:19 PM Exam Automation
\frac{\text{dy}}{\text{dx}}=\frac{-1}{2\sqrt{\text{x}^2-1}}
Therefore,
\frac{\text{dy}}{\text{dx}}=\frac{-1}{2\sqrt{\text{x}^2-1}}
Q429. The scalar product of the vector \vec{\text{a}}=\hat{\text{i}}+\hat{\text{j}}+\hat{\text{k}} with a unit vector along the sum of the 4 Marks
vectors \vec{\text{b}}=2\hat{\text{i}}+4\hat{\text{j}}-5\hat{\text{k}} and \vec{\text{c}}=\lambda\hat{\text{i}}+2\hat{\text{j}}+3\hat{\text{k}} k is equal to 1. Find the value of \lambda\ and
hence find the unit vector along \vec{\text{b}}+\vec{\text{c}}.

Ans: (2\hat{\text{i}}+4\hat{\text{j}}-5\hat{\text{k}})+(\lambda\hat{\text{i}}+2\hat{\text{j}}+\hat{3\text{k}})
=(2+\lambda)\hat{\text{i}}+6\hat{\text{j}}-2\hat{\text{K}}
Therefore, unit vector along
(2\hat{\text{i}}+4\hat{\text{j}}-5\hat{\text{k}})+(\lambda\hat{\text{i}}+2\hat{\text{j}}+\hat{3\text{k}})
is given us:
\frac{(2+\lambda)\hat{\text{i}}+6\hat{\text{j}}-2\hat{\text{k}}}{\sqrt{(2+\lambda)^2+6^2+(-2)^2}}
\Rightarrow\frac{(2+\lambda)\hat{\text{i}}+6\hat{\text{j}}-2\hat{\text{k}}}{\sqrt{4+4\lambda+\lambda^2+36+4}}
\Rightarrow\frac{(2+\lambda)\hat{\text{i}}+6\hat{\text{j}}-2\hat{\text{k}}}{\sqrt{\lambda^2+4\lambda+44}}
Scalar product of:
(\hat{\text{i}}+\hat{\text{j}}+\hat{\text{k}}​​)
With this unit vector is:
\Rightarrow(\hat{\text{i}}+\hat{\text{j}}+\hat{\text{k}}​​).\frac{(2+\lambda)\hat{\text{i}}+6\hat{\text{j}}-2\hat{\text{k}}}{\sqrt{\lambda^2+4\lambda+44}}=1
\Rightarrow\frac{(2+\lambda)+6-2}{\sqrt{\lambda^2+4\lambda+44}}=1
\Rightarrow\sqrt{\lambda^2+4\lambda+44}=\lambda+6
\Rightarrow\lambda^2+4\lambda+44=(\lambda+6)^2
\Rightarrow\lambda^2+4\lambda+44=\lambda^2+12\lambda+36
\Rightarrow8\lambda=8
\Rightarrow\lambda=1
Hence, the value of \lambda\text{ is }1.
Q430. Discuss the continuity of the following functions at the indicated point: 4 Marks
\text{f}\text{(x)}=\begin{cases}\frac{2\text{x}+\text{x}^2}{\text{x}}, & \text{x} \neq0\\0,&\text{ x} = 0\end{cases}\text{at x}=0

Ans: Given,
\Rightarrow\text{f}\text{(x)}=\frac{2\text{x}+\text{x}^2}{\text{x}},\text{x}>0
\Rightarrow\text{f}\text{(x)}=\frac{-2\text{x}+\text{x}^2}{\text{x}},\text{x}>0
\Rightarrow\text{f}\text{(x)}=0,\ \text{x}=0
\Rightarrow\text{f}\text{(x)}=\text{x+2},\ \text{x}>0
\Rightarrow\text{f}\text{(x)}=\text{x-2},\ \text{x}<0
\Rightarrow\text{f}\text{(x)}=0,\ \text{x}=0
We observe
(\text{LHL at x }=0)
\lim\limits_{\text{x} \rightarrow 0^-}\text{f}\text{(x)}=\lim\limits_{\text{x} \rightarrow 0}\text{f}\text{(0-h)}
\lim\limits_{\text{h} \rightarrow 0}\text{f}\text{(-h)}=\lim\limits_{\text{h} \rightarrow 0}-\text{h-2}
=-2
(\text{RHL at x}=0)
\lim\limits_{\text{h} \rightarrow 0^+}\text{f}\text{(x)}=\lim\limits_{\text{h} \rightarrow 0}\text{f}\text{(0+h)}
\lim\limits_{\text{h} \rightarrow 0^+}\text{f}\text{(x)}=\lim\limits_{\text{h} \rightarrow 0}\text{f}\text{(0+h)}
\lim\limits_{\text{h} \rightarrow 0}\text{f}\text{(h)}=\lim\limits_{\text{h} \rightarrow 0}\text{h+2}
=2
\Rightarrow\lim\limits_{\text{x} \rightarrow 0^-}\text{f}\text{(x)}\neq\lim\limits_{\text{x} \rightarrow 0^+}\text{f}\text{(x)}
Hence, f(x) is discontinuous at x = 0.
Q431. If xy = e(x–y), then show that \frac{\text{dy}}{\text{dx}} = \frac{\text{y (x - 1)}}{\text{x (y + 1)}}. 4 Marks

Ans: \text{x}\frac{\text{dy}}{\text{dx}} + \text{y} = \text{e}^{\text{x - y}} \bigg(1 - \frac{\text{dy}}{\text{dx}}\bigg)


= \text{xy} \bigg(1 - \frac{\text{dy}}{\text{dx}}\bigg)
\Rightarrow \frac{\text{dy}}{\text{dx}} = \frac{\text{xy - y}}{\text{x + xy}} = \frac{\text{y (x - 1)}}{\text{x(1 + y)}}
Q432. Verify the Rolle’s theorem for each of the functions: 4 Marks
\text{f(x)}=\log(\text{x}^2+2)-\log3\text{ in }[-1,1].

Ans: We have, \text{f(x)}=\log(\text{x}^2+2)-\log3


1. Logarithmic functions are continuous in their domain.
Hence, \text{f(x)}=\log(\text{x}^2+2)-\log3 is continuous in [-1,1]
2. \text{f}'(\text{x})=\frac{1}{\text{x}^2+2}\cdot2\text{x}-0
=\frac{2\text{x}}{\text{x}^2+2}, which exists in (-1,1).
Hence, f(x) is differentiable in (-1,1).
3. \text{f}(-1)=\log\big[(-1)^2+2\big]-\log3=\log3-\log3=0 and
\text{f}(1)=\log(1^2+2)-\log3=\log3-\log3=0
\Rightarrow\ \text{f}(-1)=\text{f}(1)
Conditions of Rolle’s theorem are satisfied.
Hence, there exists a real number c such that
\text{f}'(\text{c})=0
\Rightarrow\ \frac{2\text{c}}{\text{c}^2+2}=0
\Rightarrow\ \text{c}=0\in(-1,1)
Hence, Rolle’s theorem has been verified.
Q433. Examine the continuity of the function 4 Marks
\text{f}\text{(x)}=\begin{cases}3\text{x}-2 &, \text{ if x} \leq 0\\\text{x}+1 &, \text{ if x} > 0\end{cases}\text{at x}=0
Also sketch the graph of this function.

Ans: Given function is,


\text{f}\text{(x)}=\begin{cases}3\text{x}-2 &, \text{ if x} \leq 0\\\text{x}+1 &, \text{ if x} > 0\end{cases}\text{at x}=0\ ...(\text{ii})
We need to check whether f(x) is continuous at x = 0 or not.
For this we need to check L.H.L, R.H.L and value of function at x = 0
Clearly,
f(0) = 3*0 - 2 = -2 [from equation ii]
\text{L.H.L}=​​\lim\limits_{\text{h} \rightarrow 0}(0-\text{h})=\lim\limits_{\text{h} \rightarrow 0}\text{f}(-\text{h})
=\lim\limits_{\text{h} \rightarrow 0}\big\{3(-\text{h})-2\big\}=-2
\text{R.H.L}=\lim\limits_{\text{h} \rightarrow 0}\text{f}(0+\text{h})=\lim\limits_{\text{h} \rightarrow 0}\text{f(h)}
=\lim\limits_{\text{h} \rightarrow 0}\big\{\text{h}+1\big\}=0+1=1
As, \text{L.H.L}\neq\text{R.H.L}
\therefore\ \text{f(x)} is discontinuous at x = 0
This can also be proved by plotting f(x) on cartesian plane.

https://bls.smartstudies.co.in/#/exam/pdf-preview/c59cb220-8e86-4716-9ff7-82aec16b1ade/1 94/158
5/26/24, 6:19 PM Exam Automation
For x >0 ,we need to plot
y=x+1
put y = 0, we get x = -1 and for second point we put x = 0 and thus get y = 1
Two points are enough to plot the straight line.
Two coordinates are (-1, 0) and (0, 1)
For \text{x}\leq0, we need to plot
y = 3x - 2
put x = 0 then y = -2
On putting y = 0 we get \text{x}=\frac{2}{3}
Two coordinates are (0, -2) and \Big(\frac{2}{3},0\Big)

It can be seen from graph that there is breakage in curve at (0, 0)


Thus, it is discontinuous at x = 0
Q434. Differentiate the following functions with respect to x: 4 Marks
(1+\cos\text{x})^\text{x}

Ans: Let \text{y}=(1+\cos\text{x})^\text{x}\ .....(\text{i})


Taking log on both the sides,
\log\text{y}=\log(1+\cos\text{x})^\text{x}
\log\text{y}=\text{x}\log(1-\cos\text{x})
Differentiating with respect to x,
\frac{1}{\text{y}}\frac{\text{dy}}{\text{dx}}=\text{x}\frac{\text{d}}{\text{dx}}\log(1+\cos\text{x})+\log(1+\cos\text{x})\frac{\text{d}}{\text{dx}}(\text{x})
\Rightarrow\frac{1}{\text{y}}\frac{\text{dy}}{\text{dx}}=\text{x}\frac{1}{(1+\cos\text{x})}\frac{\text{d}}{\text{dx}}(1+\cos\text{x})+\log(1+\cos\text{x})(1)
\Rightarrow\frac{1}{\text{y}}\frac{\text{dy}}{\text{dx}}=\frac{\text{x}}{(1+\cos\text{x})}(0-\sin\text{x})+\log(1+\cos\text{x})
\Rightarrow\frac{1}{\text{y}}\frac{\text{dy}}{\text{dx}}=\log(1+\cos\text{x})-\frac{\text{x}\sin\text{x}}{(1+\cos\text{x})}
\Rightarrow\frac{\text{dy}}{\text{dx}}=\text{y}\Big[\log(1+\cos\text{x})-\frac{\text{x}\sin\text{x}}{(1+\cos\text{x})}\Big]
\Rightarrow\frac{\text{dy}}{\text{dx}}=(1+\cos\text{x})^\text{x}\Big[\log(1+\cos\text{x})-\frac{\text{x}\sin\text{x}}{(1+\cos\text{x})}\Big]
[Using equation (i)]
Q435. If y = sin-1 \Bigg[\frac{\text{5x + 12}\sqrt{1 - \text{x}^{2}}}{13}\Bigg],\text{ find }\frac{\text{dy}}{\text{dx}}. 4 Marks

Ans: Let \frac{5}{13}= \cos \alpha and x = \sin\theta


\therefore\sin\alpha=\frac{12}{13},\ \cos\theta=\sqrt{1-\text{x}^{2}}
\therefore\text{y}=\sin^{-1}(\sin\theta\cos\alpha+\cos\theta\sin\alpha)=\sin^{-1}[\sin(\theta+\alpha)]
=\theta+\alpha=\sin^{-1}\text{x}+\text{constant}
\therefore\text{dy}/\text{dx}=\frac{1}{\sqrt{1-\text{x}^{2}}}.
Q436. \text{If y} =\text{x}^{x},\text{prove that } \frac{\text{d}^{2}\text{y}}{\text{dx}^{2}} -\frac{1}{\text{y}}\bigg(\frac{\text{dy}}{\text{dx}}\bigg)^{2} -\frac{\text{y}}{\text{x}} =0. 4 Marks

Ans: \text{y} = \text{x}^{\text{x}}\therefore\log\text{ y } =\text{x}\log\text{x},


Taking log of both sides
\Rightarrow\frac{1}{\text{y}}\frac{\text{dy}}{\text{dx}} =\log\text{x} + 1 ,
Diff. w r t “x”
\Rightarrow\frac{1}{\text{y}}\frac{\text{d}^{2}\text{y}}{\text{dx}^{2}} -\frac{1}{\text{y}^{2}}\bigg(\frac{\text{dy}}{\text{dx}}\bigg)^{2} = \frac{1}{\text{x}},
Diff. w r t “x”
\Rightarrow\frac{\text{d}^{2}\text{y}}{\text{dx}^{2}} -\frac{1}{\text{y}}\bigg(\frac{\text{dy}}{\text{dx}}\bigg)^{2} - \frac{\text{y}}{\text{x}} - 0 .
Q437. Determine if \text{f(x)}=\begin{cases}\text{x}^2\sin\frac{1}{\text{x}},&\text{ x}\neq0\\0,&\text{x}=0\end{cases} is a continuous function? 4 Marks

Ans: The given function f is \text{f(x)}=\begin{cases}\text{x}^2\sin\frac{1}{\text{x}},&\text{ x}\neq0\\0,&\text{x}=0\end{cases}


It is evident that f is defind at all points of the real line.
Let c be a real number.
Case I:
If \text{c}\neq0, then \text{f(c)}=\text{c}^2\sin\frac{1}{\text{c}}
\lim\limits_{{\text{x}}\rightarrow\text{c}}\text{f(x)}=\lim\limits_{{\text{x}}\rightarrow\text{c}}\Big(\text{x}^2\sin\frac{1}
{\text{x}}\Big)=\Big(\lim\limits_{{\text{x}}\rightarrow\text{c}}\sin\frac{1}{\text{x}}\Big)=\text{c}^2\sin\frac{1}{\text{c}}
\therefore\ \lim\limits_{{\text{x}}\rightarrow\text{c}}\text{f(x)}=\text{f(c)}
So, f is continuouse at all points \text{x}\neq0
Case II:
If c = 0, then f(0) = 0
\lim\limits_{{\text{x}}\rightarrow0^-}\text{f(x)}=\lim\limits_{{\text{x}}\rightarrow0^-}\Big(\text{x}^2\sin\frac{1}{\text{x}}\Big)=\lim\limits_{{\text{x}}\rightarrow0}\Big(\text{x}^2\sin\frac{1}
{\text{x}}\Big)
It is know that -1\leq\sin\frac{1}{\text{x}}\leq1,\text{x}\neq0
\Rightarrow-\text{x}^2\leq\text{x}^2\sin\frac{1}{\text{x}}\leq\text{x}^2
\Rightarrow\lim\limits_{{\text{x}}\rightarrow0}(-\text{x}^2)\leq\lim\limits_{{\text{x}}\rightarrow0}\Big(\text{x}^2\sin\frac{1}{\text{x}}\Big)\leq\lim\limits_{{\text{x}}\rightarrow0}\text{x}^2
\Rightarrow0\leq\lim\limits_{{\text{x}}\rightarrow0}\Big(\text{x}^2\sin\frac{1}{\text{x}}\Big)\leq0
\Rightarrow\lim\limits_{{\text{x}}\rightarrow0}\Big(\text{x}^2\sin\frac{1}{\text{x}}\Big)=0
\Rightarrow\lim\limits_{{\text{x}}\rightarrow0^-}\text{f(x)}=0
Similarly, \lim\limits_{{\text{x}}\rightarrow0^+}\Big(\text{x}^2\sin\frac{1}{\text{x}}\Big)=\lim\limits_{{\text{x}}\rightarrow0}\Big(\text{x}^2\sin\frac{1}{\text{x}}\Big)=0
\therefore\ \lim\limits_{{\text{x}}\rightarrow0^-}\text{f(x)}=\text{f}(0)=\lim\limits_{{\text{x}}\rightarrow0^-}\text{f(x)}
So, f is continuous at x = 0
From the above observations, it can be continuouse that f is continuous at every point of the real line.
Thus, f is a continuous function.
Q438. Verify the Rolle’s Theorem for the function \text{f(x)}=\text{e}^{\text{x}}\cos\text{x}\text{ in}\Big[-\frac{\pi}{2},\frac{\pi}{2}\Big]. 4 Marks

Ans: The given function is \text{f(x)}=\text{e}^{\text{x}}\cos\text{x}.


Since \cos\text{x} and ex are everywhere continuous and differentialbem f(x) being a product of these two is continuous on \Big[-\frac{\pi}{2},\frac{\pi}{2}\Big] and differentiable on \Big(-
\frac{\pi}{2},\frac{\pi}{2}\Big).
Also,
\text{f}\Big(-\frac{\pi}{2}\Big)=\text{f}\Big(\frac{\pi}{2}\Big)=0
Thus f(x) satisfies all the conditions of Rolle's theorem.
Now, we have to show that there exists \text{c}\in\Big(-\frac{\pi}{2},\frac{\pi}{2}\Big) such that \text{f}'(\text{c})=0.
We have
\text{f(x)}=\text{e}^{\text{x}}\cos\text{x}
\Rightarrow\text{f}'(\text{x})=\text{e}^{\text{x}}(\cos\text{x}-\sin\text{x})
\therefore\text{f}'(\text{x})=0
\Rightarrow\text{e}^{\text{x}}(\cos\text{x}-\sin\text{x})=0
\Rightarrow\sin\text{x}-\cos\text{x}=0
\Rightarrow\tan\text{x}=1
\Rightarrow\text{x}=\frac{\pi}{4}
Since \text{c}=\frac{\pi}{4}\in \Big(\frac{-\pi}{2},\frac{\pi}{2}\Big) such that \text{f}'(\text{c})=0.

https://bls.smartstudies.co.in/#/exam/pdf-preview/c59cb220-8e86-4716-9ff7-82aec16b1ade/1 95/158
5/26/24, 6:19 PM Exam Automation
Hence, Rolle's theorem is verified.
Q439. Differentiate \sqrt{\tan\text{ x}} w.r.t. x from first principles. 4 Marks

Ans: \frac{\text{dy}}{\text{dx}}=\lim\limits_{\Delta\rightarrow0}\frac{\sqrt{\tan(\text{x +}\Delta\text{x})}-\sqrt{\tan\text{x}}}{\Delta\text{x}}


=\lim\limits_{\Delta\text{x}\rightarrow0}\frac{1}{\Big(\sqrt{\tan\text{( x + }\Delta\text{x})}+\sqrt{\tan\text{x}}\Big)}\cdot\frac{[\tan\text{(x +}\Delta\text{x})-\tan\text{x}]}{\Delta\text{x}}
=\frac{1}{2\sqrt{\tan\text{x}}}\lim\limits_{\Delta\text{x}\rightarrow0}\frac{\tan\text{(x +}\Delta\text{x - x})[1+\tan\text{ (x +}\Delta\text{x}).\tan\text{x}]}{\Delta\text{x}}
=\frac{1}{2\sqrt{\tan\text{x}}}.1.(1+\tan^{2}\text{x})=\frac{\sec^{2}\text{x}}{2\sqrt{\tan\text{x}}}.
Q440. Verify Rolle's theorem for the following function on the indicated intervals 4 Marks
\text{f}(\text{x})=\sin2\text{x}\text{ on }\Big[0,\frac{\pi}{2}\Big]

Ans: The given function is \text{f}(\text{x})=\sin2\text{x}


Since, \sin2\text{x} is everywhere continuous and differentiable.
Therefore \sin2\text{x} is continuous on \Big[0,\frac{\pi}{2}\Big], and differentiable on \Big(0,\frac{\pi}{2}\Big)
Also,
\text{f}\Big(\frac{\pi}{2}\Big)=\text{f}(0)=0
Thus, f(x) satisfies all the conditions of Rolle's theorem.
Now, we have to show that there exists \text{c}\in\Big(0,\frac{\pi}{2}\Big) such that f'(c) = 0.
We have
\text{f}(\text{x})=\sin2\text{x}
\Rightarrow\text{f}'(\text{x})=2\cos2\text{x}
\Rightarrow\text{f}'(\text{x})=0
\Rightarrow2\cos2\text{x}=0
\Rightarrow\cos2\text{x}=0
\Rightarrow\text{x}=\frac{\pi}{4}
Thus, \text{c}=\frac{\pi}{4}\in\Big(0,\frac{\pi}{2}\Big) such that \text{f}'(\text{c})=0.
Hence, Rolle's theorem is verified .
Q441. If sin y = x sin (a + y), prove that \frac{\text{dy}}{\text{dx}}=\frac{\sin^{2}\text{(a + y)}}{\sin\text{a}}. 4 Marks

Ans: \sin\text{y}=\text{x}\sin\text{ (a + y)}\Rightarrow\cos\text{y }\frac{\text{dy}}{\text{dx}}=\sin\text{(a + y)}+\text{x}\cos\text{(a + y)}\frac{\text{dy}}{\text{dx}}........\text{(i)}


\Rightarrow\frac{\text{dy}}{\text{dx}}=\frac{\text{sin (a + y)}}{\cos\text{ y - x }\cos\text{(a + y)}}
From (i), x = \frac{\sin\text{y}}{\sin\text{(a + y)}}\Rightarrow\frac{\text{dy}}{\text{dx}}=\frac{\text{sin (a + y)}}{\cos\text{y}-\frac{\sin\text{y}}{\sin\text{(a + y)}}\cdot\cos\text{(a + y)}}
\Rightarrow\frac{\text{dy}}{\text{dx}}=\frac{\sin^{2}\text{(a + y)}}{\sin(\text{a + y - y)}}=\frac{\sin^{2}\text{(a + y)}}{\sin\text{a}}.
Q442. Differentiate the following functions with respect to x: 4 Marks
\text{x}\sin2\text{x}+5^{\text{x}}+\text{k}^\text{k}+(\tan^2\text{x})^3

Ans: Let \text{y}=\text{x}\sin2\text{x}+5^{\text{x}}+\text{k}^\text{k}+(\tan^2\text{x})^3


Differentiate it with respect to x,
\frac{\text{dy}}{\text{dx}}=\frac{\text{d}}{\text{dx}}\big[\text{x}\sin2\text{x}+5^{\text{x}}+\text{k}^\text{k}+(\tan^2\text{x})^3\big]
=\frac{\text{d}}{\text{dx}}(\text{x}\sin2\text{x})+\frac{\text{d}}{\text{dx}}(5^\text{x})+\frac{\text{d}}{\text{dx}}(\text{k}^\text{k})+\frac{\text{d}}{\text{dx}}\big(\tan^6\text{x}\big)
=\Big[\text{x}\frac{\text{d}}{\text{dx}}(\sin2\text{x})+\sin2\text{x}\frac{\text{d}}{\text{dx}}(\text{x})\Big] \\ +5^\text{x}\log5+0+6\tan^5\text{x}\frac{\text{d}}{\text{dx}}(\tan\text{x})
[Using product rule and chain rule]
=\Big[\text{x}\cos2\text{x}\frac{\text{d}}{\text{dx}}(2\text{x})+\sin2\text{x}\Big]+5^\text{x}\log5+6\tan^5\text{x}\sec^2\text{x}
=2\text{x}\cos2\text{x}+\sin2\text{x}+5^\text{x}\log5+6\tan^5\text{x}\sec^2\text{x}
So,
\frac{\text{d}}{\text{dx}}\big(\text{x}\sin2\text{x}+5^\text{x}+\text{k}^\text{k}+(\tan^2\text{x})^3\big) \\
=2\text{x}\cos2\text{x}+\sin2\text{x}+5^\text{x}\log5+6\tan^5\text{x}\sec^2\text{x}
Q443. Differentiate the following functions with respect to x: 4 Marks
\log(\text{cosec x}-\cot\text{x})

Ans: Consider \text{y}=\log(\text{cosec x}-\cot\text{x})


Differentiate with respect to x,
\frac{\text{dy}}{\text{dx}}=\frac{\text{d}}{\text{dx}}\log\big(\text{cosec x}-\cot\text{x}\big)
=\frac{1}{(\text{cosec x}-\cot\text{x})}\times\big(-\text{cosec x}\cot\text{x}+\text{cosec}^2\text{x}\big)
[Using chain rule]
=\frac{1}{(\text{cosec x}-\cot\text{x})}\times\big(-\text{cosec x}\cot\text{x}+\text{cosec}^2\text{x}\big)
=\frac{\text{cosec x}(\text{cosec x}-\cos\text{x})}{(\text{cosec x}-\cot\text{x})}
=\text{cosec x}
Hence, the solution is, \frac{\text{d}}{\text{dx}}\big(\log(\text{cosec x}-\cot\text{x})\big)=\text{cosec x}
Q444. If \text{f(x)}=\begin{cases}\frac{\cos^2\text{x}-\sin^2\text{x}}{\sqrt{\text{x}^2+1}-1},&\text{x}\neq0\\\text{k},&\text{x}=0\end{cases} is continuous at x = 0, find k. 4 Marks

Ans: Given,
\text{f(x)}=\begin{cases}\frac{\cos^2\text{x}-\sin^2\text{x}}{\sqrt{\text{x}^2+1}-1},&\text{x}\neq0\\\text{k},&\text{x}=0\end{cases}
If f(x) is continuous at x = 0, then
\lim_\limits{\text{x}\rightarrow 0}\text{f(x)}=\text{f}(0)
\Rightarrow\lim_\limits{\text{x}\rightarrow 0}\frac{\cos^2\text{x}-\sin^2\text{x}}{\sqrt{\text{x}^2+1}-1}=\text{k}
\Rightarrow\lim_\limits{\text{x}\rightarrow 0}\frac{1-\sin^2\text{x}-\sin^2\text{x}-1}{{\sqrt{\text{x}^2+1}-1}}=\text{k}
\Rightarrow\lim_\limits{\text{x}\rightarrow 0}\frac{-2\sin^2\text{x}}{{\sqrt{\text{x}^2+1}-1}}=\text{k}
\Rightarrow\lim_\limits{\text{x}\rightarrow 0}\frac{-2(\sin^2\text{x})\big({\sqrt{\text{x}^2+1}+1}\big)}{\big({\sqrt{\text{x}^2+1}-1}\big)\big({\sqrt{\text{x}^2+1}+1}\big)}=\text{k}
\Rightarrow\lim_\limits{\text{x}\rightarrow 0}\frac{-2(\sin^2\text{x})\big({\sqrt{\text{x}^2+1}+1}\big)}{\text{x}^2}=\text{k}
\Rightarrow-2\lim_\limits{\text{x}\rightarrow 0}\frac{(\sin^2\text{x})\big({\sqrt{\text{x}^2+1}+1}\big)}{\text{x}^2}=\text{k}
\Rightarrow-2\lim_\limits{\text{x}\rightarrow 0}\Big(\frac{\sin\text{x}}{\text{x}}\Big)^2\lim_\limits{\text{x}\rightarrow 0}\Big({\sqrt{\text{x}^2+1}-1}\Big)
\Rightarrow-2\times-1\times(1+1)=\text{k}
\Rightarrow\text{k}=-4
Q445. Find \frac{\text{dy}}{\text{dx}} 4 Marks
\text{y}=\text{x}^{\text{x}}+(\sin\text{x})^\text{x}

Ans: We have, \text{y}=\text{x}^{\text{x}}+(\sin\text{x})^\text{x}


\Rightarrow\text{y}=\text{e}^{\log\text{x}^\text{x}}+\text{e}^{\log(\sin\text{x})^\text{x}}
\Rightarrow\text{y}=\text{e}^{\text{x}\log\text{x}}+\text{e}^{\text{x}\log\sin\text{x}}
Differentiating with respect to x using chain rule and product rule,
\frac{\text{dy}}{\text{dx}}=\frac{\text{d}}{\text{dx}}\big(\text{e}^{\text{x}\log\text{x}}\big)+\frac{\text{d}}{\text{dx}}\big(\text{e}^{\text{x}\log\sin\text{x}}\big)
=\text{e}^{\text{x}\log\text{x}}\frac{\text{d}}{\text{dx}}(\text{x}\log\text{x})+\text{e}^{\text{x}\log\sin\text{x}}\frac{\text{d}}{\text{dx}}(\text{x}\log\sin\text{x})
=\text{e}^{\text{x}\log\text{x}}\Big[\text{x}\frac{\text{d}}{\text{dx}}(\log\text{x})+\log\text{x}\frac{\text{d}}{\text{dx}}(\text{x})\Big] \\
+\text{e}^{\log(\sin\text{x})^\text{x}}\Big[\text{x}\frac{\text{d}}{\text{dx}}(\log\sin\text{x})+\log\sin\text{x}\frac{\text{d}}{\text{dx}}(\text{x})\Big]
=\text{x}^{\text{x}}\Big[\text{x}\Big(\frac{1}{\text{x}}\Big)+\log\text{x}(1)\Big] \\ +(\sin\text{x})^\text{x}\Big[\text{x}\Big(\frac{1}{\sin\text{x}}\Big)\frac{\text{d}}{\text{dx}}
(\sin\text{x})+\log\sin\text{x}\big]
=\text{x}^{\text{x}}\big[1+\log\text{x}\big]+(\sin\text{x})^\text{x}\Big[\text{x}\Big(\frac{1}{\sin\text{x}}\Big)(\cos\text{x})+\log\sin\text{x}\Big]
=\text{x}^{\text{x}}\big[1+\log\text{x}\big]+(\sin\text{x})^\text{x}\big[\text{x}\cot\text{x}+\log\sin\text{x}\big]
Q446. \text{Find} \frac{\text{dy}}{\text{dx}} \text{if (x}^{2} + \text{y}^2)^{2} = \text{xy.} 4 Marks

https://bls.smartstudies.co.in/#/exam/pdf-preview/c59cb220-8e86-4716-9ff7-82aec16b1ade/1 96/158
5/26/24, 6:19 PM Exam Automation
Ans: \text{(x}^{2} + \text{y}^{2})^{2} = \text{xy} \Rightarrow 2 \text{(x}^{2} + \text{y}^{2}) \bigg(\text{2x + 2y} \frac{\text{dy}}{\text{dx}}\bigg) = \text{x} \frac{\text{dy}}{\text{dx}} + \text{y}
\Rightarrow \text{4y} \frac{\text{dy}}{\text{dx}}\text{(x}^{2} + \text{y}^{2}) -\text{(x}^{2} + \text{y}^{2})- \text{x} \frac{\text{dy}}{\text{dx}} = \text{y - 4x} \text{(x}^{2} + \text{y}^{2})
\therefore \frac{\text{dy}}{\text{dx}} = \frac{\text{y - 4x}\text{(x}^{2} + \text{y}^{2})}{\text{4y}\text{(x}^{2} + \text{y}^{2}) - \text{x}}
Q447. Find \frac{\text{dy}}{\text{dx}} in the following cases: 4 Marks
4\text{x}+3\text{y}=\log\big(4\text{x}-3\text{y}\big)

Ans: We have, 4\text{x}+3\text{y}=\log\big(4\text{x}-3\text{y}\big)


Differentiating with respect to x, we get,
\frac{\text{d}}{\text{dx}}\big(4\text{x}\big)+\frac{\text{d}}{\text{dx}}\big(3\text{y}\big)=\frac{\text{d}}{\text{dx}}\big\{\log\big(4\text{x}-3\text{y}\big)\big\}
\Rightarrow4+3\frac{\text{dy}}{\text{dx}}=\frac{1}{\big(4\text{x}-3\text{y}\big)}\frac{\text{d}}{\text{dx}}\big(4\text{x}-3\text{y}\big)
\Rightarrow4+3\frac{\text{dy}}{\text{dx}}=\frac{1}{\big(4\text{x}-3\text{y}\big)}\Big(4-3\frac{\text{dy}}{\text{dx}}\Big)
\Rightarrow3\frac{\text{dy}}{\text{dx}}+\frac{3}{\big(4\text{x}-3\text{y}\big)}\frac{\text{dy}}{\text{dx}}=\frac{4}{\big(4\text{x}-3\text{y}\big)}-4
\Rightarrow3\frac{\text{dy}}{\text{dx}}\Big\{1+\frac{1}{(4\text{x}-3\text{y})}\Big\}=4\Big\{\frac{1}{(4\text{x}-3\text{y})}-1\Big\}
\Rightarrow3\frac{\text{dy}}{\text{dx}}\Big\{\frac{4\text{x}-3\text{y}+1}{(4\text{x}+3\text{y})}\Big\}=4\Big\{\frac{1-4\text{x}-3\text{y}}{4\text{x}-3\text{y}}\Big\}
\Rightarrow\frac{\text{dy}}{\text{dx}}=\frac{4}{3}\Big\{\frac{1-4\text{x}+3\text{y}}{(4\text{x}-3\text{y})}\Big\}\Big(\frac{4\text{x}-3\text{y}}{4\text{x}-3\text{y}+1}\Big)
\Rightarrow\frac{\text{dy}}{\text{dx}}=\frac{4}{3}\Big(\frac{1-4\text{x}+3\text{y}}{4\text{x}-3\text{y}+1}\Big)
Q448. Find \frac{\text{dy}}{\text{dx}} in the following cases: 4 Marks
(x + y)2 = 2axy

Ans: We Have, (x + y)2 = 2axy


Differentiating with respect to x, we get,
\Rightarrow\frac{\text{d}}{\text{dx}}\big(\text{x}+\text{y}\big)^2=\frac{\text{d}}{\text{dx}}\big(2\text{axy}\big)
\Rightarrow2(\text{x}+\text{y})\frac{\text{d}}{\text{dx}}(\text{x}+\text{y})=2\text{a}\Big[\text{x}\frac{\text{dy}}{\text{dx}}+\text{y}\frac{\text{d}}{\text{dx}}(\text{x})\Big]
\Rightarrow2(\text{x}+\text{y})\Big[1+\frac{\text{dy}}{\text{dx}}\Big]=2\text{a}\Big[\text{x}\frac{\text{dy}}{\text{dx}}+\text{y}(1)\Big]
\Rightarrow2(\text{x}+\text{y})+2(\text{x}+\text{y})\frac{\text{dy}}{\text{dx}}=2\text{a}\text{x}\frac{\text{dy}}{\text{dx}}+2\text{ay}
\Rightarrow\frac{\text{dy}}{\text{dx}}\big[2(\text{x}+\text{y})-2\text{a}\text{x}\big]=2\text{ay}-2(\text{x}+\text{y})
\Rightarrow\frac{\text{dy}}{\text{dx}}=\frac{2[\text{ay}-\text{x}-\text{y}]}{2[\text{x}+\text{y}-\text{a}\text{x}]}
\Rightarrow\frac{\text{dy}}{\text{dx}}=\Big(\frac{\text{ay}-\text{x}-\text{y}}{\text{x}+\text{y}-\text{a}\text{x}}\Big)
Q449. Verify Rolle's theorem of the following function on the indicated interval 4 Marks
\text{f}(\text{x})=\cos2{\text{x}}\text{ on }[0,\pi]

Ans: Here
\text{f}(\text{x})=\cos2{\text{x}}\text{ on }[0,\pi]
We know that, cosine function is continuous and differentiable every where, so f(x) is continuous is [0,\pi] and differentiable is (0,\pi).
Now,
\text{f}(0)=\cos0=1
\text{f}(\pi)=\cos(2\pi)=1
\Rightarrow\text{f}(0)=\text{f}(\pi)
So, Rolle's theorem is applicable, so there must exist a point \text{c}\in(0,\pi) such that f'(c) = 0.
Now,
\text{f}(\text{x})=\cos2\text{x}
\text{f}'(\text{x})=-2\sin2\text{x}
So, \text{f}'(\text{c})=0
\Rightarrow-2\sin2\text{c}=0
\Rightarrow\sin2\text{c}=0
\Rightarrow2\text{c}=0 or 2\text{c}=\pi
\Rightarrow\text{c}=0 or \text{c}=\frac{\pi}{2}\in(0,\pi)
Hence, Rolle's theorem is verified.
Q450. Find \frac{\text{dy}}{\text{dx}}, when 4 Marks
\text{x}=\text{a}(\cos\theta+\theta\sin\theta) and \text{y}=\text{a}(\sin\theta-\theta\sin\theta-\theta\cos\theta)

Ans: We have, \text{x}=\text{a}(\cos\theta+\theta\sin\theta) and \text{y}=\text{a}(\sin\theta-\theta\sin\theta-\theta\cos\theta)


\Rightarrow\frac{\text{dx}}{\text{d}\theta}=\text{a}\Big[\frac{\text{d}}{\text{d}\theta}\cos\theta+\frac{\text{d}}{\text{d}\theta}(\theta\sin\theta)\Big] and
\frac{\text{dy}}{\text{d}\theta}=\text{a}\Big[\frac{\text{d}}{\text{d}\theta}\sin\theta-\frac{\text{d}}{\text{d}\theta}(\theta\cos\theta)\Big]
\Rightarrow\frac{\text{dx}}{\text{d}\theta}=\text{a}\Big[-\sin\theta+\theta\frac{\text{d}}{\text{d}\theta}(\sin\theta)+\sin\theta\frac{\text{d}}{\text{d}\theta}(\theta)\Big] and
\frac{\text{dy}}{\text{d}\theta}=\text{a}\Big[\cos\theta-\left\{\theta\frac{\text{d}}{\text{d}\theta}(\cos\theta)+\cos\theta\frac{\text{d}}{\text{d}\theta}(\theta)\right\}\Big]
\Rightarrow\frac{\text{dx}}{\text{d}\theta}=\text{a}\big[-\sin\theta+\theta\cos\theta\big] and
\frac{\text{dy}}{\text{d}\theta}=\text{a}\big[\cos\theta+\theta\sin\theta-\cos\theta\big]
\Rightarrow\frac{\text{dx}}{\text{d}\theta}=\text{a}\theta\cos\theta and \frac{\text{dy}}{\text{d}\theta}=\text{a}\theta\sin\theta
\therefore\frac{\text{dy}}{\text{dx}}=\frac{\frac{\text{dy}}{\text{d}\theta}}{\frac{\text{dx}}{\text{d}\theta}}=\frac{\text{a}\theta\sin\theta}{\text{a}\theta\cos\theta}=\tan\theta
Q451. Differentiate the following functions with respect to x: 4 Marks
\cos^{-1}\Big\{\frac{\cos\text{x}+\sin\text{x}}{\sqrt{2}}\Big\},-\frac{\pi}{4}<\text{x}<\frac{\pi}{4}

Ans: Let \text{y}=\cos^{-1}\Big\{\frac{\cos\text{x}+\sin\text{x}}{\sqrt{2}}\Big\}


\text{y}=\cos^{-1}\Big\{\cos\text{x}\Big(\frac{1}{\sqrt{2}}\Big)+\sin\text{x}\Big(\frac{1}{\sqrt{2}}\Big)\Big\}
=\cos^{-1}\Big\{\cos\text{x}\cos\Big(\frac{\pi}{4}\Big)+\sin\text{x}\sin\text{x}\Big(\frac{\pi}{4}\Big)\Big\}
\text{y}=\cos^{-1}\Big[\cos\Big(\text{x}-\frac{\pi}{4}\Big)\Big]
Here, -\frac{\pi}{4}<\text{x}<\frac{\pi}{4}
\Rightarrow\ \Big(-\frac{\pi}{4}-\frac{\pi}{4}\Big)<\Big(\text{x}-\frac{\pi}{4}\Big)<\Big(\frac{\pi}{4}-\frac{\pi}{4}\Big)
\Rightarrow-\frac{\pi}{2}<\Big(\text{x}-\frac{\pi}{4}\Big)<0
So, from equation (i),
\text{y}=-\Big(\text{x}-\frac{\pi}{4}\Big)
\Big[\text{Since}, \cos^{-1}(\cos\theta)=-\theta,\text{ if }\theta\in [-\pi, 0]\Big]
\text{y}=-\text{x}+\frac{\pi}{4}
Differentiating with respect to x,
\frac{\text{dy}}{\text{dx}}=-1
Q452. Differentiate the following functions with respect to x: 4 Marks
\text{x}^{\frac{1}{\text{x}}}

Ans: Let \text{y}=\text{x}^{\frac{1}{\text{x}}}\ .....(\text{i})


Taking log on both sides,
\log\text{y}=\log\text{x}^{\frac{1}{\text{x}}}
\Rightarrow\log\text{y}=\frac{1}{\text{x}}\log\text{x}\ \big[\because\log\text{a}^{\text{b}}=\text{b}\log\text{a}\big]
Differentiating with respect to x,
\Rightarrow\frac{1}{\text{y}}\frac{\text{dy}}{\text{dx}}=\frac{1}{\text{x}}\frac{\text{d}}{\text{dx}}(\log\text{x})+\log\text{x}\frac{\text{d}}{\text{dx}}\big(\text{x}^{-1}\big)
[Using product rule]
\Rightarrow\frac{1}{\text{y}}\frac{\text{dy}}{\text{dx}}=\frac{1}{\text{x}}\times\frac{1}{\text{x}}(\log\text{x})\times\Big(-\frac{1}{\text{x}^2}\Big)
\Rightarrow\frac{1}{\text{y}}\frac{\text{dy}}{\text{dx}}=\frac{1}{\text{x}^2}-\frac{\log\text{x}}{\text{x}^2}
\Rightarrow \frac{1}{\text{y}}\frac{\text{dy}}{\text{dx}}=\frac{(1-\log\text{x})}{\text{x}^2}

https://bls.smartstudies.co.in/#/exam/pdf-preview/c59cb220-8e86-4716-9ff7-82aec16b1ade/1 97/158
5/26/24, 6:19 PM Exam Automation
\Rightarrow \frac{\text{dy}}{\text{dx}}=\text{x}^\frac{1}{\text{x}}\Big[\frac{1-\log\text{x}}{\text{x}}\Big]
Q453. Prove that the function 4 Marks
\text{f}\text{(x)}=\begin{cases}\frac{\text{x}}{|\text{x|+2}\text{x}^2}, &\text{ x}\neq0\\\text{k}, &\text{ x}=0\end{cases}
remains discontinuous at x = 0, regardless the choice of k.

Ans: The given function can be rewritten as,


\text{f}\text{(x)}=\begin{cases}\frac{\text{x}}{\text{x}+2\text{x}^2}, & \text{x} > 0\\\frac{-\text{x}}{\text{x}-2\text{x}^2}, &\text{x} <0\\ \text{k},&\text{x}=0\end{cases}
\Rightarrow\text{f}\text{(x)}=\begin{cases}\frac{1}{2\text{x}+1}, &\text{x} > 0\\\frac{1}{2\text{x}-1}, & \text{x} <0\\\text{k},&\text{x} =0\end{cases}
We observe
\text{(LHL at x}=0)\lim\limits_{\text{x} \rightarrow 0^-}\text{f}\text{(x)}=\lim\limits_{\text{h} \rightarrow 0}\text{f}(0-\text{h)}
=\lim\limits_{\text{h} \rightarrow 0}\text{f}(-\text{h)}=\lim\limits_{\text{h} \rightarrow 0}\frac{1}{-2\text{h}-1}=-1
\text{(RHL at x}=0)\lim\limits_{\text{x} \rightarrow 0^+}\text{f}\text{(x)}=\lim\limits_{\text{h} \rightarrow 0}\text{f}(0+\text{h)}
=\lim\limits_{\text{h} \rightarrow 0}\text{f}(\text{h)}=\lim\limits_{\text{h} \rightarrow 0}\frac{1}{2\text{h}+1}=1
So,=\lim\limits_{\text{x} \rightarrow 0^-}\text{f}\text{(x)}\neq\lim\limits_{\text{x} \rightarrow 0^+}\text{f}\text{(x)} Such that
=\lim\limits_{\text{x} \rightarrow 0^-}\text{f}\text{(x)}\&\lim\limits_{\text{x} \rightarrow 0^+}\text{f}\text{(x)}
are independent of k.
Thus, f(x) is discontinuous at x = 0,
regardless of the choice of k.
Q454. Find the derivative of sin (3x + 2) w.r.t. x from first principle. 4 Marks

Ans: \frac{\Delta\text{y}}{\Delta\text{x}}=\frac{\sin(\text{3x + 3} \Delta\text{x}+2)-\sin(\text{3x + 2})}{\Delta\text{x}}


\therefore\frac{\text{dy}}{\text{dx}}=\lim_{\Delta\text{x} \to 0}\cdot\frac{2\cos\Bigg[\text{3x + 2 +}\frac{3\Delta\text{x}}{2}\Bigg]\sin\Bigg(3\frac{\Delta\text{x}}{2}\Bigg)}{\Delta\text{x}}
=2\cos(\text{ 3x + 2})\cdot\lim_{\Delta\text{x} \to 0}\frac{\sin\Big(3\frac{\Delta\text{x}}{2}\Big)}{3\frac{\Delta\text{x}}{2}}\cdot\frac{3}{2}
= 3 cos (3x + 2).
Q455. If \text{u}=\sin^{-1}\Big(\frac{2\text{x}}{1+\text{x}^2}\Big) and \text{v}=\tan^{-1}\Big(\frac{2\text{x}}{1-\text{x}^2}\Big), where -1 < x < 1 then write the value of \frac{\text{dy}}{\text{dx}}. 4 Marks

Ans: \text{u}=\sin^{-1}\Big(\frac{2\text{x}}{1+\text{x}^2}\Big) and \text{v}=\tan^{-1}\Big(\frac{2\text{x}}{1+\text{x}^3}\Big)


We know, \frac{\text{du}}{\text{dx}}=\frac{2}{1+\text{x}^2}
Using the chain rule of differentiation,
\frac{\text{dv}}{\text{dx}}=\frac{1}{1+\Big(\frac{2\text{x}}{1+\text{x}^2}\Big)^2}\cdot\frac{(1+\text{x}^2)\cdot(2\text{x})'-(1+\text{x}^2)'\cdot(2\text{x})}{(1+\text{x}^2)^2}
=\frac{(1+\text{x}^2)^2}{(1+\text{x}^2)^2+(2\text{x})^2}\cdot\frac{2(1+\text{x}^2)-(2\text{x})(2\text{x})}{(1+\text{x}^2)^2}
=\frac{2(1-\text{x}^2)}{(1+\text{x}^2)^2+(2\text{x})^2}
Using Chain Rule of Differentiation,
\frac{\text{du}}{\text{dv}}=\frac{\text{du}}{\text{dx}}\cdot\frac{\text{dx}}{\text{dv}}
=\frac{2}{1+\text{x}^2}\cdot\frac{(1+\text{x}^2)^2+(2\text{x})^2}{2(1-\text{x})^2}
=\frac{(1+\text{x}^2)^2+(2\text{x})^2}{(1+\text{x}^2)(1-\text{x}^2)}
Dividing numerator and denominator by (1 + x2)2,
\frac{\text{du}}{\text{dv}}=\frac{1+\big(\frac{2\text{x}}{1+\text{x}^2}\big)}{\frac{1-\text{x}^2}{1+\text{x}^2}}
=\frac{1+\sin^2\text{u}}{\cos\text{u}}
=\sec\text{u}(1+\tan\text{u})
Q456. Find \frac{\text{dy}}{\text{ dx}} in the following: 4 Marks
\text{y}=\sin^{-1}\Bigg(\frac{2\text{x}}{1+\text{x}^{2}}\Bigg)

Ans: The given relationship is \text{y}=\sin^{-1}\Big(\frac{2\text{x}}{1+\text{x}^{2}}\Big)


\text{y}=\sin^{-1}\Big(\frac{2\text{x}}{1+\text{x}^{2}}\Big)
\Rightarrow \sin\text{y}=\frac{2\text{x}}{1+\text{x}^{2}}
Differenting this relationship with respect to x, we obtain
\frac{\text{d}}{\text{dx}}(\sin\text{y})=\frac{\text{d}}{\text{dx}}​​\Big(\frac{2\text{x}}{1+\text{x}^{2}}\Big)
\Rightarrow\cos\text{y}\frac{\text{dy}}{\text{dx}}=\frac{\text{d}}{\text{dx}}\Big(\frac{2\text{x}}{1+\text{x}^{2}}\Big) ...(\text{i})
The function\frac{2\text{x}}{1+\text{x}^{2}}, is of the from of \frac{\text{u}}{\text{v}}.
Therefore, by quotient rule, we obtain
\frac{\text{d}}{\text{dx}}\Big(\frac{2\text{x}}{1+\text{x}^{2}}\Big)=\frac{(1+\text{x}^{2}).\frac{\text{d}}{\text{dx}}(2\text{x})-2\text{x}.\frac{\text{d}}{\text{dx}}(1 +\text{x}^{2})}{(1
+\text{x}^{2})^2}
=\frac{(1+\text{x}^{2}).2-2\text{x}.[0+2\text{x}]}{(1 +\text{x}^{2})^2}=\frac{2 + 2\text{x}^{2}-4\text{x}^{2}}{(1 +\text{x}^{2})^2}=\frac{2(1-\text{x}^{2})}{(1+\text{x}^{2})^2} ...(\text{ii})
Also, \sin\text{y}=\frac{2\text{x}}{1+\text{x}^{2}}
\Rightarrow\cos\text{y}=\sqrt{1-\sin^{2}\text{y}}=\sqrt{1-\Big(\frac{2\text{x}}{1+\text{x}^{2}}\Big)^2}=\sqrt{\frac{(1+ \text{x}^{2})^2-4\text{x}^{2}}{(1 +\text{x}^{2})^{2}}}
=\sqrt{\frac{1+\text{x}^4+2\text{x}^2-4\text{x}^2}{(1+\text{x}^2)^2}}=\sqrt{\frac{(1-\text{x}^2)^2}{(1+\text{x}^2)^2}}
=\sqrt{\frac{(1- \text{x}^{2})^{2}}{(1 +\text{x}^{2})^{2}}}=\frac{1-\text{x}^{2}}{1+\text{x}^{2}} ...\text{(iii)}
Form(1), (2), and (3), we obtain
\frac{1-\text{x}^{2}}{1+\text{x}^{2}}\times\frac{\text{dy}}{\text{dx}}=\frac{2(1-\text{x}^{2})}{(1 +\text{x}^{2})^{2}}
\Rightarrow\frac{\text{dy}}{\text{dx}}=\frac{2}{1+\text{x}^{2}}
Q457. Find \frac{\text{dy}}{\text{dx}}, when 4 Marks
\text{x}=\text{a}(\theta+\sin\theta) and \text{y}=\text{a}(1-\cos\theta)

Ans: Here, \text{x}=\text{a}(\theta+\sin\theta)


Differentiating it with respect to \theta
\frac{\text{dx}}{\text{d}\theta}=\text{a}(1+\cos\theta).....(\text{i})
And, \text{y}=\text{a}(1-\cos\theta)
Differentiating it with respect to \theta,
\frac{\text{dy}}{\text{d}\theta}=\text{a}(\theta+\sin\theta)
and
\frac{\text{dy}}{\text{d}\theta}=\text{a}\sin\theta...(\text{ii})
Using equation (i) and (ii),
=\frac{\text{a}\sin\theta}{\text{a}(1-\cos\theta)}
=\frac{\frac{2\sin\theta}{2}\frac{\cos\theta}{2}}{\frac{2\sin^{2}\theta}{2}}, \begin{Bmatrix} \text{Since, }1-\cos\theta=\frac{2\sin^{2\theta}}{2}\\\frac{2\sin\theta}{2}\frac{\cos\theta}
{2}=\sin\theta \end{Bmatrix}
=\frac{\text{dy}}{\text{dx}}=\frac{\tan}{2}
Q458. Verify Lagrange's mean value theorem for the following function on the indicated intervals. find a point 'c' in the indicated interval as stated by the Lagrange's mean value theorem. 4 Marks
f(x) = 2x - x2 on [0, 1]

Ans: We have,
f(x) = 2x - x2
Since a polynomial function is everywhere continuous and differentiable.
Therefore, f(x) is continuous on 0, 1 and differentiable on 0,1
Thus, both conditions of Lagrange's mean value theorem are satisfied.
So, there must exist at least one real number \text{c}\in0,1 such that
\text{f}'(\text{c})=\frac{\text{f}(1)-\text{f}(0)}{1-0}=\frac{\text{f}(1)-\text{f}(0)}{1}
Now,
f(x) = 2x - x2

https://bls.smartstudies.co.in/#/exam/pdf-preview/c59cb220-8e86-4716-9ff7-82aec16b1ade/1 98/158
5/26/24, 6:19 PM Exam Automation
⇒ f'(x) = 2 - 2x,
⇒ f(1) = 2 - 1
⇒ f(1) = 1,
⇒ f(0) = 0
\therefore\ \text{f}'(\text{x})=\frac{\text{f}(1)-\text{f}(0)}{1-0}
\Rightarrow2-2\text{x}=\frac{1-0}{1}
\Rightarrow-2\text{x}=1-2
\Rightarrow\text{x}=\frac{1}{2}
Thus, \text{c}=\frac{1}{2}\in(1,0) such that \text{f}'(\text{c})=\frac{\text{f}(1)-\text{f}(0)}{1-0}
Hence, Lagrange's mean value theorem is verified.
Q459. Differentiate the following functions with respect to x: 4 Marks
\sqrt{\frac{\text{a}^2-\text{x}^2}{\text{a}^2+\text{x}^2}}

Ans: Let \text{y}=\sqrt{\frac{\text{a}^2-\text{x}^2}{\text{a}^2+\text{x}^2}}


\Rightarrow\text{y}=\Big({\frac{\text{a}^2-\text{x}^2}{\text{a}^2+\text{x}^2}}\Big)^\frac{1}{2}
Differentiate it with respect to x,
\frac{\text{dy}}{\text{dx}}=\frac{\text{d}}{\text{dx}}\Big({\frac{\text{a}^2-\text{x}^2}{\text{a}^2+\text{x}^2}}\Big)^\frac{1}{2}
=\frac{1}{2}\Big({\frac{\text{a}^2-\text{x}^2}{\text{a}^2+\text{x}^2}}\Big)^{\frac{1}{2}-1}\times\frac{\text{d}}{\text{dx}}\Big(\frac{\text{a}^2-\text{x}^2}{\text{a}^2+\text{x}^2}\Big)
[Using chain rule]
=\frac{1}{2}\Big({\frac{\text{a}^2-\text{x}^2}{\text{a}^2+\text{x}^2}}\Big)^{\frac{1}{2}}\times\Bigg\{\frac{\big(\text{a}^2+\text{x}^2\big)\frac{\text{a}}{\text{dx}}\big(\text{a}^2-
\text{x}^2\big)-\big(\text{a}^2-\text{x}^2\big)\frac{\text{d}}{\text{dx}}(\text{a}^2+\text{x}^2)}{\big(\text{a}^2+\text{x}^2\big)}\Bigg\}
[Using chain rule]
=\frac{1}{2}\Big({\frac{\text{a}^2-\text{x}^2}{\text{a}^2+\text{x}^2}}\Big)^{\frac{1}{2}}\Bigg\{\frac{-2\text{x}\big(\text{a}^2+\text{x}^2\big)-2\text{x}\big(\text{a}^2-\text{x}^2\big)}
{\big(\text{a}^2+\text{a}^2\big)^2}\Bigg\}
=\frac{1}{2}\Big({\frac{\text{a}^2-\text{x}^2}{\text{a}^2+\text{x}^2}}\Big)^{\frac{1}{2}}\bigg\{\frac{-2\text{xa}^2-2\text{x}^3-2\text{xa}^2+2\text{x}^3}
{\big(\text{a}^2+\text{a}^2\big)^2}\bigg\}
=\frac{1}{2}\Big({\frac{\text{a}^2-\text{x}^2}{\text{a}^2+\text{x}^2}}\Big)^{\frac{1}{2}}\Bigg(\frac{-4\text{xa}^2}{\big(\text{a}^2+\text{x}^2\big)^3}\Bigg)
=\frac{-2\text{xa}^2}{\sqrt{\text{a}^2-\text{x}^2}\big(\text{a}^2+\text{x}^2\big)^{\frac{3}{2}}}
So,
\frac{\text{d}}{\text{dx}}\bigg(\sqrt{\frac{\text{a}^2-\text{x}^2}{\text{a}^2+\text{x}^2}}\bigg)=\frac{-2\text{xa}^2}{\sqrt{\text{a}^2-\text{x}^2}\big(\text{a}^2+\text{x}^2\big)^{\frac{3}{2}}}
Q460. Differentiate the following functions with respect to x: 4 Marks
\frac{\text{e}^\text{x}\sin\text{x}}{(\text{x}^2+2)^3}

Ans: Consider \text{y}=\frac{\text{e}^\text{x}\sin\text{x}}{(\text{x}^2+2)^3}


Differentiating it with respect to x and applying the chain and product rule, we get
\frac{\text{dy}}{\text{dx}}=\frac{(\text{x}^2+2)^3\frac{\text{d}}{\text{dx}}(\text{e}^\text{x}\sin\text{x})-\text{e}^\text{x}\sin\text{x}\frac{\text{d}}{\text{dx}}(\text{x}^2+2)^3}
{\big[(\text{x}^2+2)^3\big]^2}
=\frac{(\text{x}^2+2)^3[\text{e}^\text{x}\cos\text{x}+\sin\text{xe}^\text{x}]-\text{e}^\text{x}\sin\text{x}3(\text{x}^2+2)^2(2\text{x})}{(\text{x}^2+2)^6}
=\frac{(\text{x}^2+2)^3[\text{e}^\text{x}\cos\text{x}+\sin\text{xe}^\text{x}]-6\text{xe}^\text{x}\sin\text{x}(\text{x}^2+2)^2}{(\text{x}^2+2)^6}
=\frac{(\text{x}^2+2)^2\big[(\text{x}^2+2)(\text{e}^\text{x}\cos\text{x}+\sin\text{xe}^\text{x})-6\text{xe}^\text{x}\sin\text{x}}{(\text{x}^2+2)^6}
=\frac{\text{x}^2\text{e}^\text{x}\cos\text{x}+\text{x}^2\sin\text{xe}^\text{x}+2\text{e}^\text{x}\cos\text{x}+2\sin\text{xe}^\text{x}-6\text{xe}^\text{x}\sin\text{x}}{(\text{x}^2+2)^4}
=\frac{\text{e}^\text{x}\sin\text{x}}{(\text{x}^2+2)^3}+\frac{\text{e}^\text{x}\cos\text{x}}{(\text{x}^2+2)^3}-\frac{6\text{xe}^\text{x}\sin\text{x}}{(\text{x}^2+2)^4}
Therefore,
\frac{\text{dy}}{\text{dx}}=\frac{\text{e}^\text{x}\sin\text{x}}{(\text{x}^2+2)^3}+\frac{\text{e}^\text{x}\cos\text{x}}{(\text{x}^2+2)^3}-\frac{6\text{xe}^\text{x}\sin\text{x}}{(\text{x}^2+2)^4}
Q461. If \text{y}=\text{e}^{\text{a}\cos^{-1}\text{x}}, -1<\text{x}<1, then show that (1-\text{x}^2)\frac{\text{d}^2\text{y}}{\text{dx}^2}-\text{x}\frac{\text{dy}}{\text{dx}}-\text{a}^2\text{y}=0. 4 Marks

Ans: It is given that, \text{y}=\text{e}^{\text{a}\cos^{-1}\text{x}}


Taking logarithm on both the sides, we obtain
\log\text{y}={\text{a}\cos^{-1}\text{x}}\log\text{e}
\log\text{y}={\text{a}\cos^{-1}\text{x}}
Differentiating both side with respect to x, we obtain
\frac{1}{2}\frac{\text{dy}}{\text{dx}}=\text{a}\times\frac{-1}{\sqrt{1-\text{x}^2}}
\Rightarrow\frac{\text{dy}}{\text{dx}}=\frac{-\text{ay}}{\sqrt{1-\text{x}^2}}
By squraing both the sides, we obtain
\Big(\frac{\text{dy}}{\text{dx}}\Big)^2=\frac{\text{a}^2\text{y}^2}{{1-\text{x}^2}}
\Rightarrow(1-\text{x}^2)\Big(\frac{\text{dy}}{\text{dx}}\Big)^2=\text{a}^2\text{y}^2
(1-\text{x}^2)\Big(\frac{\text{dy}}{\text{dx}}\Big)^2=\text{a}^2\text{y}^2
Again differentiating both sides with respect to x, we obtain
\bigg(\frac{\text{dy}}{\text{dx}}\bigg)^2\frac{\text{d}}{\text{dx}}(1-\text{x}^2)+(1-\text{x}^2)\times\frac{\text{d}}{\text{dx}}\Bigg[\bigg(\frac{\text{dy}}
{\text{dx}}\bigg)^2\Bigg]=\text{a}^2\frac{\text{d}}{\text{dx}}(\text{y}^2)
\Rightarrow\bigg(\frac{\text{dy}}{\text{dx}}\bigg)^2(-2\text{x})+(1-\text{x}^2)\times2\frac{\text{dy}}{\text{dx}}.\frac{\text{d}^2\text{y}}{\text{dx}^2}=\text{a}^2.2\text{y}.\frac{\text{dx}}
{\text{dx}}
\Rightarrow-\text{x}\frac{\text{dy}}{\text{dx}}+(1-\text{x}^2)\frac{\text{d}^2\text{y}}{\text{dx}^2}=\text{a}^2.\text{y} \Big[\frac{\text{dy}}{\text{dx}}\neq0\Big]
\Rightarrow(1-\text{x})+\frac{\text{d}^2\text{y}}{\text{dx}^2}-\text{x}\frac{\text{dy}}{\text{dx}}-\text{a}^2\text{y}=0
Hence, Proved.
Q462. Verify Rolle's theorem of the following function on the indicated interval 4 Marks
\text{f}(\text{x})=\sin\text{x}+\cos\text{x}\text{ on }\Big[0,\frac{\pi}{2}\Big]

Ans: The given function is \text{f}(\text{x})=\sin\text{x}+\cos\text{x}


Since, \sin\text{x} and \cos\text{x} are everywhere differentiable and continuous,
\text{f}(\text{x})=\sin\text{x}+\cos\text{x} is continuous on \Big[0,\frac{\pi}{2}\Big] and differentiable on \Big(0,\frac{\pi}{2}\Big)
Also,
\text{f}\Big(\frac{\pi}{2}\Big)=\text{f}(0)=1
Thus, f(x) satisfies all the conditions of Rolle's theorem.
Now, we have show that there must exists \text{c}\in\Big(0,\frac{\pi}{2}\Big) such that f'(c) = 0.
We have
\text{f}(\text{x})=\sin\text{x}+\cos\text{x}
\Rightarrow\text{f}'(\text{x})=\cos\text{x}-\sin\text{x}
\therefore\ \text{f}'(\text{x})=0
\Rightarrow\cos\text{x}-\sin\text{x}=0
\Rightarrow\tan\text{x}=1
\Rightarrow\text{x}=\frac{\pi}{4}
Thus, \text{c}=0\in\Big(0,\frac{\pi}{2}\Big) such that f'(c) = 0.
Hence, Rolle's theorem is verified.
Q463. If \text{x}^\text{m}.\text{y}^\text{n}=(\text{x}+\text{y})^{\text{m}+\text{n}}, prove that: 4 Marks
\frac{\text{dy}}{\text{dx}}=\frac{\text{y}}{\text{x}}

Ans: We have,

https://bls.smartstudies.co.in/#/exam/pdf-preview/c59cb220-8e86-4716-9ff7-82aec16b1ade/1 99/158
5/26/24, 6:19 PM Exam Automation
\text{x}^\text{m}.\text{y}^\text{n}=(\text{x}+\text{y})^{\text{m}+\text{n}}\ \ \dots(\text{i})
Differentiating Eq. (i) w.r.t. x, we get
\frac{\text{d}}{\text{dx}}(\text{x}^\text{m}.\text{y}^\text{n})=\frac{\text{d}}{\text{dx}}(\text{x}+\text{y})^{\text{m}+\text{n}}
\Rightarrow\ \text{x}^\text{m}.\frac{\text{d}}{\text{dy}}\text{y}^\text{n}.\frac{\text{dy}}{\text{dx}}+\text{y}^\text{n}.\frac{\text{d}}{\text{dx}}\text{x}^\text{m} =(\text{m}+\text{n})
(\text{x}+\text{y})^{\text{m}+\text{n}-1}\frac{\text{d}}{\text{dx}}(\text{x}+\text{y})
\Rightarrow\ \text{x}^\text{m}.\text{ny}^{\text{n}-1}\frac{\text{dy}}{\text{dx}}+\text{y}^\text{n}.\text{mx}^{\text{m}-1} =(\text{m}+\text{n})
(\text{x}+\text{y})^{\text{m}+\text{n}-1}\Big(1+\frac{\text{dy}}{\text{dx}}\Big)
\Rightarrow\ \frac{\text{dy}}{\text{dx}}\big[\text{x}^\text{m}.\text{ny}^{\text{n}-1}-(\text{m}+\text{n}).(\text{x}+\text{y})^{\text{m}+\text{n}-1}\big] =(\text{m}+\text{n})
(\text{x}+\text{y})^{\text{m}+\text{n}-1}-\text{y}^\text{n}\text{mx}^{\text{m}-1}
\Rightarrow\ \frac{\text{dy}}{\text{dx}}\big[\text{nx}^\text{m}\text{y}^{\text{n}-1}-(\text{m}+\text{n})(\text{x}+\text{y})^{\text{m}+\text{n}-1}\big] =(\text{m}+\text{n}
(\text{x}+\text{y})^{\text{m}+\text{n}-1}-\frac{\text{y}^{\text{n}-1}.\text{y}.\text{mx}^{\text{m}-1}}{\text{x}}
\therefore\ \frac{\text{dy}}{\text{dx}}=\frac{\frac{(\text{m}+\text{n})(\text{x}+\text{y})^{\text{m}+\text{n}}}{(\text{x}-\text{y})}-\frac{\text{y}^{\text{n}-1}.\text{y}.\text{mx}^\text{m}}
{\text{x}}}{\frac{\text{nx}^\text{m}\text{y}^\text{n}}{\text{y}}-(\text{m}+\text{n})(\text{x}+\text{y})^{\text{m}+\text{n}}\frac{1}{(\text{x}+\text{y})}}
=\frac{\frac{\text{x}(\text{m}+\text{n})(\text{x}+\text{y})^{\text{m}+\text{n}}-(\text{x}+\text{y}).\text{y}.^{\text{n}-1}\text{y}.\text{mx}^\text{m}}{(\text{x}+\text{y}).\text{x}}}
{\frac{(\text{x}+\text{y})\text{nx}^\text{m}\text{y}^\text{n}-\text{y}(\text{m}+\text{n})(\text{x}+\text{y})^{\text{m}+\text{n}}}{(\text{x}+\text{y}).\text{y}}}
=\frac{\frac{\text{x}(\text{m}+\text{n})\text{x}^\text{m}.\text{y}^\text{n}-\text{m}(\text{x}+\text{y})\text{y}^\text{n}\text{x}^\text{m}}{(\text{x}+\text{y}).\text{x}}}
{\frac{(\text{x}+\text{y})\text{nx}^\text{m}.\text{y}^\text{n}-\text{y}(\text{m}+\text{n}).\text{x}^\text{m}.\text{y}^\text{n}}
{(\text{x}+\text{y}).\text{y}}} \big[\because(\text{x}+\text{y})^{\text{m}+\text{n}}=\text{x}^\text{m}.\text{y}^\text{n}\big]
=\frac{\text{x}^\text{m}\text{y}^\text{n}[\text{mx}+\text{nx}-\text{mx}-\text{my}].(\text{x}+\text{y})\text{y}}{\text{x}^\text{m}\text{y}^\text{n}[\text{nx}+\text{ny}-\text{my}-\text{ny}].
(\text{x}+\text{y})\text{x}}
=\frac{\text{y}}{\text{x}}\ \ \dots(\text{i})
Hence proved.
Q464. A ladder 13m long is leaning against a vertical wall. The bottom of the ladder is dragged away from the wall along the ground at the rate of 2cm/sec. How fast is the height on the wall 4 Marks
decreasing when the foot of the ladder is 5m away from the wall?

Ans:

\frac{\text{dx}}{\text{dt}}=2\text{cm}/\text{sec}
\text{y}=\sqrt{169-\text{x}^2}
\frac{\text{dy}}{\text{dt}}=-\frac{\text{x}}{\sqrt{169-\text{x}^2}}\frac{\text{dx}}{\text{dt}}
\Big(\frac{\text{dy}}{\text{dt}}\Big)_\text{x=5}=\frac{-5}{6}\text{cm}/\text{sec}
Hence height is decreasing at the rate \frac{5}{6}\text{cm}/\text{sec}
Q465. If \text{y}=\sin\Big[2\tan^{-1}\Big\{\sqrt{\frac{1-\text{x}}{1+\text{x}}}\Big\}\Big], find \frac{\text{dy}}{\text{dx}}. 4 Marks

Ans: Here, \text{y}=\sin\Big[2\tan^{-1}\Big[\sqrt{\frac{1-\text{x}}{1+\text{x}}}\Big]\Big]


Put, \text{x}=\cos2\theta, \text{So},
\text{y}=\sin\Big[2\tan^{-1}\sqrt{\frac{1-\cos2\theta}{1+\cos2\theta}}\Big]
=\sin\Big[2\tan^{-1}\sqrt{\frac{2\sin^2\theta}{2\cos^2\theta}}\Big]
=\sin\big[2\tan^{-1}\sqrt{\tan^2\theta}\big]
=\sin\big[2\tan^{-1}(\tan\theta)\big]
=\sin{2\theta}
=\sin\Big[2\times\frac{1}{2}\cos^{-1}\text{x}\Big]\ \big[\text{Since, x}=\cos2\theta\big]
=\sin\big(\sin^{-1}\sqrt{1-\text{x}^2}\big)
\text{y}=\sqrt{1-\text{x}^2}
Differentiating with respect to x using chain rule,
\frac{\text{dy}}{\text{dx}}=\frac{1}{2\sqrt{1-\text{x}^2}}\frac{\text{d}}{\text{dx}}(1-\text{x}^2)
\frac{\text{dy}}{\text{dx}}=\frac{1}{2\sqrt{1-\text{x}^2}}(-2\text{x})
\frac{\text{dy}}{\text{dx}}=\frac{-\text{x}}{\sqrt{1-\text{x}^2}}
Q466. If \text{f}\text{(x)}=\begin{cases}\frac{\text{x}-4}{\text{|x}-4|}+\text{a}, &\text{if x} <4\\\text{a}+\text{b},&\text{if x}=4\\\frac{\text{x}-4}{\text{|x}-4|}+\text{b}, & \text{if x}>4\end{cases} is 4 Marks
continuous at x = 4. Find a, b.

Ans: Given,
f(x) is continuous at x = 4 & f(4) = a + b
For f(x) to be continuous at x = 4, f(4)- = f(4)+ = f(4)
\text{L.H.L}=\text{f(4)}^-=\lim\limits_{\text{x} \rightarrow 4}\frac{\text{x}-4}{|\text{x}-4|}+\text{a}
\Rightarrow\lim\limits_{\text{h} \rightarrow 0}\frac{(4-\text{h})-4}{|(4-\text{h})-4|}+\text{a}
\Rightarrow\lim\limits_{\text{h} \rightarrow 0}\frac{(4-\text{h}-4)}{|(4-\text{h}-4)|}+\text{a}
\Rightarrow\lim\limits_{\text{h} \rightarrow 0}\frac{(-\text{h})}{|(-\text{h})|}+\text{a}
\Rightarrow\lim\limits_{\text{h} \rightarrow 0}\frac{(-\text{h})}{\text{h}}+\text{a}
\Rightarrow\text{a}-1
\text{L.H.L}=\text{f(4)}^+=\lim\limits_{\text{x} \rightarrow 0}\frac{\text{x}-4}{|\text{x}-4|}
\Rightarrow\lim\limits_{\text{h} \rightarrow 0}\frac{(4+\text{h})-4}{|(4+\text{h})-4|}
\Rightarrow\lim\limits_{\text{h} \rightarrow 0}\frac{4+\text{h}-4}{|4+\text{h}-4|}
\Rightarrow\lim\limits_{\text{h} \rightarrow 0}\frac{\text{h}}{|\text{h}|}
\Rightarrow\lim\limits_{\text{h} \rightarrow 0}\frac{1}{|1|}
\Rightarrow1
Since, f(x) is is continuous at x = 4 & f(4) = a + b
\text{f(4)}^-=\text{f(4)}^+=\text{f(4)}
\therefore\ \text{a}-1=\text{a}+\text{b}=1
\Rightarrow\text{a}-1=1
\Rightarrow\text{a}=2
\Rightarrow\text{a}+\text{b}=1
\Rightarrow\text{b}=1-2
\Rightarrow\text{b}=-1
Q467. If \text{y}=\text{e}^{\text{x}^{\text{e}^\text{x}}}+\text{x}^{\text{e}^{\text{e}^\text{x}}}+\text{e}^{\text{x}^{\text{x}^{\text{e}}}}, prove that \frac{\text{dy}} 4 Marks
{\text{dx}}=\text{e}^{\text{x}^{\text{e}^\text{x}}}\times\text{x}^{\text{e}^{\text{x}}}\Big\{\frac{\text{e}^\text{x}}
{\text{x}}+\text{e}^\text{x}\log\text{x}\Big\}+\text{e}^{\text{x}^{\text{e}^{\text{x}}}}\times\text{e}^{\text{e}^\text{x}}\Big\{\frac{1}
{\text{x}}+\text{e}^\text{x}\times\log\text{x}\Big\}+\text{e}^{\text{x}^{\text{x}^\text{e}}}\text{x}^{\text{x}^{\text{e}}}\times\text{x}^{\text{e}-1}\Big\{\text{x}+\text{e}\log\text{x}\Big\}

Ans: We have, \text{y}=\text{e}^{\text{x}^{\text{e}^\text{x}}}+\text{x}^{\text{e}^{\text{e}^\text{x}}}+\text{e}^{\text{x}^{\text{x}^{\text{e}}}}

https://bls.smartstudies.co.in/#/exam/pdf-preview/c59cb220-8e86-4716-9ff7-82aec16b1ade/1 100/158
5/26/24, 6:19 PM Exam Automation
\Rightarrow\text{y}=\text{u}+\text{v}+\text{w}
\Rightarrow\frac{\text{dy}}{\text{dx}}=\frac{\text{du}}{\text{dx}}+\frac{\text{dv}}{\text{dx}}+\frac{\text{dw}}{\text{dx}}\ .....(\text{i})
Where \text{u}=\text{e}^{\text{x}^{\text{e}^{\text{x}}}},\text{v}=\text{x}^{\text{e}^{\text{e}^{\text{x}}}}\text{ and w}=\text{e}^{\text{x}^{\text{x}^{\text{e}}}}
Now, \text{u}=\text{e}^{\text{x}^{\text{e}^{\text{x}}}}\ .....(\text{ii})
Taking log on both sides,
\log\text{u}=\log\text{e}^{\text{x}^{\text{e}^{\text{x}}}}
\Rightarrow\log\text{u}=\text{x}^{\text{e}^\text{x}}\log\text{e}
\Rightarrow\log\text{u}=\text{x}^{\text{e}^\text{x}}\ .....(\text{iii})
Taking \log on both sides,
\log\log\text{u}=\log\text{x}^{\text{e}^\text{x}}
\Rightarrow\log\log\text{u}=\text{e}^\text{x}\log\text{x}
Differentiating with respect to x,
\Rightarrow\frac{1}{\log\text{u}}\frac{\text{d}}{\text{dx}}(\log\text{u})=\text{e}^\text{x}\frac{\text{d}}{\text{dx}}(\log\text{x})+\log\text{x}\frac{\text{d}}{\text{dx}}(\text{e}^\text{x})
\Rightarrow\frac{1}{\log\text{u}}\frac{1}{\text{u}}\frac{\text{du}}{\text{dx}}=\frac{\text{e}^\text{x}}{\text{x}}+\text{e}^\text{x}\log\text{x}
\Rightarrow\frac{\text{du}}{\text{dx}}=\text{u}\log\text{u}\Big[\frac{\text{e}^\text{x}}{\text{x}}+\text{e}^\text{x}\log\text{x}\Big]
\Rightarrow\frac{\text{du}}{\text{dx}}=\text{e}^{\text{x}^{\text{e}^{\text{x}}}}\times\text{x}^{\text{e}^\text{x}}\Big[\frac{\text{e}^\text{x}}{\text{x}}+\text{e}^\text{x}\log\text{x}\Big]\ .....
(\text{A})
[Using equation (ii) and (iii)]
Now, \text{v}=\text{x}^{\text{e}^{\text{e}^\text{x}}}\ .....(\text{iv})
Taking log on both sides,
\log\text{v}=\log\text{x}^{\text{e}^{\text{e}^\text{x}}}
\Rightarrow\log\text{v}=\text{e}^{\text{e}^\text{x}}\log\text{x}
\Rightarrow\frac{1}{\text{v}}\frac{\text{dv}}{\text{dx}}=\text{e}^{\text{e}^\text{x}}\frac{\text{d}}{\text{dx}}(\log\text{x})+\log\text{x}\frac{\text{d}}
{\text{dx}}\big(\text{e}^{\text{e}^\text{x}}\big)
\Rightarrow\frac{1}{\text{v}}\frac{\text{dv}}{\text{dx}}=\text{e}^{\text{e}^\text{x}}\big(\frac{1}{\text{x}}\big)+\log\text{xe}^{\text{e}^\text{x}}\frac{\text{d}}{\text{dx}}(\text{e}^\text{x})
\Rightarrow\frac{\text{dv}}{\text{dx}}=\text{v}\Big[\text{e}^{\text{e}^\text{x}}\big(\frac{1}{\text{x}}\big)+\log\text{xe}^{\text{e}^\text{x}}\text{e}^\text{x}\Big]
\Rightarrow\frac{\text{dv}}{\text{dx}}=\text{e}^{\text{e}^{\text{e}^\text{x}}}\times\text{e}^{\text{e}^\text{x}}\Big[\frac{1}{\text{x}}+\text{e}^\text{x}\log\text{x}\Big]\ .....(\text{B})
[Using equation (4)]
Now, \text{w}=\text{e}^{\text{x}^{\text{x}^{\text{e}}}}\ .....(\text{v})
Taking log on sides,
\log\text{w}=\log\text{e}^{\text{x}^{\text{x}^{\text{e}}}}
\Rightarrow\log\text{w}=\text{x}^{\text{x}^\text{e}}\log\text{e}
\Rightarrow\log\text{w}=\text{x}^{\text{x}^{\text{e}}}\ .....(\text{vi})
Taking log on both sides,
\log\log\text{w}=\log\text{x}^{\text{x}^{\text{e}}}
\Rightarrow\log\log\text{w}=\text{x}^{\text{e}}\log\text{x}
\Rightarrow\frac{1}{\log\text{w}}\frac{\text{d}}{\text{dx}} (\log\text{w})=\text{x}^\text{e}\frac{\text{d}}{\text{dx}}(\log\text{x})+\log\text{x}\frac{\text{d}}{\text{dx}}(\text{x}^\text{e})
\Rightarrow\frac{1}{\log\text{w}}\big(\frac{1}{\text{w}}\big)\frac{\text{dw}}{\text{dx}}=\text{x}^{\text{e}}\big(\frac{1}{\text{x}}\big)\log\text{xex}^{\text{e}-1}
\Rightarrow\frac{\text{dw}}{\text{dx}}=\text{w}\log\text{w}\big[\text{x}^{\text{e}-1}+\text{e}\log\text{xx}^{\text{e}-1}\big]
\Rightarrow\frac{\text{dw}}{\text{dx}}=\text{e}^{\text{x}^{\text{x}^\text{e}}}\text{x}^{\text{x}^\text{e}}\text{x}^{\text{e}-1}(1+\text{e}\log\text{x})\ .....(\text{C})
Using equation (A), (B) and (C) in equation (i), we get
\frac{\text{dy}}{\text{dx}}=\text{e}^{\text{x}^{\text{e}^{\text{x}}}}\times\text{x}^{\text{e}^\text{x}}\Big[\frac{\text{e}^\text{x}}{\text{x}}+\text{e}^\text{x}\log\text{x}\Big] +
\text{e}^{\text{e}^{\text{e}^\text{x}}}\times\text{e}^{\text{e}^\text{x}}\Big[\frac{1}{\text{x}}+\text{e}^\text{x}\log\text{x}\Big] +
\text{e}^{\text{x}^{\text{x}^\text{e}}}\text{x}^{\text{x}^\text{e}}\text{x}^{\text{e}-1}(1+\text{e}\log\text{x})
Q468. Verify mean value theorem for the function: 4 Marks
\text{f(x)}=\sin\text{x}-\sin2\text{x in }[0,\pi].

Ans: We have, \text{f(x)}=\sin\text{x}-\sin2\text{x in }[0,\pi]


1. Since, we know that sine functions are continuous functions hence
\text{f(x)}=\sin\text{x}-\sin2\text{x} is a continuous function in [0,\pi].
2. \text{f}'(\text{x})=\cos\text{x}-\cos2\text{x}.2=\cos\text{x}-2\cos2\text{x}, which exists in (0,\pi)
So, f(x) is differentiable in (0,\pi). Continuous of mean value theorem are satisfied.
Hence, \exists\text{ c}\in(0,\pi) such that, \text{f}'(\text{c})=\frac{\text{f}(\pi)-\text{f}(0)}{\pi-0}
\Rightarrow\ \cos\text{c}-2\cos2\text{c}=\frac{\sin\pi-\sin2\pi-\sin0+\sin2.0}{\pi-0}
\Rightarrow\ 2\cos2\text{c}-\cos\text{c}=\frac{0}{\pi}
\Rightarrow\ 2.(2\cos^2\text{c}-1)-\cos\text{c}=0
\Rightarrow\ 4\cos^2\text{c}-2-\cos\text{c}=0
\Rightarrow\ 4\cos^2\text{c}-\cos\text{c}-2=0
\Rightarrow\ \cos\text{c}=\frac{1\pm\sqrt{1+32}}{8}=\frac{1\pm\sqrt{33}}{8}
\therefore\ \text{c}=\cos^{-1}\Big(\frac{1\pm\sqrt{33}}{8}\Big)
Also, \cos^{-1}\Big(\frac{1\pm\sqrt{33}}{8}\Big)\in(0,\pi)
Hence, mean value theorem has been verified.
Q469. If \text{x}=\text{a}(1+\cos\theta),\text{y}=\text{a}(\theta+\sin\theta), prove that 4 Marks

Ans: Here,
\text{x}=\text{a}(1+\cos\theta),\text{y}=\text{a}(\theta+\sin\theta),
Differentiating w.r.t.x, we get
\frac{\text{dx}}{\text{d}\theta}=-\text{a}\sin\theta\ \text{and}\ \frac{\text{dy}}{\text{d}\theta}=\text{a}+\text{a}\cos\theta
\therefore\frac{\text{dy}}{\text{dx}}=\frac{\text{a}+\text{a}\cos\theta}{-\text{a}\sin\theta}=\frac{1+\cos\theta}{-\sin\theta}
Differentiating w.r.t.x, we get
\frac{\text{d}^2\text{y}}{\text{dx}^2}=\frac{\text{d}}{\text{d}\theta}\Big\{\frac{\text{dy}}{\text{dx}}\Big\}\frac{\text{d}\theta}{\text{dx}}
\frac{\text{d}^2\text{y}}{\text{dx}^2}=-\Big\{\frac{-\sin\theta\cos\theta-\cos^2\theta}{\sin^2\theta}\Big\}\frac{\text{d}\theta}{\text{dx}}
=\frac{1+\cos\theta}{\sin^2\theta}\times\frac{-1}{\sin^2\theta}
\frac{-(1+\cos\theta)}{\sin^3\theta}
At \theta=\frac{\pi}{2}:\frac{\text{d}^2\text{y}}{\text{dx}^2}=\frac{-(1+\cos\frac{\pi}{2})}{\text{a}(\sin\frac{\pi}{2})^3}=\frac{-1}{\text{a}}
Q470. Differentiate the following functions with respect to x: 4 Marks
\sin^{-1}\Big(\frac{1}{\sqrt{1+\text{x}^2}}\Big)

Ans: Let \text{f(x)}=\sin^{-1}\Big(\frac{2^{\text{x}+1}}{1+4^\text{x}}\Big)


To find the domain, we need to find all x such that
-1\leq\frac{2^{\text{x}+1}}{1+4^\text{x}}\leq1
Since the quantity in the middle is always psitive, we need to find all x such that \frac{2^{\text{x}+1}}{1+4^\text{x}}\leq1
i.e. all x such that 2^{\text{x}+1}\leq1+4^\text{x}
We may req. write as 2\leq\frac{1}{2^\text{x}}+2^\text{x}, which is true for all x
Hence, the function is defined at all real numbers.
Putting 2^\text{x}=\tan\theta
\text{f(x)}=\sin^{-1}\Big(\frac{2^{\text{x}+1}}{1+4^\text{x}}\Big)=\sin^{-1}\Big(\frac{2^\text{x}.2}{1+(2^\text{x})^2}\Big)

https://bls.smartstudies.co.in/#/exam/pdf-preview/c59cb220-8e86-4716-9ff7-82aec16b1ade/1 101/158
5/26/24, 6:19 PM Exam Automation
=\sin^{-1}\Big[\frac{2\tan\theta}{1+\tan^2\theta}\Big]=\sin^{-1}(\sin2\theta)=2\theta=2\tan^{-1}(2^\text{x})
Thus, \text{f(x)}=2\frac{1}{1+(2^\text{x})^2}\frac{\text{d}}{\text{dx}}(2^\text{x})
=\frac{2}{1+4^\text{x}}(2^\text{x})\log2=\frac{2^{\text{x}+1}\log2}{1+4^\text{x}}
Q471. Show that the function \text{f(x)}\begin{cases}\text{x}^\text{m}\sin(\frac{1}{\text{x}}), &\text{x}\neq0 \\0 ,& \text{x}=0\end{cases} 4 Marks
Differential at x = 0, if m > 1

Ans: Let m = 2, then the function \text{f(x)}=\begin{Bmatrix}\text{x}^2\sin(\frac{1}{\text{x}}) &\text{x}\neq0 \\0 & \text{x}=0 \end{Bmatrix}
Differentiability at x = 0:
\lim_\limits{\text{x}\rightarrow0}\frac{\text{f(x)}-\text{f(0)}}{\text{x}-0}=\lim_\limits{\text{x}\rightarrow0}\frac{\text{f(x)}}
{\text{x}}=\lim_\limits{\text{x}\rightarrow0}\text{x}\sin\Big(\frac{1}{\text{x}}\Big)=0.
\big[\therefore\lim_\limits{\text{x}\rightarrow0}\text{x}\sin\Big(\frac{1}{\text{x}}\Big)\text{x}=0, as
\begin{vmatrix}\text{x}\sin\frac{1}{\text{x}}-0\end{vmatrix}=\begin{vmatrix}\text{x}\sin\frac{1}{\text{x}} \end{vmatrix}=\begin{vmatrix}\text{x} \end{vmatrix}\begin{vmatrix}\sin\frac{1}
{\text{x}} \end{vmatrix} \leq\begin{vmatrix}\text{x}\end{vmatrix}
\because\ |\sin\theta|\leq1\ \text{for all }\theta
Hence, \begin{vmatrix}\text{x}\sin\frac{1}{\text{x}} \end{vmatrix}<0\ \text{when}\ |\text{x-0}|<\epsilon|\text{x}-0|<\epsilon\big]
Therefore, f'(x) = 0, which means f is differentiable at x = 0.
Hence the given function is differentiable at x = 0.
Q472. If \text{f(x)}=\sqrt{\text{x}^2+9}, Write the value of \lim\limits_{\text{x}\rightarrow4}\frac{\text{f(x)}-\text{f}(4)}{\text{x}-4}. 4 Marks

Ans: Given: \text{f(x)}=\text{x}^2+9


Now,
\text{f}(4)=\sqrt{16+9}
=\sqrt{25}
=5
So,
\frac{\text{f(x)}-\text{f}(4)}{\text{x}-4}=\frac{\sqrt{\text{x}^2+9-5}}{\text{x}-4}
On rationalising the numeratore, we get
\frac{\text{f(x)}-\text{f}(4)}{\text{x}-4}
=\frac{\sqrt{\text{x}^2+9-5}}{\text{x}-4}\times\frac{\sqrt{\text{x}^2+9}+5}{\sqrt{\text{x}^2+9}+5}
=\frac{\text{x}^2+9-25}{(\text{x}-4)(\sqrt{\text{x}^2+9+5})}
=\frac{\text{x}^2-16}{(\text{x}-4)(\sqrt{\text{x}^2+9+5})}
=\frac{\text{x}+4}{\sqrt{\text{x}^2+9+5}}
Taking limit x → 4, we have
\lim\limits_{\text{x}\rightarrow4}\frac{\text{f(x)}-\text{f}(4)}{\text{x}-4}=\lim\limits_{\text{x}\rightarrow4}\frac{\text{x}+4}{\sqrt{\text{x}^2+9+5}}
=\frac{8}{10}
=\frac{4}{5}
Q473. If \text{y}=(\cos\text{x})^{\cos\text{x}^{\cos\text{x}^{.....\infty}}}, prove that \frac{\text{dy}}{\text{dx}}=-\frac{\text{y}^2\tan\text{x}}{(1-\text{y}\log\cos\text{x})} 4 Marks

Ans: Here,
\text{y}=(\cos\text{x})^{\cos\text{x}^{\cos\text{x}^{.....\infty}}}
\text{y}=(\cos\text{x})^\text{y}
Taking log on both the sides,
\log\text{y}=\log(\cos\text{x})^\text{y}
\log\text{y}=\text{y}\log(\cos\text{x}),\big\{\text{Since},\log\text{a}^\text{b}=\text{b}\log\text{a}\big\}
Differentiating it with resepect to x using product rule and chain rule,
\frac{1}{\text{y}}\frac{\text{dy}}{\text{dx}}=\text{y}\frac{\text{d}}{\text{dx}}\log(\cos\text{x})+\log\cos\text{x}\frac{\text{dy}}{\text{dx}}
\frac{1}{\text{y}}\frac{\text{dy}}{\text{dx}}=\text{y}\Big(\frac{1}{\cos\text{x}}\Big)\frac{\text{d}}{\text{dx}}(\cos\text{x})+\log\cos\text{x}\frac{\text{dy}}{\text{dx}}
\frac{\text{dy}}{\text{dx}}\Big(\frac{1}{\text{y}}-\log\cos\text{x}\Big)=\frac{\text{y}}{\cos\text{x}}(-\sin\text{x})
\frac{\text{dy}}{\text{dx}}\Big(\frac{1-\log\cos\text{x}}{\text{y}}\Big)=-\text{y}\tan\text{x}
\frac{\text{dy}}{\text{dx}}=\frac{\text{y}^2\tan\text{x}}{(1-\log\cos\text{x})}
Q474. Find \frac{\text{dy}}{\text{ dx}} in the following: 4 Marks
\text{y}=\sec^{-1}\bigg(\frac{1}{2\text{x}^{2}-1}\bigg), 0<\text{x}<\frac{1}{\sqrt{2}}

Ans: The given relationship is,\text{y}=\sec^{-1}\bigg(\frac{1}{2\text{x}^{2}-1}\bigg)


\text{y}=\sec^{-1}\bigg(\frac{1}{2\text{x}^{2}-1}\bigg)
\Rightarrow\sec\text{y}=\frac{1}{2\text{x}^{2}-1}
\Rightarrow\cos\text{y}=2\text{x}^{2}-1
\Rightarrow2\text{x}^{2}=1+\cos\text{y}
\Rightarrow2\text{x}^{2}=2\cos^{2}\frac{\text{y}}{2}
\Rightarrow\text{x}=\cos\frac{\text{y}}{2}
Differentiating this relationship with respect to x, we obtain
\frac{\text{d}}{\text{dx}}(\text{x})=\frac{\text{d}}{\text{dx}}\bigg(\cos\frac{\text{y}}{2}\bigg)
\Rightarrow1=-\sin\frac{\text{y}}{2}.\frac{\text{d}}{\text{dx}}\bigg(\frac{\text{y}}{2}\bigg)
\Rightarrow \frac{-1}{\sin\frac{\text{y}}{2}}=\frac{1}{2}\frac{\text{dy}}{\text{dx}}
\Rightarrow\frac{\text{dy}}{\text{dx}}=\frac{-2}{\sin\frac{\text{y}}{2}}=\frac{-2}{\sqrt{1-\cos^{2}\frac{\text{y}}{2}}}
\Rightarrow\frac{\text{dy}}{\text{dx}}=\frac{-2}{\sqrt{1-\text{x}^{2}}}
Q475. \text{If} x = \text{a } \sin \text{ 2t}(1 + \cos\text{2t}) \text{ and }y = \text{b}\cos\text{2t}(1- \cos \text{2t}), \text{find }\frac{dy}{dx} \text{ at t} = \frac{\pi}{4} 4 Marks

Ans: \frac{\text{dx}}{\text{dt}} = \text{2a } \cos \text{2t}(1 + \cos \text{2t}) - \text{2a } \sin \text{ 2t} \sin \text{ 2t}
\frac{\text{dy}}{\text{dt}} = \text{-2b} \sin \text{2t} (1 - \cos \text{2t}) + \text{2b} \cos \text{ 2t} \sin \text{ 2t}
\frac{\text{dy}}{\text{dx}}\bigg]_{\text{t} = \frac{\pi}{4}} = \frac{\text{2b}\cos \text{2t}.\sin \text{2t - 2b}\sin \text{2t}(1 - \cos \text{2t})}{\text{2a } \cos \text{2t}(1 + \cos \text{2t}) - \text{2a
} \sin \text{ 2t} \sin \text{ 2t}}\bigg]_{\text{t} = \frac{\pi}{4}} = \frac{\text{b}}{\text{a}}
Q476. If \text{x}^\text{y}-\text{y}^\text{x}=\text{a}^\text{b}, find =\frac{\text{dy}}{\text{dx}}. 4 Marks

Ans: The given function is \text{x}^\text{y}-\text{y}^x=\text{a}^\text{b}


Let \text{x}^\text{y}=\text{u} and \text{y}^\text{x}=\text{v}
Then, the function becomes \text{u}-\text{v}=\text{a}^\text{b}
\frac{\text{du}}{\text{dx}}-\frac{\text{dv}}{\text{dx}}=0\ \dots(1)
\text{u}=\text{x}^\text{y}
\Rightarrow\log\text{u}=\log(\text{x}^\text{y})
\Rightarrow\log\text{u}=\text{y}\log\text{x}
Differentiating both sides with respect to x, we obtain
\frac{1}{\text{u}}.\frac{\text{du}}{\text{dx}}=\log\text{x}\frac{\text{dy}}{\text{dx}}+\text{y}.\frac{\text{d}}{\text{dx}}(\log\text{x})
\Rightarrow\frac{\text{du}}{\text{dx}}=\text{u}\Big[\log\text{x}\frac{\text{dy}}{\text{dx}}+\text{y}.\frac{1}{\text{x}}\Big]
\Rightarrow\frac{\text{du}}{\text{dx}}=\text{x}^\text{y}\Big(\log\text{x}\frac{\text{dy}}{\text{dx}}+\frac{\text{y}}{\text{x}}\Big)\ \dots(2)
\text{v}=\text{y}^\text{x}
\Rightarrow\log\text{v}=\log(\text{y}^\text{x})
\Rightarrow\log\text{v}=\text{x}\log\text{y}
Differentiating both sides with respect to x, we obtain

https://bls.smartstudies.co.in/#/exam/pdf-preview/c59cb220-8e86-4716-9ff7-82aec16b1ade/1 102/158
5/26/24, 6:19 PM Exam Automation
\frac{1}{\text{v}}.\frac{\text{dv}}{\text{dx}}=\log\text{y}.\frac{\text{dy}}{\text{dx}}(\text{x})+\text{x}.\frac{\text{d}}{\text{dx}}(\log\text{y})
\Rightarrow\frac{\text{dv}}{\text{dx}}=\text{v}\Big(\log\text{y}.1+\text{x}.\frac{1}{\text{y}}.\frac{\text{dy}}{\text{dx}}\Big)
\Rightarrow\frac{\text{dv}}{\text{dx}}=\text{y}^\text{x}\Big(\log\text{y}+\frac{\text{x}}{\text{y}}\frac{\text{dy}}{\text{dx}}\Big)\ \dots(3)
From (1), (2) and (3), we obtain
\text{x}^\text{y}\Big(\log\text{x}\frac{\text{dy}}{\text{dx}}+\frac{\text{y}}{\text{x}}\Big)-\text{y}^\text{x}\Big(\log\text{y}+\frac{\text{x}}{\text{y}}\frac{\text{dy}}{\text{dx}}\Big)=0
\Rightarrow\text{x}^\text{y}\log\text{x}\frac{\text{dy}}{\text{dx}}-\text{x}\text{y}^{\text{x}-1}\frac{\text{dy}}{\text{dx}}+\text{x}^{\text{y}-1}\text{y}-\text{y}^\text{x}\log\text{y}=0
\Rightarrow(\text{x}^\text{y}\log\text{x}-\text{xy}^{\text{x}-1})\frac{\text{dy}}{\text{dx}}=\text{y}^\text{x}\log\text{y}-\text{x}^{\text{y}-1}\text{y}
\Rightarrow\frac{\text{dy}}{\text{dx}}=\frac{\text{y}^\text{x}\log\text{y}-\text{x}^{\text{y}-1}\text{y}}{(\text{x}^\text{y}\log\text{x}-\text{xy}^{\text{x}-1})}
Q477. If y = \log\bigg[\text{x+}\sqrt{\text{x}^{2} + \text{a}^{2}}\bigg],\text {show that } (\text{x}^{2} + \text{a}^{2})\frac{\text{d}^{2}\text{y}}{\text{dx}^{2}} + \text{x}\frac{\text{dy}}{\text{dx}} = 0. 4 Marks

Ans: Given y = \log\bigg[\text{x} +\sqrt{\text{x}^{2} + \text{a}^{2}}\bigg]


\Rightarrow\frac{\text{dy}}{\text{dx}} =\frac{1}{\text{x} + \sqrt{\text{x}^{2} + \text{a}^{2}}}.\bigg[1+ \frac{2\text{x}}{2\sqrt{\text{x}^{2} + \text{a}^{2}}}\bigg]\Rightarrow\frac{\text{dy}}
{\text{dx}} = \frac{\text{x} + \sqrt{\text{x}^{2} + \text{a}^{2}}}{\bigg(\text{x} + \sqrt{\text{x}^{2} + \text{a}^{2}}\bigg)\bigg(\sqrt{\text{x}^{2} + \text{a}^{2}}\bigg)}
\Rightarrow\frac{\text{dy}}{\text{dx}} = \frac{1}{\sqrt{\text{x}^{2} + \text{a}^{2}}}
Differentiating again w.r.t. x we get
\frac{\text{d}^{2}\text{y}}{\text{dx}^{2}} = -\frac{1}{2}(\text{x}^{2} + \text{a}^{2})^{-\frac{3}{2}}.2\text{x} = \frac{-\text{x}}{(\text{x}^{2} + \text{a}^{2})^{\frac{3}{2}}}
\Rightarrow\frac{\text{d}^{2}\text{y}}{\text{dx}^{2}} =\frac{-\text{x}}{(\text{x}^{2} + \text{a}^{2}).\sqrt{\text{x}^{2} +\text{a}^{2}}}\Rightarrow(\text{x}^{2} +
\text{a}^{2})\frac{\text{d}^{2}\text{y}}{\text{dx}^{2}} = -\frac{\text{x}}{\sqrt{\text{x}^{2} + \text{a}^{2}}}
\Rightarrow(\text{x}^{2} + \text{a}^{2}) \frac{\text{d}^{2}\text{y}}{\text{dx}^{2}} +\text{x}.\frac{\text{dy}}{\text{dx}} = 0 . [from (i)].
Q478. Find \frac{\text{dy}}{\text{dx}} of the functions given in Exercise: 4 Marks
(\cos\text{x})^\text{y}=(\cos\text{y})^\text{x}

Ans: Given: (\cos\text{x})^\text{y}=(\cos\text{y})^\text{x}\ \Rightarrow\ \log(\cos\text{x})^\text{y}=\log(\cos\text{y})^\text{x}


\Rightarrow\ \text{y}\log\cos\text{x}=\text{x}\log\cos\text{y}\ \Rightarrow\ \frac{\text{d}}{\text{dx}}(\text{y}\log\cos\text{x})=\frac{\text{d}}{\text{dx}}(\text{x}\log\cos\text{y})
\Rightarrow\ \text{y}\frac{\text{d}}{\text{dx}}\log\cos\text{x}+\log\cos\text{x}\frac{\text{dy}}{\text{dx}}=\text{x}\frac{\text{d}}{\text{dx}}\log\cos\text{y}+\log\cos\text{y}\frac{\text{d}}
{\text{dx}}\text{x}
\Rightarrow\ \text{y}\frac{1}{\cos\text{x}}\frac{\text{d}}{\text{dx}}\cos\text{x}+\log\cos\text{x}\frac{\text{dy}}{\text{dx}}=\text{x}\frac{1}{\cos\text{y}}\frac{\text{d}}
{\text{dx}}\cos\text{y}+\log\cos\text{y}
\Rightarrow\ \text{y}\frac{1}{\cos\text{x}}(-\sin\text{x})+\log\cos\text{x}\frac{\text{dy}}{\text{dx}}=\text{x}\frac{1}{\cos\text{y}}\Big(-\sin\text{y}\frac{\text{dy}}
{\text{dx}}\Big)+\log\cos\text{y}
\Rightarrow\ -\text{y}\tan\text{x}+\log\cos\text{x}.\frac{\text{dy}}{\text{dx}}=-\text{x}\tan\text{y}.\frac{\text{dy}}{\text{dx}}+\log\cos\text{y}
\Rightarrow\ \text{x}\tan\text{y}\frac{\text{dy}}{\text{dx}}+\log\cos\text{x}.\frac{\text{dy}}{\text{dx}}=\text{y}\tan\text{x}+\log\cos\text{y}
\Rightarrow\ \frac{\text{dy}}{\text{dx}}(\text{x}\tan\text{y}+\log\cos\text{x})=\text{y}\tan\text{x}+\log\cos\text{y}
\Rightarrow\ \frac{\text{dy}}{\text{dx}}=\frac{\text{y}\tan\text{x}+\log\cos\text{y}}{\text{x}\tan\text{y}+\log\cos\text{x}}
Q479. Find the value of a and b so that the function f given by \text{f(x)}=\begin{cases}1,&\text{if }\text{ x}\leq3\\\text{ax}+\text{b},&\text{if }3<\text{x}<5\\7,&\text{if }\text{ 4 Marks
x}\geq5\end{cases} is continuous x = 3 and x = 5.

Ans: Given,
\text{f(x)}=\begin{cases}1,&\text{if }\text{ x}\leq3\\\text{ax}+\text{b},&\text{if }3<\text{x}<5\\7,&\text{if }\text{ x}\geq5\end{cases}
We have,
(\text{LHL at x}= 3)=\lim_\limits{\text{x}\rightarrow3^-}\text{f(x)}=\lim_\limits{\text{h}\rightarrow0}\text{f}(3-\text{h})
=\lim_\limits{\text{h}\rightarrow0}\text{f}(3-\text{h})=\lim_\limits{\text{h}\rightarrow0}(1)=1
(\text{RHL at x}= 3)=\lim_\limits{\text{x}\rightarrow3+}\text{f(x)}=\lim_\limits{\text{h}\rightarrow0}\text{f}(3+\text{h})
=\lim_\limits{\text{h}\rightarrow0}\text{a}(3+\text{h})+\text{b}=3\text{a}+\text{b}
(\text{LHL at x}= 5)=\lim_\limits{\text{x}\rightarrow5^-}\text{f(x)}=\lim_\limits{\text{h}\rightarrow0}\text{f}(5-\text{h})
=\lim_\limits{\text{h}\rightarrow0}(\text{a}(5-\text{h})+\text{b})=5\text{a}+\text{b}
(\text{RHL at x}= 5)=\lim_\limits{\text{x}\rightarrow5+}\text{f(x)}=\lim_\limits{\text{h}\rightarrow0}\text{f}(5+\text{h})
=\lim_\limits{\text{h}\rightarrow0}7=7
If f(x) is continuous at x = 3 and 5, then
\therefore\
\lim_\limits{\text{x}\rightarrow3^-}\text{f(x)}=\lim_\limits{\text{x}\rightarrow3^+}\text{f(x)} and \lim_\limits{\text{x}\rightarrow5^-}\text{f(x)}=\lim_\limits{\text{x}\rightarrow5^+}\text{f(x)}
\Rightarrow1=3\text{a}+\text{b}\ .... (\text{i}) and 5\text{a}+\text{b}=7\ .... (\text{ii})
On solving eqs. (i) and (ii) we get
\text{a}=3 and \text{b}=-8
Q480. In the following, find the value of the constant k so that the given function is continuous at the indicated point: 4 Marks
\text{f(x)}=\begin{cases}(\text{x}-1)\tan\frac{\pi\text{x}}{2},&\text{if}\text{ x}\neq1\\\text{k},&\text{if}\text{ x}=1\end{cases}\text{at x} = 1

Ans: Let \text{x}-1=\text{y}


\Rightarrow\text{x}=\text{y}+1
Thus,
\lim_\limits{\text{x}\rightarrow 1}(\text{x}-1)\tan\frac{\pi\text{x}}{2}=\lim_\limits{\text{y}\rightarrow0}\text{y}\tan\frac{\pi(\text{y}+1)}{2}
=\lim_\limits{\text{y}\rightarrow0}\text{y}\tan\Big(\frac{\pi\text{y}}{2}+\frac{\pi}{2}\Big)
=-\lim_\limits{\text{y}\rightarrow0}\text{y}\cot\frac{\pi\text{y}}{2}
=-\lim_\limits{\text{y}\rightarrow0}\text{y}\frac{\cot\frac{\pi\text{y}}{2}}{\sin\frac{\pi\text{y}}{2}}
=-\lim_\limits{\text{y}\rightarrow0}\text{y}\frac{\cot\frac{\pi\text{y}}{2}}{\frac{\Big(\sin\frac{\pi\text{y}}{2}\Big)\frac{\pi}{2}}{\frac{\pi}{2}}}
=-\lim_\limits{\text{y}\rightarrow0}\frac{\cot\frac{\pi\text{y}}{2}}{\frac{\Big(\sin\frac{\pi\text{y}}{2}\Big)\frac{\pi}{2}}{\frac{\pi\text{y}}{2}}}
=-\lim_\limits{\text{y}\rightarrow0}\frac{2}{\pi}\frac{\cot\frac{\pi\text{y}}{2}}{\frac{\Big(\sin\frac{\pi\text{y}}{2}\Big)\frac{\pi}{2}}{\frac{\pi\text{y}}{2}}}
=-\frac{2}{\pi}\lim_\limits{\text{y}\rightarrow0}\cos\frac{\pi\text{y}}{2}
=-\frac{2}{\pi}
Since the function is continuous, LHL = RHL.
Thus, \text{k}=-\frac{2}{\pi}
Q481. Find which of the function: 4 Marks
\text{f(x)}=\begin{cases}|\text{x}|\cos\frac{1}{\text{x}},&\text{if x}\neq0\\0,&\text{if x}=0\end{cases}
at x = 0

Ans: We have, \text{f(x)}=\begin{cases}|\text{x}|\cos\frac{1}{\text{x}},&\text{if x}\neq0\\0,&\text{if x}=0\end{cases} at x = 0


At x = 0, \text{L.H.L}=\lim\limits_{\text{h}\rightarrow0}|\text{x}|\cos\frac{1}{\text{x}}
=\lim\limits_{\text{h}\rightarrow0}|0-\text{h}|\cos\frac{1}{0-\text{h}}
=\lim\limits_{\text{h}\rightarrow0}\text{h}\cos\frac{1}{\text{h}}
= 0 × [an oscillating number between -1 and 1] = 0
\text{R.H.L}=\lim\limits_{\text{x}\rightarrow0^+}|\text{x}|\cos\frac{1}{\text{x}}
=\lim\limits_{\text{h}\rightarrow0}|0+\text{h}|\cos\frac{1}{0+\text{h}}
=\lim\limits_{\text{h}\rightarrow0}\text{h}\cos\frac{1}{\text{h}}
= 0 × [an oscillating number between -1 and 1] = 0
Also f(0) = 0 (given)
Thus, L.H.L = R.H.L = f(0)
Hence, f(x) is discontinuous at x = 0.
Q482. Differentiate the following with respect to x: 4 Marks

https://bls.smartstudies.co.in/#/exam/pdf-preview/c59cb220-8e86-4716-9ff7-82aec16b1ade/1 103/158
5/26/24, 6:19 PM Exam Automation
\cos^{-1}\Big(\frac{1-\text{x}}{1+\text{x}}\Big)

Ans: Let \text{y}=\cot^{-1}\Big(\frac{1-\text{x}}{1+\text{x}}\Big)


Put \text{x}=\tan\theta,\text{ So}
\text{y}=\cot^{-1}\Big(\frac{1-\tan\theta}{1+\tan\theta}\Big)
=\cot^{-1}\bigg(\frac{\tan\frac{\pi}{4}-\tan\theta}{1+\tan\frac{\pi}{4}\tan\theta}\bigg)
=\cot^{-1}\Big[\tan\Big(\frac{\pi}{4}-\theta\Big)\Big]
=\cot^{-1}\Big[\cot\Big(\frac{\pi}{2}-\frac{\pi}{4}+\theta\Big)\Big]
=\frac{\pi}{4}+\theta
\text{y}=\frac{\pi}{4}+\tan^{-1}\text{x}\ \big[\text{Since x}=\tan\theta\big]
Differentiating it with respect do x,
\frac{\text{dy}}{\text{dx}}=0+\frac{1}{1+\text{x}^2}
\frac{\text{dy}}{\text{dx}}=\frac{1}{1+\text{x}^2}
Q483. Prove that: \int\limits_0^\text{a}\text{f(x)}\text{dx}=\int\limits_0^\text{a}\text{f}(\text{a}-\text{x})\text{dx} 4 Marks
and hence evaluate \int\limits_0^{\frac{\pi}{2}}\frac{\text{x}}{\sin\text{x}+\cos\text{x}}\text{dx}

Ans: \int\limits_0^\text{a}\text{f(x)}\text{dx}=\int\limits_0^\text{a}\text{f}(\text{a}-\text{x})\text{dx}
put \text{x}=\text{a}-\text{t},\ \text{dx}=-\text{dt}
Upper limit t = a - x = a - a = 0
Lower limit = t = a - x = a - 0 = a
=\int\limits_0^\text{a}\text{f}(\text{a}-\text{t})\text{dt}=\int\limits_0^\text{a}\text{f}(\text{a}-\text{x})\text{dx}
Let \text{I}=\int\limits_0^{\frac{\pi}{2}}\frac{\text{x}}{\sin\text{x}+\cos\text{x}}\text{dx}\ \dots(\text{i})
\Rightarrow\text{I}=\int\limits_0^{\frac{\pi}{2}}\frac{\frac{\pi}{2}-\text{x}}{\sin\Big(\frac{\pi}{2}-\text{x}\Big)+\cos\Big(\frac{\pi}{2}-\text{x}\Big)}\text{dx}\
\dots(\text{ii}) \therefore\text{I}=\int\limits_0^{\frac{\pi}{2}}\frac{\frac{\pi}{2}-\text{x}}{\cos\text{x}\sin\text{x}}\text{dx}
Adding, (i) and (ii) we get
2\text{I}=\frac{\pi}{2}\int\limits_0^{\frac{\pi}{2}}\frac{\text{x}}{\sin\text{x}+\cos\text{x}}\text{dx}=\frac{\pi}{2\sqrt{2}}\int\limits_0^{\frac{\pi}{2}}\frac{1}{\cos\Big(\text{x}-\frac{\pi}
{4}\Big)}\text{dx}
=\frac{\pi}{2\sqrt{2}}\int\limits_0^{\frac{\pi}{2}}\sec\Big(\text{x}-\frac{\pi}{4}\Big)\text{dx}
\Rightarrow2\text{I}=\frac{\pi}{2\sqrt{2}}\Bigg\{\log\bigg|\sec\Big(\text{x}-\frac{\pi}{4}\Big)+\tan\Big(\text{x}-\frac{\pi}{4}\Big)\bigg|\Bigg\}_0^{\frac{\pi}{2}}
=\frac{\pi}{2\sqrt{2}}\Big\{\log\big(\sqrt{2}+1\big)-\log\big(\sqrt{2}-1\Big\} or \frac{\pi}{4\sqrt{2}}\log\Big(\frac{\sqrt{2}+1}{\sqrt{2}-1}\Big)
Q484. Find the points on the curve y = x3 - 3x, where the tangent to the curve is parallel to the chord joining (1, -2) and (2, 2). 4 Marks

Ans: Here,
y = x3 − 3x
y is polynomial function, so it is continuous and differentiable, so Lagrange's mean value theorem is applicable thus there exists a point c such that,
\text{f}'(\text{c})=\frac{\text{f}(\text{b})-\text{f}(\text{a})}{\text{b}-\text{a}}
\Rightarrow3\text{c}^2-3=\frac{\text{f}(2)-\text{f}(1)}{2-1}
\Rightarrow3\text{c}^2-3=\frac{2+2}{1}
\Rightarrow3\text{c}^2=7
\Rightarrow\text{c}=\pm\sqrt{\frac{7}{3}}
\Rightarrow\text{y}=\pm\frac{2}{3}\sqrt{\frac{7}{3}}
So, (\text{c},\text{y})=\Big(\pm\sqrt{\frac{7}{3}},\pm\frac{2}{3}\sqrt{\frac{7}{3}}\Big) is the required point.
Q485. Verify Rolle's theorem of the following function on the indicated interval 4 Marks
\text{f}(\text{x})=\log(\text{x}^2+2)-\log3\text{ on }[-1,1]

Ans: The given function is \text{f}(\text{x})=\log(\text{x}^2+2)-\log3, which can be rewritten as


\text{f}(\text{x})=\log\Big(\frac{\text{x}^2+2}{3}\Big)
We know that, logarithmic function is differentiable and so continuous in its domain \text{f}(\text{x})=\log\Big(\frac{\text{x}^2+2}{3}\Big) is continuous is [-1,1] and differentiable is (-1,1).
Also,
f(1) = f(-1) = 0
Thus, f(x) satisfies all the conditions of Rolle's theorem.
Now, we have show that there must exists \text{c}\in(-1,1) such that f'(c) = 0.
We have,
\text{f}(\text{x})=\log\Big(\frac{\text{x}^2+2}{3}\Big)
\text{f}'(\text{x})=\frac{3(2\text{x})}{\text{x}^2+2}=\frac{6\text{x}}{\text{x}^2+2}
\therefore\ \text{f}'(\text{x})=0
\Rightarrow\frac{6\text{x}}{\text{x}^2+2}=0
\Rightarrow\text{x}=0
Thus, \text{c}=0\in(-1,1) such that f'(c) = 0.
Hence, Rolle's theorem is verified.
Q486. \text{If y = }\sqrt{\text{x}}+\frac{1}{\sqrt{\text{x}}},\text{ show that }\text{2x}\frac{\text{dy}}{\text{dx}}+\text{y}=2\sqrt{\text{x}}. 4 Marks

Ans: Getting \frac{\text{dy}}{\text{dx}}=\frac{1}{2\sqrt{\text{x}}}-\frac{1}{\text{2x}\sqrt{\text{x}}}


\therefore\text{ 2x}\frac{\text{dy}}{\text{dx}}=\sqrt{\text{x}}-\frac{1}{\sqrt{\text{x}}}
\text{ 2x}\frac{\text{dy}}{\text{dx}}+\text{y}=\Bigg(\sqrt{\text{x}}-\frac{1}{\sqrt{\text{x}}}\Bigg)+\Bigg(\sqrt{\text{x}}+\frac{1}{\sqrt{\text{x}}}\Bigg)=2\sqrt{\text{x}}.
Q487. Examine that sin |x| is a continuous function. 4 Marks

Ans: It is given function is \text{f(x)} = \sin|\text{x}|


The given function f is defined for real number and f can be written as the composition of two functions, as
f = goh, where, \text{g(x}) =| \text{x}|\ \text {and}\ \text{h(x)} = \sin\text{x}
First we have to prove that \text{g(x}) =| \text{x}|\ \text {and}\ \text{h(x)} = \sin\text{x} are continuous functions.
g(x) = lxl can be written as
\text{g(x)}=\begin{cases}-\text{x},&\text{if}\ \text{x}<{0}\\\text{x},& \text{if}\ \text{x}\geq0\end{cases}
Now, g is defined for all real number.
Let k be a real number.
Case I: If k < 0,
Then g(k) = -k
And ^{\ \ \text{lim}}_{\text{x}\rightarrow\text{k}}\text{g(x)} = ^{\ \ \text{lim}}_{\text{x}\rightarrow\text{k}}(-\text{x}) = -\text{k}
Thus ^{\ \ \text{lim}}_{\text{x}\rightarrow\text{k}}\text{g(x)} =\text{g(k)}
Therefore, g is continuous at all points x, i.e. x > 0
Case II: If k > 0,
Then g(k) = k and
^{\ \ \text{lim}}_{\text{x}\rightarrow\text{k}}\text{g(x)} =^{\ \ \text{lim}}_{\text{x}\rightarrow\text{k}}\text{x}=\text{k}
Thus ^{\ \ \text{lim}}_{\text{x}\rightarrow\text{k}}\text{g(x)} =\text{g(k)}
Therefore, g is continuous at all points x, i.e. x < 0
Case III: If k = 0,
Then, g(k) = g(0) = 0
^{\ \ \text{lim}}_{\text{x}\rightarrow\text{0}^{-}}\text{g(x)} = ^{\ \ \text{lim}}_{\text{x}\rightarrow\text{0}^{-}}(-\text{x}) = 0
^{\ \ \text{lim}}_{\text{x}\rightarrow\text{0}^{+}}\text{g(x)} = ^{\ \ \text{lim}}_{\text{x}\rightarrow\text{0}^{+}}(\text{x}) = 0

https://bls.smartstudies.co.in/#/exam/pdf-preview/c59cb220-8e86-4716-9ff7-82aec16b1ade/1 104/158
5/26/24, 6:19 PM Exam Automation
\therefore^{\ \ \text{lim}}_{\text{x}\rightarrow\text{0}^{-}}\text{g(x)} = ^{\ \ \text{lim}}_{\text{x}\rightarrow\text{0}^{+}}\text g({\text x}) =\text{g}( 0)
Therefore, g is continuous at x = 0
From the above 3 cases, we get that g is continuous at all points.
h(x) = sinx
We know that h is defined for every real number.
Let k be a real number.
Now, put x = k + h
If x → k, then h → 0
^{\ \ \text{lim}}_{\text{x}\rightarrow\text{k}}\text{h(x)} =^{\ \ \text{lim}}_{\text{x}\rightarrow\text{k}}\sin\text{x}
= ^{\ \ \text{lim}}_{\text{h}\rightarrow\text{0}}\sin(\text{k}+\text{h})
= ^{\ \ \text{lim}}_{\text{h}\rightarrow\text{0}}[\sin\text{k}\cos\text{h} + \cos\text{k}\sin\text{h}]
= ^{\ \ \text{lim}}_{\text{h}\rightarrow\text{0}}\sin\text{k}\cos\text{h} +^{\ \ \text{lim}}_{\text{h}\rightarrow\text{0}}\cos\text{k}\sin\text{h}
= \sin\text{k}\cos0 + \cos\text{k}\sin0
=\sin \text{k}
\therefore\ ^{\ \ \text{lim}}_{\text{x}\rightarrow\text{k}}\text{h(x)} =\text{g(k)}
Thus, h(x) = cos x is continuous function.
We know that for real valued functions g and h, such that (goh) is defined at k, if g is continuous at k and if f is continuous at g(k),
Then (fog) is continuous at k.
Therefore, \text{ f(x)} = \text{(gof)(x)} = \text{g(h(x))} = \text{g}(\sin \text{x)}= |\sin\text{x}|is a continuous function.
Q488. Find which of the function: 4 Marks
\text{f(x)}=\begin{cases}|\text{x}-\text{a}|\sin\frac{1}{\text{x}},&\text{if x}\neq0\\0,&\text{if x }=\text{a}\end{cases}
at x = a

Ans: We have, \text{f(x)}=\begin{cases}|\text{x}-\text{a}|\sin\frac{1}{\text{x}},&\text{if x}\neq0\\0,&\text{if x }=\text{a}\end{cases} at x = a.


At x = a \text{L.H.L}=\lim\limits_{\text{h}\rightarrow\text{a}^-}|\text{x}-\text{a}|\sin\frac{1}{\text{x}-\text{a}}
=\lim\limits_{\text{h}\rightarrow0}|\text{a}-\text{h}-\text{a}|\sin\Big(\frac{1}{\text{a}-\text{h}-\text{a}}\Big)
=\lim\limits_{\text{h}\rightarrow0}-\text{h}\sin\Big(\frac{1}{\text{h}}\Big)\ [\because\sin(-\theta)=-\sin\theta]
= 0 × [an oscillating number between -1 and 1] = 0
\text{R.H.L}=\lim\limits_{\text{x}\rightarrow\text{a}^+}|\text{x}-\text{a}|\sin\Big(\frac{1}{\text{x}-\text{a}}\Big)
=\lim\limits_{\text{h}\rightarrow0}|\text{a}+\text{h}-\text{a}|\sin\Big(\frac{1}{\text{a}+\text{h}-\text{a}}\Big)
=\lim\limits_{\text{h}\rightarrow0}\text{h}\sin\frac{1}{\text{h}}
= 0 × [an oscillating number between -1 and 1] = 0
and f(a) = 0
\therefore L.H.L = R.H.L = f(a)
So, f(x) is continuous at x = a.
Q489. If \sin^2\text{y}+\cos\text{xy}=\text{k}, find \frac{\text{dy}}{\text{dx}} at \text{x}=1,\text{y}=\frac{\pi}{4} 4 Marks

Ans: Here, \text{e}^{\text{x}}+\text{e}^\text{y}=\text{e}^{\text{x}+\text{y}}


Differentiating with respect to x using chain rule,
\Rightarrow \frac{\text{d}}{\text{dx}}\big(\text{e}^{\text{x}}\big)+\frac{\text{d}}{\text{dx}}\text{e}^{\text{y}}=\frac{\text{d}}{\text{dx}}\big(\text{e}^{\text{x}+\text{y}}\big)
\Rightarrow \text{e}^\text{x}+\text{e}^{\text{y}}\frac{\text{dy}}{\text{dx}}=\text{e}^{\text{x}+\text{y}}\frac{\text{d}}{\text{dy}}(\text{a}+\text{y})
\Rightarrow \text{e}^{\text{x}}+\text{e}^\text{y}\frac{\text{dy}}{\text{dx}}=\text{e}^{\text{x}+\text{y}}\Big[1+\frac{\text{dy}}{\text{dx}}\Big]
\Rightarrow\text{e}^{\text{x}}\frac{\text{dy}}{\text{dx}}-\text{e}^{\text{x}+\text{y}}\frac{\text{dy}}{\text{dx}}=\text{e}^{\text{x}+\text{y}}-\text{e}^{\text{x}}
\Rightarrow\frac{\text{dy}}{\text{dx}}\big(\text{e}^{\text{y}}-\text{e}^{\text{x}+\text{y}}\big)=\text{e}^{\text{x}+\text{y}}-\text{e}^{\text{x}}
\Rightarrow \frac{\text{dy}}{\text{dx}}=\Big(\frac{\text{e}^\text{x}\times\text{e}^\text{y}-\text{e}^\text{x}}{\text{e}^\text{y}-\text{e}^\text{x}\times\text{e}^\text{y}}\Big)
\Rightarrow \frac{\text{dy}}{\text{dx}}=\frac{\text{e}^\text{x}\big(\text{e}^\text{y}-1\big)}{\text{e}^\text{y}\big({1-\text{e}}^\text{x}\big)}
\Rightarrow \frac{\text{dy}}{\text{dx}}=-\frac{\text{e}^\text{x}\big(\text{e}^\text{y}-1\big)}{\text{e}^\text{y}\big({\text{e}^\text{x}-1}\big)}
Q490. Find the value of k if the function: f (x) = \begin{matrix} kx^{2}& x\geq 1 & \\ 4 & x< 1 & \\ \end{matrix} \text{is continuous at x = 1} 4 Marks

Ans: \lim\limits_{ x \rightarrow 1^{-}} f (x) = \lim\limits_{ h \rightarrow{0}} k ( 1 + h)^{2} = k \dots\dots\dots\text{(i)}


f(1) = k \dots\dots\dots\dots\dots\text{(ii)}
\lim\limits_{ x \rightarrow 1^{-}} = 4 \dots\dots\dots\dots\text{(iii)}
\text{As f (x) is continuous at x} = 1 , \therefore k = 4
Q491. Differentiate the following functions with respect to x: 4 Marks
\tan^{-1}\Big(\frac{\sqrt{\text{x}}+\sqrt{\text{a}}}{1-\sqrt{\text{xa}}}\Big)

Ans: Let \text{y}=\tan^{-1}\Big(\frac{\sqrt{\text{x}}+\sqrt{\text{a}}}{1-\sqrt{\text{xa}}}\Big)


\text{y}=\tan^{-1}\sqrt{\text{x}}+\tan^{-1}\sqrt{\text{a}}
\Big[\text{Since},\tan^{-1}\text{x}+\tan^{-1}\text{y}=\tan^{-1}\frac{\text{x}+\text{y}}{1-\text{xy}}\Big]
Differentiating it with respect to x using chain rule,
\frac{\text{dy}}{\text{dx}}=\frac{\text{d}}{\text{dx}}(\tan^{-1}\sqrt{\text{x}})+\frac{\text{d}}{\text{dx}}(\tan^{-1}\sqrt{\text{a}})
=\frac{1}{1+\big(\sqrt{\text{x}}\big)^2}\times\frac{\text{d}}{\text{dx}}\big(\sqrt{\text{x}}\big)+0
=\Big(\frac{1}{1+\text{x}}\Big)\Big(\frac{1}{2\sqrt{\text{x}}}\Big)
\frac{\text{dy}}{\text{dx}}=\frac{1}{2\sqrt{\text{x}}(1+\text{x})}
Q492. If x = \sin \text{t} and \text{y} = \sin \text{pt,} prove that (1 - x^{2}) \frac{\text{d}^{2} \text{y}}{\text{dx}^{2}} - x \frac{\text{dy}}{\text{dx}} + \text{P}^{2}\text{y} = 0. 4 Marks

Ans: \text{x} = \sin \text{t} \Rightarrow \frac{\text{dx}}{\text{dt}} = \cos \text{t}


\text{} = \sin \text{pt} \Rightarrow \frac{\text{dy}}{\text{dt}} = \text{p} \cos \text{pt}
\frac{\text{dy}}{\text{dx}} = \frac{\text{p} \cos \text{pt}}{\cos \text{t}}
\frac{\text{d}^{2}\text{y}}{\text{dx}^{2}} = \frac{\cos \text{t} (-\text{p}^{2} \sin \text{pt)} - \text{p} \cos \text{pt} (-\sin \text{t})}{\cos^{2}\text{t}}. \frac{\text{dt}}{\text{dx}}
= \frac{\text{-p}^{2} \sin \text{pt} \cos \text{t} + \text{p} \cos \text{pt} \sin \text{t}}{\cos^{3} \text{t}}
\text{Now} (1 - \text{x}^{2}) \frac{\text{d}^{2}\text{y}}{\text{dx}^{2}} - \text{x} \frac{\text{dy}}{\text{dx}} + \text{p}^{2} \text{y} = 0 \Bigg[ \text{Substituting values of y,}\frac{\text{dy}}
{\text{dx}} \& \frac{\text{d}^{2}\text{y}}{\text{dx}^{2}}\Bigg]
Q493. Verify Lagrange's mean value theorem for the following function on the indicated intervals. find a point 'c' in the indicated interval as stated by the Lagrange's mean value theorem. 4 Marks
\text{f}(\text{x})=\text{x}+\frac{1}{\text{x}}\text{ on }[1,3]

Ans: Here,
\text{f}(\text{x})=\text{x}+\frac{1}{\text{x}}\text{ on }[1,3]
f(x) attains a unique value for each \text{x}\in[1,3], so it is continuous
\text{f}'(\text{x})=1-\frac{1}{\text{x}^2} is defined for each \text{x}\in(1,3)
⇒ f(x) is differentiable in (1,3), so Lagrange's mean value theorem is applicable, so there exist a point \text{c}\in(1,3) such that
\text{f}'(\text{c})=\frac{\text{f}(3)-\text{f}(1)}{3-1}
\Rightarrow1-\frac{1}{\text{c}^2}=\frac{\Big(3+\frac{1}{3}-(1+1)\Big)}{2}
\Rightarrow1-\frac{1}{\text{c}^2}=\frac{\frac{10}{3}-2}{2}
\Rightarrow1-\frac{1}{\text{c}^2}=\frac{4}{3\times2}
\Rightarrow1-\frac{2}{3}=\frac{1}{\text{c}^2}
\Rightarrow\text{c}^2=3
\text{c}=\sqrt{3}\in(1,3) such that \text{f}'(\text{c})=\frac{\text{f}(1)-\text{f}(0)}{1-0}

https://bls.smartstudies.co.in/#/exam/pdf-preview/c59cb220-8e86-4716-9ff7-82aec16b1ade/1 105/158
5/26/24, 6:19 PM Exam Automation
So, Lagrange's mean value theorem is verified.
Q494. In the following, determine the values of constants involved in the definition so that the given function is continuous: 4 Marks
\text{f(x)}=\begin{cases}\frac{\text{k}\cos\text{x}}{\pi-2\text{x}},&\text{x}<\frac{{\pi}}{2}\\3,&\text{x}=\frac{\pi}{2}\\\frac{3\tan\text{x}}{2\text{x}-\pi},&\text{x}>\frac{\pi}{2}\end{cases}

Ans: Since the f(x) function is continuous at \text{x}=\frac{\pi}{2} therefore


LHL of f(x) at \text{x}=\frac{\pi}{2} is
=\lim\limits_{\text{x}\rightarrow\frac{\pi}{2}^-}\text{f(x)}
=\lim\limits_{\text{h}\rightarrow0}\text{f}\Big(\text{h}-\frac{\pi}{2}\Big)
=\lim\limits_{\text{h}\rightarrow0}\frac{\text{k}\cos\text{f}\Big(\text{h}-\frac{\pi}{2}\Big)}{\pi-2\text{f}\Big(\text{h}-\frac{\pi}{2}\Big)}
=\lim\limits_{\text{h}\rightarrow0}\frac{\text{k}\sin\text{h}}{2\pi-2\text{h}}
=\frac{\text{k}}{2}\lim\limits_{\text{h}\rightarrow0}\frac{\sin(\pi-\text{h})}{(\pi-\text{h})}
=\frac{\text{k}}{2}
Again,
\text{f}\Big(\frac{\pi}{2}\Big)=3
Hence,
\text{LHL}=\text{f}\Big(\frac{\pi}{3}\Big)
\frac{\text{k}}{2}=3
\text{k}=6
Q495. Differentiate the following functions with respect to x: 4 Marks
\sqrt{\tan^{-1}\big(\frac{\text{x}}{2}\big)}

Ans: Let, \text{y}=\sqrt{\tan^{-1}\big(\frac{\text{x}}{2}\big)}


\Rightarrow\ \text{y}=\Big(\tan^{-1}\big(\frac{\text{x}}{2}\big)\Big)^\frac{1}{2}
Differentiate it with respect to x,
\frac{\text{dy}}{\text{dx}}=\frac{\text{d}}{\text{dx}}\Big(\tan^{-1}\big(\frac{\text{x}}{2}\big)\Big)^\frac{1}{2}
=\frac{1}{2}\Big(\tan^{-1}\frac{\text{x}}{2}\Big)^{\frac{1}{2}-1}\frac{\text{d}}{\text{dx}}\Big(\tan^{-1}\frac{\text{x}}{2}\Big)
=\frac{1}{2}\Big(\tan^{-1}\frac{\text{x}}{2}\Big)^{\frac{-1}{2}}\times\frac{1}{1+\big(\frac{\text{x}}{2}\big)^2}\times\frac{\text{d}}{\text{dx}}\big(\frac{\text{x}}{2}\big)
=\frac{4}{4\sqrt{\tan^{-1}\big(\frac{\text{x}}{2}\big)}\big(4+\text{x}^2\big)}
=\frac{1}{\big(4+\text{x}^2\big)\sqrt{\tan^{-1}\big(\frac{\text{x}}{2}\big)}}
So,
\frac{\text{d}}{\text{dx}}\bigg(\sqrt{\tan^{-1}\big(\frac{\text{x}}{2}\big)}\bigg)=\frac{1}{\big(4+\text{x}^2\big)\sqrt{\tan^{-1}\big(\frac{\text{x}}{2}\big)}}
Q496. If \text{x}=2\cos\text{t}-\cos2\text{t},\text{y}=2\sin\text{t}-\sin2\text{t}, find \frac{\text{d}^2\text{y}}{\text{dx}^2}\ \text{at}\ \text{t}=\frac{\pi}{2}. 4 Marks

Ans: Given,
\text{x}=2\cos\text{t}-\cos\text{2}\text{t}
\text{y}=2\sin\text{t}-\sin\text{2t}
Differentiating w.r.t. t,
\frac{\text{dy}}{\text{dx}}=2(-\sin\text{t})-2(-\sin\text{2t})
\Rightarrow\frac{\text{dy}}{\text{dt}}=2\cot-2\cos\text{2t}
Dividing both:
\frac{\text{dy}}{\text{dx}}=\frac{2(\cos\text{t}-\cos\text{2t})}{2(\sin\text{2t}-\sin\text{t})}
Differentiating w.r.t. t,
\Rightarrow\frac{\text{d}\frac{\text{dy}}{\text{dx}}}{\text{dt}}=\frac{(\sin\text{2t}-\sin\text{t})(-\sin\text{t}+2\sin\text{2t})-(\cos\text{t}-\cos\text{2t})(2\cos\text{2t}-\cos\text{t})}
{(\sin\text{2t}-\sin\text{t})^2}
Dividing:
\frac{\text{d}^2\text{y}}{\text{dx}^2}=\frac{(\sin\text{2t}-\sin\text{t})(2\sin\text{t}-\sin\text{t})-(\cos\text{t}-\cos\text{2t})(2\cos\text{2t}-\cos\text{t})}{2(\sin\text{2t}-\sin)^3}
Putting: \text{t}=\frac{\pi}{2}
\Rightarrow\frac{\text{d}^2\text{y}}{\text{dx}^2}=\frac{1+2}{-2}=-\frac{3}{2}
Q497. If \text{f}\text{(x)}=\begin{cases}\frac{1-\cos\text{x}}{\text {x}^2}, & \text{when} \text{ x}\neq 0\\1, & \text{when}\text{ x} = 0\end{cases} Show that f(x) is discontinuous at x = 0. 4 Marks

Ans: Given,
\text{f}\text{(x)}=\frac{1-\cos \text{x}}{\text{x}^2}, \text{when x}\neq0
\text{f}\text{(x)}=1, \text{when x}=0
consider
\lim\limits_{\text{x} \rightarrow 0}\text{f}\text{(x)}=\lim\limits_{\text{x} \rightarrow 0}\frac{1-\cos \text{x}}{\text{x}^2}
\lim\limits_{\text{x} \rightarrow 0}\text{f}\text{(x)}=\lim\limits_{\text{x} \rightarrow 0}\frac{2\sin^2\frac{\text{x}}{2}}{\text{x}^2}
\lim\limits_{\text{x} \rightarrow 0}\text{f}\text{(x)}=\lim\limits_{\text{x} \rightarrow 0}\frac{2\sin^2\frac{\text{x}}{2}}{\frac{4\text{x}^2}{4}}
\lim\limits_{\text{x} \rightarrow 0}\text{f}\text{(x)}=\lim\limits_{\text{x} \rightarrow 0}\frac{2\sin\frac{\text{x}^2}{2}}{\frac{4\text{x}^2}{2}}
\lim\limits_{\text{x} \rightarrow 0}\text{f}\text{(x)}=\frac{2}{4}\lim\limits_{\text{x} \rightarrow 0}\frac{\sin\frac{\text{x}}{2}}{\frac{\text{x}}{2}}
\lim\limits_{\text{x} \rightarrow 0}\text{f}\text{(x)}=\frac{1}{2}(1)
Given f(0) = 1
\therefore\lim\limits_{\text{x} \rightarrow 0}\text{f}\text{(x)}\neq\text{f}(0)
Thus, f(x) is discontinuous at x = 0.
Q498. Find: \int(\sin\text{x}.\sin2\text{x}.\sin3\text{x})\text{dx} 4 Marks

Ans: \int(\sin\text{x}\sin\ 2\ \text{x}\sin\ 3\text{x} )\text{dx}


=\frac{1}{2}\int(2\sin\text{x}\sin2\text{x})\sin\text{3x}\text{dx}
=\frac{1}{2}\int[\cos\text{(x)- cos}\ (3\text{x})]\sin \text{3x}\text{ dx}
=\frac{1}{4}\int2\cos\text{x}\sin3\text{x}\text{dx}-\frac{1}{4}\int2\cos3\text{x}\sin3\text{x}\text{ dx}
=\frac{1}{4}\int(\sin4\text{x}+\sin2\text{x})\text{dx}-\frac{1}{4}\int\sin6\text{x}\text{dx}
=-\frac{1}{4}\Big(\frac{\cos4\text{x}}{4}+\frac{\cos2\text{x}}{2}\Big)+\frac{1}{24}\cos6\text{x}+\text{C}
Q499. Find \frac{\text{dy}}{\text{ dx}} in the following: 4 Marks
\text{y}=\cos^{-1}\Bigg(\frac{2\text{x}}{1+\text{x}^{2}}\Bigg), -1<\text{x}<1

Ans: The given relationship is,\text{y}=\cos^{-1}\Bigg(\frac{2\text{x}}{1+\text{x}^{2}}\Bigg)


\text{y}=\cos^{-1}\Bigg(\frac{2\text{x}}{1+\text{x}^{2}}\Bigg)
\Rightarrow\cos\text{y}=\frac{2\text{x}}{1+\text{x}^{2}}
Differentiating this relationship with respect to x, we obtain
\frac{\text{d}}{\text{dx}}(\cos\text{y})=\frac{\text{d}}{\text{dx}}\bigg(\frac{2\text{x}}{1+\text{x}^{2}}\bigg)
Using chain rule, we obtain
\frac{\text{d}}{\text{dx}}(\cos\text{y})=\frac{\text{d}}{\text{dx}}.\bigg(\frac{2\text{x}}{1+\text{x}^{2}}\bigg)
(-\sin\text{y})\frac{\text{dy}}{\text{dx}}=\frac{\text{d}}{\text{dx}}.\bigg(\frac{2\text{x}}{1+\text{x}^{2}}\bigg)
\Rightarrow-\sqrt{1-\cos^{2}\text{y}}\frac{\text{dy}}{\text{dx}}=\frac{(1+\text{x}^{2})\times2-2\text{x}.2\text{x}}{(1+\text{x}^{2})^{2}}
\Bigg[\sqrt{1-\bigg(\frac{2\text{x}}{1+\text{x}}\bigg)^{2}}\Bigg]\frac{\text{dy}}{\text{dx}}=\Bigg[\frac{2(1-\text{x}^{2})}{(1+\text{x})^{2}}\Bigg]
\sqrt{\frac{(1+\text{x})^2-4\text{x}^2}{(1+\text{x}^2)^2}}\frac{\text{dy}}{\text{dx}}=\frac{-2(1-\text{x})^2}{(1+\text{x}^2)^2}
\sqrt{\frac{(1-\text{x}^{2})^2}{(1+\text{x}^2)^2}}\frac{\text{dy}}{\text{dx}}=\frac{-2(1-\text{x})^2}{(1+\text{x}^2)^2}
\Rightarrow{\frac{1-\text{x}^{2}}{1+\text{x}^2}}\frac{\text{dy}}{\text{dx}}=\frac{-2(1-\text{x})^2}{(1+\text{x}^2)^2}
\Rightarrow\frac{\text{dy}}{\text{dx}}=\frac{-2}{1+\text{x}^2}
Q500. Verify Lagrange's mean value theorem for the following function on the indicated intervals. find a point 'c' in the indicated interval as stated by the Lagrange's mean value theorem. 4 Marks

https://bls.smartstudies.co.in/#/exam/pdf-preview/c59cb220-8e86-4716-9ff7-82aec16b1ade/1 106/158
5/26/24, 6:19 PM Exam Automation
2
f(x) = x - 2x + 4 on [1, 5]

Ans: We have,
f(x) = x2 - 2x + 4
Since a polynomial function is everywhere continuous and differentiable.
Therefore, f(x) is continuous on [1, 5] and differentiable on (1, 5).
Thus, both conditions of Lagrange's mean value theorem are satisfied.
So, there must exist at least one real number \text{c}\in(1,5) such that
\text{f}'(\text{c})=\frac{\text{f}(5)-\text{f}(-1)}{5-1}=\frac{\text{f}(5)-\text{f}(-1)}{4}
Now, f(x) = x2 - 2x + 4
⇒ f'(x) = 2x - 2
⇒ f(5) = 25 - 10 + 4 = 19
⇒ f(1) = 1 - 2 + 4 = 3
\therefore\ \text{f}'(\text{x})=\frac{\text{f}(5)-\text{f}(-1)}{4}
\Rightarrow2\text{x}-2=\frac{19-3}{4}
\Rightarrow2\text{x}-2-4=0
\Rightarrow\text{x}=\frac{6}{2}=3
Thus, \text{c}=3\in(1,5) such that \text{f}'(\text{c})=\frac{\text{f}(5)-\text{f}(-1)}{5-1}
Hence, Lagrange's mean value theorem is verified.
Q501. \text{If } x^{\text{m }} \text{y}^{\text{n}} = ({x + \text{y)}^{\text{m + n}}}, \text{prove that} \frac{\text{d}^{2}\text{y}}{\text{d}x^{2}} = 0. 4 Marks

Ans: \text{x}^{\text{m}} . \text{y}^{\text{n}} = \text{(x + y)}^{\text{m + n}}


\Rightarrow \text{m} \log \text{x + n} \log \text{y = (m + n)} \log \text{(x + y)}
\Rightarrow \frac{\text{m}}{\text{x}} + \frac{\text{n}}{\text{y}}.\frac{\text{dy}}{\text{dx}} = \frac{\text{m + n}}{\text{x + y}} \bigg(1 + \frac{\text{dy}}{\text{dx}}\bigg)
\Rightarrow \frac{\text{dy}}{\text{dx}} = \frac{\text{y}}{\text{x}} \text{ }\text{ }\text{ }\text{ }\text{ }\dots\text{(i)}
\frac{\text{d}^{2}{\text{y}}}{\text{dx}^{2}} = \frac{\text{x}\frac{\text{dy}}{\text{dx}}-\text{y}}{\text{x}^{2}} \text{ }\text{ }\text{ }\text{ }\text{ } \dots\text{(ii) }
= \frac{\text{x}{\frac{\text{y}}{\text{x}} -\text{y}}}{\text{x}^{2}} \text{ }\text{ }\text{ }\text{ }\text{ }\dots\text{using (i)}
=0
Q502. Differentiatet \tan^{-1}\Big(\frac{x}{\sqrt{1-x^2}}\Big) with respect to \sin^{-1}(2x\sqrt{1-x^2}). 4 Marks

Ans: \text{let u}=\tan^{-1}\Big(\frac{\text{x}}{\sqrt{1-\text{x}^2}}\Big);\ \text{v}=\sin^{-1}(2\text{x}\sqrt{1-\text{x}^2});\ \text{x}=\sin\theta\ \therefore\ \theta=\sin^{-1}\text{x}


\therefore\ \text{u}=\tan^{-1}\Big(\frac{\sin\ \theta}{\sqrt{1-\sin^2\theta}}\Big)=\tan^{-1}(\tan\theta)=\theta=\sin^{-1}\text{x}
\&\ \text{v}=\sin^{-1}(2\text{x}\sqrt{1-\text{x}^2})=\sin^{-1}(\sin2\theta)=2\theta=2\sin^{-1}\text{x}
\frac{\text{du}}{\text{dx}}=\frac{1}{\sqrt{1-\text{x}^2}},\ \frac{\text{dv}}{\text{dx}}=\frac{2}{\sqrt{1-\text{x}^2}}
\therefore\ \ \frac{\text{du}}{\text{dv}}=\frac{1}{\sqrt{1-\text{x}^2}}\times\frac{\sqrt{1-\text{x}^2}}{2}=\frac{1}{2}
(\text{In case, if x}=\cos\theta\ \text{than answer is }-\frac{1}{2})
Q503. If y = 3e2x + 2e3x, prove that 4 Marks
\frac{\text{d}^{2}\text{y}}{\text{dx}^{2}}-5\frac{\text{dy}}{\text{dx}}+\text{6y}=0.

Ans: \frac{\text{dy}}{\text{dx}}=6\text{ e}^{\text{2x}}+6\cdot\text{e}^{\text{3x}}


\frac{\text{d}^{2}\text{y}}{\text{dx}^{2}}=12\text{ e}^{\text{2x}}+18\cdot\text{e}^{\text{3x}}
\Rightarrow\frac{\text{d}^{2}\text{y}}{\text{dx}^{2}}-5\frac{\text{dy}}{\text{dx}}+6\text{y} = (12 e2x + 18 e 3x) -5 (6 e2x + 6 e3x ) + 6 (3 e2x + 2 e3x)
= 30 e2x - 30 e2x + 30 e3x - 30 e3x = 0.
Q504. Find \frac{\text{dy}}{\text{dx}} 4 Marks
\text{y}=\sin\text{x}\sin2\text{x}\sin3\text{x}\sin4\text{x}

Ans: Here,
\text{y}=\sin\text{x}\cdot\sin2\text{x}\cdot\sin3\text{x}\cdot\sin4\text{x}\ .....(\text{i})
Taking log on both the sides,
\log\text{y}=\log(\sin\text{x}\cdot\sin2\text{x}\cdot\sin3\text{x}\cdot\sin4\text{x})
\log\text{y}=\log\sin\text{x}+\log\sin2\text{x}+\log\sin3\text{x}+\log\sin4\text{x}
Differentiating it with respect to x using chain rule,
\frac{1}{\text{y}}\frac{\text{dy}}{\text{dx}}=\frac{\text{d}}{\text{dx}}\log\sin\text{x}+\frac{\text{d}}{\text{dx}}\log\sin2\text{x}+\frac{\text{d}}{\text{dx}}\log\sin3\text{x}+\frac{\text{d}}
{\text{dx}}\log\sin4\text{x}
=\frac{1}{\sin\text{x}}\frac{\text{d}}{\text{dx}}(\sin\text{x})+\frac{1}{\sin2\text{x}}\frac{\text{d}}{\text{dx}}(\sin2\text{x})+\frac{1}{\sin3\text{x}}\frac{\text{d}}{\text{dx}}
(\sin3\text{x})+\frac{1}{\sin4\text{x}}\frac{\text{d}}{\text{dx}}(\sin4 \text{x})
=\frac{1}{\sin\text{x}}(\cos\text{x})+\frac{1}{\sin2\text{x}}(\cos2\text{x})\frac{\text{d}}{\text{dx}}(2\text{x}) \\ +\frac{1}{\sin3\text{x}}(\cos3\text{x})\frac{\text{d}}{\text{dx}}
(3\text{x})+\frac{1}{\sin4\text{x}}(\cos4\text{x})\frac{\text{d}}{\text{dx}}(4\text{x})
\frac{1}{\text{y}}\frac{\text{dy}}{\text{dx}}=\big[\cot\text{x}+\cot2\text{x}(2)+\cot3\text{x}(3)+\cot4\text{x}(4)\big]
\frac{\text{dy}}{\text{dx}}=\text{y}\big[\cot\text{x}+2\cot2\text{x}+3\cot\text{x}3\text{x}+4\cot4\text{x}\big]
\frac{\text{dy}}{\text{dx}}=(\sin\text{x}\sin2\text{x}\sin3\text{x}\sin4\text{x}) \\ \big[\cot\text{x}+2\cot2\text{x}+3\cot\text{x}3\text{x}+4\cot4\text{x}\big]
[Using equation (i)]
Q505. Discuss the continuity of the function f, where f is defined by: 4 Marks
\text{f(x)}= \begin{cases}-2,\ \text{if}\ \text{x}\leq-1 \\\text{2x},\text{if}\ -1<\text{x}\leq1\\2,\text{if}\ \text{x}>1\end{cases}

Ans: \text{f(x)}= \begin{cases}-2,\ \text{if}\ \text{x}\leq-1 \\\text{2x},\text{if}\ -1<\text{x}\leq1\\2,\text{if}\ \text{x}>1\end{cases}


The function f is defined at all points of the real line.
When x < -1, we have f(x) = -2, which it is constant and so is continuous.
At x = -1
^{\ \ \text{Lt}}_{\text{x}\rightarrow\text{-1}^{-}}\text{f(x)} = ^{\ \ \text{Lt}}_{\text{x}\rightarrow\text{-1}^{-}}(-{2}) = -2
^{\ \ \text{Lt}}_{\text{x}\rightarrow\text{-1}^{+}}\text{f(x)} = ^{\ \ \text{Lt}}_{\text{x}\rightarrow\text{-1}^{+}}(\text{2x}) = 2(-1)= -2
Also f(-1) = -2
\therefore\ ^{\ \ \text{Lt}}_{\text{x}\rightarrow\text{-1}^{-}}\text{f(x)} = ^{\ \ \text{Lt}}_{\text{x}\rightarrow\text{-1}^{+}}\text{f(x)} = \text{f}(-1)
\therefore f is continuous at x = -1
in the interval -1 < x < 1, we have f(x) = 2 x, which being a linear polynomial, is continuous.
At x = -1
^{\ \ \text{Lt}}_{\text{x}\rightarrow\text{-1}^{-}}\text{f(x)} = ^{\ \ \text{Lt}}_{\text{x}\rightarrow\text{-1}^{-}}(\text{2x}) = 2(1)= 2
^{\ \ \text{Lt}}_{\text{x}\rightarrow\text{-1}^{+}}\text{f(x)} = ^{\ \ \text{Lt}}_{\text{x}\rightarrow\text{-1}^{+}}({2}) = 2
Also f(1) = 2(1) = 2
\therefore\ ^{\ \ \text{Lt}}_{\text{x}\rightarrow\text{-1}^{-}}\text{f(x)} = ^{\ \ \text{Lt}}_{\text{x}\rightarrow\text{-1}^{+}}\text{f(x)} = \text{f}(1)
\therefore f is continuous at x = 1
When x > 1, we have f(x) = 2, which is constant and so it is continuous.
Q506. Differentiate (x2 – 5x + 8) (x3 + 7x + 9) in three ways mentioned below: 4 Marks
by logarithmic differentiation.

Ans: y = (x2 - 5x + 8) (x3 + 7x + 9)


\Rightarrow\ \log\text{y}=\log(\text{x}^2-5\text{x}+8)+\log(\text{x}^3+7\text{x}+9)
\Rightarrow\ \frac{\text{d}}{\text{dx}}\log\text{y}=\frac{\text{d}}{\text{dx}}\log(\text{x}^2-5\text{x}+8)+\frac{\text{d}}{\text{dx}}\log(\text{x}^3+7\text{x}+9)

https://bls.smartstudies.co.in/#/exam/pdf-preview/c59cb220-8e86-4716-9ff7-82aec16b1ade/1 107/158
5/26/24, 6:19 PM Exam Automation
\Rightarrow\ \frac{1}{\text{y}}\frac{\text{dy}}{\text{dx}}=\frac{1}{\text{x}^2-5\text{x}+8}\frac{\text{d}}{\text{dx}}(\text{x}^2-5\text{x}+8)+\frac{1}{\text{x}^3+7\text{x}+9}\frac{\text{d}}
{\text{dx}}(\text{x}^3+7\text{x}+9)
\Rightarrow\ \frac{1}{\text{y}}\frac{\text{dy}}{\text{dx}}=\frac{1}{\text{x}^2-5\text{x}+8}(2\text{x}-5)+\frac{1}{\text{x}^3+7\text{x}+9}(3\text{x}^2+7)
\Rightarrow\ \frac{\text{dy}}{\text{dx}}=\text{y}\Big[\frac{2\text{x}-5}{\text{x}^2-5\text{x}+8}+\frac{3\text{x}^2+7}{\text{x}^3-7\text{x}+9}\Big]
\Rightarrow\ \frac{\text{dy}}{\text{dx}}=\text{y}\Big[\frac{(2\text{x}-5)(\text{x}^3+7\text{x}+9)+(3\text{x}^2+7)(\text{x}^2-5\text{x}+8)}{(\text{x}^2-5\text{x}+8)
(\text{x}^3+7\text{x}+9)}\Big]
\Rightarrow\ \frac{\text{dy}}{\text{dx}}=\text{y}\Big[\frac{2\text{x}^4+14\text{x}^2+18\text{x}-5\text{x}^3-35\text{x}-45+3\text{x}^4-15\text{x}^3+24\text{x}^2+7\text{x}^2-35\text{x}+56}
{(\text{x}^2-5\text{x}+8)(\text{x}^3+7\text{x}+9)}\Big]
\Rightarrow\ \frac{\text{dy}}{\text{dx}}=\text{y}\Big[\frac{5\text{x}^4-20\text{x}^3+45\text{x}^2-52\text{x}+11}{(\text{x}^2-5\text{x}+8)(\text{x}^3+7\text{x}+9)}\Big]
\Rightarrow\ \frac{\text{dy}}{\text{dx}}=(\text{x}^2-5\text{x}+8)(\text{x}^3+7\text{x}+9)\Big[\frac{5\text{x}^4-20\text{x}^3+45\text{x}^2-52\text{x}+11}{(\text{x}^2-5\text{x}+8)
(\text{x}^3+7\text{x}+9)}\Big]\ \text{[From eq.(i)}]
\Rightarrow\ \frac{\text{dy}}{\text{dx}}=5\text{x}^4-20\text{x}^3+45\text{x}^2-52\text{x}+11\ \dots\text{(iv)}
From eq. (ii), (iii) and (iv), we can say that value of \frac{\text{dy}}{\text{dx}} is same obtained by three different methods
Q507. Differentiate \tan^{-1} \bigg(\frac{\sqrt{1 + x^{2} - 1}}{x}\bigg) w.r.t. \sin^{-1} \frac{2x}{1 + x^{2}}, \text{ if } x \in (-1, 1) 4 Marks

Ans: \text{Let} \text{ u} = \tan^{-1} \frac{\sqrt{1 + \text{x}^{2} - 1}}{\text{x}}


\text{Put } \text{ x } = \tan \theta \Rightarrow \theta = \tan^{-1} \text{x}
\therefore \text{ u } = \tan^{-1} \bigg[\frac{\sec \theta - 1}{\tan \theta}\bigg]
= \tan^{-1} \bigg[\frac{1 - \cos \theta}{\sin \theta}\bigg]
= \tan^{-1} \big(\tan \frac{\theta}{2}\big)
= \frac{\theta}{2} = \frac{1}{2} \tan^{-1} \text{x}
\Rightarrow \frac{\text{du}}{\text{dx}} = \frac{1}{2 (1 + \text{x}^{2})}
\text{v} = \sin^{-1} \bigg(\frac{\text{2x}}{1 + \text{x}^{2}}\bigg)
= 2\tan^{-1} \text{x}
\Rightarrow \frac{\text{dv}}{\text{dx}} = \frac{2}{1 + \text{x}^{2}}
\therefore \frac{\text{du}}{\text{dv}} = \frac{\text{du/dx}}{\text{dv/dx}} = \frac{1}{4}
Q508. The equation of the path traced by a roller-coaster is given by the polynomial f(x) = a (x + 9) (x + 1) (x - 3). If the roller-coaster crosses y-axis at a point (0, -1), answer the following: 4 Marks

1. Find the value of ‘a’.


2. Find f"(x) at x = 1.

Ans: 1. -1=\text{a}(-27)\Rightarrow\text{a}=\frac{1}{27}
2. \text{f}(\text{x})=\frac{1}{27}(\text{x}+9)(\text{x}+1)(\text{x}-3)
=\frac{1}{27}(\text{x}^3+7\text{x}^2-21\text{x}-27)
\text{f}'(\text{x})=\frac{1}{27}(3\text{x}^2+14\text{x}-21)
\text{f}''(\text{x})=\frac{6\text{x}+14}{27}
\text{f}''(1)=\frac{20}{27}
Q509. In the following, find the value of the constant k so that the given function is continuous at the indicated point: 4 Marks
\text{f(x)}=\begin{cases}\frac{\text{x}^2+\text{x}^2-16\text{x}+20}{(\text{x}-2)^2},&\text{ x}\neq2\\\text{k},&\text{x}=2\end{cases}

Ans: Given,
\text{f(x)}=\begin{cases}\frac{\text{x}^2+\text{x}^2-16\text{x}+20}{(\text{x}-2)^2},&\text{ x}\neq2\\\text{k},&\text{x}=2\end{cases}
\Rightarrow\text{f(x)}=\begin{cases}\frac{\text{x}^2+\text{x}^2-16\text{x}+20}{\text{x}^2-4\text{x}+4},&\text{ x}\neq2\\\text{k},&\text{x}=2\end{cases}
\Rightarrow\text{f(x)}=\begin{cases}\text{x}+5,&\text{ x}\neq2\\\text{k},&\text{x}=2\end{cases}
If f(x) is continuous at x = 2, then
\lim_\limits{\text{x}\rightarrow2}\text{f(x)}=\text{f}(2)
\Rightarrow\lim_\limits{\text{x}\rightarrow2}\text{(x}+5)=\text{k}
\Rightarrow\text{k}=2+5=7
Q510. If \tan^1\Big(\frac{\text{y}}{\text{x}}\Big)=\log\sqrt{\text{x}^2+\text{y}^2}, Prove that \frac{\text{dy}}{\text{dx}}=\frac{\text{x}+\text{y}}{\text{x}-\text{y}}. 4 Marks

Ans: \log(\text{x}^2+\text{y}^2)=2\tan^1\frac{\text{y}}{\text{x}}
We need to find the derivative of y with respect to x.
Differentiating with respect to x on both sides.
\log(\text{x}^2+\text{y}^2)=2\tan^1\frac{\text{y}}{\text{x}}
Where we know that \frac{\text{dy}}{\text{dx}}=\text{y}_1
\frac{2(\text{x}+\text{yy}_1)}{\text{x}^2+\text{y}^2}=\frac{2(\text{x}\text{y}_1-\text{y})}{\text{x}^2+\text{y}^2}
Cancelling \text{x}^2+\text{y}^2 on both sides and multiplying 2 we get
2\text{x}+2\text{yy}_1=2\text{xy}_1-2\text{y}
Cancelling out 2 on both sides
\text{x}+\text{yy}_1=\text{xy}_1-\text{y}
\text{x}+\text{y}=\text{xy}_1-\text{yy}_1
Shifting the term which has \text{y}_1 on one side
\text{x}+\text{y}=\text{y}_1(\text{x}-\text{y})
\text{y}_1=\frac{\text{x}+\text{y}}{\text{x}-\text{y}}
Substituting \frac{\text{dy}}{\text{dx}}=\text{y}_1
\frac{\text{dy}}{\text{dx}}=\frac{\text{x}+\text{y}}{\text{x}-\text{y}}
Hence Proved.
Q511. Differentiate \frac{\text{x}}{\sin\text{x}} w.r.t. \sin\text{x}. 4 Marks

Ans: Let \text{u}=\frac{\text{x}}{\sin\text{x}} and \text{v}=\sin\text{x}


\therefore\ \frac{\text{du}}{\text{dx}}=\frac{\sin\text{x}\cdot\frac{\text{d}}{\text{dx}}\text{x}-\text{x}\cdot\frac{\text{d}}{\text{dx}}\sin\text{x}}{(\sin\text{x})^2}
=\frac{\sin\text{x}-\text{x}\cos\text{x}}{\sin^2\text{x}}
and \frac{\text{dv}}{\text{dx}}=\frac{\text{d}}{\text{dx}}\sin\text{x}=\cos\text{x}
\therefore\ \frac{\text{du}}{\text{dv}}=\frac{\frac{\text{du}}{\text{dx}}}{\frac{\text{dv}}{\text{dx}}}=\frac{\sin\text{x}-\frac{\text{x}\cos\text{x}}{\sin^2\text{x}}}{\cos\text{x}}
=\frac{\sin\text{x}-\text{x}\cos\text{x}}{\sin^2\text{x}\cos\text{x}}
=\frac{\frac{\sin\text{x}-\text{x}\cos\text{x}}{\cos\text{x}}}{\frac{\sin^2\text{x}\cos\text{x}}{\cos\text{x}}}
=\frac{\tan\text{x}-\text{x}}{\sin^2\text{x}}
Q512. Verify the Rolle’s theorem for each of the functions: 4 Marks
\text{f(x)}=\sin^4\text{x}+\cos^4\text{x}\text{ in }\Big[0,\frac{\pi}{2}\Big].

Ans: We have, \text{f(x)}=\sin^4\text{x}+\cos^4\text{x}\text{ in }\Big[0,\frac{\pi}{2}\Big]


We know that \sin\text{x} and \cos\text{x} are continuoud and diferentiabe

https://bls.smartstudies.co.in/#/exam/pdf-preview/c59cb220-8e86-4716-9ff7-82aec16b1ade/1 108/158
5/26/24, 6:19 PM Exam Automation
\therefore\ \sin^4\text{x} and \cos^4\text{x} and hence \sin^4\text{x}+\cos^4\text{x} is continuous and differentiable
Now \text{f}(0)=0+1=1 and \text{f}\Big(\frac{\pi}{2}\Big)=1+0=1
\Rightarrow\ \text{f}(0)=\text{f}\Big(\frac{\pi}{2}\Big)
So, conditions of Rolle's theorem are satisfied.
Hence, there exists atleast one \text{c}\in\Big(0,\frac{\pi}{2}\Big) such that f(c) = 0
\therefore\ 4\sin^3\text{c}\cos\text{c}-4\cos^3\text{c}\sin\text{c}=0
\Rightarrow\ 4\sin\text{c}\cos\text{c}(\sin^2\text{c}-\cos^2\text{c})=0
\Rightarrow\ 4\sin\text{c}\cos\text{c}(-\cos2\text{c})=0
\Rightarrow-2\sin2\text{c}\cdot\cos2\text{c}=0
\Rightarrow\ \sin4\text{c}=0
\Rightarrow\ 4\text{c}=\pi
\Rightarrow\ \text{c}=\frac{\pi}{4}\in\Big(0,\frac{\pi}{2}\Big)
Hence, Rolle's theorem has been verified.
Q513. If f(x) = x3 + 7x2 + 8x - 9, find f(4). 4 Marks

Ans: f(x) = x3 + 7x2 + 8x - 9 is a polynomial function. So, it is differentiable every.


\text{f}'(4)=\lim_\limits{\text{h}\rightarrow0}\frac{\text{f}(4+\text{h})-\text{h}(4)}{\text{h}}
=\lim_\limits{\text{h}\rightarrow0}\frac{[(4+\text{h})^3+7(4+\text{h})^2+8(4+\text{h})-9]-[64+112+32-9]}{\text{h}}
=\lim_\limits{\text{h}\rightarrow0}\frac{[64+\text{h}^3+48\text{h}+12\text{h}^2+112+7\text{h}^2+56\text{h}+32+8\text{h}-9]-[210-9]}{\text{h}}
=\lim_\limits{\text{h}\rightarrow0}\frac{\text{h}^3+19\text{h}^2+112\text{h}+210-9-210+9}{\text{h}}
=\lim_\limits{\text{h}\rightarrow0}\frac{\text{h}^3+19\text{h}^2+112\text{h}}{\text{h}}
=\lim_\limits{\text{h}\rightarrow0}\frac{\text{h}(\text{h}^2+19\text{h}+112)}{\text{h}}
\text{f}'(4)=112
Q514. If the lines \frac{\text{x}-1}{-3}=\frac{\text{y}-2}{2\lambda}=\frac{\text{z}-3}{2} and \frac{\text{x}-1}{3\lambda}=\frac{\text{y}-1}{2}=\frac{\text{z}-6}{-5} are perpendicular, find the value 4 Marks
of \lambda. Hence find whether the lines are intersecting or not.

Ans: Given,
\frac{\text{x}-1}{-3}=\frac{\text{y}-2}{2\lambda}=\frac{\text{z}-3}{2}
\frac{\text{x}-1}{3\lambda}=\frac{\text{y}-1}{2}=\frac{\text{z}-6}{-5}
Since, Both lines are perpendicular to each other,
\therefore-3(3\lambda)+2\lambda(2)+2(-5)=0
\Rightarrow-9\lambda+4\lambda-10=0
\Rightarrow-5\lambda=10
\Rightarrow\lambda=-2
Now,
\begin{bmatrix}\text{x}_2-\text{x}_1 & \text{y}_2-\text{y}_1 & \text{z}_2-\text{z}_1 \\\text{a}_1 & \text{b}_1 & \text{c}_1\\\text{a}_2 & \text{b} _2 & \text{c}_2\end{bmatrix}
On comparing \frac{\text{x}-\text{x}_1}{\text{a}_1}=\frac{\text{y}-\text{y}_1}{\text{b}_1}=\frac{\text{z}-\text{z}_1}{\text{c}_1}
or \frac{\text{x}-\text{x}_2}{\text{a}_2}=\frac{\text{y}-\text{y}_2}{\text{b}_2}=\frac{\text{z}-\text{z}_2}{\text{c}_2}
the above given equation,
We have,
​\text{a}_1=-3,\text{a}_2=-6,\text{x}_1=1,\text{x}_2=1
​\text{b}_1=-4,\text{b}_2=2,\text{y}_1=2,\text{y}_2=1
​\text{c}_1=2,\text{b}_2=-5,\text{z}_1=3,\text{z}_2=6
Then,
\begin{bmatrix}1-1&1-2&6-3\\-3&-4&2\\-6&2&-5\end{bmatrix}
\Rightarrow(20-4)-(-1)[15-(-12)]+3(-6-24)
\Rightarrow-27-90
\Rightarrow-63\neq0
\therefore Lines are not intersecting
Q515. Find the points of discontinuity, if any of the following function: 4 Marks
\text{f(x)}=\begin{cases}\frac{\sin3\text{x}}{\text{x}},&\text{if }\text{ x}\neq0\\4,&\text{if }\text{ x}=0\end{cases}

Ans: When \text{x}\neq0, then


\text{f(x)}=\frac{\sin3\text{x}}{\text{x}}
We know that \sin3\text{x} as well as the identity function x are everwhere continuous.
So, the quotient function \frac{\sin3\text{x}}{\text{x}} is continuous at each \text{x}\neq0
Let us consider the point x = 0
Given, \text{f(x)}=\begin{cases}\frac{\sin3\text{x}}{\text{x}},&\text{if }\text{ x}\neq0\\4,&\text{if }\text{ x}=0\end{cases}
We have
(\text{LHL at x}=0)=\lim_\limits{\text{x}\rightarrow0^-}\text{f(x)}=\lim_\limits{\text{h}\rightarrow0}\text{f}(0-\text{h})
=\lim_\limits{\text{h}\rightarrow0}\text{f}(-\text{h})=\lim_\limits{\text{h}\rightarrow0}\Big(\frac{\sin(-3\text{h})}{-
\text{h}}\Big)=\lim_\limits{\text{h}\rightarrow0}\Big(\frac{3\sin(3\text{h})}{3\text{h}}\Big)=3
(\text{RHL at x}=0)=\lim_\limits{\text{x}\rightarrow0^+}\text{f(x)}=\lim_\limits{\text{h}\rightarrow0}\text{f}(0+\text{h})
=\lim_\limits{\text{h}\rightarrow0}\text{f}(\text{h})=\lim_\limits{\text{h}\rightarrow0}\Big(\frac{\sin(3\text{h})}{\text{h}}\Big)=\lim_\limits{\text{h}\rightarrow0}\Big(\frac{3\sin(3\text{h})}
{3\text{h}}\Big)=3
Also, f(0) = 4
\therefore\ \lim_\limits{\text{x}\rightarrow0^-}\text{f(x)}=\lim_\limits{\text{x}\rightarrow0^+}\text{f(x)}\neq\text{f}(0)
Thus, f(x) is discontinuous at x = 0
Hence, the only point of discontinuity for f(x) is x = 0
Q516. If the derivative of tan-1 (a + bx) takes the value 1 at x = 0, prove that 1 + a2 = b. 4 Marks

Ans: Here, \frac{\text{d}}{\text{dx}}\big[\tan^{-1}(\text{a}+\text{bx})\big]=1\text{ at x}=0


So, using chain rule,
\Big[\Big\{\frac{1}{1+(\text{a}+\text{bx})^2}\Big\}\frac{\text{d}}{\text{dx}}(\text{a}+\text{bx})\Big]_{\text{x}=0}=0
\Big[\frac{1}{1+(\text{a}+\text{bx})^2}\times(\text{b})\Big]_{\text{x}=0}=1
\Rightarrow \frac{\text{b}}{1+(\text{a}+0)^2}=1
\Rightarrow \text{b}=1+\text{a}^2
Q517. If xy + yx = ab, then find \frac{\text{dy}}{\text{dx}}. 4 Marks

Ans: xy + yx = ab
Let u + v = ab, where xy = u and yx = v.
\therefore \frac{\text{du}}{\text{dx}} + \frac{\text{dv}}{\text{dx}} = 0 \text{ }\text{ }\text{ }\text{ }\text{ }\dots\text{(i)}
\text{y} = \log \text{x} = \log \text{u} \Rightarrow\frac{\text{du}}{\text{dx}}=\text{x}^{\text{y}} \bigg[\frac{\text{y}}{\text{x}} + \log \text{x}. \frac{\text{dy}}{\text{dx}}\bigg]
\text{x} \log \text{y} = \log \text{v} \Rightarrow \frac{\text{dv}}{\text{dx}} = \text{y}^{\text{x}} \bigg[\frac{\text{x}}{\text{y}} \frac{\text{dy}}{\text{dx}} + \log \text{y}\bigg]
\text{putting in (i)} \text{x}^{\text{y}} \bigg[\frac{\text{y}}{\text{x}} + \log \text{x} \frac{\text{dy}}{\text{dx}}\bigg] + \text{y}^{\text{x}} \bigg[\frac{\text{x}}{\text{y}}\frac{\text{dy}}{\text{dx}}
+ \log \text{y} \bigg]= 0
\Rightarrow \frac{\text{dy}}{\text{dx}} = - \frac{\text{y}^{\text{x}} \log \text{y + y.x}^{\text{y - 1}}}{\text{x}^{\text{y}}. \log \text{x + x.y}^{\text{x - 1}}}
Q518. Find the points of discontinuity, if any of the following function: 4 Marks

https://bls.smartstudies.co.in/#/exam/pdf-preview/c59cb220-8e86-4716-9ff7-82aec16b1ade/1 109/158
5/26/24, 6:19 PM Exam Automation
\text{f(x)}=\begin{cases}\text{x}^3-\text{x}^2+2\text{x}-2,&\text{if }\text{ x}\neq1\\4,&\text{if }\text{ x}=1\end{cases}

Ans: When ​\text{x}\neq1, then


\text{f(x)}=\text{x}^3-\text{x}^2+2\text{x}-2
We know that a polynomial function is everywhere continuous.
So, \text{f(x)}=\text{x}^3-\text{x}^2+2\text{x}-2 is continuous at each ​\text{x}\neq1
At x = 1, we have
(\text{LHL at x}=1)=\lim_\limits{\text{x}\rightarrow1^-}\text{f(x)}=\lim_\limits{\text{h}\rightarrow0}\text{f}(1-\text{h})
=\lim_\limits{\text{h}\rightarrow0}\Big((1-\text{h})^3-(1-\text{h})^2+2(1-\text{h})-2\Big)=1-1+2-2=0
(\text{RHL at x}= 1)=\lim_\limits{\text{x}\rightarrow1^+}\text{f(x)}=\lim_\limits{\text{h}\rightarrow0}\text{f}(1+\text{h})
=\lim_\limits{\text{h}\rightarrow0}\Big((1+\text{h})^3-(1+\text{h})^2+2(1+\text{h})-2\Big)=1-1+2-2=0
Also, f(1) = 4
\therefore\ \lim_\limits{\text{x}\rightarrow1^-}\text{f(x)}=\lim_\limits{\text{x}\rightarrow1^+}\text{f(x)}\neq\text{f}(1)
Thus, f(x) is discontinuous at x = 1
Hence, the only point of discontinuity for f(x) is x = 1
Q519. If \text{x}=\text{a}(1-\cos^3\theta),\text{y}=\text{a}\sin^3\theta, Prove that \frac{\text{d}^2\text{y}}{\text{dx}^2}=\frac{32}{27\text{a}}\text{ at}\ \theta=\frac{\pi}{6} 4 Marks

Ans: Here,
\text{x}=\text{a}(1-\cos^3\theta),\text{y}=\text{a}\sin^3\theta
Differentiating w.r.t. x, we get
\frac{\text{dx}}{\text{d}\theta}=3\text{a}\cos2\theta\sin\theta\text{ and }\frac{\text{dy}}{\text{d}\theta}=3\text{a}\sin2\theta\cos\theta]
\frac{\text{dy}}{\text{dx}}=\frac{3\text{a}\sin^2\theta\cos\theta}{3\text{a}\cos^2\theta\sin\theta}=\tan\theta
Differentiating w.r.t. x, we get
\frac{\text{d}^2\text{y}}{\text{dx}^2}=\sec^2\theta\frac{\text{d}\theta}{\text{dx}}
=\frac{\sec^2\theta}{3\text{a}\cos^2\theta\sin\theta}
=\frac{\sec^4\theta}{3\text{a}\sin\theta}
\therefore\frac{\text{d}^2\text{y}}{\text{dx}}\text{ at }\theta=\frac{\pi}{6}
\frac{\text{d}^2\text{y}}{\text{dx}}=\frac{\Big(\sec\frac{\text{x}}{6}\Big)^4}{3\text{a}\sin\frac{\text{x}}{6}}=\frac{32}{27\text{a}}
Q520. \text{If y}=\cos^{-1}\bigg(\frac{3\text{x}+4\sqrt{1-\text{x}^2}}{5}\bigg),\ \text{find}\ \frac{\text{dy}}{\text{dx}}\dot{} 4 Marks

Ans: =\cos^{-1}\bigg[\frac{3}{5}\text{x}+\frac{4}{5}\sqrt{1-\text{x}^2}\bigg]
=\cos^{-1}\bigg[\frac{3}{5}\dot{}\cos\theta+\frac{4}{5}\sin\theta\bigg]\ \text{where x}=\cos\theta
=\cos^{-1}\big[\cos\alpha\dot{}\cos\theta+\sin\alpha\dot{}\sin\theta\big],\;\because\ \text{if}\: \frac{3}{5}=\cos\alpha,\text{then}\frac{4}{5}=\sin \ \alpha
=\cos^{-1}\big[\cos\big(\alpha-\theta\big)\big]=\alpha-\theta=\cos^{-1}\big(3/5\big)-\cos^{-1}\text{x}
\Rightarrow\frac{\text{dy}}{\text{dx}}=\frac{1}{\sqrt{1-\text{x}^2}}
Q521. Find all points of discontinuity of f, where f is defined by: 4 Marks
\text f(\text x)=\begin{cases}2\text{x}+3, \text {if}\ \ \text x\leq2 \\2\text{x} - 3, \text {if}\ \ \text x > 2\end{cases}

Ans: Here \text{f(x)} = \begin{cases}2\text{x} +3, \text{if}\ \text{x}\leq2\\ 2\text{x}-3, \ \text{if}\ \text{x} > 2\end{cases}
Function f is defined for all points of thr real line.
Let c be any real number.
Three cases arise:
Case I: c < 2
^{\ \ \text{Lt}}_{\text{x}\rightarrow\text{c}}\text{f(x)} = ^{\ \ \text{Lt}}_{\text{x}\rightarrow\text{c}}(2\text{x}+ 3) = 2\text{c} + 3
Also f(c) = 2c + 3
\therefore f is continuous at all points x < 2.
Case II: c > 2
^{\ \ \text{Lt}}_{\text{x}\rightarrow\text{c}}\text{f(x)} = ^{\ \ \text{Lt}}_{\text{x}\rightarrow\text{c}}(2\text{x}- 3) = 2\text{c} - 3
Also f(c) = 2c - 3
\therefore\ ^{\ \ \text{Lt}}_{\text{x}\rightarrow\text{c}}\text{f(x)}= \text{f(c)}
\therefore f is continuous at all points x >2.
Case III: c = 2
^{\ \ \text{Lt}}_{\text{x}\rightarrow\text{2}{-}}\text{f(x)} = ^{\ \ \text{Lt}}_{\text{x}\rightarrow\text{2}{-}}(2\text{x}+ 3) = 4 + 3 = 7
^{\ \ \text{Lt}}_{\text{x}\rightarrow\text{2}{+}}\text{f(x)} = ^{\ \ \text{Lt}}_{\text{x}\rightarrow\text{2}{=}}(2\text{x}- 3) = 4 - 3 = 1
\therefore\ ^{\ \ \text{Lt}}_{\text{x}\rightarrow\text{2}^{-}}\text{f(x)}\neq \ ^{\ \ \text{Lt}}_{\text{x}\rightarrow\text{2}^{+}}\text{f(x)}
\therefore f is not continuous at x = 2
\therefore x = 2 is the only point of discontinuity of f.
Q522. Differentiate \tan^{-1}\bigg(\frac{\sqrt{1 - \text{x}^{2}}}{\text{x}}\bigg)with respect to \cos^{-1}(2 \text{x}\sqrt{1- \text{x}^{2}}), when \text{x}\neq0. 4 Marks

Ans: let \text{u} = \tan^{-1}\bigg(\frac{\sqrt{1-\text{x}^{2}}}{\text{x}}\bigg),\text{v} = \cos^{-1}\bigg(2 \text{x}\sqrt{1- \text{x}^{2}}),\text{x} = \cos\theta\therefore\theta =\cos^{-1}\text{x}


\therefore\text{u} = \tan^{-1}\bigg(\frac{\sqrt{1-\cos^{2}\theta}}{\cos\theta}\bigg) = \tan^{-1}(\tan\theta) = \theta = \cos^{-1}\text{x}
and \text{v} = \cos^{-1}( 2\cos\theta\sqrt{1 -\cos^{2}\theta}) = \cos^{-1}(\sin2\theta) = \cos^{-1}\bigg(\cos\bigg(\frac{\pi}{2} - 2 \theta\bigg)\bigg)
= \frac{\pi}{2} - 2\theta =\frac{\pi}{2} - 2\cos^{-1}\text{x}
\frac{\text{du}}{\text{dx}} = \frac{-1}{\sqrt{1-\text{x}^{2}}},\frac{\text{dv}}{\text{dx}} = \frac{2}{\sqrt{1-\text{x}^{2}}}
\therefore\frac{\text{du}}{\text{dv}} =\frac{-1}{\sqrt{1- \text{x}^{2}}}\times\frac{\sqrt{1-\text{x}^{2}}}{2} =\frac{-1}{2}
( In case, If x = sin θ then answer is \frac{1}{2}).
Q523. Verify Lagrange's mean value theorem for the following function on the indicated intervals. find a point 'c' in the indicated interval as stated by the Lagrange's mean value theorem. 4 Marks
\text{f}(\text{x})=\sqrt{25-\text{x}^2}\text{ on }[-3,4]

Ans: We have,
\text{f}(\text{x})=\sqrt{25-\text{x}^2}
Here, f(x) will exist, if
25-\text{x}^2\geq0
\Rightarrow\text{x}^2\leq25
\Rightarrow-5\leq\text{x}\leq5
Since for each \text{x}\in[-3,4], the function f(x) attains a unique definite value.
So, f(x) is continuous on [-3, 4]
Also, \text{f}'(\text{x})=\frac{1}{2\sqrt{25-\text{x}^2}}(-2\text{x})=\frac{-\text{x}}{\sqrt{25-\text{x}^2}} exists for all \text{x}\in(-3,4)
So, f(x) is differentiable on (-3, 4).
Thus, both the conditions of Lagrange's theorem are satisfied.
Consequently, there exist some \text{c}\in(-3,4) such that
\text{f}'(\text{c})=\frac{\text{f}(4)-\text{f}(-3)}{4+3}=\frac{\text{f}(4)-\text{f}(-3)}{7}
Now, \text{f}(\text{x})=\sqrt{25-\text{x}^2}
\text{f}'(\text{x})=\frac{-\text{x}}{\sqrt{25-\text{x}^2}},\text{f}(-3)=4,\text{f}(4)=3
\therefore\ \text{f}'(\text{x})=\frac{\text{f}(4)-\text{f}(-3)}{4+3}
\Rightarrow\frac{-\text{x}}{\sqrt{25-\text{x}^2}}=\frac{3-4}{7}
\Rightarrow49\text{x}^2=25-\text{x}^2
\Rightarrow\text{x}=\pm\frac{1}{\sqrt2}
Thus, \text{c}=\pm\frac{1}{\sqrt2}\in(-3,4) such that \text{f}'(\text{c})=\frac{\text{f}(4)-\text{f}(-3)}{4-(-3)}.

https://bls.smartstudies.co.in/#/exam/pdf-preview/c59cb220-8e86-4716-9ff7-82aec16b1ade/1 110/158
5/26/24, 6:19 PM Exam Automation
Hence, Lagrange's mean value theorem is verified.
Q524. Differentiate the function given in Exercise: 4 Marks
\text{x}^{\text{x}\cos\text{x}}+\frac{\text{x}^2+1}{\text{x}^2-1}

Ans: Let \text{x}^{\text{x}\cos\text{x}}+\frac{\text{x}^2+1}{\text{x}^2-1}


Putting \text{u}=\text{x}^{\text{x}\cos\text{x}}\text{ and v}=\frac{\text{x}^2+1}{\text{x}^2-1},\text{we have y}=\text{u}+\text{v}
\therefore\ \frac{\text{dy}}{\text{dx}}=\frac{\text{du}}{\text{dx}}+\frac{\text{dv}}{\text{dx}}\ \dots\text{(i)}
Now \text{u}=\text{x}^{\text{x}\cos\text{x}}\ \ \Rightarrow\ \log \text{u}=\log\text{x}^{\text{x}\cos\text{x}}\cos \text{x}=\text{x} \cos\text{x} \log\text{x}
\Rightarrow\ \frac{1}{\text{u}}\frac{\text{du}}{\text{dx}}=\frac{\text{d}}{\text{dx}}(\text{x}\cos\text{x}\log\text{x})
\Rightarrow\ \frac{1}{\text{u}}\frac{\text{du}}{\text{dx}}=\frac{\text{d}}{\text{dx}}(\text{x}).\cos\text{x}\log\text{x}+\text{x}\frac{\text{d}}{\text{dx}}
(\cos\text{x})\log\text{x}+\text{x}\cos\text{x}\frac{\text{d}}{\text{dx}}(\log\text{x})
\Rightarrow\ \frac{1}{\text{u}}\frac{\text{du}}{\text{dx}}=1.\cos\text{x}\log\text{x}+\text{x}(-\sin\text{x})\log\text{x}+\text{x}\cos\text{x}\frac{1}{\text{x}}
\Rightarrow\ \frac{\text{du}}{\text{dx}}=\text{u}(\cos\text{x}\log\text{x}-\text{x}\sin\text{x}\log\text{x}+\cos\text{x})
\Rightarrow\ \frac{\text{du}}{\text{dx}}=\text{x}^{\text{x}\cos\text{x}}(\cos\text{x}\log\text{x}-\text{x}\sin\text{x}\log\text{x}+\cos\text{x})\ \dots\text{(ii)}
Again \text{v}=\frac{\text{x}^2+1}{\text{x}^2-1}\ \Rightarrow\ \frac{\text{dv}}{\text{dx}}=\frac{(\text{x}^2-1)\frac{\text{d}}{\text{dx}}(\text{x}^2+1)-(\text{x}^2+1)\frac{\text{d}}{\text{dx}}
(\text{x}^2-1)}{(\text{x}^2-1)^2}
\Rightarrow\ \frac{\text{dv}}{\text{dx}}=\frac{(\text{x}^2-1)2\text{x}-(\text{x}^2+1)2\text{x}}{(\text{x}^2-1)^2}\ \Rightarrow\ \frac{\text{dv}}{\text{dx}}=\frac{2\text{x}^3-
2\text{x}-2\text{x}^3-2\text{x}}{(\text{x}^2-1)^2}
\Rightarrow\ \frac{\text{dv}}{\text{dx}}=\frac{-4\text{x}}{(\text{x}^2-1)^2}\ \dots\ \text{(iii)}
Putting the values from eq. (ii) and (iii) in eq. (i),
\frac{\text{dy}}{\text{dx}}=\text{x}^{\text{x}\cos\text{x}}(\cos\text{x}\log\text{x}-\text{x}\sin\text{x}\log\text{x}+\cos\text{x})+\frac{-4\text{x}}{(\text{x}^2-1)^2}
Q525. Show that the function defined by f(x) = | cos x | is a continuous function. 4 Marks

Ans: It is given function is \text{f(x)} = |\cos\text{x}|


The given function f is defined for real number and f can be written as the composition of two functions, as
f = goh, where, \text{g(x}) =| \text{x}|\ \text {and}\ \text{h(x)} = \cos\text{x}
First we have to prove that \text{g(x}) =| \text{x}|\ \text {and}\ \text{h(x)} = \cos\text{x} are continuous functions.
g(x) = lxl can be written as
\text{g(x)}=\begin{cases}-\text{x},&\text{if}\ \text{x}<{0}\\\text{x},& \text{if}\ \text{x}\geq0\end{cases}
Now, g is defined for all real number.
Let k be a real number.
Case I: If k < 0,
Then g(k) = -k
And ^{\ \ \text{lim}}_{\text{x}\rightarrow\text{k}}\text{g(x)} = ^{\ \ \text{lim}}_{\text{x}\rightarrow\text{k}}(-\text{x}) = -\text{k}
Thus ^{\ \ \text{lim}}_{\text{x}\rightarrow\text{k}}\text{g(x)} =\text{g(k)}
Therefore, g is continuous at all points x, i.e. x > 0
Case II: If k > 0,
Then g(k) = k and
^{\ \ \text{lim}}_{\text{x}\rightarrow\text{k}}\text{g(x)} =\text{g(k)}
Therefore, g is continuous at all points x, i.e. x < 0
Case III: If k = 0,
Then, g(k) = g(0) = 0
^{\ \ \text{lim}}_{\text{x}\rightarrow\text{0}^{-}}\text{g(x)} = ^{\ \ \text{lim}}_{\text{x}\rightarrow\text{0}^{-}}(-\text{x}) = 0
^{\ \ \text{lim}}_{\text{x}\rightarrow\text{0}^{+}}\text{g(x)} = ^{\ \ \text{lim}}_{\text{x}\rightarrow\text{0}^{+}}(\text{x}) = 0
\therefore^{\ \ \text{lim}}_{\text{x}\rightarrow\text{0}^{-}}\text{g(x)} = ^{\ \ \text{lim}}_{\text{x}\rightarrow\text{0}^{+}}\text g({\text x}) =\text{g}( 0)
Therefore, g is continuous at x = 0
From the above 3 cases, we get that g is continuous at all points.
h(x) = cosx
We know that h is defined for every real number.
Let k be a real number.
Now, put x = k + h
If x → k, then h → 0
^{\ \ \text{lim}}_{\text{x}\rightarrow\text{k}}\text{h(x)} =^{\ \ \text{lim}}_{\text{x}\rightarrow\text{k}}\cos\text{x}
= ^{\ \ \text{lim}}_{\text{h}\rightarrow\text{0}}\cos(\text{k}+\text{h})
= ^{\ \ \text{lim}}_{\text{h}\rightarrow\text{0}}[\cos\text{k}.\cos\text{h} - \sin\text{k}.\sin\text{h}]
= ^{\ \ \text{lim}}_{\text{h}\rightarrow\text{0}}\cos\text{k}\cos\text{h} - ^{\ \ \text{lim}}_{\text{h}\rightarrow\text{0}}\sin\text{k}\sin\text{h}
= \cos \text{k}\cos0 - \sin\text{k}\sin0
= \cos \text{k} \times 1 - \sin \times\ 0
=\cos \text{k}
\therefore\ ^{\ \ \text{lim}}_{\text{x}\rightarrow\text{k}}\text{h(x)} =\text{h(k)}
Thus, h(x) = cos x is continuous function.
We know that for real valued functions g and h, such that (goh) is defined at k, if g is continuous at k and if f is continuous at g(k),
Then (fog) is continuous at k.
Therefore,\text{ f(x)} = \text{(gof)(x)} = \text{g(h(x))} = \text{g}(\cos \text{x)}= |\cos\text{x}| is a continuous function.
Q526. Verify Rolle's theorem for the following function on the indicated intervals 4 Marks
f(x) = x2 - 8x + 12 on [2, 6]

Ans: Given:
f(x) = x2 - 8x + 12
We know that a polynomial function is everywhere derivable and hence continuous.
So, being a polynomial function f(x) is continuous and derivable on [2, 6].
f(2) = (2)2 - 8(2) + 12 = 4 - 16 + 12 = 0
f(6) = (6)2 - 8(6) + 12 = 36 - 48 + 12 = 0
\therefore f(2) = f(6) = 0
Thus, all the conditions of rolle's theorem are satisfied.
Now, we have to show that there exist \text{c}\in(2, 6) such that f'(c) = 0
We have
f(x) = x2 - 8x + 12
⇒ f'(x) = 2x - 8
\therefore f'(x) = 0
⇒ 2x - 8 = 0
⇒x=4
Thus, \text{c}=4\in(2,6) such that f'(c) = 0
Q527. If \text{y}=500\text{e}^{7\text{x}}+600\text{e}^{-7\text{x}} prove that \frac{\text{d}^2\text{y}}{\text{dx}^2}=49\text{y}. 4 Marks

Ans: It is given that, \text{y}=500\text{e}^{7\text{x}}+600\text{e}^{-7\text{x}}


Then
\frac{\text{dy}}{\text{dx}}=500.\frac{\text{d}}{\text{dx}}(\text{e}^{7\text{x}})+600.\frac{\text{d}}{\text{dx}}(\text{e}^{-7\text{x}})

https://bls.smartstudies.co.in/#/exam/pdf-preview/c59cb220-8e86-4716-9ff7-82aec16b1ade/1 111/158
5/26/24, 6:19 PM Exam Automation
=500.\text{e}^{7\text{x}}.\frac{\text{d}}{\text{dx}}(7\text{x})+600.\text{e}^{-7\text{x}}.\frac{\text{d}}{\text{dx}}(-7\text{x})
=3500\text{e}^{7\text{x}}-4200\text{e}^{-7\text{x}}
\therefore\frac{\text{d}^2\text{y}}{\text{dx}^2}=3500.\frac{\text{d}}{\text{dx}}(\text{e}^{7\text{x}})-4200.\frac{\text{d}}{\text{dx}}({-7\text{x}})
=3500.e^{7\text{x}}.\frac{\text{d}}{\text{dx}}(7\text{x})-4200.\text{e}^{-7\text{x}}.\frac{\text{d}}{\text{dx}}(-7\text{x})
=7\times3500.\text{e}^{7\text{x}}+7\times4200.\text{e}^{-7\text{x}}
=49\times500\text{e}^{7\text{x}}+49\times600\text{e}^{-7\text{x}}
=49(500\text{e}^{7\text{x}}+600\text{e}^{-7\text{x}})
=49\text{y}
Hence proved
Q528. A binary operation * on the set {0, 1, 2, 3, 4, 5} is defined as: 4 Marks
a * b = \begin{matrix} \text{a + b} & \text{if} & \text{a + b < 6} \\ \text{a + b - 6,} & \text{if} & \text{a + b }\geq6 \\ \end{matrix}.
Show that zero is the identity for this operation and each element 'a' of the set is, invertible with 6 – a, being the inverse of 'a'.

Ans: since a * 0 = a + 0 = a
and 0 * a = 0 + a = a
Note: \forall a \in {0, 1, 2, 3, 4, 5}
\therefore 0 is the identity for *.
Also,\forall a \in {0, 1, 2, 3, 4, 5}, a * (6 – a) = a + (6 – a) – 6
= 0 (which is identity)
Each element ‘a’ of the set is invertible with (6 – a), being the inverse of ‘a’.
Q529. Differentiate the functions given in Exercise: 4 Marks
\Big(\text{x}+\frac{1}{\text{x}}\Big)^\text{x}+\text{x}^{\Big(1+\frac{1}{\text{x}}\Big)}

Ans: Let \text{y}=\Big(\text{x}+\frac{1}{\text{x}}\Big)^\text{x}+\text{x}^{\Big(\text{x}+\frac{1}{\text{x}}\Big)}


Putting \text{y}=\Big(\text{x}+\frac{1}{\text{x}}\Big)^\text{x}=\text{u and }\text{x}^{\Big(\text{x}+\frac{1}{\text{x}}\Big)}=\text{v}
y = u + v \ \therefore\ \frac{\text{dy}}{\text{dx}}=\frac{\text{du}}{\text{dx}}+\frac{\text{dv}}{\text{dx}}\ \dots\text{(i)}
Now \text{u}=\Big(\text{x}+\frac{1}{\text{x}}\Big)^\text{x}\ \Rightarrow\ \log\text{u}=\log \Big(\text{x}+\frac{1}{\text{x}}\Big)^\text{x}=\text{x}\log\Big(\text{x}+\frac{1}{\text{x}}\Big)
\Rightarrow\ \frac{1}{\text{u}}\frac{\text{du}}{\text{dx}}=\text{x}.\frac{1}{\Big(\text{x}+\frac{1}{\text{x}}\Big)}\frac{\text{d}}{\text{dx}}\Big(\text{x}+\frac{1}
{\text{x}}\Big)+\log\Big(\text{x}+\frac{1}{\text{x}}\Big).1
\Rightarrow\ \frac{1}{\text{u}}\frac{\text{du}}{\text{dx}}=\text{x}.\frac{1}{\Big(\text{x}+\frac{1}{\text{x}}\Big)}\Big(\text{x}+\frac{1}{\text{x}}\Big)+\log\Big(\text{x}+\frac{1}{\text{x}}\Big).1
\Rightarrow\ \frac{\text{du}}{\text{dx}}=\text{u}\Big[\frac{\text{x}^2-1}{\text{x}^2+1}+\log\Big(\text{x}+\frac{1}{\text{x}}\Big)\Big]=\Big(\text{x}+\frac{1}
{\text{x}}\Big)^\text{x}\Big[\frac{\text{x}^2-1}{\text{x}^2+1}+\log\Big(\text{x}+\frac{1}{\text{x}}\Big)\Big]\ \dots\text{(ii)}
Again \text{v}=\text{x}^{\Big(\text{x}+\frac{1}{\text{x}}\Big)}\ \Rightarrow\ \log\text{v}=\log\text{x}^{\Big(\text{x}+\frac{1}{\text{x}}\Big)}=\Big(\text{x}+\frac{1}{\text{x}}\Big)\log\text{x}
\Rightarrow\ \frac{1}{\text{v}}\frac{\text{dv}}{\text{dx}}=\Big(\text{x}+\frac{1}{\text{x}}\Big).\frac{1}{\text{x}}+\log\text{x}.\Big(\frac{-1}{\text{x}^2}\Big)\ \Rightarrow\ \frac{\text{dv}}
{\text{dx}}=\text{v}\Big[\frac{1}{\text{x}}\Big(\text{x}+\frac{1}{\text{x}}\Big)-\frac{1}{\text{x}^2}\log\text{x}\Big]
\Rightarrow\ \frac{\text{dv}}{\text{dx}}=\text{x}^{\Big(\text{x}+\frac{1}{\text{x}}\Big)}\Big[\frac{1}{\text{x}}\Big(\text{x}+\frac{1}{\text{x}}\Big)-\frac{1}{\text{x}^2}\log\text{x}\Big]\
\dots\text{(iii)}
Putting the values from eq. (ii) and (iii) in eq. (i),
\frac{\text{dy}}{\text{dx}}=\Big(\text{x}+\frac{1}{\text{x}}\Big)^\text{x}\Big[\frac{\text{x}^2-1}{\text{x}^2+1}+\log\Big(\text{x}+\frac{1}{\text{x}}\Big)\Big]+\text{x}^{\Big(\text{x}+\frac{1}
{\text{x}}\Big)}\Big[\frac{1}{\text{x}}\Big(\text{x}+\frac{1}{\text{x}}\Big)-\frac{1}{\text{x}}^2\log\text{x}\Big]
Q530. If \text{y}=\tan^{-1}\Big(\frac{\sqrt{1+\text{x}}-\sqrt{1-\text{x}}}{\sqrt{1+\text{x}}+\sqrt{1+\text{x}}}\Big), find \frac{\text{dy}}{\text{dx}}. 4 Marks

Ans: Here, \text{y}=\tan^{-1}\Big(\frac{\sqrt{1+\text{x}}-\sqrt{1-\text{x}}}{\sqrt{1+\text{x}}+\sqrt{1+\text{x}}}\Big)


Put \text{x}=\cos2\theta
\therefore \text{y}=\tan^{-1}\Big(\frac{\sqrt{1+\cos2\theta}-\sqrt{1-\cos2\theta}}{\sqrt{1+\cos2\theta}+\sqrt{1-\cos2\theta}}\Big)
=\tan^{-1}\Big(\frac{\sqrt{2\cos^2\theta}-\sqrt{2\sin^2\theta}}{\sqrt{2\cos^2\theta}+\sqrt{2\sin^2\theta}}\Big)
=\tan^{-1}\Big(\frac{\sqrt{2}(\cos\theta-\sin\theta)}{\sqrt{2}(\cos\theta+\sin\theta)}\Big)
=\tan^{-1}\bigg(\frac{\frac{\cos\theta-\sin\theta}{\cos\theta}}{\frac{\cos\theta+\sin\theta}{\cos\theta}}\bigg)
[Dividing numerator and denomainator by \cos\theta]
=\tan^{-1}\bigg(\frac{\frac{\cos\theta}{\cos\theta}-\frac{\sin\theta}{\cos\theta}}{\frac{\cos\theta}{\cos\theta}+\frac{\sin\theta}{\cos\theta}}\bigg)
=\tan^{-1}\Big(\frac{1-\tan\theta}{1+\tan\theta}\Big)
=\tan^{-1}\bigg(\frac{\tan\frac{\pi}{4}-\tan\theta}{1+\tan\frac{\pi}{4}\times\tan\theta}\bigg)
=\tan^{-1}\Big[\tan\big(\frac{\pi}{4}-\theta\big)\Big]
=\frac{\pi}{4}-\theta
=\frac{\pi}{4}-\frac{1}{2}\cos^{-1}\text{x}\ (\text{Using x}=\cos2\theta)
Differentiate it with respect to x,
\frac{\text{dy}}{\text{dx}}=0-\frac{1}{2}\Big(\frac{-1}{\sqrt{1-\text{x}^2}}\Big)
\therefore \frac{\text{dy}}{\text{dx}}=\frac{1}{2\sqrt{1-\text{x}^2}}
Q531. Differentiate the following w.r.t. x: 4 Marks
\tan^{-1}\Big(\sqrt{\frac{1-\cos\text{x}}{1+\cos\text{x}}}\Big),\frac{-\pi}{4}<\text{x}<\frac{\pi}{4}

Ans: Let \text{y}=\tan^{-1}\Big(\sqrt{\frac{1-\cos\text{x}}{1+\cos\text{x}}}\Big)


=\tan^{-1}\Bigg(\sqrt{\frac{1-1+2\sin^2\frac{\text{x}}{2}}{1+2\cos^2\frac{\text{x}}{2}-1}}\Bigg) \Big[\because\cos=1-2\sin^2\frac{\text{x}}{2}2\cos^2\frac{\text{x}}{2}-1\Big]
=\tan^{-1}\bigg(\tan\frac{\text{x}}{2}\bigg)=\frac{\text{x}}{2}
On differentiating w. r. t. x, we get
\frac{\text{dy}}{\text{dx}}=\frac{1}{2}
Q532. Differentiate \sin^{-1}\Big(2\text{x}\sqrt{1-\text{x}^2}\Big) with respect to \tan^{-1}\Big(\frac{\text{x}}{\sqrt{1-\text{x}^2}}\Big), if -\frac{1}{\sqrt{2}}<\text{x}<\frac{1}{\sqrt{2}} 4 Marks

Ans: Let, \text{u}=\sin^{-1}(2\text{x}\sqrt{1-\text{x}^2})


Put \text{x}=\sin\theta
\Rightarrow\text{u}=\sin^{-1}\Big(2\sin\theta\sqrt{1-\sin^2\theta}\Big)
\Rightarrow\text{u}=\sin^{-1}(2\sin\theta\cos\theta)
\Rightarrow\text{u}=\sin^{-1}(\sin2\theta)\ .....(\text{i})
Let \text{v}=\tan^{-1}\Big(\frac{\text{x}}{\sqrt{1-\text{x}}}\Big)
\Rightarrow\text{v}=\tan^{-1}\Big(\frac{\sin\theta}{\sqrt{1-\sin^2\theta}}\Big)
\Rightarrow\text{v}=\tan^{-1}\Big(\frac{\sin\theta}{\cos\theta}\Big)
\Rightarrow\text{v}=\tan^{-1}(\tan\theta)\ .....(\text{ii})
Here, -\frac{1}{\sqrt{2}}<\text{x}<\frac{1}{\sqrt{2}}
\Rightarrow-\frac{1}{\sqrt{2}}<\sin\theta<\frac{1}{\sqrt{2}}
\Rightarrow-\frac{\pi}{4}<\theta<\frac{\pi}{4}
So, from equation (i),
\text{u}= 2\theta\bigg[\text{Since,}\sin^{-1}(\sin\theta)=\theta,\text{if }\theta\in\bigg[-\frac{\pi}{2},\frac{\pi}{2}\bigg]\bigg]
\Rightarrow\text{u}=2\sin^{-1}\text{x}[\text{Since, x}=\sin\theta]
Differentiating it with respect to x,
\frac{\text{du}}{\text{dx}}=\frac{2}{\sqrt{1-\text{x}^2}}\ .....(\text{iii})
From equation (ii),
\text{v}=\theta\bigg[\text{Since,}\tan^{-1}(\tan\theta)=\theta,\text{if }\theta\in\Big(-\frac{\pi}{2},\frac{\pi}{2}\Big)\bigg]
\Rightarrow\text{v}=\sin^{-1}\text{x}[\text{since, x}=\sin\theta]

https://bls.smartstudies.co.in/#/exam/pdf-preview/c59cb220-8e86-4716-9ff7-82aec16b1ade/1 112/158
5/26/24, 6:19 PM Exam Automation
Differentiating it with respect to x,
\frac{\text{dv}}{\text{dx}}=\frac{1}{\sqrt{1-\text{x}^2}}\ .....\text{(iv)}
Dividing equation (iii) by (iv)
\frac{\frac{\text{du}}{\text{dx}}}{\frac{\text{dv}}{\text{dx}}}=\frac{2}{\sqrt{1-\text{x}^2}}\times\frac{\sqrt{1-\text{x}^2}}{1}
\therefore\frac{\text{du}}{\text{dv}}=2
Q533. Find a point on the curve y = (x - 3)2, where the tangent is parallel to the chord joining the points (3, 0) and (4, 1). 4 Marks

Ans: Consider, y = (x - 3)2, which is continuous in x1, = 3 and x2 = 4 i.e., [3, 4]


Also, y' = 2(x - 3).1 = 2(x - 3) which exists in (3, 4).
Hence, by mean value theorem there exists a point on the curve at which tangent drawn is parallel to the chord joining the points (3, 0) and (4, 1).
Thus, \text{f}'(\text{c})=\frac{\text{f}(4)-\text{f}(3)}{4-3}
\Rightarrow\ 2(\text{c}-3)=\frac{(4-3)^2-(3-3)^2}{4-3}
\Rightarrow\ 2\text{c}-6=\frac{1-0}{1} \Rightarrow\ \text{c}=\frac{7}{2}
For \text{x}=\frac{7}{2}, \text{y}=\Big(\frac{7}{3}-3\Big)^2=\Big(\frac{1}{2}\Big)^2=\frac{1}{4}
So, \Big(\frac{7}{2},\frac{1}{4}\Big) is the point on the curve at which tangent drawn is parallel to the chord joining the points (3, 0) and (4, 1).
Q534. Prove that \int\limits_0^\text{a}\text{f(x)}\text{dx}=\int\limits\text{f}(\text{a}-\text{x})\text{dx}, and hence evaluate \int\limits_0^1\text{x}^2(1-\text{x})^\text{n}\text{dx}. 4 Marks

Ans: \int\limits^{\text{a}}_0\text{f(x)}\text{dx}=\int\limits^{\text{a}}_0\text{f}(\text{a}-\text{x})\text{dx}
Taking R.H.S.
=\int\limits^{\text{a}}_0\text{f}(\text{a}-\text{x})\text{dx}
Put \text{a}-\text{x}=\text{t}
-\text{dx}=\text{dt}
-\int\limits^{0}_\text{a}\text{f(t)}\text{dt}
by 2nd Property,
\therefore-\int\limits^{\text{b}}_{\text{a}}\text{f(x)}\text{dx}=\int\limits^{\text{a}}_{\text{b}}\text{f(x)}\text{dx}
=\int\limits^{\text{a}}_\text{0}\text{f(t)}\text{dt}
by 1st property,
\therefore\int\limits^{\text{b}}_{\text{a}}\text{f(x)}=\int\limits^{\text{b}}_\text{a}\text{f(t)}\text{dt}
\Rightarrow\int\limits^{\text{a}}_0\text{f(x)}\text{dx}=\text{LHS}
Now,
\text{I}=\int\limits^{1}_0\text{x}^2(1-\text{x})^{\text{n}}\text{dx}
\Rightarrow\int\limits^{1}_0[\text{x}^{\text{n}}+\text{x}^{\text{n}+2}-2\text{x}^{\text{n}+1}]\text{dx}
\text{I}=\Big[\frac{\text{x}^{\text{n}+1}}{\text{n}+1}\big]^1_0+\Big[\frac{\text{x}^{\text{n}+3}}{\text{n}+3}\big]^{1}_0-2\Big[\frac{\text{x}^{\text{n}+2}}{\text{n}+2}\big]^{1}_0
​\text{I}=\frac{1}{\text{n}+1}+\frac{1}{\text{n}+3}-\frac{2}{\text{n}+2}
Q535. Let \text{f}\text{(x)}=\frac{\log\Big(1+\frac{\text{x}}{\text{a}}\Big)-\log\Big(1-\frac{\text{x}}{\text{b}}\Big)}{\text{x}},\text{x}\neq0 Find the value of f at x = 0. So that f becomes continuous 4 Marks
at x = 0.

Ans: Given, \text{f}\text{(x)}=\frac{\log\Big(1+\frac{\text{x}}{\text{a}}\Big)-\log\Big(1-\frac{\text{x}}{\text{b}}\Big)}{\text{x}},\text{x}\neq0


If f(x) is continuous at x = 0, then
\lim\limits_{\text{x} \rightarrow 0}\text{f}\text{(x)}=\text{f}(0)
\Rightarrow\lim\limits_{\text{x} \rightarrow 0}\begin{pmatrix}\frac{\log\Big(1+\frac{\text{x}}{\text{a}}\Big)-\log\Big(1-\frac{\text{x}}{\text{b}}\Big)}{\text{x}}\end{pmatrix}=\text{f}(0)
\Rightarrow\lim\limits_{\text{x} \rightarrow 0}\begin{pmatrix}\frac{\log\Big(1+\frac{\text{x}}{\text{a}}\Big)}{\frac{\text{ax}}{\text{a}}}-\frac{\log\Big(1-\frac{\text{x}}{\text{b}}\Big)}
{\frac{\text{bx}}{\text{b}}}\end{pmatrix}=\text{f}(0)
\Rightarrow\lim\limits_{\text{x} \rightarrow 0}\begin{pmatrix}\frac{\log\Big(1+\frac{\text{x}}{\text{a}}\Big)}{\frac{\text{x}}{\text{a}}}\end{pmatrix}-\Big(-\frac{1}
{\text{b}}\Big)\lim\limits_{\text{x} \rightarrow 0}\begin{pmatrix}\frac{\log\Big(1-\frac{\text{x}}{\text{b}}\Big)}{\frac{-\text{x}}{\text{b}}}\end{pmatrix}=\text{f}(0)
\Rightarrow\frac{1}{\text{a}}\times1-\Big(-\frac{1}{\text{b}}\Big)\times1=\text{f}(0) \Big[\text{Using :}\lim_{\text{x} \rightarrow 0}\frac{\text{log(1}+\text{x)}}{\text{x}}=1\Big]
\Rightarrow\frac{1}{\text{a}}+\frac{1}{\text{b}}=\text{f}(0)
\Rightarrow\frac{\text{a}+\text{b}}{\text{ab}}=\text{f}(0)
Q536. Discuss the continuity of the function f, where f is defined by: 4 Marks
\text{f(x)}= \begin{cases}\ 2\text{x},\ \ \text{if}\ \text{x}<0 \\0,\ \ \ \ \text{if}\ 0\leq\text{x}\leq1\\4\text{x},\ \ \ \text{if}\ \text{x}>1\end{cases}

Ans: The given function is \text{f(x)}= \begin{cases}\ 2\text{x},\ \ \text{if}\ \text{x}<0 \\0,\ \ \ \ \text{if}\ 0\leq\text{x}\leq1\\4\text{x},\ \ \ \text{if}\ \text{x}>1\end{cases}
The function f is defined at all points of the real line.
Then, we have 5 cases i.e. k < 0, k = 0, 0 < k < 1, k = 1 or k < 1.
Now, Case I: k < 0
Then, f(k) = 2k
^{\ \ \text{lim}}_{\text{x}\rightarrow\text{k}}\text{f(x)} = ^{\ \ \text{lim}}_{\text{x}\rightarrow\text{k}}(\text{2x}) = \text{2k} = \text{f(k)}
Thus, ^{\ \ \text{lim}}_{\text{x}\rightarrow\text{k}}\text{f(x)} = \text{f(k)}
Hence, f is continuous at all points x, s.t. x < 0.
Case II: k = 0
f(0) = 0
^{\ \ \text{lim}}_{\text{x}\rightarrow\text{0}^{-}}\text{f(x)} = ^{\ \ \text{lim}}_{\text{x}\rightarrow\text{0}^{-}}(\text{2x}) = 2\times 0 = 0
^{\ \ \text{lim}}_{\text{x}\rightarrow\text{1}^{+}}\text{f(x)} = ^{\ \ \text{lim}}_{\text{x}\rightarrow\text{1}^{+}}(0) = 0
\Rightarrow\ ^{\ \ \text{lim}}_{\text{x}\rightarrow\text{k}^{-}}\text{f(x)} =^{\ \ \text{lim}}_{\text{x}\rightarrow\text{k}^{+}}\text{f(x)}=\text{f(k)}
Hence, f is not continuous at x = 0.
Case III: 0 < k < 1
Then, f(k) = 0
^{\ \ \text{lim}}_{\text{x}\rightarrow\text{k}}\text{f(x)} = ^{\ \ \text{lim}}_{\text{x}\rightarrow\text{k}}(0) = 0 = \text{f(k)}
Thus, ^{\ \ \text{lim}}_{\text{x}\rightarrow\text{k}}\text{f(x)} = \text{f(k)}
Hence, f is continuous in (0, 1).
Case IV: k = 1
Then f(k) = f(1) = 0
^{\ \ \text{lim}}_{\text{x}\rightarrow\text{1}^{-}}\text{f(x)} = ^{\ \ \text{lim}}_{\text{x}\rightarrow\text{1}^{-}}(0) = 0
^{\ \ \text{lim}}_{\text{x}\rightarrow\text{1}^{+}}\text{f(x)} = ^{\ \ \text{lim}}_{\text{x}\rightarrow\text{1}^{+}}(\text{4x}) = 4\times 1 = 4
\Rightarrow\ ^{\ \ \text{lim}}_{\text{x}\rightarrow\text{k}^{-}}\text{f(x)} \neq ^{\ \ \text{lim}}_{\text{x}\rightarrow\text{k}^{+}}\text{f(x)}
Hence, f is not continuous at x = 1.
Case V: k < 1
Then, f(k) = 4k
^{\ \ \text{lim}}_{\text{x}\rightarrow\text{k}}\text{f(x)} = ^{\ \ \text{lim}}_{\text{x}\rightarrow\text{k}}(\text{4x}) = 4\text{k} = \text{f(k)}
Thus ^{\ \ \text{lim}}_{\text{x}\rightarrow\text{k}}\text{f(x)} = \text{f(k)}
Hence, f is contionuous at all points x, s.t. x > 1.
Therefore, x = 1 is the only point of
discontinuity of f.
Q537. If \text{x}=\text{a}\sec\theta,\text{y}=b\tan\theta prove that \frac{\text{d}^2\text{y}}{\text{dx}^2}=-\frac{\text{b}^4}{\text{a}^2\text{y}^3} 4 Marks

Ans: Here,
\text{x}=\text{a}\sec\theta,\text{y}=b\tan\theta

https://bls.smartstudies.co.in/#/exam/pdf-preview/c59cb220-8e86-4716-9ff7-82aec16b1ade/1 113/158
5/26/24, 6:19 PM Exam Automation
Differentiating w.r.t.x, we get
\frac{\text{dx}}{\text{d}\theta}=\text{a}\sec\theta\tan\theta\ \text{and}\ \frac{\text{dy}}{\text{d}\theta}=b\sec^2\theta
\therefore\frac{\text{dy}}{\text{dx}}=\frac{\text{dy}}{\text{d}\theta}\times\frac{\text{d}\theta}{\text{dx}}=\frac{\text{b}\sec^2\theta}{\text{a}\sec\theta\tan\theta}=\frac{\text{b}\
\text{cosec}\theta}{\text{a}}
Differentiating w.r.t.x, we get
\frac{\text{d}^2\text{y}}{\text{dx}^2}=\frac{\text{b}}{\text{a}}\times-\text{cosec}\theta\cot\theta\times\frac{\text{d}\theta}{\text{dx}}
=-\frac{\text{b}}{\text{a}}\times\text{cosec}\theta\cot\theta\times\frac{1}{\text{a}\sec\theta\tan\theta}
=\frac{-\text{b}}{\text{a}^2}\times\frac{1}{\tan^3\theta}
\therefore\frac{\text{d}^2\text{y}}{\text{dx}^2}=\frac{-\text{b}}{\frac{\text{a}^2\text{y}^3}{\text{b}^3}}=-\frac{\text{b}^4}{\text{a}^2\text{y}^3}
Hence proved
Q538. If \text{y}=(\log\text{x})^{\text{x}}+\text{x}^{\log\text{x}}, then find \frac{\text{dy}}{\text{dx}}. 4 Marks

Ans: \text{y}=(\log\text{x})^{\text{x}}+\text{x}^{\log\text{x}}
Let \text{u}=(\log\text{x}^{\text{x}}) and \text{v}=\text{x}^{\log\text{x}}
Differentiating the above w.r.t. x, we get
\frac{\text{dy}}{\text{dx}}=\frac{\text{du}}{\text{dx}}+\frac{\text{dv}}{\text{dx}}\ ...(\text{i})
Now, \text{u}=(\log\text{x}^{\text{x}})
\log\text{u}=\text{x}.\log(\log\text{x})
\Rightarrow\frac{1}{\text{u}}.\frac{\text{du}}{\text{dx}}=\frac{1}{\log\text{x}}+\log(\log\text{x})
\frac{\text{du}}{\text{dx}}=(\log\text{x})^{\text{x}}\Big[\frac{1}{\log\text{x}}+\log(\log\text{x})\Big]\ ...(\text{ii})
\text{v}=\text{x}^{\log\text{x}}
\log\text{v}=\log\text{x}^2
\frac{1}{\text{v}}.\frac{\text{dv}}{\text{dx}}=2\log\text{x}.\frac{1}{\text{x}}
\frac{\text{dv}}{\text{dx}}=\text{x}^{\log\text{x}}\Big[\frac{2\log\text{x}}{\text{x}}\Big]\ ...(\text{3})
\frac{\text{dv}}{\text{dx}}=(\log\text{x}^{\text{x}})\Big[\frac{1}{\log\text{x}}+\log(\log\text{x})\Big]\\+\text{x}^{\log\text{x}}\Big[\frac{2\log\text{x}}{\text{x}}]
Q539. Find \frac{\text{dy}}{\text{dx}} 4 Marks
\text{y}=\text{x}^{\cos\text{x}}+(\sin\text{x})^{\tan\text{x}}

Ans: We have, \text{y}=\text{x}^{\cos\text{x}}+(\sin\text{x})^{\tan\text{x}}


\text{y}=\text{e}^{\log\text{x}^{\cos\text{x}}}+\text{e}^{\log(\sin\text{x})^{\tan\text{x}}}
\text{y}=\text{e}^{\cos\text{x}\log\text{x}}+\text{e}^{\tan\text{x}\log\sin\text{x}}
Differentiating with respect to x using chain rule,
\frac{\text{dy}}{\text{dx}}=\frac{\text{d}}{\text{dx}}\big(\text{e}^{\cos\text{x}\log\text{x}}\big)+\frac{\text{d}}{\text{dx}}\big(\text{e}^{\tan\text{x}\log\sin\text{x}}\big)
=\text{e}^{\cos\text{x}\log\text{x}}\frac{\text{d}}{\text{dx}}(\cos\text{x}\log\text{x}) \\ +\text{e}^{\tan\text{x}\log\sin\text{x}}\frac{\text{d}}{\text{dx}}(\tan\text{x}\log\sin\text{x})
=\text{e}^{\log\text{x}^{\cos\text{x}}}\Big[\cos\text{x}\frac{\text{d}}{\text{dx}}(\log\text{x})+\log\text{x}\frac{\text{d}}{\text{dx}}(\cos\text{x})\Big] \\
+\text{e}^{\log(\sin\text{x})^{\tan\text{x}}}\Big[\tan\text{x}\frac{\text{d}}{\text{dx}}\log\sin\text{x}+\log\sin\text{x}\frac{\text{d}}{\text{dx}}(\tan\text{x})\Big]
=\text{x}^{\cos\text{x}}\Big[\cos\text{x}\Big(\frac{1}{\text{x}}\Big)+\log\text{x}(-\sin\text{x})\Big] \\ +(\sin\text{x})^{\tan\text{x}}\Big[\tan\text{x}\Big(\frac{1}
{\sin\text{x}}\Big)\frac{\text{d}}{\text{dx}}(\sin)\text{x}+\log\sin\text{x}(\sec^2\text{x})\Big]
=\text{x}^{\cos\text{x}}\Big[\frac{\cos\text{x}}{\text{x}}-\sin\text{x}\log\text{x}\Big] \\ +(\sin\text{x})^{\tan\text{x}}\Big[\tan\text{x}\Big(\frac{1}{\sin\text{x}}\Big)
(\cos\text{x})+\sec^2\text{x}\log\sin\text{x}\Big]
=\text{x}^{\cos\text{x}}\Big[\frac{\cos\text{x}}{\text{x}}-\sin\text{x}\log\text{x}\Big] \\ +(\sin\text{x})^{\tan\text{x}}\big[1+\sec^2\text{x}\log\sin\text{x}\big]
Q540. Using mean value theorem, prove that there is a point on the curve y = 2x2 - 5x + 3 between the points A(1, 0) and B(2, 1), where tangent is parallel to the chord AB. Also, find that point. 4 Marks

Ans: We have, y = 2x2 - 5x + 3 which is continuous in [1, 2] as it is a polynomial function.


Also, y’ = 4x - 5, which exists in (1, 2).
By mean value theorem, \exists\text{ c}\in(1,2) at which drawn tangent is parallel to the chord AB,
where A and B are (1, 0) and (2, 1), respectively.
\therefore\ \text{f}'(\text{c})=\frac{\text{f}(2)-\text{f}(1)}{2-1}
\Rightarrow\ 4\text{c}-5=\frac{(8-10+3)-(2-5+3)}{1}
\Rightarrow\ 4\text{c}-5=1
\Rightarrow\ \text{c}=\frac{6}{4}=\frac{3}{2}\in(1,2)
For \text{x}=\frac{3}{2},\ \text{y}=2\Big(\frac{3}{2}\Big)^2-5\Big(\frac{3}{2}\Big)+3
=2\times\frac{9}{4}-\frac{15}{2}+3=\frac{9-15+6}{2}=0
Hence, \Big(\frac{3}{2},0\Big) is the point on the curve y = 2x2 - 5x + 3 between the points
A(1, 0) and B(2, 1) where tangent is parallel to the chord AB.
Q541. Find all points of discontinuity of f, where f is defined by: 4 Marks
\text {f(x)}=\begin{cases}\frac{\text x}{|\text x|},\ \ \text {if x}<0\\-1,\ \text{if x} \geq 0\end{cases}

Ans: Here \text {f(x)}=\begin{cases}\frac{\text x}{|\text x|},\ \ \text {if x}<0\\-1,\ \text{if x} \geq 0\end{cases}
Function f is defined for all points of the real line.
Let c be any real number.
Three cases arise:
Case I: c < 0
^{\ \ \text{Lt}}_{\text{x}\rightarrow\text{c}}\text{f(x)} = ^{\ \ \text{Lt}}_{\text{x}\rightarrow\text{c}}\frac{\text{x}}{|\text{x}|} = ^{\ \ \text{Lt}}_{\text{x}\rightarrow\text{c}}\big(\frac{\text{x}}
{\text{-x}}\big) =^{\ \ \text{Lt}}_{\text{x}\rightarrow\text{c}}(-1) = -1
Also \text{f(c)} = \frac{\text c} {|\text c|} = \frac{\text{c}}{-\text{c}} = -1
\therefore\ ^{\ \ \text{Lt}}_{\text{x}\rightarrow\text{c}}\text{f(x)} = \text{f(c)}
\therefore f is continuous at all point x < 0
Case II: c > 0
^{\ \ \text{Lt}}_{\text{x}\rightarrow\text{c}}\text{f(x)} = ^{\ \ \text{Lt}}_{\text{x}\rightarrow\text{c}}\frac{\text{x}}{|\text{x}|} = ^{\ \ \text{Lt}}_{\text{x}\rightarrow\text{c}}\big(\frac{\text{x}}
{\text{x}}\big) =^{\ \ \text{Lt}}_{\text{x}\rightarrow\text{c}}(1) = 1
Also \text{f(c)} = \frac{\text c} {|\text c|} = \frac{\text{c}}{\text{c}} = 1
\therefore\ ^{\ \ \text{Lt}}_{\text{x}\rightarrow\text{c}}\text{f(x)} = \text{f(c)}
\therefore f is continuous at all points x > 0.
Case III: c = 0
^{\ \ \text{Lt}}_{\text{x}\rightarrow\text{c}^{-}}\text{f(x)} = ^{\ \ \text{Lt}}_{\text{x}\rightarrow\text{0}^{-}}\frac{\text{x}}{|\text{x}|} = ^{\ \
\text{Lt}}_{\text{x}\rightarrow\text{0}^{-}}\big(\frac{\text{x}}{\text{-x}}\big) =^{\ \ \text{Lt}}_{\text{x}\rightarrow\text{0}^{-}}(-1) = -1
^{\ \ \text{Lt}}_{\text{x}\rightarrow\text{c}^{+}}\text{f(x)} = ^{\ \ \text{Lt}}_{\text{x}\rightarrow\text{0}^{+}}\frac{\text{x}}{|\text{x}|} = ^{\ \
\text{Lt}}_{\text{x}\rightarrow\text{0}^{+}}\big(\frac{\text{x}}{\text{x}}\big) =^{\ \ \text{Lt}}_{\text{x}\rightarrow\text{0}^{+}}(1) = 1
\therefore\ ^{\ \ \text{Lt}}_{\text{x}\rightarrow\text{c}^{-}}\text{f(x)} = ^{\ \ \text{Lt}}_{\text{x}\rightarrow\text{c}^{+}}\text{f(x)}
\therefore f is discontinuous at x = 0.
Q542. If (cos x)y = (sin y)x, find \frac{\text{dy}}{\text{dx}}. 4 Marks

Ans: (cos x)y = (sin y)x


\Rightarrow y log cos x = x log sin y
\therefore log (cos x). \frac{\text{dy}}{\text{dx}} - y . tan x = log sin y + x cot y . \frac{\text{dy}}{\text{dx}}
\therefore\frac{\text{dy}}{\text{dx}} (log cos x - x cot y) = log sin y + y tan x
\therefore\frac{\text{dy}}{\text{dx}}=\frac{\log\sin\text{y + y tan x}}{\text{log cos x - x}\cdot\text{cot y}}.

https://bls.smartstudies.co.in/#/exam/pdf-preview/c59cb220-8e86-4716-9ff7-82aec16b1ade/1 114/158
5/26/24, 6:19 PM Exam Automation
Q543. If \sin^{-1}\Big(\frac{3}{\text{x}}\Big)+\sin^{-1}\Big(\frac{4}{\text{x}}\Big)=\frac{\pi}{2}, then find the value of x. 4 Marks

Ans: \sin^{-1}\Big(\frac{3}{\text{x}}\Big)+\sin^{-1}\Big(\frac{4}{\text{x}}\Big)=\frac{\pi}{2}
\Rightarrow\sin^{-1}\Big(\frac{3}{\text{x}}\Big)=\frac{\pi}{2}-\sin^{-1}\Big(\frac{4}{\text{x}}\Big)=\cos^{-1}\Big(\frac{4}{\text{x}}\Big)
\Rightarrow\sin^{-1}\Big(\frac{3}{\text{x}}\Big)=\sin^{-1}\bigg(\sqrt{1-\frac{16}{\text{x}^2}}\bigg)
\Rightarrow\Big(\frac{3}{\text{x}}\Big)^2=\frac{\text{x}^2-16}{\text{x}^2}
\Rightarrow\text{x}^2=25
\Rightarrow\text{x}=\pm5,\ \text{x}=-5 (rejected)
\therefore\text{x}=5
Q544. Discuss the continuity of the function f, where f is defined by 4 Marks
\text{f(x)}= \begin{cases}\ 3,\ \ \text{if}\ 0\leq \text{x}\leq 1 \\4,\ \ \text{if}\ 1<\text{x}<3\\5,\ \text{if}\ 3\leq\text{x}\leq10\end{cases}

Ans: The given function is \text{f(x)}= \begin{cases}\ 3,\ \ \text{if}\ 0\leq \text{x}\leq 1 \\4,\ \ \text{if}\ 1<\text{x}<3\\5,\ \text{if}\ 3\leq\text{x}\leq10\end{cases}
The function f is definedc at all points of the
interval [0, 10].
Let k be the point in the interval [0, 10].
Then, we have 5 cases i. e. 0\leq\text{k}<1, \text{k}= 1, 1<\text{k}
<3, \text{k} = 3\ \text{or}\ 3 < \text{k}\leq 10.
Now, Case I: 0\leq\text{k}<1
Then, f(k) = 3
^{\ \ \text{lim}}_{\text{x}\rightarrow\text{k}}\text{f(x)} = ^{\ \ \text{lim}}_{\text{x}\rightarrow\text{k}}(3) = 3 = \text{f(k)}
Thus, ^{\ \ \text{lim}}_{\text{x}\rightarrow\text{k}}\text{f(x)} = \text{f(k)}
Hence, f is continuous in the interval [0, 10].
Case II: k = 1
f(1) = 3
^{\ \ \text{lim}}_{\text{x}\rightarrow\text{1}^{-}}\text{f(x)} = ^{\ \ \text{lim}}_{\text{x}\rightarrow\text{1}^{-}}(3) = 3
^{\ \ \text{lim}}_{\text{x}\rightarrow\text{1}^{+}}\text{f(x)} = ^{\ \ \text{lim}}_{\text{x}\rightarrow\text{1}^{+}}(4) = 4
\Rightarrow\ ^{\ \ \text{lim}}_{\text{x}\rightarrow\text{k}^{-}}\text{f(x)} \neq ^{\ \ \text{lim}}_{\text{x}\rightarrow\text{k}^{+}}\text{f(x)}
Hence, f is not continuous at x = 1.
Case III: 1 < k < 3
Then, f(x) = 4
^{\ \ \text{lim}}_{\text{x}\rightarrow\text{k}}\text{f(x)} = ^{\ \ \text{lim}}_{\text{x}\rightarrow\text{k}}(4) = 4 = \text{f(k)}
Thus, ^{\ \ \text{lim}}_{\text{x}\rightarrow\text{k}}\text{f(x)} = \text{f(k)}
Hence, f is continuous in (1, 3).
Case IV: k = 3
^{\ \ \text{lim}}_{\text{x}\rightarrow\text{3}^{-}}\text{f(x)} = ^{\ \ \text{lim}}_{\text{x}\rightarrow\text{3}^{-}}(4) = 4
^{\ \ \text{lim}}_{\text{x}\rightarrow\text{3}^{+}}\text{f(x)} = ^{\ \ \text{lim}}_{\text{x}\rightarrow\text{3}^{+}}(5) = 5
\Rightarrow\ ^{\ \ \text{lim}}_{\text{x}\rightarrow\text{k}^{-}}\text{f(x)} \neq ^{\ \ \text{lim}}_{\text{x}\rightarrow\text{k}^{+}}\text{f(x)}
Hence, f is not continuous at x = 3.
Case V: 3 < \text{k}\leq 10
Then, f(k) = 5
^{\ \ \text{lim}}_{\text{x}\rightarrow\text{k}}\text{f(x)} = ^{\ \ \text{lim}}_{\text{x}\rightarrow\text{k}}(5) = 5 = \text{f(k)}
Thus ^{\ \ \text{lim}}_{\text{x}\rightarrow\text{k}}\text{f(x)} = \text{f(k)}
Hence, f is contionuous at all points of the
interval [3, 10].
Therefore, x = 1 and 3 are the points of
discontinuity of f.
Q545. If \tan(\text{x}+\text{y})+\tan(\text{x}+\text{y})=1, find \frac{\text{dy}}{\text{dx}} 4 Marks

Ans: We have, \tan(\text{x}+\text{y})+\tan(\text{x}-\text{y})=1


Differentiating with respect to x, we get,
\Rightarrow\ \frac{\text{d}}{\text{dx}}\tan(\text{x}+\text{y})+\frac{\text{d}}{\text{dx}}\tan(\text{x}+\text{y})=\frac{\text{d}}{\text{dx}}(1)
\Rightarrow\ \sec^2(\text{x}+\text{y})\frac{\text{d}}{\text{dx}}(\text{x}+\text{y})+\sec^2(\text{x}-\text{y})\frac{\text{d}}{\text{dx}}(\text{x}-\text{y})=0
\Rightarrow\ \sec^2(\text{x}+\text{y})\Big[1+\frac{\text{dy}}{\text{dx}}\Big]+\sec^2(\text{x}-\text{y})\Big[1-\frac{\text{dy}}{\text{dx}}\Big]=0
\Rightarrow\ \sec^2(\text{x}+\text{y})\frac{\text{dy}}{\text{dx}}-\sec^2(\text{x}-\text{y})\frac{\text{dy}}{\text{dx}} \\ =-\big[\sec^2(\text{x}-\text{y})+\sec^2(\text{x}-\text{y})\big]
\Rightarrow\frac{\text{dy}}{\text{dx}}\big[\sec^2(\text{x}+\text{y})-\sec^2(\text{x}-\text{y})\big] \\ =-\big[\sec^2(\text{x}+\text{y})+\sec^2(\text{x}-\text{y})\big]
\Rightarrow\frac{\text{dy}}{\text{dx}}=\frac{\sec^2(\text{x}+\text{y})+\sec^2(\text{x}-\text{y})}{\sec^2(\text{x}-\text{y})-\sec^2(\text{x}+\text{y})}
Q546. Differentiate the following functions from first principles: 4 Marks
eax+b.

Ans: Let f(x) = eax+b


⇒ f(x + h) = ea(x+h)+b
\therefore\frac{\text{d}}{\text{dx}}(\text{f(x)})=\lim\limits_{\text{h}\rightarrow0}\frac{\text{f}(\text{x}+\text{h})-\text{f}(\text{x})}{\text{h}}
=\lim\limits_{\text{h}\rightarrow0}\frac{\text{e}^{\text{a}(\text{x}+\text{h})+\text{b}}-\text{e}^{(\text{ax}+\text{b})}}{\text{h}}
=\lim\limits_{\text{h}\rightarrow0}\frac{\text{e}^{\text{ax}+\text{b}}\text{e}^{\text{ah}}-\text{e}^{\text{ax}+\text{b}}}{\text{h}}
=\lim\limits_{\text{h}\rightarrow0}\text{ e}^{\text{ax}+\text{b}}\left\{\frac{(\text{e}^{\text{ah}}-1)}{\text{ah}}\right\}\times\text{a}
=\text{ae}^{\text{ax}+\text{b}} \lim\limits_{\text{h}\rightarrow0}\left\{\frac{(\text{e}^{\text{ah}}-1)}{\text{ah}}\right\}
=\text{ae}^{\text{ax}+\text{b}}
So,
\frac{\text{d}}{\text{dx}}(\text{e}^{\text{ax}}+\text{b})=\text{ae}^{\text{ax}+\text{b}}
Q547. If \text{y}=\text{e}^{2\text{x}}(\text{ax}+\text{b}), show that \text{y}_2-\text{4}\text{y}_1+4\text{y}=0 4 Marks

Ans: \text{y}=\text{e}^{2\text{x}}(\text{ax}+\text{b}),
Differentiating w.r.t.x,
\Rightarrow\frac{\text{dy}}{\text{dx}}=\text{e}^{2\text{x}}(\text{a})+2(\text{ax}+b)(\text{e}^{2\text{x}})
\Rightarrow\frac{\text{dy}}{\text{dx}}=\text{ae}^{2\text{x}}+2\text{y}
Differentiating w.r.t.x,
\Rightarrow\frac{\text{dy}}{\text{dx}}=2\text{ae}^{2\text{x}}+2\frac{\text{dy}}{\text{dx}}
\Rightarrow\frac{\text{d}^2\text{y}}{\text{dx}^2}=2\frac{\text{dy}}{\text{dx}}+2\text{ae}^{2\text{x}}+4\text{y}-4\text{y}=2\frac{\text{dy}}{\text{dx}}+2\frac{\text{dy}}{\text{dx}}-4\text{y}
\Rightarrow\frac{\text{d}^2\text{y}}{\text{dx}^2}-4\frac{\text{dy}}{\text{dx}}+4\text{y}=0
\Rightarrow\text{y}_2-4\text{y}_1+4\text{y}=0
Hence proved
Q548. Find the relationship between 'a' and 'b' so that the function 'f' defind by \text{f(x)}=\begin{cases}\text{ax}+1,&\text{if }\text{ x}\leq3\\\text{bx}+3,&\text{if }\text{ x}>3\end{cases} is 4 Marks
continuous at x = 3.

Ans: Given,
\text{f(x)}=\begin{cases}\text{ax}+1,&\text{if }\text{ x}\leq3\\\text{bx}+3,&\text{if }\text{ x}>3\end{cases}
We have,
(\text{LHL at x}= 3)=\lim_\limits{\text{x}\rightarrow3^-}\text{f(x)}=\lim_\limits{\text{h}\rightarrow0}\text{f}(3-\text{h})

https://bls.smartstudies.co.in/#/exam/pdf-preview/c59cb220-8e86-4716-9ff7-82aec16b1ade/1 115/158
5/26/24, 6:20 PM Exam Automation
=\lim_\limits{\text{h}\rightarrow0}\text{a}(3-\text{h})+1=3\text{a}+1
(\text{RHL at x}= 3)=\lim_\limits{\text{x}\rightarrow3^+}\text{f(x)}=\lim_\limits{\text{h}\rightarrow0}\text{f}(3+\text{h})
=\lim_\limits{\text{h}\rightarrow0}\text{b}(3+\text{h})+3=3\text{b}+3
If f(x) is continuous at x = 3, then
\lim_\limits{\text{x}\rightarrow3^-}\text{f(x)}=\lim_\limits{\text{x}\rightarrow3^+}\text{f(x)}
\Rightarrow3\text{a}+1=3\text{b}+3
\Rightarrow3\text{a}-3\text{b}=2
Hence, the required relationship between a & b is 3a - 2b = 2
Q549. Is the function defined by \text{f(x)}= \begin{cases}\text{x} + 5,\ \ \text{if x}\leq 1 \\\text{x} - 5,\ \ \text{if x}>1\end{cases} 4 Marks
a continuous function?

Ans: Here \text{f(x)}= \begin{cases}\text{x} + 5,\ \ \text{if x}\leq 1 \\\text{x} - 5,\ \ \text{if x}>1\end{cases}
Function f is defined at all points of the real line.
Let c be any real number.
Three cases arise:
Case I: c < 1
^{\ \ \text{Lt}}_{\text{x}\rightarrow\text{c}}\text{f(x)} = ^{\ \ \text{Lt}}_{\text{x}\rightarrow\text{c}}\text{(x} + 5) = \text{c} + 5
f(c) = c + 5
\therefore\ ^{\ \ \text{Lt}}_{\text{x}\rightarrow\text{c}}\text{f(x)} = \text{f(c)}
\therefore f is continuous at all points x < 1.
Case II: c > 1
^{\ \ \text{Lt}}_{\text{x}\rightarrow\text{c}}\text{f(x)} = ^{\ \ \text{Lt}}_{\text{x}\rightarrow\text{c}}\text{f(x} - 5) = \text{c} - 5
f(c) = c - 5
\therefore\ ^{\ \ \text{Lt}}_{\text{x}\rightarrow\text{c}}\text{f(x)} = \text{f(c)}
\therefore f is continuous at all points x > 1.
Case III: c = 1
^{\ \ \text{Lt}}_{\text{x}\rightarrow\text{1}^{-}}\text{f(x)} = ^{\ \ \text{Lt}}_{\text{x}\rightarrow\text{1}^{-}}\text{(x} + 5) = 1 + 5 = 6
^{\ \ \text{Lt}}_{\text{x}\rightarrow\text{1}^{+}}\text{f(x)} = ^{\ \ \text{Lt}}_{\text{x}\rightarrow\text{1}^{+}}\text{(x} - 5) = 1 - 5 = -4
\therefore\ ^{\ \ \text{Lt}}_{\text{x}\rightarrow\text{1}^{-}}\text{f(x)} \neq ^{\ \ \text{Lt}}_{\text{x}\rightarrow\text{1}^{+}}\text{f(x)}
\therefore f is discontinuous at x = 1.
Q550. Find all points of discontinuity of f, where f is defined by: 4 Marks
\text {f(x)}=\begin{cases}\frac{\left|\text x\right|}{\text x},\ \ \text {if x}\neq0\\0,\ \ \ \ \text{if x} = 0\end{cases}

Ans: Here \text {f(x)}=\begin{cases}\frac{\left|\text x\right|}{\text x},\ \ \text {if x}\neq0\\0,\ \ \ \ \text{if x} = 0\end{cases}
Function f is defined for all points of the real line.
Let c be any real number.
Three cases arise:
Case I: c < 0
^{\ \ \text{Lt}}_{\text{x}\rightarrow\text{c}}\text{f(x)} = ^{\ \ \text{Lt}}_{\text{x}\rightarrow\text{c}}\frac{|\text{x}|}{\text{x}} = ^{\ \ \text{Lt}}_{\text{x}\rightarrow\text{c}}\big(\frac{\text{-
x}}{\text{x}}\big) =^{\ \ \text{Lt}}_{\text{x}\rightarrow\text{c}}(-1) = -1
Also \text{f(c)} = \frac{|c|} {3} = \frac{-\text{c}}{\text{c}} = -1
\therefore\ ^{\ \ \text{Lt}}_{\text{x}\rightarrow\text{c}}\text{f(x)} = \text{f(c)}
\therefore f is continuous at all point x < 0
Case II: c > 0
^{\ \ \text{Lt}}_{\text{x}\rightarrow\text{c}}\text{f(x)} = ^{\ \ \text{Lt}}_{\text{x}\rightarrow\text{c}}\frac{|\text{x}|}{\text{x}} = ^{\ \ \text{Lt}}_{\text{x}\rightarrow\text{c}}\big(\frac{\text{x}}
{\text{x}}\big) =^{\ \ \text{Lt}}_{\text{x}\rightarrow\text{c}}(1) = 1
Also \text{f(c)} = \frac{|c|} {\text c} = \frac{\text{c}}{\text{c}} = 1
\therefore\ ^{\ \ \text{Lt}}_{\text{x}\rightarrow\text{c}}\text{f(x)} = \text{f(c)}
\therefore f is continuous at all points x > 0
Case III: c = 0
^{\ \ \text{Lt}}_{\text{x}\rightarrow\text{c}^{-}}\text{f(x)} = ^{\ \ \text{Lt}}_{\text{x}\rightarrow\text{0}^{-}}\frac{|\text{x}|}{\text{x}} = ^{\ \
\text{Lt}}_{\text{x}\rightarrow\text{0}^{-}}\big(\frac{\text{-x}}{\text{x}}\big) =^{\ \ \text{Lt}}_{\text{x}\rightarrow\text{0}^{-}}(-1) = -1
^{\ \ \text{Lt}}_{\text{x}\rightarrow\text{c}^{+}}\text{f(x)} = ^{\ \ \text{Lt}}_{\text{x}\rightarrow\text{0}^{+}}\frac{|\text{x}|}{\text{x}} = ^{\ \
\text{Lt}}_{\text{x}\rightarrow\text{0}^{+}}\big(\frac{\text{x}}{\text{x}}\big) =^{\ \ \text{Lt}}_{\text{x}\rightarrow\text{0}^{+}}(1) = 1
\therefore\ ^{\ \ \text{Lt}}_{\text{x}\rightarrow\text{c}^{-}}\text{f(x)} \neq ^{\ \ \text{Lt}}_{\text{x}\rightarrow\text{c}^{+}}\text{f(x)}
\therefore f is discontinuous at x = 0.
Q551. Show that \text{f}\text{(x)}=\begin{cases}\frac{\sin 3\text{x}}{\tan2\text{x}},&\text{if } \text{x}<0\\\frac{3}{2},&\text{if }\text{x} = 0\\\frac{\log(1+3\text{x})} 4 Marks
{\text{e}^{2\text{x}}},&\text{if}\text{ x}>0\end{cases} is discontinuous at x = 0.

Ans: Given,
\text{f}\text{(x)}=\begin{cases}\frac{\sin 3\text{x}}{\tan2\text{x}},&\text{if } \text{x}<0\\\frac{3}{2},&\text{if }\text{x} = 0\\\frac{\log(1+3\text{x})}{\text{e}^{2\text{x}}},&\text{if}\text{
x}>0\end{cases}
We observe
\text{(LHL at x}=0)=\lim\limits_{\text{x} \rightarrow 0^-}\text{f}\text{(x)}\\=\lim\limits_{\text{h} \rightarrow 0}\text{f}(0-\text{h)}=\lim\limits_{\text{h} \rightarrow 0}\text{f}(-\text{h)}
=\lim\limits_{\text{h} \rightarrow 0}\Big(\frac{\sin3(-\text{h})}{\tan2(-\text{h})}\Big)=\lim\limits_{\text{h} \rightarrow 0}\Big(\frac{\sin3\text{h}}{\tan2\text{h}}\Big)
=\lim\limits_{\text{h} \rightarrow 0}\Bigg(\frac{\frac{3\sin3\text{h}}{3\text{h}}}{\frac{2\tan2\text{h}}{2\text{h}}}\Bigg)
=\frac{\lim\limits_{\text{h} \rightarrow 0}\Big(\frac{3\sin3\text{h}}{3\text{h}}\Big)}{\lim\limits_{\text{h} \rightarrow 0}\Big(\frac{2\tan2\text{h}}{2\text{h}}\Big)}
=\frac{3\lim\limits_{\text{h} \rightarrow 0}\Big(\frac{\sin3\text{h}}{3\text{h}}\Big)}{2\lim\limits_{\text{h} \rightarrow 0}\Big(\frac{\tan2\text{h}}{2\text{h}}\Big)}
=\frac{3\times1}{2\times1}=\frac{3}{2}
\text{(RHL at x}=0)=\lim\limits_{\text{x} \rightarrow 0^+}\text{f}\text{(x)}\\=\lim\limits_{\text{h} \rightarrow 0}\text{f}(0+\text{h)}=\lim\limits_{\text{h} \rightarrow 0}\text{f}(\text{h)}
=\lim\limits_{\text{h} \rightarrow 0}\Big(\frac{\log(1+3\text{h})}{\text{e}^{2\text{h}}-1}\Big)
=\lim\limits_{\text{h} \rightarrow 0}\Bigg(\frac{3\text{h}\frac{\log(1+3\text{h})}{3\text{h}}}{2\text{h}\frac{(\text{e}^{2\text{h}-1})}{2\text{h}}}\Bigg)
\frac{3}{2}\lim\limits_{\text{h} \rightarrow 0}\Bigg(\frac{\frac{\log(1+3\text{h})}{3\text{h}}}{\frac{(\text{e}^{2\text{h}}-1)}{2\text{h}}}\Bigg)
=\frac{3}{2}\frac{\lim\limits_{\text{h} \rightarrow 0}\Big(\frac{\log(1+3\text{h})}{3\text{h}}\Big)}{\lim\limits_{\text{h} \rightarrow 0}\Big(\frac{(\text{e}^{2\text{h}-1})}{2\text{h}}\Big)}
=\frac{3\times1}{2\times1}=\frac{3}{2}
And, \text{f}(0)=\frac{3}{2}
\therefore\lim\limits_{\text{h} \rightarrow 0}\text{f}\text{(x)}=\lim\limits_{\text{h} \rightarrow 0^+}\text{f}\text{(x)}=\text{f}(0)
Thus, f(x) is continuous at x = 0.
Q552. AB is the diameter of a circle and C is any point on the circle. Show that the area of triangle ABC is maximum, when it is an isosceles triangle. 4 Marks

Ans:

https://bls.smartstudies.co.in/#/exam/pdf-preview/c59cb220-8e86-4716-9ff7-82aec16b1ade/1 116/158
5/26/24, 6:20 PM Exam Automation

Correct Figure
Let the length of sides of \triangle\ \text{ABC} are, AC = x and BC = y
⇒ x2 + y2 = 4r2 and Area A = \frac{1}{\text{2}}\text{xy}
\text{A}=\frac{1}{2}\text{x}\sqrt{4\text{r}^2-\text{x}^2}\ \ \text{or}\ \text{S}=\frac{\text{x}^2}{4}(4\text{r}^2-\text{x}^2)
\text{S}=\frac{1}{4}[4\text{r}^2\text{x}^2-\text{x}^4]
\therefore\ \frac{\text{ds}}{\text{dx}}=\frac{1}{4}[8\text{r}^2\text{x}-4\text{x}^3]
\frac{\text{ds}}{\text{dx}}=0\Rightarrow2\text{r}^2=\text{x}^2\Rightarrow\text{x}=\sqrt2\text{r}
\text{and}\ \text{y}=\sqrt{4\text{r}^2-2\text{r}^2}=\sqrt2\text{r}
\text{and}\frac{\text{d}^2\text{s}}{\text{ds}^2}=\frac{1}{4}[8\text{r}^2-12\text{x}^2]=\frac{1}{4}[8\text{r}^2-24\text{r}^2]<0
\therefore\ For maximum area, x = y i.e., \triangle\ is isosceles.
Q553. Differentiate the following with respect to x: 4 Marks
\sin^{-1}\bigg(\frac{2^{\text{x} + 1}.3^{\text{x}}}{1 + (36)^{\text{x}}}\bigg)

Ans: Let \text{y} = \sin^{-1}\bigg(\frac{2^{\text{x} + 1 }.3^{\text{x}}}{1 +(36)^{\text{x}}}\bigg) = \sin^{-1}\bigg(\frac{2.2^{\text{x}}.3^{\text{x}}}{1 + (6^{2})^{\text{x}}}\bigg) =


\sin^{-1}\bigg(\frac{2.6^{\text{x}}}{1 + (6^{\text{x}})^{2}}\bigg)
Let 6^{\text{x}} = \tan\theta\Rightarrow\theta = \tan^{-1}(6^{\text{x}})
\therefore\text{y} = \sin^{-1}\bigg(\frac{2\tan\theta}{1 + \tan^{2}\theta}\bigg)\Rightarrow\text{y} = \sin^{-1}(\sin2\theta)
\Rightarrow\text{y} = 2 \theta\Rightarrow\text{y} = 2.\tan^{-1}(6^{\text{x}})
\Rightarrow\frac{\text{dy}}{\text{dx}} = \frac{2}{1 + (6^{\text{x}})^{2}}.\log_{e}6.6^{\text{x}}\Rightarrow\frac{\text{dy}}{\text{dx}} = \frac{2.6^{\text{x}}.\log_{e}6}{1 + 36^{\text{x}}}.
Q554. If (\text{x}-\text{y})\text{e}^{\frac{\text{x}}{\text{x}-\text{y}}}=\text{a}, prove that \text{y}\frac{\text{dy}}{\text{dx}}+\text{x}=2\text{y} 4 Marks

Ans: Consider the given function, (\text{x}-\text{y})\text{e}^{\frac{\text{x}}{\text{x}-\text{y}}}=\text{a}.


We need to prove that \text{y}\frac{\text{dy}}{\text{dx}}+\text{x}=2\text{y}.
Differentiating the given equation w.r.t 'x' we get
(\text{x}-\text{y})\Bigg[\text{e}^{\frac{\text{x}}{\text{x}-\text{y}}}\Bigg(\frac{(\text{x}-\text{y})-\text{x}\big(1-\frac{\text{dy}}{\text{dx}}\big)}{(\text{x}-
\text{y})^2}\Bigg)\Bigg]+\text{e}^\frac{\text{x}}{\text{x}-\text{y}}\Big(1-\frac{\text{dy}}{\text{dx}}\Big)=0
\Rightarrow\frac{(\text{x}-\text{y})-\text{x}\Big(1-\frac{\text{dy}}{\text{dx}}\Big)}{(\text{x}-\text{y})}+\Big(1-\frac{\text{dy}}{\text{dx}}\Big)=0
\Rightarrow\Big(1+\frac{\text{dy}}{\text{dx}}\Big)\Big(1-\frac{\text{x}}{\text{x}-\text{y}}\Big)+1=0
\Rightarrow\Big(1+\frac{\text{dy}}{\text{dx}}\Big)\Big(\frac{-\text{y}}{\text{x}-\text{y}}\Big)+1=0
\Rightarrow-\text{y}+\text{y}\frac{\text{dy}}{\text{dx}}+\text{x}-\text{y}=0
\Rightarrow\text{y}\frac{\text{dy}}{\text{dx}}+\text{x}=2\text{y}
Q555. If \sqrt{1-\text{x}^2}+\sqrt{1-\text{y}^2}=\text{a}(\text{x}-\text{y}), prove that \frac{\text{dy}}{\text{dx}}=\sqrt{\frac{1-\text{y}^2}{1-\text{x}^2}}. 4 Marks

Ans: We have, \sqrt{1-\text{x}^2}+\sqrt{1-\text{y}^2}=\text{a}(\text{x}-\text{y})


On putting \text{x}=\sin\alpha and \text{y}=\sin\beta, we get
\sqrt{1-\sin^2\alpha}+\sqrt{1-\sin\beta}=\text{a}(\sin\alpha-\sin\beta)
\Rightarrow\ \cos\alpha+\cos\beta=\text{a}(\sin\alpha-\sin\beta)
\Rightarrow\ 2\cos\frac{\alpha+\beta}{2}\cdot\cos\frac{\alpha-\beta}{2}=2\text{a}\cos\frac{\alpha+\beta}{2}\sin\frac{\alpha-\beta}{2}
\Rightarrow\ \cos\frac{\alpha-\beta}{2}=\text{a}\sin\frac{\alpha-\beta}{2}
\Rightarrow\ \cot\frac{\alpha-\beta}{2}=\text{a}
\Rightarrow\ \frac{\alpha-\beta}{2}=\cot^{-1}\text{a}
\Rightarrow\ \alpha-\beta=2\cot^{-1}\text{a}
\Rightarrow\ \sin^{-1}\text{x}-\sin^{-1}\text{y}=2\cot^{-1}\text{a} [\because\text{x}=\sin\alpha\text{ and y}=\sin\beta]
Differentiating both sides w.r.t. x, we get
\frac{1}{\sqrt{1-\text{x}^2}}-\frac{1}{\sqrt{1-\text{y}^2}}\frac{\text{dy}}{\text{dx}}=0
\therefore\ \frac{\text{dy}}{\text{dx}}=\frac{\sqrt{1-\text{y}^2}}{\sqrt{1-\text{x}^2}}
Q556. Differentiate the following functions with respect to x: 4 Marks
\sin\Big(\frac{1+\text{x}^2}{1-\text{x}^2}\Big)

Ans: Let \text{y}=\sin\Big(\frac{1+\text{x}^2}{1-\text{x}^2}\Big)


Differentiate it with respect to x,
\frac{\text{dy}}{\text{dx}}=\frac{\text{d}}{\text{dy}}\Big(\sin\Big(\frac{1+\text{x}^2}{1-\text{x}^2}\Big)\Big)
=\cos\Big(\frac{1+\text{x}^2}{1-\text{x}^2}\Big)\frac{\text{d}}{\text{dx}}\Big(\frac{1+\text{x}^2}{1-\text{x}^2}\Big)
[Using chain rule]
=\cos\Big(\frac{1+\text{x}^2}{1-\text{x}^2}\Big)\Bigg[\frac{(1-\text{x}^2)\frac{\text{d}}{\text{dx}}(1+\text{x}^2)-(1+\text{x}^2)\frac{\text{d}}{\text{dx}}(1-\text{x}^2)}{(1-\text{x})^2}\Bigg]
[Using chain rule]
=\cos\Big(\frac{1+\text{x}^2}{1-\text{x}^2}\Big)\bigg[\frac{(1-\text{x}^2)(2\text{x})-(1+\text{x}^2)(-2\text{x})}{(1-\text{x}^2)^2}\bigg]
=\cos\Big(\frac{1+\text{x}^2}{1-\text{x}^2}\Big)\Big[\frac{2\text{x}-2\text{x}^3+2\text{x}+2\text{x}^3}{(1-\text{x}^2)^2}\Big]
=\frac{4\text{x}}{\big(1-\text{x}^2\big)^2}\cos\Big(\frac{1+\text{x}^2}{1-\text{x}^2}\Big)
So,
\frac{\text{d}}{\text{dx}}\Big(\sin\Big(\frac{1+\text{x}^2}{1-\text{x}^2}\Big)\Big)=\frac{4\text{x}}{\big(1-\text{x}^2\big)^2}\cos\Big(\frac{1+\text{x}^2}{1-\text{x}^2}\Big)
Q557. Differentiate the following functions with respect to x: 4 Marks
(\tan\text{x})^\frac{1}{\text{x}}

Ans: Let \text{y}=(\tan\text{x})^\frac{1}{\text{x}}\ .....(\text{i})


Taking log on both the sides,
\log\text{y}=\log(\tan\text{x})^\frac{1}{\text{x}}
\log\text{y}=\frac{1}{\text{x}}\log(\tan\text{x})\ \big[\text{Since}, \log\text{a}^\text{b}=\text{b}\log\text{a}\big]
Differentiating it with respect to x using product rule and chain rule,
\frac{1}{\text{y}}\frac{\text{dy}}{\text{dx}}=\frac{1}{\text{x}}\frac{\text{d}}{\text{dx}}\log(\tan\text{x})+\log(\tan\text{x})\frac{\text{d}}{\text{dx}}\Big(\frac{1}{\text{x}}\Big)
=\frac{1}{\text{x}}\times\frac{1}{\tan\text{x}}\frac{\text{d}}{\text{dx}}(\tan\text{x})+\log(\tan\text{x})\Big(-\frac{1}{\text{x}^2}\Big)
\frac{1}{\text{y}}\frac{\text{dy}}{\text{dx}}=\frac{1}{\text{x}\tan\text{x}}(\sec^2\text{x})-\frac{\log(\tan\text{x})}{\text{x}^2}
\frac{\text{dy}}{\text{dx}}=\text{y}\Big[\frac{\sec^2\text{x}}{\text{x}\tan\text{x}}-\frac{\log(\tan\text{x})}{\text{x}^2}\Big]
\frac{\text{dy}}{\text{dx}}=(\tan\text{x})^\frac{1}{\text{x}}\Big[\frac{\sec^2\text{x}}{\text{x}\tan\text{x}}-\frac{\log(\tan\text{x})}{\text{x}^2}\Big]
[Using equation (i)]
Q558. Find all point of discontinuity of the function \text{f(t)}=\frac{1}{\text{t}^2+\text{t}-2}, where \text{t}=\frac{1}{\text{x}-1} 4 Marks

Ans: \text{f(t)}=\frac{1}{\text{t}^2+\text{t}-2}, where \text{t}=\frac{1}{\text{x}-1}

https://bls.smartstudies.co.in/#/exam/pdf-preview/c59cb220-8e86-4716-9ff7-82aec16b1ade/1 117/158
5/26/24, 6:20 PM Exam Automation
Clearly \text{t}=\frac{1}{\text{x}-1} is discontinuous at x = 1
For \text{x}\neq1, we get
\text{f(t)}=\frac{1}{\text{t}^2+\text{t}-2}=\frac{1}{(\text{t}-2)(\text{t}-1)}
This is discontinuous at t = -2 and t = 1
For t = -2, \text{t}=\frac{1}{\text{x}-1}
\Rightarrow\text{x}=\frac{1}{2}
For t = 1, \text{t}=\frac{1}{\text{x}-1}
⇒x=2
Hence, f is discontinuous at \text{x}=\frac{1}{2}, x = 1 and x = 2
Q559. \text{If (x}-\text{a})^2+(\text{y}-\text{b})^2=\text{c}^2, for some c > 0 , prove that 4 Marks
\frac{\Big[1+\Big(\frac{\text{dy}}{\text{dx}}\Big)^2\Big]^{\frac{3}{2}}}{\frac{\text{d}^2\text{y}}{\text{dx}^2}}
is a constant independent of a and b.

Ans: It is given that, \text{(x}-\text{a})^2+(\text{y}-\text{b})^2=\text{c}^2


Differentiating both sides with respect to x, we obtain
\frac{\text{d}}{\text{dx}}[(\text{x}-\text{a})^2]+\frac{\text{d}}{\text{dx}}[(\text{y}-\text{b})^2]=\frac{\text{d}}{\text{dx}}(\text{c}^2)
\Rightarrow\ 2(\text{x}-\text{a}).\frac{\text{d}}{\text{dx}}(\text{x}-\text{a})+2(\text{y}-\text{b}).\frac{\text{d}}{\text{dx}}(\text{y}-\text{b})=0
\Rightarrow\ \frac{\text{dy}}{\text{dx}}=\frac{-(\text{x}-\text{a})}{\text{y}-\text{b}}\ \dots(1)
\therefore\frac{\text{d}^2\text{y}}{\text{dx}^2}=\frac{\text{d}}{\text{dx}}\Big[\frac{-(\text{x}-\text{a})}{\text{y}-\text{b}}\Big]
=-\Bigg[\frac{(\text{y}-\text{b}).\frac{\text{d}}{\text{dx}}(\text{x}-\text{a})-(\text{x}-\text{a}).\frac{\text{d}}{\text{dx}}(\text{y}-\text{b})}{(\text{y}-\text{b})^2}\Bigg]
=-\Bigg[\frac{(\text{y}-\text{b})-(\text{x}-\text{a}).\frac{\text{dy}}{\text{dx}}}{(\text{y}-\text{b})^2}\Bigg]
=-\Bigg[\frac{(\text{y}-\text{b})-(\text{x}-\text{a}).\Big\{\frac{-(\text{x}-\text{a})}{\text{y}-\text{b}}\Big\}}{(\text{y}-\text{b})^2}\Bigg]\ \ [\text{using (1)}]
=-\Big[\frac{(\text{y}-\text{b})^2+(\text{x}-\text{a})^2}{(\text{y}-\text{b})^3}\Big]
\therefore\ \Bigg[\frac{1+\Big(\frac{\text{dy}}{\text{dx}}\Big)^2}{\frac{\text{d}^2\text{y}}{\text{dx}^2}}\Bigg]^{\frac{3}{2}} =\frac{\Bigg[1+\frac{(\text{x}-\text{a})^2}{(\text{y}-
\text{b})^2}\Bigg]^{\frac{3}{2}}}{-\Bigg[\frac{(\text{y}-\text{b})^2+(\text{x}-\text{a})^2}{(\text{y}-\text{b})^3}\Bigg]} =\frac{\Bigg[\frac{(\text{y})-\text{b})^2+(\text{x}-\text{a})^2}{(\text{y}-
\text{b})^2}\Bigg]^{\frac{3}{2}}}{-\Bigg[\frac{(\text{y}-\text{b})^2+(\text{x}-\text{a})^2}{(\text{y}-\text{b})^3}\Bigg]}
=\frac{\Bigg[\frac{\text{c}^2}{(\text{y}-\text{b})^2}\Bigg]^{\frac{3}{2}}}{-\frac{\text{c}^2}{(\text{y}-\text{b})^3}}=\frac{\frac{\text{c}^3}{(\text{y}-\text{b})^3}}{-\frac{\text{c}^2}{(\text{y}-
\text{b})^3}}
= -c, which is constant and is independent of a and b
Hence, proved.
Q560. If \text{y}=(\sin\text{x})^{(\sin\text{x})^{(\sin\text{x})^{....\infty}}}, prove that \frac{\text{y}^2\cot\text{x}}{(1-\text{y}\log\sin\text{x})} 4 Marks

Ans: Here,
\text{y}=(\sin\text{x})^{(\sin\text{x})^{(\sin\text{x})^{....\infty}}}
\Rightarrow\text{y}=(\sin\text{x})^\text{y}
Taking log on both sides,
\log\text{y}=\log(\sin\text{x})^{\text{y}}
\log\text{y}=\text{y}(\log\sin\text{x})
Differentiating it with respect to x, using product rule,
\frac{1}{\text{y}}\frac{\text{dy}}{\text{dx}}=\text{y}\frac{\text{d}}{\text{dx}}(\log\sin\text{x})+\log\sin\text{x}\frac{\text{dy}}{\text{dx}}
\frac{1}{\text{y}}\frac{\text{dy}}{\text{dx}}=\text{y}\frac{1}{\sin\text{x}}\frac{\text{d}}{\text{dx}}(\sin\text{x})+\log\sin\text{x}\frac{\text{dy}}{\text{dx}}
\frac{\text{dy}}{\text{dx}}\Big(\frac{1}{\text{y}}-\log\sin\text{x}\Big)=\frac{\text{y}}{\sin\text{x}}(\cot\text{x})
\frac{\text{dy}}{\text{dx}}\Big(\frac{1-\text{y}\log\sin\text{x}}{\text{y}}\Big)=\text{y}\cot\text{x}
\frac{\text{dy}}{\text{dx}}=\Big(\frac{\text{y}^2\cot\text{x}}{1-\text{y}\log\sin\text{x}}\Big)
Q561. If \text{x}=\sin\text{t} and \text{y}=\sin\text{pt}, prove that (1-\text{x}^2)\frac{\text{d}^2\text{y}}{\text{dx}^2}-\text{x}\frac{\text{dy}}{\text{dx}}+\text{p}^2\text{ y}=0. 4 Marks

Ans: We have, \text{x}=\sin\text{t} and \text{y}=\sin\text{pt}


\therefore\ \frac{\text{dx}}{\text{dt}}=\cos\text{t} and \frac{\text{dy}}{\text{dt}}=\text{p}\cos\text{pt}
\Rightarrow\ \frac{\text{dy}}{\text{dx}}=\frac{\frac{\text{dy}}{\text{dt}}}{\frac{\text{dx}}{\text{dt}}}=\frac{\text{p}\cdot\cos\text{pt}}{\cos\text{t}}
\Rightarrow\ \text{y}'=\frac{\text{p}\cdot\cos\text{pt}}{\cos\text{t}}
\Rightarrow\ \text{y}'^2\cos^2\text{t}=\text{p}^2\cos^2\text{pt}
\Rightarrow\ \text{y}'^{2}(1-\sin^2\text{t})=\text{p}^2(1-\sin^2\text{pt})
\Rightarrow\ \text{y}'^2(1-\text{x}^2)=\text{p}^2(1-\text{y}^2)
Differentiating above w.r.t. x, we get
2\text{y}'\text{y}''(1-\text{x}^2)-2\text{xy}'^2=\text{p}^2(-2\text{yy}')
\Rightarrow\ \text{y}''(1-\text{x}^2)-\text{xy}'=-\text{p}^2\text{y}
\Rightarrow\ \text{y}''(1-\text{x}^2)-\text{xy}'+\text{p}^2\text{y}=0
Q562. Prove that the function \text{f(x)}=\begin{cases}\frac{\sin\text{x}}{\text{x}},&\text{x}<0\\\text{x}+1,&\text{x}\geq0\end{cases} is everywhere continuous. 4 Marks

Ans: When x < 0, we have


​\text{f(x)}=\frac{\sin\text{x}}{\text{x}}
We know that \sin\text{x} as well as the identity function x are everywhere continuous.
So, the quotient function ​\text{f(x)}=\frac{\sin\text{x}}{\text{x}} is continuous at each x < 0
When x > 0, we have f(x) = x + 1, which is a polynomial function.
Therefore, f(x) is continuous at each x > 0
Now, Let us consider the point x = 0
Given, \text{f(x)}=\begin{cases}\frac{\sin\text{x}}{\text{x}},&\text{x}<0\\\text{x}+1,&\text{x}\geq0\end{cases}
We have,
(\text{LHL at x}= 0)=\lim_\limits{\text{x}\rightarrow0^-}\text{f(x)}=\lim_\limits{\text{h}\rightarrow0}\text{f}(0-\text{h})
=\lim_\limits{\text{h}\rightarrow0}\text{f}(-\text{h})=\lim_\limits{\text{h}\rightarrow0}\Big(\frac{\sin(-\text{h})}{-\text{h}}\Big)=\lim_\limits{\text{h}\rightarrow0}\Big(\frac{\sin(\text{h})}
{\text{h}}\Big)=1
(\text{RHL at x}= 0)=\lim_\limits{\text{x}\rightarrow0^+}\text{f(x)}=\lim_\limits{\text{h}\rightarrow0}\text{f}(0+\text{h})
=\lim_\limits{\text{h}\rightarrow0}\text{f}(\text{h})=\lim_\limits{\text{h}\rightarrow0}(\text{h}+1)=1
Also,
\text{f}(0)=0+1=1
\therefore\ \lim_\limits{\text{x}\rightarrow0^-}\text{f(x)}=\lim_\limits{\text{x}\rightarrow0^+}\text{f(x)}=\text{f}(0)
Thus, f(x) is continuous at x = 0
Hence, f(x) is everywhere continuous.
Q563. If \text{y}=\text{e}^{\text{x}}+\text{e}^{-\text{x}}, prvoe that \frac{\text{dy}}{\text{dx}}=\sqrt{\text{y}^2-4} 4 Marks

Ans: Differentiating with respect to x,


\frac{\text{dy}}{\text{dx}}=\frac{\text{d}}{\text{dx}}\big(\text{e}^{\text{x}}+\text{e}^{-\text{x}}\big)
=\frac{\text{d}}{\text{dx}}\text{e}^{\text{x}}+\frac{\text{d}}{\text{dx}}{\text{e}}^{-\text{x}}
=\text{e}^{\text{x}}+\text{e}^{-\text{x}}\frac{\text{d}}{\text{dx}}\big(-\text{x}\big)
[Using chain rule]
=\text{e}^{\text{x}}+\text{e}^{-\text{x}}(-1)
=\big(\text{e}^{\text{x}}-\text{e}^{-\text{x}}\big)
=\sqrt{\big(\text{e}^{\text{x}}-\text{e}^{-\text{x}}\big)^2-4\text{e}^{\text{x}}\times\text{e}^{-\text{x}}}

https://bls.smartstudies.co.in/#/exam/pdf-preview/c59cb220-8e86-4716-9ff7-82aec16b1ade/1 118/158
5/26/24, 6:20 PM Exam Automation
\Big[\text{Since},(\text{a}-\text{b}=\sqrt{(\text{a}+\text{b})^2-4\text{ab}}\Big]
=\sqrt{\text{y}^2-4}
\big[\text{Since e}^\text{x}+\text{e}^{-\text{x}}=\text{y}\big]
Hence, the solution is, \frac{\text{dy}}{\text{dx}}=\sqrt{\text{y}^2-4}
Q564. If \text{y}=\text{x}\sin(\text{a}+\text{y}), prove that \frac{\text{dy}}{\text{dx}}=\frac{\sin^2(\text{a}+\text{y})}{\sin(\text{a}+\text{y})-\text{y}\cos(\text{a}+\text{y})} 4 Marks

Ans: Here,
\text{y}=\text{x}\sin(\text{a}+\text{y})
Differentiating with respect to x using the chain rule and product rule,
\frac{\text{dy}}{\text{dx}}=\text{x}\frac{\text{d}}{\text{dx}}\sin(\text{a}+\text{y})+\sin(\text{a}+\text{y})\frac{\text{dx}}{\text{dx}}
\frac{\text{dy}}{\text{dx}}=\text{x}\cos(\text{a}+\text{y})\frac{\text{dy}}{\text{dx}}+\sin(\text{a}+\text{y})
(1-\text{x}\cos(\text{a}+\text{y}))\frac{\text{dy}}{\text{dx}}=\sin(\text{a}+\text{y})
\frac{\text{dy}}{\text{dx}}=\frac{\sin(\text{a}+\text{y})}{(1-\text{x}\cos(\text{a}+\text{y}))}
\frac{\text{dy}}{\text{dx}}=\frac{\sin(\text{a}+\text{y})}{\Big(1-\frac{\text{y}}{\sin(\text{a}+\text{y})}\cos(\text{a}+\text{y})\Big)}\ \Big[\text{Since}\frac{\text{y}}
{\sin(\text{a}+\text{y})}=\text{x}\Big]
\frac{\text{dy}}{\text{dx}}=\frac{\sin^2(\text{a}+\text{y})}{\sin(\text{a}+\text{y})-\text{y}\cos(\text{a}+\text{y})}
Q565. \text{If y = x}^{x},\text{Prove that } \frac{\text{d}^{2}\text{y}}{\text{dx}^{2}} - \frac{\text{1}}{\text{y}}\bigg(\frac{dy}{dx}\bigg)^{2} - \frac{y}{x} = 0. 4 Marks

Ans: \text{y = x}^{\text{x}}\Rightarrow \log \text{y = x.}\log\text{x}


\Rightarrow \frac{1}{\text{y}}\frac{\text{dy}}{\text{dx}} = (1 + \log \text{x})
\Rightarrow\frac{1}{\text{y}}\frac{\text{d}^{2}{\text{y}}}{\text{ dx}^{2}} - \frac{1}{\text{y}^{2}} \bigg(\frac{\text{dy}}{\text{dx}}\bigg)^{2} = \frac{1}{\text{x}}
\Rightarrow \frac{\text{d}^{2}\text{y}}{\text{dx}^{2}} - \frac{1}{\text{y}} \bigg(\frac{\text{dy}}{\text{dx}}\bigg)^{2}-\frac{\text{y}}{\text{x}} = 0
Q566. If \text{y}=\sqrt{\text{x}^2+\text{a}^2}, prvoe that \text{y}\frac{\text{dy}}{\text{dx}}-\text{x}=0 4 Marks

Ans: Here, \text{y}=\sqrt{\text{x}^2+\text{a}^2}


Differentiating with respect to x,
\frac{\text{dy}}{\text{dx}}=\frac{\text{d}}{\text{dx}}\big(\sqrt{\text{x}^2+\text{a}^2}\big)
=\frac{1}{2\sqrt{\text{x}^2+\text{a}^2}}\frac{\text{d}}{\text{dx}}\big(\text{x}^2+\text{a}^2\big)
[Using chain rule]
=\frac{1}{2\sqrt{\text{x}^2+\text{a}^2}}\times(2\text{x})
=\frac{\text{x}}{\sqrt{\text{x}^2+\text{a}^2}}
\frac{\text{dy}}{\text{dx}}=\frac{\text{x}}{\text{y}}\ \Big[\text{Since},\sqrt{\text{x}^2+\text{a}^2}=\text{y}\Big]
\Rightarrow \text{y}\frac{\text{dy}}{\text{dx}}=\text{x}
\Rightarrow\text{y}\frac{\text{dy}}{\text{dx}}-\text{x}=0
Q567. Determine if f defined by: 4 Marks
\text{f(x)}=\begin{cases}\text{x}^{2} \sin\frac{1}{\text{x}}, \text{if} \ \text{x}\neq0\\0, \ \ \ \ \ \ \ \ \ \ \ \text{if}\ \text{x} = 0\end{cases}

Ans: It is given that \text{f(x)}=\begin{cases}\text{x}^{2} \sin\frac{1}{\text{x}}, \text{if} \ \text{x}\neq0\\0, \ \ \ \ \ \ \ \ \ \ \ \text{if}\ \text{x} = 0\end{cases}
We know that f is defined at all point of the real line.
Let k be a real number.
Case I: \text{k} \neq 0,
Then \text{f(k)} =\text{k}^{2} \sin\frac{1}{\text{k}}
^{\ \ \text{lim}}_{\text{x}\rightarrow\text{k}}\text{f(x)} = ^{\ \ \text{lim}}_{\text{x}\rightarrow\text{k}}\Big(\text{x}^{2}\sin\frac{1}{\text{x}}\Big) = \text{k}^{2}\sin \frac{\text{1}}{\text{k}}
\therefore\ ^{\ \ \text{lim}}_{\text{x}\rightarrow\text{k}}\text{f(x)} = \text{f(k)}
Thus, f is continuous at all points x that is \text{x}\neq0.
Case II: k = 0
Then f(k) = f(0) = 0
^{\ \ \text{lim}}_{\text{x}\rightarrow\text{0}^{-}}\text{f(x)} = ^{\ \ \text{lim}}_{\text{x}\rightarrow\text{0}^{-}}\Big(\text{x}^{2}\sin\frac{1}{\text{x}}\Big) =^{\ \
\text{lim}}_{\text{x}\rightarrow\text{0}^{-}}\Big( \text{x}^{2}\sin \frac{\text{1}}{\text{x}}\Big)
We know that -1 \leq\sin\frac{1}{\text {x}}\leq1, \text{x}\neq0
\rightarrow \text{x}^{2}\leq \text{x}^{2}\sin\frac{1}{\text {x}}\leq0
\Rightarrow^{\ \ \text{lim}}_{\text{x}\rightarrow\text{0}}\Big(\text{x}^{2}\sin\frac{1}{\text{x}}\Big) = 0
\Rightarrow^{\ \ \text{lim}}_{\text{x}\rightarrow\text{0}^{-}}\text{f(x)} = 0
similarly,
^{\ \ \text{lim}}_{\text{x}\rightarrow\text{0}^{+}}\text{f(x)} = ^{\ \ \text{lim}}_{\text{x}\rightarrow\text{0}^{+}}\Big(\text{x}^{2}\sin\frac{1}{\text{x}}\Big) =^{\ \
\text{lim}}_{\text{x}\rightarrow\text{0}}\Big( \text{x}^{2}\sin \frac{\text{1}}{\text{x}}\Big) = 0
^{\ \ \text{lim}}_{\text{x}\rightarrow\text{0}^{-}}\text{f(x)} = \text{f(0)}= ^{\ \ \text{lim}}_{\text{x}\rightarrow\text{0}^{+}}\text{f(x)}
Therefore , f is continuous at x = 0.
Therefore, f is has no point of discontinuity.
Q568. If \sec\Big(\frac{\text{x}+\text{y}}{\text{x}-\text{y}}\Big)=\text{a} prove that \frac{\text{dx}}{\text{dx}}=\frac{\text{y}}{\text{x}} 4 Marks

Ans: We have, \sec\Big(\frac{\text{x}+\text{y}}{\text{x}-\text{y}}\Big)=\text{a}


\Rightarrow\frac{\text{x}+\text{y}}{\text{x}-\text{y}}=\sec^{-1}({\text{a}})
Differentiate with respect to x, we get,
\Rightarrow\bigg[\frac{(\text{x}-\text{y})\frac{\text{d}}{\text{dx}}(\text{x}+\text{y})-(\text{x}+\text{y})\frac{\text{d}}{\text{dx}}(\text{x}-\text{y})}{(\text{x}-\text{y}}\bigg]=0
\Rightarrow(\text{x}-\text{y})\Big(1+\frac{\text{d}}{\text{dx}}\Big)-(\text{x}+\text{y})\Big(1-\frac{\text{d}}{\text{dx}}\Big)=0
\Rightarrow(\text{x}-\text{y})+(\text{x}-\text{y})\frac{\text{dy}}{\text{dx}}-(\text{x}+\text{y})+(\text{x}+\text{y})\frac{\text{dy}}{\text{dx}}=0
\Rightarrow\frac{\text{dy}}{\text{dx}}[\text{x}-\text{y}+\text{x}+\text{y}]=\text{x}+\text{y}-\text{x}+\text{y}
\Rightarrow\frac{\text{dy}}{\text{dx}}(2\text{x})=2\text{y}
\Rightarrow\frac{\text{dy}}{\text{dx}}=\frac{\text{y}}{\text{x}}
Q569. Find \frac{\text{dy}}{\text{dx}} of the functions given in Exercise: 4 Marks
\text{x}^\text{y}+\text{y}^\text{x}=1

Ans: Given: \text{x}^\text{y}+\text{y}^\text{x}=1\ \Rightarrow\ \text{u}+\text{v}=1,\text{where u}=\text{x}^\text{y}\text{ and v}=\text{y}^\text{x}


\therefore\ \frac{\text{d}}{\text{dx}}\text{u}+\frac{\text{d}}{\text{dx}}\text{v}=\frac{\text{d}}{\text{dx}}1\ \Rightarrow\ \frac{\text{du}}{\text{dx}}+\frac{\text{dv}}{\text{dx}}=0\
\dots\text{(i)}
Now \text{u}=\text{x}^\text{y}\ \Rightarrow\ \log\text{u}=\log\text{x}^\text{y}=\text{y}\log\text{x}
\Rightarrow\ \frac{\text{d}}{\text{dx}}\log\text{u}=\frac{\text{d}}{\text{dx}}(\text{y}\log\text{x})\ \Rightarrow\ \frac{1}{\text{u}}\frac{\text{du}}{\text{dx}}=\text{y}\frac{\text{d}}
{\text{dx}}\log\text{x}+\log\text{x}\frac{\text{dy}}{\text{dx}}
\Rightarrow\ \frac{1}{\text{u}}\frac{\text{du}}{\text{dx}}=\text{y}.\frac{1}{\text{x}}+\log\text{x}.\frac{\text{dy}}{\text{dx}}\ \Rightarrow\ \frac{\text{du}}
{\text{dx}}=\text{u}\Big(\frac{\text{y}}{\text{x}}+\log\text{x}.\frac{\text{dy}}{\text{dx}}\Big)
\Rightarrow\ \frac{\text{du}}{\text{dx}}=\text{x}^\text{y}\Big(\frac{\text{y}}{\text{x}}+\log\text{x}.\frac{\text{dy}}{\text{dx}}\Big)=\text{x}^\text{y}\frac{\text{y}}
{\text{x}}+\text{x}^\text{y}\log\text{x}.\frac{\text{dy}}{\text{dx}}
\Rightarrow\ \frac{\text{du}}{\text{dx}}=\text{x}^{\text{y}-1}\text{y}+\text{x}^\text{y}\log\text{x}.\frac{\text{dy}}{\text{dx}}\ \dots\text{(ii)}
Again \text{v}=\text{y}^\text{x}\ \Rightarrow\ \log\text{v}=\log\text{y}^\text{x}=\text{x}\log\text{y}
\Rightarrow\ \frac{\text{d}}{\text{dx}}\log\text{v}=\frac{\text{d}}{\text{dx}}(\text{x}\log\text{y})\ \Rightarrow\ \frac{1}{\text{v}}\frac{\text{dv}}{\text{dx}}=\text{x}\frac{\text{d}}
{\text{dx}}\log\text{y}+\log\text{y}\frac{\text{d}}{\text{dx}}\text{x}
\Rightarrow\ \frac{1}{\text{v}}\frac{\text{dv}}{\text{dx}}=\text{x}.\frac{1}{\text{y}}\frac{\text{dy}}{\text{dx}}+\log\text{y}.1\ \Rightarrow\ \frac{\text{dv}}
{\text{dx}}=\text{v}\Big(\frac{\text{x}}{\text{y}}\frac{\text{dy}}{\text{dx}}+\log\text{y}\Big)

https://bls.smartstudies.co.in/#/exam/pdf-preview/c59cb220-8e86-4716-9ff7-82aec16b1ade/1 119/158
5/26/24, 6:20 PM Exam Automation
\Rightarrow\ \frac{\text{dv}}{\text{dx}}=\text{y}^\text{x}\Big(\frac{\text{x}}{\text{y}}\frac{\text{dy}}{\text{dx}}+\log\text{y}\Big)=\text{y}^\text{x} \frac{\text{x}}{\text{y}}\frac{\text{dy}}
{\text{dx}}+\text{y}^\text{x}\log\text{y}
\Rightarrow\ \frac{\text{dy}}{\text{dx}}=\text{y}^{\text{x}-1}\text{x}\frac{\text{dy}}{\text{dx}}+\text{y}^\text{x}\log\text{y}\ \dots\text{(iii)}
Putting values from eq. (ii) and (iii) in eq. (i),
\text{x}^{\text{x}-1}\text{y}+\text{x}^\text{y}\log\text{x}.\frac{\text{dy}}{\text{dx}}+\text{y}^{\text{x}-1}\text{x}\frac{\text{dy}}{\text{dx}}+\text{y}^\text{x}\log\text{y}=0
\Rightarrow\ \frac{\text{dy}}{\text{dx}}(\text{x}^\text{y}\log\text{x}+\text{y}^{\text{x}-1}\text{x})=-\text{x}^{\text{y}-1}\text{y}-\text{y}^\text{x}\log\text{y}
\Rightarrow\ \frac{\text{dy}}{\text{dx}}=\frac{-(\text{x}^{\text{y}-1}\text{y}-\text{y}^\text{x}\log\text{y})}{\text{x}^\text{y}\log\text{x}+\text{y}^{\text{x}-1}\text{x}}
Q570. If \text{y}\log(1+\cos\text{x}), prove that \frac{\text{d}^3\text{y}}{\text{dx}^3}+\frac{\text{d}^\text{y}}{\text{dx}^2}.\frac{\text{d}\text{y}}{\text{dx}}=0 4 Marks

Ans: \text{y}\log(1+\cos\text{x}),
Differentiating w.r.t.x, we get
\Rightarrow\frac{\text{dy}}{\text{dx}}=\frac{1}{1+\cos\text{x}}\times-\sin\text{x}=\frac{-\sin\text{x}}{1+\cos\text{x}}
Differentiating w.r.t.x, we get
\Rightarrow\frac{\text{d}^2\text{y}}{\text{dx}^2}=-\Big[\frac{(1+\cos\text{x})\cos\text{x}-\sin(-\sin\text{x})}{(1+\cos\text{x})^2}\Big]
\Rightarrow\frac{\text{d}^2\text{y}}{\text{dx}^2}=-\Big[\frac{\cos\text{x}+\cos^2\text{x}+\sin^2\text{x}}{(1+\cos\text{x})^2}\Big]=-\Big[\frac{1+\cos\text{x}}
{(1+\cos\text{x})^2}\Big]=\frac{-1}{1+\cos\text{x}}
Differentiating w.r.t.x
\Rightarrow\frac{\text{d}^2\text{y}}{\text{dx}^3}=-\Big(\frac{+1}{(1+\cos\text{x})^2}\times+\sin\text{x}\Big)\\=-\Big(\frac{-\sin\text{x}}{1+\cos\text{x}}\Big)\times\Big(\frac{-1}
{1+\cos\text{x}}\Big)=-\frac{\text{dy}}{\text{dx}}.\frac{\text{d}^2\text{y}}{\text{dx}^2}
\Rightarrow\frac{\text{d}^3\text{y}}{\text{dx}^3}+\frac{\text{d}^2\text{y}}{\text{dx}^2}.\frac{\text{dy}}{\text{dx}}=0
Q571. Differentiate the functions given in Exercise: 4 Marks
\text{x}^\text{x}-2^{\sin\text{x}}

Ans: Let \text{y}=\text{x}^\text{x}-2^{\sin\text{x}}


Putting \text{u}=\text{x}^\text{x}\text{ and v }=2^{\sin\text{x}}
\Rightarrow\ \text{y}=\text{u}-\text{v}\ \Rightarrow\ \frac{\text{dy}}{\text{dx}}=\frac{\text{du}}{\text{dx}}-\frac{\text{dv}}{\text{dx}}\ \dots\text{(i)}
Now, u = xx \ \Rightarrow\ \log\text{u}=\log\text{x}^\text{x}=\text{x}\log\text{x}
\therefore\ \frac{\text{d}}{\text{dx}}\log\text{u}=\frac{\text{d}}{\text{dx}}(\text{x}\log\text{x}) \Rightarrow\ \frac{1}{\text{u}}\frac{\text{du}}{\text{dx}}=\text{x}\frac{\text{d}}
{\text{dx}}\log\text{x}+\log\text{x}\frac{\text{d}}{\text{dx}}\text{x}
\Rightarrow\ \frac{1}{\text{u}}\frac{\text{du}}{\text{dx}}=\text{x}\frac{1}{\text{x}}+\log\text{x}.1 \Rightarrow\ \frac{1}{\text{u}}\frac{\text{du}}{\text{dx}}=1+\log\text{x}
\Rightarrow\ \frac{\text{du}}{\text{dx}}=\text{u}(1+\log\text{x}) =\text{x}^\text{x}(1+\log\text{x})\ \dots\text{(ii)}
Again, \text{v}=2^{\sin\text{x}}\ \Rightarrow\ \frac{\text{dv}}{\text{dx}}=\frac{\text{d}}{\text{dx}}2^{\sin\text{x}}
\Rightarrow\ \frac{\text{dv}}{\text{dx}}=2^{\sin \text{x}}\log2\frac{\text{d}}{\text{dx}}\sin\text{x}=\ \Big[\because\frac{\text{d}}
{\text{dx}}\text{a}^{\text{f(x)}}=\text{a}^{\text{f(x)}}\log\text{a}\frac{\text{d}}{\text{dx}}\text{f(x)}\Big]
\frac{\text{dv}}{\text{dx}}=2^{\sin\text{x}}(\log2).\cos\text{x}=\cos\text{x}.2^{\sin\text{x}}\log2\ \dots\text{(iii)}
Putting the values from eq. (ii) and (iii) in eq. (i),
\frac{\text{dy}}{\text{dx}}=\text{x}^\text{x}(1+\log\text{x})-\cos\text{x}.2^{\sin\text{x}}\log2
Q572. Find the points of discontinuity, if any of the following function: 4 Marks
\text{f(x)}=\begin{cases}\frac{\text{x}^4-16}{\text{x}-2},&\text{if }\text{ x}\neq2\\16,&\text{if }\text{ x}=2\end{cases}

Ans: Given, \text{f(x)}=\begin{cases}\frac{\text{x}^4-16}{\text{x}-2},&\text{if }\text{ x}\neq2\\16,&\text{if }\text{ x}=2\end{cases}


When \text{x}\neq2, then
\text{f(x)}=\frac{\text{x}^4-16}{\text{x}-2}=\frac{\text{x}^4-2^4}{\text{x}-2}=\frac{(\text{x}^2-4)(\text{x}-2)(\text{x}+2)}{\text{x}-2}=(\text{x}^2+4)(\text{x}+2)
We know that a polynomial function is everywhere continuous.
Therefore, the functions (x2 + 4) and (x + 2) are everywhere continuous.
So, the product function x2 + 4x + 2 is everywhere continuous.
Thus, f(x) is continuous at every \text{x}\neq2
At x = 2, we have
(\text{LHL at x}=2)=\lim_\limits{\text{x}\rightarrow2^-}\text{f(x)}=\lim_\limits{\text{h}\rightarrow0}\text{f}(2-\text{h})
=\lim_\limits{\text{h}\rightarrow0}\big[(2-\text{h})^2+4\big](2-\text{h}+2)=8(4)=32
(\text{RHL at x}= 2)=\lim_\limits{\text{x}\rightarrow2^+}\text{f(x)}=\lim_\limits{\text{h}\rightarrow0}\text{f}(2+\text{h})
=\lim_\limits{\text{h}\rightarrow0}\big[(2+\text{h})^2+4\big](2+\text{h}+2)=8(4)=32
Also, f(2) = 16
\therefore\ \lim_\limits{\text{x}\rightarrow2^-}\text{f(x)}=\lim_\limits{\text{x}\rightarrow2^+}\text{f(x)}\neq\text{f}(2)
Thus, f(x) is discontinuous at x = 2
Hence, the only point of discontinuity for f(x) is x = 2
Q573. If u, v and w are functions of x, then show that 4 Marks
\frac{\text{d}}{\text{dx}}(\text{u. v. w)}=\frac{\text{du}}{\text{dx}}\text{ v. w}+\text{u }\frac{\text{dv}}{\text{dx}}\text{ w}+\text{u.v}\frac{\text{dw}}{\text{dx}}
in two ways - first by repeated application of product rule, second by logarithmic
differentiation.

Ans: Given: u, v and w are functions of x.


To prove: \frac{\text{d}}{\text{dx}}(\text{u. v. w)}=\frac{\text{du}}{\text{dx}}\text{ y.w}+\text{u }\frac{\text{dv}}{\text{dx}}\text{ w}+\text{u.v}\frac{\text{dw}}{\text{dx}}
1. By repeated application of product rule:
\text{L.H.S } \frac{\text{d}}{\text{dx}}(\text{u.v.w)}=\frac{\text{d}}{\text{dx}}[(\text{uv).w}]=\text{uv}\frac{\text{d}}{\text{dx}}\text{w}+\text{w}\frac{\text{d}}{\text{dx}}\text{(uv)}
=\text{uv}\frac{\text{dw}}{\text{dx}}+\text{w}\Big[\text{u}\frac{\text{d}}{\text{dx}}\text{v}+\text{v}\frac{\text{d}}{\text{dx}}\text{u}\Big]=\text{uv}\frac{\text{dw}}
{\text{dx}}+\text{uw}\frac{\text{dv}}{\text{dx}}+\text{vw}\frac{\text{du}}{\text{dx}}
=\frac{\text{du}}{\text{dx}}\text{.v.w}+\text{u}\frac{\text{dv}}{\text{dx}}\text{.w}+\text{u.v.}\frac{\text{dw}}{\text{dx}}=\text{R.H.S}\ \ \ \text{Hence proved}.
2. By Logarithmic differentiation:
Let y = uvw \ \Rightarrow\ \log\text{y}=\log(\text{u.v.w)}
\Rightarrow\ \log\text{y}=\log\text{u}+\log\text{v}+\log\text{w}\ \Rightarrow\ \frac{\text{d}}{\text{dx}}\log\text{y}=\frac{\text{d}}{\text{dx}}\log\text{u}+\frac{\text{d}}
{\text{dx}}\log\text{v}+\frac{\text{d}}{\text{dx}}\log\text{w}
\Rightarrow\ \frac{1}{\text{y}}\frac{\text{dy}}{\text{dx}}=\frac{1}{\text{u}}\frac{\text{du}}{\text{dx}}+\frac{1}{\text{v}}\frac{\text{dv}}{\text{dx}}+\frac{1}{\text{w}}\frac{\text{dw}}{\text{dx}}\
\Rightarrow\ \frac{\text{dy}}{\text{dx}}=\text{y}\Big[\frac{1}{\text{u}}\frac{\text{du}}{\text{dx}}+\frac{1}{\text{v}}\frac{\text{dv}}{\text{dx}}+\frac{1}{\text{w}}\frac{\text{dw}}{\text{dx}}\Big]
Putting y = uvw, we get
\frac{\text{d}}{\text{dx}}(\text{u.v.w)}=\text{uvw}\Big[\frac{1}{\text{u}}\frac{\text{du}}{\text{dx}}+\frac{1}{\text{v}}\frac{\text{dv}}{\text{dx}}+\frac{1}{\text{w}}\frac{\text{dw}}
{\text{dx}}\Big]
\Rightarrow\ \frac{\text{d}}{\text{dx}}\text{(u.v.w)}=\frac{\text{du}}{\text{dx}}\text{.v.w}+\text{u.}\frac{\text{dv}}{\text{dx}}\text{.w}+\text{u.v.}\frac{\text{dw}}{\text{dx}}\ \ \ \text{Hence
proved}.
Q574. Differentiate the following functions with respect to x: 4 Marks
\tan^{-1}\Big(\frac{\text{x}}{1+6\text{x}^3}\Big)

Ans: Let \text{y}=\tan^{-1}\Big(\frac{\text{x}}{1+6\text{x}^3}\Big)


\Rightarrow\ \text{y}=\tan^{-1}\Big(\frac{3\text{x}-2\text{x}}{1+(3\text{x})(2\text{x})}\Big)
\Rightarrow\text{y}=\tan^{-1}3\text{x}-\tan^{-1}2\text{x}
\Big[\text{Since}, \tan^{-1}\text{x}-\tan^{-1}\text{y}=\tan^{-1}\Big(\frac{\text{x}-\text{y}}{1+\text{xy}}\Big)\Big]
Differentiate it with respect to x using chain rule,
\frac{\text{dy}}{\text{dx}}=\frac{1}{1+(3\text{x})^2}\frac{\text{d}}{\text{dx}}(3\text{x})-\frac{1}{1+(2\text{x})^3}\frac{\text{d}}{\text{dx}}(2\text{x})

https://bls.smartstudies.co.in/#/exam/pdf-preview/c59cb220-8e86-4716-9ff7-82aec16b1ade/1 120/158
5/26/24, 6:20 PM Exam Automation
\Rightarrow\frac{\text{dy}}{\text{dx}}=\frac{1}{1+9\text{x}^2}(3)-\frac{1}{1+4\text{x}^2}(2)
\therefore\frac{\text{dy}}{\text{dx}}=\frac{3}{1+9\text{x}^2}-\frac{2}{1+4\text{x}^2}
Q575. If \text{x}=\cot\text{t and y}=\sin\text{t}, prove that \frac{\text{dy}}{\text{dx}}=\frac{1}{\sqrt{3}}\text{ at t}=\frac{2\pi}{3} 4 Marks

Ans: We have, \text{x}=\cos\text{t} and \text{y}=\sin\text{t}


\Rightarrow\frac{\text{dx}}{\text{dt}}=\frac{\text{d}}{\text{dt}}(\cos\text{t}) and \frac{\text{dy}}{\text{dt}}=\frac{\text{d}}{\text{dt}}(\sin\text{t})
\Rightarrow\frac{\text{dx}}{\text{dt}}=-\sin\text{t} and \frac{\text{dy}}{\text{dt}}=\cos\text{t}
\therefore\frac{\frac{\text{dy}}{\text{dt}}}{\frac{\text{dx}}{\text{dt}}}=\frac{\cos\text{t}}{-\sin\text{t}}=-\cot{\text{t}}
Now, \big(\frac{\text{dy}}{\text{dx}}\big)_{\text{t}=\frac{2\pi}{3}}=-\cot\big(\frac{2\pi}{3}\big)=\frac{1}{\sqrt{3}}
Q576. At what points on the following curves, is the tangent parallel to x-axis? 4 Marks
y = 12(x + 1)(x - 2) on [-1, 2]

Ans: Let f(x) = 12(x + 1)(x - 2) ...(1)


⇒ f(x) = 12(x2 - x - 2)
⇒ f(x) = 12x2 - 12x - 24
Since f(x) is a polynomial function, f(x) is continuous on [-1, 2] and differentiable on (1, 2).
Also,
f(2) = f(-1) = 0
Thus, all the conditions of Rolle's theorem are satisfied.
Consequently, there exists at least one point \text{c}\in(-1,2) for which f'(c) = 0.
But \text{f}'(\text{c})=0
\Rightarrow24\text{c}-12=0
\Rightarrow\text{c}=\frac{1}{2}
\therefore\ \text{f}(\text{c})=\text{f}\Big(\frac{1}{2}\Big)=-12\Big(\frac{3}{2}\Big)\Big(\frac{3}{2}\Big)=-27
By the geometrical interepretetion of Rolle's theorem, \Big(\frac{1}{2}\Big),-27 is the point on y = 12(x + 1)(x - 2) where the tangent is parallel to the x-axis.
Q577. Differentiate \sin^{-1}\Big(4\text{x}\sqrt{1-4\text{x}^2}\Big) with respect to \sqrt{1-4\text{x}^2}, if: 4 Marks
\text{x}\in\Big(-\frac{1}{2},-\frac{1}{2\sqrt{2}}\Big)

Ans: Let \text{u}=\sin^{-1}\Big(4\text{x}\sqrt{1-4\text{x}^2}\Big)


Put 2\text{x}=\cos\theta \text{ So},
\Rightarrow\text{u}=\sin^{-1}\Big(2\times\cos\theta\sqrt{1-\cos^2\theta}\Big)
\Rightarrow\text{u}=\sin^{-1}(2\cos\theta\sin\theta)
\Rightarrow\text{u}=\sin^{-1}(\sin2\theta)\ .....(\text{i})
Let, \text{v}=\sqrt{1-4\text{x}^2}\ .....(\text{ii})
Here,
\text{x}\in\Big(\frac{1}{2},-\frac{1}{2\sqrt{2}}\Big)
\Rightarrow2\text{x}\in\Big(-1,-\frac{1}{\sqrt{2}},\Big)
\Rightarrow\theta\in\Big(\frac{3\pi}{4},\pi\Big)
So, from equation (i),
\text{u}=\pi-2\theta
\Big[\text{Since},\sin^{-1}(\sin\theta)=\pi-\theta,\text{ if }\theta\in\Big[-\frac{\pi}{2},\frac{3\pi}{2}\Big]\Big]
\Rightarrow\text{u}=\pi-2\cos^{-1}(2\text{x})\big[\text{Since},2\text{x}=\cos\theta\big]
Differentiating it with respect to x using chain rule,
\frac{\text{du}}{\text{dx}}=0-2\bigg(\frac{-1}{\sqrt{1-(2\text{x})^2}}\bigg)\frac{\text{d}}{\text{dx}}(2\text{x})
\Rightarrow\frac{\text{du}}{\text{dx}}=\frac{-2}{\sqrt{1-4\text{x}^2}}(2)
\Rightarrow\frac{\text{du}}{\text{dx}}=\frac{-4}{\sqrt{1-4\text{x}^2}}\ .....(\text{vi})
From equation (iv)
\frac{\text{dv}}{\text{dx}}=\frac{4}{\sqrt{1-4\text{x}^2}}
But, \text{x}\in\Big(-\frac{1}{2},-\frac{1}{2\sqrt{2}}\Big)
\frac{\text{dv}}{\text{dx}}=\frac{-4(-\text{x})}{\sqrt{1-4(-\text{x})^2}}
\frac{\text{dv}}{\text{dx}}=\frac{4\text{x}}{\sqrt{1-4\text{x}^2}}\ .....(\text{vii})
Dividing equation (vi) by (vii),
\frac{\frac{\text{du}}{\text{dx}}}{\frac{\text{dv}}{\text{dx}}}=\frac{-4}{\sqrt{1-4\text{x}^2}}\times\frac{\sqrt{1-4\text{x}^2}}{4\text{x}}
\frac{\text{du}}{\text{dv}}=-\frac{1}{\text{x}}
Q578. Differentiate the following functions with respect to x: 4 Marks
\text{e}^{\sqrt{\cot\text{x}}}

Ans: Let, \text{y}=\text{e}^\sqrt{{\cot\text{x}}}


\Rightarrow\ \text{y}=\text{e}^{(\cot\text{x})^\frac{1}{2}}
Differentiate with respect to x we get,
\frac{\text{dy}}{\text{dx}}=\frac{\text{d}}{\text{dx}}\Big(\text{e}^{(\cot\text{x})^\frac{1}{2}}\Big)
=\text{e}^{(\cot\text{x})^\frac{1}{2}}\frac{\text{d}}{\text{dx}}(\cot\text{x})^\frac{1}{2}
[Using chain rule]
=\text{e}^\sqrt{\cot\text{x}}\times\frac{1}{2}(\cot\text{x})^{\frac{1}{2}-1}\frac{\text{d}}{\text{dx}}(\cot\text{x})
=-\frac{\text{e}^\sqrt{\cot\text{x}}\times\text{cosec}^2\text{x}}{2\sqrt{\cot\text{x}}}
So,
\frac{\text{d}}{\text{dx}}\Big(\text{e}^\sqrt{\cot\text{x}}\Big)=-\frac{\text{e}^\sqrt{\cot\text{x}}\times\text{cosec}^2\text{x}}{2\sqrt{\cot\text{x}}}
Q579. Find the value of k in this question, so that the function f is continuous at the indicated point: 4 Marks
\text{f(x)}=\begin{cases}\frac{1-\cos\text{kx}}{\text{x}\sin\text{x}},&\text{if x}\neq0\\\frac{1}{2},&\text{if x}=0\end{cases} at x = 0.

Ans: We have, \text{f(x)}=\begin{cases}\frac{1-\cos\text{kx}}{\text{x}\sin\text{x}},&\text{if x}\neq0\\\frac{1}{2},&\text{if x}=0\end{cases} at x = 0


At x = 0, \text{ LHL }=\lim\limits_{\text{x}\rightarrow0}\frac{1-\cos\text{kx}}{\text{x}\sin\text{x}}=\lim\limits_{\text{x}\rightarrow0}\frac{1-\cos\text{k}(0-\text{h})}{(0-\text{h})\sin(0-
\text{h})}
=\lim\limits_{\text{x}\rightarrow0}\frac{1-\cos(-\text{kh})}{-\text{h}\sin(-\text{h})}
=\lim\limits_{\text{x}\rightarrow0}\frac{1-\cos\text{kh}}{-\text{h}\sin\text{h}} [\because \cos(-\theta)-\cos\theta,\sin(-\theta)=-\sin\theta]
=\lim\limits_{\text{x}\rightarrow0}\frac{1-1+2\sin^2\frac{\text{kh}}{2}}{\text{h}\sin\text{h}} \bigg[\because\cos\theta=1-2\sin^2\frac{\theta}{2}\bigg]
=\lim\limits_{\text{x}\rightarrow0}\frac{2\sin^2\frac{\text{kh}}{2}}{\text{h}\sin\text{h}}
=\lim\limits_{\text{x}\rightarrow0}\frac{2\sin\frac{\text{kh}}{2}}{\frac{\text{kh}}{2}}\cdot\frac{\sin\frac{\text{kh}}{2}}{\frac{\text{kh}}{2}}\cdot\frac{1}{\frac{\sin\text{h}}
{\text{h}}}\cdot\frac{\text{k}^2\frac{\text{h}}{4}}{\text{h}}
=\frac{2\text{k}^2}{4}=\frac{\text{k}^2}{2}\Big[\because\lim\limits_{\text{x}\rightarrow0}\frac{\sin\text{h}}{\text{h}}=1\Big]
Also, f(0)=\frac{1}{2}\Rightarrow\frac{\text{k}^2}{2}=\frac{1}{2}
\Rightarrow\text{k}=\pm1
Q580. Find the points of discontinuity, if any of the following function: 4 Marks
\text{f(x)}=\begin{cases}|\text{x}-3|,&\text{if }\text{ x}\geq1\\\frac{\text{x}^2}{4}-\frac{3\text{x}}{2}+\frac{13}{4},&\text{if }\text{ x}<1\end{cases}

Ans: When x >, then


f(x) = |x - 3|
Since modulus function is a continuous function, f(x) is continuous for each x > 1
When x < 1, then

https://bls.smartstudies.co.in/#/exam/pdf-preview/c59cb220-8e86-4716-9ff7-82aec16b1ade/1 121/158
5/26/24, 6:20 PM Exam Automation
\text{f(x)}=\frac{\text{x}^2}{4}-\frac{3\text{x}}{2}+\frac{13}{4}
Since, x2 & 3x are continuous being polynomial functions, x2 & 3x will also be continuous.
Also, \frac{13}{4} is continuous being a polymomial function.
\Rightarrow\frac{\text{x}^2}{4}-\frac{3\text{x}}{2}+\frac{13}{4} is continuous for eqch x < 1
⇒ f(x) is continuous for each x < 1
At x = 1, we have
(\text{LHL at x}=1)=\lim_\limits{\text{x}\rightarrow1^-}\text{f(x)}=\lim_\limits{\text{h}\rightarrow0}\text{f}(1-\text{h})
=\lim_\limits{\text{h}\rightarrow0}\Big[\frac{(1-\text{h}^2)}{4}-\frac{3(1-\text{h})}{2}+\frac{13}{4}\Big]=\frac{1}{4}-\frac{3}{2}+\frac{13}{4}=2
(\text{RHL at x}= 1)=\lim_\limits{\text{x}\rightarrow1^+}\text{f(x)}=\lim_\limits{\text{h}\rightarrow0}\text{f}(1+\text{h})
=\lim_\limits{\text{h}\rightarrow0}\big[|1+\text{h}-3|\big]=|-2|=2
Also, \text{f}(1)=|1-3|=|-2|=2
Thus, \lim_\limits{\text{x}\rightarrow1^-}\text{f(x)}=\lim_\limits{\text{x}\rightarrow1^+}\text{f(x)}=\text{f}=(1)
Hence, f(x) is continuous at x = 1
Thus, the given function is now where discontinuous.
Q581. Differentiate \tan^{-1}\Big(\frac{1-\text{x}}{1+\text{x}}\Big) with respect to \sqrt{1-\text{x}^2}, if -1 < x < 1. 4 Marks

Ans: Let, \text{u}=\tan^{-1}\Big(\frac{1-\text{x}}{1+\text{x}}\Big)


Put \text{x}=\tan\theta
\Rightarrow\theta=\tan^{-1}\text{x}
\Rightarrow\text{u}=\tan^{-1}\Big(\frac{1-\tan\theta}{1+\tan\theta}\Big)
\Rightarrow\text{u}=\tan^{-1}\Big[\tan\Big(\frac{\pi}{4}-\theta\Big)\Big]\ .....(\text{i})
Here,
-1<\text{x}<1
\Rightarrow-1<\tan\theta<1
\Rightarrow-\frac{\pi}{4}<\theta<\frac{\pi}{4}
\Rightarrow\frac{\pi}{4}>-\theta>\frac{\pi}{4}
\Rightarrow-\frac{\pi}{4}<-\theta<\frac{\pi}{4}
\Rightarrow0<\frac{\pi}{4}-\theta<\frac{\pi}{2}
So, from equation (i)
\text{u}=\frac{\pi}{4}-\theta
\Big[\text{Since,}\tan^{-1}(\tan\theta)=\theta,\text{if }\theta\in\Big(-\frac{\pi}{2},\frac{\pi}{2}\Big)\Big]
\Rightarrow\text{u}=\frac{\pi}{4}-\tan^{-1}\text{x}
\frac{\text{du}}{\text{dx}}=0-\Big(\frac{1}{1+\text{x}}\Big)
\Rightarrow\frac{\text{du}}{\text{dx}}=-\frac{1}{1+\text{x}^2}\ .....(\text{ii})
And
Let, \text{v}=\sqrt{1-\text{x}^2}
\frac{\text{dv}}{\text{dx}}=\frac{1}{2\sqrt{1-\text{x}^2}}\times\frac{\text{d}}{\text{dx}}(1-\text{x}^2)
\Rightarrow\frac{\text{dv}}{\text{dx}}=\frac{1}{2\sqrt{1-\text{x}^2}}(-2\text{x})
\Rightarrow\frac{\text{dv}}{\text{dx}}=\frac{-\text{x}}{\sqrt{1-\text{x}^2}}\ .....(\text{iii})
Dividing equation (ii) by (iii),
\frac{\frac{\text{du}}{\text{dx}}}{\frac{\text{dv}}{\text{dx}}}=-\frac{1}{1+\text{x}^2}\times\frac{\sqrt{1-\text{x} ^2}}{-\text{x}}
\therefore\frac{\text{du}}{\text{dv}}=\frac{\sqrt{1-\text{x}^2}}{\text{x}(1+\text{x}^2)}
Q582. Differentiate \tan^{-1}\Big(\frac{\text{x}}{\sqrt{1-\text{x}^2}}\Big) with respect to \sin^{-1}\Big(2\text{x}\sqrt{1-\text{x}^2}\Big), if -\frac{1}{\sqrt{2}}<\text{x}<\frac{1}{\sqrt{2}} 4 Marks

Ans: Let, \text{u}=\tan^{-1}\Big(\frac{\text{x}}{\sqrt{1-\text{x}^2}}\Big)


Put \text{x}=\sin\theta
\Rightarrow\theta=\sin^{-1}\text{x}
\Rightarrow\text{u}=\tan^{-1}\Big(\frac{\sin\theta}{\sqrt{1-\sin^{2}\theta}}\Big)
\Rightarrow\text{u}=\tan^{-1}\Big(\frac{\sin\theta}{\cos\theta}\Big)
\Rightarrow\text{u}=\tan^{-1}(\tan\theta)\ .....(\text{i})
And
Let, \text{v}=\sin^{-1}(2\text{x}\sqrt{1-\text{x}^2})
\text{v}=\sin^{-1}(2\sin\theta\sqrt{1-\sin^{2}\theta})
\text{v}-\sin^{-1}(2\sin\theta\cos\theta)
\text{v}=\sin^{-1}(\sin2\theta)\ .....(\text{ii})
Here,
-\frac{1}{\sqrt{2}}<\text{x}<\frac{1}{\sqrt{2}}
\Rightarrow-\frac{1}{\sqrt{2}}<\sin\theta<\frac{1}{\sqrt{2}}
\Rightarrow-\frac{\pi}{4}<\theta<\frac{\pi}{4}
So, from equation (i)
\text{u}=\theta\Big[\text{since,}\tan^{-1}(\tan\theta)=\theta,\text{if }\theta\in\Big(-\frac{\pi}{2},\frac{\pi}{2}\Big)\Big]
\Rightarrow\text{u}=\sin^{-1}\text{x}
Differentiatiating it with respect to x,
\frac{\text{du}}{\text{dx}}=\frac{1}{\sqrt{1-\text{x}^2}}\ .....(\text{iii})
From equation (ii),
\text{v}=2\theta\Big[\text{Since,}\sin^{-1}(\sin\theta)=\theta,\text{if }\theta\in\Big[-\frac{\pi}{2},\frac{\pi}{2}\Big]\Big]
\Rightarrow\text{v}=2\sin^{-1}\text{x}
Differentiating it with respect to x,
\frac{\text{dv}}{\text{dx}}=\frac{2}{\sqrt{1-\text{x}^2}}\ .....(\text{iv})
Dividing equation (iii) by (iv),
\frac{\frac{\text{du}}{\text{dx}}}{\frac{\text{dv}}{\text{dx}}}=\Big(\frac{1}{\sqrt{1-\text{x}^2}}\Big)\Big(\frac{\sqrt{1-\text{x}^2}}{2}\Big)
\therefore\frac{\text{du}}{\text{dv}}=\frac{1}{2}
Q583. show that \text{f}\text{(x)}=\begin{cases}\frac{\text{x}-|\text{x}|}{2}, & \text{when} \text{ x}\neq 0\\2, & \text{when}\text{ x} = 0\end{cases} is discontinuous at x = 0. 3 Marks

Ans: We want, to check the continuty of the function at x = 0.


\text{LHL}=\lim\limits_{\text{x} \rightarrow 0^-}\text{f}\text{ (x)}=\lim\limits_{\text{h} \rightarrow 0}\text{f}(0-\text{x)}
=\lim\limits_{\text{h} \rightarrow 0}\frac{-\text{h}-|-\text{h|}}{2}=\lim\limits_{\text{h} \rightarrow 0}\frac{-\text{h}-\text{h}}{2}=0
\text{RHL}=\lim\limits_{\text{x} \rightarrow 0^+}\text{f}\text{ (x)}=\lim\limits_{\text{h} \rightarrow 0}\text{f}(0+\text{h)}
=\lim\limits_{\text{h} \rightarrow 0}\frac{\text{h}-\text{(|h|)}}{2}=0
\text{f}(0)=2
Thus, \text{LHL}=\text{RHL}\neq\text{f}(0)
Hence,The function is discotinuous at x = 0
This is rem ovable discontinuty.
Q584. Differentiate the functions with respect to x. 3 Marks
\frac{\sin(\text{ax + b)}}{\cos(\text{cx + d})}

Ans: \text{Let y} = \frac{\sin(\text{ax + b)}}{\cos(\text{cx + d})}


Using quotient rule,

https://bls.smartstudies.co.in/#/exam/pdf-preview/c59cb220-8e86-4716-9ff7-82aec16b1ade/1 122/158
5/26/24, 6:20 PM Exam Automation
\therefore \frac{\text{dy}}{\text{dx}} = \frac{\cos(\text{cx + d})\frac{\text{d}}{\text{dx}}\sin\text(\text{ax + b})-\sin(\text{ax + b)}\frac{\text{d}}{\text{dx}}\cos(\text{cx} + \text{d})}
{\cos^2(\text{cx} + \text{d})}
= \frac{\cos(\text{cx + d})\cos\text(\text{ax + b})\frac{\text{d}}{\text{dx}}(\text{ax + b)}-\sin(\text{ax + b)}\left\{-\sin(\text{cx + d)}\right\}\frac{\text{d}}{\text{dx}}(\text{cx} + \text{d})}
{\cos^2(\text{cx} + \text{d})}
= \frac{\cos(\text{cx + d})\cos\text(\text{ax + b})(\text{a)}+\sin(\text{ax + b})\sin(\text{cx + d)}(\text{c})}{\cos^2(\text{cx} + \text{d})}
Q585. If f(x) is defined by f(x) x2. find f(2). 3 Marks

Ans: Given: f(x) = x2.


We know a polynomial function is everywhere differentiable. Therefore f(x) is differentiable at x = 2.
\text{f}'(2)=\lim_\limits{\text{k}\rightarrow0}\text{f}\frac{(2+\text{h})-\text{f}(2)}{\text{h}}
\Rightarrow\text{f}'(2)=\lim_\limits{\text{k}\rightarrow0}\text{f}\frac{(2+\text{h})2-22}{\text{h}}
\Rightarrow\text{f}'(2)=\lim_\limits{\text{k}\rightarrow0}\text{f}\frac{(4+\text{h}2-4\text{h})-4}{\text{h}}
\Rightarrow\text{f}'(2)=\lim_\limits{\text{k}\rightarrow0}\text{f}\frac{\text{h}(\text{h}+4)}{\text{h}}
\Rightarrow\text{f}'(2)=4
Q586. Find the derivative of the function f defined by f(x) = mx + c at x = 0. 3 Marks

Ans: Given: f(x) = mx + c


Clearly, being a polynomial function, is differentiable everywhere. Therefore the derivative of f at x is given by:
\text{f}'(\text{x})=\lim_\limits{\text{h}\rightarrow0}\frac{\text{f}(\text{x}+\text{h}-\text{f(x)})}{\text{h}}
\Rightarrow\text{f}'(\text{x})=\lim_\limits{\text{h}\rightarrow0}\frac{\text{m}(\text{x}+\text{h})+\text{c}-\text{mx}-\text{c}}{\text{h}}
\Rightarrow\text{f}'(\text{x})=\lim_\limits{\text{h}\rightarrow0}\frac{\text{mx}+\text{mh}+\text{c}-\text{mx}-\text{c}}{\text{h}}
\Rightarrow\text{f}'(\text{x})=\lim_\limits{\text{h}\rightarrow0}\frac{\text{mh}}{\text{h}}
\Rightarrow\text{f}'(\text{x})=\text{m}
Thus, \text{f}'(0)=\text{m}
Q587. Differentiate the following functions with respect to x: 3 Marks
\text{e}^{\sin\sqrt{\text{x}}}

Ans: Let, \text{y}=\text{e}^{\sin\sqrt{\text{x}}}


Differentiate it with respect to x,
\frac{\text{dy}}{\text{dx}}=\frac{\text{d}}{\text{dx}}\big(\text{e}^{\sin\sqrt{\text{x}}}\big)
=\text{e}^{\sin\sqrt{\text{x}}}\frac{\text{d}}{\text{dx}}\big(\sin\sqrt{\text{x}}\big)
[Using chain rule]
=\text{e}^{\sin\sqrt{\text{x}}}\times\cos\sqrt{\text{x}}\frac{\text{d}}{\text{dx}}\sqrt{\text{x}}
[Using chain rule]
=\text{e}^{\sin\sqrt{\text{x}}}\times\cos\sqrt{\text{x}}\times\frac{1}{2\sqrt{\text{x}}}
=\frac{1}{2\sqrt{\text{x}}}\times\cos\sqrt{\text{x}}\times\text{e}^{\sin\sqrt{\text{x}}}
So,
\frac{\text{d}}{\text{dx}}=\big(\text{e}^{\sin^\sqrt{\text{x}}}\big)=\frac{1}{2\sqrt{\text{x}}}\cos\sqrt{\text{x}}\times\text{e}^{\sin\sqrt{\text{x}}}
Q588. Differentiate the following functions with respect to x: 3 Marks
\tan^{-1}\Big(\frac{\text{a}+\text{x}}{1-\text{ax}}\Big)

Ans: Let \text{y}=\tan^{-1}\Big(\frac{\text{a}+\text{x}}{1-\text{ax}}\Big)


\text{y}=\tan^{-1}\text{a}+\tan^{-1}\text{x}
\Big[\text{Since},\tan^{-1}\text{x}+\tan^{-1}\text{y}=\tan^{-1}\Big(\frac{\text{x}+\text{y}}{1-\text{xy}}\Big)\Big]
Differentiating it with respect to x,
\frac{\text{dy}}{\text{dx}}=\frac{\text{d}}{\text{dx}}(\tan^{-1}\text{a})+\frac{\text{d}}{\text{dx}}(\tan^{-1}\text{x})
=0+\frac{1}{1+\text{x}^2}
\frac{\text{dy}}{\text{dx}}=\frac{1}{1+\text{x}^2}
Q589. If \text{y}=\text{ae}^{2\text{x}}+\text{be}^{-\text{x}}, show that \frac{\text{d}^2\text{y}}{\text{dx}^2}-\frac{\text{dy}}{\text{dx}}-2\text{y}=0 3 Marks

Ans: Here,
\text{y}=\text{ae}^{2\text{x}}+\text{be}^{-\text{x}}
Differentiating w.r.t.x, we get
\frac{\text{dy}}{\text{dx}}=2\text{a}\text{e}^{2\text{x}}
Differentiating w.r.t.x, we get
\Rightarrow\frac{\text{d}^2\text{y}}{\text{dx}^2}=4\text{a}\text{e}^{2\text{x}}
\Rightarrow\frac{\text{d}^2\text{y}}{\text{dx}^2}=2\text{a}\text{e}^{2\text{x}}
\Rightarrow\frac{\text{d}^2\text{y}}{\text{dx}^2}=\frac{\text{dy}}{\text{dx}}+2\text{y}
\Rightarrow\frac{\text{d}^2\text{y}}{\text{dx}^2}-\frac{\text{dy}}{\text{dx}}-2\text{y}=0
Hence proved.
Q590. If \text{y}=3\text{e}^{2\text{x}}+2\text{e}^{3\text{x}} prove that \frac{\text{d}^2\text{y}}{\text{dx}^2}-5\frac{\text{dy}}{\text{dx}}+6\text{y}=0 3 Marks

Ans: \text{y}=3\text{e}^{2\text{x}}+2\text{e}^{3\text{x}}
Differentiating w.r.t.x, we get
\frac{\text{dy}}{\text{dx}}=6\text{e}^{2\text{x}}+6\text{e}^{3\text{x}}
Differentiating w.r.t.x, we get
\frac{\text{d}^2\text{y}}{\text{dx}^2}=12\text{e}^{2\text{x}}+18\text{e}^{3\text{x}}
\Rightarrow\frac{\text{d}^2\text{y}}{\text{dx}^2}=5(6\text{e}^{2\text{x}}+6\text{e}^{3\text{x}})-6(3\text{e}^{2\text{x}}+2\text{e}^{3\text{x}})
\Rightarrow\frac{\text{d}^2\text{y}}{\text{dx}^2}=5\Big(\frac{\text{dy}}{\text{dx}}\Big)-6\text{y}
\Rightarrow\frac{\text{d}^2\text{y}}{\text{dx}^2}-5\Big(\frac{\text{dy}}{\text{dx}}\Big)+6\text{y}=0
Q591. Differentiate the following w.r.t. x: 3 Marks
\sin\text{x}^2+\sin^2\text{x}+\sin^2(\text{x}^2)

Ans: Let \text{y}=\sin\text{x}^2+\sin^2\text{x}+\sin^2(\text{x}^2)


\Rightarrow\ \frac{\text{dy}}{\text{dx}}=\frac{\text{d}}{\text{dx}}\sin\big(\text{x}^2\big)+\frac{\text{d}}{\text{dx}}\big(\sin\text{x}\big)^2+\frac{\text{d}}{\text{dx}}(\sin\text{x}^2)^2
=\cos\big(\text{x}^2\big)\frac{\text{d}}{\text{dx}}\big(\text{x}^2\big)+2\sin\text{x}\cdot\frac{\text{d}}{\text{dx}}\sin\text{x}+2\sin^2\cdot\frac{\text{d}}{\text{dx}}\sin\text{x}^2
=2\text{x}\cos\text{x}^2+2\cdot\sin\text{x}\cdot\cos\text{x}+2\sin\text{x}^2\cos\text{x}^2\cdot\frac{\text{d}}{\text{dx}}\text{x}^2
=2\text{x}\cos\text{x}^2+2\cdot\sin\text{x}\cdot\cos\text{x}+2\sin\text{x}^2\cos\text{x}^2\cdot2\text{x}
=2\text{x}\cos\text{x}^2+\sin2\text{x}+\sin\big(2\text{x}^2\big)\cdot2\text{x}
=2\text{x}\cos\text{x}^2+2\text{x}\cdot\sin2\big(\text{x}^2\big)+\sin2\text{x}
Q592. Differentiate the following functions with respect to x: 3 Marks
\sin(\log\sin\text{x})

Ans: Consider \text{y}=\sin(\log\sin\text{x})


Differentiate with respect to x,
\frac{\text{dy}}{\text{dx}}=\frac{\text{d}}{\text{dx}}\sin(\log\sin\text{x})
=\cos(\log\sin\text{x})\frac{\text{d}}{\text{dx}}(\log\sin\text{x})
[Using chain rule]
=\cos(\log\sin\text{x})\times\frac{1}{\sin\text{x}}\frac{\text{d}}{\text{dx}}0\sin\text{x}
=\cos(\log\sin\text{x})\frac{\cos\text{x}}{\sin\text{x}}

https://bls.smartstudies.co.in/#/exam/pdf-preview/c59cb220-8e86-4716-9ff7-82aec16b1ade/1 123/158
5/26/24, 6:20 PM Exam Automation
=\cos(\log\sin\text{x})\times\cot\text{x}
Hence, the solution is, \frac{\text{d}}{\text{dx}}(\sin(\log\sin\text{x}))=\cos(\log\sin\text{x})\text{x}\cot\text{x}
Q593. Differentiate the following w.r.t. x: 3 Marks
\log\Big(\text{x}+\sqrt{\text{x}^2+\text{a}}\Big)

Ans: Let \text{y}=\log\Big(\text{x}+\sqrt{\text{x}^2+\text{a}}\Big)


\Rightarrow\ \frac{\text{dy}}{\text{dx}}=\frac{1}{\big(\text{x}+\sqrt{\text{x}^2+\text{a}}\big)}\cdot\frac{\text{d}}{\text{dx}}\Big[\text{x}+\sqrt{\text{x}^2+\text{a}}\Big]
=\frac{1}{\big(\text{x}+\sqrt{\text{x}^2+\text{a}}\big)}\Big[1+\frac{1}{2}(\text{x}^2+\text{a})^{\frac{-1}{2}}\cdot2\text{x}\Big]
=\frac{1}{\big(\text{x}+\sqrt{\text{x}^2+\text{a}}\big)}\cdot\Big(1+\frac{\text{x}}{\sqrt{\text{x}^2+\text{a}}}\Big)
-\frac{\big(\sqrt{\text{x}^2+\text{a}}+\text{x}\big)}{\big(\text{x}+\sqrt{\text{x}^2+\text{a}}\big)\big(\sqrt{\text{x}^2+\text{a}}\big)}
=\frac{1}{\big(\sqrt{\text{x}^2+\text{a}}\big)}
Q594. Differentiate the following w.r.t. x: 3 Marks
\sec^{-1}\Big(\frac{1}{4\text{x}^3-3\text{x}}\Big),0<\text{x}<\frac{1}{\sqrt{2}}

Ans: Let \text{y}=\sec^{-1}\Big(\frac{1}{4\text{x}^3-3\text{x}}\Big)


On putting \text{x}=\cos\theta, we get
\text{y}=\sec^{-1}\frac{1}{4\cos^3\theta-3\cos\theta}
=\sec^{-1}\frac{1}{\cos3\theta}
=\sec^{-1}(\sec3\theta)
=3\theta
=3\cos^{-1}\text{x} \big[\because\theta=\cos^{-1}\text{x}\big]
\therefore\ \frac{\text{dy}}{\text{dx}}=\frac{\text{d}}{\text{dx}}(3\cos^{-1}\text{x})
=\frac{-3}{\sqrt{1-\text{x}^2}}
Q595. Discuss the continuity of the following functions at the indicated point: 3 Marks
\text{f}\text{(x)}=\begin{cases}(\text{x}-\text{a}){\sin}\Big(\frac{1}{\text{x}-\text{a}}\Big) & \text{x} \neq \text{a}\\\ 0, & \text{ x} = \text{a}\end{cases}\text{at x}=\text{a}

Ans: We want, to check the continuity at x = 0.


\text{LHL}=\lim\limits_{\text{x} \rightarrow 0^-}\text{f}\text{ (x)}=\lim\limits_{\text{h} \rightarrow 0}\text{f}(0-\text{ h)}=\lim\limits_{\text{x} \rightarrow 0}(-\text{h)}^2
\sin\Big(\frac{1}{\text{-h}}\Big)=0
\text{RHL}=\lim\limits_{\text{x} \rightarrow 0^+}\text{f}\text{ (x)}=\lim\limits_{\text{h} \rightarrow 0}\text{f}(0+\text{h)}=\lim\limits_{\text{x} \rightarrow 0}\text{h}^2
\sin\Big(\frac{1}{\text{h}}\Big)=0
\text{f}(0)=0
Thus, LHL = RHL = f(0) = 0
Hence, the function is continuous at x = 0.
Q596. Differentiate the following functions with respect to x: 3 Marks
(\log\sin\text{x})^2

Ans: Let \text{y}=(\log\sin\text{x})^2


Differentiate it with respect to x we get,
\frac{\text{dy}}{\text{dx}}=\frac{\text{d}}{\text{dx}}(\log\sin\text{x})^2
=2(\log\sin\text{x})\frac{\text{d}}{\text{dx}}(\log\sin\text{x})
=2(\log\sin\text{x})\times\frac{1}{\sin\text{x}}\frac{\text{d}}{\text{dx}}(\sin\text{x})
=2(\log\sin\text{x})\times\frac{1}{\sin\text{x}}\cos\text{x}
=2(\log\sin\text{x})\cot\text{x}
So,
\frac{\text{d}}{\text{dx}}(\log\sin\text{x})^2=2(\log\sin\text{x})\cot\text{x}
Q597. If \text{y}=\tan^{-1} show that (1+\text{x}^2)\frac{\text{d}^2\text{y}}{\text{dx}^2}+2\text{x}\frac{\text{dy}}{\text{dx}}=0 3 Marks

Ans: \text{y}=\tan^{-1}
Differentiating w.r.t.x, we get
\Rightarrow\frac{\text{dy}}{\text{dx}}=\frac{1}{1+\text{x}^2}
\Rightarrow(1+\text{x}^2)\frac{\text{dy}}{\text{dx}}=1
Differentiating w.r.t.x, we get
\Rightarrow(1+\text{x}^2)\frac{\text{d}^2\text{y}}{\text{dx}^2}+2\text{x}\frac{\text{dy}}{\text{dx}}=0
Hence proved
Q598. In the following, find the value of the constant k so that the given function is continuous at the indicated point: 3 Marks
\text{f(x)}=\begin{cases}\frac{\text{x}^2-25}{\text{x}-5},&\text{x}\neq5\\\text{k},&\text{x}=5\end{cases}\text{at x} =5

Ans: Given,
\text{f(x)}=\begin{cases}\frac{\text{x}^2-25}{\text{x}-5},&\text{x}\neq5\\\text{k},&\text{x}=5\end{cases}
\Rightarrow\text{f(x)}=\begin{cases}\frac{(\text{x}-5)(\text{x}+5)}{\text{x}-5},&\text{x}\neq5\\\text{k},&\text{x}=5\end{cases}
\Rightarrow\text{f(x)}=\begin{cases}{\text{x}-5},&\text{x}\neq5\\\text{k},&\text{x}=5\end{cases}
If f(x) is continuous at x = 5, then,
\lim_\limits{\text{x}\rightarrow5}\text{f(x)}=\text{f}(5)
\Rightarrow\lim_\limits{\text{x}\rightarrow5}\text{(x}+5)=\text{k}
\Rightarrow\text{k}=5+5=10
Q599. Give an example of a function which is continuos but not differentiable at at a point. 3 Marks

Ans: Consider a function, \text{f(x)}=\begin{cases}\text{x}, & \text{x}> 0\\-\text{x}, & \text{x}\leq 0\end{cases}
This mod function is continuous at x = 0 but not differentiable at x = 0.
Continuity at x - 0, We have:
(LHL at x = 0)
\lim_\limits{\text{x}\rightarrow0^{-}}\text{f(x)}
=\lim_\limits{\text{x}\rightarrow0}\text{f}(0-\text{h})
=\lim_\limits{\text{x}\rightarrow0}-(0-\text{h})
=0
(RHL at x = 0)
\lim_\limits{\text{x}\rightarrow0^{+}}\text{f(x)}
=\lim_\limits{\text{x}\rightarrow0}\text{f}(0+\text{h})
=\lim_\limits{\text{x}\rightarrow0}(0+\text{h})
=0
and f(0) = 0
Thus, \lim_\limits{\text{x}\rightarrow0^{-}}\text{f(x)}=\lim_\limits{\text{x}\rightarrow0^{+}}\text{f(x)}=\text{f}(0).
Hence, f(x) is continuous at x = 0.
Now, we will check the differentiability at x = 0, we have:
(LHL at x = 0)
\lim_\limits{\text{x}\rightarrow0^{-}}\frac{\text{f(x)}-\text{f}(0)}{\text{x}-0}
=\lim_\limits{\text{h}\rightarrow0}\frac{\text{f}(0-\text{h})-\text{f}(0)}{0-\text{h}-0}
=\lim_\limits{\text{h}\rightarrow0}\frac{-(0-\text{h})-0}{-\text{h}}=-1

https://bls.smartstudies.co.in/#/exam/pdf-preview/c59cb220-8e86-4716-9ff7-82aec16b1ade/1 124/158
5/26/24, 6:20 PM Exam Automation
(RHL at x = 0)
\lim_\limits{\text{x}\rightarrow0^{+}}\frac{\text{f(x)}-\text{f}(0)}{\text{x}-0}
=\lim_\limits{\text{h}\rightarrow0}\frac{\text{f}(0+\text{h})-\text{f}(0)}{0+\text{h}-0}
=\lim_\limits{\text{h}\rightarrow0}\frac{(0+\text{h})-0}{-\text{h}}=1
Thus, \lim_\limits{\text{h}\rightarrow0^{-}}\text{f(x)}\neq\lim_\limits{\text{h}\rightarrow0^{+}}\text{f(x)}
Hence f(x) is not differentiable at x = 0.
Q600. If \text{y}=\sqrt{\log\text{x}+\sqrt{\log\text{x}+\sqrt{\log\text{x}+\ .... \text{to }\infty}}}, prove that (2\text{y}-1)\frac{\text{dy}}{\text{dx}}=\frac{1}{\text{x}} 3 Marks

Ans: We have, \text{y}=\sqrt{\log\text{x}+\sqrt{\log\text{x}+\sqrt{\log\text{x}+\ .... \text{to }\infty}}}


\Rightarrow\text{y}=\sqrt{\log\text{x}+\text{y}}
Squaring both sides, we get,
\text{y}^2=\log\text{x}+\text{y}
=2\text{y}\frac{\text{dy}}{\text{dx}}=\frac{1}{\text{x}}+\frac{\text{dy}}{\text{dx}}
\Rightarrow\frac{\text{dy}}{\text{dx}}(2\text{y}-1)=\frac{1}{\text{x}}
Q601. If \text{x}=\text{a}(\theta-\sin\theta)\text{ and},\text{y}=\text{a}(1+\cos\theta), find \frac{\text{dy}}{\text{dx}}\text{ at }\theta=\frac{\pi}{3} 3 Marks

Ans: Here,
\text{x}=\text{a}(\theta-\sin\theta)\text{ and y}=\text{a}(1+\cos\theta)
Then,
\frac{\text{dx}}{\text{d}\theta}=\frac{\text{d}}{\text{d}\theta}\big[\text{a}(\theta-\sin\theta)\big]=\text{a}(1-\cos\theta)
\frac{\text{dx}}{\text{d}\theta}=\frac{\text{d}}{\text{d}\theta}\big[\text{a}(1+\sin\theta)\big]=\text{a}(1-\sin\theta)
\therefore\frac{\text{dy}}{\text{dx}}=\Bigg[\frac{\frac{\text{dy}}{\text{d}\theta}}{\frac{\text{dx}}{\text{d}\theta}}=\frac{-\text{a}\sin\theta}{\text{a}(1-\cos\theta)}\Bigg]_{\theta=\frac{\pi}
{3}}
=-\frac{\sin\frac{\pi}{2}}{1-\cos\frac{\pi}{3}}=\frac{\frac{\sqrt{3}}{2}}{1-\frac{1}{2}}=-\sqrt{3}
Q602. If \text{y}=\text{e}^\text{x}\cos\text{x}, prove that \frac{\text{d}^2\text{y}}{\text{dx}^2}=2\text{e}^\text{x}\cos(\text{x}+\frac{\pi}{2}). 3 Marks

Ans: Here
\text{y}=\text{e}^\text{x}\cos\text{x}
Differentiating w.r.t.x, we get
\frac{\text{dy}}{\text{dx}}=\text{e}^\text{x}\cos\text{x}-\text{e}^\text{x}\sin\text{x}=\text{e}^\text{x}(\cos\text{x}-\sin\text{x})
Differentiating w.r.t.x, we get
\frac{\text{d}^2\text{y}}{\text{d}\text{x}^2}=\text{e}^\text{x}(\cos\text{x}-\sin\text{x})+\text{e}^\text{x}(-\sin\text{x}-\cos\text{x})
=\text{e}^\text{x}\cos\text{x}-\text{e}^\text{x}\sin\text{x}-\text{e}^\text{x}\sin\text{x}-\text{e}^\text{x}\cos\text{x}
=-2\text{e}^\text{x}\sin\text{x}
=2\text{e}^\text{x}\cos(\text{x}+\frac{\pi}{2})
Q603. Differentiate the following functions with respect to x: 3 Marks
\sin(3\text{x}+5)

Ans: Consider \text{y}=\sin(3\text{x}+5)


Differentiate y with the respect to x,
\frac{\text{dy}}{\text{dx}}=\frac{\text{d}}{\text{dx}}\big(\sin(3\text{x}+5)\big)
=\cos(3\text{x}+5)\frac{\text{d}}{\text{dx}}(3\text{x}+5)
[using chain rule]
=\cos(3\text{x}+5)\times[3(1)+0]
=3\cos(3\text{x}+5)
Hence, the solution is \frac{\text{d}}{\text{dx}}(\sin(3\text{x}+5))=3\cos(3\text{x}+5)
Q604. If \text{y}=\sin^{-1}\Big(\frac{2\text{x}}{1+\text{x}^2}\Big), write the value of \frac{\text{dy}}{\text{dx}}\text{ for x}>1. 3 Marks

Ans: We have, \text{y}=\sin^{-1}\Big(\frac{2\text{x}}{1+\text{x}^2}\Big)


Putting \text{x}=\tan\theta
\Rightarrow 1 <\tan\theta<\infty
\Rightarrow\frac{\pi}{4}<\theta<\frac{\pi}{2}
\frac{\pi}{2}<2\theta<\pi
\therefore\text{y}=\sin^{-1}(\sin2\theta)
\Rightarrow\text{y}=\sin^{-1}\big\{\sin(\pi-2\theta)\big\}
\Rightarrow\text{y}=\pi-2\theta
\Rightarrow\text{y}=\pi-2\tan^{-1}\text{x}
\Rightarrow\frac{\text{dy}}{\text{dx}}=0-\frac{2}{1+\text{x}^2}
\Rightarrow\frac{\text{dy}}{\text{dx}}=-\frac{2}{1+\text{x}^2}
Q605. Differentiate the following functions from first principles: 3 Marks
e3x.

Ans: Let f(x) = e3x


⇒ f(x + h) = e3(x + h)
\frac{\text{d}}{\text{dx}}(\text{f(x)})=\lim\limits_{\text{h}\rightarrow0}\frac{\text{f}(\text{x}+\text{h})-\text{f}(\text{x})}{\text{h}}
=\lim\limits_{\text{h}\rightarrow0}\frac{\text{e}^{3(\text{x}+\text{h})}-\text{e}^{3\text{x}}}{\text{h}}
=\lim\limits_{\text{h}\rightarrow0}\frac{\text{e}^{3\text{x}}\text{e}^{3\text{h}}-\text{e}^{3\text{x}}}{\text{h}}
=\lim\limits_{\text{h}\rightarrow0}\text{e}^{3\text{x}}\left\{\frac{(\text{e}^{3\text{h}}-1)}{3\text{h}}\right\}\times3
=3\text{e}^{3\text{x}}\Big[\text{Since, }\lim\limits_{\text{h}\rightarrow0}\frac{\text{e}^\text{x}-1}{\text{x}}=1\Big]
Hence,
\frac{\text{d}}{\text{dx}}(\text{e}^{3\text{x}})=3\text{e}^{3\text{x}}
Q606. Differentiate the following functions with respect to x: 3 Marks
\tan5\text{x}^\circ

Ans: Let, \text{y}=\tan5\text{x}^\circ


\Rightarrow\ \text{y}=\tan\Big(5\text{x}\times\frac{\pi}{180}\Big)
Differentiate it with respect to x we get,
\frac{\text{dy}}{\text{dx}}=\frac{\text{d}}{\text{dx}}\tan\Big(5\text{x}\times\frac{\pi}{180}\Big)
=\sec^2\Big(5\text{x}\times\frac{\pi}{180}\Big)\frac{\text{d}}{\text{dx}}\Big(5\text{x}\times\frac{\pi}{108}\Big)
[Using chain rule]
=\Big(\frac{5\text{x}}{180}\Big)\sec^2\Big(5\text{x}\times\frac{\pi}{180}\Big)
=\frac{5\pi}{180}\sec^2(5\text{x}^\circ)
Hence, \frac{\text{d}}{\text{dx}}(\tan5\text{x}^\circ)=\frac{5\pi}{180}\sec^2(5\text{x}^\circ)
Q607. Differentiate the following w.r.t. x: 3 Marks
\sin\sqrt{\text{x}}+\cos^2\sqrt{\text{x}}

Ans: Let \text{y}=\sin\sqrt{\text{x}}+\big(\cos^2\sqrt{\text{x}}\big)^2


\therefore\ \frac{\text{dy}}{\text{dx}}=\frac{\text{d}}{\text{dx}}\sin\Big(\text{x}^{\frac{1}{2}}\Big)+\frac{\text{d}}{\text{dx}}\Big[\cos\Big(\text{x}^{\frac{1}{2}}\Big)\Big]^2
=\cos\text{x}^{\frac{1}{2}}\cdot\frac{\text{d}}{\text{dx}}\text{x}^{\frac{1}{2}}+\cos\Big(\text{x}^{\frac{1}{2}}\Big)\frac{\text{d}}{\text{dx}}\Big[\cos\Big(\text{x}^{\frac{1}{2}}\Big)\Big]
=\cos\sqrt{\text{x}}\cdot\frac{1}{2\sqrt{\text{x}}}+2\cos\sqrt{\text{x}}\Big[-\sin\sqrt{\text{x}}\cdot\frac{1}{2\sqrt{\text{x}}}\Big]

https://bls.smartstudies.co.in/#/exam/pdf-preview/c59cb220-8e86-4716-9ff7-82aec16b1ade/1 125/158
5/26/24, 6:20 PM Exam Automation
=\frac{1}{2\sqrt{\text{x}}}\Big[\cos\big(\sqrt{\text{x}}\big)-\sin\big(2\sqrt{\text{x}}\big)\Big]
Q608. Verify Lagrange's mean value theorem for the following function on the indicated intervals. find a point 'c' in the indicated interval as stated by the Lagrange's mean value theorem. 3 Marks
\text{f}(\text{x})=\sin\text{x}-\sin2\text{x}-\text{x}\text{ on }[0,\pi]

Ans: We have,
\text{f}(\text{x})=\sin\text{x}-\sin2\text{x}-\text{x}
Since, \sin\text{x},\sin2\text{x}\ \&\ \text{x} are everywhere continuous and differentiable.
Therefore, f(x) is continuous on [0,\pi] and differentiable on (0,\pi)
Concequently, there exist some \text{c}\in(0,\pi)such that
\text{f}'(\text{c})=\frac{\text{f}(\pi)-\text{f}(0)}{\pi-0}=\frac{\text{f}(\pi)-\text{f}(0)}{\pi}
Now, \text{f}(\text{x})=\sin\text{x}-\sin2\text{x}-\text{x}
\text{f}'(\text{x})=\cos\text{x}-2\cos2\text{x}-1,\text{f}(\pi)=-\pi,\text{f}(0)=0
\therefore\ \text{f}'(\text{x})=\frac{\text{f}(\pi)-\text{f}(0)}{\pi-0}
\Rightarrow\cos\text{x}-2\cos2\text{x}-1=-1
\Rightarrow\cos\text{x}-2\cos2\text{x}=0
\Rightarrow\cos\text{x}-4\cos^2\text{x}=-2
\Rightarrow4\cos^2\text{x}-\cos\text{x}-2=0
\Rightarrow\cos\text{x}=\frac{1}{8}\big(1\pm\sqrt{33}\big)
\Rightarrow\text{x}=\cos^{-1}\Big[\frac{1}{8}\big(1\pm\sqrt{33}\big)\Big]
Thus, \text{c}=\cos^{-1}\Big(\frac{1\pm\sqrt{33}}{8}\Big)\in(0,\pi) such that \text{f}'(\text{c})=\frac{\text{f}(\pi)-\text{f}(0)}{\pi-0}.
Hence, Lagrange's mean value theorem is verified.
Q609. Differentiate w.r.t. x the function in Exercise: 3 Marks
\frac{\cos^{-1}\frac{\text{x}}{2}}{\sqrt{2\text{x}+7}},-2<\text{x}<2

Ans: Let \text{y}=\frac{\cos^{-1}\frac{\text{x}}{2}}{\sqrt{2\text{x}+7}}


By quotient rule, we obtain
\frac{\text{dy}}{\text{dx}}=\frac{\sqrt{2\text{x}+7}\frac{\text{d}}{\text{dx}}\Big(\cos^{-1}\frac{\text{x}}{2}\Big)-\Big(\cos^{-1}\frac{\text{x}}{2}\Big)\frac{\text{d}}{\text{dx}}
(\sqrt{2\text{x}+7)}}{(\sqrt{2\text{x}+7})^2}
=\frac{\sqrt{2\text{x}+7}\Bigg[\frac{1}{\sqrt{1-\Big(\frac{\text{x}}{2}\Big)^2}}.\frac{\text{d}}{\text{dx}}\Big(\frac{\text{x}}{2}\Big)\Bigg]-\Big(\cos^{-1}\frac{\text{x}}{2}\Big)\frac{1}
{2\sqrt{2\text{x}+7}}.\frac{\text{d}}{\text{dx}}(2\text{x}+7)}{2\text{x}+7}
=\frac{\sqrt{2\text{x}+7}\frac{-1}{\sqrt{4-\text{x}^2}}-\Big(\cos^{-1}\frac{\text{x}}{2}\Big)\frac{2}{2\sqrt{2\text{x}+7}}}{2\text{x}+7}
=\frac{-\sqrt{2\text{x}+7}}{\sqrt{4-\text{x}^2}\times(2\text{x}+7)}-\frac{\cos^{-1}\frac{\text{x}}{2}}{(\sqrt{2\text{x}+7})(2\text{x}+7)}
=-\Bigg[\frac{1}{\sqrt{4-\text{x}^2}\sqrt{2\text{x}+7}}+\frac{\cos^{-1}\frac{\text{x}}{2}}{(2\text{x}+7)^{\frac{3}{2}}}\Bigg]
Q610. If \text{y}=\log\sqrt{\tan\text{x}}, write \frac{\text{dy}}{\text{dx}}. 3 Marks

Ans: We have, \text{y}=\log\sqrt{\tan\text{x}}


\Rightarrow\text{y}=\log(\tan\text{x})^\frac{1}{2}
\Rightarrow\text{y}=\frac{1}{2}\log(\tan\text{x})\big[\because\log\text{a}^\text{b}=\text{b}\log\text{a}\big]
\Rightarrow\frac{\text{dy}}{\text{dx}}=\frac{1}{2}\times\frac{1}{\tan\text{x}}\frac{\text{d}}{\text{dx}}(\tan\text{x})
\Rightarrow\frac{\text{dy}}{\text{dx}}=\frac{1}{2}\times\frac{1}{\tan\text{x}}(\sec^2\text{x})
\Rightarrow\frac{\text{dy}}{\text{dx}}=\frac{1}{2\frac{\sin\text{x}}{\cos\text{x}}}\times\cos^2\text{x}
\Rightarrow\frac{\text{dy}}{\text{dx}}=\frac{1}{2\sin\text{x}\cos\text{x}}
\Rightarrow\frac{\text{dy}}{\text{dx}}=\frac{1}{\sin2\text{x}}
\Rightarrow\frac{\text{dy}}{\text{dx}}=\text{cosec }2\text{x}
Q611. Using Rolle's theorem, find points on the curve \text{y}=16-\text{x}^2,\text{x}\in[-1,1], where tagent is parallel to x-axis. 3 Marks

Ans: The equation of the curve is,


\text{y}=16-\text{x}^2\ ....(1)
Let P(x1, y1) be a point on it where the tangent is parallel to x-axis.
Then,
\Big(\frac{\text{dy}}{\text{dx}}\Big)_\text{p}=0\ ....(2)
Differentiating (1) with respect to x, we get
\frac{\text{dy}}{\text{dx}}=-2\text{x}
\Rightarrow\Big(\frac{\text{dy}}{\text{dx}}\Big)_\text{p}=-2\text{x}_1
\Rightarrow-2\text{x}_1=0 (from(2))
\Rightarrow\text{x}_1=0
P(x1, y1) lies on the curve y = 16 - x2
\therefore\text{y}_1=16-\text{x}_1^2
When x1 = 0,
y1 = 16
Hence, (0, 16) is the required point.
Q612. Discuss the applicability of the Rolle's theorem for the following function on the indicated interval 3 Marks
\text{f}(\text{x})=\begin{cases}-4\text{x}+5,&0\leq\text{x}\leq1\\2\text{x}-3,&1<\text{x}\leq2\end{cases}

Ans: The given function is


\text{f}(\text{x})=\begin{cases}-4\text{x}+5,&0\leq\text{x}\leq1\\2\text{x}-3,&1<\text{x}\leq2\end{cases}
At x = 0, we have
\lim\limits_{\text{x}\rightarrow1^-}\text{f}(\text{x})=\lim\limits_{\text{h}\rightarrow0}\text{f}(1-\text{h})=\lim\limits_{\text{h}\rightarrow0}[-4(1-\text{h}+5)]=1
\lim\limits_{\text{x}\rightarrow1^+}\text{f}(\text{x})=\lim\limits_{\text{h}\rightarrow0}\text{f}(1+\text{h})=\lim\limits_{\text{h}\rightarrow0}[2(1+\text{h}-3)]=-1
\therefore\ \lim\limits_{\text{x}\rightarrow1^-}\text{f}(\text{x})\neq\lim\limits_{\text{x}\rightarrow1^+}\text{f}(\text{x})
Thus, f(x) is discontinuous at x = 1.
Hence, Rolle's theorem is not applicable for the given function.
Q613. Differentiate the following functions with respect to x: 3 Marks
\tan(\text{x}^\circ+45^\circ)

Ans: Let, \text{y}=\tan(\text{x}^\circ+45^\circ)


\Rightarrow\text{y}=\tan\Big\{(\text{x}+45)\frac{\pi}{180}\Big\}
Differentiating it with respect to x we get,
\frac{\text{dx}}{\text{dy}}=\frac{\text{d}}{\text{dx}}\tan\Big\{(\text{x}+45)\frac{\pi}{180}\Big\}
=\sec^2\Big\{(\text{x}+45)\frac{\pi}{180}\Big\}\times\frac{\text{d}}{\text{dx}}(\text{x}+45)\frac{\pi}{180}
[Using chain rule]
=\frac{\pi}{180}\sec^2(\text{x}^\circ+45^\circ)
So,
=\frac{\text{d}}{\text{dx}}\Big\{\tan(\text{x}^\circ+45^\circ)\Big\}=\frac{\pi}{180}\sec^2(\text{x}^\circ+45^\circ)
Q614. Differentiate (x2 – 5x + 8) (x3 + 7x + 9) in three ways mentioned below: 3 Marks
by using product rule

Ans: Let y = (x2 - 5x + 8)(x3 + 7x + 9) ....(i)


\frac{\text{dy}}{\text{dx}}=(\text{x}^2-5\text{x}+8)\frac{\text{d}}{\text{dx}}(\text{x}^3+7\text{x}+9)+(\text{x}^3+7\text{x}+9)\frac{\text{d}}{\text{dx}}(\text{x}^2-5\text{x}+8)

https://bls.smartstudies.co.in/#/exam/pdf-preview/c59cb220-8e86-4716-9ff7-82aec16b1ade/1 126/158
5/26/24, 6:20 PM Exam Automation
\Rightarrow\ \frac{\text{dy}}{\text{dx}}=(\text{x}^2-5\text{x}+8)(3\text{x}^2+7)+(\text{x}^3+7\text{x}+9)(2\text{x}-5)
\Rightarrow\ \frac{\text{dy}}{\text{dx}}=3\text{x}^4+7\text{x}^2-15\text{x}^3-35\text{x}+24\text{x}^2+56+2\text{x}^4-5\text{x}^3+14\text{x}^2-35\text{x}+18\text{x}-45
\Rightarrow\ \frac{\text{dy}}{\text{dx}}=5\text{x}^4-20\text{x}^3+45\text{x}^2+11\ \dots\text{(ii)}
Q615. Differentiate the following functions with respect to x: 3 Marks
3^{\text{e}^{\text{x}}}

Ans: Let, \text{y}=3^{\text{e}^{\text{x}}}


Differentiate it with respect to x,
\frac{\text{dy}}{\text{dx}}=\frac{\text{d}}{\text{dx}}\Big(3^{\text{e}^\text{x}}\Big)
=3^{\text{e}^\text{x}}\log3\frac{\text{d}}{\text{dx}}\big(\text{e}^\text{x}\big)
[Using chain rule]
=\text{e}^\text{x}\times3^{\text{e}^\text{x}}\log3
So,
\frac{\text{d}}{\text{dx}}\Big(3^{\text{e}^\text{x}}\Big)=\text{e}^\text{x}\times3^{\text{e}^\text{x}}\log3
Q616. If \text{f(x)}=\begin{cases}2\text{x}^2+\text{k},&\text{if }\text{ x}\geq0\\-2\text{x}^2+\text{k},&\text{if }\text{ x}<0\end{cases}, then what should be the value of k so that f(x) is continuous 3 Marks
at x = 0.

Ans: It is given that function is continous at x = 0 then,


\text{LHL}=\text{RHL}=\text{f}(0)\ ....(\text{i})
Now, \text{f}(0)=2\times0+\text{k}=\text{k}
​\text{LHL}=\lim_\limits{\text{x}\rightarrow0^-}\text{f(x)}\lim_\limits{\text{h}\rightarrow0}=\lim_\limits{\text{h}\rightarrow0}-2(-\text{h})^2+\text{k}=\text{k}
\text{RHL}=\lim_\limits{\text{x}\rightarrow0^+}\text{f(x)}=\lim_\limits{\text{h}\rightarrow0}\text{f}(0+\text{h})=\lim_\limits{\text{h}\rightarrow0}2(\text{h}^2)+\text{k}=\text{k}
Thus, the function will be continuous for any \text{k}\in\text{R}
Q617. Differentiate: 3 Marks
\tan(\text{x}^\circ+45^\circ)

Ans: Let, \text{y}=\tan(\text{x}^\circ+45^\circ)


\Rightarrow \text{y}=\tan\Big\{(\text{x}+45)\frac{\pi}{180}\Big\}
Differentiate it with respect to x we get,
\frac{\text{dy}}{\text{dx}}=\frac{\text{d}}{\text{dx}}\tan\Big\{(\text{x}+45)\frac{\pi}{180}\Big\}
=\sec^2\Big\{(\text{x}+45)\frac{\pi}{180}\Big\}\times\frac{\text{d}}{\text{dx}}(\text{x}+45)\frac{\pi}{180}
[Using chain rule]
=\frac{\pi}{180}\sec^2(\text{x}^\circ+45^\circ)
So,
\frac{\text{d}}{\text{dx}}\big\{\tan(\text{x}^\circ+45^\circ)\big\}=\frac{\pi}{180}\sec^2(\text{x}^\circ+45^\circ)
Q618. Find \frac{\text{dy}}{\text{dx}} of the functions given in Exercise: 3 Marks
\text{xy}=\text{e}^{(\text{x}-\text{y})}

Ans: Given: \text{xy}=\text{e}^{\text{x}-\text{y}}\ \Rightarrow\ \log\text{xy}\ \log=\text{e}^{\text{x}-\text{y}}


\Rightarrow\ \log\text{x}+\log\text{y}=(\text{x}-\text{y})\log\text{e}\ \Rightarrow\ \log\text{x}+\log\text{y}=(\text{x}-\text{y})\ \ [\because\log\text{e}=1]
\Rightarrow\ \frac{\text{d}}{\text{dx}}\log\text{x}+\frac{\text{d}}{\text{dx}}\log\text{y}=\frac{\text{d}}{\text{dx}}(\text{x}-\text{y})\ \Rightarrow\ \frac{1}{\text{x}}+\frac{1}
{\text{y}}\frac{\text{dy}}{\text{dx}}=1-\frac{\text{dy}}{\text{dx}}
\Rightarrow\ \frac{1}{\text{y}}\frac{\text{dy}}{\text{dx}}+\frac{\text{dy}}{\text{dx}}=1-\frac{1}{\text{x}}\ \Rightarrow\ \frac{\text{dy}}{\text{dx}}\Big(\frac{1}
{\text{y}}+1\Big)=\frac{\text{x}-1}{\text{x}}
\Rightarrow\ \frac{\text{dy}}{\text{dx}}\Big(\frac{1+\text{y}}{\text{y}}\Big)=\frac{\text{x}-1}{\text{x}}\ \Rightarrow\ \frac{\text{dy}}{\text{dx}}=\frac{\text{y}(\text{x}-1)}{\text{x}(1+\text{y})}
Q619. Differentiate the following functions with respect to x: 3 Marks
\tan^{-1}(\text{e}^{\text{x}})

Ans: Consider \text{y}=\tan^{-1}(\text{e}^{\text{x}})


Differentiate with respect to x,
\frac{\text{dy}}{\text{dx}}=\frac{\text{d}}{\text{dx}}\big(\tan^{-1}\text{e}^{\text{x}}\big)
=\frac{1}{1+\big(\text{e}^{2\text{x}}\big)^2}=\frac{\text{d}}{\text{dx}}\big(\text{e}^\text{x}\big)
[using chain rule]
=\frac{1}{1+\text{e}^{2\text{x}}}\times\text{e}^\text{x}
=\frac{\text{e}^\text{x}}{1+\text{e}^{2\text{x}}}
Hence, the solution is, \frac{\text{d}}{\text{dx}}\big(\tan^{-1}\text{e}^\text{x}\big)=\frac{\text{e}^\text{x}}{1+\text{e}^{2\text{x}}}
Q620. Find the second order derivatives of the following functions: 3 Marks
\text{y}=\text{x}.\cos\text{x}

Ans: let \text{y}=\text{x}.\cos\text{x}


Then,
\frac{\text{dy}}{\text{dx}}=\frac{\text{d}}{\text{dx}}(\text{x}.\cos\text{x})=\cos\text{x}.\frac{\text{d}}{\text{dx}}(\text{x})+\text{x}\frac{\text{d}}{\text{dx}}(\cos\text{x})
=\cos.1+\text{x}(-\sin\text{x})=\cos\text{x}-\text{x}\sin\text{x}
\therefore\frac{\text{d}^2\text{y}}{\text{dx}^2}=\frac{\text{d}}{\text{dx}}[\cos\text{x}-\text{x}\sin\text{x}]=\frac{\text{d}}{\text{dx}}(\cos\text{x})-\frac{\text{d}}{\text{dx}}
(\text{x}\sin\text{x})
=-\sin\text{x}-\Big[\sin\text{x}.\frac{\text{d}}{\text{dx}}(\text{x})+\text{x}.\frac{\text{d}}{\text{dx}}(\sin\text{x})\Big]
=-\sin\text{x}=(\sin\text{x}+\text{x}\cos\text{x})
Q621. If \text{x}=\text{e}^{\frac{\text{x}}{\text{y}}}, prove that \frac{\text{dy}}{\text{dx}}=\frac{\text{x}-\text{y}}{\text{x}\log\text{x}}. 3 Marks

Ans: We have, \text{x}=\text{e}^{\frac{\text{x}}{\text{y}}}


Differentiating both sides w.r.t. x, we get
\therefore\ 1=\text{e}^{\frac{\text{x}}{\text{y}}}\frac{\text{d}}{\text{dx}}\Big(\frac{\text{x}}{\text{y}}\Big)
\Rightarrow\ 1=\text{e}^{\frac{\text{x}}{\text{y}}}\bigg[\frac{\text{y}\cdot1-\text{x}\frac{\text{dy}}{\text{dx}}}{\text{y}^2}\bigg]
\Rightarrow\ \text{y}^2=\text{y}\cdot\text{e}^{\frac{\text{x}}{\text{y}}}-\text{x}\cdot\frac{\text{dy}}{\text{dx}}\cdot\text{e}^{\frac{\text{x}}{\text{y}}}
\Rightarrow\ \frac{\text{dy}}{\text{dx}}=\frac{\text{y}\Big(\text{e}^{\frac{\text{x}}{\text{y}}}-\text{y}\Big)}{\text{x}\cdot\text{e}^{\frac{\text{x}}{\text{y}}}}
=\frac{\text{e}^{\frac{\text{x}}{\text{y}}}-\text{y}}{\frac{\frac{\text{x}}{\text{y}}\text{e}^{\frac{\text{x}}{\text{y}}}}{}}
=\frac{\text{x}-\text{y}}{\text{x}\cdot\log\text{x}} \Big[\because\ \text{x}=\text{e}^{\frac{\text{x}}{\text{y}}}\Rightarrow\log\text{x}=\frac{\text{x}}{\text{y}}\Big]
Hence proved
Q622. A function f(x) is defined as, 3 Marks
\text{f}\text{(x)}=\begin{cases}\frac{\text{x}^2-\text{x}-6}{\text{x}-3}&; &\text{if} \text{x}\neq3\\5 &;&\text{if}\text{ x}=3\end{cases}
show that f(x) is continuous that x = 3.

Ans: We have, to check the continuity at x = 3.


\text{L.H.L}=\lim\limits_{\text{x} \rightarrow 3^-}\text{f}\text{(x)}=\lim\limits_{\text{h} \rightarrow 0}\text{f}(3-\text{h})=\lim\limits_{\text{h} \rightarrow 0}\frac{(3-\text{h})^2-(3-
\text{h})-6}{(3-\text{h})-3}
=\lim\limits_{\text{h} \rightarrow 0}\frac{\text{h}^2-5\text{h}}{-\text{h}}=\lim\limits_{\text{h} \rightarrow 0}-\text{h}+5=5
\text{R.H.L}=\lim\limits_{\text{x} \rightarrow 3^+}\text{f}\text{(x)}=\lim\limits_{\text{h} \rightarrow 0}\text{f}(3\text{+h)}=\lim\limits_{\text{h} \rightarrow 0}\frac{(3+\text{h})^2-
(3+\text{h})-6}{(3+\text{h})-3}
=\lim\limits_{\text{h} \rightarrow 0}\frac{\text{h}^2+5\text{h}}{\text{h}}=\lim\limits_{\text{h}\rightarrow 0}\text{h}+5=5
\text{f}(3)=5

https://bls.smartstudies.co.in/#/exam/pdf-preview/c59cb220-8e86-4716-9ff7-82aec16b1ade/1 127/158
5/26/24, 6:20 PM Exam Automation
Thus, We have, LHL = RHL = f(3) = 5
So,The function is continus at x = 3
Q623. Prove that the function f(x) = 5x – 3 is continuous at x = 0, at x = – 3 and at x = 5. 3 Marks

Ans: Here f(x) = 5x - 3


1. \ \ \ \text{Lt}\ \ \ \ \ \ \text{f(x)}\\ \text{x} \rightarrow 0 = \ \ \ \text{Lt}\ \ \ \ \ \ (5\text{x}-3) = 5(0) - 3 = 0 - 3 = -3\\ \ \ \ \ \text{x} \rightarrow 0
Now f is defined at x = 0
and f(0) = 5(0) - 3 = 0 - 3 = -3
\therefore \ \ \text{Lt}\ \ \ \ \ \ \ \ \ \ \ \text{f(x)} = \text{f}(0) = -3\\ \ \ \ \text{x}\rightarrow0
\therefore f is continous at x = 0
2. \ \ \ \text{Lt}\ \ \ \ \ \ \text{f(x)}\\ \text{x} \rightarrow -3= \ \ \ \text{Lt}\ \ \ \ \ \ (5\text{x}-3) = 5(-3) - 3 = -15- 3 = -18\\ \ \ \ \ \text{x} \rightarrow -3
Now f is defined at x = -3
and f(-3) = 5(-3) - 3 = -15 - 3 = -18
\therefore \ \ \text{Lt}\ \ \ \ \ \ \ \ \ \ \ \text{f(x)} = \text{f}(-3) = -18\\ \ \ \ \text{x}\rightarrow-3
\therefore f is continous at x = -3
3. \ \ \ \text{Lt}\ \ \ \ \ \ \text{f(x)}\\ \text{x} \rightarrow 5= \ \ \ \text{Lt}\ \ \ \ \ \ (5\text{x}-3) = 5(5) - 3 = 25- 3 = 22\\ \ \ \ \ \text{x} \rightarrow 5
Now f is defined at x = 5
and f(5) = 5(5) - 3 = 25 - 3 = 22
\therefore \ \ \text{Lt}\ \ \ \ \ \ \ \ \ \ \ \text{f(x)} = \text{f}(5) = 22\\ \ \ \ \text{x}\rightarrow5
\therefore f is continous at x = 5
Q624. Differentiate the following functions with respect to x: 3 Marks
\tan(\text{e}^{\sin\text{x}})

Ans: Consider \text{y}=\tan(\text{e}^{\sin\text{x}})


Differentiate with respect to x,
\frac{\text{dy}}{\text{dx}}=\frac{\text{d}}{\text{dx}}\big[\tan\text{e}^{\sin\text{x}}\big]
=\sec^2\big(\text{e}^{\sin\text{e}}\big)\frac{\text{d}}{\text{dx}}\big(\text{e}^{\sin\text{x}}\big)
[Using chain rule]
=\sec^2\big(\text{e}^{\sin\text{x}}\big)\times\text{e}^{\sin\text{x}}\times\frac{\text{d}}{\text{dx}}(\sin\text{ x})
Q625. Differentiate the following functions with respect to x: 3 Marks
\log\Big\{\cot\Big(\frac{\pi}{4}+\frac{\pi}{2}\Big)\Big\}

Ans: \frac{\text{d}}{\text{dx}}\Big[\log\Big\{\cot\Big(\frac{\Pi}{4}+\frac{\pi}{2}\Big)\Big\}\Big]
\frac{1}{\cot\Big(\frac{\pi}{4}+\frac{\text{x}}{2}\Big)}\times\Big(-\text{cosec}^2\Big(\frac{\pi}{4}+\frac{\text{x}}{2}\Big)\Big)\times\frac{1}{2}
\frac{-1}{2\cos\Big(\frac{\pi}{4}+\frac{\text{x}}{2}\Big)\sin\Big(\frac{\pi}{4}+\frac{\text{x}}{2}\Big)}=-\frac{1}{\sin2\Big(\frac{\pi}{4}+\frac{\text{x}}{2}\Big)}
=-\frac{1}{\sin2\Big(\frac{\pi}{4}+\frac{\text{x}}{2}\Big)}=-\frac{1}{\cos\text{x}}=-\sec\text{x}
Q626. Differentiate \log(1+\text{x}^2) with respect to \tan^{-1}\text{x} 3 Marks

Ans: Let \text{u}=\log(1+\text{x}^2)


Differentiating it with respect to x using chain rule,
\frac{\text{du}}{\text{dx}}=\frac{1}{(1+\text{x}^2)}\frac{\text{d}}{\text{dx}}(1+\text{x}^2)
=\frac{1}{(1+\text{x}^2)}(2\text{x})
\frac{\text{du}}{\text{dx}}=\frac{2\text{x}}{(1+\text{x}^2)}\ .....(\text{i})
Let \text{v}=\tan^{-1}\text{x}
Differentiating it with respect to x,
\frac{\text{dv}}{\text{dx}}=\frac{1}{1+\text{x}^2}\ .....(\text{ii})
Dividing equation (i) by (ii),
\frac{\frac{\text{du}}{\text{dx}}}{\frac{\text{dv}}{\text{dx}}}=\frac{2\text{x}}{(1+\text{x}^2)}\times\frac{(1+\text{x}^2)}{1}
\frac{\text{du}}{\text{dx}}=2\text{x}
Q627. If x and y are connected parametrically by the equations given in Exercise without eliminating the parameter, Find \frac{\text{dy}}{\text{dx}}. 3 Marks
\text{x}=\text{a}\sec\theta,\text{y}=\text{b}\tan\theta

Ans: The given equations are \text{x}=\text{a}\sec\theta\text{ and y}=\text{b}\tan\theta


Then, \frac{\text{dx}}{\text{d}\theta}= \text{a}.\frac{\text{d}}{\text{d}\theta}(\sec\theta)=\text{a}\sec\theta\tan\theta
\frac{\text{dy}}{\text{d}\theta}=\text{b}\frac{\text{d}}{\text{d}\theta}(\tan\theta)=\text{b}\sec^2\theta
\therefore\ \frac{\text{dy}}{\text{dx}}=\frac{\Big(\frac{\text{dy}}{\text{d}\theta}\Big)}{\Big(\frac{\text{dx}}{\text{d}\theta}\Big)}=\frac{\text{b}\sec^2\theta}
{\text{a}\sec\theta\tan\theta}=\frac{\text{b}}{\text{a}}\sec\theta\cot\theta =\frac{\text{b}\cos\theta}{\text{a}\cos\theta\sin\theta}=\frac{\text{b}}{\text{a}}\times\frac{1}
{\sin\theta}=\frac{\text{b}}{\text{a}}\ \text{cose}\theta
Q628. Prove that the function f given by: 3 Marks
\text{f(x)} = |\text{x} - 1|, \text{x} \in \text{R}
is not differentiable at x = 1.

Ans: Given: \text{f(x)} = |\text{x} - 1|\ \therefore \ \text{f(1)} = |1 - 1| = 0


\text{R}\text{f}{'}(1) = ^{\ \ \text{lim}}_{\text{h}\rightarrow\text{0}}\frac{\text{f(1 + h)}-\text{f}(1)}{\text{h}} = ^{\ \ \text{lim}}_{\text{h}\rightarrow\text{0}}\frac{|1 + \text{h} - 1|-0}
{\text{h}} = ^{\ \ \text{lim}}_{\text{h}\rightarrow\text{0}}\frac{|\text{h}|}{\text{h}}= ^{\ \ \text{lim}}_{\text{h}\rightarrow\text{0}}\frac{\text{h}}{\text{h}}=1
\text{And}\ \text{L}\text{f}{'}(1) = ^{\ \ \text{lim}}_{\text{h}\rightarrow\text{0}}\frac{\text{f(1 - h)}-\text{f}(1)}{-\text{h}} = ^{\ \ \text{lim}}_{\text{h}\rightarrow\text{0}}\frac{|1 - \text{h} -
1|-0}{-\text{h}} = ^{\ \ \text{lim}}_{\text{h}\rightarrow\text{0}}\frac{|-\text{h}|}{\text{h}}= ^{\ \ \text{lim}}_{\text{h}\rightarrow\text{0}}\frac{-\text{h}}{\text{h}}=-1
Since \text{R }\text{f}{'}(1)\neq \text{L}{\text{f}}{'}(1)
Therefore, f(x) is not differentiable at x =1.
Q629. Write the value of the derivative of f(x) = |x − 1| + |x − 3| at x = 2. 3 Marks

Ans: Given: f(x) = |x - 1| + |x - 3|


\Rightarrow\text{f(x)}=\begin{cases}-(\text{x}-1)-(\text{x}-3), & \text{x}<1\\ \text{x}-1-(\text{x}-3),& 1\leq\text{x}<3\\(\text{x}-1)+(\text{x}-3),&\text{x}\geq3\end{cases}
\Rightarrow\text{f(x)}=\begin{cases}-2\text{x}+4, & \text{x}<1\\ 2,& 1\leq\text{x}<3\\2\text{x}-4,&\text{x}\geq3\end{cases}
Wecheck differentiable at x = 2
(LHL at x = 2)
\lim\limits_{\text{x}\rightarrow2^{-}}\frac{\text{f(x)}-\text{f}(2)}{\text{x}-2}
=\lim\limits_{\text{h}\rightarrow0}\frac{\text{f}(2-\text{h})-\text{f}(2)}{2-\text{h}-2}
=\lim\limits_{\text{h}\rightarrow0}\frac{2-2}{-\text{h}}
=0
Q630. \text{If y}=\cos^{-1}\text{x},\text{ Find }\frac{\text{d}^2\text{y}}{\text{dx}^2}\text{ in terms of y alone}. 3 Marks

Ans: \text{y}=\cos^{-1}\text{x}\ \ \dots(1)


\therefore\ \frac{\text{dy}}{\text{dx}}=-\frac{1}{\sqrt{1-\text{x}^2}}=-(1-\text{x}^2)^{-\frac{1}{2}}
\therefore\ \frac{\text{dy}}{\text{dx}}=\frac{1}{2}(1-\text{x}^2)^{\frac{-3}{2}}(-2\text{x})=-\frac{\text{x}}{(1-\text{x}^2)^{\frac{3}{2}}}=-\frac{\cos\text{y}}{(1-\cos^2\text{y})^{\frac{3}{2}}}\ \
[\because\text{of }(1)]
=-\frac{\cos\text{y}}{(1-\cos^2\text{y})^{\frac{3}{2}}}=-\frac{\cos\text{y}}{\sin^3\text{y}}=-\frac{\cos\text{y}}{\sin\text{y}}.\frac{1}{\sin^2\text{y}}=-\cot\text{y cosec}^2\text{y}.
Q631. If \text{y}=\text{e}^{-\text{x}}\cos\text{x}, show that \frac{\text{d}^2\text{y}}{\text{dx}^2}=2\text{e}^{-\text{x}}\sin\text{x}. 3 Marks

Ans: Here,

https://bls.smartstudies.co.in/#/exam/pdf-preview/c59cb220-8e86-4716-9ff7-82aec16b1ade/1 128/158
5/26/24, 6:20 PM Exam Automation
\text{y}=\text{e}^{-\text{x}}\cos\text{x},
differentiating w.r.t.x, we get
\frac{\text{dy}}{\text{dx}}=-\text{e}^{-\text{x}}\cos\text{x}
=-\text{e}-\text{x}\sin\text{x}+\text{e}-\text{x}\cos\text{x}
differentiating w.r.t.x, we get
\frac{\text{d}^2\text{y}}{\text{dx}^2}=-\text{e}^{-\text{x}}\cos\text{x}-\text{e}^{-\text{x}}\sin\text{x}-\text{e}^{\text{-x}}\cos\text{x}
=2\text{e}^{-\text{x}}\sin\text{x}
Q632. Differentiate the following w.r.t. x: 3 Marks
\tan^{-1}(\sec\text{x}+\tan\text{x}),\frac{-\pi}{2}<\text{x}<\frac{\pi}{2}

Ans: Let \text{y}=\tan^{-1}(\sec\text{x}+\tan\text{x})


\therefore\ \frac{\text{dy}}{\text{dx}}=\frac{\text{d}}{\text{dx}}\tan^{-1}(\sec\text{x}+\tan\text{x})
=\frac{1}{1+(\sec\text{x}+\tan\text{x})^2}\cdot\frac{\text{d}}{\text{dx}}(\sec\text{x}+\tan\text{x})
=\frac{1}{1+\sec^2\text{x}+\tan^2\text{x}+2\sec\text{x}\cdot\tan\text{x}}\big[\sec\text{x}\cdot\tan\text{x}+\sec^2\text{x}\big]
=\frac{1}{\big(\sec^2\text{x}+\sec^2\text{x}+2\sec\text{x}\cdot\tan\text{x}\big)}\cdot\sec\text{x}\cdot(\sec\text{x}+\tan\text{x})
=\frac{1}{2\sec\text{x}(\tan\text{x}+\sec\text{x})}\cdot\sec\text{x}(\sec\text{x}+\tan\text{x})=\frac{1}{2}
Q633. If \text{y}=\text{e}^\text{x}(\sin\text{x}+\cos\text{x}) prove that \frac{\text{d}^2\text{y}}{\text{dx}^2}-2\frac{\text{dy}}{\text{dx}}+2\text{y}=0 3 Marks

Ans: \text{y}=\text{e}^\text{x}(\sin\text{x}+\cos\text{x})
Differentiating w.r.t.x, we get
\Rightarrow\frac{\text{dy}}{\text{dx}}=\text{e}^\text{x}(\cos\text{x}-\sin\text{x})+(\sin\text{x}+\text{cos}\text{x})\text{e}^\text{x}
\Rightarrow\frac{\text{dy}}{\text{dx}}=\text{y}+\text{e}^\text{x}(\cos\text{x}-\sin\text{x})
Differentiating w.r.t.x, we get
\Rightarrow\frac{\text{d}^2\text{y}}{\text{dx}^2}=\frac{\text{dy}}{\text{dx}}+\text{e}^\text{x}(-\sin\text{x}-\cos\text{x})+(\cos\text{x}-\sin\text{x})\text{e}^\text{x}
=\frac{\text{dy}}{\text{dx}}-\text{y}+(\cos\text{x}-\sin\text{x})\text{e}^\text{x}
Adding and substracting y on RHS
\Rightarrow\frac{\text{d}^2\text{y}}{\text{dx}^2}=\frac{\text{dy}}{\text{dx}}+\text{e}^\text{x}(-\sin\text{x}-\cos\text{x})+(\cos\text{x}-\sin\text{x})\text{e}^\text{x}
\Rightarrow\frac{\text{d}^2\text{y}}{\text{dx}^2}-2\frac{\text{dy}}{\text{dx}}+2\text{y}=0
Q634. Find the value of k for which \text{f(x)}=\begin{cases}\frac{1-\cos4\text{x}}{8\text{x}^2},&\text{when x}\neq0\\\text{k},&\text{when x}=0\end{cases} is continous at x = 0. 3 Marks

Ans: Given,
\text{f(x)}=\begin{cases}\frac{1-\cos4\text{x}}{8\text{x}^2},&\text{when x}\neq0\\\text{k},&\text{when x}=0\end{cases}
If f(x) is continuous at x = 0, then
\lim_\limits{\text{x}\rightarrow 0}\text{f(x)}=\text{f}(0)
\Rightarrow\lim_\limits{\text{x}\rightarrow 0}\frac{1-\cos4\text{x}}{8\text{x}^2}=\text{f}(0)
\Rightarrow\lim_\limits{\text{x}\rightarrow 0}\frac{2\sin^22\text{x}}{8\text{x}^2}=\text{f}(0)
\Rightarrow\frac{2}{2}\lim_\limits{\text{x}\rightarrow 0}\frac{\sin^22\text{x}}{4\text{x}^2}=\text{f}(0)
\Rightarrow\frac{2}{2}\lim_\limits{\text{x}\rightarrow 0}\Big(\frac{\sin2\text{x}}{2\text{x}^2}\Big)^2=\text{f}(0)
\Rightarrow1\times1=\text{f}(0)
\Rightarrow\text{k}=1 (\because\text{f}(0)=\text{k})
Q635. Differentiate the following w.r.t. x: 3 Marks
2^{\cos^2}\text{x}

Ans: Let \text{y}=2^{\cos^2}\text{x}


Taking logarithm on both sides, we get
\log\text{y}=\log2\cos^2\text{x}
\Rightarrow\ \frac{\text{d}}{\text{dy}}(\log\text{y})\cdot\frac{\text{dy}}{\text{dx}}=\log2\frac{\text{d}}{\text{dx}}(\cos^2\text{x})
\Rightarrow\ \frac{1}{\text{y}}\cdot\frac{\text{dy}}{\text{dx}}=\log2(2\cos\text{x})\frac{\text{d}}{\text{dx}}\cos\text{x}
\Rightarrow\ \frac{1}{\text{y}}\cdot\frac{\text{dy}}{\text{dx}}=\log2\cdot2\cos\text{x}\cdot(-\sin\text{x})
\Rightarrow\ \frac{1}{\text{y}}\cdot\frac{\text{dy}}{\text{dx}}=\log2\cdot[-(\sin2\text{x})]
\Rightarrow\ \frac{\text{dy}}{\text{dx}}=-\text{y}\cdot\log2(\sin2\text{x})
\Rightarrow\ \frac{\text{dy}}{\text{dx}}=-2^{\cos^2}\text{x}\cdot\log2(\sin2\text{x}) \big[\because\text{y}=2^{\cos^2}\text{x}\big]
Q636. Differentiate the following w.r.t.x: \sin(\tan^{-1}\text{e}^{-\text{x}}) 3 Marks

Ans: \text{Let}\ \text{y}=\sin(\tan^{-1}\text{e}^{-\text{x}})


\therefore\ \frac{\text{dy}}{\text{dx}}=\cos(\tan^{-1}\text{e}^{-\text{x}}).\frac{\text{d}}{\text{dx}}(\tan^{-1}\text{e}^{-\text{x}})= \bigg[\because\frac{\text{d}}
{\text{dx}}\sin\text{f(x)}=\cos\text{f(x)}\frac{\text{d}}{\text{dx}}\text{f(x)}\bigg]
=\cos(\tan^{-1}\text{e}^{-\text{x}}).\frac{1}{1+(\text{e}^{-\text{x}})^{2}}\frac{\text{d}}{\text{dx}}\text{e}^{-\text{x}}= \bigg[\because\frac{\text{d}}{\text{dx}}\ \tan^{-1} \text{f(x)}=\frac{1}
{(\text{f(x)})^{2}}\frac{\text{d}}{\text{dx}}\text{f(x)}\bigg]
=\cos(\tan^{-1}\text{e}^{-\text{x}}).\frac{1}{1+\text{e}^{-\text{2x}}}\text{e}^{\text{-x}}\frac{\text{d}}{\text{dx}}({-\text{x}})
=\frac{-\text{e}^{-\text{x}}.\cos(\tan^{-1}\text{e}^{-\text{x}})}{1+\text{e}^{-2\text{x}}}
Q637. Differentiate the following functions from first principles: 3 Marks
e-x.

Ans: Consider f(x) = e-x


⇒ f(x + h) = e-(x+h)
\frac{\text{d}}{\text{dx}}(\text{f}(\text{x}))=\frac{\lim}{\text{h}\rightarrow0}\frac{\text{f}(\text{x}+\text{h})-\text{f}(\text{x})}{\text{h}}
=\frac{\lim}{\text{h}\rightarrow0}\frac{\text{e}^{-(\text{x}+\text{h})}\text{e}^{-\text{x}}}{\text{h}}
=\frac{\lim}{\text{h}\rightarrow0}\frac{\text{e}^{-\text{x}}\times\text{e}^{-\text{h}}-\text{e}^{-\text{x}}}{\text{h}}
=\frac{\lim}{\text{h}\rightarrow0}\text{e}^{-\text{x}}\left\{\Big(\frac{\text{e}^{-\text{h}}-1}{-\text{h}}\Big)\right\}\times(-1)
\Big[\text{Since, }\frac{\lim}{\text{h}\rightarrow0}\frac{\text{e}^\text{h}-1}{\text{h}}=1\Big]
=-\text{e}^{-\text{x}}
So,
\frac{\text{d}}{\text{dx}}(\text{e}^{-\text{x}})=-\text{e}^{-\text{x}}
Q638. If \text{y}=\text{ax}^{\text{n+1}}+\text{bx}^{-\text{n}} and \text{x}^2\frac{\text{d}^2\text{y}}{\text{dx}^2}=\lambda\text{y} then write the value of \lambda 3 Marks

Ans: \text{y}=\text{ax}^{\text{n}+1}+\text{b}\text{x}^{-\text{-n}}
and \text{x}^2\frac{\text{d}^2\text{y}}{\text{dx}^2}=\lambda\text{y}
Now,
\frac{\text{dy}}{\text{dx}}=\text{a}(\text{n}+1)\text{x}^{\text{n}}-\text{bn x}^{-\text{n-1}}
and \frac{\text{d}^2\text{y}}{\text{dx}^2}=\text{an}(\text{n}+1)\text{x}^{\text{n}+1}-\text{bn}(-\text{n}-1)\text{x}^{-\text{n}-2}
Now, \text{x}^2\frac{\text{d}^2\text{y}}{\text{dx}^2}=\lambda\text{y}[\text{given}]
\Rightarrow\text{x}^2[\text{an}(\text{n}+1)\text{x}^{\text{n}-1}+\text{bn}(\text{n}+1)\text{x}^{-\text{n}-2}]=\lambda(\text{ax}^{\text{n+1}}+\text{b x}^{-\text{n}})
\Rightarrow\text{an}(\text{n}+1)\text{x}^{\text{n}+1}+\text{bn}(\text{n}+1)\text{x}^{-\text{n}}=\lambda\text{ax}^{\text{n+1}}+\text{b x}^{-\text{n}}
\Rightarrow\text{n}(\text{n}+1)\text{ax}^{\text{n}+1}+\text{bx}^{-n}=\lambda\text{ax}^{\text{n}+1}+\text{dx}^{\text{-n}}
\Rightarrow\lambda=\text{n}(\text{n}+1)
Q639. If \text{y}=\log\Big(\sqrt{\text{x}}+\frac{1}{\sqrt{\text{x}}}\Big), prove that \frac{\text{dy}}{\text{dx}}=\frac{\text{x}-1}{2\text{x}(\text{x}+1)} 3 Marks

Ans: We have, \text{y}=\log\Big(\sqrt{\text{x}}+\frac{1}{\sqrt{\text{x}}}\Big)


Differentiating with respect to x,

https://bls.smartstudies.co.in/#/exam/pdf-preview/c59cb220-8e86-4716-9ff7-82aec16b1ade/1 129/158
5/26/24, 6:20 PM Exam Automation
\frac{\text{dy}}{\text{dx}}=\frac{\text{d}}{\text{dx}}\log\Big(\sqrt{\text{x}}+\frac{1}{\sqrt{\text{x}}}\Big)
=\frac{1}{\sqrt{\text{x}}+\frac{1}{\sqrt{\text{x}}}}\frac{\text{d}}{\text{dx}}\Big(\sqrt{\text{x}}+\frac{1}{\sqrt{\text{x}}}\Big)
=\frac{\sqrt{\text{x}}}{\text{x}+1}\Big(\frac{1}{2\sqrt{\text{x}}}-\frac{1}{2\text{x}\sqrt{\text{x}}}\Big)
=\frac{1}{2}\frac{\sqrt{\text{x}}}{\text{x}+1}\Big(\frac{\text{x}-1}{\text{x}\sqrt{\text{x}}}\Big)
=\frac{\text{x}-1}{2\text{x}(\text{x}+1)}
So,
\frac{\text{dy}}{\text{dx}}=\frac{\text{x}-1}{2\text{x}(\text{x}+1)}
Q640. Differentiate the following functions with respect to x: 3 Marks
\sin(\log\text{x})

Ans: Consider \text{y}=\sin(\log\text{x})


Differentiate it with respect to x,
\frac{\text{dy}}{\text{dx}}=\frac{\text{d}}{\text{dx}}\sin(\log\text{x})
=\cos(\log\text{x})\frac{\text{d}}{\text{dx}}(\log\text{x})
[Using chain rule]
=\frac{1}{\text{x}}\cos(\log\text{x})
Hence, the solution is \frac{\text{d}}{\text{dx}}=(\sin(\log\text{x}))=\frac{1}{\text{x}}\cos(\log\text{x})
Q641. If \text{y}=\frac{\log\text{x}}{\text{x}}, show that \frac{\text{d}^2\text{y}}{\text{dx}^2}=\frac{2\log\text{x}-3}{\text{x}^3}. 3 Marks

Ans: Here,
\text{y}=\frac{\log\text{x}}{\text{x}},
Differentiating w.r.t.x, we get
\frac{\text{d}\text{y}}{\text{dx}}=\frac{1-\log\text{x}}{\text{x}^2}
Differentiating w.r.t.x, we get
\frac{\text{d}^2\text{y}}{\text{dx}^2}=\frac{-\text{x}-2\text{x}(1-\log\text{x})}{\text{x}^4}
=\frac{-\text{x}-2\text{x}+2\text{x}\log\text{x}}{\text{x}^4}
=\frac{-3+2\log\text{x}}{\text{x}^3}
=\frac{2\log\text{x}-3}{\text{x}^3}
Hence proved
Q642. Differentiate the function given in Exercise: 3 Marks
\sqrt{\frac{(\text{x}-1)(\text{x}-2)}{(\text{x}-3)(\text{x}-4)(\text{x}-5)}}

Ans: Let \text{y}=\sqrt{\frac{(\text{x}-1)(\text{x}-2)}{(\text{x}-3)(\text{x}-4)(\text{x}-5)}}=\Big(\frac{(\text{x}-1)(\text{x}-2)}{(\text{x}-3)(\text{x}-4)(\text{x}-5)}\Big)^{\frac{1}{2}}\ \dots\text{(i)}


Taking logs on both sides, we have
\log\text{y}=\frac{1}{2}[\log(\text{x}-1)+\log(\text{x}-2)-\log(\text{x}-3)-\log(\text{x}-4)-\log(\text{x}-5)]
\therefore\ \frac{1}{\text{y}}\frac{\text{dy}}{\text{dx}}=\frac{1}{2}\Big[\frac{1}{\text{x}-1}\frac{\text{d}}{\text{dx}}(\text{x}-1)+\frac{1}{\text{x}-2}\frac{\text{d}}{\text{dx}}(\text{x}-2)-\frac{1}
{\text{x}-3}\frac{\text{d}}{\text{dx}}(\text{x}-3)-\frac{1}{\text{x}-4}\frac{\text{d}}{\text{dx}}(\text{x}-4)-\frac{1}{\text{x}-5}\frac{\text{d}}{\text{dx}}(\text{x}-5)\Big]
\Rightarrow\ \frac{\text{dy}}{\text{dx}}=\frac{1}{2}\text{y}\Big[\frac{1}{\text{x}-1}+\frac{1}{\text{x}-2}-\frac{1}{\text{x}-3}-\frac{1}{\text{x}-4}-\frac{1}{\text{x}-5}\Big]
\Rightarrow\ \frac{\text{dy}}{\text{dx}}=\frac{1}{2}\sqrt{\frac{(\text{x}-1)(\text{x}-2)}{(\text{x}-3)(\text{x}-4)(\text{x}-5)}}\Big[\frac{1}{\text{x}-1}+\frac{1}{\text{x}-2}-\frac{1}{\text{x}-3}-
\frac{1}{\text{x}-4}-\frac{1}{\text{x}-5}\Big]\ \text{[From eq.(i)}]
Q643. If f(x) is an odd function, then write whether f'(x) is even of odd. 3 Marks

Ans: We have, f(x) is an odd function.


\Rightarrow\text{f}(-\text{x})=-\text{f}(\text{x})
\Rightarrow\frac{\text{d}}{\text{dx}}\big\{\text{f}(-\text{x})\big\}=-\frac{\text{d}}{\text{dx}}\big\{\text{f}(\text{x})\big\}
\Rightarrow\text{f}'(-\text{x})\frac{\text{d}}{\text{dx}}(-\text{x})=-\text{f}'\text{(x)}
\Rightarrow\text{f}'(-\text{x})\times(-1)=-\text{f}'\text{(x)}
\Rightarrow\text{f}'(-\text{x})=-\text{f}'\text{(x)}
\Rightarrow\text{f}'(-\text{x})=\text{f}'\text{(x)}
Thus, f'(x) is an even function.
Q644. Using the fact that \sin(\text{A}+\text{B})=\sin\text{A}\cos\text{B}+\cos\text{A}\sin\text{B}and the differentiation, obtain the sum formula for cosines. 3 Marks

Ans: \sin(\text{A}+\text{B})=\sin\text{A}\cos\text{B}+\cos\text{A}\sin\text{B}
Differentiating both sides with respect to x, we obtain
\frac{\text{d}}{\text{dx}}[\sin(\text{A}+\text{B})]=\frac{\text{d}}{\text{dx}}(\sin\text{A}\cos\text{B})+\frac{\text{d}}{\text{dx}}(\cos\text{A}\sin\text{B})
\Rightarrow\ \cos(\text{A}+\text{B}).\frac{\text{d}}{\text{dx}}(\text{A}+\text{B})
=\cos\text{B}.\frac{\text{d}}{\text{dx}}(\sin\text{A})+\sin\text{A}.\frac{\text{d}}{\text{dx}}(\cos\text{B})+\sin\text{B}.\frac{\text{d}}{\text{dx}}(\cos\text{A})+\cos\text{A}.\frac{\text{d}}
{\text{dx}}(\sin\text{B})
\Rightarrow\ \cos(\text{A}+\text{B}).\frac{\text{d}}{\text{dx}}(\text{A}+\text{B})
=\cos\text{B}.\cos\text{A}\frac{\text{dA}}{\text{dx}}+\sin\text{A}(-\sin\text{B})\frac{\text{dB}}{\text{dx}}+\sin\text{B}(-\sin\text{A})\frac{\text{dA}}
{\text{dx}}+\cos\text{A}\cos\text{B}\frac{\text{dB}}{\text{dx}}
\Rightarrow\ \cos(\text{A}+\text{B}).\Big[\frac{\text{dA}}{\text{dx}}+\frac{\text{dB}}{\text{dx}}\Big]
=(\cos\text{A}\cos\text{B}-\sin\text{A}\sin\text{B}).\Big[\frac{\text{dA}}{\text{dx}}+\frac{\text{dB}}{\text{dx}}\Big]
\therefore\ \cos(\text{A}+\text{B})=\cos\text{A}\cos\text{B}-\sin\text{A}\sin\text{B}
Q645. Show that \text{f}\text{(x)}=\begin{cases}1+\text{x}^2,&\text{if } 0\leq\text{x}\leq 1\\2-\text{x},&\text{if }\text{x} > 1\end{cases} is discontinuous at x = 1. 3 Marks

Ans: Given,
\text{f}\text{(x)}=\begin{cases}1+\text{x}^2,&\text{if } 0\leq\text{x}\leq 1\\2-\text{x},&\text{if }\text{x} > 1\end{cases}
We observe
\text{(LHL at x}=1)\lim\limits_{\text{x} \rightarrow 1^-}\text{f}\text{(x)}=\lim\limits_{\text{h} \rightarrow 0}\text{f}(1-\text{h)}
=\lim\limits_{\text{h} \rightarrow 0}(1+1-\text{h)}^2=\lim\limits_{\text{h} \rightarrow 0}(2+\text{h}^2-\text{2h)}=2
\text{(RHL at x}=1)\lim\limits_{\text{x} \rightarrow 1^+}\text{f}\text{(x)}=\lim\limits_{\text{h} \rightarrow 0}\text{f}(1+\text{h)}
=\lim\limits_{\text{h} \rightarrow 0}(2-(1+\text{h))}=\lim\limits_{\text{h} \rightarrow 0}(1-\text{h)}=1
\lim\limits_{\text{x} \rightarrow 1^-}\text{f}\text{(x)}\neq\lim\limits_{\text{x} \rightarrow 1^+}\text{f}\text{(x)}
Thus, f(x) is discontinuous at x = 1.
Q646. Find which of the function: 3 Marks
\text{f(x)}=\begin{cases}3\text{x}+5,&\text{if x}\geq2\\\text{x}^2,&\text{if x}<2\end{cases}
at x = 2

Ans: We have, \text{f(x)}=\begin{cases}3\text{x}+5,&\text{if x}\geq2\\\text{x}^2,&\text{if x}<2\end{cases}


At x = 2, \text{L.H.L}​=\​ lim\limits_{\text{x}\rightarrow2^-}(\text{x})^2
=\lim\limits_{\text{h}\rightarrow0}(2-\text{h}^2)=\lim\limits_{\text{h}\rightarrow0}(4+\text{h}^2-4\text{h})=4
And \text{R.H.L}=\lim\limits_{\text{x}\rightarrow2^+}(3\text{x}+5)
=\lim\limits_{\text{h}\rightarrow0}\big[3(2+\text{h})+5\big]=11
Since, L.H.L ≠ R.H.L at x = 2
So, f(x) is discontinuous at x = 2.
Q647. Let g(x) be the inverse of an invertible function f(x) which is derivable at x = 3. If f(3) = 3 and f(3) = 9, write the value of g'(9). 3 Marks

Ans: We have, f(3) = 9, f'(3) = 9


and g(x) = f-1 (x)

https://bls.smartstudies.co.in/#/exam/pdf-preview/c59cb220-8e86-4716-9ff7-82aec16b1ade/1 130/158
5/26/24, 6:20 PM Exam Automation
\Rightarrow(\text{gof})(\text{x})=\text{x}
\Rightarrow\text{g}\{\text{f(x)}\}=\text{x}
\Rightarrow\frac{\text{d}}{\text{dx}}\big[\text{g}\{\text{f}\text{(x)}\}\big] = 1
\Rightarrow\text{g}'\big\{\text{f}\text{(x)}\big\}\frac{\text{d}}{\text{dx}}\big\{\text{f}(\text{x})\big\}=1
\Rightarrow\text{g}'\big\{\text{f}\text{(x)}\big\}\times\text{f}'\text{(x)}=1
Putting x = 3, we get,
\text{g}'\big\{\text{f}(3)\big\}\times\text{f}'(3)=1
\Rightarrow\text{g}'(9)\times9=1\big[\because\text{f}(3)=9,\text{f}'(3)=9\big]
\Rightarrow\text{g}'(9)=\frac{1}{9}
Q648. Discuss the continuity of the function f(x) at the point x = 0, where 3 Marks
\text{f}\text{(x)}=\begin{cases}\text{x}, & \text{x} > 0\\1,&\text{x}=0\\\text{-x}, & \text{x} > 0\end{cases}

Ans: Given,
\text{f}\text{(x)}=\begin{cases}\text{x}, & \text{x} > 0\\1,&\text{x}=0\\\text{-x}, & \text{x} > 0\end{cases}
\text{(LHL at x}=0)=\lim\limits_{\text{x} \rightarrow 0^-}\text{f}\text{(x)}=\lim\limits_{\text{h} \rightarrow 0}\text{f}(0-\text{h)}
=\lim\limits_{\text{h} \rightarrow 0}\text{f}\text{(-h)}=\lim\limits_{\text{h} \rightarrow 0}-(-\text{h)}=0
\text{(RHL at x}=0)=\lim\limits_{\text{x} \rightarrow 0^+}\text{f}\text{(x)}=\lim\limits_{\text{h} \rightarrow 0}\text{f}(0+\text{h)}
\lim\limits_{\text{h} \rightarrow 0}\text{f}\text{(h)}=0
And, \text{f}(0)=1
\therefore\lim\limits_{\text{x} \rightarrow 0^-}\text{f}\text{(x)}=\lim\limits_{\text{x} \rightarrow 0^+}\text{f}\text{(x)}\neq\text{f}(0).
Hence, f(x) is discontinuous at x = 0.
Q649. If \text{f(x)}=\begin{cases}\frac{\text{x}^2}{2},&\text{if }0\leq\text{ x}\leq1\\2\text{x}^2-3\text{x}+\frac{3}{2},&\text{if }1<\text{x}\leq2\end{cases} Show that f is continuous at x = 1. 3 Marks

Ans: We want to discuss the continuity of the function at x = 1


We need to prove that
​\text{LHL}=​​\text{RHL}=\text{f}(1)
\text{f}(1)=\frac{1^2}{2}=\frac{1}{2}
\text{LHL}=\lim_\limits{\text{x}\rightarrow1^-}\text{f(x)}=\lim_\limits{\text{h}\rightarrow0}\text{f}(1-\text{h})=\lim_\limits{\text{h}\rightarrow0}\frac{(1-\text{h})^2}{2}=\frac{1}{2}
\text{RHL}=\lim_\limits{\text{x}\rightarrow1^+}\text{f(x)}=\lim_\limits{\text{h}\rightarrow0}(1+\text{h})=\lim_\limits{\text{h}\rightarrow0}2(1+\text{h}^2)-3(1+\text{h})+\frac{3}{2}
=2-3+\frac{3}{2}=\frac{1}{2}
Thus, \text{LHL}=\text{RHL}=\text{f}(1)=\frac{1}{2}
Hence, function is continuous at x = 1
Q650. Find \frac{\text{dy}}{\text{dx}} in the following cases: 3 Marks
\tan^{-1}\big(\text{x}^2+\text{y}^2\big)=\text{a}

Ans: We have, \tan^{-1}\big(\text{x}^2+\text{y}^2\big)=\text{a}


Differentiating with respect to x, we get,
\frac{\text{d}}{\text{dx}}\big[\tan^{-1}\big(\text{x}^2+\text{y}^2\big)\big]=\frac{\text{d}}{\text{dx}}(\text{a})
\Rightarrow\frac{1}{1+(\text{x}^2+\text{y}^2)^2}\times\frac{\text{d}}{\text{dx}}\big(\text{x}^2+\text{y}^2\big)=0
\Rightarrow\Big[\frac{1}{1+(\text{x}^2+\text{y}^2)^2}\Big]\Big(2\text{x}+2\text{y}\frac{\text{dy}}{\text{dx}}\Big)=0
\Rightarrow2\text{x}+2\text{y}\frac{\text{dy}}{\text{dx}}=0
\Rightarrow\text{x}+\text{y}\frac{\text{dy}}{\text{dx}}=0
\Rightarrow\frac{\text{dy}}{\text{dx}}=-\frac{\text{x}}{\text{y}}
Q651. Find \frac{\text{dy}}{\text{dx}} of the functions expressed in parametric: 3 Marks
\sin\text{x}=\frac{2\text{t}}{1+\text{t}},\ \tan\text{y}=\frac{2\text{t}}{1-\text{t}^2}.

Ans: We have, \sin\text{x}=\frac{2\text{t}}{1+\text{t}} and \tan\text{y}=\frac{2\text{t}}{1-\text{t}^2}


Let \text{t}=\tan\text{z}
\therefore\ \sin\text{x}=\frac{2\tan\text{z}}{1+\tan^2\text{z}}=\sin2\text{z}
\therefore\ \text{x}=2\text{z}
Also \tan\text{y}=\frac{2\tan\text{z}}{1-\tan^2\text{z}}=\tan2\text{z}
\therefore\ \text{y}=2\text{z}
\therefore\ \text{y}=\text{x}
\therefore\ \frac{\text{dy}}{\text{dx}}=1
Q652. Find the value of k in this question, so that the function f is continuous at the indicated point: 3 Marks
\text{f(x)}=\begin{cases}\frac{2^{\text{x}+2}-16}{4^\text{x}-16},&\text{if x}\neq2\\\text{k},&\text{if x}=2\end{cases} at x = 2.

Ans: Consider, \text{f(x)}=\begin{cases}\frac{2^{\text{x}+2}-16}{4^\text{x}-16},&\text{if x}\neq2\\\text{k},&\text{if x}=2\end{cases} at x = 2


Since, f(x) is continuous at x = 2.
\therefore L.H.L = R.H.L = f(2)
At x = 2, =\lim\limits_{\text{h}\rightarrow2}\frac{2^\text{x}\cdot2^2-2^4}{4^\text{x}-4^2}=\lim\limits_{\text{h}\rightarrow2}\frac{4\cdot(2^\text{x}-4)}{(2^\text{x})^2-(4)^2}
=\lim\limits_{\text{h}\rightarrow2}\frac{4\cdot(2^\text{x}-4)}{(2^\text{x}-4)-(2^\text{x}+4)}
=\lim\limits_{\text{h}\rightarrow2}\frac{4}{2^\text{x}+4}=\frac{8}{4}=\frac{1}{2}
But f(2) = k
\therefore\ \text{k}=\frac{1}{2}
Q653. Examine the continuity of f, where f is defined by 3 Marks
​\text{f(x)}=\begin{cases} \sin{\text{x}- \cos\text{x}}, \text{if} \ \text{x}\neq0\\-1, \ \ \ \ \ \ \ \ \ \ \ \ \ \ \ \text{if}\ \text{x} = 0\end{cases}
Find the values of k so that the function f is continuous at the indicated point in Exercises 26 to 29.

Ans: It is given that ​\text{f(x)}=\begin{cases} \sin{\text{x}- \cos\text{x}}, \text{if} \ \text{x}\neq0\\-1, \ \ \ \ \ \ \ \ \ \ \ \ \ \ \ \text{if}\ \text{x} = 0\end{cases}
We know that f is defined at all point of the real line.
Let k be a real number.
Case I: \text{k} \neq 0,
Then \text{f(k)} =\sin\text{k}- \cos\text{k}
^{\ \ \text{lim}}_{\text{x}\rightarrow\text{k}}\text{f(x)} = ^{\ \ \text{lim}}_{\text{x}\rightarrow\text{k}}(\sin\text{x}- \cos\text{x}) = \sin\text{k} - \cos\text{k}
\therefore\ ^{\ \ \text{lim}}_{\text{x}\rightarrow\text{k}}\text{f(x)} = \text{f(k)}
Thus, f is continuous at all points x that is \text{x}\neq0.
Case II: \text{k} = 0
Then f(k) = f(0) = 0
^{\ \ \text{lim}}_{\text{x}\rightarrow\text{0}^{-}}\text{f(x)} = ^{\ \ \text{lim}}_{\text{x}\rightarrow\text{0}^{-}}(\sin\text{x} - \cos\text{x}) = \sin0 - \cos0 = 0 - 1 = - 1
^{\ \ \text{lim}}_{\text{x}\rightarrow\text{0}^{+}}\text{f(x)} = ^{\ \ \text{lim}}_{\text{x}\rightarrow\text{0}^{+}}(\sin\text{x} - \cos\text{x}) = \sin0 - \cos0 = 0 - 1 = - 1
\therefore \ ^{\ \ \text{lim}}_{\text{x}\rightarrow\text{0}^{-}}\text{f(x)} = ^{\ \ \text{lim}}_{\text{x}\rightarrow\text{0}^{+}}\text{f(x)} = \text{f(0)}
Therefore , f is continuous at x = 0.
Therefore, f is has no point of discontinuity.
Q654. Differentiate the functions given in Exercise: 3 Marks
(\log\text{x})^{\cos\text{x}}

Ans: Let \text{y}=(\log\text{x})^{\cos\text{x}}\ \dots\text{(i)}


Taking logs on both sides, we have

https://bls.smartstudies.co.in/#/exam/pdf-preview/c59cb220-8e86-4716-9ff7-82aec16b1ade/1 131/158
5/26/24, 6:20 PM Exam Automation
\log\text{y}=\log(\log\text{x})^{\cos\text{x}}=\cos\text{x}\log(\log\text{x})
\therefore\ \frac{\text{d}}{\text{dx}}\log\text{y}=\frac{\text{d}}{\text{dx}}\ [\cos\text{x}\log(\log\text{x)}]
\Rightarrow\ \frac{1}{\text{y}}\frac{\text{dy}}{\text{dx}}=\cos\text{x}\frac{\text{d}}{\text{dx}}\log(\log\text{x})+\log(\log\text{x})\frac{\text{d}}{\text{dx}}\cos\text{x}\ \ \text{[By product
rule}]
\Rightarrow\ \frac{1}{\text{y}}\frac{\text{dy}}{\text{dx}}=\cos\text{x}\frac{1}{\log\text{x}}\frac{\text{d}}{\text{dx}}(\log\text{x})+\log(\log\text{x})(-\sin\text{x})
\Rightarrow\ \frac{1}{\text{y}}\frac{\text{dy}}{\text{dx}}=\frac{\cos\text{x}}{\log\text{x}}.\frac{1}{\log\text{x}}-\sin\text{x}\log(\log\text{x})
\Rightarrow\ \frac{\text{dy}}{\text{dx}}=\text{y}\Big[\frac{\cos\text{x}}{\log\text{x}}-\sin\text{x}\log(\log\text{x})\Big] =(\log\text{x})^{\cos\text{x}}\Big[\frac{\cos\text{x}}{\log\text{x}}-
\sin\text{x}\log(\log\text{x})\Big]
Q655. Examine the continuity of the function f(x) = x3 + 2x2 - 1 at x = 1. 3 Marks

Ans: We know that, function f will be continuous at x = a, if \lim\limits_{\text{x}\rightarrow\text{a}^-}\text{f(x)}=\lim\limits_{\text{x}\rightarrow\text{a}^+}\text{f(x)}=\text{f(a)}.


Consider, f(x) = x3 + 2x2 - 1 at x = 1.
\lim\limits_{\text{x}\rightarrow1^+}\text{f(x)}=\lim\limits_{\text{h}\rightarrow0}(1+\text{h})^3+2(1+\text{h})^2-1=2
and
\lim\limits_{\text{x}\rightarrow1^-}\text{f(x)}=\lim\limits_{\text{h}\rightarrow0}(1-\text{h})^3+2(1-\text{h})^2-1=2
\because\ \lim\limits_{\text{x}\rightarrow1^+}\text{f(x)}=\lim\limits_{\text{x}\rightarrow1^-}\text{f(x)}
And f(1) = 1 + 2 - 1 = 2
Thus, f(x) is continuous at x = 1.
Q656. Differentiate the following w.r.t. x: 3 Marks
\cos\big(\tan\sqrt{\text{x}+1}\big)

Ans: Let \text{y}=\cos\big(\tan\sqrt{\text{x}+1}\big)


\therefore\ \frac{\text{dy}}{\text{dx}}=\frac{\text{d}}{\text{dx}}\cos\big(\tan\sqrt{\text{x}+1}\big) =-\sin\big(\tan\sqrt{\text{x}+1}\big)\cdot\frac{\text{d}}
{\text{dx}}\big(\tan\sqrt{\text{x}+1}\big)
=-\sin\big(\tan\sqrt{\text{x}+1}\big).\sec^2\sqrt{\text{x}+1}.\frac{\text{d}}{\text{dx}}(\text{x}+1)^{\frac{1}{2}} \Big[\because\frac{\text{d}}{\text{dx}}(\tan\text{x})=\sec^2\text{x}\Big]
=-\sin\big(\tan\sqrt{\text{x}+1}\big).\big(\sec\sqrt{\text{x}+1}\big)^2.\frac{1}{2}(\text{x}+1)^{\frac{1}{2}}.\frac{\text{d}}{\text{dx}}(\text{x}+1)
=\frac{-1}{2\sqrt{\text{x}+1}}.\sin\big(\tan\sqrt{\text{x}+1}\big).\sec^2\big(\sqrt{\text{x}+1}\big)
Q657. If \text{x}=\frac{1+\log\text{t}}{\text{t}^2},\text{y}=\frac{3+2\log\text{t}}{\text{t}}, find \frac{\text{dy}}{\text{dx}} 3 Marks

Ans: \text{x}=\frac{1+\log\text{t}}{\text{t}^2},\text{y}=\frac{3+2\log\text{t}}{\text{t}}
\frac{\text{dy}}{\text{dt}}=\frac{\text{t}^2\big(\frac{1}{\text{t}}\big)-(1+\log\text{t})(2\text{t})}{\text{t}^4} \\ =\frac{\text{t}-2\text{t}-2\text{t}\log\text{t}}{\text{t}^4}=\frac{-2\log\text{t}-1}
{\text{t}^3}
\frac{\text{dy}}{\text{dt}}=\frac{\text{t}\big(\frac{2}{\text{t}}\big)-(3+2\log\text{t})(1)}{\text{t}^2} \\ =\frac{2-3-2\log\text{t}}{\text{t}^2}=\frac{-2\log\text{t}-1}{\text{t}^2}
\frac{\text{dy}}{\text{dx}}=\frac{\frac{\text{dy}}{\text{dt}}}{\frac{\text{dx}}{\text{dt}}}=\frac{\frac{-2\log\text{t}-1}{\text{t}^2}}{\frac{-2\log\text{t}-1}{\text{t}^3}}=\text{t}
Q658. Find the values of k so that the function f is continuous at the indicated point: 3 Marks
\text{f(x)}= \begin{cases}\text{k}\text{x}+1,\ \text{if}\ \text{x}\leq{\pi}\\ \cos\text{x}, \ \ \ \ \text{if}\ \text{x} >{\pi}\end{cases}
\text{at}\ \text{x} = {\pi}

Ans: Here \text{f(x)}= \begin{cases}\text{k}\text{x}+1,\ \text{if}\ \text{x}\leq{\pi}\\ \cos\text{x}, \ \ \ \ \text{if}\ \text{x} >{\pi}\end{cases}
​^{\ \ \text{Lt}}_{\text{x}\rightarrow{\pi}^{-}}\text{f(x)}= ^{\ \ \text{Lt}}_{\text{x}\rightarrow{\pi}^{-}}(\text{k}\text{x}+1)
\left[\text{Put}\ \text{x} = \pi -\text{h}, \text{h}>0\ \text{so that}\ \text{h}\rightarrow0\ \text{as}\ \text{x}\rightarrow\pi^{-}\right]
= ^{\ \ \text{Lt}}_{\text{x}\rightarrow0}\left\{\text{k}(\pi - \text{h})+1\right\}={\text{k}}(\pi - 0) = 1 = \text{k}\pi + 1
^{\ \ \text{Lt}}_{\text{x}\rightarrow\pi^{+}}\text{f(x)} =^{\ \ \text{Lt}}_{\text{x}\rightarrow\pi^{+}}\cos\pi
\left[\text{Put}\ \text{x} = \pi +\text{h}, \text{h}>0\ \text{so that}\ \text{h}\rightarrow0\ \text{as}\ \text{x}\rightarrow\pi^{+}\right]
^{\ \ \text{Lt}}_{\text{h}\rightarrow0}\cos(\pi + \text{h}) = ^{\ \ \text{Lt}}_{\text{h}\rightarrow0}(\cos\pi\cos\text{h} - \sin\pi\sin\text{h})
\cos \pi. 1 - \sin \pi. 0 = \cos\pi = -1
Since f(x) is continuous at \text{x} = \pi
\therefore\ ^{\ \ \text{Lt}}_{\text{x}\rightarrow{\pi}^{-}}\text {f(x}) =^{\ \ \text{Lt}}_{\text{x}\rightarrow{\pi}^{+}}\text{f(x)}
\therefore \text{k}\pi = -1 \Rightarrow \text{k} = -\frac{1}{\pi}
Q659. Differentiate w.r.t. x the function in Exercise: 3 Marks
(5\text{x})^{3\cos2\text{x}}

Ans: Let \text{y}=(5\text{x})^{3\cos2\text{x}}


Taking logaritthm on both the sides, we obtain
\log\text{y}=3\cos2\text{x}\log5\text{x}
Differentiating both sides with respect to x, we obtain
\frac{1}{\text{y}}\frac{\text{dy}}{\text{dx}}=3\Big[\log5\text{x}.\frac{\text{d}}{\text{dx}}(\cos2\text{x})+\cos2\text{x}.\frac{\text{d}}{\text{dx}}(\log5\text{x})\Big]
\Rightarrow\ \frac{\text{dy}}{\text{dx}}=3\text{x}\Big[\log5\text{x}(-\sin2\text{x}).\frac{\text{d}}{\text{dx}}(2\text{x})+\cos2\text{x}.\frac{1}{5\text{x}}.\frac{\text{d}}{\text{dx}}
(5\text{x})\Big]
\Rightarrow\ \frac{\text{dy}}{\text{dx}}=3\text{y}\Big[-2\sin2\text{x}\log5\text{x}+\frac{\cos2\text{x}}{\text{x}}\Big]
\Rightarrow\ \frac{\text{dy}}{\text{dx}}=3\text{y}\Big[\frac{3\cos2\text{x}}{\text{x}}-6\sin2\text{x}\log5\text{x}\Big]
\Rightarrow\ \frac{\text{dy}}{\text{dx}}=(5\text{x})^{\text{x}\cos2\text{x}}\Big[\frac{3\cos2\text{x}}{\text{x}}-6\sin2\text{x}\log5\text{x}\Big]
Q660. Differentiate following w.r.t. x: 3 Marks
\sin^\text{n}\big(\text{ax}^2+\text{bx}+\text{c}\big)

Ans: Let \text{y}=\sin^\text{n}\big(\text{ax}^2+\text{bx}+\text{c}\big)


\Rightarrow\ \frac{\text{dy}}{\text{dx}}=\frac{\text{d}}{\text{dx}}\Big[\sin\big(\text{ax}^2+\text{bx}+\text{c}\big)\Big]^\text{n}
=\text{n}\cdot\Big[\sin\big(\text{ax}^2+\text{bx}+\text{c}\big)\Big]^{\text{n}-1}\cdot\frac{\text{d}}{\text{dx}}\sin\big(\text{ax}^2+\text{bx}+\text{c}\big)
=\text{n}\cdot\sin^{\text{n}-1}\big(\text{ax}^2+\text{bx}+\text{c}\big)\cdot\cos\big(\text{ax}^2+\text{bx}+\text{c}\big)\cdot\frac{\text{d}}{\text{dx}}\big(\text{ax}^2+\text{bx}+\text{c}\big)
=\text{n}\cdot\sin^{\text{n}-1}\big(\text{ax}^2+\text{bx}+\text{c}\big)\cdot\cos\big(\text{ax}^2+\text{bx}+\text{c}\big)\cdot(2\text{ax + b})
=\text{n}\cdot(2\text{ax + b})\cdot\sin^{\text{n}-1}\big(\text{ax}^2+\text{bx}+\text{c}\big)\cdot\cos\big(\text{ax}^2+\text{bx}+\text{c}\big)
Q661. Determine the value of the constant k so that the function 3 Marks
\text{f}\text{(x)}=\begin{cases}\frac{\sin2\text{x}}{5\text{x}}, &\text{if}\text{ x}\neq0\\\text{k}, &\text{if}\text{ x}=0\end{cases} is continuous at x = 0.

Ans: We have given that the funtion is continuous at x = 0


So, LHL = RHL = f(0) ....(i)
Now,
\text{LHL}=\lim\limits_{\text{x} \rightarrow 0^-}\text{f}\text{(x)}=\lim\limits_{\text{h} \rightarrow 0}\text{f}(0-\text{h)}=\lim\limits_{\text{h} \rightarrow 0}\frac{\sin2(-\text{h})}{5(-
\text{h})}
=\lim\limits_{\text{h} \rightarrow 0}\frac{-\sin2\text{h}}{-5\text{h}}=\lim\limits_{\text{h} \rightarrow 0}\frac{\sin2\text{h}}{2\text{h}}\times\frac{2\text{h}}{5\text{h}}=\frac{2}{5}
\text{f}(0)=\text{k}
Using(i), \text{k}=\frac{2}{5}
Q662. If \text{f}\text{(x)}=\begin{cases}\frac{\sin3\text{x}}{\text{x}},& \text{when}\text{ x}\neq0 \\1,&\text{when} \text{ x}=0\end{cases} Find whether f(x) is continuous at x = 0. 3 Marks

Ans: Given,
\text{f}\text{(x)}=\frac{\text{x}^2-1}{\text{x}-1},\text{ if}\text{ x}\neq1
\text{f}\text{(x)}=2,\text{ if}\text{ x}=1
We observe
\text{(LHL at x = 1)}
\lim\limits_{\text{x} \rightarrow 1^-}\text{f}\text{ (x)}=\lim\limits_{\text{x} \rightarrow 0}(1-\text{h})

https://bls.smartstudies.co.in/#/exam/pdf-preview/c59cb220-8e86-4716-9ff7-82aec16b1ade/1 132/158
5/26/24, 6:20 PM Exam Automation
\lim\limits_{\text{x} \rightarrow 0}(1-\text{h})=\lim\limits_{\text{x} \rightarrow 0}\frac{(1-\text{h})^2-1}{(1-\text{h})^2-1}
\lim\limits_{\text{x} \rightarrow 0}\frac{1-\text{h}^2-2\text{h}-1}{1-\text{h}-1}
\lim\limits_{\text{x} \rightarrow 0}\frac{\text{h}^2-2\text{h}}{-\text{h}}
\lim\limits_{\text{x} \rightarrow 0}2-\text{h}
=2
(\text{RHL at x}=1)
\lim\limits_{\text{x} \rightarrow 1^+}\text{f}\text{(x)}=\lim\limits_{\text{h} \rightarrow 0}(1+\text{h)}
\lim\limits_{\text{h} \rightarrow 0}(1-\text{h)}=\lim\limits_{\text{h} \rightarrow 0}\frac{(1+\text{h})^2-1}{(1+\text{h})-1}
\lim\limits_{\text{h} \rightarrow 0}\frac{1+\text{h}^2+2\text{h}-1}{1+\text{h}-1}
\lim\limits_{\text{h} \rightarrow 0}\frac{\text{h}^2+2\text{h}}{\text{h}}
\lim\limits_{\text{h} \rightarrow 0}\text{h}+2
=2
Also f(x) = 2
\lim\limits_{\text{x} \rightarrow 1^-}\text{f}\text{(x)}=\lim\limits_{\text{x} \rightarrow 1^+}\text{f}\text{(x)}=\text{f}(1)
Hence f(x) is continuous at x = 1.
Q663. If \text{y}=\sec^{-1}\Big(\frac{\text{x}+1}{\text{x}-1}\Big)+\sin^{-1}\Big(\frac{\text{x}-1}{\text{x}+1}\Big),\text{x}>0. Find \frac{\text{dy}}{\text{dx}}. 3 Marks

Ans: Here, \text{y}=\sec^{-1}\Big(\frac{\text{x}+1}{\text{x}-1}\Big)+\sin^{-1}\Big(\frac{\text{x}-1}{\text{x}+1}\Big)


\text{y}=\cos^{-1}\Big(\frac{\text{x}+1}{\text{x}-1}\Big)+\sin^{-1}\Big(\frac{\text{x}-1}{\text{x}+1}\Big)
\Big[\text{Since, } \sec^{-1}(\text{x})=\cos^{-1}\Big(\frac{1}{\text{x}}\Big)\Big]
\text{y}=\frac{\pi}{2} \Big[\text{Since}, \cos^{-1}\text{x}+\sin^{-1}=\frac{\pi}{2}\Big]
Differentiating with respect to x,
\frac{\text{dy}}{\text{dx}}=0
Q664. Find the points of discontinuity, if any of the following function: 3 Marks
\text{f(x)}=\begin{cases}\frac{\text{e}^{\text{x}}}{\log_\text{e}(1+2\text{x})},&\text{if }\text{ x}\neq0\\7,&\text{if }\text{ x}=0\end{cases}

Ans: Given, \text{f(x)}=\begin{cases}\frac{\text{e}^{\text{x}}}{\log_\text{e}(1+2\text{x})},&\text{if }\text{ x}\neq0\\7,&\text{if }\text{ x}=0\end{cases}


We have,
\lim_\limits{\text{x}\rightarrow0}\text{f(x)}=\lim_\limits{\text{x}\rightarrow0}\frac{\text{e}^{\text{x}}}{\log_\text{e}
(1+2\text{x})}=\lim_\limits{\text{x}\rightarrow0}\frac{\big(\frac{\text{e}^{\text{x}}-1}{\text{x}}\Big)}{\Big(\frac{2\log_\text{e}(1+2\text{x})}{2\text{x}}\Big)}
=\frac{1}{2}\times\frac{\big(\frac{\text{e}^{\text{x}}-1}{\text{x}}\Big)}{\Big(\frac{2\log_\text{e}(1+2\text{x})}{2\text{x}}\Big)}=\frac{1}{2}
It is given that f(0) = 7
\lim_\limits{\text{x}\rightarrow0}\text{f(x)}\neq\text{f}(0)
Hence, the given function is discontinuous at x = 0 and continuous elsewhere.
Q665. Find \frac{\text{dy}}{\text{dx}} when x and y are connected by the relation: 3 Marks
\sec(\text{x}+\text{y})=\text{xy}

Ans: Consider, \sec(\text{x}+\text{y})=\text{xy}


\Rightarrow\ \sec(\text{x}+\text{y})\cdot\tan(\text{x}+\text{y})\cdot\frac{\text{d}}{\text{dx}}(\text{x}+\text{y})=\text{x}\cdot\frac{\text{d}}{\text{dx}}\text{y}+\text{y}\cdot\frac{\text{d}}
{\text{dx}}\text{x}
\Rightarrow\ \sec(\text{x}+\text{y})\cdot\tan(\text{x}+\text{y})\cdot\Big(1+\frac{\text{dy}}{\text{dx}}\Big)=\text{x}\frac{\text{dy}}{\text{dx}}+\text{y}
\Rightarrow\ \sec(\text{x}+\text{y})\tan(\text{x}+\text{y})+\sec(\text{x}+\text{y})\cdot\tan(\text{x}+\text{y})\cdot\frac{\text{dy}}{\text{dx}}=\text{x}\frac{\text{dy}}{\text{dx}}+\text{y}
\Rightarrow\ \sec(\text{x}+\text{y})\cdot\tan(\text{x}+\text{y})\cdot\frac{\text{dy}}{\text{dx}}-\text{x}\frac{\text{dy}}{\text{dx}}=\text{y}-\sec(\text{x}+\text{y})\tan(\text{x}+\text{y})
\Rightarrow\ \frac{\text{dy}}{\text{dx}}\big[\sec(\text{x}+\text{y})\cdot\tan(\text{x}+\text{y})-\text{x}\big]=\text{y}-\sec(\text{x}+\text{y})\cdot\tan(\text{x}+\text{y})
\therefore\ \frac{\text{dy}}{\text{dx}}=\frac{\text{y}-\sec(\text{x}+\text{y})\cdot\tan(\text{x}+\text{y})}{\sec(\text{x}+\text{y})\cdot\tan(\text{x}+\text{y})-\text{x}}
Q666. Discuss the continuity of \text{f}\text{(x)}=\begin{cases}2\text{x}-1, & \text{x} < 0\\2\text{x}+1, & \text{x} \geq 0\end{cases}\text{at}\text{ x}=0 3 Marks

Ans: \text{f}\text{(x)}=\begin{cases}2\text{x}-1, & \text{x} < 0\\2\text{x}+1, & \text{x} \geq 0\end{cases}\text{at}\text{ x}=0
\text{(LHL at x}=0)=\lim\limits_{\text{x} \rightarrow 0^-}\text{f}\text{(x)}=2(0)-1=-1
\text{(RHL at x}=0)=\lim\limits_{\text{x} \rightarrow 0^+}\text{f}\text{(x)}=2(0)+1=1
\Rightarrow\lim\limits_{\text{x} \rightarrow 0^-}\text{f}\text{(x)}\neq\lim\limits_{\text{x} \rightarrow 0^+}\text{f}\text{(x)}
Hence, f(x) is discontinuous at x = 0.
Q667. Find the values of a and b such that the function defined by 3 Marks
\text{f(x)}=\begin{cases}5,&\text{if}\ \text{x}\leq{2}\\\text{ax} + \text{b},& \text{if}\ 2<\text{x}<10\\21,&\text{if}\ \text{x}\geq10\end{cases}
is a continuous function.

Ans: \text{f(x)}=\begin{cases}5,&\text{if}\ \text{x}\leq{2}\\\text{ax} + \text{b},& \text{if}\ 2<\text{x}<10\\21,&\text{if}\ \text{x}\geq10\end{cases}


\therefore f is continuous function
\therefore f is continuous at x = 2 and x = 10
\therefore f is right continuous at x = 2 and left continuous at x = 10.
\therefore\ ^{\ \ \text{Lt}}_{\text{x}\rightarrow\text{2}^{+}}\text{f(x)} = \text{f}(2)\Rightarrow 2\text{a} + \text{b} = 5\ ...{(\text i)}
^{\ \ \text{Lt}}_{\text{x}\rightarrow\text{10}^{-}}\text{f(x)} = \text{f}(10)\Rightarrow 10\text{a} + \text{b} = 21\ ...{(\text {ii})}
Subtracting (1) from(2), we get,
8a = 16 or a = 2
\therefore from (1), 4 + 6 = 5 ⇒ b = 1
\therefore we have a = 2, b = 1
Q668. Differentiate the function given in Exercise: 3 Marks
\cos\text{x}.\cos 2\text{x}.\cos 3\text{x}

Ans: Let \text{y}=\cos\text{x}\cos2\text{x}\cos3\text{x}\ \dots\text{(i})


Taking logs on both sides, we have
\log\text{y}=\log(\cos\text{x}\cos2\text{x}\cos3\text{x}) =\log\cos\text{x}+\log\cos2\text{x}+\log\cos3\text{x}
\therefore\ \frac{\text{d}}{\text{dx}}\log\text{y}=\frac{\text{d}}{\text{dx}}\log\cos\text{x}+\frac{\text{d}}{\text{dx}}\log\cos2\text{x}+\frac{\text{d}}{\text{dx}}\log\cos3\text{x}
\Rightarrow\ \frac{1}{\text{y}}\frac{\text{d}}{\text{dx}}=\frac{1}{\cos\text{x}}\frac{\text{d}}{\text{dx}}\cos\text{x}+\frac{1}{\cos2\text{x}}\frac{\text{d}}{\text{dx}}\cos2\text{x}+\frac{1}
{\cos3\text{x}}\frac{\text{d}}{\text{dx}}\cos3\text{x}
\Rightarrow\ \frac{1}{\text{y}}\frac{\text{d}}{\text{dx}}=\frac{1}{\cos\text{x}}(-\sin\text{x})+\frac{1}{\cos2\text{x}}(-\sin2\text{x})\frac{\text{d}}{\text{dx}}2\text{x}+\frac{1}{\cos3\text{x}}(-
\sin3\text{x})\frac{\text{d}}{\text{dx}}3\text{x}
\Rightarrow\ \frac{1}{\text{y}}\frac{\text{dy}}{\text{dx}}=-\tan\text{x}-(\tan2\text{x})2-\tan3\text{x}(3)
\Rightarrow\ \frac{\text{d}}{\text{dx}}=-\text{y}(\tan\text{x}+2\tan2\text{x}+3\tan3\text{x})
\Rightarrow\ \frac{\text{d}}{\text{dx}}=-\cos\text{x}\cos2\text{x}\cos3\text{x}(\tan\text{x}+2\tan2\text{x}+3\tan3\text{x})\ \ [\text{From eq. (i)}]
Q669. Write the derivative of \sin\text{x} with respect to \cos\text{x}. 3 Marks

Ans: Let \text{u}=\sin\text{x}\text{ and v}=\cos\text{x}


\Rightarrow\frac{\text{du}}{\text{dx}}=\cos\text{x and }\frac{\text{dv}}{\text{dx}}=-\sin\text{x}
\therefore\frac{\frac{\text{du}}{\text{dx}}}{\frac{\text{dv}}{\text{dx}}}=\frac{\cos\text{x}}{-\sin\text{x}}
\Rightarrow\frac{\text{du}}{\text{dv}}=-\cot\text{x}
Q670. Find the second order derivatives of the following functions: 3 Marks
\text{x}^3+\tan\text{x}

Ans: We have

https://bls.smartstudies.co.in/#/exam/pdf-preview/c59cb220-8e86-4716-9ff7-82aec16b1ade/1 133/158
5/26/24, 6:20 PM Exam Automation
\text{y}=\text{x}^3+\tan\text{x}
Differentiating w.r.t.x, we get
\frac{\text{dy}}{\text{x}}=3\text{x}^2+\sec^2\text{x}
Differentiating again w.r.t.x, we get
\frac{\text{d}^2\text{y}}{\text{dx}^2}=6\text{x}+2\sec^2\text{x}\tan\text{x}
Q671. If \text{x}=3\sin\text{t}-\sin3\text{t},\text{y}=3\cos3\text{t}-\cos3\text{t} find \frac{\text{dy}}{\text{dx}}\text{ at t}=\frac{\pi}{3} 3 Marks

Ans: \text{x}=3\sin\text{t}-\sin3\text{t and } \text{y}=3\cos3\text{t}-\cos3\text{t}


\Rightarrow\frac{\text{dx}}{\text{dt}}=3\cos\text{t}-3\cos3\text{t}\text{ and} \\ \frac{\text{dy}}{\text{dt}}=-3\sin\text{t}-3\sin3\text{t}
\therefore\frac{\text{dy}}{\text{dx}}=\frac{\frac{\text{dy}}{\text{dt}}}{\frac{\text{dx}}{\text{dt}}}=\frac{-3\sin\text{t}+3\sin3\text{t}}{3\cos\text{t}-3\cos3\text{t}}
\Rightarrow\Big(\frac{\text{dy}}{\text{dx}}\Big)_{\text{t}=\frac{\pi}{3}}=\frac{-3\sin\frac{\pi}{3}+3\sin\pi}{3\cos\frac{\pi}{3}-3\cos\pi}
=\frac{3\times\frac{\sqrt{3}}{2}+0}{3\times\frac{1}{2}+3}
=\frac{\frac{-3\sqrt{3}}{2}}{\frac{9}{2}}
=-\frac{1}{\sqrt{3}}
Q672. Find \frac{\text{dy}}{\text{dx}} in the following cases: 3 Marks
xy = c2

Ans: We have, xy = c2
Differentiating with respect to x, we get
\frac{\text{d}}{\text{dx}}(\text{xy})=\frac{\text{d}}{\text{dx}}(\text{c}^2)
\Rightarrow\text{x}\frac{\text{dy}}{\text{dx}}+\text{y}\frac{\text{d}}{\text{dx}}(\text{x})=0
[Using product rule]
\Rightarrow\text{x}\frac{\text{dy}}{\text{dx}}+\text{y}=0
\Rightarrow\text{x}\frac{\text{dy}}{\text{dx}}=-\text{y}
\Rightarrow\frac{\text{dy}}{\text{dx}}=-\frac{\text{y}}{\text{x}}
Q673. Differentiate the functions with respect to x. 3 Marks
\cos\text{x}^{3}. \sin^{2}(\text{x}^{5})

Ans: \text{Let y} =\cos\text{x}^{3}. \sin^{2}(\text{x}^{5})


\therefore \frac{\text{dy}}{\text{dx}} =\cos\text{x}^{3} \frac{\text{d}}{\text{dx}}\sin^{2}(\text{x}^{5})+\sin^{2}(\text{x}^{5})\frac{\text{d}}{\text{dx}}\cos\text{x}^{3}
​=\cos\text{x}^{3}.2\sin(\text{x}^{5}) \frac{\text{d}}{\text{dx}}\sin(\text{x}^{5})+\sin^{2}(\text{x}^{5})(-\sin\text{x}^{3})\frac{\text{d}}{\text{dx}}\text{x}^{3}
​=\cos\text{x}^{3}.2\sin\text{x}^{5} .\cos\text{x}^{5}\frac{\text{d}}{\text{dx}}\text{x}^5+\sin^{2}(\text{x}^{5})(-\sin\text{x}^{3})3\text{x}^{2}
​=\cos\text{x}^{3}.2\sin(\text{x}^{5}) \cos(\text{x}^{5})(5\text{x}^{4})-\sin^{2}(\text{x}^{5})\sin\text{x}^{3}.3\text{x}^{2}
​=10\text{x}^{4}\cos\text{x}^{3} \sin(\text{x}^{5})\cos(\text{x}^{5})-3\text{x}^{2}\sin^{2}(\text{x}^{5})\sin\text{x}^{3}
Q674. Differentiate the following functions with respect to x: 3 Marks
\log(\cos\text{x}^2)

Ans: Consider \text{y}=\log(\cos\text{x}^2)


Differentiate it with respect to x and applying the chain and product rule, we get
\frac{\text{dy}}{\text{dx}}=\frac{\text{d}}{\text{dx}}\log(\cos\text{x}^2)
=\frac{-2\text{x}\sin\text{x}^2}{\cos\text{x}^2}
\frac{\text{dy}}{\text{dx}}=-2\text{x}\tan\text{x}^2
Therefore,
\frac{\text{dy}}{\text{dx}}=-2\text{x}\tan\text{x}^2
Q675. Discuss the continuity of the function \text{f(x)}=\begin{cases}\frac{\text{x}}{|\text{x}|},&\text{x}\neq0\\0,&\text{x}=0\end{cases} 3 Marks

Ans: When \text{x}\neq0,


\text{f(x)}=\frac{\text{x}}{|\text{x}|}=\begin{cases}\frac{-\text{x}}{\text{x}}=-1;&\text{x}<0\\\frac{\text{x}}{|\text{x}|}=1;&\text{x}>0\end{cases}
So, f(x) is a constant function when \text{x}\neq0,
Hence, is continuous for all x < 0 and x > 0
Now, Consider the point x = 0
\text{LHL}=\lim_\limits{\text{x}\rightarrow0^-}\text{f(x)}=\lim_\limits{\text{h}\rightarrow0}\text{f}(0-\text{h})=\lim_\limits{\text{h}\rightarrow0}\frac{-\text{h}}{|-\text{h}|}=-1
\text{RHL }=\lim_\limits{\text{x}\rightarrow0^+}\text{f(x)}=\lim_\limits{\text{h}\rightarrow0}\text{f}(0+\text{h})=\lim_\limits{\text{h}\rightarrow0}\frac{\text{h}}{|\text{h}|}=1
So, \text{LHL}\neq\text{RHL}
Hence, function is discontinuous at x = 0
Q676. Differentiate of the following w.r.t. x: 3 Marks
\sin^\text{m}\text{x}\cdot\cos^\text{n}\text{x}

Ans: Let \text{y}=\sin^\text{m}\text{x}\cdot\cos^\text{n}\text{x}


\Rightarrow\ \frac{\text{dy}}{\text{dx}}=\frac{\text{d}}{\text{dx}}\big[(\sin\text{x})^\text{m}\cdot(\cos\text{x})^\text{n}\big]
=(\sin\text{x})^\text{m}\cdot\frac{\text{d}}{\text{dx}}(\cos\text{x})^\text{n}+(\cos\text{x})^\text{n}\cdot\frac{\text{d}}{\text{dx}}(\sin\text{x})^\text{m}
=(\sin\text{x})^\text{m}\cdot\text{n}(\cos\text{x})^{\text{n}-1}\cdot\frac{\text{d}}{\text{dx}}\cos\text{x}+(\cos\text{x})^\text{n}\text{m}(\sin\text{x})^{\text{m}-1}\cdot\frac{\text{d}}
{\text{dx}}\sin\text{x}
=(\sin\text{x})^\text{m}\cdot\text{n}(\cos\text{x})^{\text{n}-1}(-\sin\text{x})+(\cos\text{x})^\text{n}\cdot\text{m}(\sin\text{x})^{\text{m}-1}\cos\text{x}
=-\text{n}\cdot\sin^\text{m}\text{x}\cdot\sin\text{x}\cdot\cos^\text{n}\text{x}\cdot\frac{1}{\cos\text{x}}+\text{m}\cdot\sin^\text{m}\text{x}\cdot\frac{1}
{\sin\text{x}}\cdot\cos^\text{n}\text{x}\cdot\cos\text{x}
=-\text{n}\sin^\text{m}\text{x}\cdot\cos^\text{n}\text{x}\cdot\tan\text{x}+\text{m}\sin^\text{m}\text{x}\cdot\cos^\text{n}\text{x}\cdot\cot\text{x}
=\sin^\text{m}\text{x}\cdot\cos^\text{n}\text{x}\big[\text{m}\cot\text{x}-\text{n}\tan\text{x}\big]
Q677. If \log\text{y}=\tan^{-1} show that (1+\text{x}^2)\text{y}_2+(2\text{x}-1)\text{y}_1=0 3 Marks

Ans: Here
\log\text{y}=\tan^{-1}
Differentiating w.r.t.x, we get
\frac{1}{\text{y}}\times\text{y}_1=\frac{1}{1+\text{x}^2}
\Rightarrow(1+\text{x}^2)\text{y}_1=\text{y}
\Rightarrow(1+\text{x}^2)\text{y}_2+2\text{xy}_1=\text{y}_1
\Rightarrow(1+\text{x}^2)\text{y}_2+2\text{xy}_1-\text{y}_1=0
\Rightarrow(1+\text{x}^2)\text{y}_2+(25\text{x}-1)\text{y}_1=0
hence proved
Q678. Discuss the applicability of the Rolle's theorem for the following function on the indicated interval 3 Marks
\text{f}(\text{x})=[\text{x}]\text{ for }-1\leq\text{x}\leq1, where [x] denotes the greatest integer not exceeding x.

Ans: Here, \text{f}(\text{x})=[\text{x}] and \text{x}\in[-1,1], at n = 1


\text{LHL}=\lim\limits_{\text{x}\rightarrow(1-\text{h})}[\text{x}]
=\lim\limits_{\text{h}\rightarrow0}[1-\text{h}]
=0
\text{RHL}=\lim\limits_{\text{x}\rightarrow(1+\text{h})}[\text{x}]
=\lim\limits_{\text{h}\rightarrow0}[1+\text{h}]
=1
\text{LHL}\neq\text{RHL}

https://bls.smartstudies.co.in/#/exam/pdf-preview/c59cb220-8e86-4716-9ff7-82aec16b1ade/1 134/158
5/26/24, 6:20 PM Exam Automation
So, f(x) is not continuos at 1\in[-1,1]
Hence, Rolle's theorem is not applicable on f(x) in [-1, 1].
Q679. Find the second order derivatives of the following functions: 3 Marks
\sin(\log\text{x})

Ans: Let \text{y}=\sin(\log\text{x})


Then
\frac{\text{dy}}{\text{dx}}=\frac{\text{d}}{\text{dx}}[\sin(\log\text{x})]=\cos(\log\text{x}).\frac{\text{d}}{\text{dx}}(\log\text{x})=\frac{\cos(\log\text{x})}{\text{x}}
\therefore\frac{\text{d}^2\text{y}}{\text{dx}^2}=\frac{\text{d}}{\text{dx}}\Big[\frac{\cos(\log\text{x})}{\text{x}}\Big]
=\frac{\text{x}.\frac{\text{d}}{\text{dx}}[\cos(\log\text{x})]-\cos(\log\text{x}).\frac{\text{d}}{\text{dx}}(\text{x})}{\text{x}^2}
=\frac{\text{x}.\Big[-\sin(\log\text{x}).\frac{\text{d}}{\text{dx}(\log\text{x})}\Big]-\cos(\log\text{x}.1}{\text{x}^2}
\frac{-\text{x}\sin(\log\text{x}).\frac{1}{\text{x}}-\cos(\log\text{x})}{\text{x}^2}
=\frac{[-\sin(\log\text{x})+\cos(\log\text{x})]}{\text{x}^2}
Q680. Verify Mean Value Theorem, if f(x) = x3 – 5x2 – 3x in the interval [a, b], where a = 1 and b = 3. Find all \text{c}\in(1,\ 3) for which f′(c) = 0. 3 Marks

Ans: Function is continuous in [1, 3] as it is a polynomial function and polynomial function is always continuous.
f'(x) = 3x2 -10x, f'(x) exists in [1, 3], hence derivable. Conditions of MVT theorem are satisfied, hence there exists, at least one \text{c}\in(1,\ 3) such that
\frac{\text{f}(3)-\text{f}(1)}{3-1}=\text{f}'\text{(c)}\ \Rightarrow\ \frac{-21-(-7)}{2}=3\text{c}^2-10\text{c}
\Rightarrow\ -7 = 3\text{c}^2 - 10\text{c} \ \Rightarrow\ 3\text{c}^2 - 10\text{c} + 7 = 0
\Rightarrow\ 3\text{c}^2-7\text{c}-3\text{c}+7=0 \ \Rightarrow\ \text{c}(3\text{c} -7) -1(3\text{c}-7)=0
\Rightarrow\ (3\text{c}-7)(\text{c} -1)=0 \ \Rightarrow\ (3\text{c}-7)=0\text{ or } (\text{c}-1)=0
\Rightarrow\ 3\text{c}=7 \text{ or}\text{ c}=1 \ \Rightarrow\ \text{c}=\frac{7}{3}\text{or c}=1
\therefore\ \text{c}=\frac{7}{3}\in(1, 3) and other value \in(1,\ 3)
Since \text{f}(1)\neq\text{f}(3), therefore the value of 'c' does not exist such that f(c) = 0.
Q681. If \text{y}=\text{e}^{\text{a}\cos^{-1}}\text{x} prove that (1-\text{x}^2)\frac{\text{d}^2\text{y}}{\text{dx}^2}-\text{x}\frac{\text{dy}}{\text{dx}}-\text{a}^2\text{y}=0 3 Marks

Ans: Here,
\text{y}=\text{e}^{\text{a}\cos^{-1}}\text{x}
Differentiating w.r.t.x, we get
\frac{\text{dy}}{\text{dx}}=-\text{e}^{\text{a}\cos^{-1}}\text{x}\ \times\frac{\text{a}}{\sqrt{1-\text{x}^2}}
Differentiating w.r.t.x, we get
\Rightarrow\frac{\text{d}^2\text{y}}{\text{dx}^2}=\text{e}^{\text{a}\cos^{-1}}\text{x}\ \times\frac{\text{a}^2}{1-\text{x}^2}+\frac{\text{xa }\text{e}^{\text{a}\cos^{-1}}\text{x}}{(1-
\text{x}^2)\sqrt{1-\text{x}^2}}
\Rightarrow\frac{\text{d}^2\text{y}}{\text{dx}^2}=\text{y}\times\frac{\text{a}^2}{1-\text{x}^2}-\frac{\text{x}\frac{\text{dy}}{\text{dx}}}{(1-\text{x}^2)}
\Rightarrow(1-\text{x}^2)\frac{\text{d}^2\text{y}}{\text{dx}^2}=\text{a}^2\text{y}-\text{x}\frac{\text{dy}}{\text{dx}}
\Rightarrow(1-\text{x}^2)\frac{\text{d}^2\text{y}}{\text{dx}^2}+\text{x}\frac{\text{dy}}{\text{dx}}-\text{a}^2\text{y}=0
Q682. If \text{y}=\cos^{-1}(2\text{x})+2\cos^{-1}\sqrt{1-4\text{x}^2}, 0 <\text{x}<\frac{1}{2}, find \frac{\text{dy}}{\text{dx}}. 3 Marks

Ans: Here, \text{y}=\cos^{-1}(2\text{x})+2\cos^{-1}\sqrt{1-4\text{x}^2}


Put 2\text{x}=\cos\theta
\therefore\ \text{y}=\cos^{-1}(\cos\theta)+2\cos^{-1}\sqrt{1-\cos^2\theta}
\Rightarrow \text{y}=\cos^{-1}(\cos\theta)+2\cos^{-1}(\sin\theta)
\therefore\ \text{y}=\cos^{-1}(\cos\theta)+2\cos^{-1}\Big[\cos\big(\frac{\pi}{2}\big)-\theta\Big]\ .....(\text{i})
Here, 0<\text{x}<\frac{1}{2}
\Rightarrow 0<2\text{x}<1
\Rightarrow 0<\cos\theta<1
\Rightarrow 0<\theta<\frac{\pi}{2}
And
\Rightarrow 0> -\theta>-\frac{\pi}{2}
\Rightarrow\ \frac{\pi}{2}>\big(\frac{\pi}{2}-\theta\big)>0
Q683. If \text{y}=\sqrt{\tan\text{x}+\sqrt{\tan\text{x}+\sqrt{\tan\text{x}+\ .... \text{to }\infty}}} prove that \frac{\text{dy}}{\text{dx}}=\frac{\sec^2\text{x}}{2\text{y}-1} 3 Marks

Ans: We have, \text{y}=\sqrt{\tan\text{x}+\sqrt{\tan\text{x}+\sqrt{\tan\text{x}+\ .... \text{to }\infty}}}


\Rightarrow\text{y}=\sqrt{\tan\text{x}+\text{y}}
Squaring both sides, we get,
\text{y}^2=\tan\text{x}+\text{y}
\Rightarrow2\text{y}\frac{\text{dy}}{\text{dx}}=\sec^2\text{x}+\frac{\text{dy}}{\text{dx}}
\Rightarrow\frac{\text{dy}}{\text{dx}}(2\text{y}-1)=\sec^2\text{x}
\Rightarrow\frac{\text{dy}}{\text{dx}}=\frac{\sec^2\text{x}}{2\text{y}-1}
Q684. Differentiate the functions given in Exercise: 3 Marks
(\text{x}+3)^2.(\text{x}+4)^3.(\text{x}+5)^4

Ans: Let \text{y}=(\text{x}+3)^2.(\text{x}+4)^3.(\text{x}+5)^4\ \dots\text{(i)}


Taking logs on both sides, we have
\log\text{y}=2\log(\text{x}+3)+3\log(\text{x}+4)+4\log(\text{x}+5)^4
\therefore\ \frac{\text{d}}{\text{dx}}\log\text{y}=2\frac{\text{d}}{\text{dx}}\log(\text{x}+3)+3\frac{\text{d}}{\text{dx}}\log(\text{x}+4)+4\frac{\text{d}}{\text{dx}}\log(\text{x}+5)
\Rightarrow\ \frac{1}{\text{y}}\frac{\text{dy}}{\text{dx}}=2\frac{1}{\text{x}+3}\frac{\text{d}}{\text{dx}}(\text{x}+3)+3\frac{1}{\text{x}+4}\frac{\text{d}}{\text{dx}}(\text{x}+4)+4\frac{1}
{\text{x}+5}\frac{\text{d}}{\text{dx}}(\text{x}+5)
\Rightarrow\ \frac{1}{\text{y}}\frac{\text{dy}}{\text{dx}}=\frac{2}{\text{x}+3}+\frac{3}{\text{x}+4}+\frac{4}{\text{x}+5}
\Rightarrow\ \frac{\text{dy}}{\text{dx}}=\text{y}\Big(\frac{2}{\text{x}+3}+\frac{3}{\text{x}+4}+\frac{4}{\text{x}+5}\Big)
\Rightarrow\ \frac{\text{dy}}{\text{dx}}=(\text{x}+3)^2(\text{x}+4)^3(\text{x}+5)^4\Big(\frac{2}{\text{x}+3}+\frac{3}{\text{x}+4}+\frac{4}{\text{x}+5}\Big)\ \text{[From eq.(i)]}
Q685. Verify Lagrange's mean value theorem for the following function on the indicated intervals. find a point 'c' in the indicated interval as stated by the Lagrange's mean value theorem. 3 Marks
f(x) = x2 + x - 1 on [0, 4]

Ans: Here,
f(x) = x2 + x - 1 on [0, 4]
f(x) is polynomial, so it is continuous is [0, 4] and differentiable in (0, 4)
as every polynomial is continuous and differentiable everywhere. So, Lagrange's mean value theorem is applicable, so there exist a point \text{c}\in[0,4] such that
\text{f}'(\text{c})=\frac{\text{f}(4)-\text{f}(0)}{4-0}
\Rightarrow2\text{c}+1=\frac{\big((4)^2+4-1\big)-(0-1)}{4}
\Rightarrow2\text{c}+1=\frac{19+1}{4}
\Rightarrow2\text{c}+1=5
\Rightarrow\text{c}=2\in(0,4)
Hence, Lagrange's mean value theorem is verified.
Q686. \text{If y}=\text{Ae}^{\text{mx}}+\text{Be}^{\text{nx}},\text{ show that }\frac{\text{d}^2\text{y}}{\text{dx}^2}-(\text{m}+\text{n})\frac{\text{dy}}{\text{dx}}+\text{mny}=0 3 Marks

Ans: \text{y}=\text{Ae}^\text{mx}+\text{Be}^{\text{nx}}\dots(1)
\therefore\ \frac{\text{dy}}{\text{dx}}=\text{Ame}^{\text{mx}}+\text{Bne}^{\text{nx}}\dots(2)
and \frac{\text{d}^2\text{y}}{\text{dx}^2}=\text{Am}^2\text{e}^{\text{mx}}+\text{Bn}^2\text{e}^{\text{nx}}\dots(3)
\text{L.H.S.}=\frac{\text{d}^2\text{y}}{\text{dx}^2}-(\text{m}+\text{n})\frac{\text{dy}}{\text{dx}}+\text{mny}

https://bls.smartstudies.co.in/#/exam/pdf-preview/c59cb220-8e86-4716-9ff7-82aec16b1ade/1 135/158
5/26/24, 6:20 PM Exam Automation
=(\text{Am}^2\text{e}^{\text{mx}}+\text{Bn}^2\text{e}^{\text{nx}})-(\text{m}+\text{n})(\text{Ame}^{\text{mx}}+\text{Bne}^{\text{nx}})+\text{mn}
(\text{Ae}^{\text{mx}}+\text{Be}^{\text{nx}})
[\because \text{of }(1),(2),(3)]
=\text{Am}^2\text{e}^{\text{mx}}+\text{Bn}^2\text{e}^{\text{nx}}-\text{Am}^2\text{e}^{\text{mx}}-\text{Bmne}^{\text{nx}}-\text{Amne}^{\text{mx}}
-\text{Bn}^2\text{e}^{\text{nx}}+\text{Amne}^{\text{mx}}+\text{Bmne}^{\text{nx}}
=0
=\text{R.H.S.}
Q687. Differentiate the following functions with respect to x: 3 Marks
\text{e}^{\tan3\text{x}}

Ans: Let, \text{y}=\text{e}^{\tan3\text{x}}


Differentiate with respect to x,
\frac{\text{dy}}{\text{dx}}=\frac{\text{d}}{\text{dx}}\big(\text{e}^{\tan3\text{x}}\big)
=\text{e}^{\tan3\text{x}}\frac{\text{d}}{\text{dx}}(\tan3\text{x})
[Using chain rule]
\text{e}^{\tan3\text{x}}\times\sec^23\text{x}\times\frac{\text{d}}{\text{dx}}(3\text{x})
So,
\frac{\text{d}}{\text{dx}}\big(\text{e}^{\tan3\text{x}}\big)=3\text{e}^{\tan3\text{x}}\times\sec^2 3\text{x}
Q688. If \text{y}=\log(\sin\text{x}) Prove that \frac{\text{d}^3\text{y}}{\text{dx}^3}=2\cos\text{x}\ \text{cosec}^3\text{x} 3 Marks

Ans: Here,
\text{y}=\log(\sin\text{x})
Differentiating w.r.t.x, we get
\frac{\text{dy}}{\text{dx}}=\frac{1}{\sin\text{x}}\times\cos\text{x}=\cot\text{x}
Differentiating w.r.t.x, we get
\frac{\text{d}^2\text{y}}{\text{dx}^2}=-\text{cosec}^2\text{x}
Differentiating w.r.t.x, we get
\frac{\text{d}^3\text{y}}{\text{dx}^3}=-2\text{cosec}\ \text{x}\times(-\text{cosec}\ \text{x}\cot\text{x})
=2\cot\ \text{x}\ \text{cosec}^2\text{x}=2\cos\ \text{x}\ \text{cosec}^3\text{x}
Q689. Differentiate the following w.r.t. x: 3 Marks
\log\big[\log(\log\text{x}^5)\big]

Ans: Let \text{y}=\log\big[\log(\log\text{x}^5)\big]


\therefore\ \frac{\text{dy}}{\text{dx}}=\frac{\text{d}}{\text{dx}}\log\big[\log(\log\text{x}^5)\big]
=\frac{1}{\log\log\text{x}^5}\cdot\frac{\text{d}}{\text{dx}}\big(\log\cdot\log\text{x}^5\big)
=\frac{1}{\log\log\text{x}^5}\cdot\frac{1}{\log\text{x}^5}\cdot\frac{\text{d}}{\text{dx}}\log\text{x}^5
=\frac{1}{\log\log\text{x}^5}\cdot\frac{1}{\log\text{x}^5}\cdot\frac{\text{d}}{\text{dx}}(5\log\text{x})
=\frac{5}{\text{x}\cdot\log(\log\text{x}^5)\cdot\log(\text{x}^5)}
Q690. \text{If y}=5\cos\text{x}-3\sin\text{x},\text{ prove that }\frac{\text{d}^2\text{y}}{\text{dx}^2}+\text{y}=0 3 Marks

Ans: Let \text{y}=5\cos\text{x}-3\cos\text{x}


\therefore\ \frac{\text{dy}}{\text{dx}}=-5\sin\text{x}-3\cos\text{x}
\Rightarrow\ \frac{\text{d}^2\text{y}}{\text{dx}^2}=-5\cos\text{x}+3\sin\text{x}=-(5\cos\text{x}-3\sin\text{x})=-\text{y}
\Rightarrow\ \frac{\text{d}^2\text{y}}{\text{dx}^2}=-\text{y}\ \Rightarrow\ \frac{\text{d}^2\text{y}}{\text{dx}^2}+\text{y}=0
Q691. Prove that the greatest integer function defined by 3 Marks
\text{f(x)} = [\text{x} ], 0 < \text{x} < 3
is not differentiable at x = 1 and x = 2.

Ans: Given: \text{f(x)} = [\text{x} ], 0 < \text{x} < 3


\text{R}\text{f}{'}(1) = ^{\ \ \text{lim}}_{\text{h}\rightarrow\text{0}}\frac{\text{f(1 + h)}-\text{f}(1)}{\text{h}} = ^{\ \ \text{lim}}_{\text{h}\rightarrow\text{0}}\frac{|1 + \text{h} |-1}{\text{h}} =
^{\ \ \text{lim}}_{\text{h}\rightarrow\text{0}}\frac{1-1}{\text{h}}= ^{\ \ \text{lim}}_{\text{h}\rightarrow\text{0}}\frac{0}{\text{h}}=0
\text{And}\ \text{L}\text{f}{'}(1) = ^{\ \ \text{lim}}_{\text{h}\rightarrow\text{0}}\frac{\text{f(1 - h)}-\text{f}(1)}{-\text{h}} = ^{\ \ \text{lim}}_{\text{h}\rightarrow\text{0}}\frac{|1 - \text{h}| -
1}{-\text{h}} = ^{\ \ \text{lim}}_{\text{h}\rightarrow\text{0}}\frac{|1-\text{h}|-1}{\text{h}}= ^{\ \ \text{lim}}_{\text{h}\rightarrow\text{0}}\frac{0-1}{-\text{h}}=\infty
Since \text{R }\text{f}{'}(1)\neq \text{L}{\text{f}}{'}(1)
Therefore, f(x) = [x] is not differentiable at x =1.
Similarly f(x) = [x] is not differentiable at x = 2.
Q692. Differentiate the following functions with respect to x: 3 Marks
2^{\text{x}^3}

Ans: Consider \text{y}=2^{\text{x}^3}


Differentiate it with respect to x,
\frac{\text{dy}}{\text{dx}}=\frac{\text{d}}{\text{dx}}\Big(2^{\text{x}^3}\Big)
=2^{\text{x}^3}\times\log_2\frac{\text{d}}{\text{dx}}(\text{x}^3)
[Using chain rule]
=3\text{x}^2\times2^{\text{x}^3}\times\log_2
Hence, the solution is \frac{\text{d}}{\text{dx}}\big(2^{\text{x}^3}\big)=3\text{x}^2\times2^{\text{x}^3}\log_2
Q693. Find \frac{\text{dy}}{\text{dx}} of the functions expressed in parametric: 3 Marks
\text{x}=\text{t}+\frac{1}{\text{t}},\text{ y}=\text{t}-\frac{1}{\text{t}}

Ans: Consider, \text{x}=\text{t}+\frac{1}{\text{t}} and \text{y}=\text{t}\frac{1}{\text{t}}


\Rightarrow\ \frac{\text{dx}}{\text{dt}}=1+\Big(-\frac{1}{\text{t}^2}\Big) and \frac{\text{dy}}{\text{dt}}=1-\Big(-\frac{1}{\text{t}^2}\Big)
\Rightarrow\ \frac{\text{dx}}{\text{dt}}=1-\frac{1}{\text{t}^2} and \frac{\text{dy}}{\text{dt}}=1+\frac{1}{\text{t}^2}
\Rightarrow\ \frac{\text{dx}}{\text{dt}}=\frac{\text{t}^2-1}{\text{t}^2} and \frac{\text{dy}}{\text{dt}}=\frac{\text{t}^2+1}{\text{t}^2}
\Rightarrow\ \frac{\text{dx}}{\text{dt}}=\frac{\frac{\text{dy}}{\text{dt}}}{\frac{\text{dx}}{\text{dt}}}
=\frac{\text{t}^2+\frac{1}{\text{t}^2}}{\text{t}^2-\frac{1}{\text{t}^2}}
=\frac{\text{t}^2+1}{\text{t}^2-1}
Q694. Show that the function defined by g (x) = x – [x] is discontinuous at all integral points. Here [x] denotes the greatest integer less than or equal to x. 3 Marks

Ans: g(x) = x - [x]


Let a be any integer.
^{\ \ \text{Lt}}_{\text{x}\rightarrow\text{a}^{-}}\text{g(x)} = ^{\ \ \text{Lt}}_{\text{x}\rightarrow\text{a}^{-}}\left\{\text{x-[x]}\right\}
[Put x - a - h, h > 0 so that h→0 as x→a-
= ^{\ \ \text{Lt}}_{\text{x}\rightarrow\text{0}}\left\{\text{(a}-\text{h})-(\text{a}-\text{h})\right\} = ^{\ \ \text{Lt}}_{\text{h}\rightarrow\text{0}}\left\{\text{(a}-\text{h})-(\text{a}-
\text{1})\right\}
= ^{\ \ \text{Lt}}_{\text{h}\rightarrow\text{0}}\text{(a}-\text{h}-\text{a}+1) = \text{a}- 0-\text{a} + 1 = 1
^{\ \ \text{Lt}}_{\text{x}\rightarrow\text{a}^{+}}\text{g(x)} = ^{\ \ \text{Lt}}_{\text{x}\rightarrow\text{a}^{+}}\left\{\text{x - [x]}\right\}
[Put x = a + h, h > 0 so that h→0 as x→a+]
= ^{\ \ \text{Lt}}_{\text{h}\rightarrow\text{0}}\left\{\text{(a}+\text{h})-(\text{a}+\text{h})\right\}= ^{\ \ \text{Lt}}_{\text{h}\rightarrow\text{0}}\left\{\text{(a}+\text{h})-\text{a}\right\} =^{\ \
\text{Lt}}_{\text{h}\rightarrow\text{0}}\text{(h)}
=0

https://bls.smartstudies.co.in/#/exam/pdf-preview/c59cb220-8e86-4716-9ff7-82aec16b1ade/1 136/158
5/26/24, 6:20 PM Exam Automation
\therefore\ ^{\ \ \text{Lt}}_{\text{x}\rightarrow\text{a}^{-}}\text{f(x)}\neq^{\ \ \text{Lt}}_{\text{x}\rightarrow\text{a}^{+}}\text{f(x)}
\therefore f is discontinuous at x = a
But a is any integral point.
\therefore f is discontinuous at all integgral points.
Q695. Differentiate w.r.t. x the function in Exercise: 3 Marks
(\log\text{x})^{\log\text{x}},\text{x}>1

Ans: Let \text{y}=(\log\text{x})^{\log\text{x}}


Tanking logarithm on both the sides, we obtain
\log\text{y}=\log\text{x}.\log(\log\text{x})
Differentiating both sides with respect to x, we obtain
\frac{1}{\text{y}}\frac{\text{dy}}{\text{dx}}=\frac{\text{d}}{\text{dx}}[\log\text{x}.\log(\log\text{x})]
\Rightarrow\ \frac{1}{\text{y}}\frac{\text{dy}}{\text{dx}}=\log(\log\text{x)}.\frac{\text{d}}{\text{dx}}(\log\text{x)}+\log\text{x}.\frac{\text{d}}{\text{dx}}[\log(\log\text{x})]
\Rightarrow\ \frac{\text{dy}}{\text{dx}}=\text{y}\Big[\log(\log\text{x)}.\frac{1}{\text{x}}+\log\text{x}.\frac{1}{\log\text{x}}.\frac{\text{d}}{\text{dx}}(\log\text{x})\Big]
\Rightarrow\ \frac{\text{dy}}{\text{dx}}=\text{y}\Big[\frac{1}{\text{x}}\log(\log\text{x)}+\frac{1}{\text{x}}\Big]
\therefore\ \frac{\text{dy}}{\text{dx}}=(\log\text{x)}^{\log\text{x}}\Big[\frac{1}{\text{x}}+\frac{\log(\log\text{x})}{\text{x}}\Big]
Q696. If `\text{y}=(\cos\text{x})^{(\cos\text{x})^{(\cos\text{x})\dots\infty}}, show that \frac{\text{dy}}{\text{dx}}=\frac{\text{y}^2\tan\text{x}}{\text{y}\log\cos\text{x}-1}. 3 Marks

Ans: We have, \text{y}=(\cos\text{x})^{(\cos\text{x})^{(\cos\text{x})\dots\infty}}


\Rightarrow\ \text{y}=(\cos\text{x})^\text{y}
\therefore\ \log\text{y}=\log(\cos\text{x})^\text{y}
Differentiating w.r.t. x, we get
\frac{1}{\text{y}}\cdot\frac{\text{dy}}{\text{dx}}=\text{y}\cdot\frac{\text{d}}{\text{dx}}(\log\cos\text{x})+\log\cos\text{x}\cdot\frac{\text{dy}}{\text{dx}}
\Rightarrow\ \frac{1}{\text{y}}\cdot\frac{\text{dy}}{\text{dx}}=\frac{\text{y}}{\cos\text{x}}\cdot\frac{\text{d}}{\text{dx}}(\cos\text{x})+\log\cos\text{x}\cdot\frac{\text{dy}}{\text{dx}}
\Rightarrow\ \frac{\text{dy}}{\text{dx}}\Big[\frac{1}{\text{y}}-\log\cos\text{x}\Big] =\frac{-\text{y}\sin\text{x}}{\cos\text{x}}=-\text{y}\tan\text{x}
\therefore\ \frac{\text{dy}}{\text{dx}}=\frac{-\text{y}^2\tan\text{x}}{(1-\text{y}\log\cos\text{x})}=\frac{\text{y}^2\tan\text{x}}{\text{y}\log\cos\text{x}-1}
Q697. If \text{y}=\sqrt{\text{x}+\sqrt{\text{x}+\sqrt{\text{x}+\ .... \text{to }\infty}}}, prove that \frac{\text{dy}}{\text{dx}}=\frac{1}{2\text{y}-1} 3 Marks

Ans: We have, \text{y}=\sqrt{\text{x}+\sqrt{\text{x}+\sqrt{\text{x}+\ .... \text{to }\infty}}}


Squaring both sides, we get,
y2 = x + y
\Rightarrow 2\text{y}\frac{\text{dy}}{\text{dx}}=1+\frac{\text{dy}}{\text{dx}}
\Rightarrow\frac{\text{dy}}{\text{dx}}(2\text{y}-1)=1
\Rightarrow\frac{\text{dy}}{\text{dx}}=\frac{1}{2\text{y}-1}
Q698. Find \frac{\text{dy}}{\text{ dx}} in the following: 3 Marks
\text{y}=\cos^{-1}\Bigg(\frac{{1}-\text{x}^{2}}{1+\text{x}^{2}}\Bigg), 0 <\text{x}<1

Ans: The given relationship is,


\text{y}=\cos^{-1}\Bigg(\frac{{1}-\text{x}^{2}}{1+\text{x}^{2}}\Bigg)
\Rightarrow\frac{\text{d}}{\text{dx}}\cos\text{y}=\frac{\text{d}}{\text{dx}}\Big[\frac{1-\text{x}^2}{1+\text{x}^2}\Big]
\Rightarrow-\sin\text{y}\frac{\text{dy}}{\text{dx}}=\frac{(1+\text{x}^2)(-2\text{x})-(1-\text{x}^2)2\text{x}}{(1+\text{x}^2) ^2}
\Rightarrow-\sqrt{1-\cos^2\text{y}}\frac{\text{dy}}{\text{dx}}=\frac{-2\text{x}-2\text{x}^3-2\text{x}+2\text{x}^3}{(1+\text{x}^2)^2}
\Rightarrow-\sqrt{1-\Big\{\frac{1-\text{x}^2}{1+\text{x}^2}\Big\}^2}\frac{\text{dy}}{\text{dx}}=\frac{-4\text{x}}{(1+\text{x}^2)^2}
\Rightarrow\sqrt{1-\frac{(1-\text{x}^2)^2-(1-\text{x}^2)^2}{(1+\text{x}^2)^2}}\frac{\text{dy}}{\text{dx}}=\frac{4\text{x}}{(1+\text{x}^2)^2}
\Rightarrow\sqrt{\frac{1+\text{x}^4+2\text{x}^2-1-\text{x}^4+2\text{x}^2}{(1+\text{x}^2)^2}}\frac{\text{dy}}{\text{dx}}=\frac{4\text{x}}{(1+\text{x}^2)^2}
\Rightarrow\sqrt{\frac{4\text{x}^2}{(1+\text{x}^2)^2}}\frac{\text{dy}}{\text{dx}}=\frac{4\text{x}}{(1+\text{x}^2)^2}
\Rightarrow\frac{2\text{x}}{1+\text{x}^2}\frac{\text{dy}}{\text{dx}}=\frac{4\text{x}}{1+\text{x}^2}
\frac{\text{dy}}{\text{dx}}=\frac{2}{1+\text{x}^2}
Q699. Differentiate the following functions with respect to x: 3 Marks
3^{\text{x}\log\text{x}}

Ans: Let \text{y}=3^{\text{x}\log\text{x}}


Differentiate it with respect to x we get,
\frac{\text{dy}}{\text{dx}}=\frac{\text{d}}{\text{dx}}\big(3^{\text{x}\log\text{x}}\big)
=3^{\text{x}\log\text{x}}\times\log_\text{e}3\frac{\text{d}}{\text{dx}}(\text{x}\log\text{x})
[Using chain rule]
=3^{\text{x}\log\text{x}}\times\log_\text{e}3\Big[\text{x}\frac{\text{d}}{\text{dx}}(\log\text{x})+\log\text{x}\frac{\text{d}}{\text{dx}}(\text{x})\Big]
=3^{\text{x}\log\text{x}}\times\log_\text{e}3\Big[\frac{\text{x}}{\text{x}}+\log\text{x}\Big]
=3^{\text{x}\log\text{x}}\big[1+\log\text{x}\big]\times\log_\text{e}3
So,
\frac{\text{d}}{\text{dx}}\big(3^{\text{x}\log\text{x}}\big)=3^{\text{x}\log\text{x}}\big[1+\log\text{x}\big]\log_\text{e}3
Q700. \text{If x}=\text{a}(\cos\text{t}+\text{t}\sin\text{t})\text{ and y}=\text{a}(\sin\text{t}-\text{t}\cos\text{t}),\ \text{find}\dfrac{\text{d}^2\text{y}}{\text{dx}^2} 3 Marks

Ans: It is given that, \text{x}=\text{a}(\cos\text{t}+\text{t}\sin\text{t})\text{ and y}=\text{a}(\sin\text{t}-\text{t}\cos\text{t})


\therefore\ \frac{\text{dx}}{\text{dt}}=\text{a}.\frac{\text{d}}{\text{dt}}(\cos\text{t}+\text{t}\sin\text{t})
=\text{a}\Big[-\sin\text{t}+\sin\text{t}.\frac{\text{d}}{\text{dx}}(\text{t})+\text{t}.\frac{\text{d}}{\text{dt}}(\sin\text{t})\Big]
=\text{a}[-\sin\text{t}+\sin\text{t}+\text{t}\cos\text{t}]=\text{at}\cos\text{t}
\frac{\text{dy}}{\text{dt}}=\text{a}.\frac{\text{d}}{\text{dt}}(\sin\text{t}-\text{t}\cos\text{t})
=\text{a}\Big[\cos\text{t}-\Big\{\cos\text{t}.\frac{\text{d}}{\text{dt}}(\text{t})+\text{t}.\frac{\text{d}}{\text{dt}}(\cos\text{t})\Big\}\Big]
=\text{a}[\cos\text{t}-\{\cos\text{t}-\text{t}\sin\text{t}\}]=\text{at}\sin\text{t}
\therefore\ \frac{\text{dy}}{\text{dx}}=\frac{\Big(\frac{\text{dy}}{\text{dt}}\Big)}{\Big(\frac{\text{dx}}{\text{dt}}\Big)}=\frac{\text{at}\sin\text{t}}{\text{at}\cos\text{t}}=\tan\text{t}
Then, \frac{\text{d}^2\text{y}}{\text{dx}^2}=\frac{\text{d}}{\text{dx}}\Big(\frac{\text{dy}}{\text{dx}}\Big)=\frac{\text{d}}{\text{dx}}(\tan\text{t})=\sec^2\text{t}.\frac{\text{dt}}{\text{dx}}
=\sec^2\text{t}.\frac{1}{\text{at}\cos\text{t}}\ \Big[\frac{\text{dx}}{\text{dt}}=\text{at}\cos\text{t}\Rightarrow\ \frac{\text{dt}}{\text{dx}}=\frac{1}{\text{at}\cos\text{t}}\Big]
=\frac{\sec^3\text{t}}{\text{at}}.0<\text{t}<\frac{\pi}{2}
Q701. Examine the following functions for continuity. 3 Marks
\text f(\text X)=\frac{\text X^{2} - 25}{\text {X} + 5}

Ans: \text f(\text X)=\frac{\text X^{2} - 25}{\text {X} + 5}


For f to be defined,
x + 5 \neq 0 i. e. x \neq -5
\therefore Df = Set of all real numbers except -5 = R - { -5}
Let \text{c} \neq -5 be any real number.
\therefore f(c) = \frac {\text{c}^2 - 25}{\text{c} + 25} = \frac{(\text{c}-5)(\text{c} + 5)}{\text{c}+5} = \text{c}-5
Also ^{\ \ \text{Lt}}_{\text{x}\rightarrow\text{c}}\text{f(x)} = ^{\ \ \text{Lt}}_{\text{x}\rightarrow\text{c}}\frac{\text{x}^2-25}{\text{x}+5} =\ ^{\ \
\text{Lt}}_{\text{x}\rightarrow\text{c}}\frac{({\text{x} -5)(\text{x} +5)}}{\text{x}+ 5}
= ^{\ \ \text{Lt}}_{\text{x}\rightarrow\text{c}}\text{(x}- 5) = \text{c} - 5
\therefore\ ^{\ \ \text{Lt}}_{\text{x}\rightarrow\text{c}}\text{f(x}) = \text {f(c)}
\therefore f is continuous at x = c.
But \text{c} \neq -5 is any real number.

https://bls.smartstudies.co.in/#/exam/pdf-preview/c59cb220-8e86-4716-9ff7-82aec16b1ade/1 137/158
5/26/24, 6:20 PM Exam Automation
\therefore f is continuous at every real number \text{c} \in \text{D}_\text{f}.
\therefore f is continuous function.
Q702. Examine the following functions for continuity. 3 Marks
\text f(\text x)=\begin{vmatrix}\text x-5\end{vmatrix}

Ans: Here f(x) = | x - 5|


Function f is defined for all real numbers.
Let c be any real number.
\therefore \text{f}(\text{c}) = \left| \text{c} - 5\right|
Also ^{\ \ \text{Lt}}_{\text{x}\rightarrow\text{c}}\text{f(x)} = ^{\ \ \text{Lt}}_{\text{x}\rightarrow\text{c}}\left|\text{x}- 5\right| = \left|\text{c}-5\right|
\therefore\ ^{\ \ \text{Lt}}_{\text{x}\rightarrow\text{c}}\text{f(x)} = \text{f(c)}
\therefore f is continuous at x = c,
But c is any real number
\therefore f is continuous at every real number.
Q703. Show that the function \text{f(x)}=\big|\sin\text{x}+\cos\text{x}| is continuous at \text{x}=\pi. 3 Marks

Ans: Consider, \text{f(x)}=\big|\sin\text{x}+\cos\text{x}\big|\text{ at x}=\pi


Let \text{g(x)}=\sin\text{x}+\cos\text{x}
And \text{h(x)}=|\text{x}|
\therefore\ \text{hog (x)}=\text{h}[\text{g (x)}]
=\text{h }(\sin\text{x}+\cos\text{x})
=|\sin\text{x}+\cos\text{x}|
Since, g(x) and h(x) are continuous functions and f(x) is a composite functions.
We know that composite functions of two continuous functions is also a continuous function.
Hence, \text{f(x)}=|\sin\text{x}+\cos\text{x}| is a continuous function everywhere,
So, f(x) is continuous at \text{x}=\pi.
Q704. Determine that value of the constant 'k' so that function \text{f(x)}=\begin{cases}\frac{\text{kx}}{|\text{x}|},&\text{if }\text{ x}<0\\3,&\text{if }\text{ x}\geq0\end{cases} is continuous at x = 3 Marks
0.

Ans: Given, \text{f(x)}=\begin{cases}\frac{\text{kx}}{|\text{x}|},&\text{if }\text{ x}<0\\3,&\text{if }\text{ x}\geq0\end{cases}


Since, the function is continuous at x = 0, therefore,
\lim\limits_{{\text{x}}\rightarrow0^-}\text{f(x})=\lim\limits_{{\text{x}}\rightarrow0^+}\text{f(x})=\text{f}(0)
\Rightarrow\lim\limits_{{\text{x}}\rightarrow0}\frac{-\text{kx}}{\text{x}}=\lim\limits_{{\text{x}}\rightarrow0}3=3
\Rightarrow-\text{k}=3
\Rightarrow\text{k}=-3
Q705. Discuss the continuity of the following functions at the indicated point: 3 Marks
\text{f}\text{(x)}=\begin{cases}\text{|x|}\cos\Big(\frac{1}{\text{x}}\Big), & \text{ x}\neq 0\\0 &\text{ x} = 0\end{cases}\text{at x}=0

Ans: Given,
\text{f}\text{(x)}=\text{x}\cos\Big(\frac{1}{\text{x}}\Big),\text{x}\neq0
\text{f}\text{(x)}=0,\ \text{x}=0
We observe
\lim\limits_{\text{x} \rightarrow 0}\text{f}\text{(x)}=\lim\limits_{\text{x} \rightarrow 0}\text{x}\cos\Big(\frac{1}{\text{x}}\Big)
\lim\limits_{\text{x} \rightarrow 0}\text{f}\text{(x)}=\lim\limits_{\text{x} \rightarrow 0}\text{x}\lim\limits_{\text{x} \rightarrow 0}\cos\Big(\frac{1}{\text{x}}\Big)
\lim\limits_{\text{x} \rightarrow 0}\text{f}\text{(x)}=0\times\lim\limits_{\text{x} \rightarrow 0}\cos\Big(\frac{1}{\text{x}}\Big)
=0
\lim\limits_{\text{x} \rightarrow 0}\text{f}\text{(x)}=\text{f}(0)
Hence, f(x) is continuous at x = 0.
Q706. Discuss the continuity of the function f(x) at the point \text{x}=\frac{1}{2} where 3 Marks
\text{f}\text{(x)}=\begin{cases}\text{x}, & 0\leq\text{x} < \frac{1}{2}\\\frac{1}{2},&\text{x}=\frac{1}{2}\\1-\text{x}, &\frac{1}{2}< \text{x}\leq 1\end{cases}

Ans: We want to discuss the continuity of the function at \text{x}=\frac{1}{2}.


\text{LHL}=\lim\limits_{\text{x} \rightarrow \frac{1}{2}^-}\text{f}\text{(x)}=\lim\limits_{\text{x} \rightarrow 0}\text{f}\Big(\frac{1}{2}-\text{h}\Big)=\lim\limits_{\text{x} \rightarrow 0}\frac{1}
{2}-\text{h}=\frac{1}{2}
\text{RHL}=\lim\limits_{\text{x} \rightarrow \Big(\frac{1}{2}^-\Big)}=\lim\limits_{\text{x} \rightarrow 0}\text{f}\Big(\frac{1}{2}+\text{h}\Big)=\lim\limits_{\text{x} \rightarrow 0}1-
\Big(\frac{1}{2}+\text{h}\Big)=\frac{1}{2}
\text{f}\Big(\frac{1}{2}\Big)=\frac{1}{2}
Thus, \text{LHL}=\text{RHL}=\text{f}\Big(\frac{1}{2}\Big)=\frac{1}{2}
Hence, the function is continuous at \text{x}=\frac{1}{2}.
Q707. If \text{f(x)}=\frac{2\text{x}+3\sin\text{x}}{3\text{x}+2\sin\text{x}},\text{ x}\neq0 is continuous at x = 0, then find f(0). 3 Marks

Ans: It is given that the function is continuous at x = 0


\therefore\text{LHL}=\text{RHL}=\text{f}(0)\ ....(\text{i})
\text{LHL}=\lim_\limits{\text{x}\rightarrow 0}=\lim_\limits{\text{h}\rightarrow 0}(0-\text{h})=\lim_\limits{\text{h}\rightarrow 0}\frac{2(-\text{h})+3\sin(-\text{h})}{3(-\text{h})+2\sin(-
\text{h})}
=\lim_\limits{\text{x}\rightarrow 0}\frac{-2\text{h}-3\sin\text{h}}{-3\text{h}-2\sin\text{h}}
=\lim_\limits{\text{x}\rightarrow 0}\frac{\frac{2\text{h}+3\sin\text{h}}{\text{h}}}{\frac{3\text{h}+2\sin\text{h}}{\text{h}}}
=\lim_\limits{\text{x}\rightarrow 0}\frac{2+3\frac{\sin\text{h}}{\text{h}}}{3+2\frac{\sin\text{h}}{\text{h}}}
=\frac{2+3}{3+2}=1 \Big[\because=\lim_\limits{\theta\rightarrow 0}\frac{\sin\theta}{\theta}=1\Big]
Using (i) we get
=\text{f}(0)=1
Q708. In the following, find the value of the constant k so that the given function is continuous at the indicated point: 3 Marks
\text{f(x)}=\begin{cases}\text{k}(\text{x}^2-2\text{x}),&\text{if}\text{ x}<0\\\cos\text{x},&\text{if}\text{ x}\geq0\end{cases}\text{at x} = 0

Ans: Given,
\text{f(x)}=\begin{cases}\text{k}(\text{x}^2-2\text{x}),&\text{if}\text{ x}<0\\\cos\text{x},&\text{if}\text{ x}\geq0\end{cases}
We have,
(\text{LHL at x}= 0)=\lim_\limits{\text{x}\rightarrow0^-}\text{f(x)}=\lim_\limits{\text{h}\rightarrow0}\text{f}(0-\text{h})
=\lim_\limits{\text{h}\rightarrow0}\text{f}(-\text{h})=\lim_\limits{\text{h}\rightarrow0}\text{k}(\text{h}^2+2\text{h})=0
(\text{RHL at x}= 0)=\lim_\limits{\text{x}\rightarrow0^+}\text{f(x)}=\lim_\limits{\text{h}\rightarrow0}\text{f}(0+\text{h})
=\lim_\limits{\text{h}\rightarrow0}\text{f}(\text{h})=\lim_\limits{\text{h}\rightarrow0}\cos\text{h}=1
\therefore\lim_\limits{\text{x}\rightarrow0^-}\text{f(x)}\neq\lim_\limits{\text{x}\rightarrow0^+}\text{f(x)}
Thus, no value of k exists for which f(x) is continuous at x = 0.
Q709. If f(x) = |x - 2| write whether f(2) exists or not. 3 Marks

Ans: Given: \text{f(x)}=|\text{x}-2|=\begin{cases}\text{x}-2, & \text{x}> 2\\-\text{x}+2, & \text{x}\leq 2\end{cases}


Now,
(LHL at x = 2)
\lim_\limits{\text{x}\rightarrow2^{-}}\frac{\text{f(x)}-\text{f}(2)}{\text{x}-2}
=\lim_\limits{\text{h}\rightarrow0}\frac{\text{f}(2-\text{h})-\text{f}(2)}{2-\text{h}-2}

https://bls.smartstudies.co.in/#/exam/pdf-preview/c59cb220-8e86-4716-9ff7-82aec16b1ade/1 138/158
5/26/24, 6:20 PM Exam Automation
=\lim_\limits{\text{h}\rightarrow0}\frac{(-2+\text{h}+2)-0}{-\text{h}}
=-1
(RHL at x = 2)
\lim_\limits{\text{x}\rightarrow2^{+}}\frac{\text{f(x)}-\text{f}(2)}{\text{x}-2}
=\lim_\limits{\text{h}\rightarrow0}\frac{\text{f}(2+\text{h})-\text{f}(2)}{2+\text{h}-2}
=\lim_\limits{\text{h}\rightarrow0}\frac{2+\text{h}+2-0}{\text{h}}
=1
Thus, (LHL at x = 2) \neq (RHL at x = 2)
Hence, \lim_\limits{\text{x}\rightarrow2}\frac{\text{f(x)}-\text{f}(2)}{\text{x}-2}=\text{f'}(2) does not exist.
Q710. Find the second order derivatives of the following functions: 3 Marks
\text{y}=\text{x}^3\log\text{x}

Ans: We have
\text{y}=\text{x}^3\log\text{x}
differentiating w.r.t.x, we get
\frac{\text{dy}}{\text{dy}}=3\text{x}^2\log\text{x}+\text{x}^3\times\frac{1}{\text{x}}
=3\text{x}^2\log\text{x}+\text{x}^2
differentiating w.r.t.x, we get
\frac{\text{d}^2\text{y}}{\text{dx}^2}=6\text{x}\log\text{x}+3\text{x}^2\times\frac{1}{\text{x}}+2\text{x}
=6\text{x}\log\text{x}+5\text{x}
Q711. Find which of the function: 3 Marks
\begin{cases}\frac{1-\cos2\text{x}}{\text{x}^2},&\text{ if x}\neq0\\5,&\text{if x}=0\end{cases}
at x = 0

Ans: We have, \begin{cases}\frac{1-\cos2\text{x}}{\text{x}^2},&\text{ if x}\neq0\\5,&\text{if x}=0\end{cases} at x = 0.


At x = 0, \text{L.H.L}=\lim\limits_{\text{h}\rightarrow0^-}\frac{1-\cos2\text{x}}{\text{x}^2}
=\lim\limits_{\text{h}\rightarrow0}\frac{1-\cos2(0-\text{h})}{(0-\text{h})^2}=\lim\limits_{\text{h}\rightarrow0}\frac{1-\cos2\text{h}}
{\text{h}^2}=\lim\limits_{\text{h}\rightarrow0}\frac{2\sin^2\text{h}}{\text{h}^2}=2
\text{R.H.L}=\lim\limits_{\text{h}\rightarrow0^+}\frac{1-\cos2\text{x}}{\text{x}^2}
=\lim\limits_{\text{h}\rightarrow0}\frac{1-\cos2(0+\text{h})}{(0+\text{h})^2}=\lim\limits_{\text{h}\rightarrow0}\frac{2\sin^2\text{h}}{\text{h}^2}=2
Also f(0) = 5 (given)
Since, L.H.L = R.H.L ≠ f(0)
Hence, f(x) is discontinuous at x = 0.
Q712. Differentiate the following functions with respect to x: 3 Marks
\log_7(2\text{x}-3)

Ans: Let, \text{y}=\log_7(2\text{x}-3)


\Rightarrow\ \text{y}=\frac{\log(2\text{x}-3)}{\log_7}\ \Big[\text{Since}, \log^\text{b}_\text{a}=\frac{\log\text{b}}{\log\text{a}}\Big]
Differentiate it with respect to x,
\frac{\text{dy}}{\text{dx}}=\frac{1}{\log7}\frac{\text{d}}{\text{dx}}\big(\log(2\text{x}-3)\big)
=\frac{1}{\log7}\times\frac{1}{(2\text{x}-3)}\frac{\text{d}}{\text{dx}}(2\text{x}-3)
[Using chain rule]
=\frac{2}{(2\text{x}-3)\log7}
Hence, \frac{\text{d}}{\text{dx}}(\log_7(2\text{x}-3))=\frac{2}{(2\text{x}-3)\log7}
Q713. For what value of k is the function 3 Marks
\text{f}\text{(x)}=\begin{cases}\frac{\sin5\text{x}}{3\text{x}}, &\text{if}\text{ x}\neq0\\\text{k}, &\text{if}\text{ x}=0\end{cases} is continuous at x = 0?

Ans: Given,
\text{f}\text{(x)}=\begin{cases}\frac{\sin5\text{x}}{3\text{x}}, &\text{if}\text{ x}\neq0\\\text{k}, &\text{if}\text{ x}=0\end{cases}
If f(x) is continuous at x = 0, then
\lim\limits_{\text{x} \rightarrow 0}\text{f}\text{(x)}=\text{f}(0)
\Rightarrow\lim\limits_{\text{x} \rightarrow 0}\frac{\sin5\text{x}}{3\text{x}}=\text{k}
\Rightarrow\lim\limits_{\text{x} \rightarrow 0}\frac{5\sin5\text{x}}{3\times5\text{x}}=\text{k}
\Rightarrow\frac{5}{3}\lim\limits_{\text{x} \rightarrow 0}\frac{\sin5\text{x}}{5\text{x}}=\text{k}
\Rightarrow\frac{5}{3}\times1=\text{k}
\Rightarrow\text{k}=\frac{5}{3}
Q714. Find all points of discontinuity of the function \text{f(t)}=\frac{1}{\text{t}^2+\text{t}-2}, where \text{t}=\frac{1}{\text{x}-1}. 3 Marks

Ans: We have, \text{f(t)}=\frac{1}{\text{t}^2+\text{t}-2}, where \text{t}=\frac{1}{\text{x}-1}


\therefore\ \text{f(x)}=\frac{1}{\Big(\frac{1}{\text{x}-1}\Big)^2+\frac{1}{\text{x}-1}-2}
=\frac{(\text{x}-1)^2}{1+(\text{x}-1)-2(\text{x}-1)^2}=\frac{(\text{x}-1)^2}{-(2\text{x}^2-5\text{x}+2)}=\frac{(\text{x}-1)^2}{(2\text{x}-1)(2-\text{x})}
So, f(t) is discontinuous at 2x - 1 = 0
\Rightarrow\ \text{x}=\frac{1}{2}
and 2 - x = 0 ⇒ x = 2
Also f(t) is discontinuous at x = 1, where \text{t}=\frac{1}{\text{x}-1} is discontinuous.
Q715. Determine the value of the constant k so that the function 3 Marks
\text{f}\text{(x)}=\begin{cases}\frac{\text{x}^2-3\text{x}+2}{\text{x}-1}, &\text{if}\text{ x}\neq1\\\text{k}, &\text{if}\text{ x}=1\end{cases} is continuous at x = 1

Ans: We have that the function is continuous at x = 1


\therefore LHL = RHL = f(1) ....(1)
Now,
\text{LHL}=\lim\limits_{\text{x} \rightarrow 1^-}\text{f}\text{(x)}=\lim\limits_{\text{h} \rightarrow 0}\text{f}(1-\text{h)}=\lim\limits_{\text{h} \rightarrow 0}\frac{(1-\text{h})^2-3(1-
\text{h})+2}{(1-\text{h})-1}
\lim\limits_{\text{h} \rightarrow 0}\frac{\text{h}^2+\text{h}}{-\text{h}}=\lim\limits_{\text{h} \rightarrow 0}-\text{h}-1=-1
\text{f}(1)=\text{k}
from (1), We get,
k = -1
Q716. For what value of k is the following function continuous at x = 1 3 Marks
\text{f}\text{(x)}=\begin{cases}\frac{\text{x}^2-1}{\text{x}-1}, & \text{x} \neq 1\\\text{k}, & \text{x}= 1\end{cases}

Ans: Given,
\text{f}\text{(x)}=\begin{cases}\frac{\text{x}^2-1}{\text{x}-1}, & \text{x} \neq 1\\\text{k}, & \text{x}= 1\end{cases}
If f(x) is continuous at x = 1, then
\lim\limits_{\text{x} \rightarrow 1}\text{f}\text{(x)}=\text{f}(1)
\lim\limits_{\text{x} \rightarrow 1}\frac{\text{x}^2-1}{\text{x}-1}=\text{k}
\lim\limits_{\text{x} \rightarrow 1}\frac{\text{(x}-1)(\text{x}+1)}{\text{x}-1}=\text{k}
\lim\limits_{\text{x} \rightarrow 1}(\text{x}+1)=\text{k}
\text{k}=2
Q717. Find \frac{\text{dy}}{\text{dx}} of the functions given in Exercise: 3 Marks

https://bls.smartstudies.co.in/#/exam/pdf-preview/c59cb220-8e86-4716-9ff7-82aec16b1ade/1 139/158
5/26/24, 6:20 PM Exam Automation
\text{y}^\text{x}=\text{x}^\text{y}

Ans: Given: \text{y}^\text{x}=\text{x}^\text{y}\ \Rightarrow\ \text{x}^\text{y}=\text{y}^\text{x}


\Rightarrow\ \log\text{x}^\text{y}=\log\text{y}^\text{x}\ \Rightarrow\ \text{y}\log\text{x}=\text{x}\log\text{y}
\Rightarrow\ \frac{\text{d}}{\text{dx}}(\text{y}\log\text{x})=\frac{\text{d}}{\text{dx}}(\text{x}\log\text{y})\ \Rightarrow\ \text{y}.\frac{1}{\text{x}}+\log\text{x}.\frac{\text{dy}}
{\text{dx}}=\text{x}.\frac{1}{\text{y}}\frac{\text{dy}}{\text{dx}}+\log\text{y}.1
\Rightarrow\ \Big(\log\text{x}-\frac{\text{x}}{\text{y}}\Big) \frac{\text{dy}}{\text{dx}}=\log\text{y}-\frac{\text{y}}{\text{x}}\ \Rightarrow\ \Big(\frac{\text{y}\log\text{x}-\text{x}}
{\text{y}}\Big)\frac{\text{dy}}{\text{dx}}=\frac{\text{x}\log\text{y}-\text{y}}{\text{x}}
\Rightarrow\ \frac{\text{dy}}{\text{dx}}=\frac{\text{y}(\text{x}\log\text{y}-\text{y})}{\text{x}(\text{y}\log\text{x}-\text{x})}
Q718. In the following, determine the values of constants involved in the definition so that the given function is continuous: 3 Marks
\text{f(x)}=\begin{cases}\frac{\sin2\text{x}}{5\text{x}},&\text{if }\text{ x}\neq0\\3\text{k},&\text{if }\text{ x}=0\end{cases}

Ans: Given, \text{f(x)}=\begin{cases}\frac{\sin2\text{x}}{5\text{x}},&\text{if }\text{ x}\neq0\\3\text{k},&\text{if }\text{ x}=0\end{cases}


If f(x) is continuous at x = 0, then
\Rightarrow\lim_\limits{\text{x}\rightarrow0}\frac{\sin2\text{x}}{5\text{x}}=\text{f}(0)
\Rightarrow\lim_\limits{\text{x}\rightarrow0}\frac{2\sin2\text{x}}{2\times5\text{x}}=\text{f}(0)
\Rightarrow\frac{2}{5}\lim_\limits{\text{x}\rightarrow0}\frac{\sin2\text{x}}{2\text{x}}=\text{f}(0)
\Rightarrow\frac{2}{5}=3\text{k}
\Rightarrow\text{k}=\frac{2}{15}
Q719. Find the values of k so that the function f is continuous at the indicated point: 3 Marks
\text{f(x)}\begin{cases}\frac{\text{k}\cos\text{x}}{\pi -2\text{x}}\ \text{if}\ \text{x}\neq \frac{\pi}{2}\\3, \ \ \ \ \ \ \ \ \text{if}\ \text{x} =\frac{\pi}{2}\end{cases}
\text{at} \text{x} = \frac{\pi}{2}

Ans: Here \text{f(x)}\begin{cases}\frac{\text{k}\cos\text{x}}{\pi -2\text{x}},\ \text{if}\ \text{x}\neq \frac{\pi}{2}\\3, \ \ \ \ \ \ \ \ \text{if}\ \text{x} =\frac{\pi}{2}\end{cases}
​^{\ \ \text{Lt}}_{\text{x}\rightarrow\frac{\pi}{2}}\text{f(x)}= ^{\ \ \text{Lt}}_{\text{x}\rightarrow\frac{\pi}{2}}\frac{\text{k}\cos\text{x}}{\pi - 2\text{x}} \left[ \text{Put}\ \text{x} = \frac{\pi}{2}
+ \text{h},\text{h} > 0 \ \text{so that h} \rightarrow0\ \text{as x} \rightarrow\frac{\pi}{2}\right]
^{\ \ \text{Lt}}_{\text{h}\rightarrow0}\frac{\text{k}\cos\Big(\frac{\pi}{2}+\text{h}\Big)}{{\pi}- {2}\Big(\frac{\pi}{2}-\text{h}\Big)}=^{\ \ \text{Lt}}_{\text{h}\rightarrow0}\frac{-
\text{k}\sin\text{h}}{-2\text{h}}
\frac{\text{k}}{2}\ ^{\ \ \text{Lt}}_{\text{h}\rightarrow0}\frac{\sin\text{h}}{\text{h}} = \frac{\text{k}}{2}\times1 = \frac{\text{k}}{2}
Also \text{f}(\frac{\pi}{2})= 3
Since f is continuous at \text{x}= \frac{\pi}{2}
\therefore\ ^{\ \ \text{Lt}}_{\text{x}\rightarrow{\frac{\pi}{2}}}\text {f(x}) = \text{f}\Big(\frac{\pi}{2}\Big)\Rightarrow \frac{\text{k}}{2} = 3 \Rightarrow\text{k} =6
Q720. Given the funcation \text{f(x)}=\frac{1}{\text{x}+2}. Find the points of discontinuity of the function f(f(x)). 3 Marks

Ans: \text{f}\big[\text{f(x)}\big]=\frac{1}{\frac{​​1}{\text{x}+2}+2}=\frac{\text{x}+2}{2\text{x}+5}
So, f[f(x)] is not defind at x + 2 = 0 and 2x + 5 = 0
If x + 2, then x = - 2
If 2x + 5 = 0, then \text{x}=-\frac{5}{2}
Hence, the function is dicontinuous at \text{x}=-\frac{5}{2} and -2
Q721. For what value of k is the function 3 Marks
\text{f}\text{(x)}=\begin{cases}\frac{\sin2\text{x}}{\text{x}}, & \text{x} \neq 0\\\text{k}, &\text{x} = 0\end{cases} continuous at x = 0.

Ans: Given, \text{f}\text{(x)}=\begin{cases}\frac{\sin2\text{x}}{\text{x}}, & \text{x} \neq 0\\\text{k}, &\text{x} = 0\end{cases}


If f(x) is continuous at x = 0, then
\lim\limits_{\text{x} \rightarrow 0}\text{f}\text{(x)}=\text{f}(0)
\Rightarrow\lim\limits_{\text{x} \rightarrow 0}\frac{\sin2\text{x}}{\text{x}}=\text{k}
\Rightarrow\lim\limits_{\text{x} \rightarrow 0}\frac{2\sin2\text{x}}{2\text{x}}=\text{k}
\Rightarrow2\lim\limits_{\text{x} \rightarrow 0}\frac{\sin2\text{x}}{2\text{x}}=\text{k}
\Rightarrow2\times1=\text{k}
\Rightarrow\text{k} =2
Q722. Differentiate the following w.r.t. x: 3 Marks
\frac{8^\text{x}}{\text{x}^8}

Ans: Let \text{y}=\frac{8^\text{x}}{\text{x}^8}


\Rightarrow\ \log\text{y}=\log\frac{8^\text{x}}{\text{x}^8}
\Rightarrow\ \frac{\text{d}}{\text{dy}}\log\text{y}\cdot\frac{\text{dy}}{\text{dx}}=\frac{\text{d}}{\text{dx}}\big[\log8^\text{x}-\log\text{x}^8\big]
\Rightarrow\ \frac{1}{\text{y}}\cdot\frac{\text{dy}}{\text{dx}}=\big[\text{x}.\log8-8.\log\text{x}]
On differentiating w.r.t. x, we get
\Rightarrow\ \frac{1}{\text{y}}\cdot\frac{\text{dy}}{\text{dx}}=\log8.1-8.\frac{1}{\text{x}}
\Rightarrow\ \frac{1}{\text{y}}\cdot\frac{\text{dy}}{\text{dx}}=\log8-\frac{8}{\text{x}}
\therefore\ \frac{\text{dy}}{\text{dx}}=\text{y}\Big(\log8-\frac{8}{\text{x}}\Big)=\frac{8^\text{x}}{\text{x}^8}\Big(\log8-\frac{8}{\text{x}}\Big)
Q723. Write the value of b which \text{f(x)}=\begin{cases}5\text{x}-4,&0<\text{x}\leq1\\4\text{x}^2+3\text{bx},&1<\text{x}<2\end{cases} is continuous at x = 1. 3 Marks

Ans: Given, \text{f(x)}=\begin{cases}5\text{x}-4,&0<\text{x}\leq1\\4\text{x}^2+3\text{bx},&1<\text{x}<2\end{cases}


If f(x) is continuous at x = 1, then
\lim\limits_{{\text{x}}\rightarrow1^-}\text{f(x})=\lim\limits_{{\text{x}}\rightarrow1^-}\text{f(x})=\text{f}(1)\ ...(\text{i})
Now,
\lim\limits_{{\text{x}}\rightarrow1^-}\text{f(x})=\lim\limits_{{\text{h}}\rightarrow0}\text{f}(1-\text{h})\\=\lim\limits_{{\text{h}}\rightarrow0}5(1-\text{h})-4=5-4=1
\lim\limits_{{\text{x}}\rightarrow1^+}\text{f(x})=\lim\limits_{{\text{h}}\rightarrow0}\text{f}(1+\text{h})\\=\lim\limits_{{\text{h}}\rightarrow0}4(1+\text{h})^2+3\text{b}
(1+\text{h})=4+3\text{b}
Also,
\text{f}(1)=5(1)-4=1
=\lim\limits_{{\text{x}}\rightarrow1^-}\text{f(x)}=\lim\limits_{{\text{x}}\rightarrow1^+}\text{f(x)}=\text{f}(1) [From eq. (i)]
\Rightarrow1=4+3\text{b}=1
\Rightarrow1=4+3\text{b}
\Rightarrow-3=3\text{b}
\Rightarrow\text{b}=-1
Thus, for b = -1, the function f(x) is continuous at x = 1.
Q724. In the following, find the value of the constant k so that the given function is continuous at the indicated point: 3 Marks
\text{f(x)}=\begin{cases}\text{k}+1,&\text{if}\text{ x}\leq5\\3\text{x}-5,&\text{if}\text{ x}>5\end{cases}\text{at x} =5

Ans: \text{f(x)}=\begin{cases}\text{k}+1,&\text{if}\text{ x}\leq5\\3\text{x}-5,&\text{if}\text{ x}>5\end{cases}\text{at x} =5


We have given that function is continuous at x = 5
\therefore\ \text{LHL}=\text{RHL}=\text{f}(5)\ ....(\text{i})
\text{f}(5)=5\text{k}+1
\text{LHL}=\lim_\limits{\text{x}\rightarrow5^+}\text{f(x)}=\lim_\limits{\text{h}\rightarrow0}(5+\text{h})
=\lim_\limits{\text{h}\rightarrow0}3(5+\text{h})-5=10
Thus, using (i) we get,
5\text{k}+1=10
5\text{k}=9

https://bls.smartstudies.co.in/#/exam/pdf-preview/c59cb220-8e86-4716-9ff7-82aec16b1ade/1 140/158
5/26/24, 6:20 PM Exam Automation
\text{k}=\frac{9}{5}
Q725. At what points on the following curves, is the tangent parallel to x-axis? 3 Marks
\text{y}=\text{x}^2\text{ on }[-2,2]

Ans: Let f(x) = x2


Since f(x) is a polynomial function, it is continuous on [-2, 2] and differentiable on (-2, 2)
Also, f(2) = f(-2) = 4
Thus, all the conditions of Rolle's theorem are satisfied.
Concequently, there exists at least one point \text{c}\in(-2,2) for which f'(c) = 0.
But \text{f}'(\text{c})=0
\Rightarrow2\text{c}=0
\Rightarrow\text{c}=0
\therefore\text{f}_\text{c}=\text{f}_0=0
By the geometrical interpretetion of Rolle's theorem, (0, 0) is the point on y = x2, where the tangent is parallel to the x-axis.
Q726. Differentiate the following functions with respect to x: 3 Marks
\log(\tan^{-1}\text{x})

Ans: Consider \text{y}=\log\big(\tan^{-1}\text{x}\big)


Differentiate with respect to x,
\frac{\text{dy}}{\text{dx}}=\frac{\text{d}}{\text{dx}}\log\big(\tan^{-1}\text{x}\big)
=\frac{1}{\tan^{-1}\text{x}}\times\frac{\text{d}}{\text{dx}}\big(\tan^{-1}\text{x}\big)
[Using chain rule]
=\frac{1}{\big(1+\text{x}^2\big)\tan^{-1}\text{x}}
Hence, the solution is, \frac{\text{d}}{\text{dx}}\big(\log\tan^{-1}\text{x}\big)=\frac{1}{\big(1+\text{x}^2\big)\tan^{-1}\text{x}}
Q727. Find \frac{\text{dy}}{\text{dx}},\ \text{if y}=\sin^{-1}\text{x}+\sin^{-1} \sqrt{1-\text{x}^2},\ -1\leq\text{x}\leq1 3 Marks

Ans: It is given that, \text{y}=\sin^{-1}\text{x}+\sin^{-1}\sqrt{1-\text{x}^2}


\therefore\ \frac{\text{dy}}{\text{dx}}=\frac{\text{d}}{\text{dx}}\Big[\sin^{-1}\text{x}+\sin^{-1}\sqrt{1-\text{x}^2}\Big]
\Rightarrow\ \frac{\text{dy}}{\text{dx}}=\frac{\text{d}}{\text{dx}}(\sin^{-1}\text{x})+\frac{\text{d}}{\text{dx}}(\sin^{-1}\sqrt{1-\text{x}^2})
\Rightarrow\ \frac{\text{dy}}{\text{dx}}=\frac{1}{\sqrt{1-\text{x}^2}}+\frac{1}{\sqrt{1-(\sqrt{1-\text{x}^2})^2}}.\frac{\text{d}}{\text{dx}}(\sqrt{1-\text{x}^2})
\Rightarrow\ \frac{\text{dy}}{\text{dx}}=\frac{1}{\sqrt{1-\text{x}^2}}+\frac{1}{\text{x}}.\frac{1}{2\sqrt{1-\text{x}^2}}.\frac{\text{d}}{\text{dx}}(1-\text{x}^2)
\Rightarrow\ \frac{\text{dy}}{\text{dx}}=\frac{1}{\sqrt{1-\text{x}^2}}+\frac{1}{2\text{x}\sqrt{1-\text{x}^2}}(-2\text{x})
\Rightarrow\ \frac{\text{dy}}{\text{dx}}=\frac{1}{\sqrt{1-\text{x}^2}}-\frac{1}{\sqrt{1-\text{x}^2}}
\therefore\ \frac{\text{dy}}{\text{dx}}=0
Q728. Differentiate the following functions with respect to x: 3 Marks
\tan^2\text{x}

Ans: Let,
\text{y}=\tan^2\text{x}
Differentiate it with respect to x,
\frac{\text{dy}}{\text{dx}}=2\tan\text{x }\frac{\text{d}}{\text{dx}}(\tan\text{x})\ \big[\text{using chain rule}\big]
=2\tan\text{x}\times\sec^2\text{x}
So,
\frac{\text{d}}{\text{dx}}=\big(\tan^2\text{x}\big)=2\tan\text{x }\sec^2\text{x}.
Q729. Find f(x) is continuse at x = 0, then \text{f(x)}=\frac{\text{x}}{1-\sqrt{1-\text{x}}} becomes continuous at x = 0. 3 Marks

Ans: If f(x) is continuous at x = 0, then \lim\limits_{{\text{x}}\rightarrow0}\text{f(x})=\text{f}(0)\ ....(\text{i})


Given, \text{f(x)}=\frac{\text{x}}{1-\sqrt{1-\text{x}}}
\Rightarrow\text{f(x)}=\frac{\text{x}\big(1+\sqrt{1-\text{x}}\big)}{\big(1-\sqrt{1-\text{x}}\big)\big(1+\sqrt{1-\text{x}}\big)}
\Rightarrow\text{f(x)}=\frac{\text{x}\big(1+\sqrt{1-\text{x}}\big)}{1-(1-\text{x})}
\Rightarrow\text{f(x)}=\big(1+\sqrt{1-\text{x}}\big)
\Rightarrow\lim\limits_{{\text{x}}\rightarrow0}=\big(1+\sqrt{1-\text{x}}\big)=\text{f}(0) [From eq. (i)]
\Rightarrow\text{f}(0)=2
So, for f(0) = 2, the function f(x) becomes continuous x = 0
Q730. For what value of k is the following function continuous at x = 2? 3 Marks
\text{f(x)}=\begin{cases}2\text{x}+1,&\text{if }\text{ x}<2\\\text{k},&\text{x}=2\\3\text{x}-1,&\text{x}>2\end{cases}

Ans: Given, \text{f(x)}=\begin{cases}2\text{x}+1,&\text{if }\text{ x}<2\\\text{k},&\text{x}=2\\3\text{x}-1,&\text{x}>2\end{cases}


We have,
(\text{LHL at x}= 2)=\lim_\limits{\text{x}\rightarrow2^-}\text{f(x)}=\lim_\limits{\text{h}\rightarrow0}\text{f}(2-\text{h})
=\lim_\limits{\text{h}\rightarrow0}\text{f}(2(2-\text{h})+1)=5
(\text{RHL at x}= 2)=\lim_\limits{\text{x}\rightarrow2^+}\text{f(x)}=\lim_\limits{\text{h}\rightarrow0}\text{f}(2+\text{h})
=\lim_\limits{\text{h}\rightarrow0}\text{f}(2+\text{h})=\lim_\limits{\text{h}\rightarrow0}3(2+\text{h})-1=5
Also, \text{f}(2)=\text{k}
If f(x) is continuous at x = 2, then
=\lim_\limits{\text{x}\rightarrow2^-}\text{f}(\text{x})=\lim_\limits{\text{x}\rightarrow2^+}\text{f}(\text{x})=\text{f}(2)
\Rightarrow5=5=\text{k}
Hence, for k = 5, (fx) is continuous at x = 2
Q731. In the following, determine the values of constants involved in the definition so that the given function is continuous: 3 Marks
\text{f(x)}=\begin{cases}\text{k}(\text{x}^2+3\text{x}),&\text{if }\text{ x}<0\\\cos2\text{x},&\text{if }\text{ x}\geq0\end{cases}

Ans: Given, \text{f(x)}=\begin{cases}\text{k}(\text{x}^2+3\text{x}),&\text{if }\text{ x}<0\\\cos2\text{x},&\text{if }\text{ x}\geq0\end{cases}


If f(x) is continuous at x = 0, then
\lim_\limits{\text{x}\rightarrow0^-}\text{f(x)}=\lim_\limits{\text{x}\rightarrow0^+}\text{f(x)}
\Rightarrow\lim_\limits{\text{h}\rightarrow0}\text{f}{(-\text{h)}}=\lim_\limits{\text{h}\rightarrow0}\text{f}(-\text{h})
\Rightarrow\lim_\limits{\text{h}\rightarrow0}\Big(\text{k}\big((-\text{h})^2-3\text{h}\big)\Big)=\lim_\limits{\text{h}\rightarrow0}(\cos2\text{h})
\Rightarrow0=1 [It is not possible]
Hence, there does not exist any value of k, which can make the given function continuous.
Q732. Test the continuity of the function on f(x) at the origin: 3 Marks
\text{f}\ (\text{x})=\begin{cases}\frac{\text{x}}{\text{|x|}},& \text{x}\neq0\\1, & \text{x} = 0\end{cases}

Ans: Given,
\text{f}\ (\text{x})=\text{x},\text{ x}\neq0
\text{f}\ (\text{x})=1,\text{ x}=0
We observe
\text{(LHL at x}= 0)
\lim\limits_{\text{x} \rightarrow 0^-}\text{f}\text{ (x)}=\lim\limits_{\text{h} \rightarrow 0}\text{f} \ (0-\text{h})
\lim\limits_{\text{x} \rightarrow 0^-}\text{f}\text{ (-h)}=\lim\limits_{\text{h} \rightarrow 0} \frac{\text{-h}}{\text{h}}
\lim\limits_{\text{h} \rightarrow 0}-1=-1
\text{(RHL at x}=0)

https://bls.smartstudies.co.in/#/exam/pdf-preview/c59cb220-8e86-4716-9ff7-82aec16b1ade/1 141/158
5/26/24, 6:20 PM Exam Automation
\lim\limits_{\text{x} \rightarrow 0^+}\text{f}\ \text{(x)}=\lim\limits_{\text{h} \rightarrow 0}\text{f}(0+\text{h})
\lim\limits_{\text{h} \rightarrow 0}\text{f}\ \text{(h)}=\lim\limits_{\text{h} \rightarrow 0}\frac{\text{h}}{\text{h}}
\lim\limits_{\text{h} \rightarrow 0}1=1
Hence, f(x) is discontinuous at the origin.
Q733. Find \frac{\text{dy}}{\text{dx}} in the following cases: 3 Marks
\text{e}^{\text{x}-\text{y}}=\log\Big(\frac{\text{x}}{\text{y}}\Big)

Ans: We have, \tan^{-1}\big(\text{x}^2+\text{y}^2\big)=\text{a}


Differentiating with respect to x, we get
\frac{\text{d}}{\text{dx}}\big[\tan^{-1}\big(\text{x}^2+\text{y}^2\big)\big]=\frac{\text{d}}{\text{dx}}(\text{a})
\Rightarrow\frac{1}{1+(\text{x}^2+\text{y}^2)^2}\times\frac{\text{d}}{\text{dx}}\big(\text{x}^2+\text{y}^2\big)=0
\Rightarrow\Big[\frac{1}{1+(\text{x}^2+\text{y}^2)^2}\Big]\Big(2\text{x}+2\text{y}\frac{\text{dy}}{\text{dx}}\Big)=0
\Rightarrow2\text{x}+2\text{y}\frac{\text{dy}}{\text{dx}}=0
\Rightarrow\text{x}+\text{y}\frac{\text{dy}}{\text{dx}}=0
\Rightarrow\frac{\text{dy}}{\text{dx}}=-\frac{\text{x}}{\text{y}}
Q734. Discuss the applicability of the Rolle's theorem for the following function on the indicated interval 3 Marks
\text{f}(\text{x})=\text{x}^{\frac{2}{3}}\text{ on }[-1,1]

Ans: Here, \text{f}(\text{x})=\text{x}^{\frac{2}{3}}\text{ on }[-1,1]


\text{f}'(\text{x})=\frac{2}{3\text{x}^{\frac{1}{3}}}
\text{f}'(0)=\frac{2}{3(0)^{\frac{1}{3}}}
\text{f}'(0)=\infty
So, f'(x) does not exist at \text{x}=0\in(-1,1)
⇒ f(x) is not differentialble in \text{x}\in(-1,1)
So, Rolle's theorem is not applicable on f(x) in [-1, 1].
Q735. Examine the differentialiblilty of the function f defined by \text{f(x)}=\begin{cases}2\text{x}+3 & \text{if}-3\leq\text{x}\leq-2\\\text{x}+1 & \text{if} -2\leq\text{x}\leq0\\\text{x}+2&\text{if}\ 3 Marks
0\leq\text{x}\leq1\end{cases}

Ans: \text{f(x)}=\begin{cases}2\text{x}+3 & \text{if}-3\leq\text{x}\leq-2\\\text{x}+1 & \text{if} -2\leq\text{x}\leq0\\\text{x}+2&\text{if}\ 0\leq\text{x}\leq1\end{cases}


\text{f}'(\text{x})=\begin{cases}2 & \text{if}-3\leq\text{x}\leq-2,\\1 & \text{if} -2\leq\text{x}\leq0\\1&\text{if}\ 0\leq\text{x}\leq1\end{cases}
Now,
\text{LHL}=\lim_\limits{\text{x}\rightarrow2^{-}}\text{f}'(\text{x})=\lim_\limits{\text{x}\rightarrow2^{-}}2=2
\text{RHL}=\lim_\limits{\text{x}\rightarrow2^{+}}\text{f}'(\text{x})=\lim_\limits{\text{x}\rightarrow2^{+}}1=1
Since, at \text{x}=-2,\text{LHL}\neq\text{RHL}
Hence, f(x) is not differentiable at x = -2
Again,
\text{LHL}=\lim_\limits{\text{x}\rightarrow0^{-}}\text{f}'(\text{x})=\lim_\limits{\text{x}\rightarrow0^{-}}1=1
\text{RHL}=\lim_\limits{\text{x}\rightarrow0^{+}}\text{f}'(\text{x})=\lim_\limits{\text{x}\rightarrow0^{+}}1=1
Since, at \text{x}=0,
\text{LHL}=\text{RHL}
Hence, f(x) is differentiable at x = 0
Q736. Using Rolle’s theorem, find the point on the curve \text{y}=\text{x}(\text{x}-4),\text{x}\in[0,4]. where the tangent is parallel to x-axis. 3 Marks

Ans: We have, \text{y}=\text{x}(\text{x}-4),\text{x}\in[0,4]


Since given function is polynomial it is continuous and differentiable
Also y(0) = y(4) = 0
So, conditions of Rolle's theorem are satisfied.
Hence there exists a point \text{c}\in(0,4) such that
f'(c) = 0
⇒ 2c - 4 = 0
⇒c=2
⇒ x = 2 and y(2) = 2(2 - 4) = -4
Therefore, the required point on the curve, where the tangent drawn is parallel to the x-axix is (2, -4).
Q737. State Rolle's theorem. 3 Marks

Ans: Rolle's theorem: Let f(x) be a real value function defined on the closed interval [a, b] such that
1. It is continuous on [a, b]
2. It is differentiable on (a, b)
3. f(a) = f(b)
Then, there exists a real number \text{c}\in(\text{a},\text{b}) such that f'(c) = 0.
Q738. Find the value of k for which the function \text{f(x)}=\begin{cases}\frac{\text{x}^{2} + 3\text{x} - 10}{\text{x} - 2},&\text{x}\neq2\\\text{k},&\text{x} = {2}\end{cases} is continues at x = 2. 3 Marks

Ans: Given
\text{f(x)} = \frac{\text{x}^{2} + 3\text{x} - 10}{\text{x} - 2}
Continuity
\text{x} = 2
\lim\limits_{\text{x} \rightarrow 2} \frac{\text{x}^{2} + 3\text{x} - 10}{\text{x} -2} = \text{k}
\lim\limits_{\text{x}\rightarrow 2} \frac{\text{x}^{2} + 5\text{x} - 2\text{x} - 10}{\text{x} - 2 } = \text{k}
\lim\limits_{\text{x} \rightarrow 2}\frac{\text{x} (\text{x} + 5) - 2 (\text{x} + 5)}{\text{x} - 2} = \text{k}
\lim\limits_{\text{x} \rightarrow 2} \frac{(\text{x} - 2) (\text{x} + 5)}{\text{(x} - 2)} = \text{k}
When x = 2
x+5=k
k=5+2=7
k=7
Q739. State Lagrange's mean value theorem. 3 Marks

Ans: Lagrange's Mean Value Theorem:


Let f(x) be a function defined on [a, b] such that
1. It is continuous on [a, b] and
2. It is differentiable on (a, b).
Then, there exists a real number \text{c}\in(\text{a},\text{b}) such that \text{f}'(\text{c})=\frac{\text{f}(\text{b})-\text{f}(\text{a})}{\text{b}-\text{a}}.
Q740. \text{If y}=\text{e}^{\text{y}}(\text{x}+1)=1,\text{ show that }\frac{\text{d}^2\text{y}}{\text{dx}^2}=\Big(\frac{\text{dy}}{\text{dx}}\Big)^2 3 Marks

Ans: Here \text{e}^{\text{y}}(\text{x}+1)=1


\therefore\ \log[\text{e}^{\text{y}}(\text{x}+1)]=\log1
\therefore\ \log\text{e}^{\text{y}}(\text{x}+1)=0
\therefore\ \text{y}\log\text{e}=-\log(\text{x}+1)
\therefore\ \text{y}=-\log(\text{x}+1)\ \ [\because\text{loge}=1]
\therefore\ \frac{\text{dy}}{\text{dx}}=-\frac{1}{\text{x}+1}\ \dots(1)

https://bls.smartstudies.co.in/#/exam/pdf-preview/c59cb220-8e86-4716-9ff7-82aec16b1ade/1 142/158
5/26/24, 6:20 PM Exam Automation
\therefore\ \frac{\text{d}^2\text{y}}{\text{dx}^2}=-\Bigg[\frac{(\text{x}+1).\frac{\text{d}}{\text{dx}}(1)-1.\frac{\text{d}}{\text{dx}}(\text{x}+1)}{(\text{x}+1)^2}\Bigg] =-
\Big[\frac{(\text{x}+1).0-1.1}{(\text{x}+1)^2}\Big]
=\frac{1}{(\text{x}+1)^2}=\Big(-\frac{1}{(\text{x}+1)^2}\Big)^2
\therefore\ \frac{\text{d}^2\text{y}}{\text{dx}^2}=\Big(\frac{\text{dy}}{\text{dx}}\Big)^2\ \dots[\because\text{of }(1)]
Q741. \text{If x}\sqrt{1+\text{y}}+\text{y}\sqrt{1+\text{x}}=0, for, < x < 1, prove that 3 Marks

Ans: It is given that,


\text{x}\sqrt{1+\text{y}}+\text{y}\sqrt{1+\text{x}}=0
\Rightarrow\ \text{x}\sqrt{1+\text{y}}=-\text{y}\sqrt{1+\text{x}}
Squaring both sides, we obtain
\text{x}^2(1+\text{y})=\text{y}^2(1+\text{x})
\Rightarrow\ \text{x}^2+\text{x}^2\text{y}=\text{y}^2+\text{xy}^2
\Rightarrow\ \text{x}^2-\text{y}^2=\text{xy}^2-\text{x}^2\text{y}
\Rightarrow\ \text{x}^2-\text{y}^2=\text{xy}(\text{y}-\text{x})
\Rightarrow\ (\text{x}+\text{y})(\text{x}-\text{y})=\text{xy}(\text{y}-\text{x})
\therefore\ \text{x}+\text{y}=-\text{xy}
\Rightarrow\ (1+\text{x})\text{y}=-\text{x}
\Rightarrow\ \text{y}=\frac{-\text{x}}{(1+\text{x})}
Differentiating both sides with respect to x, we obtain
\text{y}=\frac{-\text{x}}{(1+\text{x})}
\frac{\text{dy}}{\text{dx}}=\frac{(1+\text{x})\frac{\text{d}}{\text{dx}}(\text{x)}-\text{x}\frac{\text{d}}{\text{dx}}(1+\text{x})}{(1+\text{x})^2} =-\frac{(1+\text{x})-\text{x}}{(1+\text{x})^2}=-
\frac{1}{(1+\text{x})^2}
Hence, proved.
Q742. Show that the Lagrange's mean value theorem is not applicable to the function 3 Marks
\text{f}(\text{x})=\frac{1}{\text{x}}\text{ on }[-1,1]

Ans: Given,
\text{f}(\text{x})=\frac{1}{\text{x}}
Clearly, f(x) is does not exist for x = 0
Thus, the given function is discontinuous on [-1, 1]
Hence, Lagrange's mean value theorem is not applicable for the given function on [-1, 1].
Q743. Write the derivative of f(x) = |x|3 at x = 0. 3 Marks

Ans: Given: \text{f(x)}=|\text{x}^3|=\begin{cases}\text{x}^3,&\text{x}\geq0\\-\text{x}^3,&\text{x}<0\end{cases}


(LHL at x = 0)
\lim_\limits{\text{x}\rightarrow0^{-}}\frac{\text{f(x)}-\text{f}(0)}{\text{x}-0}
=\lim_\limits{\text{x}\rightarrow0}\frac{\text{f}(0-\text{h})-\text{f}(0)}{\text{x}}
=\lim_\limits{\text{x}\rightarrow0}\frac{\text{h}^3}{-\text{h}}
=0
(RHL at x = 0)
\lim_\limits{\text{x}\rightarrow0^{+}}\frac{\text{f(x)}-\text{f}(0)}{\text{x}-0}
=\lim_\limits{\text{x}\rightarrow0}\frac{\text{f}(0+\text{h})-\text{f}(0)}{\text{x}}
=\lim_\limits{\text{x}\rightarrow0}\frac{\text{h}^3-0}{-\text{h}}
=0
And f(0) = 0.
Thus, (LHL at x = 0) = (RHL at x = 0) = f(0)
Hence, \lim_\limits{\text{x}\rightarrow0}\frac{\text{f(x)}-\text{f}(0)}{\text{x}-0}=\text{f}'(0)=0.
Q744. Differentiate the following w.r.t. x: 3 Marks
\tan^{-1}\Big(\frac{3\text{a}^2\text{x}-\text{x}^3}{\text{a}^3-3\text{ax}^2}\Big),\frac{-1}{\sqrt{3}}<\frac{\text{x}}{\text{a}}<\frac{1}{\sqrt{3}}

Ans: Let \text{y}=\tan^{-1}\Big(\frac{3\text{a}^2\text{x}-\text{x}^3}{\text{a}^3-3\text{ax}^2}\Big)


Put \text{x}=\text{a}\tan\theta\Rightarrow\ \theta=\tan^{-1}\frac{\text{x}}{\text{a}}
\therefore\ \text{y}=\tan^{-1}\bigg[\frac{3\tan\theta-\tan^3\theta}{1-3\tan^2\theta}\bigg] \bigg[\because\tan3\theta=\frac{3\tan\theta-\tan^3\theta}{1-3\tan^2\theta}\bigg]
=\tan^{-1}(\tan3\theta)=3\theta
=3\tan^{-1}\frac{\text{x}}{\text{a}}\Big[\because\theta=\tan^{-1}\frac{\text{x}}{\text{a}}\Big]
\therefore\ \frac{\text{dy}}{\text{dx}}=3\cdot\frac{\text{d}}{\text{dx}}\tan^{-1}\frac{\text{x}}{\text{a}} =3\cdot\Bigg[\frac{1}{1+\frac{\text{x}^2}{\text{a}^2}}\Bigg]\cdot\frac{\text{d}}
{\text{dx}}\cdot\Big(\frac{\text{x}}{\text{a}}\Big)
=3\cdot\frac{\text{a}^2}{\text{a}^2+\text{x}^2}.=\frac{1}{\text{a}}=\frac{3\text{a}}{\text{a}^2+\text{x}^2}
Q745. If \text{y}=\sin(\log\text{x}) prove that \text{x}^2\frac{\text{d}^2\text{y}}{\text{dx}^2}+\text{x}\frac{\text{dy}}{\text{dx}}+\text{y}=0 3 Marks

Ans: Here,
\text{y}=\sin(\log\text{x})
Differentiating w.r.t.x, we get
\frac{\text{dy}}{\text{dx}}=\frac{\cos(\log\text{x})}{\text{x}}
Differentiating w.r.t.x, we get
\frac{\text{d}^2\text{y}}{\text{dx}^2}=\frac{-\sin(\log\text{x})-\cos(\log\text{x})}{\text{x}^2}
\Rightarrow\frac{\text{d}^2\text{y}}{\text{dx}^2}=\frac{-\sin(\log\text{x})}{\text{x}^2}-\frac{\cos(\log\text{x})}{\text{x}^2}{}
\Rightarrow\frac{\text{d}^2\text{y}}{\text{dx}^2}=\frac{-\text{y}}{\text{x}^2}-\frac{1}{\text{x}}\times\frac{\text{dy}}{\text{dx}}
\Rightarrow\text{x}^2\frac{\text{d}^2\text{y}}{\text{dx}^2}+\text{x}\frac{\text{dy}}{\text{dx}}+\text{y}=0
Q746. Differentiate the following functions with respect to x: 3 Marks
\tan^{-1}\Big(\frac{\sin\text{x}}{1+\cos\text{x}}\Big),\pi<\text{x}<\pi

Ans: Let \text{f(x)}=\tan^{-1}\Big(\frac{\sin\text{x}}{1+\cos\text{x}}\Big)


This function is defined for all real numbers where \cos\text{x}\neq1
\text{f(x)}=\tan^{-1}\Big(\frac{\sin\text{x}}{1+\cos\text{x}}\Big)
\Rightarrow\ \text{f(x)}=\tan^{-1}\Bigg[\frac{2\sin\big(\frac{\text{x}}{2}\big)\cos\big(\frac{\text{x}}{2}\big)}{2\cos^2\big(\frac{\text{x}}{2}\big)}\Bigg]
\Rightarrow\ \text{f(x)}=\tan^{-1}\big[\tan\big(\frac{\text{x}}{2}\big)\big]=\frac{\text{x}}{2}
Thus, \text{f'(x)}=\frac{\text{d}}{\text{dx}}\big(\frac{\text{x}}{2}\big)=\frac{1}{2}
Q747. If x and y are connected parametrically by the equations given in Exercise without eliminating the parameter, Find \frac{\text{dy}}{\text{dx}}. 3 Marks
\text{x}=\text{a}(\theta-\sin\theta),\text{y}=\text{a}(1+\cos\theta)

Ans: The given equations are \text{x}=\text{a}(\theta-\sin\theta)\text{ and y}=\text{a}(1+\cos\theta)


Then, \frac{\text{dx}}{\text{d}\theta}= \text{a}\Big[\frac{\text{d}}{\text{d}\theta}(\theta)-\frac{\text{d}}{\text{d}\theta}(\sin\theta)\Big]=\text{a}(1-\cos\theta)
\frac{\text{dy}}{\text{d}\theta}= \text{a}\Big[\frac{\text{d}}{\text{d}\theta}(1)+\frac{\text{d}}{\text{d}\theta}(\cos\theta)\Big]=\text{a}[0+(-\sin\theta)]=-\text{a}\sin\theta
\therefore\ \frac{\text{dy}}{\text{dx}}=\frac{\Big(\frac{\text{dy}}{\text{d}\theta}\Big)}{\Big(\frac{\text{dx}}{\text{d}\theta}\Big)}=\frac{-\text{a}\sin\theta}{\text{a}(1-
\cos\theta)}=\frac{-2\sin\frac{\theta}{2}\cos\frac{\theta}{2}}{2\sin^2\frac{\theta}{2}}=\frac{-\cos\frac{\theta}{2}}{\sin\frac{\theta}{2}}=-\cot\frac{\theta}{2}
Q748. Discuss the continuity of the following functions: 3 Marks
\text{f(x)} = \sin \text{x} . \cos \text{x}

Ans: Let a be an arbitrary real number then ^{\ \ \text{lim}}_{\text{x}\rightarrow\text{a}^{+}}\text{f(x)} \Rightarrow^{\ \ \text{lim}}_{\text{h}\rightarrow\text{0}}\text{f(a + h)}

https://bls.smartstudies.co.in/#/exam/pdf-preview/c59cb220-8e86-4716-9ff7-82aec16b1ade/1 143/158
5/26/24, 6:20 PM Exam Automation
\Rightarrow\ ^{\ \ \text{lim}}_{\text{h}\rightarrow\text{0}}\sin\text{(a + h)} . \cos (\text{a} + \text{h})
\Rightarrow\ ^{\ \ \text{lim}}_{\text{h}\rightarrow\text{0}}(\sin\text{a}\cos\text{ h} + \cos\text{a} \sin \text{h})(\cos \text{a}\cos\text{h}-\sin\text{a}\sin\text{h})
= (\sin \text{a}\cos0+\cos\text{a}\sin0) (\cos\text{a}\cos0 - \sin\text{a}\sin0)
=( \sin \text{a} + 0) ( \cos\text{a}-0)
= \sin \text{a} . \cos\text{a}= \text{f(a)}
Similarly, we have ^{\ \ \text{lim}}_{\text{x}\rightarrow\text{a}^{-}}\text{f(x)} = \text{f(a)}
\therefore\ ^{\ \ \text{lim}}_{\text{x}\rightarrow\text{a}^{-}}\text{f(x)}= \text{f(a)}= ^{\ \ \text{lim}}_{\text{x}\rightarrow\text{a}^{+}}\text{f(x)}
Therefore, f(x) is continuous at x = a.
Since, a is an arbitrary real number, therefore, \text{f(x)}= \sin\text{x} . \cos\text{x} is continuous.
Q749. Find \frac{\text{dy}}{\text{dx}},\ \text{if y}=12(1-\cos \text{t}),\ \text{x}=10(\text{t}-\sin\text{t}),\ -\frac{\pi}{2}<\text{t}<\frac{\pi}{2} 3 Marks

Ans: It is given that, \text{y}=12(1-\cos\text{t}),\text{x}=10(\text{t}-\sin\text{t})


\therefore\ \frac{\text{dx}}{\text{dt}}=\frac{\text{d}}{\text{dt}}[10(\text{t}-\sin\text{t})] =10.\frac{\text{d}}{\text{dt}}(\text{t}-\sin\text{t})=10(1-\cos\text{t})
\frac{\text{dy}}{\text{dt}}=\frac{\text{d}}{\text{dt}}[12(1-\cos\text{t}) =12.\frac{\text{d}}{\text{dt}}(1-\cos\text{t})=12.[0-(-\sin\text{t})]=12\sin\text{t}
\therefore\ \frac{\text{dx}}{\text{dx}}\frac{\Big(\frac{\text{dy}}{\text{dt}}\Big)}{\Big(\frac{\text{dx}}{\text{dt}}\Big)}=\frac{12\sin\text{t}}{10(1-\cos\text{t})} =\frac{12.2\sin\frac{\text{t}}
{2}.\cos\frac{\text{t}}{2}}{10.2\sin^2\frac{\text{t}}{2}}=\frac{6}{5}\cot\frac{\text{t}}{2}
Q750. Find \frac{\text{dy}}{\text{ dx}} in the following: 3 Marks
\text{y}=\tan^{-1}\Bigg(\frac{3\text{x}-\text{x}^{3}}{1-3\text{x}^{2}}\Bigg), -\frac{1}{\sqrt{3}}<\text{x}<\frac{1}{\sqrt{3}}

Ans: The given relationship is \text{y}=\tan^{-1}\Bigg(\frac{3\text{x}-\text{x}^{3}}{1-3\text{x}^{2}}\Bigg)


\Rightarrow\tan\text{y}=\frac{3\text{x}-\text{x}^{3}}{1-3\text{x}^{2}} ...\text{(i)}
\text{y}=\tan^{-1}\Bigg(\frac{3\text{x}-\text{x}^{3}}{1-3\text{x}^{2}}\Bigg)
It is known that, \tan\text{y}=\frac{3\tan\frac{\text{y}}{3}-\tan^{3}\frac{\text{y}}{3}}{1-3\tan^{2}\frac{\text{y}}{3}} ...\text{(ii)}
Comparing equations (1) and (2), we obtain
\text{x} =\tan\frac{\text{y}}{3}
Differenting this relationship with respect to x, we obtain
\frac{\text{d}}{\text{dx}}(\text{x})=\frac{\text{d}}{\text{dx}}​​\Bigg(\tan\frac{\text{y}}{3}\Bigg)
\Rightarrow1 =\sec^{2}\frac{\text{y}}{3}.\frac{\text{d}}{\text{dx}}\Bigg(\frac{\text{y}}{3}\Bigg)
\Rightarrow1 =\sec^{2}\frac{\text{y}}{3}.\frac{\text{1}}{\text{3}}\frac{\text{dy}}{\text{dx}}
\Rightarrow\frac{\text{dy}}{\text{dx}} = \frac{3}{\sec^{2}\frac{\text{y}}{3}} =\frac{3}{1 +\tan^{2}\frac{\text{y}}{3}}
\therefore \frac{\text{dy}}{\text{dx}}= \frac{3}{1+ \text{x}^{2}}
Q751. Find which of the function: 3 Marks
\text{f(x)}=\begin{cases}\frac{|\text{x}-4|}{2(\text{x}-4)},&\text{if x}\neq4\\0,&\text{if x}=4\end{cases}
at x = 4

Ans: The condition for function f to be a continuous at x = a is given by =\lim\limits_{\text{x}\rightarrow\text{a}^-}\text{f(x)}=\lim\limits_{\text{x}\rightarrow\text{a}^+}\text{f(x)}=\text{f(a)}


Consider, \text{f(x)}=\begin{cases}\frac{|\text{x}-4|}{2(\text{x}-4)},&\text{if x}\neq4\\0,&\text{if x}=4\end{cases} at x = 4.
At x = 4, \text{L.H.L}=\lim\limits_{\text{x}\rightarrow4^-}\frac{|\text{x}-4|}{2(\text{x}-4)}
=\lim\limits_{\text{h}\rightarrow0}\frac{|4-\text{h}-4|}{2\big[(4-\text{h})-4\big]}
=\lim\limits_{\text{h}\rightarrow0}\frac{|-\text{h}|}{-2\text{h}}
=\lim\limits_{\text{h}\rightarrow0}\frac{\text{h}}{-2\text{h}}=\frac{-1}{2}\text{ and f}(4)=0\neq\text{L.H.L}
So, f(x) is discontinuous at x = 4.
Q752. If \text{y}=\log|3\text{x}|,\text{x}\neq0, find \frac{\text{dy}}{\text{dx}}. 3 Marks

Ans: We have, \text{y}=\log|3\text{x}|


\Rightarrow\frac{\text{dy}}{\text{dx}}=\frac{\text{d}}{\text{dx}}(\log|3\text{x}|)
\Rightarrow\frac{\text{dy}}{\text{dx}}=\frac{1}{3\text{x}}\frac{\text{d}}{\text{dx}}(3\text{x})
\Rightarrow\frac{\text{dy}}{\text{dx}}=\frac{1}{3\text{x}}(3)
\Rightarrow\frac{\text{dy}}{\text{dx}}=\frac{1}{\text{x}}
Q753. Verify Lagrange's mean value theorem for the following function on the indicated intervals. find a point 'c' in the indicated interval as stated by the Lagrange's mean value theorem. 3 Marks
\text{f}(\text{x})=\sqrt{\text{x}^2-4}\text{ on }[2,4]

Ans: We have,
\text{f}(\text{x})=\sqrt{\text{x}^2-4}
Here, f(x) will exist,
if
\text{x}^2-4\geq0
\Rightarrow\text{x}\leq-2\text{ or }\text{x}\geq2
Since, for each \text{x}\in2,4, the function f(x) attains a unique definite value.
So, f(x) is continuous on 2, 4
Also,
\text{f}'(\text{x})=\frac{1}{2\sqrt{\text{x}^2-4}}(2\text{x})=\frac{\text{x}}{\sqrt{\text{x}^2-4}}
Exists for all \text{x}\in2,4
So, f(x) is differentiable on 2, 4.
Thus, both the conditions of Lagrange's theorem are satisfied.
Consequently, there exists some \text{c}\in2,4 such that
\text{f}'(\text{x})=\frac{1}{2\sqrt{\text{x}^2-4}}(2\text{x})=\frac{\text{x}}{\sqrt{\text{x}^2-4}}
Now,
\text{f}(\text{x})=\sqrt{\text{x}^2-4}
\text{f}'(\text{x})=\frac{1}{\sqrt{\text{x}^2-4}},\text{f}(4)=2\sqrt3,\text{f}(2)=0
\therefore\ \text{f}'(\text{x})=\frac{\text{f}(4)-\text{f}(2)}{4-2}
\Rightarrow\frac{\text{x}}{\sqrt{\text{x}^2-4}}=\frac{2\sqrt3}{2}
\Rightarrow\frac{\text{x}}{\sqrt{\text{x}^2-4}}=\sqrt3
\Rightarrow\frac{\text{x}^2}{\text{x}^2-4}=3
\Rightarrow\text{x}^2=3\text{x}^2-12
\Rightarrow\text{x}^2=6
\Rightarrow\text{x}=\pm\sqrt6
Thus, \text{c}=\sqrt6\in(2,4) such that \text{f}'(\text{c})=\frac{\text{f}(4)-\text{f}(2)}{4-2}
Hence, Lagrange's theorem is verified.
Q754. If \text{y}=\log_\text{a}\text{x},, find \frac{\text{dy}}{\text{dx}}. 3 Marks

Ans: We have, \text{y}=\log_\text{a}\text{x},


\Rightarrow\text{y}=\frac{\log\text{x}}{\log\text{a}} \Big[\because\log_\text{a}\text{b}=\frac{\log\text{b}}{\log\text{a}}\Big]
\Rightarrow\frac{\text{dy}}{\text{dx}}=\frac{1}{\log\text{a}}\frac{\text{d}}{\text{dx}}(\log\text{x})
\Rightarrow\frac{\text{dy}}{\text{dx}}=\frac{1}{\log\text{a}}\Big(\frac{1}{\text{x}}\Big)
\Rightarrow\frac{\text{dy}}{\text{dx}}=\frac{1}{\text{x}\log\text{a}}
Q755. In the following, find the value of the constant k so that the given function is continuous at the indicated point: 3 Marks
\text{f(x)}=\begin{cases}\text{k}\text{x}^2,&\text{x}\geq1\\4,&\text{x}<1\end{cases}\text{at x} =1

https://bls.smartstudies.co.in/#/exam/pdf-preview/c59cb220-8e86-4716-9ff7-82aec16b1ade/1 144/158
5/26/24, 6:20 PM Exam Automation
Ans: Given,
\text{f(x)}=\begin{cases}\text{k}\text{x}^2,&\text{x}\geq1\\4,&\text{x}<1\end{cases}
We have,
(\text{LHL at x}= 1)=\lim_\limits{\text{x}\rightarrow1^-}\text{f(x)}=\lim_\limits{\text{h}\rightarrow0}\text{f}(1-\text{h})
\lim_\limits{\text{h}\rightarrow0}4=4
(\text{RHL at x}= 1)=\lim_\limits{\text{x}\rightarrow1^+}\text{f(x)}=\lim_\limits{\text{h}\rightarrow0}\text{f}(1+\text{h})
\lim_\limits{\text{h}\rightarrow0}\text{k}(1+\text{h})^2=\text{k}
If f(x) is continuous at x = 1, then
\lim_\limits{\text{x}\rightarrow1^-}\text{f(x)}=\lim_\limits{\text{x}\rightarrow1^+}\text{f(x)}
\Rightarrow\text{k}=4
Q756. If f(0) = f(1) = 0, f'(1) = 1 and y = f(ex) ef(x), write the value of \frac{\text{dy}}{\text{dx}}\text{ at x} = 0. 3 Marks

Ans: Here,
f(0) = f(1) = 0, f'(1) = 2
And, y = f(ex)df(x)
Differentiating ti with respect to x using product rule, chain rule,
\frac{\text{dy}}{\text{dx}}=\frac{\text{d}}{\text{dx}}\big[\text{f}(\text{e}^\text{x})\times\text{e}^{\text{f(x)}}\big]
=\text{f}(\text{e}^\text{x})\frac{\text{d}}{\text{dx}}\text{e}^{\text{f(x)}}+\text{e}^{\text{f(x)}}\frac{\text{d}}{\text{dx}}\text{f}(\text{e}^\text{x})
=\text{f}(\text{e}^\text{x})\text{e}^{\text{f(x)}}\frac{\text{d}}{\text{dx}}\text{f(x)}+\text{e}^{\text{f(x)}}\times\text{f}'(\text{e}^\text{x})\frac{\text{d}}{\text{dx}}(\text{e}^\text{x})
=\text{f}(\text{e}^\text{x})\times\text{e}^{\text{f(x)}}\times\text{f}'\text{(x)}+\text{e}^{\text{f(x)}}+\text{f}'(\text{e}^\text{x})\times\text{e}^\text{x}
Put x = 0
=\text{f}(\text{e}^0)\text{e}^{\text{f}(0)}\text{f}'(0)+\text{e}^{\text{f}(0)}\text{f}^{1}(\text{e}^0)\times\text{e}^0
=\text{f}(1)\text{e}^{\text{f}(0)}\times\text{f}'(0)+\text{e}^{\text{f}(0)}\times\text{f}'(1)\times1
=0\times\text{e}^0\times\text{f}'(0)+\text{e}^02\times1
\big[\text{Since},\text{f}(0)=\text{f}(1)=0,\text{f}'(1)=2\big]
=0+1\times2\times1
=2
So,
\frac{\text{dy}}{\text{dx}}=2
Q757. Find which of the function: 3 Marks
\text{f(x)}=|\text{x}|+|\text{x}-1|\text{ at x}=1

Ans: The function f will be continuous at x = a, if =\lim\limits_{\text{h}\rightarrow\text{a}^-}\text{f(x)}=\lim\limits_{\text{h}\rightarrow\text{a}^+}\text{f(x)}=\text{f(a)}.


Consider, \text{f(x)}=|\text{x}|+|\text{x}-1|\text{ at x}=1
At x = 1, \text{L.H.L}=\lim\limits_{\text{h}\rightarrow1^-}\big[|\text{x}|+|\text{x}-1|\big]
=\lim\limits_{\text{h}\rightarrow0}\big[|1-\text{h}|+|1-\text{h}-1|\big]=1+0=1
At x = 1, \text{R.H.L}=\lim\limits_{\text{h}\rightarrow1^-}\big[|\text{x}|+|\text{x}-1|\big]
=\lim\limits_{\text{h}\rightarrow0}\big[|1+\text{h}|+|1+\text{h}-1|\big]=1+0=1
\text{f}(1)=|1|+|0|=1
Since, L.H.L = R.H.L = f(1)
Hence, f(x) is continuous at x = 1.
Q758. Differentiate the following functions with respect to x: 3 Marks
10^{(10^\text{x})}

Ans: Let \text{y}=10^{(10^\text{x})}\ .....(\text{i})


Taking log on both sides,
\log\text{y}=\log_\text{e}10^{(10^\text{x})}
\log\text{y}=10^{\text{x}}\log_\text{e}10
Differentiating with respect to x,
\Rightarrow\ \frac{1}{\text{y}}\frac{\text{dy}}{\text{dx}}=\log_\text{e}10\times10^\text{x}\log_\text{e}10
\Rightarrow\frac{1}{\text{y}}\frac{\text{dy}}{\text{dx}}=10^\text{x}\times(\log_\text{e}10)^2
\Rightarrow\frac{\text{dy}}{\text{dx}}=\text{y}\big[10^{\text{x}}\times(\log_\text{e}10)^2\big]
\therefore\ \frac{\text{dy}}{\text{dx}}=10^{(10\text{x})}\times10^\text{x}\times(\log_\text{e}10)^2
[Using equation (i)]
Q759. In the following, find the value of the constant k so that the given function is continuous at the indicated point: 3 Marks
\text{f(x)}=\begin{cases}\frac{1-\cos2\text{kx}}{\text{x}^2},&\text{if}\text{ x}\neq0\\8,&\text{if}\text{ x}=0\end{cases}\text{at x}=0

Ans: Given,
\text{f(x)}=\begin{cases}\frac{1-\cos2\text{kx}}{\text{x}^2},&\text{if}\text{ x}\neq0\\8,&\text{if}\text{ x}=0\end{cases}
If f(x) is continuous at x = 0, then
\lim_\limits{\text{x}\rightarrow 0}\text{f(x)}=\text{f}(0)
\Rightarrow\lim_\limits{\text{x}\rightarrow 0}\frac{1-\cos2\text{kx}}{\text{x}^2}=8
\Rightarrow\lim_\limits{\text{x}\rightarrow 0}\frac{2\text{k}^2\sin^2\text{kx}}{\text{k}^2\text{x}^2}=8
\Rightarrow2\text{k}^2\lim_\limits{\text{x}\rightarrow 0}\Big(\frac{\sin\text{kx}}{\text{kx}}\Big)^2=8
\Rightarrow2\text{k}^2\times1=8
\Rightarrow\text{k}^2=4
\Rightarrow\text{k}=\pm2
Q760. Find the second order derivatives of the following functions: 3 Marks
\text{y}=\log(\log\text{x})

Ans: We have,
\text{y}=\log(\log\text{x})
Differentiating w.r.t.x, we get
\frac{\text{dy}}{\text{dx}}=\frac{1}{\log\text{x}}\times\frac{1}{\text{x}}=\frac{1}{\text{x}\log\text{x}}
Differentiating w.r.t.x, we get
\frac{\text{d}^2\text{y}}{\text{dx}^2}=\frac{0-(\log\text{x}+1)}{(\text{x}\log\text{x})^2}=-\frac{(1+\log\text{x})}{(\text{x}\log\text{x})^2}
Q761. In the following, determine the values of constants involved in the definition so that the given function is continuous: 3 Marks
\text{f(x)}=\begin{cases}\text{kx}+5,&\text{if }\text{ x}\leq2\\\text{x}-1,&\text{if }\text{ x}>2\end{cases}

Ans: \text{f(x)}=\begin{cases}\text{kx}+5,&\text{if }\text{ x}\leq2\\\text{x}-1,&\text{if }\text{ x}>2\end{cases}


It is given that the function is continuous
\therefore\ \text{LHL}=\text{RHL}=\text{f}(2)\ ...(\text{i})
\text{LHL}=\lim_\limits{\text{x}\rightarrow2^-}\text{f(x)}=\lim_\limits{\text{h}\rightarrow0}\text{f}(2-\text{h})=\lim_\limits{\text{h}\rightarrow0}\text{k}(2-\text{h})+5=2\text{k}+5
\text{RHL}=\lim_\limits{\text{x}\rightarrow2^+}\text{f(x)}=\lim_\limits{\text{h}\rightarrow0}\text{f}(2+\text{h})=\lim_\limits{\text{h}\rightarrow0}(2+\text{h})-1=1
Thus, using (i) we get
2\text{k}+5=1
\text{k}=-2
Q762. Find \frac{\text{dy}}{\text{ dx}} in the following: 3 Marks
\text{x}^{3}+\text{x}^{2}\text{y} +\text{x} \text{y}^{2}+\text{y}^{3} = 81

https://bls.smartstudies.co.in/#/exam/pdf-preview/c59cb220-8e86-4716-9ff7-82aec16b1ade/1 145/158
5/26/24, 6:20 PM Exam Automation

Ans: The given relationship is \text{x}^{3}+\text{x}^{2}\text{y} +\text{x} \text{y}^{2}+\text{y}^{3} = 81


differenting this relationship with respect to x, we obtain
\frac{\text{d}}{\text{dx}}(\text{x}^{3}+\text{x}^{2}\text{y} +\text{x} \text{y}^{2}+\text{y}^{3}) =\frac{\text{d}}{\text{dx}}(81)
\Rightarrow\frac{\text{d}}{\text{dx}}(\text{x}^{3})+\frac{\text{d}}{\text{dx}}(\text{x}^{2}\text{y})+\frac{\text{d}}{\text{dx}}(\text{x}\text{y}^{2})+\frac{\text{d}}{\text{dx}}\text{(y}^{3}) =0
\Rightarrow 3\text{x}^{2}+\Big[\text{y}\frac{\text{d}}{\text{dx}}(\text{x}^{2})+\text{x}^{2}\frac{\text{dy}}{\text{dx}}\Big] +\Big[\text{y}^{2}\frac{\text{d}}{\text{dx}}
(\text{x})+\text{x}\frac{\text{dy}}{\text{dx}}(\text{y}^{2})\Big]+3\text{y}^{2}\frac{\text{dy}}{\text{dx}}=0
\Rightarrow3\text{x}^{2}+\Big[\text{y}.2\text{x}+\text{x}^{2}\frac{\text{dy}}{\text{dx}}\Big]+\Big[\text{y}^{2}.1+\text{x}.2\text{y}.\frac{\text{dy}}
{\text{dx}}\Big]+3\text{y}^{2}\frac{\text{dy}}{\text{dx}}= 0
\Rightarrow(\text{x}^{2} + 2\text{xy}+3\text{y}^{2})\frac{\text{dy}}{\text{dx}}+(3\text{x}^{2}+2\text{xy}+\text{y}^{2})= 0
\therefore\frac{\text{dy}}{\text{dx}}= \frac{-(3\text{x}^{2}+2\text{xy}+\text{y}^{2})}{(\text{x}^{2}+2\text{xy}+3\text{y}^{2})}
Q763. Differentiate the following functions with respect to x: 3 Marks
\cos(\log\text{ x})^2

Ans: Consider \text{y}=\cos(\log\text{ x})^2


Differentiate it with respect to x and applying the chain and product rule, we get
\frac{\text{dy}}{\text{dx}}=\frac{\text{d}}{\text{dx}}\cos(\log\text{ x})^2
=-\sin(\log\text{x})^2\frac{\text{d}}{\text{dx}}(\log\text{ x})^2
=-\sin(\log\text{x})^2\frac{2\log\text{x}}{\text{x}}
\frac{\text{dy}}{\text{dx}}=\frac{-2\log\text{x}\sin(\log\text{x})^2}{\text{x}}
So, The solution is \frac{\text{dy}}{\text{dx}}=\frac{-2\log\text{x}\sin(\log\text{x})^2}{\text{x}}
Q764. Differentiate the following w.r.t. x: 3 Marks
\sin^{-1}\Big(\frac{1}{\sqrt{\text{x}+1}}\Big)

Ans: Let \text{y}=\sin^{-1}\Big(\frac{1}{\sqrt{\text{x}+1}}\Big)


\therefore\ \frac{\text{dy}}{\text{dx}}=\frac{\text{d}}{\text{dx}}\sin^{-1}\Big(\frac{1}{\sqrt{\text{x}+1}}\Big)
=\frac{1}{\sqrt{-1\Big(\frac{1}{\sqrt{\text{x+1}}}\Big)^2}}\cdot\frac{\text{d}}{\text{dx}}\frac{1}{(\text{x}+1)^{\frac{1}{2}}} \Big[\because\frac{\text{d}}{\text{dx}}(\sin^{-1}\text{x})=\frac{1}
{\sqrt{1-\text{x}^2}}\Big]
=\frac{1}{\sqrt{\frac{\text{x}+1-1}{\text{x}+1}}}\cdot\frac{\text{d}}{\text{dx}}(\text{x+1})^{\frac{-1}{2}}
=\sqrt{\frac{\text{x}+1}{\text{x}}}\cdot\frac{-1}{2}(\text{x}+1)^{\frac{1}{2}-1}\cdot\frac{\text{d}}{\text{dx}}(\text{x+1})
=\frac{(\text{x}+1)^{\frac{1}{2}}}{\text{x}^{\frac{1}{2}}}\cdot\Big(-\frac{1}{2}\Big)(\text{x}+1)^{-\frac{3}{2}}
=\frac{-1}{2\sqrt{\text{x}}}\cdot\Big(\frac{1}{\text{x}+1}\Big)
Q765. Find \frac{\text{dy}}{\text{ dx}} in the following: 3 Marks
\sin^{2}\text{y}+\cos\text{xy}=\pi

Ans: The given relationship is \sin^{2}\text{y}+\cos\text{xy}=\pi


differenting this relationship with respect to x, we obtain
\frac{\text{d}}{\text{dx}}(\sin^{2}\text{y}+\cos\text{xy})=\frac{\text{d}}{\text{dx}}(\pi)
\Rightarrow\frac{\text{d}}{\text{dx}}(\sin^{2}\text{y})+\frac{\text{d}}{\text{dx}}(\cos\text{xy})=0\ ...(\text{i})
Using chain rule, we obtain
\frac{\text{d}}{\text{dx}}(\sin^{2}\text{y})= 2\sin\text{y}\frac{\text{d}}{\text{dx}}(\sin\text{y})=2\sin\text{y}\cos\text{y}\frac{\text{dy}}{\text{dx}} ...\text{(ii)}
\frac{\text{d}}{\text{dx}}(\cos\text{xy})=-\sin\text{xy}\frac{\text{d}}{\text{dx}}(\text{xy})=-\sin\text{xy}\Big[\text{y}\frac{\text{d}}{\text{dx}}(\text{x})+\text{x}\frac{\text{dy}}{\text{dx}}\Big]
=-\sin\text{xy}\Big[\text{y}.1+\text{x}\frac{\text{dy}}{\text{dx}}\Big]= -\text{y}\sin\text{xy}-\text{x}\sin\text{xy}\frac{\text{dy}}{\text{dy}} ...(\text{iii})
From (1), (2) and (3), we obtain
2\sin\text{y}\cos\text{y}\frac{\text{dy}}{\text{dx}}-\text{y}\sin\text{xy}-\text{x}\sin\text{xy}\frac{\text{dy}}{\text{dx}}=0
\Rightarrow(2\sin\text{y}\cos\text{y}-\text{x}\sin\text{xy})\frac{\text{dy}}{\text{dx}}=\text{y}\sin\text{xy}
Q766. Determine the value of the constant k so that the function \text{f(x)}=\begin{cases}\text{kx}^2,&\text{if }\text{ x}\leq2\\3,&\text{if }\text{ x}>2\end{cases} is continuous at x = 2. 3 Marks

Ans: Given, \text{f(x)}=\begin{cases}\text{kx}^2,&\text{if }\text{ x}\leq2\\3,&\text{if }\text{ x}>2\end{cases}


If f(x) is continuous at x = 2, then
\lim\limits_{\text{x}\rightarrow2^-}\text{f(x)}=\lim\limits_{\text{x}\rightarrow2^+}=\text{f}(2)\ ...(\text{i})
Now,
\lim\limits_{\text{x}\rightarrow2^-}\text{f(x)}=\lim\limits_{\text{h}\rightarrow0}\text{f}(2-\text{h})\\=\lim\limits_{\text{h}\rightarrow0}\text{k}(2-\text{h})^2=4\text{k}
And, f(2) = 3
From (i) we have,
4\text{k}=3
\Rightarrow\text{k}=\frac{3}{4}
Q767. Verify Lagrange's mean value theorem for the following function on the indicated intervals. find a point 'c' in the indicated interval as stated by the Lagrange's mean value theorem. 3 Marks
f(x) = x2 - 3x + 2 on [-1, 2]

Ans: We have
f(x) = x2 - 3x + 2
Since a polynomial function is everywhere continuous and differentiable.
Therefore, f(x) is continuous on -1, 2 and differentiable on -1, 2.
Thus, both conditions of Lagrange's mean value theorem are satisfied.
So, there must exist at least one real number \text{c}\in-1,2 such that
\text{f}'(\text{c})=\frac{\text{f}(2)-\text{f}(-1)}{2+1}
\text{f}'(\text{c})=\frac{\text{f}(2)-\text{f}(-1)}{3}
Now,
f(x) = x2 - 3x + 2
⇒ f'(x) = 2x - 3
⇒ f(2) = 0
⇒ f(-1) = (-1)2 - 3(-1) + 2
⇒ f(-1) = 6
\therefore\ \text{f}'(\text{x})=\frac{\text{f}(2)-\text{f}(-1)}{3}
\Rightarrow2\text{x}-3=-2
\Rightarrow2\text{x}-1=0
\Rightarrow\text{x}=\frac{1}{2}
Thus, \text{c}=\frac{1}{2}\in(1,2) such that \text{f}'(\text{c})=\frac{\text{f}(2)-\text{f}(-1)}{2-(-1)}
Hence, Lagrange's mean value theorem is verified.
Q768. If \text{x}=\text{a}(\cos2\text{t}+2\text{t}\sin2\text{t})\ \text{and}\ \text{y}=\text{a}(\sin2\text{t}-2\text{t}\cos2\text{t}), then find \frac{\text{d}^2\text{y}}{\text{dx}^2}. 3 Marks

Ans: \text{x}=\text{a}(\cos2\text{t}+2\text{t}\sin2\text{t})
\frac{\text{dx}}{\text{dt}}=-2\text{a}\sin2\text{t}+2\text{a}\sin2\text{t}+4\text{at}\cos2\text{t}=4\text{at}\cos2\text{t}
\text{y}=\text{a}(\sin2\text{t}-2\text{t}\cos2\text{t})
\frac{\text{dy}}{\text{dt}}=2\text{a}\cos2\text{t}-2\text{a}\cos2\text{t}+4\text{at}\sin2\text{t}=4\text{at}\sin2\text{t}
\frac{\text{dy}}{\text{dx}}=\tan2\text{t}
\frac{\text{d}^2\text{y}}{\text{dx}^2}=\frac{\text{d}}{\text{dx}}(\tan2\text{t})
\frac{\text{d}^2\text{y}}{\text{dx}^2}=2\sec^22\text{t}\frac{\text{d}}{\text{dx}}(\text{t})
\frac{\text{d}^2\text{y}}{\text{dx}^2}=2\sec^22\text{t}\times\frac{1}{4\text{at}\cos2\text{t}}

https://bls.smartstudies.co.in/#/exam/pdf-preview/c59cb220-8e86-4716-9ff7-82aec16b1ade/1 146/158
5/26/24, 6:20 PM Exam Automation
\frac{\text{d}^2\text{y}}{\text{dx}^2}=\frac{1}{2\text{a}}\sec^32\text{t}
Q769. Find \frac{\text{dy}}{\text{dx}} when x and y are connected by the relation: 3 Marks
\big(\text{x}^2+\text{y}^2\big)^2=\text{xy}

Ans: We have, \big(\text{x}^2+\text{y}^2\big)^2=\text{xy}


On differentiating both sides w.r.t. x, we get
\frac{\text{d}}{\text{dx}}\big(\text{x}^2+\text{y}^2\big)^2=\frac{\text{d}}{\text{dx}}(\text{xy})
\Rightarrow\ 2\big(\text{x}^2+\text{y}^2\big)^2\cdot\frac{\text{d}}{\text{dx}}(\text{x}^2+\text{y}^2)=\text{x}\cdot\frac{\text{d}}{\text{dx}}\text{y}+\text{y}\cdot\frac{\text{d}}
{\text{dx}}\text{x}
\Rightarrow\ 2(\text{x}^2+\text{y}^2)\cdot\Big(2\text{x}+2\text{y}\frac{\text{dy}}{\text{dy}}\Big)=\text{x}\frac{\text{dy}}{\text{dx}}+\text{y}
\Rightarrow\ 2\text{x}^2\cdot2\text{x}+2\text{x}^2\cdot2\text{y}\frac{\text{dy}}{\text{dx}}+2\text{y}^2\cdot2\text{x}+2\text{y}^2\cdot2\text{y}\frac{\text{dy}}
{\text{dx}}=\text{x}\frac{\text{dy}}{\text{dx}}+\text{y}
\Rightarrow\ \frac{\text{dy}}{\text{dx}}\big[4\text{x}^2\text{y}+4\text{y}^3-\text{x}\big]=\text{y}-4\text{x}^3-4\text{xy}^2
\therefore\ \frac{\text{dy}}{\text{dx}}=\frac{(\text{y}-4\text{x}^3-4\text{xy}^2)}{(4\text{x}^2\text{y}+4\text{y}^3-\text{x})}
Q770. \text{If y}=(\tan ^1 \text{x})^2,\text{show that }(\text{x}^2+1)^2 \ \text{y}_2+ 2\text{x}(\text{x}^2+1)_\text{y}=2 3 Marks

Ans: \text{y}=(\tan^{-1}\text{x})^2\ \dots(1)


\therefore\ \frac{\text{dy}}{\text{dx}}=2 \tan ^{-1}\text{x}.\frac{1}{1+\text{x}^2}
\Rightarrow \ (1+ \text{x}^2) \frac{\text{dy}}{\text{dx}} =2 \tan^{-1} \text{x}
\Rightarrow\ (1+\text{x}^2)^2\bigg(\frac{\text{dy}}{\text{dx}}\bigg)^2=4 (\tan ^{-1}\text{x})^2
\Rightarrow\ (1+\text{x}^2)^2 \bigg(\frac{\text{dy}}{\text{dx}}\bigg)^2= 4\text{y}\ \ [\because\text{of } (1)]
Differenitiating both sides w.r.t.x, we get,
(1 +\text{x}^2)^2.2 \frac{\text{dy}}{\text{dx}}\frac{\text{d}^2\text{y}}{\text{dx}^2}+\bigg(\frac{\text{dy}}{\text{dx}}\bigg)^2.2(1+\text{x}^2).2 \text{x}=4 \frac{\text{dy}}{\text{dx}}
Divide both sides by 2 \frac{\text{dy}}{\text{dx}}, we get,
(1+\text{x}^2)^2.\frac{\text{d}^2\text{y}}{\text{dx}^2}+2 \text{x}(1+\text{x}^2)\frac{\text{dy}}{\text{dx}}=2
\text{Or}\ \ (\text{x}{^2}+1)\text{y}_2+2 \text{x}(\text{x}^2+1)\text{y}_1=2
Q771. If \text{y}=\cos^{-1}\text{x}, Find \frac{\text{d}^2\text{y}}{\text{dx}^2} in terms of y alone. 3 Marks

Ans: Here,
\text{y}=\cos^{-1}\text{x},
Differentiating w.r.t.x, we get
\frac{\text{dy}}{\text{dx}}=\frac{1}{\sqrt{1-\text{x}^2}}
Differentiating w.r.t.x, we get
\frac{\text{d}^2\text{y}}{\text{dx}^2}=\frac{-2\text{x}}{2\sqrt{1-\text{x}^2}^\frac{3}{2}}=\frac{-\text{x}}{(1-\text{x}^2)}
Now,
\text{y}=\cos^{-1}\text{x}
\Rightarrow\text{x}=\cos\text{y}
\Rightarrow\frac{\text{d}^2\text{y}}{\text{dx}^2}=\frac{-\cos\text{y}}{(1-\cos^2\text{y})^\frac{3}{2}}=-\frac{\cos\text{y}}{(\sin^2\text{y})^\frac{3}{2}}=-\cot\text{y}\ \text{cosec}^2\text{y}
Q772. If y = xx, find \frac{\text{dy}}{\text{dx}}\text{at x}=\text{e} 3 Marks

Ans: We have, y = xx .....(i)


Taking log on both sides,
\log\text{y}=\log\text{x}^\text{x}
\Rightarrow\log\text{y}=\text{x}\log\text{x}
\Rightarrow\frac{1}{\text{y}}\frac{\text{dy}}{\text{dx}}=\text{x}\frac{\text{d}}{\text{dx}}(\log\text{x})+\log\text{x}\frac{\text{d}}{\text{dx}}(\text{x})
\Rightarrow\frac{1}{\text{y}}\frac{\text{dy}}{\text{dx}}=\text{x}\Big(\frac{1}{\text{x}}\Big)+\log\text{x}(1)
\Rightarrow\frac{1}{\text{y}}\frac{\text{dy}}{\text{dx}}=1+\log\text{x}
\Rightarrow\frac{\text{dy}}{\text{dx}}=\text{y}(1+\log\text{x})
\Rightarrow\frac{\text{dy}}{\text{dx}}=\text{x}^\text{x}(1+\log\text{x})
[Using equation (i)]
Putting x = e, we get,
\frac{\text{dy}}{\text{dx}}=\text{e}^\text{e}(1+\log_\text{e}\text{e})
\Rightarrow\frac{\text{dy}}{\text{dx}}=\text{e}^\text{e}(1+1)\big[\because\log_\text{e}\text{e}=1\big]
\Rightarrow\frac{\text{dy}}{\text{dx}}=2\text{e}^\text{e}
Q773. Differentiate the following functions with respect to x: 3 Marks
\sin^{-1}\Big(\frac{1-\text{x}^2}{1+\text{x}^2}\Big)+\sec^{-1}\Big(\frac{1+\text{x}^2}{1-\text{x}^2}\Big),\text{x}\in\text{R}

Ans: Let \text{y}=\sin^{-1}\Big(\frac{1-\text{x}^2}{1+\text{x}^2}\Big)+\sec^{-1}\Big(\frac{1+\text{x}^2}{1-\text{x}^2}\Big)


\Rightarrow\text{y}=\sin^{-1}\Big(\frac{1-\text{x}^2}{1+\text{x}^2}\Big)+\cos^{-1}\Big(\frac{1+\text{x}^2}{1-\text{x}^2}\Big)
\Big[\text{Since},\sec^{-1}\text{x}=\cos^{-1}\big(\frac{1}{\text{x}}\big)\Big]
\Rightarrow \text{y}=\frac{\pi}{2}\Big[\text{Since}, \sin^{-1}\text{x}+\cos^{-1}\text{x}=\frac{\pi}{2}\Big]
Differentiate it with respect to x,
\therefore \frac{\text{dy}}{\text{dx}}=0
Q774. If ex+y - x = 0, prove that \frac{\text{dy}}{\text{dx}}=\frac{1-\text{x}}{\text{x}} 3 Marks

Ans: Here,
ex+y - x = 0
ex+y = x .....(i)
Differentiating it with respect to x using chain rule,
\frac{\text{d}}{\text{dx}}\big(\text{x}^{\text{x}+\text{y}}\big)=\frac{\text{d}}{\text{dx}}(\text{x})
\text{e}^{\text{x}+\text{y}}\frac{\text{d}}{\text{dx}}(\text{x}+\text{y})=1
\text{x}\Big[1+\frac{\text{dy}}{\text{dx}}\Big]=1
[Using euqation (i)]
1+\frac{\text{dy}}{\text{dx}}=\frac{1}{\text{x}}
\frac{\text{dy}}{\text{dx}}=\frac{1}{\text{x}}-1
\frac{\text{dy}}{\text{dx}}=\frac{1-\text{x}}{\text{x}}
Q775. Differentiate the following w.r.t. x: 3 Marks
(\sin\text{x})^{\cos\text{x}}

Ans: Let \text{y}=(\sin\text{x})^{\cos\text{x}}


\Rightarrow\ \log\text{y}=\log(\sin\text{x})^{\cos\text{x}}=\cos\text{x}\log(\sin\text{x})
Differentiate both sides w.r.t.x, we get
\Rightarrow\ \frac{1}{\text{y}}\cdot\frac{\text{d}}{\text{dx}}=\cos\text{x}\cdot\frac{\text{d}}{\text{dx}}(\log\sin\text{x})+\log\sin\text{x }\frac{\text{d}}{\text{dx}}(\cos\text{x})
=\cos\text{x}\cdot\frac{1}{\sin\text{x}}\cdot\frac{\text{d}}{\text{dx}}(\sin\text{x})+\log\sin\text{x}\cdot(-\sin\text{x})
=\cos\text{x}\cdot\cos\text{x}-\log(\sin\text{x})\cdot\sin\text{x}
\therefore\ \frac{\text{dy}}{\text{dx}}=\text{y}\big[\cot\text{x}\cos\text{x}-\sin\text{x}\cdot\log(\sin\text{x})\big]
=(\sin\text{x})^{\cos\text{x}}\big[\cot\text{x}\cos\text{x}-\sin\text{x}\cdot\log(\sin\text{x})\big]
Q776. Find the value of k in this question, so that the function f is continuous at the indicated point: 3 Marks
\text{f(x)}=\begin{cases}3\text{x}-8,&\text{if x}\leq5\\2\text{k},&\text{if x}>5\end{cases} at x = 5.

https://bls.smartstudies.co.in/#/exam/pdf-preview/c59cb220-8e86-4716-9ff7-82aec16b1ade/1 147/158
5/26/24, 6:20 PM Exam Automation
Ans: We have, \text{f(x)}=\begin{cases}3\text{x}-8,&\text{if x}\leq5\\2\text{k},&\text{if x}>5\end{cases} at x = 5.
Since, f(x) is continuous at x = 5.
\therefore L.H.L = R.H.L = f(5)
Now, \text{L.H.L}=\lim\limits_{\text{h}\rightarrow5^-}(3\text{x}-8)=\lim\limits_{\text{h}\rightarrow0}[(5-\text{h})-8]
=\lim\limits_{\text{h}\rightarrow0}\ [15-3\text{h}-8]=7
\text{R.H.L}=\lim\limits_{\text{h}\rightarrow5^+}2\text{k}=\lim\limits_{\text{h}\rightarrow0}2\text{k}=2\text{x}=7 [\because\ \text{L.H.L}=\text{R.H.L}]
And f(5) = 3 × 5 - 8 = 7
2\text{k}=7\Rightarrow\ \text{k}=\frac{7}{2}
Q777. If \text{y}=\text{x}\sin\text{y}, prove that \frac{\text{dx}}{\text{dx}}=\frac{\sin^2\text{y}}{(1-\text{x}\cos\text{y})} 3 Marks

Ans: Here,
\text{y}=\text{x}\sin\text{y}
Differentiate with respect to x,
\Rightarrow\frac{\text{dy}}{\text{dx}}=\frac{\text{d}}{\text{dx}}(\text{x}\sin\text{y})
\Rightarrow\frac{\text{dy}}{\text{dx}}=\text{x}\frac{\text{d}}{\text{dx}}(\sin\text{y})+\sin\text{y}\frac{\text{d}}{\text{dx}}(\text{x})
[Using product rule]
\Rightarrow\frac{\text{dy}}{\text{dx}}=\text{x}\cos\frac{\text{dy}}{\text{dx}}+\sin\text{y}(1)
\Rightarrow\frac{\text{dy}}{\text{dx}}(1-\text{x}\cos\text{y})=\sin\text{y}
\Rightarrow\frac{\text{dy}}{\text{dx}}=\frac{\sin\text{y}}{1-\text{x}\cos\text{y}}
Q778. Verify Lagrange's mean value theorem for the following function on the indicated intervals. find a point 'c' in the indicated interval as stated by the Lagrange's mean value theorem. 3 Marks
f(x) = 2x2 - 3x + 1 on [1, 3]

Ans: Here,
f(x) = 2x2 - 3x + 1 on [1, 3]
We know that a polynomial function is continuous and differentiable.
So, f(x) is continuous in [1, 3] and f(x) differentiable in (1, 3).
So, Lagrange's mean value theorem is applicable.
So, there must exist at least one real number \text{c}\in(1,3) such that
\text{f}'(\text{c})=\frac{\text{f}(3)-\text{f}(-1)}{3-1}
\Rightarrow4\text{c}-3=\frac{(2(3)^2-3(3)+1)-(2-3+1)}{3-1}
\Rightarrow4\text{c}-3=\frac{10}{2}
\Rightarrow4\text{c}=5+3
\Rightarrow4\text{c}=8
\Rightarrow\text{c}=2\in(1,3)
Hence, Lagrange's mean value theorem is verified.
Q779. If \text{y}=\text{x}+\tan\text{x}, show that \cos^2\text{x}\frac{\text{d}^2\text{y}}{\text{dx}^2}-2\text{y}+2\text{x}=0 3 Marks

Ans: \text{y}=\text{x}+\tan\text{x},
differentiating w.r.t.x, we get
\Rightarrow\frac{\text{dy}}{\text{dx}}=1+\sec^2\text{x}
differentiating w.r.t.x, we get
\Rightarrow\frac{\text{d}^2\text{y}}{\text{dx}^2}=0+2\sec^2\times\tan\text{x}
\Rightarrow\frac{\text{d}^2\text{y}}{\text{dx}^2}=\frac{2\sin\text{x}}{\cos^3\text{x}}
\Rightarrow\cos^2\times\frac{\text{d}^2\text{y}}{\text{dx}^2}=2\tan\text{x}+2\text{x}-2\text{x}
\Rightarrow\cos^2\times\frac{\text{d}^2\text{y}}{\text{dx}^2}=2(\text{x}+\tan\text{x})-2\text{x}
\Rightarrow\cos^2\times\frac{\text{d}^2\text{y}}{\text{dx}^2}=2\text{y}-2\text{x}
\Rightarrow\cos^2\times\frac{\text{d}^2\text{y}}{\text{dx}^2}-2\text{y}+2\text{x}=0
Q780. If x and y are connected parametrically by the equations given in Exercise without eliminating the parameter, Find \frac{\text{dy}}{\text{dx}}. 3 Marks
\text{x}=\text{a}(\cos\theta+\theta\sin\theta),\text{y}=\text{a}(\sin\theta-\theta\cos\theta)

Ans: The given equations are \text{x}=\text{a}(\cos\theta+\theta\sin\theta)\text{ and y}=\text{a}(\sin\theta-\theta\cos\theta)


Then, \frac{\text{dx}}{\text{d}\theta}= \text{a}\Big[\frac{\text{d}}{\text{d}\theta}\cos\theta+\frac{\text{d}}{\text{d}\theta}(\theta\sin\theta)\Big] =\text{a}\Big[-
\sin\theta+\theta\frac{\text{d}}{\text{d}\theta}(\sin\theta)+\sin\theta\frac{\text{d}}{\text{d}\theta}(\theta)\Big]
=\text{a}[-\sin\theta+\theta\cos\theta+\sin\theta]=\text{a}\theta\cos\theta
\frac{\text{dy}}{\text{d}\theta}=\text{a}\Big[\frac{\text{d}}{\text{d}\theta}(\sin\theta)-\frac{\text{d}}{\text{d}\theta}(\theta\cos\theta)\Big] =\text{a}\Big[\cos\theta-\Big\
{\theta\frac{\text{d}}{\text{d}\theta}(\cos\theta)+\cos\theta.\frac{\text{d}}{\text{d}\theta}(\theta)\Big\}\Big]
=\text{a}[\cos\theta+\theta\sin\theta-\cos\theta]
=\text{a}\theta\sin\theta
\therefore\ \frac{\text{dy}}{\text{dx}}=\frac{\Big(\frac{\text{dy}}{\text{d}\theta}\Big)}{\Big(\frac{\text{dx}}{\text{d}\theta}\Big)}=\frac{\text{a}\theta\sin\theta}
{\text{a}\theta\cos\theta}=\tan\theta
Q781. If f(x) is an even function, then write whether f'(x) is even of odd. 3 Marks

Ans: Here,
f(x) is even function, so
f(-x) = f(x)
Differentiating it with respect to x,
\frac{\text{d}}{\text{dx}}(\text{f}(-\text{x}))=\frac{\text{d}}{\text{dx}}(\text{f}(\text{x}))
\text{f}'(-\text{x})\frac{\text{d}}{\text{dx}}(-\text{x})=\text{f}'\text{(x)}
\text{f}'(-\text{x})\times(-1)=\text{f}'(\text{x})
-\text{f}'(-\text{x})=\text{f}'(\text{x})
\text{f}'(-\text{x})=-\text{f}'(\text{x})
So,
f'(x) is odd function.
Q782. If \text{x}=\text{a}\cos\text{nt}-\text{b}\sin\text{nt} and \frac{\text{d}^2\text{x}}{\text{dt}^2}=\lambda\text{x} then find the value of \lambda. 3 Marks

Ans: we have
\text{x}=\text{a}\cos\text{nt}-\text{b}\sin\text{nt}
\Rightarrow\frac{\text{dx}}{\text{dt}}=-\text{a}\sin(\text{nt})\times\text{n}-\text{bn}\cos)\text{nt}
\Rightarrow\frac{\text{d}^2\text{y}}{\text{dt}^2}=-\text{an}^2\cos(\text{nt})+\text{bn}^2\sin(\text{nt})
since, \frac{\text{d}^2\text{y}}{\text{dt}^2}=\lambda\text{x}
\Rightarrow-\text{an}^2\cos(\text{nt})+\text{bn}^2\sin(\text{nt})=\lambda(\text{a}\cos\text{nt}-\text{b}\sin\text{nt})
\Rightarrow\lambda=\text{n}^2
Q783. Find \frac{\text{dy}}{\text{dx}} in the following cases: 3 Marks
\frac{\text{x}^2}{\text{a}^2}+\frac{\text{y}^2}{\text{b}^2}=1

Ans: Given,
\frac{\text{x}^2}{\text{a}^2}+\frac{\text{y}^2}{\text{b}^2}=1
Differentiating with resepct to x,
\frac{\text{d}}{\text{dx}}\Big(\frac{\text{x}^2}{\text{a}^2}+\frac{\text{y}^2}{\text{b}^2}\Big)=\frac{\text{d}}{\text{dx}}(1)

https://bls.smartstudies.co.in/#/exam/pdf-preview/c59cb220-8e86-4716-9ff7-82aec16b1ade/1 148/158
5/26/24, 6:20 PM Exam Automation
\Rightarrow\frac{\text{d}}{\text{dx}}\Big(\frac{\text{x}^2}{\text{a}^2}\Big)+\frac{\text{d}}{\text{dx}}\Big(\frac{\text{y}^2}{\text{b}^2}\Big)=0
\Rightarrow\frac{1}{\text{a}^2}(2{\text{x}})+\frac{1}{\text{b}^2}(2\text{y})\frac{\text{d}}{\text{dx}}=0
\Rightarrow\frac{2\text{y}}{\text{b}^2}\frac{\text{dy}}{\text{dx}}=-\frac{2{\text{x}}}{\text{a}^2}
\Rightarrow\frac{\text{dy}}{\text{dx}}=-\Big(\frac{2{\text{x}}}{\text{a}^2}\Big)\Big(\frac{\text{b}^2}{2\text{y}}\Big)
\Rightarrow\frac{\text{dy}}{\text{dx}}=-\frac{{\text{b}^2\text{x}}}{\text{a}^2\text{y}}
Q784. If \text{y}=\sqrt{\cos\text{x}+\sqrt{\cos\text{x}+\sqrt{\cos\text{x}+\ .... \text{to }\infty}}}, prove that \frac{\text{dy}}{\text{dx}}=\frac{\sin\text{x}}{1-2\text{y}} 3 Marks

Ans: Here,
\text{y}=\sqrt{\cos\text{x}+\sqrt{\cos\text{x}+\sqrt{\cos\text{x}+\ .... \text{to }\infty}}}
\text{y}=\sqrt{\cos\text{x}+\text{y}}
Squaring both the sides,
\text{y}^2=\cos\text{x}+\text{y}
Differentiating it with respect to x,
2\text{y}\frac{\text{dy}}{\text{dx}}=-\sin\text{x}+\frac{\text{dy}}{\text{dx}}
\frac{\text{dy}}{\text{dx}}(2\text{y}-1)=-\sin\text{x}
\frac{\text{dy}}{\text{dx}}=\frac{-\sin\text{x}}{(2\text{y}-1)}
\frac{\text{dy}}{\text{dx}}=\frac{\sin\text{x}}{1-2\text{y}}
Q785. If \text{y}=\text{x}+\text{e}^\text{x}, find \frac{\text{d}^2\text{x}}{\text{dy}^2}. 2 Marks

Ans: Here,
\text{y}=\text{x}+\text{e}^\text{x}
\Rightarrow\frac{\text{dy}}{\text{dx}}=1+\text{e}^\text{x}
\Rightarrow\frac{\text{dy}}{\text{dx}}=\frac{1}{1+\text{e}^\text{x}}
\Rightarrow\frac{\text{d}^2\text{y}}{\text{dx}^2}=\frac{-\text{e}^\text{x}}{(1+\text{e}^\text{x})^2}
\frac{\text{dx}}{\text{dy}}=-\frac{-\text{e}^\text{x}}{(1+\text{e}^\text{x})^3}
Q786. Find \frac{\text{dx}}{{\text{dy}}} in the following: 2 Marks
2\text{x} + 3\text{y} = \sin\text{y}

Ans: The given relationship is 2\text{x} + 3\text{y} = \sin\text{y}


Differentiating this relationship with respect to x, we obtain
\frac{\text{d}}{\text{dx}}(2\text{x}) + \frac{\text{d}}{\text{dx}}(3\text{y)} =\frac{\text{d}}{\text{dx}}( \sin\text{y})
\Rightarrow 2 + 3\frac{\text{dy}}{\text{dx}}= \cos\text{y}\frac{\text{dy}}{\text{dx}} [By using chain rule]
\Rightarrow 2 = (\cos\text{y} - 3) \frac{\text{dy}}{\text{dx}}
\therefore\ \frac{\text{dy}}{\text{dx}}=\frac{2}{ \cos\text{y} - 3}
Q787. If \text{y}=1-\text{x}+\frac{\text{x}^2}{2!}-\frac{\text{x}^3}{3!}+\frac{\text{x}^4}{4!}+...\infty then write \frac{\text{d}^2\text{y}}{\text{dx}^2} in terms of y. 2 Marks

Ans: Here,
\text{y}=1-\text{x}+\frac{\text{x}^2}{2!}-\frac{\text{x}^3}{3!}+\frac{\text{x}^4}{4!}+...\infty
Thus
\Rightarrow\frac{\text{dy}}{\text{dx}}=-1+\frac{2\text{x}}{2!}-\frac{3\text{x}^2}{3!}+\frac{4\text{x}^3}{4!}...\infty
=-1+\text{x}-\frac{\text{x}^2}{2!}+\frac{\text{x}^3}{3!}...\infty
\Rightarrow\frac{\text{d}^2\text{y}}{\text{dx}^2}=1-\frac{2\text{x}}{2!}+\frac{3\text{x}^2}{3!}-\frac{4\text{x}^3}{4!}+...\infty
\Rightarrow\frac{\text{d}^2\text{y}}{\text{dx}^2}=1-\frac{2\text{x}}{2!}+\frac{3\text{x}^2}{3!}-\frac{4\text{x}^3}{4!}+...\infty
=1-\text{x}+\frac{\text{x}^2}{2!}-\frac{\text{x}^3}{3!}+...\infty
=\text{y}
Q788. If \text{f(x)}=\begin{cases}\frac{\sin^{-1}\text{x}}{\text{x}},&\text{x}\neq0\\\text{k},&\text{x}=0\end{cases} is continuous at x = 0, write the value of k. 2 Marks

Ans: Given, \text{f(x)}=\begin{cases}\frac{\sin^{-1}\text{x}}{\text{x}},&\text{x}\neq0\\\text{k},&\text{x}=0\end{cases}


If f(x) is continuous at x = 0, then
\lim\limits_{{\text{x}}\rightarrow0}\text{f(x})=\text{f}(0)
\Rightarrow\lim\limits_{{\text{x}}\rightarrow0}\text{f(x})\Big(\frac{\sin^{-1}\text{x}}{\text{x}}\Big)=\text{f}(0)
\Rightarrow\lim\limits_{{\text{x}}\rightarrow0}\Big(\frac{\sin^{-1}\text{x}}{\text{x}}\Big)=\text{k}
\Rightarrow\text{k}=1 \bigg[\because\ \lim\limits_{{\text{x}}\rightarrow0}\Big(\frac{\sin^{-1}\text{x}}{\text{x}}\Big)=1\bigg]
Q789. If \text{y}=\cot\text{x} show that \frac{\text{d}^2\text{y}}{\text{dx}^2}+2\text{y}\frac{\text{dy}}{\text{dx}}=0 2 Marks

Ans: \text{y}=\cot\text{x}
Differentiating w.r.t.x
\Rightarrow\frac{\text{dy}}{\text{dx}}=-\text{cosec}^2\text{x}
Differentiating w.r.t.x
\Rightarrow\frac{\text{d}^2\text{y}}{\text{dx}^2}=-[2\text{cosec}\text{ x}(-\text{cosec}^2\times\cot\text{x})]=-2\frac{\text{dy}}{\text{dx}}.\text{y}
\Rightarrow\frac{\text{d}^2\text{y}}{\text{dx}^2}+2\text{y}\frac{\text{dy}}{\text{dx}}=0
Hence proved.
Q790. Find \frac{\text{dx}}{{\text{dy}}} in the following: 2 Marks
\text{ax} + \text{by}^{2} = \cos\text{y}

Ans: The given relationship is \text{ax} + \text{by}^{2} = \cos\text{y}


Differentiating this relationship with respect to x, we obtain
\frac{\text{d}}{\text{dx}}(\text{ax}) + \frac{\text{d}}{\text{dx}}(\text{by}^{2}) =\frac{\text{d}}{\text{dx}}( \cos\text{y})
\Rightarrow \text{a + b}\frac{\text{d}}{\text{dx}}(\text{y}^{2}) =\frac{\text{d}}{\text{dx}}( \cos\text{y}) ...(\text{i})
Using chain rule, we obtain \frac{\text{d}}{\text{dx}}(\text{y}^{2}) = 2\text{y}\frac{\text{dy}}{\text{dx}}\ \text{and}\ \frac{\text{d}}{\text{dx}}( \cos\text{y}) = -\sin\text{y}\frac{\text{dy}}
{\text{dx}} ...{\text{(ii)}}
Form (i) and (ii), we obtain
\text{a + b}\times 2\text{y}\frac{\text{dy}}{\text{dx}} = -\sin\text{y}\frac{\text{dy}}{\text{dx}}
\Rightarrow(2\text{by} + \sin\text{y})\frac{\text{dy}}{\text{dx}} = -\text{a}
\therefore\frac{\text{dy}}{\text{dx}} =\frac{\text{-a}}{2\text{by} + \sin\text{y}}
Q791. Find the second order derivatives of the function given in Exercise: 2 Marks
\text{x }\cos\text{x}

Ans: Let \text{y}=\text{x}\cos\text{x}


\therefore\ \frac{\text{dy}}{\text{dx}}=\text{x}(-\sin\text{x})+\cos\text{x}.1=-\text{x}\sin\text{x}+\cos\text{x}
and \frac{\text{d}^2\text{y}}{\text{dx}^2}=-\text{x}\cos\text{x}-\sin\text{x}.1-\sin\text{x}=-\text{x}\cos\text{x}-2\sin\text{x}
Q792. If x = at2, y = 2at, then find \frac{\text{d}^2\text{y}}{\text{dx}^2}. 2 Marks

Ans: Given,
x = at2 and y = 2at
On differentiating both sides w.r.t.t, we get,
\frac{\text{dx}}{\text{dt}}=2\text{at and}\frac{\text{dy}}{\text{dt}}=2\text{a}
Therefore,
\frac{\text{dy}}{\text{dx}}=\frac{2\text{a}}{2\text{at}}=\frac{1}{\text{t}}
Now, \frac{\text{d}^2\text{y}}{\text{dx}^2}=\frac{\text{d}}{\text{dx}}\Big(\frac{\text{dy}}{\text{dx}}\Big)\times\frac{\text{dt}}{\text{dx}}

https://bls.smartstudies.co.in/#/exam/pdf-preview/c59cb220-8e86-4716-9ff7-82aec16b1ade/1 149/158
5/26/24, 6:20 PM Exam Automation
=\frac{\text{d}}{\text{dt}}\Big(\frac{1}{\text{t}}\Big)\times\frac{1}{2\text{at}}=-\frac{1}{\text{t}^2}\times\frac{1}{\text{2at}}=-\frac{1}{\text{2at}^3}
Q793. If f(x) = x + 1 then write the value of \frac{\text{d}}{\text{dx}}\text{ fof }\text{(x)}. 2 Marks

Ans: Here,
f(x) = x + 1
(fof)(x) = f(f(x))
= f(x + 1)
= (x + 1) + 1
(fof)(x) = x + 2
Differentiating it with respect to x,
\frac{\text{d}}{\text{dx}}(\text{fof})\text{(x)}=\frac{\text{d}}{\text{dx}}\text{(x)}+\frac{\text{d}}{\text{dx}}(2)
=1+0
\frac{\text{d}}{\text{dx}}\text{(fof)}\text{(x)}=1
Q794. Find the second order derivatives of the function given in Exercise: 2 Marks
\log(\log\text{x})

Ans: Let \text{y}=\log(\log\text{x})


\therefore\ \frac{\text{dy}}{\text{dx}}=\frac{1}{\log\text{x}}.\frac{1}{\text{x}}=\frac{1}{\text{x}\log\text{x}}
\therefore\ \frac{\text{d}^2\text{y}}{\text{dx}^2}=\frac{(\text{x}\log\text{x}).\frac{\text{d}}{\text{dx}}(1)-1.\frac{\text{d}}{\text{dx}}(\text{x}\log\text{x})}{(\text{x}\log\text{x})^2}
=\frac{(\text{x}\log\text{x}).0-1.\Big(\text{x}.\frac{1}{\text{x}}+\log\text{x}.1\Big)}{(\text{x}\log\text{x})^2}
=\frac{0-(1+\log\text{x})}{(\text{x}\log\text{x})}
=-\frac{(1+\log\text{x})}{(\text{x}\log\text{x})^2}
Q795. Find \frac{\text{dy}}{\text{dx}}, when 2 Marks
\text{x}=\text{a}(1-\cos\theta)\text{ and y}=\text{a}(\theta+\sin\theta)\text{ at }\theta=\frac{\pi}{1}

Ans: We have, \text{x}=\text{a}(1-\cos\theta)\text{ and y}=\text{a}(\theta+\sin\theta)


\therefore\frac{\text{dx}}{\text{d}\theta}=\frac{\text{d}}{\text{d}\theta}[\text{a}(1-\cos\theta)]=\text{a}(\sin\theta)
and
\frac{\text{dx}}{\text{d}\theta}=\frac{\text{d}}{\text{d}\theta}[\text{a}(\theta+\sin\theta)]=\text{a}(1+\cos\theta)
\therefore\Big[\frac{\text{dy}}{\text{dx}}\Big]_{\theta=\frac{\pi}{2}}=\bigg[\frac{\frac{\text{dy}}{\text{d}\theta}}{\frac{\text{dx}}{\text{d}\theta}}\bigg]_{\theta=\frac{\pi}{2}} \\
=\Big[\frac{\text{a}(1+\cos\theta)}{\text{a}(\sin\theta)}\Big]_{\theta=\frac{\pi}{2}}=\frac{\text{a}(1+0)}{\text{a}}=1
Q796. If x and y are connected parametrically by the equations given in Exercise without eliminating the parameter, Find \frac{\text{dy}}{\text{dx}}. 2 Marks
\text{x}=\sin\text{t, y}=\cos2\text{t}

Ans: The given equations are \text{x}=\sin\text{ t and y}=\cos=2\text{t}


Then, \frac{\text{dx}}{\text{dt}}=\frac{\text{d}}{\text{dt}}(\sin\text{t)}=\cos\text{t}
\frac{\text{dy}}{\text{dt}}=\frac{\text{d}}{\text{dt}}(\cos2\text{t})=-\sin2\text{t}.\frac{\text{d}}{\text{dt}}(2\text{t})=-2\sin2\text{t}
\therefore\ \frac{\text{dy}}{\text{dx}}=\frac{\Big(\frac{\text{dy}}{\text{dt}}\Big)}{\Big(\frac{\text{dx}}{\text{dt}}\Big)}=\frac{-2\sin2\text{t}}{\cos\text{t}}=\frac{-2.2\sin\text{t}\cos\text{t}}
{\cos\text{t}}=-4\sin\text{t}
Q797. Form the differential equation representing the family of curves y2 = m(a2 - x2) by eliminating the arbitrary constants ‘m’ and ‘a’. 2 Marks

Ans: The given equation is \text{y}^2 =\text{m}\big(\text{a}^2-\text{x}^2\big), where m and a are arbitrary constants.
\text{y}^2=\text{m}\big(\text{a}^2-\text{x}^2\big)\ \dots\text{(i)}
Differentiate (i) w.r.t. x,
2\text{y}\frac{\text{dy}}{\text{dx}}=-2 \text{mx}\ \dots(\text{ii})
\Rightarrow -2\text{m}=2\frac{\text{y}}{\text{x}}\frac{\text{dy}}{\text{dx}}
Differentiate (ii) w.r.t. x,
2\begin{bmatrix}\text{y}\frac{\text{d}^2\text{y}}{\text{dx}^2}+\Big(\frac{\text{dy}}{\text{dx}}\Big)^2\end{bmatrix}=-2\text{m}\ \dots\text{(iii)}
From (ii)and (iii), we get
2\begin{bmatrix}\text{y}\frac{\text{d}^2\text{y}}{\text{dx}^2}+\Big(\frac{\text{dy}}{\text{dx}}\Big)^2\end{bmatrix}=2\frac{\text{y}}{\text{x}}\frac{\text{dy}}{\text{dx}}
\text{y}\frac {\text{d}^2\text{y}}{\text{dx}^2}+\Big(\frac{\text{dy}}{\text{dx}}\Big)^2 -\Big(\frac{\text{y}}{\text{x}}\Big)\frac{\text{dy}}{\text{dx}}=0
Therefore, the required differential equation is \text{y}\frac{\text{d}^2\text{y}}{\text{dx}^2}+\Big(\frac{\text{dy}}{\text{dx}}\ \Big)^2- \Big(\frac{\text{y}}{\text{x}}\Big)\frac{\text{dy}}
{\text{dx}}=0
Q798. Differentiate the following with respect to x: 2 Marks
\cos^{-1}(\sin\text{ x})

Ans: Let \text{f(x)}=\cos^{-1}(\sin\text{x})


We observe that this function is defined for all real numbers.
\text{f(x)}=\cos^{-1}(\sin\text{x})
=\cos^{-1}\Big[\cos\Big(\frac{\pi}{2}-\text{x}\Big)\Big]=\frac{\pi}{2}-\text{x}
Thus, \text{f(x)}=\frac{\text{d}}{\text{dx}}\Big(\frac{\pi}{2}-\text{x}\Big)=-1
Q799. if f(1) = 4, f'(1) = 2, find the value of the derivative of \log\Big(\text{f}\big(\text{e}^\text{x}\big)\Big) w.r.t x at the point x = 0. 2 Marks

Ans: We have, f(1) = 4 and f'(1) = 2


Let \text{y}=\log\big\{\text{f}\big(\text{e}^\text{x}\big)\big\}
\Rightarrow\frac{\text{dy}}{\text{dx}}=\frac{\text{d}}{\text{dx}}\Big[\log\big\{\text{f}\big(\text{e}^\text{x}\big)\big\}\Big]
\Rightarrow\frac{\text{dy}}{\text{dx}}=\frac{1}{\text{f}\big(\text{e}^\text{x}\big)}\times\frac{\text{d}}{\text{dx}}\big\{\text{f}\big(\text{e}^\text{x}\big)\big\}
\Rightarrow\frac{\text{dy}}{\text{dx}}=\frac{1}{\text{f}(\text{e}^\text{x})}\times\text{f}'\big(\text{e}^\text{x}\big)\times\frac{\text{d}}{\text{dx}}\big(\text{e}^\text{x}\big)
\Rightarrow\frac{\text{dy}}{\text{dx}}=\frac{\text{e}^\text{x}\text{f}'\big(\text{e}^\text{x}\big)}{\text{f}\big(\text{e}^\text{x}\big)}
Putting x = 0, we get,
\Rightarrow\frac{\text{dy}}{\text{dx}}=\frac{\text{e}^0\text{f}'\big(\text{e}^0\big)}{\text{f}\big(\text{e}^0\big)}
\Rightarrow\frac{\text{dy}}{\text{dx}}=\frac{1\text{f}'(1)}{\text{f}(1)}
\Rightarrow\frac{\text{dy}}{\text{dx}}=\frac{2}{4}\big[\because\text{f}'(1)=2\text{ and f}(1)=4\big]
\Rightarrow\frac{\text{dy}}{\text{dx}}=\frac{1}{2}
Q800. If \text{f(x)}=\log_\text{e}(\log_\text{e}\text{x}), then write the value of f'(e). 2 Marks

Ans: We have, \text{f(x)}=\log_\text{e}(\log_\text{e}\text{x})


Differentiating with respect to x,
\text{f}'\text{(x)}=\frac{1}{\log_\text{e}\text{x}}\frac{\text{d}}{\text{dx}}(\log_\text{e}\text{x})
\Rightarrow\text{f}'\text{(x)}=\frac{1}{\log_\text{e}\text{x}}\Big(\frac{1}{\text{x}}\Big)
\Rightarrow\text{f}'\text{(e)}=\frac{1}{\log_\text{e}\text{e}}\Big(\frac{1}{\text{e}}\Big) \big[\because\text{x}=\text{e}\big]
\Rightarrow\text{f}'\text{(e)}=\frac{1}{\text{x}}\big[\because\log_\text{e}=1\big]
Q801. Examine the continuity of the function f(x) = 2x2 – 1 at x = 3. 2 Marks

Ans: Here
f(x) = 2x2 - 1
^{\ \ \text{Lt}}_{\text{x}\rightarrow{3}}\text{f(x)}=^{\ \ \text{Lt}}_{\text{x}\rightarrow{3}}(2\text{x}^3-1) = 2(3)^2 - 1
= 2(9) - 1 = 18 - 1 = 17
Now f is defined at x = 3
and f(x) = 2(3)2 - 1 = 2(9) - 1 = 18 - 1 = 17

https://bls.smartstudies.co.in/#/exam/pdf-preview/c59cb220-8e86-4716-9ff7-82aec16b1ade/1 150/158
5/26/24, 6:20 PM Exam Automation
\therefore\ \ \text{Lt}\ \ \ \ \text{f(x)} = \text{f(3)} = 17\\ \ \ \ \ \text{x}\rightarrow3
\therefore f is continous at x = 3.
Q802. Differentiate the following w.r.t.x: \frac{\text{e}^\text{x}}{\sin\text{x}} 2 Marks

Ans: \text{Let}\ \text{y}=\frac{\text{e}^\text{x}}{\sin\text{x}}


\therefore\frac{\text{dy}}{\text{dx}}=\frac{\sin\text{x}.\frac{\text{d}}{\text{dx}}(\text{e}^{\text{x}})-\text{e}^{\text{x}}.\frac{\text{d}}{\text{dx}}(\sin\text{x})}{\sin^{2}\text{x}}
=\frac{\sin\text{x}.\text{e}^{\text{x}}-\text{e}^{\text{x}}.\cos\text{x}}{\sin^{2}\text{x}}=\text{e}^{\text{x}}\bigg(\frac{\sin\text{x}-\cos\text{x}}{\sin^{2}\text{x}}\bigg)
Q803. Find the values of k so that the function f is continuous at the indicated point: 2 Marks
\text{f(x)}=\begin{cases}\text{k}\text{x}+1,&\text{if}\ \text{x}\leq{5}\\3\text{x} - 5,& \text{if}\ \text{x}> 5\end{cases}
\text{at} \text{x} = 5

Ans: Here \text{f(x)}=\begin{cases}\text{k}\text{x}+1,&\text{if}\ \text{x}\leq{5}\\3\text{x} - 5,& \text{if}\ \text{x}> 5\end{cases}


^{\ \ \text{Lt}}_{\text{x}\rightarrow\text{5}^{-}}\text{f(x)}=^{\ \ \text{Lt}}_{\text{x}\rightarrow\text{5}^{-}}(\text{k}\text{x} + 1) = 5\text{k} + 1
^{\ \ \text{Lt}}_{\text{x}\rightarrow\text{5}^{+}}\text{f(x)}=^{\ \ \text{Lt}}_{\text{x}\rightarrow\text{5}^{+}}({3}\text{x} - 5) = 15 - 5 = 10
Since f is continuous at x = 1
\therefore\ ^{\ \ \text{Lt}}_{\text{x}\rightarrow\text{5}^{-}}\text{f(x)}=^{\ \ \text{Lt}}_{\text{x}\rightarrow\text{5}^{+}}\text{f(x)}
\therefore 5 \text{k} + 1 = 10 \Rightarrow 5\text{k} = 9 \Rightarrow\text{k} = \frac{9}{5}
Q804. Discuss the applicability of the Rolle's theorem for the following function on the indicated interval 2 Marks
\text{f}(\text{x})=2\text{x}^2-5\text{x}+3\text{ on }[1,3]

Ans: The given function \text{f}(\text{x})=2\text{x}^2-5\text{x}+3\text{ on }[1,3].


The domain of f is given to be (1, 3).
It is a polynomial function.
Thus, it is everywhere derivable and hence continuous.
But
f(1) = 0 and f(3) = 6
\Rightarrow\text{f}(3)\neq\text{f}(1)
Hence, Rolle's theorem is not applicable for the given function.
Q805. If \text{x}=\text{t}^2 and \text{y}=\text{t}^3 then find \frac{\text{d}^2\text{y}}{\text{dx}^2} 2 Marks

Ans: Here,
\text{x}=\text{t}^2\ \text{and}\ \text{y}=\text{t}^3
\Rightarrow\frac{\text{dx}}{\text{dt}}=2\text{t}\ \text{and}\ \frac{\text{dy}}{\text{dt}^2}=3\text{t}^2
\therefore\frac{\text{dy}}{\text{dx}}=\frac{3\text{t}}{2}
\Rightarrow\frac{\text{d}^2\text{y}}{\text{dx}^2}=\frac{3}{2}\frac{\text{dt}}{\text{dx}}=\frac{3}{4\text{t}}
Q806. If \frac{\pi}{2}\leq\text{x}\leq\frac{3\pi}{2} and \text{y}=\sin^{-1}(\sin\text{x}), find \frac{\text{dy}}{\text{dx}}. 2 Marks

Ans: Here,
\text{y}=\sin^{-1}(\sin\text{x}),\text{x}\in\Big[\frac{\pi}{2},\frac{3\pi}{1}\Big]
\Big[\text{Since},\sin^{-1}(\sin\text{x})=\text{x},\text{if x}\in\Big[-\frac{\pi}{2},\frac{\pi}{2}\Big]\Big]
\text{y}=\pi-\text{x}
Differentiating it with respect to x,
\frac{\text{dy}}{\text{dx}}=\frac{\text{d}}{\text{dx}}(\pi-\text{x})
0-1
\frac{\text{dy}}{\text{dx}}=-1
Q807. If |x| < 1 and \text{y}=1+\text{x}+\text{x}^2+\ .....\ \text{to}\ \infty, then find the value of \frac{\text{dy}}{\text{dx}}. 2 Marks

Ans: We have, \text{y}=1+\text{x}+\text{x}^2+\ .....\ \text{to}\ \infty,


\Rightarrow\text{y}=\frac{1}{1-\text{x}}
[\because It is a G.P with first term 1 and common ration x]
\Rightarrow\frac{\text{dy}}{\text{dx}}=\frac{\text{d}}{\text{dx}}\Big(\frac{1}{1-\text{x}}\Big)
\Rightarrow\frac{\text{dy}}{\text{dx}}=-\frac{1}{(1-\text{x})^2}\frac{\text{d}}{\text{dx}}(1-\text{x})
\Rightarrow\frac{\text{dy}}{\text{dx}}=-\frac{1}{(1-\text{x})^2}(-1)
\Rightarrow\frac{\text{dy}}{\text{dx}}=-\frac{1}{(1-\text{x})^2}
Q808. If \text{x}=2\text{ at},\text{y}=\text{at}^2, where a is a constant, then find \frac{\text{d}^2\text{y}}{\text{dx}^2}\text{ at}\text{ x}=\frac{1}{2}. 2 Marks

Ans: Here,
\text{x}=2\text{at}\ \text{and}\ \text{y}=\text{at}^2
Differentiating w.r.t.t, we get
\frac{\text{dx}}{\text{dt}}=2\text{a}\ \text{and}\ \frac{\text{dy}}{\text{dt}}=2\text{at}
\therefore\frac{\text{dy}}{\text{dx}}=\frac{2\text{at}}{2\text{a}}=\text{t}
Differentiating w.r.t.t, we get
\frac{\text{d}^2\text{y}}{\text{dx}^2}=1\times\frac{\text{dt}}{\text{dx}}=\frac{1}{2\text{a}}
Now \Big[\frac{\text{d}^2\text{y}}{\text{dx}^2}\Big]_{\text{x}=\frac{1}{2}}=\frac{1}{2\text{a}}
Q809. Find \frac{\text{dy}}{\text{ dx}} in the following: 2 Marks
\sin^{2}\text{x}+\cos^{2}\text{y}=1

Ans: The given relationship is \sin^{2}\text{x}+\cos^{2}\text{y}=1


differenting this relationship with respect to x, we obtain
\frac{\text{d}}{\text{dx}}(\sin^{2}\text{x}+\cos^{2}\text{y})=\frac{\text{d}}{\text{dx}}(1)
\Rightarrow\frac{\text{d}}{\text{dx}}(\sin^{2}\text{x})+\frac{\text{d}}{\text{dx}}(\cos^{2}\text{y})=0
\Rightarrow 2\sin\text{x}.\frac{\text{d}}{\text{dx}}(\sin\text{x})+2\cos\text{y}.\frac{\text{d}}{\text{dx}}(\cos\text{y})=0
\Rightarrow2\sin\text{x}\cos\text{x}+2\cos\text{y}(-\sin\text{y}).\frac{\text{dy}}{\text{dx}}=0
\Rightarrow\sin2\text{x}-\sin2\text{y}\frac{\text{dy}}{\text{dx}}=0
\therefore\frac{\text{dy}}{\text{dx}}=\frac{\sin2\text{x}}{\sin2\text{y}}
Q810. Differentiate the functions with respect to x. 2 Marks
\sin(\text{ax + b})

Ans: \text{Let y} = \sin(\text{ax + b})


\therefore \frac{\text{dy}}{\text{dx}} = \cos(\text{ax + b})\frac{\text{d}}{\text{dx}}\text{(ax + b)}
= \cos(\text{ax + b)}\text{(a + 0)} = \text{a}\cos\text{(ax + b)}
Q811. Find the value of =\frac{\text{dy}}{\text{dx}} at \theta=\frac{\pi}{3}, if \text{x}=\cos\theta-\cos2\theta,\ \text{y}=\sin\theta-\sin2\theta. 2 Marks

Ans: \frac{\text{dx}}{\text{d}\theta}=\sin\theta+2\sin\ 2\theta


\frac{\text{dy}}{\text{d}\theta}=\cos\theta-2\cos2\theta
\therefore\frac{\text{dy}}{\text{dx}}=\frac{\cos\theta-2\cos2\theta}{-\sin\theta+2\sin2\theta}
\therefore\frac{\text{dy}}{\text{dx}}\Big|_{\theta-\frac{\pi}{3}}=\sqrt{3}
Q812. If \text{y}=\sin^{-1}\Big(\frac{1-\text{x}^2}{1+\text{x}^2}\Big)+\cos^{-1}\Big(\frac{1-\text{x}^2}{1+\text{x}^2}\Big), find \frac{\text{dy}}{\text{dx}}. 2 Marks

Ans: Here,
\text{y}=\sin^{-1}\Big(\frac{1-\text{x}^2}{1+\text{x}^2}\Big)+\cos^{-1}\Big(\frac{1-\text{x}^2}{1+\text{x}^2}\Big)
\Big[\text{Since},\sin^{-1}\text{x}+\cos^{-1}=\frac{\pi}{2}\Big]
https://bls.smartstudies.co.in/#/exam/pdf-preview/c59cb220-8e86-4716-9ff7-82aec16b1ade/1 151/158
5/26/24, 6:20 PM Exam Automation
So,
\text{y}=\frac{\pi}{2}
Differentiating it with respect to x,
\frac{\text{dy}}{\text{dx}}=0
Q813. If \text{x}=\text{f}(\text{t}) and \text{y}=\text{g}(\text{t}), then write the value of \frac{\text{d}^2\text{y}}{\text{dx}^2}. 2 Marks

Ans: We are given


\text{x}=\text{f}(\text{t})
\text{Y}=\text{g}(\text{t})
Then \frac{\text{dy}}{\text{dx}}=\frac{\text{dy}}{\text{dt}}\times\frac{\text{dt}}{\text{dx}}=\frac{\text{g}'(\text{t})}{\text{f}'(\text{t})}
\therefore\frac{\text{d}^2\text{y}}{\text{dx}^2}=\frac{\text{d}}{\text{dx}}\frac{\text{f}'(\text{t})\text{g}'\text({t})-\text{g}'(\text{t})\text{f}''\text({t})}{[\text{f}'(\text{t})]^3}
Q814. Is every continuous function differentiable? 2 Marks

Ans: No, function may be continuous at a point but may not be differentiable at that point .
For example: function f(x) = |x| is continuous at x = 0 but it is not differentiable at x = 0.
Q815. For what value of \lambda is the function defined by 2 Marks
\text{f(x)}= \begin{cases}\lambda(\text{x}^{2} - 2\text{x}), \text{if}\ \text{x} \leq0\\ \text{4x} + 1,\ \ \ \ \ \ \ \text{if}\ \text{x} > 0\end{cases}

Ans: \text{f(x)}= \begin{cases}\lambda(\text{x}^{2} - 2\text{x}), \text{if}\ \text{x} \leq0\\ \text{4x} + 1,\ \ \ \ \ \ \ \text{if}\ \text{x} > 0\end{cases}
At x = 0
^{\ \ \text{Lt}}_{\text{x}\rightarrow\text{0}^{-}}\text{f(x)} = ^{\ \ \text{Lt}}_{\text{x}\rightarrow\text{0}^{-}}\lambda(\text{x}^{2} - \text{2x}) = \lambda(0 - 0) = 0
^{\ \ \text{Lt}}_{\text{x}\rightarrow\text{0}^{+}}\text{f(x)} = ^{\ \ \text{Lt}}_{\text{x}\rightarrow\text{0}^{+}}(4\text{x} + 1) = 4(0) + 1 = 0 + 1 = 1
\therefore \ ^{\ \ \text{Lt}}_{\text{x}\rightarrow\text{0}^{+}}\text{f(x)} \neq ^{\ \ \text{Lt}}_{\text{x}\rightarrow\text{0}^{+}}\text{f(x })
At x = 1
^{\ \ \text{Lt}}_{\text{x}\rightarrow\text{1}}\text{f(x)} = ^{\ \ \text{Lt}}_{\text{x}\rightarrow\text{1}}(4\text{x} + 1) = 4 + 1 = 5
Also f(1) = 4 + 1 = 5
\therefore\ ^{\ \ \text{Lt}}_{\text{x}\rightarrow\text{c}}\text{f(x)} = \text{f(1)}
\therefore f is continuous at x = 1 whatever value of \lambda be.
Q816. 12 cards numbered 1 to 12 (one number on one card), are placed in a box and mixed up thoroughly. Then a card is drawn at random from the box. If it is known that the number on the 2 Marks
drawn card is greater than 5, find the probability that the card bears an odd number.

Ans: A: card bears odd number


B: Number on the card is greater than 5
\text{A}\cap \text{B}\text{ {7, 9, 1}}
\text{P}\Big(\frac{\text{A}}{\text{B}}\Big)=\frac{\text{P}(\text{A}\cap\text{B})}{\text{P}(\text{B})}=\frac{\frac{3}{12}}{\frac{7}{2}}=\frac{3}{7}
Q817. If \text{A}=\begin{bmatrix}-3& 6\\-2&4\end{bmatrix} then show that A3 = A. 2 Marks

Ans: \text{A}^2=\begin{bmatrix}-3&6\\-2&4\end{bmatrix}\begin{bmatrix}-3&6\\-2&4\end{bmatrix}=\begin{bmatrix}-3&6\\-2&4\end{bmatrix}=\text{A}
\Rightarrow\text{A}^3=\text{A}^2.\text{A}=\text{A}.\text{A}=\text{A}^2=\text{A}
Q818. Find: \int\limits_\text{a}^\text{b}\frac{\log\text{x}}{\text{x}}\text{dx} 2 Marks

Ans: Put \log\text{x}=\text{t}


\Rightarrow\frac{1}{\text{x}}\text{dx}=\text{dt}
Where a and b are loga and logb
\therefore\text{I}=\int\limits^{\log\text{b}}_{\log\text{a}}\text{t}\text{dt}
=\Big[\frac{\text{t}^2}{2}\Big]^{\log\text{b}}_{\log\text{a}}
=\frac{1}{2}\Big[(\log\text{b})^2-(\log\text{a})^2\big]
Q819. If x and y are connected parametrically by the equations given in Exercise without eliminating the parameter, Find \frac{\text{dy}}{\text{dx}}. 2 Marks
\text{x}=4\text{t, y}=\frac{4}{\text{t}}

Ans: The given equations are \text{x}=4\text{t, and y}=\frac{4}{\text{t}}


\frac{\text{dx}}{\text{dt}}=\frac{\text{d}}{\text{dt}}(4\text{t)}=4
\frac{\text{dy}}{\text{dt}}=\frac{\text{d}}{\text{dt}}\Big(\frac{4}{\text{t}}\Big)=4.\frac{\text{d}}{\text{dt}}\Big(\frac{1}{\text{t}}\Big)=4.\Big(\frac{-1}{\text{t}^2}\Big)=\frac{-4}{\text{t}^2}
\therefore\ \frac{\text{dy}}{\text{dx}}=\frac{\Big(\frac{\text{dy}}{\text{dt}}\Big)}{\Big(\frac{\text{dx}}{\text{dt}}\Big)}=\frac{\Big(\frac{-4}{\text{t}^2}\Big)}{4}=\frac{-1}{\text{t}^2}
Q820. If \text{f(x)}=\begin{cases}\frac{1-\cos\text{x}}{\text{x}^2},&\text{x}\neq0\\\text{k},&\text{x}=0\end{cases} is continuous at x = 0, find k. 2 Marks

Ans: Since f(x) is continuous at x = 0,


\text{f}(0)=\lim\limits_{{\text{x}}\rightarrow0}\text{f(x})
\Rightarrow\text{k}=\lim\limits_{{\text{x}}\rightarrow0}\frac{1-\cos\text{x}}{\text{x}^2}
\Rightarrow\text{k}=\lim\limits_{{\text{x}}\rightarrow0}\frac{2\sin^2\frac{\text{x}}{2}}{\text{x}^2}
\Rightarrow\text{k}=\frac{1}{2}
Q821. Write the points of non-differentiability of f(x) = |log |x||. 2 Marks

Ans: Here,
f(x) = |log |x||
f(x) will always positive and let two points x = 1 and x = -1
f(x) = 0
The function f(x) = |log |x|| is not differentiable at x = -1 and 1.
Q822. If \text{y}=\mid\log_\text{e}\text{x}\mid find \frac{\text{d}^2\text{y}}{\text{dx}^2} 2 Marks

Ans: Here,
\text{y}=\mid\log_\text{e}\text{x}\mid
=\begin{cases}-\log_\text{e}\text{x}&\text{if}&0<\text{x}< 1\\\log_\text{e}\text{x}&\text{if}&\text{x}>1\end{cases}
Differentiating w.r.t.x, we get
\frac{\text{dy}}{\text{dx}}=\begin{cases}\frac{-1}{\text{x}}&\text{if}&0<\text{x}<1\\\frac{1}{\text{x}}&\text{if}&\text{x}>1\end{cases}
Differentiating w.r.t.x, we get
\frac{\text{d}^2\text{y}}{\text{dx}^2}=\begin{cases}\frac{1}{\text{x}^2}&\text{if}&0<\text{x}<1\\\frac{-1}{\text{x}^2}&\text{if}&\text{x}>1 \end{cases}
Q823. If x and y are connected parametrically by the equations given in Exercise without eliminating the parameter, Find \frac{\text{dy}}{\text{dx}}. 2 Marks
\text{x}=\text{a}\cos\theta,\text{y}=\text{b}\cos\theta

Ans: The given equations are \text{x}=\text{a}\cos\theta\text{ and y}=\text{b}\cos\theta


Then, \frac{\text{dx}}{\text{d}\theta}=\frac{\text{d}}{\text{d}\theta}(\text{a}\cos\theta)=\text{a}(-\sin\theta)=-\text{a}\sin\theta
\frac{\text{dy}}{\text{d}\theta}=\frac{\text{d}}{\text{d}\theta}(\text{b}\cos\theta)=\text{b}(-\sin\theta)=-\text{b}\sin\theta
\therefore\ \frac{\text{dy}}{\text{dx}}=\frac{\Big(\frac{\text{dy}}{\text{d}\theta}\Big)}{\Big(\frac{\text{dx}}{\text{d}\theta}\Big)}=\frac{-\text{b}\sin\theta}{-\text{a}\sin\theta}=\frac{\text{b}}
{\text{a}}
Q824. Find the second order derivatives of the function given in Exercise: 2 Marks
\text{x}^3\log\text{x}

Ans: Let \text{y}=\text{x}^3\log\text{x}


\therefore\ \frac{\text{dy}}{\text{dx}}=\text{x}^3.\frac{1}{\text{x}}+\log\text{x}.3\text{x}^2=\text{x}^2+3\text{x}^2\log\text{x}=\text{x}^2(1+3\log\text{x})
\therefore\ \frac{\text{d}^2\text{y}}{\text{dx}^2}=\text{x}^2.\frac{3}{\text{x}}+(1+3\log\text{x}).2\text{x}
=3\text{x}+2\text{x}(1+3\log\text{x})=\text{x}[3+2(1+3\log\text{x})]

https://bls.smartstudies.co.in/#/exam/pdf-preview/c59cb220-8e86-4716-9ff7-82aec16b1ade/1 152/158
5/26/24, 6:20 PM Exam Automation
=\text{x}[3+2+6\log\text{x}]=\text{x}(5+6\log\text{xz})
Q825. \text{If y}=\text{sin}^{-1}(6\text{x}\sqrt{1-9\text{x}^2}), -\frac{1}{3\sqrt{2}}<\text{x}<\frac{1}{3\sqrt{2}},\text{then find}\frac{\text{dy}}{dx} 2 Marks

Ans: \text{y}=\text{sin}^{-1}(6\text{x}\sqrt{1-9\text{x}^2}),-\frac{1}{3\sqrt2}<\text{x}<\frac{1}{3\sqrt2}
\text{put 3x}=\text{sin}\ \theta
\Rightarrow\theta=\text{sin}^{-1}3\text{x}
\text{y}=\text{sin}^{-1}(\text{sin}\ 2\theta)
=2\theta
=2\ \text{sin}^{-1}3\text{x}
\therefore\ \frac{\text{dy}}{\text{dx}}=\frac{6}{\sqrt{1-9\text{x}^2}}
Q826. If \text{f(x)}=\begin{cases}\frac{\text{x}^2-16}{\text{x}-4},&\text{if }\text{ x}\neq4\\\text{k},&\text{if }\text{ x}=4\end{cases} is continuous at x = 4, find k. 2 Marks

Ans: Given, \text{f(x)}=\begin{cases}\frac{\text{x}^2-16}{\text{x}-4},&\text{if }\text{ x}\neq4\\\text{k},&\text{if }\text{ x}=4\end{cases}


If f(x) is continuous at x = 4, then
\lim\limits_{{\text{x}}\rightarrow4}\text{f(x})=\text{f(4)}
\Rightarrow\lim\limits_{{\text{x}}\rightarrow4}\Big(\frac{\text{x}^2-16}{\text{x}-4}\Big)=\text{k}
\Rightarrow\lim\limits_{{\text{x}}\rightarrow4}\frac{(\text{x}+4)(\text{x}-4)}{(\text{x}-4)}=\text{k}
\Rightarrow\lim\limits_{{\text{x}}\rightarrow4}(\text{x}+4)=\text{k}
\Rightarrow\text{k}=8
Q827. Find the derivative of \text{x}^{\log\text{x}} w.r.t. \log\text{x}. 2 Marks

Ans: Let \text{u}=\text{x}^{\log\text{x}} and \text{v}=\log\text{x}


Now, \log\text{u}=(\log\text{x})^2
\Rightarrow\frac{1}{\text{u}}\frac{\text{du}}{\text{dx}}=2\log\text{x}.\frac{1}{\text{x}}
\Rightarrow\frac{\text{du}}{\text{dx}}=\frac{2\log\text{x}}{\text{x}}\text{x}^{\log\text{x}}
Again \text{v}=\log\text{x}\Rightarrow\frac{\text{dv}}{\text{dx}}\frac{1}{\text{x}}
\therefore\frac{\text{du}}{\text{dv}}=2\text{x}^{\log\text{x}}\log\text{x}
Q828. For what value of \lambda is the function \text{f(x)}=\begin{cases}\lambda(\text{x}^2-2\text{x}),&\text{if }\text{ x}\leq0\\4\text{x}+1,&\text{if }\text{ x}>0\end{cases} continuous at x = 0? 2 Marks
What about continuity at \text{x}=\pm1?

Ans: The given function f is \text{f(x)}=\begin{cases}\lambda(\text{x}^2-2\text{x}),&\text{if }\text{ x}\leq0\\4\text{x}+1,&\text{if }\text{ x}>0\end{cases}


If f is continuous at x = 0, then
\lim_\limits{\text{x}\rightarrow0^-}\text{f(x)}=\lim_\limits{\text{x}\rightarrow0^+}\text{f(x)}=\text{f}(0)
\Rightarrow\lim_\limits{\text{x}\rightarrow0^-}\lambda(\text{x}^2-2\text{x})=\lim_\limits{\text{x}\rightarrow0^+}(4\text{x}+1)=\lambda(0^2-2\times0)
\Rightarrow\lambda(0^2-2\times0)=4\times0+1=0
\Rightarrow0=1=0 which is not possible
Therefore, there is no value of \lambda for which f(x) is continuous at x = 0
At x = 1
\Rightarrow\text{f}(1)=4\text{x}+1=4\times1+1=5
\Rightarrow\lim_\limits{\text{x}\rightarrow1}(4\text{x}+1)=4\times1+1=5
\therefore\ \lim_\limits{\text{x}\rightarrow1}\text{f(x)}=\text{f}(1)
Therefore, for any value of \lambda, f is continuous at x = 1
At x = -1 we have,
\Rightarrow\text{f}(-1)=\lambda(1+2)=3\lambda
\Rightarrow\lim_\limits{\text{x}\rightarrow-1}\lambda(1+2)=3\lambda
\therefore\ \lim_\limits{\text{x}\rightarrow-1}\text{f(x)}=\text{f}(-1)
Therefore, for any values of \lambda, f is continuous at x = -1
Q829. Show that the function f defined by f(x) = (x – 1) ex + 1 is an increasing function for all x > 0. 2 Marks

Ans: \text{f(x)}=(\text{x}-1)\text{e}^{\text{x}}+1
\text{f}'\text{(x)}=(\text{x}-1)\text{e}^{\text{x}}+\text{e}^{\text{x}}
=\text{x}\text{e}^{\text{x}}-\text{e}^{\text{x}}+\text{e}^{\text{x}}
=\text{x}\text{e}^{\text{x}}
Given: x > 0
We know,
=\text{e}^{\text{x}}>0
\Rightarrow\text{x}\text{e}^{\text{x}}>0
={\text{f}'\text{(x)}}>0,\forall{\text{x}}>0
So, f(x) is increasing on for all x > 0.
Q830. Differentiate (x2 – 5x + 8) (x3 + 7x + 9) in three ways mentioned below: 2 Marks
by expanding the product to obtain a single polynomial.

Ans: y = (x2 - 5x + 8) (x3 + 7x + 9)


\Rightarrow\ \text{y}=\text{x}^5+7\text{x}^3+9\text{x}^2-5\text{x}^4-35\text{x}^2-45\text{x}+8\text{x}^3+56\text{x}+72
\Rightarrow\ \text{y}=\text{x}^5-5\text{x}^4+15\text{x}^3-26\text{x}^2+11\text{x}+72
\frac{\text{dy}}{\text{dx}}=5\text{x}^4-20\text{x}^3+45\text{x}^2-52\text{x}+11\ \dots\text{(iii)}
Q831. Differentiate the functions with respect to x. 2 Marks
\cos(\sqrt{\text{x})}

Ans: \text{Let y} =\cos(\sqrt{\text{x})}


\therefore \frac{\text{dy}}{\text{dx}}= -\sin\sqrt{\text{x}}\frac{\text{d}}{\text{dx}}\sqrt{\text{x}} = -\sin\sqrt{\text{x}}.\frac{1}{2}(\text{x})^{\frac{-1}{2}}=\frac{-\sin\sqrt{\text{x}}}
{2\sqrt{\text{x}}}
Q832. Differentiate w.r.t. x the function in Exercise: 2 Marks
(3\text{x}^2-9\text{x}+5)^9

Ans: Let \text{y}=(3\text{x}^2-9\text{x}+5)^9


Using chain rule, we obtain
\frac{\text{dy}}{\text{dx}}=\frac{\text{d}}{\text{dx}}(3\text{x}^2-9\text{x}+5)^9
=9(3\text{x}^2-9\text{x}+5)^8.\frac{\text{d}}{\text{dx}}(3\text{x}^2-9\text{x}+5)
=9(3\text{x}^2-9\text{x}+5)^8.(6\text{x}-9)
=9(3\text{x}^2-9\text{x}+5)^8.3(2\text{x}-3)
=27(3\text{x}^2-9\text{x}+5)^8.3(2\text{x}-3)
Q833. Find the second order derivatives of the following functions: 2 Marks
\log(\sin\text{x})

Ans: We have,
\text{y}=\log(\sin\text{x})
Differentiating w.r.t.x, we get
\frac{\text{dy}}{\text{dx}}=\frac{1}{\sin\text{x}}\times\cos\text{x}=\cot\text{x}
Differentiating w.r.t.x, we get

https://bls.smartstudies.co.in/#/exam/pdf-preview/c59cb220-8e86-4716-9ff7-82aec16b1ade/1 153/158
5/26/24, 6:20 PM Exam Automation
\frac{\text{d}^2\text{y}}{\text{dx}^2}=-\text{cosec}^2\text{x}
Q834. If \text{y}=\sin^{-1}(\sin\text{x}),-\frac{\pi}{2}\leq\text{x}\leq\frac{\pi}{2}. Then, wrrite tha value of \frac{\text{dy}}{\text{dx}}\text{ for x}\in\Big(-\frac{\pi}{2},\frac{\pi}{2}\Big). 2 Marks

Ans: We have, \text{y}=\sin^{-1}(\sin\text{x})


\Rightarrow\text{y}=\text{x}
\Big[\because\sin^{-1}(\sin\text{x})=\text{x},\text{ if x}\in\Big[-\frac{\pi}{2},\frac{\pi}{2}\Big]\Big]
\Rightarrow\frac{\text{dy}}{\text{dx}}=\frac{\text{d}}{\text{dx}}(\text{x})
\Rightarrow\frac{\text{dy}}{\text{dx}}=1
Q835. Verify Rolle’s theorem for the function \text{f(x)}=\text{x}^2+2\text{x}-8,\text{x}\in[-4,2]. 2 Marks

Ans: Consider \text{f(x)}=\text{x}^2+2\text{x}-8,\text{x}\in[-4,\ 2]


Function is continuous in [-4,2] as it is a polynomial function and polynomial function is always continuous.
\text{f}'\text{(x)}=2\text{x}+2, \text{f}'\text{(x)} exists in [-4, 2], hence derivable.
\text{f}(-4)=0\text{ and f }(2)=0
\therefore\ \text{f}(-4)=\text{f}(2)
Conditions of Rolle’s theorem are satisfied, hence there exists, at least one \text{c}\in(-4,2) such that \text{f}'\text{(c)}=0
\Rightarrow\ 2\text{c}+2=0\ \Rightarrow\ \text{c}=-1
Q836. If P(not A) = 0.7, P(B) = 0.7 and \text{P}\Big(\frac{\text{B}}{\text{A}}\Big)=0.5, then find \text{P}\Big(\frac{\text{A}}{\text{B}}\Big). 2 Marks

Ans: \text{P}(\bar{\text{A}})=0.7
\Rightarrow1-\text{P(A)}=0.7
\Rightarrow\text{P(A)}=0.3
\text{P}(\text{A}\cap\text{B})=\text{P(A)}\cdot\text{P}\Big(\frac{\text{B}}{\text{A}}\Big)
\text{P}(\text{A}\cap\text{B})=0.3\times0.5
\text{P}(\text{A}\cap\text{B})=0.15
\text{P}\Big(\frac{\text{A}}{\text{B}}\Big)=\frac{\text{P}(\text{A}\cap\text{B})}{\text{P(B)}}
\text{P}\Big(\frac{\text{A}}{\text{B}}\Big)=\frac{0.15}{0.7}
\text{P}\Big(\frac{\text{A}}{\text{B}}\Big)=\frac{15}{70}\text{ or }\frac{3}{14}
Q837. Find \frac{\text{dy}}{\text{dx}}, when 2 Marks
\text{x}=\text{a}\cos\theta and \text{y}=\text{b}\sin\theta

Ans: We have, \text{x}=\text{a}\cos \theta and \text{y}=\text{b}\cos\theta


\Rightarrow\frac{\text{dx}}{\text{d}\theta}=-\text{a}\sin\theta and \frac{\text{dy}}{d\theta}=\text{b}\cos\theta
\therefore\frac{\text{dy}}{{\text{d}}\theta}=\frac{\frac{\text{dy}}{\text{d}\theta}}{\frac{\text{dx}}{\text{d}\theta}}=\frac{\text{b}\cos\theta}{-\text{a}\sin\theta}=\frac{-\text{b}}
{\text{a}}\cot\theta
Q838. Find the differential of \sin^2\text{x} w.r.t. \text{e}^{\cos\text{x}}. 2 Marks

Ans: Let \text{u}=\sin^2\text{x} and \text{v}= \text{e}^{\cos\text{x}}


we need to differentiate u w.r.t. v.
i.e, \frac{\text{du}}{\text{dv}}
\frac{\text{du}}{\text{dv}}=\frac{\text{du}}{\text{dv}},\frac{\text{dx}}{\text{dx}}
\frac{\text{du}}{\text{dv}}=\frac{\text{du}}{\text{dx}},\frac{\text{dx}}{\text{dv}}
\frac{\text{du}}{\text{dv}}=\frac{\frac{\text{du}}{\text{dx}}}{\frac{\text{dv}}{\text{dx}}}
Calcutating \frac{\text{du}}{\text{dv}}
\text{u}=\sin^2\text{x}
Differentiating w.r.t. x.
\frac{\text{du}}{\text{dx}}=\frac{\text{d}(\sin^2\text{x})}{\text{dx}}
\frac{\text{du}}{\text{dx}}=2\sin\text{x}.\frac{\text{d}(\sin\text{x})}{\text{dx}}
\frac{\text{du}}{\text{dx}}=2\sin\text{x}.\cos\text{x}
Calcutating \frac{\text{dv}}{\text{dx}}
\text{v}=\text{e}^{\cos\text{x}}
Differentiating w.r.t. x.
\frac{\text{dv}}{\text{dx}}=\text{e}^{\cos\text{x}}.\frac{\text{d}(\text{e}^{\cos\text{x}})}{\text{dx}}
\frac{\text{dv}}{\text{dx}}=\text{e}^{\cos\text{x}}.(-\sin\text{x})
\frac{\text{dv}}{\text{dx}}=-\sin\text{x}.\text{e}^{\cos\text{x}}
Therefore
\frac{\text{du}}{\text{dv}}=\frac{\frac{\text{du}}{\text{dx}}}{\frac{\text{dv}}{\text{dx}}}
=\frac{2\sin\text{x}.\cos\text{x}}{-\sin\text{x}.\text{e}^{\cos\text{x}}}
\frac{-2.\cos\text{x}}{\text{e}^{\cos\text{x}}} or -2\cos\text{x}\text{e}^{-\cos\text{x}}
Q839. Differentiate the functions with respect to x. 2 Marks
\cos(\sin \text{x})

Ans: \text{Let y} = \cos(\sin\text{x})


\therefore \frac{\text{dy}}{\text{dx}} = -\sin(\sin\text{x})\frac{\text{d}}{\text{dx}}\sin\text{x} = -\sin(\sin\text{x})\cos\text{x}
Q840. If \text{f}(\text{x})=\sqrt{\text{tan}\sqrt{\text{x}}}, then find \text{f}'\Big(\frac{\pi^2}{16}\Big). 2 Marks

Ans: Given, \text{f}'(\text{x})=\sqrt{\tan\sqrt{\text{x}}}


\frac{\text{d}}{\text{dx}}\text{f}.(\text{x})=\frac{\text{d}}{\text{dx}}\sqrt{\tan\sqrt{\text{x}}}
\text{f}'(\text{x})-\frac{1}{2}(\tan\sqrt{\text{x}})^{{\frac{-1}{2}}},
\frac{1}{2}(\text{x)}^{\frac{-1}{2}},\sec^2\sqrt{\text{x}}
\text{f}'(\text{x})=\frac{\sec^2\sqrt{\text{x}}}{4\sqrt{\text{x}}\sqrt{\tan\sqrt{\text{x}}}}
Now, put (\text{x})=\frac{\pi^2}{16}
\text{f}'\Big(\frac{\pi^2}{16}\Big)=\frac{\sec^2}{\sqrt[4]{{\frac{\pi^2}{16}}}}.\frac{\sqrt{\frac{\pi^2}{16}}}{\tan\sqrt{\frac{\pi^2}{16}}}
\text{f}'\Big(\frac{\pi^2}{16}\Big)=\frac{\sec^2}{4\times\frac{\pi}{4}}. \frac{\frac{\pi}{4}}{\sqrt{\tan\frac{\pi}{4}}}
=\frac{2}{\pi\times1}
=\frac{2}{\pi}
Q841. Find \frac{\text{d}^2\text{y}}{\text{dx}^2}, where \text{y}=\log\Big(\frac{\text{x}^2}{\text{e}^2}\Big) 2 Marks

Ans: Here
\text{y}=\log\Big(\frac{\text{x}^2}{\text{e}^2}\Big)
Differentiating w.r.t.x, we get
\frac{\text{dy}}{\text{dx}}=\frac{1}{\frac{\text{x}^2}{\text{e}^2}}\times\frac{2\text{x}}{\text{e}^2}=\frac{2}{\text{x}}
Differentiating w.r.t.x, we get
\frac{\text{d}^2\text{y}}{\text{dx}^2}=\frac{-2}{\text{x}^2}
Q842. Differentiate the functions with respect to x. 2 Marks
\sin(\text{x}^{2} + 5)

Ans: \text{Let y} = \sin(\text{x}^{2} + 5)


\therefore \frac{\text{dy}}{\text{dx}} = \cos(\text{x}^{2} + 5)\frac{\text{d}}{\text{dx}}(\text{x}^{2} + 5) = \cos (\text{x}^{2} + 5)(2\text{x} + 0) = 2\text{x}\cos(\text{x}^{2} +5)
Q843. Verify Mean Value Theorem, if f(x) = x2 - 4x - 3 in the interval [a, b], where a = 1 and b = 4. 2 Marks

https://bls.smartstudies.co.in/#/exam/pdf-preview/c59cb220-8e86-4716-9ff7-82aec16b1ade/1 154/158
5/26/24, 6:20 PM Exam Automation
Ans: Function is continuous in [1, 4] as it is a polynomial function and polynomial function is always continuous. f'(x) = 2x - 4, f'(x) exists in [1, 4], hence derivable. Conditions of MVT theorem
are satisfied, hence there exists, at least one \text{c}\in(1,\ 4) such that.
\frac{\text{f}(4)-\text{(f)}(1)}{4-1}=\text{f}'\text{(c)}\ \Rightarrow\ \frac{-3-(-6)}{3}=2\text{c}-4
\Rightarrow\ 1=2\text{c}-4\ \Rightarrow\ \text{c}=\frac{5}{2}
Q844. Differentiate the following w.r.t.x: \frac{\cos\text{x}}{\log\text{x}},\text{x}>0 2 Marks

Ans: \text{Let y}=\frac{\cos\text{x}}{\log\text{x}}


\therefore\ \frac{\text{dy}}{\text{dx}} =\frac{(\log\text{x}).\frac{\text{d}}{\text{dx}}(\cos\text{x})-\cos\text{x}.\frac{\text{d}}{\text{dx}}(\log\text{x})}{(\log\text{x})^{2}}
=\frac{(\log\text{x}).(-\sin\text{x})-\cos\text{x}\cdot\frac{\text{1}}{\text{x}}}{(\log\text{x})^{2}}=\frac{-\sin\text{x}\log\text{x}-\frac{\cos\text{x}}{\text{x}}}{(\log\text{x}^{2})}
=\frac{-\text{x}\sin\text{x}\ \log\text{x}-\cos\text{x}}{\text{x}(\log\text{x})^2}
Q845. If \text{x}=\text{a}\cos\theta;\text{y}=\text{b}\sin\theta, then find \frac{\text{d}^2\text{y}}{\text{dx}^2}. 2 Marks

Ans: \text{x}=\text{a}\cos\theta\Rightarrow\frac{\text{dx}}{\text{d}\theta}=-\text{a}\sin\theta
\text{y}=\text{b}\sin\theta\Rightarrow\frac{\text{dy}}{\text{dx}}=\text{b}\cos\theta
\frac{\text{dy}}{\text{dx}}=\frac{\frac{\text{dy}}{\text{d}\theta}}{\frac{\text{dx}}{\text{d}\theta}}=\frac{\text{b}\cos\theta}{-\text{a}\sin\theta}
\frac{\text{b}}{\text{a}}.\cot\theta
\frac{\text{d}^2\text{y}}{\text{dx}^2}=\frac{-\text{b}}{\text{a}}(\text{cosec}^2\theta).\frac{\text{d}\theta}{\text{dx}}
\frac{\text{b}}{\text{a}}.\text{cosec}^2\theta.\frac{-1}{\text{a}\sin\theta}=\frac{-\text{b}}{\text{a}^2}\text{cosec}^2\theta
Q846. Find the second order derivatives of the function given in Exercise: 2 Marks
\text{e}^\text{x}\sin5\text{x}

Ans: Let \text{y}=\text{e}^\text{x}\sin5\text{x}


\therefore\ \frac{\text{dy}}{\text{dx}}=\text{e}^\text{x}\sin\text{x}+5\text{e}^\text{x}\cos5\text{x}
\therefore\ \frac{\text{d}^2\text{y}}{\text{dx}^2}=\text{e}^\text{x}\sin5\text{x}+5\text{e}^\text{x}\cos5\text{x}+5\text{e}^\text{x}\cos5\text {x}-25\text{ e}^\text{x}\sin5\text{x}
=10\text{e}^\text{x}\cos5\text{x}-24\text{e}^\text{x}\sin5\text{x}=2\text{e}^\text{x}(5\cos5\text{x}-12\sin5\text{x})
Q847. Find \frac{\text{dy}}{\text{ dx}} in the following: 2 Marks
\text{xy} + \text{y}^{2} = \tan\text{x + y}

Ans: The given relationship is \text{xy} + \text{y}^{2} = \tan\text{x + y}


differenting this relationship with respect to x, we obtain
\frac{\text{d}}{\text{dx}}(\text{xy} + \text{y}^{2}) = \frac{\text{d}}{\text{dx}}(\tan\text{x} + \text{y})
\frac{\text{d}}{\text{dx}}(\text{xy})+\frac{\text{d}}{\text{dx}}\text{(y}^{2}) = \frac{\text{d}}{\text{dx}}(\tan\text{x})+\frac{\text{dy}}{\text{dx}}
\Rightarrow \Big[\text{y}.\frac{\text{d}}{\text{dx}}(\text{x})+\text{x}.\frac{\text{dy}}{\text{dx}}\Big] +2\text{y} \frac{\text{dy}}{\text{dx}}=\sec^{2}\text{x}+\frac{\text{dy}}{\text{dx}}\ \
[\text{Using product rule and chain rule]}
\Rightarrow(\text{x}+2\text{y}-1)\frac{\text{dy}}{\text{dx}}=\sec^{2}\text{x}-\text{y}
\therefore\frac{\text{dy}}{\text{dx}}= \frac{\sec^{2}\text{x}-\text{y}}{(\text{x} + 2\text{y}-1)}
Q848. Find \frac{\text{dy}}{\text{dx}}, when 2 Marks
x = at2 and y = 2at

Ans: We have x = at2 and y = 2 at


\Rightarrow\frac{\text{dx}}{\text{dx}}=2\text{at}\text{ and }\frac{\text{dy}}{\text{dx}}=2\text{a}
\therefore\frac{\text{dy}}{\text{dx}}=\frac{\frac{\text{dy}}{\text{dt}}}{\frac{\text{dx}}{\text{dt}}}=\frac{2\text{a}}{2\text{at}}=\frac{1}{\text{t}}
Q849. Determine whether \text{f(x)}=\begin{cases}\frac{\sin\text{x}^2}{\text{x}},&\text{x}\neq0\\0,&\text{x}=0\end{cases} is continuous at x = 0 or not. 2 Marks

Ans: Given, \text{f(x)}=\begin{cases}\frac{\sin\text{x}^2}{\text{x}},&\text{x}\neq0\\0,&\text{x}=0\end{cases}


We have
\lim\limits_{{\text{x}}\rightarrow0}\text{f(x})=\lim\limits_{{\text{x}}\rightarrow0}\frac{\sin\text{x}^2}{\text{x}}
=\lim\limits_{{\text{x}}\rightarrow0}\frac{\text{x}\sin\text{x}^2}{\text{x}^2}
=\lim\limits_{{\text{x}}\rightarrow0}\frac{\sin\text{x}^2}{\text{x}^2}\lim\limits_{{\text{x}}\rightarrow0}\text{x}
=1\times0
=0
=\text{f}(0)
\therefore\ \lim\limits_{{\text{x}}\rightarrow0}\text{f(x)}=\text{f}(0)
Hence, f(x) is continuous at x = 0.
Q850. If \pi\leq\text{x}\leq3\pi and \text{y}=\cos^{-1}(\cos\text{x}), find \frac{\text{dy}}{\text{dx}}. 2 Marks

Ans: We have,
\text{y}=\cos^{-1}(\cos\text{x})
\Rightarrow\text{y}=2\pi-\text{x}
\big[\because\cos^{-1}(\cos\text{x})=2\pi-\text{x},\text{if x}\in[\pi,2{\pi}]\big]
\Rightarrow\frac{\text{dy}}{\text{dx}}=\frac{\text{d}}{\text{dx}}(2\pi-\text{x})
\Rightarrow\frac{\text{dy}}{\text{dx}}=0-1
\Rightarrow\frac{\text{dy}}{\text{dx}}=-1
Q851. If \text{f}'\text{(x)}=\sqrt{2\text{x}^2-1} and \text{y}=\text{f}(\text{x}^2), then find \frac{\text{dy}}{\text{dx}}\text{at x}=1. 2 Marks

Ans: Here,
\text{f}'\text{(x)}=\sqrt{2\text{x}^2-1}
and \text{y}=\text{f}\big(\text{x}^2\big)
Differentiating it with respect to x,
\frac{\text{dy}}{\text{dx}}=\frac{\text{d}}{\text{dx}}\text{f}\big(\text{x}^2\big)
=\text{f}'\big(\text{x}^2\big)\frac{\text{d}}{\text{dx}}\big(\text{x}^2\big)
=\text{d}'\big(\text{x}^2\big)\times2\text{x}
\frac{\text{dy}}{\text{dx}}=2\text{xf}'\big(\text{x}^2\big)
Put x = 1
\frac{\text{dy}}{\text{dx}}=2(1)\text{f}'(1)
=2\times\text{f}'(1)
\frac{\text{dy}}{\text{dx}}=2\times1
\big[\text{Since},\text{f}'(1)=\sqrt{2(1)^2-1}=\sqrt{2-1}=1\big]
\frac{\text{dy}}{\text{dx}}=2
Q852. Find the second order derivatives of the function given in Exercise: 2 Marks
\text{e}^{6\text{x}}\cos3\text{x}

Ans: Let \text{y}=\text{e}^{6\text{x}}\cos3\text{x}


\therefore\ \frac{\text{dy}}{\text{dx}}=6\text{e}^{6\text{x}}\cos3\text{x}-3\text{e}^{6\text{x}}\sin3\text{x}
\therefore\ \frac{\text{d}^2\text{y}}{\text{dx}^2}=36\text{e}^{6\text{x}}\cos3\text{x}-18\text{e}^{6\text{x}}\sin3\text{x}-18\text{e}^{6\text{x}}\sin3\text
{x}-9\text{e}^{6\text{x}}\cos3\text{x}
=27\text{e}^{6\text{x}}\cos3\text{x}-36\text{e}^{6\text{x}}\sin3\text{x}=9\text{e}^{6\text{x}}(3\cos\text{x}-4\sin3\text{x})
Q853. Show that the vectors \hat{\text{i}}-2\hat{\text{j}}+3\hat{\text{k}},\ -2\hat{\text{i}}+3\hat{\text{j}}-4\hat{\text{k}} and \hat{\text{i}}-3\hat{\text{j}}+5\hat{\text{k}} are coplanar. 2 Marks

Ans: Let \vec{\text{a}}=\hat{\text{i}} -2\hat{\text{j}}+3\hat{ \text{k}},


\vec{\text{b}} =-2\hat{\text{i}} +3\hat{\text{j}}- 4\hat{\text{k}}

https://bls.smartstudies.co.in/#/exam/pdf-preview/c59cb220-8e86-4716-9ff7-82aec16b1ade/1 155/158
5/26/24, 6:20 PM Exam Automation
\vec{\text{c}}=\hat{\text{i}}-3\hat{\text{j}}+5\hat{\text{k}}
\Big[\vec{\text{a}}\ \vec{\text{b}}\ \vec{\text{c}}\Big]= \begin{vmatrix}1& -2& 3\\-2& 3& -4\\1& -3 &5\end{vmatrix}
=1(15-12)+2(-10+4)+3(6-3)
=3-12+9
=0
Therefore, \vec{\text{a}},\ \vec{\text{b}},\ \vec{\text{c}} are coplanar.
Hence proved.
Q854. If an operation * on the set of integers Z is defined by a * b = 2a2 + b, then find (i) whether it is a binary or not, and (ii) if a binary, then is it commutative or not. 2 Marks

Ans: 1. \forall\text{A},\ \text{b}\in \text{z},\ \text{a}*\text{b}=\text{2a}^2+\text{b}\in\text{Z} \therefore*\ is binnary


2. 1,\ 2 \in \text{Z},\ 1*2=2\times1^2+2=4
2*1=2\times(2)^2+1=9
1*2\not =2*1
\therefore* is not commutative.
Q855. Differentiate x2 with respect to x3. 2 Marks

Ans: Let u = x2 and v = x3


\Rightarrow\frac{\text{du}}{\text{dx}}=2\text{x and }\frac{\text{dv}}{\text{dx}}=3\text{x}^2
\therefore\frac{\text{du}}{\text{dv}}=\frac{\frac{\text{du}}{\text{dx}}}{\frac{\text{dv}}{\text{dx}}}=\frac{2\text{x}}{3\text{x}^2}=\frac{2}{3\text{x}}
Q856. Differentiate the following w.r.t.x: \log(\cos\text{e}^{\text{x}}) 2 Marks

Ans: \text{Let}\ \text{y}=\log(\cos\text{e}^{\text{x}})


\therefore\ \frac{\text{dy}}{\text{dx}}=\frac{1}{\cos\text{e}^{\text{x}}}\frac{\text{d}}{\text{dx}}(\cos\text{e}^\text{x})\ \ \bigg[\because\frac{\text{d}}{\text{dx}}\log\text{f(x)}=\frac{1}
{\text{f(x)}}\frac{\text{d}}{\text{dx}}\text{f(x)}\bigg]
=\frac{1}{\cos\text{e}^{\text{x}}}(-\sin\text{e}^\text{x})\frac{\text{d}}{\text{dx}}\text{e}^\text{x}= \bigg[\because\frac{\text{d}}{\text{dx}}\cos\text{f(x)}=-\sin\text{f(x)}\frac{\text{d}}
{\text{dx}}\text{f(x)}\bigg]
=-(\tan\text{e}^\text{x})\text{e}^\text{x}=-\text{e}^\text{x}(\tan\text{e}^\text{x})
Q857. Differentiate the functions with respect to x. 2 Marks
\sec(\tan(\sqrt{\text{x}}))

Ans: \text{Let y} = \sec(\tan\sqrt{\text{x}})


\therefore \frac{\text{dy}}{\text{dx}} = \sec(\tan\sqrt{\text{x}})\tan(\tan\sqrt{\text{x}})\frac{\text{d}}{\text{dx}}(\tan\sqrt{\text{x}})
= \sec(\tan\sqrt{\text{x}})\tan(\tan\sqrt{\text{x}})\sec^{2}\sqrt{\text x}\frac{\text{d}}{\text{dx}}\sqrt{\text{x}}
= \sec(\tan\sqrt{\text{x}})\tan(\tan\sqrt{\text{x}})\sec^{2}\sqrt{\text x}.\frac{1}{2}\text{x}^{\frac{1}{2}-1}
= \sec(\tan\sqrt{\text{x}})\tan(\tan\sqrt{\text{x}})\sec^{2}\sqrt{\text x}.\frac{1}{2\sqrt{\text{x}}}
Q858. If x and y are connected parametrically by the equations given in Exercise without eliminating the parameter, Find \frac{\text{dy}}{\text{dx}}. 2 Marks
\text{x}=\cos\theta-\cos2\theta,\text{y}=\sin\theta-\sin2\theta

Ans: The given equations are \text{x}=\cos\theta-\cos2\theta\text{ and y}=\sin\theta-\sin2\theta


Then, \frac{\text{dx}}{\text{d}\theta}=\frac{\text{d}}{\text{d}\theta}(\cos\theta-\cos2\theta)=\frac{\text{d}}{\text{d}\theta}(\cos\theta)-\frac{\text{d}}{\text{d}\theta}(\cos2\theta)
=-\sin\theta-(-2\sin2\theta)=2\sin2\theta-\sin\theta
\frac{\text{dy}}{\text{d}\theta}=\frac{\text{d}}{\text{d}\theta}(\sin-\sin2\theta)=\frac{\text{d}}{\text{d}\theta}(\sin\theta)-\frac{\text{d}}{\text{d}\theta}(\sin2\theta)
=\cos\theta-2\cos2\theta
\therefore\ \frac{\text{dy}}{\text{dx}}=\frac{\Big(\frac{\text{dy}}{\text{d}\theta}\Big)}{\Big(\frac{\text{dx}}{\text{d}\theta}\Big)}=\frac{\cos\theta-2\cos2\theta}{2\sin2\theta-\sin\theta}
Q859. Differentiate w.r.t. x the function in Exercise: 2 Marks
\sin^3\text{x}+\cos^6\text{x}

Ans: Let \text{y}=\sin^3\text{x}+\cos^6\text{x}


\therefore\ \frac{\text{dy}}{\text{dx}}=\frac{\text{d}}{\text{dx}}(\sin^3\text{x})+\frac{\text{d}}{\text{dx}}(\cos^6\text{x})
=3\sin^2\text{x}.\frac{\text{d}}{\text{dx}}(\sin\text{x})+6\cos^5\text{x}.\frac{\text{d}}{\text{dx}}(\cos\text{x})
=3\sin^2\text{x}.\cos\text{x}+6\cos^5\text{x}.(-\sin\text{x})
=3\sin\text{x}\cos\text{x}(\sin\text{x}-2\cos^4\text{x})
Q860. Define differentiability of a function at a point. 2 Marks

Ans: Let f(x) be a real valued function defined on an open interval (a, b) and let \text{c}\in(\text{a, b}).
Then f(x) is said to be differentiable or derivable at x = c iff \lim_\limits{\text{x}\rightarrow{\text{c}}}\frac{\text{f(x)}-\text{f(c)}}{\text{x}-\text{c}} exists finitely.
or, \text{f}'(\text{c})=\lim_\limits{\text{x}\rightarrow{\text{c}}}\frac{\text{f(x)}-\text{f(c)}}{\text{x}-\text{c}}.
Q861. If \text{y}=\sec^{-1}\Big(\frac{\text{x}+1}{\text{x}-1}\Big)+\sin^{-1}\Big(\frac{\text{x}-1}{\text{x}+1}\Big), then write the value of \frac{\text{dy}}{\text{dx}}. 2 Marks

Ans: We have, \text{y}=\sec^{-1}\Big(\frac{\text{x}+1}{\text{x}-1}\Big)+\sin^{-1}\Big(\frac{\text{x}-1}{\text{x}+1}\Big)


\Rightarrow\text{y}=\cos^{-1}\Big(\frac{\text{x}-1}{\text{x}+1}\Big)+\sin^{-1}\Big(\frac{\text{x}-1}{\text{x}+1}\Big)
\Big[\because\sec^{-1}\text{x}=\cos^{-1}\big(\frac{1}{\text{x}}\big)\Big]
\Rightarrow\text{y}=\frac{\pi}{2}\Big[\because\sin^{-1}\text{x}+\cos^{-1}\text{x}=\frac{\pi}{2}\Big]
\Rightarrow\frac{\text{dy}}{\text{dx}}=0
Q862. Differentiate w.r.t. x the function in Exercise: 2 Marks
\cos(\text{a}\cos\text{x}+\text{b}\sin\text{x}),\ for some constant a and b.

Ans: Let \text{y}=\cos(\text{a}\cos\text{x}+\text{b}\sin\text{x})


By using chain rule, we obtain
\frac{\text{dy}}{\text{dx}}=\frac{\text{d}}{\text{dx}}\cos(\text{a}\cos\text{x}+\text{b}\sin\text{x})
\Rightarrow\ \frac{\text{dy}}{\text{dx}}=-\sin(\text{a}\cos\text{x}+\text{b}\sin\text{x}).\frac{\text{d}}{\text{dx}}(\text{a}\cos\text{x}+\text{b}\sin\text{x})
=-\sin(\text{a}\cos\text{x}+\text{b}\sin\text{x}).[\text{a}(-\sin\text{x})+\text{b}\cos\text{x}]
(\text{a}\sin\text{x}-\text{b}\cos\text{x})-\sin(\text{a}\cos\text{x}+\text{b}\sin\text{x})
Q863. If \text{y}^2\cos\bigg(\frac{1}{\text{x}}\bigg)=\text{a}^2, then find \frac{\text{dy}}{\text{dx}}. 2 Marks

Ans: \text{y}^2\cos\bigg(\frac{1}{\text{x}}\bigg)=\text{a}^2
Differentiate with respect to x,
\text{y}^2\frac{\text{d}}{\text{dx}}\cos\bigg(\frac{1}{\text{x}}\bigg)+\cos\bigg(\frac{1}{\text{x}}\bigg)\frac{\text{d}}{\text{dx}}\text{y}^2=\frac{\text{d}}{\text{dx}}\text{a}^2
-\text{y}^2\sin\frac{1}{\text{x}}\frac{\text{d}}{\text{dx}}\bigg(\frac{1}{\text{x}}\bigg)+\frac{\cos}{\text{x}}2\text{y}\frac{\text{dy}}{\text{dx}}=0
\text{y}^2\sin\frac{1}{\text{x}}\bigg(\frac{1}{\text{x}^2}\bigg)+2\text{y}.\cos\frac{1}{\text{x}}\frac{\text{dy}}{\text{dx}}=0
2\text{y}.\cos\frac{1}{\text{x}}\frac{\text{dy}}{\text{dx}}=\frac{\text{y}^2}{\text{x}^2}\sin\bigg(\frac{1}{\text{x}}\bigg)
\frac{\text{dy}}{\text{dx}}=\frac{-\text{y}^2\sin\bigg(\frac{1}{\text{x}}\bigg)}{2\text{y}\text{x}^2\cos\bigg(\frac{1}{\text{x}}\bigg)}
\therefore\frac{\text{dy}}{\text{dx}}=-\frac{\text{y}^2}{2\text{x}^2}\tan\bigg(\frac{1}{\text{x}}\bigg)
Q864. Discuss the continuity of the following functions: 2 Marks
\text{f(x)} = \sin \text{x} + \cos \text{x}

Ans: Let a be an arbitrary real number then ^{\ \ \text{lim}}_{\text{x}\rightarrow\text{a}^{+}}\text{f(x)} \Rightarrow^{\ \ \text{lim}}_{\text{h}\rightarrow\text{0}}\text{f(a + h)}
\Rightarrow ^{\ \ \text{lim}}_{\text{h}\rightarrow\text{0}}\sin\text{(a + h)} + \cos (\text{a} + \text{h})
\Rightarrow ^{\ \ \text{lim}}_{\text{h}\rightarrow\text{0}}(\sin\text{a}\cos\text{ h} + \cos\text{a} \sin \text{h}+\cos \text{a}\cos\text{h}-\sin\text{a}\sin\text{h})
= \sin \text{a}\cos0+\cos\text{a}\sin0 + \cos\text{a}\cos0 - \sin\text{a}\sin0
= \sin \text{a} + 0 + \cos\text{a}-0
= \sin \text{a} + \cos\text{a}= \text{f(a)}
https://bls.smartstudies.co.in/#/exam/pdf-preview/c59cb220-8e86-4716-9ff7-82aec16b1ade/1 156/158
5/26/24, 6:20 PM Exam Automation
Similarly, we have ^{\ \ \text{lim}}_{\text{x}\rightarrow\text{a}^{-}}\text{f(x)} = \text{f(a)}
\therefore\ ^{\ \ \text{lim}}_{\text{x}\rightarrow\text{a}^{-}}\text{f(x)}= \text{f(a)}= ^{\ \ \text{lim}}_{\text{x}\rightarrow\text{a}^{+}}\text{f(x)}
Therefore, f(x) is continuous at x = a.
Since, a is an arbitrary real number, therefore, \text{f(x)}= \sin\text{x} + \cos\text{x} is continuous.
Q865. Discuss the continuity and differntiability of f(x) = |log |x||. 2 Marks

Ans: f(x) = |log |x||


Since, it is an absolute function. So, it is continuous function. The graph of the function is as below:-

From the graph, it is clear that f(x) is not differentiable at x = -1, 1 but continuous for all x.
Q866. Find \frac{\text{dy}}{\text{ dx}} in the following: 2 Marks
\text{x}^{2}+\text{xy} + \text{y}^{2} =100

Ans: The given relationship is \text{x}^{2}+\text{xy} + \text{y}^{2} =100


differenting this relationship with respect to x, we obtain
\frac{\text{d}}{\text{dx}}(\text{x}^{2}+\text{xy} + \text{y}^{2}) = \frac{\text{d}}{\text{dx}}(100)
\frac{\text{d}}{\text{dx}}(\text{x}^{2})+\frac{\text{d}}{\text{dx}}(\text{xy})+\frac{\text{d}}{\text{dx}}\text{(y}^{2}) =0\ [\text{Derivative of constant function is 0}]
\Rightarrow 2\text{x}+ \Big[\text{y}.\frac{\text{d}}{\text{dx}}(\text{x})+\text{x}.\frac{\text{dy}}{\text{dx}}\Big] +2\text{y} \frac{\text{dy}}{\text{dx}}=0\ \ [\text{Using product rule and chain
rule]}
\Rightarrow\ 2\text{x}+\text{y}.1+\text{x}.\frac{\text{dy}}{\text{dx}}+2\text{y}\frac{\text{dy}}{\text{dx}}=0
\Rightarrow\ 2\text{x}+\text{y}+(\text{x}+2\text{y})\frac{\text{dy}}{\text{dx}}=0
\therefore\frac{\text{dy}}{\text{dx}}= -\frac{2\text{x}+\text{y}}{\text{x} + 2\text{y}}
Q867. Define continuity of a function at a point. 2 Marks

Ans: Continuity at a point:


A function f(x) is said to be continuous at a point x = a of its domain, \lim\limits_{{\text{x}}\rightarrow\text{a}}\text{f(x)}=\text{f(a)}
Thus, f(x) is continuous at \text{x}=\text{a}\Leftrightarrow\lim\limits_{{\text{x}}\rightarrow\text{a}}\text{f(x)}=\text{f}
(\text{a})\Leftrightarrow\lim\limits_{{\text{x}}\rightarrow\text{a}^-}\text{f(x)}=\lim\limits_{{\text{x}}\rightarrow\text{a}^+}\text{f(x)}=\text{f}(\text{a})
Q868. If -\frac{\pi}{2}<\text{x}<0\text{ and y}=\tan^{-1}\sqrt{\frac{1-\cos2\text{x}}{1+\cos2\text{x}}}, find \frac{\text{dy}}{\text{dx}}. 2 Marks

Ans: We have, \text{y}=\tan^{-1}\sqrt{\frac{1-\cos2\text{x}}{1+\cos2\text{x}}}


\Rightarrow\text{y}=\tan^{-1}\sqrt{\frac{2\sin^2\text{x}}{2\cos^2\text{x}}}
\Rightarrow\text{y}=\tan^{-1}\sqrt{\tan^2\text{x}}
\Rightarrow\text{y}=\tan^{-1}(\tan\text{x})
\Big[\because\tan^{-1}(\tan\text{x})=-\text{x},\text{if x}\in\big(-\frac{\pi}{2},0\big)\Big]
\Rightarrow\frac{\text{dy}}{\text{dx}}=-1
Q869. Find: \int\text{e}^{\text{x}}(\cos\text{x}-\sin\text{x})\text{cosec}^\text{x}\text{dx} 2 Marks

Ans: \int\text{e}^\text{x} (\cos\text{x}-\sin\text{x})\ \text{cosec}^2 \text{x}\ \text{dx}


=-\int\text{e}^\text{x}[\text{cosec}\ \text{x}-\text{cosec}\ \text{x}\cot\ \text{x}]\text{dx}
=-\text{e}^\text{x}\text{cosec}\ \text{x}\ +\text{c}
Q870. Differentiate \tan^{-1}\Big(\frac{1+\cos\text{x}}{\sin\text{x}}\Big) with respect to x. 2 Marks

Ans: Let \tan^{-1}\Big(\frac{1+\cos\text{x}}{\sin\text{x}}\Big)=\text{y}


\text{y}=\tan^{-1}\bigg(\frac{2\cos^2\frac{\text{x}}{2}}{2\sin\frac{\text{x}}{2}\cos\frac{\text{x}}{2}}\bigg)
=\tan^{-1}\Big(\cot\frac{\text{x}}{2}\Big)
=\tan^{-1}\Big[\tan\Big(\frac{\pi}{2}-\frac{\text{x}}{2}\Big)\Big]
\text{y}=\frac{\pi}{2}-\frac{\text{x}}{2}
\frac{\text{dy}}{\text{dx}}=0-\frac{1}{2}=\frac{-1}{2}
Q871. If \text{f}:[-5,\ 5]\rightarrow\ \text{R} is a differentiable function and if f'(x) does not vanish anywhere, then prove that \text{f}(-5)\neq\text{f}(5). 2 Marks

Ans: For, Rolle’s theorem, if


f is continuous is [a, b]
f is derivable in [a, b]
f(a) = f(b)
Then, \text{f}'\text{(c)}=0,\ \text{c}\in(\text{a},\ \text{b)}
It is given that f is continuous and derivable, but \text{f}'\text{(c)}\neq0
\Rightarrow\ \text{f(a)}\neq\text{f(b)}
\Rightarrow\ \text{f}(-5)\neq\text{f}(5)
Q872. \text{If y}=500\text{e}^{7\text{x}}+600\text{e}^{-7\text{x}},\text{ show that }\frac{\text{d}^2\text{y}}{\text{dx}^2}=49\text{y} 2 Marks

Ans: \text{y}=500\text{e}^{7\text{x}}+600\text{e}^{-7\text{x}}\dots(1)
\frac{\text{dy}}{\text{dx}}=3500\text{e}^{7\text{x}}-4200\text{e}^{-7\text{x}}
\frac{\text{d}^2\text{y}}{\text{dx}^2}=24500\text{e}^{7\text{x}}+29400\text{e}^{-7\text{x}}
\Rightarrow\ \frac{\text{d}^2\text{y}}{\text{dx}^2}=49(500\text{e}^{7\text{x}}+600\text{e}^{-7\text{x}})
\Rightarrow\ \frac{\text{d}^2\text{y}}{\text{dx}^2}=49\text{y}\ \ [\because\text{of }1]
Q873. Find \frac{\text{dy}}{\text{dx}} when x and y are connected by the relation: 2 Marks
\tan^{-1}(\text{x}^2+\text{y}^2)=\text{a}

Ans: Consider, \tan^{-1}(\text{x}^2+\text{y}^2)=\text{a}


\text{x}^2+\text{y}^2=\tan\text{a}
\Rightarrow\ 2\text{x}+2\text{y}\frac{\text{dy}}{\text{dx}}=0
\Rightarrow\ 2\text{y}\cdot\frac{\text{dy}}{\text{dx}}=-2\text{x}
\Rightarrow\ \frac{\text{dy}}{\text{dx}}=\frac{-2\text{x}}{2\text{y}}=\frac{-\text{x}}{\text{y}}
Q874. Find the value of c in Rolle’s theorem for the function \text{f(x)} = \text{x}^{3} - \text{3x in} [ -\sqrt{3}, 0]. 2 Marks

Ans: \text{f(x)} = \text{x}^{3} - \text{3x}


\therefore \text{f}'\text{(c)} = 3\text{ c}^{2} - 3 = 0
\therefore \text{c}^{2} = 1 \Rightarrow \text{c} = \pm 1.
Rejecting c = 1 as it does not belong to (-\sqrt{3, 0)},
we get c = –1.
Q875. Find: \int\sqrt{3-2\text{x}-\text{x}^2}\text{dx} 2 Marks

https://bls.smartstudies.co.in/#/exam/pdf-preview/c59cb220-8e86-4716-9ff7-82aec16b1ade/1 157/158
5/26/24, 6:20 PM Exam Automation
Ans: \int\sqrt{3-2\text{x}-\text{x}^2}\text{ dx}
=\int\sqrt{3-( 2\text{x}-\text{x}^2})\text{ dx}
Adding and subtracting 12
\Rightarrow\int\sqrt{3-(\text{x}^2+2\text{x}+1^2-1^2)}\text{ dx}
\Rightarrow\int\sqrt{3-(\text{x}^2+2\text{x}+1^2)+1)}\text{ dx}
\Rightarrow\int\sqrt{3+1-(\text{x}^2+2\text{x}+1^2)}\text{ dx}
\int\sqrt{4-(\text{x}^2+2\text{x}+1^2)}\text{ dx}
\Rightarrow\int\sqrt{2^2-(\text{x}+1)^2}\text{ dx}
=\frac{1}{2}(\text{x}+1)\sqrt{2^2-(\text{x}-1)^2}+\frac{2^2}{2}\sin^{-1}\frac{(\text{x+1})}{2}+\text{C}
=\frac{1}{2}(\text{x}+1)\sqrt{4-(\text{x}^2+1^2+2\text{x})^2}+\frac{2^2}{2}\sin^{-1}\frac{(\text{x+1})}{2}+\text{C}
=\frac{1}{2}(\text{x}+1)\sqrt{3-2\text{x}-\text{x}^2}+2\sin^{-1}\frac{(\text{x}+1)}{2}+\text{C}
Q876. If \text{y}=\mid\text{x}-\text{x}^2\mid, then find \frac{\text{d}^2\text{y}}{\text{dx}^2} 2 Marks

Ans: We have
\text{y}=\mid\text{x}-\text{x}^2\mid
\Rightarrow\text{y}=\mid\text{x}(1-\text{x})\mid
\Rightarrow\text{y}=\begin{cases}\text{x} -\text{x}^2 &\text{if}&0\leq\times\leq1\\\text{x}^2-\text{x} & \text{if}&\text{x}<0\ \text{or}\ \text{x}>1\end{cases}
\Rightarrow\frac{\text{dy}}{\text{dx}}=\begin{cases}1-2\text{x}&\text{if}&0\leq\times\leq1\\2\text{x}-1&\text{if}&\text{x}\times0\ \text{or}\ \text{x}>1 \end{cases}
\Rightarrow\frac{\text{d}^2\text{y}}{\text{dx}^2}=\begin{cases}-2&\text{if}&0\leq\times\leq1\\2&\text{if}&\text{x}<\ \text{or}\ \text{x}\geq\end{cases}
Q877. Differentiate w.r.t. x the function in Exercise: 2 Marks
\sin^{-1}(\text{x}\sqrt{\text{x)}},\ 0\leq\text{x}\leq1

Ans: Let \text{y}=\sin^{-1}(\text{x}\sqrt{\text{x)}}


Using chain rule, we obtain
\frac{\text{dy}}{\text{dx}}=\frac{\text{d}}{\text{dx}}\sin^{-1}(\text{x}\sqrt{\text{x})}
=\frac{1}{\sqrt{1-(\text{x}\sqrt{\text{x})^2}}}\times\frac{\text{d}}{\text{dx}}(\text{x}\sqrt{\text{x}})
=\frac{1}{\sqrt{1-(\text{x}\sqrt{\text{x})^2}}}.\frac{\text{d}}{\text{dx}}\Big(\text{x}^{\frac{3}{2}}\Big)
=\frac{1}{\sqrt{1-(\text{x}\sqrt{\text{x})^2}}}\times\frac{3}{2}.\text{x}^{\frac{1}{2}}
=\frac{3\sqrt{\text{x}}}{2\sqrt{1-\text{x}^3}}
=\frac{3}{2}\sqrt{\frac{\text{x}}{1-\text{x}^3}}
Q878. If y = x|x|, find \frac{\text{dy}}{\text{dx}}\text{ for x 0}<0. 2 Marks

Ans: We have, y = x|x|


\Rightarrow\text{y}=\text{x}(-\text{x})\big(\because\text{x}>0\big)
\Rightarrow\text{y}=-\text{x}^2
\Rightarrow\frac{\text{dy}}{\text{dx}}=\frac{\text{d}}{\text{dx}}(-\text{x}^2)
\Rightarrow\frac{\text{dy}}{\text{dx}}=-2\text{x}
Q879. If * is defined on the set R of all real numbers by *:\text{a}\ *\text{b}=\sqrt{\text{a}^2+\text{b}^2}, find the identity element, if it exists in with respect to *. 2 Marks

Ans: let b be the identity element, then


\text{a*b}=\text{b*}\ \text{a}=\text{a}
\text{a*b} =\sqrt{\text{(a)}^2+\text{(b)}^2} =\text{a}
\Rightarrow\text{(a)}^2+\text{(b)}^2=\text{(a)}^2
\Rightarrow \text{b} =0
Similarly,
\text{b*a}= \sqrt{\text{(b)}^2+\text{(a)}^2}=\text{a}
\Rightarrow\text{(b)}^2+\text{(a)}^2 =\text{(a)}^2
\Rightarrow\text{b}=0
Q880. \text{Find } \frac{\text{dy}}{\text{dx}} \text{ at t} = \frac{2\pi}{3} \text{ when x 10 (t} -\sin \text{t) and y = 12 } (1 - \cos \text{t}). 2 Marks

Ans: \frac{\text{dy}}{\text{dt}} = 12 \sin \text{t}, \frac{\text{dx}}{\text{dt}} = 10 \text{ }(1 - \cos \text{t})


\therefore \frac{\text{dy}}{\text{dx}} = \frac{6}{5} \times \frac{\sin \text{t}}{1 - \cos \text{t}}
\frac{\text{dy}}{\text{dx}}\bigg|_{\text{t} = \frac{2\pi}{3}} = \frac{6}{5\sqrt{3}}

https://bls.smartstudies.co.in/#/exam/pdf-preview/c59cb220-8e86-4716-9ff7-82aec16b1ade/1 158/158

You might also like